Sie sind auf Seite 1von 1481

Kritik ist Kaput

Kritik ist Kaput........................................................................................................... 1

**GENERAL K ANSWERS** ..................................................................................... 16

**Framework** ...................................................................................................... 17

Fiat Good: 2AC ........................................................................................................ 18

General Defense of the Aff: 2AC (1/2) .................................................................... 20

Floating PICs Bad: 2AC (Long) (~50 sec.) ................................................................. 22

Floating PICs Bad: 2AC (Short) (<20 sec.) ................................................................ 25

Do the Plan Perm: 2AC............................................................................................ 27

#1 Steals Aff Ground: 1AR ....................................................................................... 29

#4 Ext. Turns Offense: 1AR...................................................................................... 31

A2 Plan Focus Bad: 1AR ....................................................................................... 33

A2 Plan is only a tiny part of the speech/Discourse of 1AC is ~9 min.: 1AR ......... 35

Must Have an Alternative: 2AC ............................................................................... 37

Hasty Generalization Bad: 2AC ............................................................................... 38

Law Transformative: 2AC (1/2) ............................................................................... 40

Policymaking Good: 1AR (1/2) ............................................................................. 45

A2 Only Learn As Spectators: 1AR ....................................................................... 57

Policy Debate Good ................................................................................................ 59

Switch-side Debate Good (1/3) ............................................................................... 61

Debate Solves Authoritarianism ............................................................................. 66

Roleplaying Good (1/3) ........................................................................................... 67

Traditional Debate Good (1/2) ................................................................................ 74

Traditional Debate Accesses Peformativity ............................................................. 78

Competition Good .................................................................................................. 79

**Permutations** .................................................................................................. 80

Juxtaposition Perm: 2AC ......................................................................................... 80

Juxtaposition Perm: 1AR ......................................................................................... 82

Campbell Perm: 2AC ............................................................................................... 89

Campbell Perm: 1AR ............................................................................................... 90

Strategic Essentialism Perm: 2AC............................................................................ 91

Strategic Essentialism Perm: 1AR............................................................................ 92

Bleiker Perm: 2AC ................................................................................................... 93

Perm Solves: Coalitions Key .................................................................................... 94

Perm Solves: Hybridization Effective ...................................................................... 95

Page 1 of 1481
Perm Solves: Multifaceted Resistance Best ............................................................ 96

Perm Solves: Radicalism Dooms the Movement ..................................................... 97

Perm Solves: Working within Institutions Key to Change ....................................... 98

**Classic Turns** .................................................................................................. 101

Derrida Turn: 2AC ................................................................................................. 102

Fear of Co-optation Turn: 2AC .............................................................................. 103

Fear of Co-optation Turn: 1AR .............................................................................. 104

The Fetish: 2AC ..................................................................................................... 106

The Fetish: 1AR ..................................................................................................... 109

Authenticity Impossible: 1AR ................................................................................ 112

Kulynych Turn: 2AC ............................................................................................... 113

Kulynych Turn: 1AR ............................................................................................... 114

Praxis Turn: 2AC .................................................................................................... 116

Praxis Turn:1AR..................................................................................................... 117

Praxis Turn: 2AR .................................................................................................... 119

Praxis Turn: Ext ..................................................................................................... 120

Presymbolism Turn: 2AC ....................................................................................... 121

Presymbolism Turn: 1AR ....................................................................................... 123

Rejection Bad Turn: 2AC ....................................................................................... 124

Rejection Bad Turn: 1AR ....................................................................................... 125

Rejection Bad Turn: Ext ......................................................................................... 127

Ricouer Turn: 2AC ................................................................................................. 128

Ricoeur Turn: 1AR ................................................................................................. 130

Ricoeur Turn: Ext .................................................................................................. 133

Romanticization Turn: 2AC ................................................................................... 134

Romanticization Turn: 1AR ................................................................................... 135

Romanticization Turn: 2AR ................................................................................... 137

Said Turn: 2AC....................................................................................................... 139

Academic Work Spurs Activism: Ext (1/2) ............................................................. 141

Academics as Politics is Bad (1/2) ......................................................................... 145

Criticism Destroys Agency ..................................................................................... 149

Criticism is Nihilistic (1/4) ..................................................................................... 150

**Postmodernism Bad** ...................................................................................... 158

Floating Subjectivity Bad (1/3) .............................................................................. 158

**Pragmatism** ................................................................................................... 165

Pragmatism Good: 2AC (1/3) ................................................................................ 166

Plan focus good: Rorty (1/2) ................................................................................. 173

Page 2 of 1481
**Realism**.......................................................................................................... 179

Realism Good: 2AC (1/2) ....................................................................................... 180

#1 Mearsheimer: 1AR ........................................................................................... 186

#1 Mearsheimer: Ext............................................................................................. 188

#2 Guzzini: 1AR ..................................................................................................... 191

#2 Guzzini: Ext....................................................................................................... 194

#3 Murray: 1AR ..................................................................................................... 197

#3 Murray: Ext ...................................................................................................... 199

Democratic Realism Solves the Links .................................................................... 201

Violence is Endemic .............................................................................................. 203

Realism Good: Prevents Nuclear War ................................................................... 207

Realism Good: Prevents War (1/3) ....................................................................... 208

Realism Good: Militarism Solves War (1/2) .......................................................... 215

Realism Good: Militarism Solves Genocide ........................................................... 220

Realism Good: Militarism Solves Democracy ........................................................ 223

Alt Bad: Could Make Things Worse ....................................................................... 225

Alt Fails: Realism Inevitable (1/2) .......................................................................... 226

Alt Fails: Realism Will Reasset Itself ...................................................................... 229

IR is Realist Now (1/2) ........................................................................................... 230

Miscalculation Inevitable ...................................................................................... 234

Perm Solves: Realism Necessary to Understand Parts of IR .................................. 237

A2 9/11 Disproves Realism ................................................................................ 238

A2 Cold War Disproves Realism (1/2) ................................................................ 239

A2 Cold War End Proves Liberalism ................................................................... 244

A2 Cooperation Good (1/2) ............................................................................... 245

A2 Democracy Solves War ................................................................................. 249

A2 Defense Solves ............................................................................................. 250

A2 Human Nature .............................................................................................. 251

A2 Mindset Shift ................................................................................................ 253

A2 Realism Assumes States Rational ................................................................. 256

A2 Realism Constructs Threats .......................................................................... 258

A2 Realism is Amoral ......................................................................................... 259

A2 Realism is a Self-Fulfilling Prophecy (1/2) ..................................................... 261

A2 Social Constructivism (1/3) ........................................................................... 266

A2 State/Sovereignty Bad .................................................................................. 273

**Calculability/Util** ............................................................................................ 275

Utilitarianism Good: 2AC (1/2) .............................................................................. 276

Page 3 of 1481
Utilitarianism Good: 1AR ...................................................................................... 280

Calculability Good: 2AC (1/2) ................................................................................ 283

A2 Tyranny of Survival (1/2) .............................................................................. 288

A2 Ontology First: 2AC....................................................................................... 294

A2 Your Impact is Inevitable: 2AC ...................................................................... 296

A2 Your Impact is Inevitable: 1AR ...................................................................... 298

A2 Your Impact = Bare Life: 2AC (1/3) ............................................................... 299

A2 No Value to Life: 2AC (1/3) ........................................................................... 306

No Value To Life Justifies Genocide ................................................................... 312

No Value To Life Justifies Nazism....................................................................... 313

Theres Always Value To Life ................................................................................. 314

A2 Communication Scholar Framework: 2AC .................................................... 317

**Democratic Talk** ............................................................................................ 320

Democratic Talk Turn: 2AC (1/2) ........................................................................... 321

Democratic Talk Turn: 1AR (1/3) ........................................................................... 326

Debate Solves Democratic Talk: Ext ...................................................................... 333

Democratic Talk Key to Autonomy: Ext ................................................................. 336

Democratic Talk Key to Checking Right: Ext .......................................................... 338

Restoring Public Sphere Solves Oppression: Ext ................................................... 341

Talk is Action: Ext .................................................................................................. 343

**Performance** ................................................................................................. 345

A2 Performativity (1/2) ...................................................................................... 346

Performance is Commodified (1/2)....................................................................... 350

Performance Fails ................................................................................................. 354

**Link Answers: General** ................................................................................... 355

A2 The Case is Apolitical/Has No Theory ........................................................... 355

**Alternative Answers: General** ....................................................................... 356

Individual Action Fails ........................................................................................... 356

Mann .................................................................................................................... 358

Power Vaccuum .................................................................................................... 361

**SPECIFIC K ANSWERS** .................................................................................... 364

**Apocalyptic Rhetoric** ..................................................................................... 365

Perm Solvency ...................................................................................................... 366

Apocalyptic Rhetoric Good (1/3)........................................................................... 367

**Badiou** ........................................................................................................... 374

A2 Badiou: 2AC .................................................................................................. 375

Perm Solvency (1/3) .............................................................................................. 377

Page 4 of 1481
Human Rights Solve .............................................................................................. 384

Double Bind .......................................................................................................... 386

Alternative Fractures Coalitions ............................................................................ 388

Divorcing Politics from State Bad .......................................................................... 390

**Baudrillard** .................................................................................................... 393

Baudrillard Destroys Social Change (1/2) .............................................................. 394

Alternative Masks Violence................................................................................... 398

Our Representations Solve ................................................................................... 400

Baudrillard is Wrong (1/2) .................................................................................... 402

A2 Disaster Porn (1/3) ....................................................................................... 406

**Butler** ............................................................................................................ 413

Butler Answers: 2AC (1/2)..................................................................................... 414

A2 Legal Categories Bad .................................................................................... 418

**Biopolitics**...................................................................................................... 420

Agamben Answers: 2AC (1/6) ............................................................................... 421

#2 Alternative Kills Liberation: 1AR (1/2) .............................................................. 437

#5 Perm: 1AR ........................................................................................................ 442

#5 Perm: Ext.......................................................................................................... 445

#7 Good Biopower: 1AR (1/2) ............................................................................... 447

#9 Essentialism: 1AR (1/2) .................................................................................... 452

#9 Essentialism: Ext............................................................................................... 458

#10 Criticism Causes Powerlessness: 1AR (1/2) .................................................... 461

#10 Criticism Causes Powerlessness: Ext (1/3) ..................................................... 465

A2 Neilson Conclude Negative: 1AR .................................................................. 472

#11 Agamben Misunderstands Sovereignty: 1AR ................................................. 474

#11 Agamben Misunderstands Sovereignty: Ext (1/2) .......................................... 477

#13 Praxis: 1AR ..................................................................................................... 483

#14 Liberalism Doesnt Cause Exception: 1AR ...................................................... 485

Agamben Collapses the State ............................................................................... 488

**Foucault** ........................................................................................................ 490

Foucault Answers: 2AC (1/3)................................................................................. 491

#2 Perm: 1AR ........................................................................................................ 500

Juxtaposition Solves: 1AR (1/2) ............................................................................. 501

#5 Demands on the State Good: 1AR (1/4) ........................................................... 505

#6 Nihilism (Cook): 1AR (1/2) ................................................................................ 515

#10 Reformism Good: 1AR .................................................................................... 520

Alt Fails: Body Cannot Be a Site of Resistance....................................................... 522

Page 5 of 1481
Alt Fails: Cannot Escape Subjectivity ..................................................................... 523

Alt Fails: Geneologies Dont Produce Change ....................................................... 524

Alt Fails: Remains Enmeshed in Power ................................................................. 525

Alt Fails: Praxis ...................................................................................................... 526

Alt Fails: Suspicion ................................................................................................ 531

**Benjamin** ....................................................................................................... 533

Benjamin Answers: 2AC ........................................................................................ 534

**Chaloupka** ..................................................................................................... 536

Chaloupka Answers: 2AC (1/3).............................................................................. 537

**CLS** ...................................................................... Error! Bookmark not defined.

CLS Answers: 2AC (1/4) ......................................................................................... 547

#4 Permutation: 1AR (1/2) .................................................................................... 557

#7 Experiential Deconstruction Turn: 1AR ............................................................ 561

A2 Religious Institution Rationalized Oppression: 1AR ...................................... 564

#8 Liberalism Good Turn: 1AR .............................................................................. 566

No Links (1/2) ........................................................................................................ 569

Turns: Ricoeur ....................................................................................................... 573

Turns: Judicial Oppression .................................................................................... 576

Turns: Criticism Perpetuates Capitalism ............................................................... 578

Turns: Law Key to Solving Atrocity ........................................................................ 580

Turns: Law Key to Solving Exploitation ................................................................. 583

Turns: Rights Good (1/4) ....................................................................................... 586

Turns: Alternative Causes Rights Rollback ............................................................ 595

Turns: Minorities................................................................................................... 597

Turn: Working in System Good (1/2) .................................................................... 599

Indeterminacy False (1/4) ..................................................................................... 604

A2 Language Makes Law Indeterminate: 2AC ................................................... 614

CLS Recreates Oppression (1/2) ............................................................................ 616

CLS is Nihilistic ...................................................................................................... 621

No Alternative (1/2) .............................................................................................. 623

Alternative Fails: Elitism ........................................................................................ 629

Alternative Fails: Fractures Movement ................................................................. 632

Alternative Fails: Indeterminacy Kills Criticism ..................................................... 634

Alternative Fails: Historical Record of Marxism .................................................... 636

Alternative Fails: Non-Rights Strategies Bad ......................................................... 639

Alternative Fails: Praxis (1/3) ................................................................................ 641

A2 Thats Not Our Indeterminacy Thesis: 1AR ................................................... 649

Page 6 of 1481
A2 Reification: 2AC ............................................................................................ 651

A2 Rights Tradeoff: 2AC ..................................................................................... 653

A2 Feminist Jurisprudence: 2AC ........................................................................ 655

A2 Fem K of Intl Law: 2AC .................................................................................. 657

**CRT** ................................................................................................................ 660

CRT Answers: 2AC (1/4) ........................................................................................ 661

#5 Perm: 1AR ........................................................................................................ 671

**Cuomo**........................................................................................................... 673

Preventing Nuke War Is a Prerequisite to Positive Peace ..................................... 674

Negative Peace Key to Positive Peace ................................................................... 676

Absolutism Bad ..................................................................................................... 678

**Deep Ecology** ................................................................................................ 679

Permutation Solvency: 2AC................................................................................... 679

Permutation Solvency: 1AR .................................................................................. 680

Anthro Good/Inevitable (1/3) ............................................................................... 681

Human Intervention Good .................................................................................... 687

Deep Ecology Justifies Ecocide (1/2) ..................................................................... 688

Deep Ecology Reinscribes Anthropocentrism (1/2)............................................... 691

Deep Ecology Justifies Nazism: 2AC ...................................................................... 695

Deep Ecology Justifies Nazism: Ext (1/2) ............................................................... 700

A2 Were Not Fascists: 1AR ............................................................................... 705

Deep Ecology Justifies State/Capitalism................................................................ 706

Deep Ecology Creates Suffering ............................................................................ 707

Case Comes First ................................................................................................... 708

Alternative Fails: Bad Activism .............................................................................. 709

Alternative Fails: Premodern Society Bad ............................................................. 711

Asteroid Turn ........................................................................................................ 712

HIV Turn ................................................................................................................ 714

African AIDS Outweighs ........................................................................................ 716

Singularity Turn ..................................................................................................... 718

**Deleuze and Guattari** .................................................................................... 720

Perms .................................................................................................................... 721

Alternative Increases Oppression ......................................................................... 722

Deleuze Bad (General) .......................................................................................... 724

D & G Exclude Women.......................................................................................... 725

A2 Life is Carbon ................................................................................................ 726

A2 Death Doesnt Destroy Being: 2AC (1/2) ...................................................... 728

Page 7 of 1481
A2 Life is Meaningless Because the Sun Will Go Out: 2AC ................................. 733

**Derrida** .......................................................................................................... 736

A2 Deconstruction............................................................................................... 737

A2 New International (1/2) ................................................................................ 739

**Discourse Kritiks (General)** ............................................................................ 744

Discourse Kritik Answers: 2AC (1/3)...................................................................... 745

Newspeak Turn: 1AR ......................................................................................... 754

#2 Newspeak Turn: Ext (1/5)................................................................................. 757

#4 Censorship Bad Turns: 1AR .............................................................................. 770

#4 Censorship Bad Turns: Ext (1/4) ....................................................................... 772

#7 Discourse Focus Trades off with Action: 1AR ................................................... 783

#7 Discourse Focus Trades off with Action: Ext..................................................... 786

#8 Alternative Fails: 1AR ....................................................................................... 788

Holocaust Trivialization Answers: 2AC (1/3) ......................................................... 790

A2 Representation Links (1/4) ........................................................................... 797

A2 Indigenous Peoples Labels Bad: 2AC ............................................................ 807

EPrime Answers: 2AC (1/3) ................................................................................... 808

EPrime Bad (Jack Attack!) ..................................................................................... 816

**Fear Bad** ........................................................................................................ 818

A2 Fear of Death Bad: 2AC (1/5) ........................................................................ 819

#3 Good Fear of Death: 1AR (1/2) ......................................................................... 829

#4 Repression Turn: 1AR (1/3) .............................................................................. 832

#5 Fear is Key to Love: 1AR ................................................................................... 837

#6 Inaction Turn: 1AR ........................................................................................... 838

#7 Fear Solves War: 1AR ....................................................................................... 839

Spectacle of Death Good (1/4) .............................................................................. 842

**Empire**........................................................................................................... 852

Movements Fail .................................................................................................... 853

Alternative Causes Violence.................................................................................. 855

Alternative is False Radicalism .............................................................................. 857

Capitalism is Sustainable ....................................................................................... 859

Resistance Fails ..................................................................................................... 861

Alternative = Oppression ...................................................................................... 863

Alternative Fractures Other Movements .............................................................. 865

Alternative Causes Terrorism ................................................................................ 868

**Exceptionalism (USC)** .................................................................................... 870

Exceptionalism Answers: 2AC ............................................................................... 871

Page 8 of 1481
**Feminism**....................................................................................................... 874

Feminism Answers: 2AC (1/2) ............................................................................... 875

White Feminism Bad: 1AR .................................................................................... 885

**Gift** ................................................................................................................ 888

A2 The Gift: 2AC (1/4)........................................................................................ 889

Anti-Globalization Turn: 1AR (1/2) ........................................................................ 898

Anti-Globalization Movements Up Now (1/2)....................................................... 903

Provisional Truth Turn: 2AC (1/2) ......................................................................... 909

Provisional Truth: 1AR .......................................................................................... 914

**Global/Local** .................................................................................................. 916

Micropolitics Only Benefit Privileged .................................................................... 917

Localism Causes Oppression (1/2) ........................................................................ 919

Globalism Key to Resistance ................................................................................. 924

Alternative Kills Movements ................................................................................. 926

Rejection Bad ........................................................................................................ 928

A2 Localism........................................................................................................ 929

Permutation .......................................................................................................... 930

**Habeas Corpus** .............................................................................................. 932

Habeas Corpus Answers: 2AC (1/3)....................................................................... 933

**Habermas** ...................................................................................................... 941

Habermas Answers: 2AC ....................................................................................... 942

**Heidegger**...................................................................................................... 944

Ethics Turn ............................................................................................................ 945

Ontological Fascism Turn: 2AC .............................................................................. 947

Ontology = Nazism: 1AR........................................................................................ 950

Ontology = Nazism: Ext (1/3) ................................................................................ 951

A2 We Dont Advocate Nazism: 1AR.................................................................. 959

A2 Nazism is Inauthentic: 1AR ........................................................................... 960

Heidegger Kills Change.......................................................................................... 963

Heidegger Irrelevent ............................................................................................. 965

Rejecting Tech Leads to Extinction........................................................................ 967

Alternative Fails: Lapses Into Ontic Thought ......................................................... 969

Alternative Fails: Tech Returns ............................................................................. 972

Alternative Causes Suffering ................................................................................. 974

Alternative Causes Paralysis (1/2) ......................................................................... 976

Heidegger Was a Nazi ........................................................................................... 981

Anti-Humanism Justifies Genocide ....................................................................... 984

Page 9 of 1481
Liberal Humanism Solves Oppression ................................................................... 987

Humanism Solves Genocide .................................................................................. 990

A2 Reject Technology: 2AC ................................................................................ 992

A2 Spanos: 2AC (1/3) ............................................................................................ 994

A2 Spanos: 2AC (3/3) ............................................................................................ 999

HR Bad Answers: 2AC (1/4) ................................................................................. 1001

#3 Essentialism Turn: 1AR ................................................................................... 1012

#5 Relativist Apologism Turn: 1AR ...................................................................... 1014

#8 Permutation: 1AR (1/3) .................................................................................. 1016

#10 Zizek Presymbolism: 1AR (1/2)..................................................................... 1024

No Link ................................................................................................................ 1028

Relativism Is Self-Refuting................................................................................... 1029

Defense: Non-Westerners Want Dignity ............................................................. 1031

A2 Foundationalism Bad .................................................................................. 1033

A2 Morality Is Culturally Created ..................................................................... 1034

K = Imperialist ..................................................................................................... 1035

**Kappeler** ...................................................................................................... 1037

Kappeler Answers: 2AC (1/5) .............................................................................. 1038

#5 Alternative Causes Violence: 1AR (1/2) .......................................................... 1050

#7 Negation: 1AR ................................................................................................ 1054

#8 Subversion: 1AR ............................................................................................. 1057

#12 Authenticity: 1AR ......................................................................................... 1058

**Kato** ............................................................................................................. 1061

Kato Answers: 2AC (1/4) ..................................................................................... 1062

**Levinas/Derrida** ........................................................................................... 1073

A2 Infinite Responsibility (1/3) ........................................................................ 1074

Levinas Destroys Ethics (1/2) .............................................................................. 1082

Levinas/Derrida Destroy Ethics ........................................................................... 1087

**Nietzsche**..................................................................................................... 1090

Nietzsche Answers: 2AC (1/6) ............................................................................. 1091

Nietzsche = Nihilism ............................................................................................ 1105

Nietzsche Legitimizes Genocide (1/2) ................................................................. 1108

Nietzsche Legitimizes Patriarchy ......................................................................... 1113

Alternative Causes Annihilation .......................................................................... 1115

Nihilism Fails ....................................................................................................... 1117

Nihilism Causes Terrorism (1/2) .......................................................................... 1120

Nihilism Causes Terrorism (2/2) .......................................................................... 1123

Page 10 of 1481
Nihilism is the Root Cause of Violence ................................................................ 1125

Nihilism Causes Authoritarianism ....................................................................... 1127

**Nonviolence** ................................................................................................ 1129

Nonviolence Answers: 2AC (1/6) ........................................................................ 1130

#2 Pragmatic Pacifism Perm: 1AR (1/2) .............................................................. 1145

A2 Violence Snowballs: 1AR ............................................................................ 1149

#5 Violence Inevitable: 1AR ................................................................................ 1152

#7 Pacifism Allows Atrocity: 1AR ........................................................................ 1154

Pacifism = State Collusion (1/2) .......................................................................... 1156

Embracing Violence = Nonviolence ..................................................................... 1162

Pacifism = Violence (1/3) .................................................................................... 1163

Pacifism Doesnt Solve Violence ......................................................................... 1168

Pacifist Activism Fails: General............................................................................ 1170

Pacifist Activism Fails: Law is Violent .................................................................. 1173

Pacifist Activism Fails: Final Solution (1/3) .......................................................... 1176

Pacifist Activism Fails: Final Solution (3/3) .......................................................... 1181

Civil Disobedience Fails (1/2) .............................................................................. 1183

A2 Violence Alienates the People: 2AC............................................................ 1187

A2 Non-Violence Key to Prevent Eradication of Movement: 2AC .................... 1188

Pacifism Bad: War Good (1/2)............................................................................. 1190

Pacifism Bad: Unethical ...................................................................................... 1195

Pacifism Causes Oppression ................................................................................ 1196

Pacifism Causes Aggression (1/2)........................................................................ 1198

**Normativity** ................................................................................................. 1203

Normativity Answers: 2AC (1/7) ......................................................................... 1204

#3 Permutation: 1AR........................................................................................... 1223

#3 Permutation: Ext ............................................................................................ 1225

#5 Sublime Justice: 1AR ...................................................................................... 1227

#7 Alt Reinscribes Subject: 1AR (1/2) .................................................................. 1230

#9 Normativity Good: 1AR .................................................................................. 1236

#10 Simulation/Roleplaying Good: 1AR (1/3) ..................................................... 1238

#11 Alt Lapses Back into NLT: 1AR ...................................................................... 1245

#11 Alt Lapses Back into NLT: Ext........................................................................ 1248

Normative Thought Inevitable (1/3) ................................................................... 1250

Alternative Fails .................................................................................................. 1257

Pragmatism Good ............................................................................................... 1259

**Nuclearism** .................................................................................................. 1261

Page 11 of 1481
Nuclearism Answers: 2AC (1/3) .......................................................................... 1262

#1 Permutation: 1AR........................................................................................... 1270

#5 Fear of Nuc Weapons Solves Usage: 1AR ....................................................... 1274

#5 Fear of Nuc Weapons Solves Usage: Ext ........................................................ 1276

#5 Nuclear Imagery Good: 1AR ........................................................................... 1287

A2 Nuclear Numbing: 2AC ............................................................................... 1289

A2 Nuclear Deterrence Immoral: 2AC (1/2) ..................................................... 1291

A2 Proliferation K: 2AC .................................................................................... 1295

**Religion** ....................................................................................................... 1297

Wrath of God Answers: 2AC (1/6)....................................................................... 1298

#1 Finite Quantum States: 1AR ........................................................................... 1312

A2 Cant Disprove Gods Existence: 1AR .......................................................... 1314

#7 Religious Suffering: 1AR (1/3) ........................................................................ 1315

A2 Those Ppl Werent Real Christians: 1AR ..................................................... 1321

#8 Evilution Disproves Religion: 1AR ................................................................... 1323

Evolution Contradicts Christianity: Ext (1/2) ....................................................... 1324

A2 Evolution Is Only a Theory: 1AR.................................................................. 1328

A2 Evolution Contradicts Thermodynamics: 1AR ............................................ 1329

A2 No Transitional Fossils: 1AR ........................................................................ 1330

#12 Sexual Abuse: 1AR........................................................................................ 1332

Christianity = Sex Abuse: Ext (1/3) ...................................................................... 1334

A2 Life Without God Pointless: 1AR................................................................. 1342

A2 Life Without God is Terrifying: 1AR ............................................................ 1344

Alternative Hurts Religion ................................................................................... 1345

**Securitization** .............................................................................................. 1347

Security Good: Helps Marginalized People ......................................................... 1348

Alt Bad: Allows Suffering to Continue ................................................................. 1349

Alt Fails: Engagement/Nonengagement Doublebind .......................................... 1350

Alt Fails: Securitizes Itself .................................................................................... 1351

Perm Solves: Starting Point ................................................................................. 1352

Perm Solves: Must Act ........................................................................................ 1353

A2 Dillon: 2AC .................................................................................................. 1355

**Speaking for Others** .................................................................................... 1357

A2 Speaking for Others: 2AC (1/2) ................................................................... 1357

#3 Retreat: 1AR ................................................................................................... 1362

#3 Retreat: Ext .................................................................................................... 1364

#6 Perm: 1AR ...................................................................................................... 1365

Page 12 of 1481
#9 Reductionism: 1AR ......................................................................................... 1366

The Alternative is a Fantasy ................................................................................ 1368

**State Bad, Juhdge** ........................................................................................ 1370

Strategic Use of State Good ................................................................................ 1371

State is Key to Solving Oppression (1/2) ............................................................. 1374

State Key to Solving War (1/2) ............................................................................ 1380

Alternative Creates Worse Oppression (1/2) ...................................................... 1385

Alternative Causes Nuclear War ......................................................................... 1391

Permutation Solvency (1/3) ................................................................................ 1394

No Link ................................................................................................................ 1400

No Alternative..................................................................................................... 1402

A2 Borders: 2AC............................................................................................... 1404

**Terror Talk**................................................................................................... 1406

Terror Talk Answers: 2AC (1/5) ........................................................................... 1407

Terror Discourse Good: 1AR ............................................................................... 1419

Counterspeech Solves: 1AR ................................................................................ 1421

**Threat Construction** .................................................................................... 1423

Threat Construction Answers: 2AC (1/3) ............................................................ 1424

#2 Threat Rhetoric Deters War: 1AR ................................................................... 1432

#5 Realism Inevitable: 1AR.................................................................................. 1434

#7 Scenario Analysis Good: 1AR (1/3) ................................................................. 1436

#9 Prefer Our Args: 1AR ...................................................................................... 1441

Dillon Supports Acting Against Terrorism ........................................................... 1442

**Zizek: Psychopolitics**.................................................................................... 1444

Lacan Destroys Social Change (1/2) .................................................................... 1445

Lacan = Being Towards Death ............................................................................. 1450

Lacan = Oppression ............................................................................................. 1453

A2 Stavrakakis: 2AC ......................................................................................... 1456

Marxism Answers: 2AC (1/2) .............................................................................. 1458

Brown Turns (1/2) ............................................................................................... 1462

Permutation Key to Socialism ............................................................................. 1467

**Miscellaneous** ............................................................................................. 1469

A2 Art (1/2)...................................................................................................... 1470

A2 Love ............................................................................................................ 1475

A2 Poetry ......................................................................................................... 1477

A2 Silence ........................................................................................................ 1479

A2 Third World Bad ....................................................................................... 1481

Page 13 of 1481
Page 14 of 1481
Page 15 of 1481
**GENERAL K ANSWERS**

Page 16 of 1481
**Framework**

Page 17 of 1481
Fiat Good: 2AC
Next, our interpretation is that plan is a yes/no question. If its better than the squo or a competng policy option, we
win. Thats good because

A. It is the most predictable because the resolution asks a question about federal action. The lack of
individual agency stipulations in the resolution mean that introducing such questions are outside the
scope of the subject matter we were asked to prepare to debate. We would be happy to address such
concerns under different resolutions

B. It facilitates the best policy analysis because it ensures that we are not forced to compare aff apples
versus neg oranges

C. Aff choice justifiesthey can run critical affirmatives if they want and we will engage themthey should
reciprocally respect our choice to play the fiat game

D. Our affirmative impact claims necessitateclaims of individual agency beg the question of the efficacy of
liberal politics, and we impact turn such claims by proving that their drive for unfettered autonomy lets
the government get away with destroying the world

E. Most educationalkritiks are run in debate because graduate assistants like to talk about their course
readings with debaterswe lack the foundational understanding to engage in high speed discourse
about such arguments until weve done our homework, whereas high school civics provides adequate
grounding for policy debate. We think that there should be two debate leagues: a policy circuit for
undergrads and a critical circuit for grad students.

F. Even if we lose the fiat debate, we still get to leverage our aff impacts against those of the kritikthe
discursive (or other) mechanism through which their alternative solves is just as available to our message
about the necessity of authoritarianism. We are both theoretical kritiks of the status quo

Page 18 of 1481
Page 19 of 1481
General Defense of the Aff: 2AC
(1/2)
PERM DO BOTH

PERM DO THE PLAN AND ALL OF THE ALTERNATIVE EXCEPT THE PARTS THAT
LINK TO PLAN

POLICYMAKING PROVIDES A UNIQUE SPACE TO BECOME EDUCATED ABOUT


CRITICAL ADVOCACY, THE ONLY ALTERNATIVE IS THE CREATION OF A NEW ELITE

Coverstone 95
[Alan, Princeton High School, An Inward Glance: A Response to Mitchells Outward Activist Turn,
www.wfu.edu/Student-organizations/debate/MiscSites/DRGArticles/Coverstone1995China.htm, acc 3-16-05//uwyo-
ajl]

Yet, Mitchell goes too far. In two important areas, his argument is slightly miscalibrated. First,
Mitchell underestimates the value of debate as it is currently practiced. There is greater value in
the somewhat insular nature of our present activity than he assumes. Debate's inward focus
creates an unusual space for training and practice with the tools of modem political discourse.
Such space is largely unavailable elsewhere in American society. Second, Mitchell overextends
his concept of activism. He argues fervently for mass action along ideological lines. Such a
turn replaces control by society's information elite with control by an elite all our own. More
than any other group in America today, practitioners of debate should recognize the subtle
issues upon which political diversity turns. Mitchell's search for broad themes around which to
organize mass action runs counter to this insight. As a result, Mitchell's call for an outward
activist turn threatens to subvert the very values it seeks to achieve.

Page 20 of 1481
Page 21 of 1481
Floating PICs Bad: 2AC (Long)
(~50 sec.)
Next, Floating PICs are bad:

1. Steals all aff ground- the plan is the foundation for all affirmative offense in
debate, allowing the negative to defend the plan crushes our ability to answer
arguments, including their K. In a world where affirmatives are able to generate
foundational offense separate from the plan, the negatives ability to debate is
severely compromised, plan focus is best for both teams.
2. Not educational- there is little education to be gained from allowing the
negative to agree that the plan is a good idea in totality and that there was
something wrong with the Construction of the iac, this justifies allowing the
negative to Criticize the spelling of our tags, while advocating the plan.
Affirmatives rarely win in this world.
3. Undermines Reciprocal Burdens- allowing the negative to advocate the plan
means that the negatives burden has shifted from disproving the plan to
disproving anything that the affirmative has said; that is too easy on negatives,
especially on a tiny topic with lots of generic negative ground. Their argument
justifies affirmatives defending the text of the INC but not the justifications of
the INC. It also justifies severing out of everything that is not the plan.
4. We Turn their offensive arguments- They should have to win the framework
debate in order to win that their K comes before the affirmative, allowing them
to win because there is a small risk that something was wrong with the aff,
separate from the plan, means that we dodge a discussion of methodology and
epistemology and its relationship to the aff, they should have to win that there
is a meaningful relationship, not that there could be a meaningful relationship.
They dodge a discussion of these questions, preventing any benefits of making
affs defend their whole iac.
5. This has to be a voting issue, we have to go for this argument just to get back to
ground zero; this should be a non-issue.

Page 22 of 1481
Page 23 of 1481
Page 24 of 1481
Floating PICs Bad: 2AC (Short)
(<20 sec.)
Next, Floating PICs are bad

1. Steals Aff ground- Floating PICs steal the only option that affs have to generate
offense, the plan.
2. Not educational- Floating PICs justify negatives defending the plan and
criticizing the spelling of our tags, crushing education.
3. Not Reciprocal- the affirmative cannot agree with a bulk of the neg strat and k
their reps, we would have to win a framework arg too.
4. We turn their offense- they sidestep a discussion of epistemology and its
effects on policymaking, not defending the plan provides more meaningful
education.
5. This has to be a voting issue, we have to go for this argument just to get back to
ground zero.

Page 25 of 1481
Page 26 of 1481
Do the Plan Perm: 2AC

Perm- do the plan.

Perm solves-
1. That the negative can divorce themselves from the bad representations of the
IAC surely means that we can too. If it really is just as easy as saying, we defend
the plan but not the representations of the IAC; then there is no reason why we
would not be able to do the same thing.

2. No theoretical reason why the perm is illegit, they might win substantive
reasons why the our representations are tied to our plan, but that is a reason
why they also would not be able to advocate it separate from the rest of the
IAC, if the very utterance of the rest of the iac ties it to the plan, then that is
irrevocable.

3. And we will defend that the perm is a test of the competitiveness of part of
their alternative- the part that advocates the plan, which is decidedly not
competitive, a remedy to this non-competitive nature would be to disallow the
negative to advocate the plan.

Page 27 of 1481
Page 28 of 1481
#1 Steals Aff Ground: 1AR
Extend the 2AC #1- Floating PICs destroy all affirmative Ground; the plan is the
only way for affirmatives to generate offense in debate. If the negative is allowed
to defend the plan as well, then there is no residual IAC offense that we can claim,
and the 2AC has to start from scratch, meaning that affirmatives always start at a
disadvantage. This pits the block against the IAR, which means affs rarely ever win.

If instead the aff is able to generate offense in the IAC that does not stem from the
plan but something else, then debate for the negative becomes difficult as they
not only have to disprove the plan but everything else.

Page 29 of 1481
Page 30 of 1481
#4 Ext. Turns Offense: 1AR
Extend the 2AC #4-

Any reason that they win that it is important for us to defend the non-plan parts of
the IAC, we will win are reasons why they shouldnt defend the plan.

If the negative did not defend our plan, but solely engaged in a criticism of our
representations, then that would facilitate a discussion of how our
representations related to and affected our plan. By choosing to defend the plan
absent from the rest of the IAC, they have limited our discussion to just one of
language, rather than including broader issues of epistemology. This short-circuits
any reason why it would be good or educational to examine the representations
because they have severed them from

Page 31 of 1481
Page 32 of 1481
A2 Plan Focus Bad: 1AR
1. We will outweigh any of their arguments plan focus is bad

A. Ground- Both teams benefit immensely from plan focus debate, their
argument would not be possible in a world where we didnt read a plan,
most negative args would be rendered meaningless
B. Education- the alternative is res-focused debate, which prevents us from
delving into the more interesting aspects of the resolution by
parametrisizing it.

Page 33 of 1481
Page 34 of 1481
A2 Plan is only a tiny part of
the speech/Discourse of 1AC is
~9 min.: 1AR
They say that the plan is relatively unimportant, this is just not true:

1. The plan is the foundation for the rest of the affirmative, taking the plan out
of the affirmative would render the IAC fairly nonsensical, just because the
plan can be read quickly does not render it meaningless.
2. This is untrue from the standpoint of the negative as well, the plan is what
they get before the round, not the entire text of the affirmative, it is the
focus of the debate in a literal as well as figurative sense.

Page 35 of 1481
Page 36 of 1481
Must Have an Alternative: 2AC
NEXT, LACK OF ALT IS BAD

A. We need a text to provide us with ground to perm the kritiksuch arguments are critical tests of
the link

B. Utopian alternatives destroy debate because we can never win that the plan is better than
perfection

C. Vague alternatives are moving targets that prevent us from linking offense
D. It guts their solvency because their argument will never gain political traction, all of which are voters for
fairness and education

Page 37 of 1481
Hasty Generalization Bad: 2AC

HASTY GENERALIZATION

A. There are many instances where advocating government change is goodthese instances would
still vote to the K

B. Call to reject doesnt justify its utilitarian basisthere are still plenty of reasons to do the plan

Page 38 of 1481
Page 39 of 1481
Law Transformative: 2AC (1/2)

IT IS IMPORTANT THAT EACH ONE OF US DEFENDS THE TRANSFORMATIVE POWER


OF THE LAW- WORLDWIDE RIGHTS AND FREEDOM DEPEND ON IT

KENNEDY 06
(Anthony, Supreme Court Justice, Remarks at the Annual Meeting of the American Bar Association,
Federal News Service, August 12, 2006, Lexis)

I sense, President Greco, as indicated in your remarks, that we are at another turning point in the history of the law. The Constitution gave us
judges. It's really remarkable that it did. Remember that attacks and complaints against judges were one of the indictments, one of the allegations, in the Declaration of
Independence. The framers had been pushed around by judges. And what did they do? They created a judiciary and gave them life tenure. Why did they do that? Because they
were confident that the process of reason, the slow elaboration of the principles of justice through the case-by-case method, was the surest way to interpret the Constitution.
The framers knew that they were not prescient enough, and they were not brazen enough, to specify all of the elements of justice. They knew this could become apparent only
over time. They knew that the whole purpose of the Constitution is to rise above the inequities and the injustices that you can't see. But now we are in an era where I sense
something different happening. We know the truth needs no translation. There's a word for truth in every language. We know that the world is getting smaller. We know that

the rule of law is essential. We hear a lot about security. But our best security, ultimately our only security, is in the world of ideas. And I sense a slight
foreboding. I sense that we are not making the case as well as we ought. It could be, to use a Pacific metaphor, that the tide has gone out and

we're on the beach. But a tsunami of expectations and discontent and demands and dissatisfaction may soon

sweep in upon us. We must explain to the rest of the world the meaning, the essentiality and the purpose
of the rule of law as it's understood by the American people and by other democracies throughout the
world. And we must begin to do a better job of it, and we must begin that now. (Applause.) I was here in Hawaii, Governor
Lingle, just a few months ago and met with the University of Hawaii law students. And I asked them, "What does the rule of law mean?" You know, I never heard that term when
I was in law school. And lawyers bandy it about a lot. Should it not be defined? If you parse it as a grammarian might, it doesn't always work. You might have a dictator with
laws that are known and that are enforced, but that can't be the rule of law. The rule of law does not exist just because a dictator makes the trains run on time. And so I tried to
define the rule of law. And before doing so, there were certain caveats. There are certain risks. The phrase has a resonance, an allure, that you're reluctant to destroy. And
we're often reluctant to talk about universal truths lest our efforts at formulating their specifics seem too bland, too insufficient, for the great purpose behind the phrase. So
there's a risk, when we talk about the rule of law, that you say too little or that you say too much; that you say too little and you're facile, thereby preventing us from discovering
other truths; that you say too much and that you're prolix. There's a reluctance to open the bidding so that every interest group has its particular interest, its particular goal,
incorporated in the rule of law. I always wanted to teach a law school course in constitutional law to some very bright students who had never read the Constitution. And the
way I'd do it is I'd say, "Now, here it is, but you can't read it. I want you to tell me what you think the Constitution should contain if it's a model Constitution." They'd look. I'd say,
"Now, don't peek." And just as an academic trick, I would get them interested. I've done the same thing for you, and I'm glad it's dark, because I don't want you to look at it. I've
given you a little definition of the rule of law. I have one for all the Kameamea students. What would you put in your definition of the rule of law? Would you talk about process,
knowing that there are certain truths that are not evident to us now, that we're blind to the injustices and the prejudices of our own times? So you just talk about process? That
really doesn't suffice. It's not elevating enough. So you must talk about substance. What is the substance which you include? I suggested that the rule of law has three parts.
This is simply a working definition. If we were in the law school class at the University of Hawaii, or if we had more time, you could probably make some suggestions for how
this should be improved. But I think it's important for us to begin assessing where we are in this campaign to explain the meaning of freedom, the meaning of the rule of law, to

There's a jury that's out. It's half the world. The verdict is not yet in. The
a doubting world. My friends, make no mistake:

commitment to accept the western idea of democracy has not yet been made, and they are waiting for
you to make the case. I suggest that the rule of law has three parts. The first is that the law is binding on the
government and all of its officials. This may seem a rather self-evident matter, but it's a proposition that most government officials in most countries do
not fully understand. If an administrative agency and an administrator in that agency is charged with giving you a permit, the permit is not given to you as a matter of grace. It's

The rule of law binds the


given to you because you're entitled to it, and it's his or her duty to give it to you. Very few countries in the world understand this.

government and all of its officials. This is an essential lesson that must be taught if the corruption and the greed and the graft President Greco referred to
are eliminated. The second part of the rule of law is there for you on the little slip. It is, I think, in a sense, the most troubling for me. I'm not sure that it's complete. It says that

the rule of law must respect the dignity, equality and human rights of every person. And then there's a second sentence,
and the second sentence says that the people are entitled to have a voice in the laws that govern them . So there's a process

element. But it isn't just process, because the right to participate in government is nothing less than the right to help shape

your own destiny. And the framers of our Constitution made it very clear that each generation has a share, has a chance to
determine its own destiny, to determine its own direction. What are human rights? Is it the right to subsistence, the right to enough to
eat, the right to breathe clean air, the right to an education? At this point the rule of law, as we, I think, would want to define it, may depart from the idea of a model constitution.
These are two different things. In the Constitution of the United States, there are a series of essentially negative commands. "Congress shall make no law restricting free
speech or the free press." "There shall be no unreasonable search and seizures." These are negative commands. It's easier to have the Ten Commandments -- "Thou shalt not
steal" -- than the Sermon on the Mount -- "Thou shalt love thy neighbor." It's harder to enforce the latter. But what about affirmative rights? Aren't there some basic human
entitlements? You see a man on a steam grate in the cold winter in Washington, D.C. and you say, "Well, you have the right to a jury trial, and you actually have a right to own

if the rule of law is to have meaning, substance,


a newspaper." He'd say, "I'm cold. I'm hungry. I want to eat." Americans must understand that

hope, inspiration for the rest of the world, it must be coupled with the opportunity to improve human
existence. I became interested a few years ago in water systems in Africa, and I have attended a few lectures about it. Not long ago I heard a speaker say the following.
He asked this question: "How many hours of human labor per year are spent in the continent of Africa getting clean water?" This is work that falls on the shoulders of women.
The answer was 8 billion hours a year. I was sitting in an audience like yours, thinking, "Now, did he say 8 million? No, that can't work out. Was it 80 million?" The answer is 8
The
billion. And I asked him about it later. He said, "This is very conservative, because I'm just talking about the water that's clean when it gets back to the source."

biggest single cause of infant mortality in Africa and other undeveloped nations is diarrhea. Children with
a slight body mass dehydrate quickly, and there's nothing for the heart to pump against. The heart can't
pump if it's dry. This can be fixed. This is not rocket science. One of the reasons it can't be fixed, under present

Page 40 of 1481
conditions, is that governments are corrupt. And people have a right to improve their lives, to gain basic
security, without corrupt governments depriving them of the very means of existence. CONTINUED ON
NEXT PAGE-

Page 41 of 1481
Page 42 of 1481
Law Transformative (2/2)

--KENNEDY 06 CONT--

My third suggestion -- and it can only be a suggestion; it would be presumptuous to say that I can define the rule of law -- my third suggestion for you to think about surprised

every person has a right to know what the laws are and to enforce them without
me when I wrote it, and it was this, that

fear of retaliation or retribution. This is almost a process-sounding precept, but it's again substantive as well. It's part of your identity,
it's part of your self-definition, to know the laws that protect you, to know the laws that are respected by
your neighbors and friends and family. This is part of who you are. And you're entitled to know this, and
you're entitled to enforce them. I was talking with some lawyers and judges not long ago from Bangladesh. They told me that a standard criminal
sentence works something like this: A fine of three dollars or nine to 12 months in jail, and at least 1,000 people a year spend a year in jail for want of the three dollars. I said,
"Well, I'm not a man of great means, but I'll write you a check for $1,000. That'll take care of 333 people." And they said, "Well, no, but then there'd be no deterrence." Is a

we must
nation, is a people, is a culture, is a society able to embrace the western idea of the rule of law under such conditions? I suggest to you the answer is no. And

find some ways to link the rule of law with real progress in improving the condition of humankind. We must
have some measures to assure that the vast aid, the work of the NGOs, the work of this association, has some immediate, visible, tangible return so that we can make the
case. You were gracious to mention my remarks, President Greco, in San Francisco, when you last met in that city. We talked about the criminal justice system. And I

mentioned at the time a book by Alexander Solzhenitsyn called "One Day in the Life of Ivan Denisovich." And it occurred to me, when we were coming here to
Hawaii, that Solzhenitsyn might be relevant in a somewhat different connection. He was a writer whom I greatly admired. He had escaped from the Soviet Union and from a
gulag in order to write about that experience, and he was living in the United States. He was invited to Harvard to give the most important address given every year to the
Harvard students. It was in the mid or late '70s. I was living in California at the time. I was thrilled that my hero was addressing the Harvard College. And this was pre-fax and
Internet days, so it took me one or two days to get the text of his remarks, the text of his remarks from The New York Times. And I was shocked, stunned, terribly disappointed

to read his remarks, in which heattacked the West, and particularly the law and the legal system. And he said that any
society that defines the tissues of human existence in legalistic terms is condemned to spiritual
mediocrity. My hero was saying this about my profession, about the Constitution that is America's self-identity, about the Constitution that Americans still think as
defining who they are as a people?I reflected on it for a few days, and then I got the answer. We just define law differently than Solzhenitsyn

did. From his era, from his culture, law was a dictat, a ucas (ph) -- a command, a mandate. In sum, it was a cold decree.
That's not the meaning of law as our nation and our co- democracies define it. For us, law is a liberating
force. It's a promise. It's a covenant. It says that you can hope, you can dream, you can dare, you can plan.
You have joy in your existence. That's the meaning of the law as Americans understand it, and that's the
meaning of the law that we must explain to a doubting world where the verdict is still out. You can make
this case. You must make this case. And that is because freedom -- your freedom, my freedom and the
freedom of the next generation -- hangs in the balance. I'm confident you will do this.

Page 43 of 1481
Page 44 of 1481
Policymaking Good: 1AR (1/2)
FIRST, EXTEND THE COVERSTONE 95 EVIDENCE. POLICY DEBATE CREATES A SAFE
SPACE ALLOWING US TO TEST IDEAS, BECOMING EDUCATED ENOUGH TO HOLD
ELITES ACCOUNTABLE, STOPPING THE RISE OF NEW OPPRESSION

SECOND, DEBATE IS CIVIL SOCIETY: IT IS THE ROLE OF CRITICAL INTELLECTUALS TO


FORM A PUBLIC POLICY SPHERE CONSTITUTED AROUND SPECIFIC POLICY IDEAS.
WE ARE NOT THE GOVERNMENT, BUT BY ORIENTING OURSELVES TOWARDS THE
STATE WE CAN ENSURE EFFECTIVE POLITICS.

HABERMAS 98
[Jurgen, Prof. Philosophy at U. of Frankfurt, The Inclusion of the Other, p. 31//uwyo-
crowe]

A law is valid in the moral sense when it could be accepted by everybody from the perspective
of each individual. Because only general laws fulfill the condition that they regulate matters in the equal interest of all, practical
reason finds expression in the generalizability or universalizability of the interests expressed in
the law. Thus a person takes the moral point of view when he deliberates like a democratic
legislator on whether the practice that would result from the general observance of a
hypothetically proposed norm could be accepted by all those possibly affected viewed as
potential co-legislators. Each person participates in the role of co-legislator in a cooperative
enterprise and thereby adopts an intersubjectively extended perpective from which it can be
determined whether a controversial norm can count as generalizable from the point of view of
each participant. Pragmatic and ethical reasons, which retain their internal connection to the interests and
self0understanding of individual persons, also play a role in these deliberations; but these agent-relative reasons no longer
count as rational motives and value-orientations of individual persons but as epistemic contributions to a discourse in which norms are
Because a legislative practice can only be
examined with the aim of reaching a communicative agreement.
undertaken jointly, a monological, egocentric operation of the generalization test in the manner
of the Golden Rule will not suffice.

Page 45 of 1481
Page 46 of 1481
Policymaking Good: 1AR (2/2)
THIRD, TECHNICAL, COMPETITIVE DEBATE IS A DIALECTICAL METHOD THAT
TEACHES STUDENTS ABOUT INTERPLAY BETWEEN ARGUMENTS, TRAINING THEM
FOR POLICY ENGAGEMENT

Mitchell 2000
[Gordon R., the brilliant DOD at Pitt, Preface to Strategic Deception: Rhetoric, Science and Politics in Missle Defense
Advocacy, Michigan State University Press, 2000, xvi//uwyo]

intercollegiate policy debate is an odd and magical place, where a keen spirit
The world of
of competition drives debaters to amass voluminous research in preparation for tournaments, and
where the resulting density of ideas spurts speakers to cram arguments into strictly timed presentation periods during
contest rounds. Expert judges trained in policy analysis keep track of such contests as they
unfold at breakneck speed, with speakers routinely delivering intricate argumentation at
over 300 words per minute. To the uninitiated onlooker, this style of debate reveals itself as an unintelligible
charade, something like a movie-length Federal Express commercial or an auctioneering competition gone bad. But there
are rich rewards for participants who master policy debate's special vocabulary, learn its
arcane rules, and acclimate themselves to the style of rapid-fire speaking needed to keep up
with the flow of arguments. The rigorous dialectical method of debate analysis cultivates a
panoramic style of critical thinking that elucidates subtle interconnections among
multiple positions and perspectives on policy controversies. The intense pressure of
debate competition instills a relentless research ethic in participants. An inverted
pyramid dynamic embedded in the format of contest rounds teaches debaters to
synthesize and distill their initial positions down to the most cogent propositions for their
final speeches.

FOURTH, ONLY STATE-CENTERED DISCUSSION ABOUT POLITICS CAN REVERSE THE


TREND TOWARD TOTALITARIANISM. THIS DESTROYS DEBATE

TORGERSON 99
[Douglas, Prof and Chair Dept. Political Studies @ Trent U., The Promise of Green Politics: Environmentalism and the
Public Sphere, Duke University Press//uwyo-crowe]

One rationale for Arendt's emphasis on the intrinsic value of politics is that this value has been so neglected by
modernity that politics itself is threatened. Without a celebration of the intrinsic value of politics,
neither functional nor constitutive political activity has any apparent rationale for continuing
once its ends have been achieved. Functional politics might well be replaced by a technocratic
management of advanced industrial society. A constitutive politics intent on social transformation might well be eclipsed
by the coordinated direction of a cohesive social movement. In neither ease would any need be left for what Arendt takes to be the essence of
politics: there would be no need for debate.

Green authoritarianism, following in the footsteps of Hobbes, has been all too ready to reduce politics to
governance. Similarly, proponents of deep ecology, usually vague about politics, at least have been able to recognize totalitarian dangers
in a position that disparages public opinion in favor of objective management." Any attempt to plot a comprehensive strategy for a cohesive

Page 47 of 1481
green movement, moreover, ultimately has to adopt a no-nonsense posture while erecting clear standards by which to identify and
excommunicate the enemy that is within.

Green politics from its inception, however, has challenged the officialdom of advanced industrial society by invoking the cultural idiom of the
carnivalesque. Although tempted by visions of tragic heroism, as we saw in chapter, green politics has also celebrated the irreverence of the
comic, of a world turned upside down to crown the fool. In a context of political theater, instrumentalism is often attenuated, at least
momentarily displaced by a joy of performance. The comic dimension of political action can also be more than episodic. The image of the
Lilliputians tying up the giant suggests well the strength and flexibility of a decentered constitutive politics. In a functional context, green politics
offers its own technology of foolishness in response to the dysfunctions of industrialism, even to the point of exceeding the comfortable limits of
a so-called responsible foolishness.

Highlighting the comic, these tendencies within green politics begin to suggest an intrinsic value to politics. To the extent that this
value is recognized, politics is inimical to authoritarianism and offers a poison pill to the
totalitarian propensities of an industrialized mass society." To value political action for its own
sake, in other words, at least has the significant extrinsic value of defending against the antipolitical
inclinations of modernity. But what is the intrinsic value of politics? Arendt would locate this value in the virtuosity of political
action, particularly as displayed in debate. Although political debate surely has extrinsic value, this does not exhaust its value. Debate is a
language game that, to be played well, cannot simply be instrumentalized for the services it can
render but must also he played for its own sake. Any game pressed into the service of external goals tends to lose its
playful quality; it ceases to be fun.

Page 48 of 1481
Page 49 of 1481
Policymaking Good: Ext (1/3)

ACADEMIC SWITCH-SIDE DEBATING TEACHES STUDENTS HOW TO ORGANIZE


INFORMATION AND DEFEND ARGUMENTS, RESISTING TOTALITARIAN
INFORMATION OVERLOAD

Coverstone 95
[Alan, Princeton High School, An Inward Glance: A Response to Mitchells Outward
Activist Turn, www.wfu.edu/Student-
organizations/debate/MiscSites/DRGArticles/Coverstone1995China.htm, acc 3-16-
05//uwyo-ajl]

Mitchell's argument underestimates the nature of academic debate in three ways. First, debate
trains students in the very skills required for navigation in the public sphere of the
information age. In the past, political discourse was controlled by those elements who
controlled access to information. While this basic reality will continue in the future, its
essential features will change. No longer will mere possession of information determine
control of political life. Information is widely available. For the first time in human history we
face the prospect of an entirely new threat. The risk of an information overload is already
shifting control of political discourse to superior information managers. It is no longer
possible to control political discourse by limiting access to information. Instead, control
belongs to those who are capable of identifying and delivering bits of information to a
thirsty public. Mitchell calls this the "desertification of the public sphere."

The public senses a deep desire for the ability to manage the information around them. Yet,
they are unsure how to process and make sense of it all. In this environment, snake charmers
and charlatans abound. The popularity of the evening news wanes as more and more
information becomes available. People realize that these half hour glimpses at the news do not
even come close to covering all available information. They desperately want to select
information for themselves. So they watch CNN until they fall asleep. Gavel to gavel coverage of
political events assumes top spots on the Nielsen charts. Desperate to decide for themselves,
the public of the twenty-first century drinks deeply from the well of information. When
they are finished, they find they are no more able to decide. Those who make decisions are
envied and glorified.

Debate teaches individual decision-making for the information age. No other academic
activity available today teaches people more about information gathering, assessment,
selection, and delivery. Most importantly, debate teaches individuals how to make and
defend their own decisions. Debate is the only academic activity that moves at the speed
of the information age. Time is required for individuals to achieve escape velocity. Academic
debate holds tremendous value as a space for training.

Mitchell's reflections are necessarily more accurate in his own situation. Over a decade of
debate has well positioned him to participate actively and directly in the political process. Yet
the skills he has did not develop overnight. Proper training requires time. While there is a
tremendous variation in the amount of training required for effective navigation of the public
sphere, the relative isolation of academic debate is one of its virtues. Instead of turning
students of debate immediately outward, we should be encouraging more to enter the oasis. A

Page 50 of 1481
thirsty public, drunk on the product of anyone who claims a decision, needs to drink from the
pool of decision-making skills. Teaching these skills is our virtue.

Page 51 of 1481
Page 52 of 1481
Policymaking Good: Ext (2/3)
DEBATE TRAINS STUDENTS TO BECOME ACTIVISTS BY TESTING THEIR OPINIONS
AND BECAUSE OF ITS COVERT NATURE BECOMING OUTWARDLY POLITICAL
THREATENS TO HAVE US INFILTRATED

Coverstone 95
[Alan, Princeton High School, An Inward Glance: A Response to Mitchells Outward
Activist Turn, www.wfu.edu/Student-
organizations/debate/MiscSites/DRGArticles/Coverstone1995China.htm, acc 3-16-
05//uwyo-ajl]

Mitchell's argument underestimates the risks associated with an outward turn. Individuals
trained in the art and practice of debate are, indeed, well suited to the task of entering the
political world. At some unspecified point in one's training, the same motivation and focus
that has consumed Mitchell will also consume most of us. At that point, political action
becomes a proper endeavor. However, all of the members of the academic debate
community will not reach that point together. A political outward turn threatens to corrupt
the oasis in two ways. It makes our oasis a target, and it threatens to politicize the training
process.

As long as debate appears to be focused inwardly, political elites will not feel threatened.
Yet one of Mitchell's primary concerns is recognition of our oasis in the political world. In this
world we face well trained information managers. Sensing a threat from "debate," they will
begin to infiltrate our space. Ready made information will increase and debaters will eat
it up. Not yet able to truly discern the relative values of information, young debaters will
eventually be influenced dramatically by the infiltration of political elites. Retaining our
present anonymity in political life offers a better hope for reinvigorating political
discourse.

As perhaps the only truly non-partisan space in American political society, academic debate
holds the last real possibility for training active political participants. Nowhere else are
people allowed, let alone encouraged, to test all manner of political ideas. This is the
process through which debaters learn what they believe and why they believe it. In many
ways this natural evolution is made possible by the isolation of the debate community. An
example should help illustrate this idea.

Like many young debaters, I learned a great deal about socialism early on. This was not
crammed down my throat. Rather, I learned about the issue in the free flow of information that
is debate. The intrigue of this, and other outmoded political arguments, was in its relative
unfamiliarity. Reading socialist literature avidly, I was ready to take on the world. Yet I only had
one side of the story. I was an easy mark for the present political powers. Nevertheless, I
decided to fight City Hall. I had received a parking ticket which I felt was unfairly issued. Unable
to convince the parking department to see it my way, I went straight to the top. I wrote the
Mayor a letter. In this letter, I accused the city of exploitation of its citizens for the purpose of
capital accumulation. I presented a strong Marxist critique of parking meters in my town. The
mayor's reply was simple and straightforward. He called me a communist. He said I was being
silly and should pay the ticket. I was completely embarrassed by the entire exchange. I thought I
was ready to start the revolution. In reality, I wasn't even ready to speak to the Mayor. I did

Page 53 of 1481
learn from the experience, but I did not learn what Gordon might have hoped. I learned to stop
reading useless material and to keep my opinions to myself.

Do we really want to force students into that type of situation? I wrote the mayor on my own.
Debaters will experiment with political activism on their own. This is all part of the natural
impulse for activism which debate inspires. Yet, in the absence of such individual
motivation, an outward turn threatens to short circuit the learning process. Debate
should capitalize on its isolation. We can teach our students to examine all sides of an
issue and reach individual conclusions before we force them into political exchanges. To
prematurely turn debaters out threatens to undo the positive potential of involvement in
debate.

Page 54 of 1481
Page 55 of 1481
Policymaking Good: Ext (3/3)
OUTWARD ACTIVISM RISKS CREATING A NEW HOMOGENEOUS ELITE, CRUSHING
IDEOLOGICAL DISSENT, TURNING THEIR ARGUMENT

Coverstone 95
[Alan, Princeton High School, An Inward Glance: A Response to Mitchells Outward
Activist Turn, www.wfu.edu/Student-
organizations/debate/MiscSites/DRGArticles/Coverstone1995China.htm, acc 3-16-
05//uwyo-ajl]

My third, and final reaction to Mitchell's proposal, targets his desire for mass action. The
danger is that we will replace mass control of the media/government elite with a mass
control of our own elite. The greatest virtue of academic debate is its ability to teach
people that they can and must make their own decisions. An outward turn, organized
along the lines of mass action, threatens to homogenize the individual members of the
debate community. Such an outcome will, at best, politicize and fracture our community. At
worst, it will coerce people to participate before making their own decisions.

Debate trains people to make decisions by investigating the subtle nuances of public policies.
We are at our best when we teach students to tear apart the broad themes around which
traditional political activity is organized. As a result, we experience a wide array of
political views within academic debate. Even people who support the same proposals or
candidates do so for different and inconsistent reasons. Only in academic debate will two
supporters of political views argue vehemently against each other. As a group, this reality
means that mass political action is doomed to fail. Debaters do not focus on the broad themes
that enable mass unity. The only theme that unites debaters is the realization that we are all
free to make our own decisions. Debaters learn to agree or disagree with opponents with
respect. Yet unity around this theme is not easily translated into unity on a partisan
political issue. Still worse, Mitchell's proposal undermines the one unifying principle.

Mitchell must be looking for more. He is looking for a community wide value set that
discourages inaction. This means that an activist turn necessarily will compel political action
from many who are not yet prepared. The greatest danger in this proposal is the
likelihood that the control of the media/government elite will be replaced by control of our
own debate elite.

Emphasizing mass action tends to discourage individual political action. Some will decide
that they do not need to get involved, but this is by far the lesser of two evils. Most will decide
that they must be involved whether or not they feel strongly committed to the issue. Mitchell
places the cart before the horse. Rather than letting ideas and opinions drive action as they
do now, he encourages an environment where action drives ideas for many people.
Young debaters are particularly vulnerable. They are likely to join in political action out of
a desire to "fit in." This cannot be what Mitchell desires. Political discourse is a dessert now
because there are more people trying to "fit in" that there are people trying to break out.

Page 56 of 1481
A2 Only Learn As Spectators: 1AR

FIRST, NOT TRUE DEBATES ABOUT DETAINMENT TRAIN US TO HOLD


POLICYMAKERS ACCOUNTABLE FOR THEIR DECISIONS. IF WE CANT HAVE A
DEBATE, WE WONT KNOW WHAT TO DO WHEN WE CONFRONT REACTIONARIES.
CROSS-APPLY COVERSTONE

SECOND, TURN VIEWING DEBATE DECISIONS AS ACTIVISM, IN AND OF


THEMSELVES, CRUSHES ACTUAL POLITICAL ACTIVITY. WINNING A TOURNAMENT
BECOMES GOOD ENOUGH CREATING NIHILISTS WHO NEVER ACTUALLY LOBBY THE
GOVERNMENT.

THIRD, THIS IS EMPIRICALLY DENIED BY THE MASSES OF DEBATERS WHO GO ON


TO BECOME SOCIAL ACTIVISTS AND PROGRESSIVE ATTORNEYS. WE WOULDNT
HAVE PEOPLE LIKE GORDON MITCHELL DOING WORK IN MISSILE DEFENSE
OPACITY IF IT WERENT FOR THE SAFE SPACE OF SWITCH SIDE DEBATE

FOURTH, WORLDY ACADEMIC WORK IS DEMOCRATIZING AND SPURS ACTIVISM

Gordon R. Mitchell, Assistant Professor of Communication, University of Pittsburgh,

ARGUMENTATION AND ADVOCACY, Fall 1998, p. 47.

argumentative agency involves the capacity to contextualize and employ the skills and
In basic terms the notion of

strategies of argumentative discourse in fields of social action, especially wider spheres of public deliberation. Pursuit of argumentative
agency charges academic work with democratic energy by linking teachers and students with civic organizations, social movements, citizens and other actors engaged in live

argumentative agency links decontextualized argumentation


public controversies beyond the schoolyard walls. As a bridging concept,

skills such as research, listening, analysis, refutation and presentation, to the broader political telos of
democratic empowerment. Argumentative agency fills gaps left in purely simulation-based models of
argumentation by focusing pedagogical energies on strategies for utilizing argumentation as a driver of progressive social change. Moving beyond an exclusively
skill-oriented curriculum, teachers and students pursuing argumentative agency seek to put argumentative tools to the test by employing them in situations beyond the space of
the classroom. This approach draws from the work of Kincheloe (1991), who suggests that through "critical constructivist action research," students and teachers cultivate their

own senses of agency and work to transform the world around them .

Page 57 of 1481
Page 58 of 1481
Policy Debate Good
CRITICAL THEORY DIMINISHES THE BENEFIT OF POLICY DEBATE

Jentleson 2002
[Bruce, Dir. Terry Sanford Inst. Public Policy and Prof. Pub Plcy and Pol. Sci. @ Duke, The Need for Praxis:
Bringing Policy Debate Back In, International Security 26:4, Spring, ASP//uwyo-ajl]

To be sure, political science and international relations have produced and

continue to produce scholarly work that does bring important policy insights.

Still it is hard to deny that contemporary political science and international

relations as a discipline put limited value on policy relevancetoo little, in my

view, and the discipline suffers for it. The problem is not just the gap between

theory and policy but its chasmlike widening in recent years and the limited

valuation of efforts, in Alexander Georges phrase, at bridging the gap. The

events of September 11 drive home the need to bring policy relevance back in

to the discipline, to seek greater praxis between theory and practice.

AND DECENTRALIZED PUBLIC DEBATE IS NECESSARY OT TRANSFORM BUREACRACY

Martin 90
[Brian, Bureacracy, www.uow.edu.au/arts/sts/bmartin/pubs/90uw/uw08.html, 9-23-06//uwyo-ajl]

All of this can be quite useful and often effective, and should not be rejected. But working through
bureaucracy on the inside, or demanding policy changes from the outside, does little to transform
bureaucracy itself. In fact, working through bureaucracy can reinforce the legitimacy and sway of
bureaucracy itself. In addition, campaigns oriented towards working through bureaucracy or applying
pressure for change at the top tend to become bureaucratised themselves.

Another important orientation adopted by many social activists is towards building self-managing
organisational forms for their own activities, such as cooperative enterprises or egalitarian action groups.
Self-managing organisational forms are an alternative to bureaucracy. Direct experience in self-managing
groups strengthens the sense of community and commitment to social action and also provides
understanding and individual strength to resist pressures for bureaucratisation in the wider society. In as
much as social movements organise themselves as decentralised self-managing groups, linked by
federations and networks, and self-consciously set out to develop and extend such structures, they
provide a strong challenge to the domination of bureaucratic forms of social organisation.

Page 59 of 1481
Page 60 of 1481
Switch-side Debate Good (1/3)
CRITICAL DISTANCE & *PUBLICLY* ADVOCATING ARGUMENTS WITH WHICH YOU
DISAGREE ARE ETHICALLY IMPORTANT:

Dennis G. Day, Professor, Speech, University of Wisconsin-Madison, CENTRAL STATES SPEECH JOURNAL,
February 1966, p. 7.

All must recognize and accept personal responsibility to present, when necessary, as forcefully as
possible, opinions and arguments with which they may personally disagree.
To present persuasively the arguments for a position with which one disagrees is, perhaps, the greatest
need and the highest ethical act in democratic debate. It is the greatest need because most minority
views, if expressed at all, are not expressed forcefully and persuasively. Bryce, in his perceptive analysis of
America and Americans, saw two dangers to democratic government: the danger of not ascertaining
accurately the will of the majority and the danger that minorities might not effectively express
themselves. In regard to the second danger, which he considered the greater of the two, he suggested:
The duty, therefore, of a patriotic statesman in a country where public opinion rules, would seem to be
rather to resist and correct than to encourage the dominant sentiment. He will not be content with trying
to form and mould and lead it, but he will confront it, lecture it, remind it that it is fallible, rouse it -out of
its self-complacency
To present persuasively arguments for a position with which one disagrees is the highest ethical act in
debate because it sets aside personal interests for the benefit of the common good. Essentially, for the
person who accepts decision by debate, the ethics of the decision-making process are superior to the
ethics of personal conviction on particular subjects for debate. Democracy is a commitment to means, not
ends. Democratic society accepts certain ends, i.e., decisions, because they have been arrived at by
democratic means. We recognize the moral priority of decision by debate when we agree to be bound by
that decision regardless of personal conviction. Such an agreement is morally acceptable because the
decision-making process guarantees our moral integrity by guaranteeing the opportunity to debate for a
reversal of the decision.
Thus, personal conviction can have moral significance in social decision-making only so long as the
integrity of debate is maintained. And the integrity of debate is maintained only when there is a full and
forceful confrontation of arguments and evidence relevant to decision. When an argument is not
presented or is not presented as persuasively as possible, then debate fails. As debate fails decisions
become less "wise." As decisions become less wise the process of decision-making is questioned.

And finally, if and when debate is set aside for the alternative method of decision-making by authority,
the personal convictions of individuals within society lose their moral significance as determinants of
social choice.

Page 61 of 1481
Page 62 of 1481
Switch-Side Debate Good (2/3)
SWITCH SIDE DEBATING IS PROFOUNDLY MORAL AND GUARDS AGAINST
ABSOLUTISM

Gary Alan Fine, Professor of Sociology at Northwestern University, Gifted Tongues, 2001, p. 54-
55.

Despite these concerns, most individuals with whom I discussed the issue felt that debating both sides of
an issue was valuable, perhaps the greatest benefit of the activity, teaching the value of respect for
differing opinions, multiple perspectives, and the dangers of absolutism. For some the ability to argue
both sides of an issue is profoundly moral:

I have seen some people become cynical as a result. I would hope with students I teach that they learn
some ethical responsibilities. But I think what debate does is allow students to seriously consider
important questions from both sides of the issue and see other perspectives before they become
committed themselves to a position. I have students who will say, Well, I cant argue against this,
because I really believe it. But after theyve done some research they are not so certain of their
convictions. They at least can see the other side. I think they become more humane as a result of looking
at both sides. (interview)

The ability to see both points of view has the potential in this view to make one more humane and less
self-righteous. Others suggest that not only does debating both sides of a position not weaken ones
position, but it strengthens it, perhaps by inoculating one to opposing arguments. Many debaters have
strong political positions, which the activity seems to do nothing to diminish:

I think what happens is that you leam that there are two sides to every issue. I think most debaters come
down on one side or the other in their mind, but they are able to argue both sides. And I think that is an
important thing to be able to do. I mean because it makes what you believe in, it makes that belief even
more justified, because you do know both sides. (interview)

The ability to take a position that is contrary to ones own beliefs has several benefits: making one
appreciate the perspective of ones foes, making ones own thoughts more complex, and helping one
become aware of counterarguments. Perhaps this stance does suggest that positions are gamelike, but
it is a game that corresponds to the way that much political decision making operates in the real world.

Page 63 of 1481
Page 64 of 1481
Switch-Side Debate Good (3/3)
SWITCH-SIDE DEBATING IS NECESSARY TO EXAMINE DIVERSE POLITICAL AGENDAS
AND POLITICS. THE SOLUTION IS NOT TO SILENCE ALL REPRESENTATION; ITS TO
MASSIVELY PROLIFERATE REPRESENTATIONS AND LET THE DEBATE EXAMINE THE
WORTHINESS OF INDIVIDUAL REPRESENTATIONS WHICH CAN SUBVERT THE
SYSTEM. EVERY TIME ANOTHER IMAGE IS REPRESENTED, IT MAKES OVERALL
MARGINALIZATION LESS EASY.

Ann Marie Baldonado, Fall 1996 http://www.emory.edu/ENGLISH/Bahri/Representation.html, accessed 3/23/01

This questioning is particularly important when the representation of the subaltern is involved. The problem does not rest solely with the fact that often marginalized groups do
not hold the 'power over representation' (Shohat 170); it rests also in the fact that representations of these groups are both flawed and few in numbers. Shohat asserts that
dominant groups need not preoccupy themselves too much with being adequately represented. There are so many different representations of dominant groups that negative
images are seen as only part of the "natural diversity" of people. However, "representation of an underrepresented group is necessarily within the hermeneutics of domination,

since
overcharged with allegorical significance." (170) The mass media tends to take representations of the subaltern as allegorical, meaning that

representations of the marginalized are few, the few available are thought to be representative of all
marginalized peoples. The few images are thought to be typical, sometimes not only of members of a particular minority group, but of all minorities in general. It
is assumed that subalterns can stand in for other subalterns. A prime example of this is the fact that actors of particular ethnic
backgrounds were often casted as any ethnic "other". (Some examples include Carmen Miranda HYPERLINK "http://www.emory.edu/ENGLISH/Bahri/carmen.gif" in The
Gang's All Here (1943), Ricardo Mantalban in Sayonara (1957), and Rudolph Valentino in The Son of the Sheik ). This collapsing of the image of the subaltern reflects not only
ignorance but a lack of respect for the diversity within marginalized communities. Shohat also suggests that representations in one sphere--the sphere of popular culture--
effects the other spheres of representation, particularly the political one: The denial of aesthetic representation to the subaltern has historically formed a corollary to the literal
denial of economic, legal, and political representation. The struggle to 'speak for oneself' cannot be separated from a history of being spoken for, from the struggle to speak and
be heard. (173) It cannot be ignored that representations effect the ways in which actual individuals are perceived. Although many see representations as harmless likenesses,
they do have a real effect on the world. They are meant to relay a message and as the definition shows, 'influence opinion and action'. We must ask what ideological work these

Both the scarcity


representations accomplish. Representations or the 'images or ideas formed in the mind' have vast implications for real people in real contexts.

and the importance of minority representations yield what many have called " the burden of
representation". Since there are so few images, negative ones can have devastating affects on the real
lives of marginalized people. We must also ask, if there are so few, who will produce them? Who will be the
supposed voice of the subaltern? Given the allegorical character of these representations, even subaltern writers, artists, and scholars are asking who can really
speak for whom? When a spokesperson or a certain image is read as metonymic, representation becomes more difficult and dangerous. Solutions for this conundrum are
difficult to theorize. We can call for increased "self representation" or the inclusion of more individuals from 'marginalized' groups in 'the act of representing', yet this is easier
said then done. Also, the inclusion of more minorities in representation will not necessarily alter the structural or institutional barriers that prevent equal participation for all in
representation. Focusing on whether or not images are negative or positive, leaves in tact a reliance on the "realness' of images, a "realness" that is false to begin with. Finally,
I again turn to Spivak and her question, 'Can the Subaltern Speak'. In this seminal essay, Spivak emphasizes the fact that representation is a sort of speech act, with a speaker
and a listener. Often, the subaltern makes an attempt at self-representation, perhaps a representation that falls outside the 'the lines laid down by the official institutional
structures of representation' (306). Yet, this act of representation is not heard. It is not recognized by the listener, perhaps because it does not fit in with what is expected of the
representation. Therefore, representation by subaltern individuals seems nearly impossible. Despite the fact that Spivak's formulation is quite accurate, there must still be an
effort to try and challenge status quo representation and the ideological work it does. The work of various 'Third world' and minority writers, artists, and filmmakers attest to the
possibilities of counter-hegemonic, anti-colonial subversion. It is obvious that representations are much more than plain 'likenesses'. They are in a sense ideological tools that
can serve to reinforce systems of inequality and subordination; they can help sustain colonialist or neocolonialist projects. A great amount of effort is needed to dislodge

this force is not


dominant modes of representation. Efforts will continue to be made to challenge the hegemonic force of representation, and of course,

completely pervasive, and subversions are often possible. 'Self representation' may not be a complete
possibility, yet is still an important goal.

Page 65 of 1481
Debate Solves Authoritarianism

DEBATE INVERTS DOCILITY AND AUTHORITARIANISM

N. Kirk Evans, two time NDT first-round and graduate student at U Chicago, [eDebate] We Other
Debaters, Feb 27, 2002, http://www.ndtceda.com/archives/200202/0747.html, accessed February 27,
2002

Although critics of debate (e.g., Kevin Sanchez) appropriate Foucauldian language such as describing
debate as ?the pedagogy devoted to scholarship and training in good conduct,? I can?t help but wonder if
there is a little ?repressive hypothesis? discourse going on here. ?For a long time, the story goes, we
supported a repressive/calculating/veritas-seeking/flogocentric/docile body producing regime, and we
continue to be dominated by it even today. The image of the stratego-spewtron is emblazoned on our
restrained, (un)mute, and hypocrtical debating.? I don?t like certain aspects of debate as it is currently
practiced. Some of my objections are political (e.g., under-representation of minorities, propensity of elite
schools to dominate). Some are aesthetic (e.g., lack of clarity among most debaters). My problem with
criticisms such as Kevin S?s or William S?s or Jack S?s is that they lump something together called
?debate? and criticize it from afar (if that isn?t rendering something standing reserve and then surveying
it with an enlightened imperial gaze, I don?t know what is). Somehow the sentiment seems to be lurking
about that we?d all be free, uninhibited, and unrepressed beings if the debate-machine hadn?t turned us
into assembly-line products of technostrategic thinking. Ummm? repressive hypothesis. The reality is that
proto-debaters enter high school with 8-9 years of educational training to be docile subjects and liberal
humanists. If debate still maintains vestiges of these systems of thought, I think it has more to do with
what people bring to the ?institution? of debate than what debate teaches them. Debaters are taught to
question authorit(ies), and there is certainly a higher degrees of activism (both liberal and conservative)
among debaters than among their non-debate counterparts.

Page 66 of 1481
Roleplaying Good (1/3)

AND, WE MUST POSIT OURSELVES AS THE GOVERNMENT

Rawls, Political Philosopher, 1999 (John, The Law of Peoples, p. 56-7)

How is the ideal of public reason realized by citizens who are not government officials? In a representative government, citizens vote for representativeschief executives,
legislators, and the likenot for particular laws (except at a state or local level where they may vote directly on referenda questions, which are not usually fundamental

, citizens are to think of themselves as if they were legislators and ask


questions). To answer this question, we say that, ideally

themselves what statutes, supported by what reasons satisfying the criterion of reciprocity, they would think it most reasonable to
enact. When firm and widespread, the disposition of citizens to view themselves as ideal legislators, and to repudiate
government officials and candidates for public office who violate public reason, forms part of the political
and social basis of liberal democracy and is vital for its enduring strength and vigor. Thus in domestic society
citizens fulfill their duty of civility and support the idea of public reason, while doing what they can to hold
government officials to it. This duty, like other political rights and duties, is an intrinsically moral duty. I emphasize that it is not a legal duty,
for in that case it would be incompatible with freedom of speech. Similarly, the ideal of the public reason of free and equal peoples is realized, or satisfied, whenever chief
executives and legislators, and other government officials, as well as candidates for public office, act from and follow the principles of the Law of Peoples and explain to other
peoples their reasons for pursuing or revising a peoples foreign policy and affairs of state that involve other societies. As for private citizens, we say, as before, that ideally

citizens are to think of themselves as if they were executives and legislators and ask themselves what
foreign policy supported by what considerations they would think it most reasonable to advance. Once again, when firm and
widespread, the disposition of citizens to view themselves as ideal executives and legislators, and to repudiate government

officials and candidates for public office who violate the public reason of free and equal peoples, is part of the political and social basis of peace

and understanding among peoples

AND, ROLE-PLAYING DEBATES PROMOTE PREPARE US FOR REAL WORLD ACTIVISM


BY GIVING US A BETTER UNDERSTANDING OF HOW POLICY WORKS, MAKING US
AFFECTIVE AGENTS TO ACHIEVE CHANGE. THIS ALLOWS US AS INDIVIDUALS TO
BECOME ACTORS WHO COULD INDEED TRANSFORM INTERNATIONAL POLITICS.

Joyner 1999
[Christopher, Professor international Law @ University of Georgetown, Teaching International Law:
Views from an international relations political scientist].

The debate exercises carry several specific educational objectives. First, students on each team must work
together to refine a cogent argument that compellingly asserts their legal position on a foreign policy
issue confronting the United States. In this way, they gain greater insight into the real-world legal
dilemmas faced by policy makers. Second, as they work with other members of their team, they realize
the complexities of applying and implementing international law, and the difficulty of bridging the gaps
between United States policy and international legal principles, either by reworking the former or
creatively reinterpreting the latter. Finally, research for the debates forces students to become
familiarized with contemporary issues on the United States foreign policy agenda and the role that
international law plays in formulating and executing these policies. 8 The debate thus becomes an
excellent vehicle for pushing students beyond stale arguments over principles into the real world of policy
analysis, political critique, and legal defense.

Page 67 of 1481
Page 68 of 1481
Roleplaying Good (2/3)
ROLEPLAYING IS KEY TO SOCIAL JUSTICE LEARNING WHAT THE STATE SHOULD
DO ALLOWS US TO ACHIEVE THE ALTERNATIVES GOALS

Richard Rorty, philosopher, ACHIEVING OUR COUNTRY: LEFTIST THOUGHT IN TWENTIETH-


CENTURY AMERICA, 1998, p. 98-99

The cultural Left often seems convinced that the nation-state is obsolete, and that there is therefore no
point in attempting to revive national politics. The trouble with this claim is that the government of our
nation-state will be, for the foreseeable future, the only agent capable of making any real difference in
the amount of selfishness and sadism inflicted on Americans. It is no comfort to those in danger of being
immiserated by globalization to be told that, since national governments are now irrelevant, we must
think up a replacement for such governments. The cosmopolitan super-rich do not think any
replacements are needed, and they are likely to prevail. Bill Readings was right to say that the nation-
state [has ceased] to be the elemental unit of capitalism, but it remains the entity which makes decisions
about social benefits, and thus about social justice. The current leftist habit of taking the long view and
looking beyond nationhood to a global polity is as useless as was faith in Marxs philosophy of history, for
which it has become a substitute. Both are equally irrelevant to the question of how to prevent the
reemergence of hereditary castes, or of how to prevent right-wing populists from taking advantage of
resentment at that reemergence. When we think about these latter questions, we begin to realize that
one of the essential transformations which the cultural Left will have to undergo is the shedding of its
semi- conscious anti-Americanism, which it carried over from the rage of the late Sixties. This Left will
have to stop thinking up ever more abstract and abusive names for "the system" and start trying to
construct inspiring images of the country. Only by doing so can it begin to form alliances with people
outside the academyand, specifically, with the labor unions. Outside the academy, Americans still want
to feel patriotic. They still want to feel part of a nation which can take control of its destiny and make
itself a better place. If the Left forms no such alliances, it will never have any effect on the laws of the
United States. To form them will require the cultural Left to forget about Baudrillard's account of America
as Disneylandas a country of simulacraand to start proposing changes in the laws of a real country,
inhabited by real people who are enduring unnecessary suffering, much of which can be cured by
governmental action. Nothing would do more to resurrect the American Left than agreement on a
concrete political platform, a People's Charter, a list of specific reforms. The existence of such a list
endlessly reprinted and debated, equally familiar to professors and production workers, imprinted on the
memory both of professional people and of those who clean the professionals' toiletsmight revitalize
leftist politics.

Page 69 of 1481
Page 70 of 1481
Roleplaying Good (3/3)
ROLE PLAYING IN DEBATE IS ESSENTIAL TO BREAK DOWN ASSUMPTIONS, DEVELOP
CRITICAL THINKING SKILLS, AND DECONSTRUCT THE STATE

Joyner, Professor International Law @ Georgetwon, 99 (Christopher TEACHING INTERNATIONAL


LAW: VIEWS FROM AN INTERNATIONAL RELATIONS POLITICAL SCIENTIST ILSA Journal of International &
Comparative Law, Spring, 5 ILSA J Int'l & Comp L 377)

Use of the debate can be an effective pedagogical tool for education in the social sciences. Debates,
like other role-playing simulations, help students understand different perspectives on a policy issue by
adopting a perspective as their own. But, unlike other simulation games, debates do not require that a
student participate directly in order to realize the benefit of the game. Instead of developing policy
alternatives and experiencing the consequences of different choices in a traditional role-playing game,
debates present the alternatives and consequences in a formal, rhetorical fashion before a judgmental
audience. Having the class audience serve as jury helps each student develop a well-thought-out opinion
on the issue by providing contrasting facts and views and enabling audience members to pose challenges
to each debating team.
These debates ask undergraduate students to examine the international legal implications of various
United States foreign policy actions. Their chief tasks are to assess the aims of the policy in question,
determine their relevance to United States national interests, ascertain what legal principles are involved,
and conclude how the United States policy in question squares with relevant principles of international
law. Debate questions are formulated as resolutions, along the lines of: "Resolved: The United States
should deny most-favored-nation status to China on human rights grounds;" or "Resolved: The United
States should resort to military force to ensure inspection of Iraq's possible nuclear, chemical and
biological weapons facilities;" or "Resolved: The United States' invasion of Grenada in 1983 was a lawful
use of force;" or "Resolved: The United States should kill Saddam Hussein." In addressing both sides of
these legal propositions, the student debaters must consult the vast literature of international law,
especially the nearly 100 professional law-school-sponsored international law journals now being
published in the United States. This literature furnishes an incredibly rich body of legal analysis that often
treats topics affecting United States foreign policy, as well as other more esoteric international legal
subjects. Although most of these journals are accessible in good law schools, they are largely unknown to
the political science community specializing in international relations, much less to the average
undergraduate. [*386]
By assessing the role of international law in United States foreign policy- making, students realize that
United States actions do not always measure up to international legal expectations; that at times,
international legal strictures get compromised for the sake of perceived national interests, and that
concepts and principles of international law, like domestic law, can be interpreted and twisted in order
to justify United States policy in various international circumstances. In this way, the debate format
gives students the benefits ascribed to simulations and other action learning techniques, in that it
makes them become actively engaged with their subjects, and not be mere passive consumers. Rather
than spectators, students become legal advocates, observing, reacting to, and structuring political and
legal perceptions to fit the merits of their case.
The debate exercises carry several specific educational objectives. First, students on each team must
work together to refine a cogent argument that compellingly asserts their legal position on a foreign
policy issue confronting the United States. In this way, they gain greater insight into the real-world legal
dilemmas faced by policy makers. Second, as they work with other members of their team, they realize
the complexities of applying and implementing international law, and the difficulty of bridging the gaps
between United States policy and international legal principles, either by reworking the former or
creatively reinterpreting the latter. Finally, research for the debates forces students to become

Page 71 of 1481
familiarized with contemporary issues on the United States foreign policy agenda and the role that
international law plays in formulating and executing these policies. 8 The debate thus becomes an
excellent vehicle for pushing students beyond stale arguments over principles into the real world of
policy analysis, political critique, and legal defense.

Page 72 of 1481
Page 73 of 1481
Traditional Debate Good (1/2)
SWITCH-SIDE PLAN-FOCUSED DEBATE ENSURES EVERY COMPETITOR MUST
EVALUATE BOTH SIDES OF POTENTIAL POLICIES. THEY ENCOURAGE DEBATES
WITHOUT CLASH. THIS UNCRITICAL FORM OF DEBATE ELIMINATES OUR CAPACITY
TO ENGAGE IN SOCRATIC QUESTIONING, THE ONLY FIREWALL AGAINST GENOCIDE

Dana R. Villa, Laurance S. Rockefeller Fellow at the University Center for Human Values, Princeton
University, Political Theory, April 1998 v26 n2 p147(26)

Arendt sees the categorical imperative as an absolute in the Platonic/authoritarian sense, standing above men and the realm of human affairs, measuring them without any concern for
context, specificity, or the "fundamental relativity" of the "interhuman realm."(30) Arendt emphasizes this inheritance of Platonism because she sees it as inculcating a habit of mechanical,

The more judgment is identified with the application of a rule or an unvarying standard, the
unthinking judgment.

more our powers of judgment atrophy, and the less we are able to "stop and think" in the Socratic sense.
Moreover, the insistence that judgment is dependent on such standards leads to a "crisis in judgment" when these standards are revealed to be without effective power. This, according to
Arendt, is what happens in the course of the modern age, as new and unprecedented moral and political phenomena reveal the hollowness and inadequacy of the "reliable universal rules"

. This process--call it the crisis in authority or, to use Nietzsche's symbolic formulation, the "death of God"--comes to its conclusion
the tradition had offered

with the advent of the evils of totalitarianism, evils so unprecedented that they "have clearly exploded our
categories of political thought and our standards for moral judgment."(31) The failure of the inherited wisdom of the past, the fact of
a radical break in our tradition, throws us back upon our own resources. Potentially, Arendt notes, the crisis is liberating, as it frees the faculty of judgment from its subservience to objectivist
regimes such as Plato's ideas or Kant's categorical imperative. As Arendt puts it in "Understanding and Politics": Even though we have lost yardsticks by which to measure, and rules under
which to subsume the particular, a being whose essence is beginning may have enough of origin within himself to understand without preconceived categories and to judge without the set of
customary rules which is morality.(32) The hope that the "crisis in authority" will lead to the rebirth of a genuinely autonomous faculty of judgment runs up against Arendt's own deeply

Minus the presence of Socrates


ingrained sense that ordinary individuals will find it difficult indeed to wean themselves from pregiven categories and rules.

(who, like an electric ray, paralyzes his partners in dialogue, forcing them to stop and think), the likely
result of such a crisis is thankfulness for anything that props up the old set of standards or provides the semblance of a
new one. Responding to Hans Jonas's call for a renewed inquiry into ultimate, metaphysical grounds for judgment at a conference on her work in 1972, Arendt declared her pessimism that "a
new god will appear," and went on to observe: If you go through such a situation [as totalitarianism] the first thing you know is the following: you never know how somebody will act. You
have the surprise of your life! This goes throughout all layers of society, and it goes throughout various distinctions between men. And if you want to make a generalization, then you could say
that those who were still very firmly convinced of the so-called old values were the first to be ready to change their old values for a new set of values, provided they were given one. And I am
afraid of this, because I think that the moment you give anybody a new set of values--or this famous "bannister"--you can immediately exchange it. And the only thing the guy gets used to is
having a "bannister" and a set of values, no matter.(33) Arendt thought that the natural tendency of the ordinary person, when faced with the destruction of one set of authoritative rules,

Socratic
would not be Socratic examination and perplexity (which only further dissolves the customary), but rather a grasping for a new code, a new "bannister." Thinking, especially

thinking, dissolves grounds, it does not stabilize them. It is, as Arendt says, a "dangerous and resultless enterprise," one that can just as easily lead
to cynicism and nihilism as to independent judgment and a deepened moral integrity.(34) Arendt agrees with the analysis Kant gives in "What Is Enlightenment?": most people would simply
prefer not to make the effort that independent judgment demands, let alone risk the taken-for-granted moral presuppositions of their existence. Yet however real this aversion to thinking or

"paralysis" is, Arendt holds onto the Socratic possibility that ordinary individuals will remain open to the "winds
of thought." She profoundly agrees with Socrates that it is only through such examination that the individual is likely to
avoid complicity with the moral horrors perpetrated by popular political regimes. Socratic thinking--which, in
its relentless negativity, is the very opposite of all foundational or professional philosophical thinking --liberates the faculty of judgment from the

tyranny of rules and custom. In this way, it prevents the individual from being "swept away unthinkingly by what everybody else does and believes in."(35) Independent
judgment is, according to Arendt, the "by-product" of this liberating effect of thinking; it "realizes" thinking "in the world of appearances."(36 ) Thinking may not be able

to "make friends" of citizens as Socrates had hoped, but it can "prevent catastrophes, at least for myself, in the rare moments when the chips are
down."(37)

Page 74 of 1481
Page 75 of 1481
Traditional Debate Good (2/2)

TRADITIONAL DEBATE IS RE-PRESENTATION. WE DONT CLAIM TO SPEAK A HIGHER


TRUTH OR KNOW WHAT IS BEST FOR OTHERS. WE DEBATE THOSE ISSUES
CONTINGENTLY. THE ONLY PEOPLE WHO SAY THAT DEBATE SHOULD BE PURELY
REPRESENTATIVE ARE THE ACTIVISM/CRITIQUE CROWD.

Ann Marie Baldonado, Fall 1996 http://www.emory.edu/ENGLISH/Bahri/Representation.html, accessed 3/23/01

1. Presence, bearing, air; Appearance; impression on the sight. 2. An Image, likeness, or reproduction in
some manner of a thing; A material image or figure; a reproduction in some material or tangible form; in
later use, a drawing or painting. (of a person or thing); The action or fact of exhibiting in some visible
image or form; The fact of expressing or denoting by means of a figure or symbol; symbolic action or
exhibition. 3. The exhibition of character and action upon the stage; the performance of a play; Acting,
simulation, pretense. 4. The action of placing a fact, etc., before another or others by means of discourse;
a statement or account, esp. one intended to convey a particular view or impression of a matter in order
to influence opinion or action. 5. A formal and serious statement of facts, reasons, or arguments, made
with a view to effecting some change, preventing some action, etc.; hence, a remonstrance, protest,
expostulation. 6. The action of presenting to the mind or imagination; an image thus presented; a clearly
conceived idea or concept; The operation of the mind in forming a clear image or concept; the faculty of
doing this. 7. The fact of standing for, or in place of, some other thing or person, esp. with a right or
authority to act on their account; substitution of one thing or person for another. 8. The fact of
representing or being represented in a legislative or deliberative assembly, spec. in Parliament; the
position, principle, or system implied by this; The aggregate of those who thus represent the elective
body.
from The Oxford English Dictionary

Representation is presently a much debated topic not only in postcolonial studies and academia, but in
the larger cultural milieu. As the above dictionary entry shows, the actual definitions for the word alone
are cause for some confusion. The Oxford English Dictionary defines representation primarily as
"presence" or "appearance." There is an implied visual component to these primary definitions.
Representations can be clear images, material reproductions, performances and simulations.
Representation can also be defined as the act of placing or stating facts in order to influence or affect the
action of others. Of course, the word also has political connotations. Politicians are thought to 'represent'
a constituency. They are thought to have the right to stand in the place of another. So above all, the term
representation has a semiotic meaning, in that something is 'standing for' something else. These various yet
related definitions are all implicated in the public debates about representation. Theorists interested in Postcolonial
studies, by closely examining various forms of representations, visual, textual and otherwise, have teased out the
different ways that these "images" are implicated in power inequalities and the subordination of the 'subaltern'.

Representations-- these 'likenesses'--come in various forms: films, television, photographs, paintings, advertisements
and other forms of popular culture. Written materials--academic texts, novels and other literature, journalistic pieces--
are also important forms of representation. These representations, to different degrees, are thought to be somewhat
realistic, or to go back to the definitions, they are thought be 'clear' or state 'a fact'. Yet how can simulations or
"impressions on the sight" be completely true? Edward Said, in his analysis of textual representations of the Orient in
Orientalism, emphasizes the fact that representations can never be exactly realistic:

In any instance of at least written language, there is no such thing as a delivered presence, but a re-presence, or a
representation. The value, efficacy, strength, apparent veracity of a written statement about the Orient therefore relies
very little, and cannot instrumentally depend, on the Orient as such. On the contrary, the written statement is a

Page 76 of 1481
presence to the reader by virtue of its having excluded, displaced, made supererogatory any such real thing as "the
Orient". (21)

Representations, then can never really be 'natural' depictions of the orient. Instead, they are constructed images,
images that need to be interrogated for their ideological content.

In a similar way, Gayatri Chakravorty Spivak makes a distinction between Vertretung and Darstellung. The
former she defines as "stepping in someone's place. . .to tread in someone's shoes." Representation in
this sense is "political representation," or a speaking for the needs and desires of somebody or something.
Darstellung is representation as re-presentation, "placing there." Representing is thus "proxy and
portrait," according to Spivak. The complicity between "speaking for" and "portraying" must be kept in
mind ("Practical Politics of the Open End," The Post-Colonial Critic: Interviews, Strategies, Dialogues.)

Page 77 of 1481
Traditional Debate Accesses Peformativity

TRADITIONAL DEBATE COOPTS THEIR PERFORMANCE GOOD OFFENSE. IT


INCORPORATES STYLE WITHOUT ELIMINATING SUBSTANCE

Jeff Parcher, February 26, 2001, www.ndtceda.com

BTW - my notions do not eliminate the notion of performance - they merely contextualize them within a
discussion that can be limited and fair. It merely requires the performance be relevant by a reasonable
criteria (ie the resolution). Also, debates have speaker points. It seems fairly obvious to me that the
debate ballot is a clear dichotomy. One affirms or negates the resolution/plan and then gives speaker
points to reward or punish performance. Obviously, I realize that performance impacts truth. But that's
only a reason why a focus on the resolutional question coopts the performative criteria. Of course a good
performance gets rewared in both points and in the decision itself. That's why we don't need to make it
JUST about performance. We already take the perfromance into account inevitably. Mixing it further
simply makes us drift aimlessly.

PLAN FOCUSED DEBATES ALWAYS PROVIDE A CLEARER, FAIRER, AND MORE


EDUCATION FRAMEWORK

Jeff Parcher, February 26, 2001, www.ndtceda.com

This is absolutely devastating to the performance arguments. And even if we could hodge-podge together
some inevtiably subjective criteria in each individual debate, they simply could never match the benefits
of debate provided by a clear plan/resolution focus. Performance debates would be incredibly repetitive
in that they would always be 90% about methodolgy rather than the substance of performances. Because
the limits to possible performances are so large - both sides would always have an incentive to focus on
methodology rather than substance. The affirmative will be on an endless search to coopt the negative
performance (in the words of the Fort, "We are in solidarity with these words"). The negative on an
endless search to exclude the affirmative performance through topicality or general kritiks. Rarely do I
think we would ever have debates which engaged the two performances. The current puryeyors of this
type of debate have certainly relied much more on competitiveness arguments than on actual substantive
engagement (as far as I've seen anyway).

Page 78 of 1481
Competition Good

COMPETITION IS IS NECESSARY FOR SOCIAL ORGANIZATION

Gary Olson and Jean- Franois Lyotard, Resisting a Discourse of Mastery: A


Conversation with Jean-Franois Lyotard, JAC 15.3, 1995,
http://jac.gsu.edu/jac/15.3/Articles/1.htm, accessed 1/21/02

Second, competition is not competition between different groups in a cultural reality. Not at all. The
notion of competition as a male model is a notion I reject, maybe because I am a male, but, in fact,
because there is not any other way to understand the domination of the competitive pattern in our
society. I mean, this system has competed against all other systems, all the other ways of organizing
human communities. And we can consider human history not as a linear succession with a sort of
causality between each segment of this line, but as the opposite, as the contingent and different ways in
which human communities have tried to organizeexactly in the same terms that so-called life has
fortuitously produced different forms of living beings. And between these different entitiesanimals,
vegetables, human beings, or human communitiescompetition was necessarily open. They are all open
systems; they need to grasp energy from outside in order to maintain themselves, and if they have to
grasp energy from outside, they are competitive with other systems. Thats true for animals, even
vegetables, and for human communities. And thats how our system, now, won against other ways that
communities have tried to organize themselves, and it has internalized competition itself in order to
continue to be able to grasp outside and inside energies as much as possible. Its not a male idea; there is
no argument against it. There is no doubt: its not a male idea. And Im sure women are perfectly able to
understand this, even if they hate it; so do I. But we are in this condition.

Page 79 of 1481
**Permutations**
Juxtaposition Perm: 2AC

PERM DO BOTH, CRITICISM WITHOUT OPPOSITION CAUSES COOPERTATION,


ONLY JUXTAPOSITION ALLOWS CONSTANT CRITICISM

Edelman 87
[Prof. Pol Sci @ Wisconsin, September, U. of Minn, Constructing the Political Spectacle]

Opposition in expressed opinion accordingly make for social stability: they are almost
synonymous with it, for they reaffirm and reify what everyone already knows and accepts. To
express a prochoice or an anti-abortion position is to affirm that the opposite position is being
expressed as well and to accept the opposition as a continuing feature of public discourse. The
well established, thoroughly anticipated and therefore ritualistic reaffirmation of the differences
institutionalizes mboth rhetorics minimizing the chance of major shifts and leaving the regime
wide discretion; for there will be anticipated support and opposition no matter what forms of
action or inaction occur. As long as there is substantial expression of opinion on both sides of an
issue, social stability persists and so does regime discretion regardless of the exact numbers or
of marginal shifts in members. The persistence of unresolved problems with conflicting meaning
is vital. It is not the expression of opposition but of consensus that makes for instability. Wher
statements need not be defended against counterstatements they are readily changed or
inverted. Consensual agreements about the foreign enemy of ally yield readily to acceptance of
the erstwhile enemy as ally and the former ally as enemy, but opinions about abortion are likely
to persist. Rebellion and revolution do not ferment in societies in which there has been a long
history of the ritualized exchange of opposing views of issues accepted as important, but rather
where such exchanges have been lacking, so that a consensus on common action to oust the
regime is easily built.

Page 80 of 1481
Page 81 of 1481
Juxtaposition Perm: 1AR
EXTEND THE 2AC EDELMAN EVIDENCE. PURE CRITIQUE FAILS BECAUSE IT FLIPS THE
BINARISM, NOT ENGAGING THE DISCOURSE IT CRITICIZED, CREATING A NEW
MONOLITHIC HEGEMONY. ONLY THE PERM THAT COMBINES THE 1AC AND THE
CRITICISM CREATES CONSTANT CRITICISM, USING THE AFF AS A TARGET, SOLVING
BETTER THAN THE ALTERNATIVE

ALSO, ALL OF THEIR PERM THEORY AND LINK ARGUMENTS DONT APPLY BECAUSE
THE PERM COMBINES THE WHOLE 1AC AND THE CRITICISM, USING THAT
CONTRADICTION TO CONSIDER BOTH SIDES, IMPACT TURNING THEIR ARGUMENT

ALSO, COMBINING THE AFF AND THE K SOLVES BETTER

Said 94
[Edward W., Representations of the Intellectual: The 1993 Reich Lectures, Vintage, 1994, 60]

Because the exile sees things both in terms of what has been left behind and what is actual hear
and now, there is a double perspective that never sees things in isoaltion. Every scene or
situation in the new country necessarily draws on its counterpart in the old country.
Intellectually, this means that an idea or expreience is always counterposed with another,
therefore, making them both appear in a sometimes new and unpredictable light: from that
justaposition, one gets a better, perhaps more universal idea of how to think say, about a
human rights issue in one situation by comparison with another. I have felt that most of the
alarmist and deeply flawed discussions of Islamic fundamentalism in the West have been
intellectually invidious precisely because they have not been compared with Jewish or Christian
fundamentalism, both equally prevalent and reprehensible in my own experience of the Middle
East. What is usually thought of as a simple issue of judgment against an approved enemy, in
double or exile perspective impels a Western intellectual to see a much wider picture,
with the requirement now of taking a position as a secularist (or not) on all theocratic
tendencies, not just against the conventionally designated ones.

ALSO, PURE CRITICISM FAILS, ONLY COMBINATION OF CONTRADICTORY IDEAS


SOLVES

Walt 98
[Stephen M., Prof. Pol. Sci, U. of Chicago, International Relations: one world, many theories, Foreign
Policy, March 22, LN]

Page 82 of 1481
No single approach can capture all the complexity of contemporary world politics.
Therefore, we are better off with a diverse array of competing ideas rather than a single
theoretical orthodoxy. Competition between theories helps reveal their strengths and
weaknesses and spurs subsequent refinements, while revealing flaws in conventional
wisdom. Although we should take care to emphasize inventiveness over invective, we should
welcome and encourage the heterogeneity of contemporary scholarship.

Page 83 of 1481
Page 84 of 1481
Juxtapositon Perm: 2AR
THE EDELMAN PERMUTATION IS THE ONLY ADVOCACY WHICH PROVIDES FOR
CONSTANT CRITICISM. JUXTAPOSITION TAKES THE WHOLE AFFIRMATIVE SPEECH
ACT AND THE WHOLE NEGATIVE CRITICISM AND ALLOWS YOU TO VOTE FOR THE
PROCESS OF CONSTANT CRITICISM. IT USES THE PLAN TO UPHOLD THE SYSTEM AS
A TARGET FOR THE NEG CRITICISM. WITHOUT THAT, THE CRITICISM BECOMES
INVERTED, EMBODYING ITS OWN OPPOSITE.

ALSO, NONE OF THEIR SPECIFIC EVIDENCE APPLIES. ITS AN IN-ROUND


PERMUTATION ABOUT OUR SPEECDH ACTS AND THE BEST WAY TO MAINTAIN THE
INTEGRITY OF CRITICISM

Page 85 of 1481
Page 86 of 1481
Juxtaposition Perm: Ext
AND, JUXTAPOSING THE AFF AND THE ALTERNATIVE CREATES EFFECTIVE,
CONSTANT CRITICISM, OVERCOMING THE HEGEMONY OF CRITIQUE

Connolly 2002
[William E., Prof. of Pol. Sci. @ John Hopkins U., Identity/Difference: Democratic Negotiations of Political
Paradox, Minneapolis: University of Minnesota Press, September 2002, 180-1]

Another way to pose the paradox is this: The human animal is essentially incomplete without
social form and a common language, institutional setting, set of political traditions, and political
forum for inunciating public purposes are indispensible to the acquisition of an identity and the
commonalities essential to life. But every form of social completion and enablement also
contains subjugations and cruelties within it. Politics, then, is the medium through which these
ambiguities can be engaged and confronted, shifted and stretched. It is simultaneously a
medum through which common purposes are crystalized and the consummage means by which
their transcription into musical harmonies is exposed, contested, disturbed, and unsettled. A
society that enables politics as this ambiguous medium is a good society because it enables the
paradox of difference to find expression in public life

AND, JUXTAPOSITION OF INCOMPATIBLE IDEAS AVOIDS THE PROBLEMS OF


TRADITIONAL THEORY AND ENABLES A PROCESS OF CONSTANT CRITICISM

Marcus '98
[George E., Professor of Anthro at Rice University, Ethnography through Thick and
Thin, Princeton: Princeton University Press, 1998, 186-7//uwyo-ajl]

The postmodern notions of heterotopia (Foucault), juxtapositions, and the blocking together
of incommensurables (Lyotard) have served to renew the long-neglected practice of
comparison in anthropology, but in altered ways. Juxtapositions do not have the obvious
meta-logic of older styles of comparison in anthropology (e.g., controlled comparisons
within a cultural area or "natural" geographical region); rather, they emerge from putting
questions to an emergent object of study whose controus are not known beforehand, but
are themselves a contribution of making an account which has different, complexly
connected real-world sites of investigation. The postmodern object of study is ultimately
mobile and multiply situated, so any ethnography of such an object will have a
comparative dimension that is integral to it, in the form of juxtapositions of seeming
incommensurables or phenomena that might conventionally have appeared to be "world
apart." Comparison reenters the very act of ethnographic specificity by a postmodern
vision of seemingly improbably juxtapositions, the global collapsed into and made and
integral part of a parallel, related local situations rather than something monolithic and
external to them. This move toward comparison as heterotopia firmly deterritorializes culture

Page 87 of 1481
in ethnographic writing and simulates accounts of cultures composed in a landscape for which
there is as yet no developed theoretical comparison

Page 88 of 1481
Campbell Perm: 2AC

PERM DO THE PLAN WHILE ENDORSING THE CRITICISM EXIGENCIES DEMAND


ACTION EVEN IN THE FACE OF CRITICISM

Campbell 98
[David, Intl Relations Prof @ UM, National Deconstruction: Violence, Identity, and Justice in Bosnia,
Minneapolis: University of Minnesota Press, 1998, 186]

The undecidable within the decision does not, however, prevent the decision nor avoid its urgency. As
Derrida observes, a just decision is always required immediately, right away. This necessary haste has
unavoidable consequences because the pursuit of infinite information and the unlimited knowledge of
conditions, rules or hypothetical imperatives that could justify it are unavailable in the crush of time. Nor
can the crush of time be avoided, even by unlimited time, because the moment of decision as such
always remains a finite moment of urgency and precipitation. The decision is always structurally finite,
it aalways marks the interruption of the juridico- or ethico- or politico-cognitive deliberation that
precedes it, that must precede it. That is why, invoking Kierkegaard, Derrida, declares that the instant of
decision is a madness.

The finite nature of the decision may be a madness in the way it renders possible the impossible, the
infinite character of justice, but Derrida argues for the necessity of this madness. Most importantly,
Derrida argues for the necessity of this madness. Most importantly, although Derridas argument
concerning the decision has, to this pint, been concerned with an account of the procedure by which a
decision is possible, it is with respect to the ncessity of the decision that Derrida begins to formulate an
account of the decision that bears upon the content of the decision. In so doing, Derridas argument
addresses more directly more directly, I would argue than is acknowledged by Critchley the concern
that for politics (at least for a progressive politics) one must provide an account of the decision to combat
domination.

That undecidability resides within the decision, Derrida argues, that justice exceeds law and calculation,
that the unpresentable exceeds the determinalbe cannot and should not serve as alibi for staying out of
juridico-political battles, within an institution or a state, or between institutions or states and others.
Indeed, incalculable justice requires us to calculate. From where do these insistences come? What is
behind, what is animating, these imperatives? It is both the character of infinite justice as a heteronomic
relationship to the other, a relationship that because of its undecidability multiplies responsibility, and the
fact that left to itself, the incalculable and given (donatrice) idea of justice is always very close to
the bad, even to the worst, for it can always be reappropriated by the most perverse calculation.
The necessity of calculating the incalculable thus responds to a duty a duty that inhabits the
instant of madness and compels the decision to avoid the bad, the perverse calculation, even
the worst. This is the duty that also dwells with deconstructive thought and makes it the starting
point, the at least necessary condition, for the organization of resistance to totalitarianism in all
its forms. And it is a duty that responds to practical political concerns when we recognize that
Derrida names the bad, the perverse, and the worst as those violences we recognize all too well
without yet having thought them through, the crimes of xenophobia, racism, anti-Semitism,
religious or nationalist fanaticism.

Page 89 of 1481
Campbell Perm: 1AR

EXTEND THE 2AC CAMPBELL 98 EVIDENCE. WHEN FACED WITH UNDECIDABLE


SITUATIONS AND THE STAKES ARE AS HIGH AS THE 1AC, YOU HAVE TO ACT IN THE
FACE OF CRITICISM OR RISK POLITICAL PARALYSIS BECAUSE EVERY ACTION SEEMS
DOOMED, ALLOWING OPPRESSION AND VIOLENCE TO REIGN UNCHECKED

Page 90 of 1481
Strategic Essentialism Perm: 2AC

PERMUTATION: THE PLAN IS A STRATEGIC ESSENTIALISM THAT CREATES SPACE


FOR ACTIVIST POLITICS
(THE CRITIQUE IS A FALSE CHOICE THAT IMPEDES ACTIVISM.)

Sankaran Krishna, Professor, Political Science, University of Hawaii, Alternatives v. 18, 1993, p. 400-401.

The dichotomous choice presented in this excerpt is straightforward: one either indulges in total critique,
delegitimizing all sovereign truths, or one is committed to nostalgic, essentialist unities that have
become obsolete and have been the grounds for all our oppressions. In offering this dichotomous choice,
Der Derian replicates a move made by Chaloupka in his equally dismissive critique of the move
mainstream nuclear opposition, the Nuclear Freeze movement of the early 1980s, that, according to him,
was operating along obsolete lines, emphasizing facts and realities, while a postmodern President
Reagan easily outflanked them through an illusory Star Wars program (See KN: chapter 4) Chaloupka
centers this difference between his own supposedly total critique of all sovereign truths (which he
describes as nuclear criticism in an echo of literary criticism) and the more partial (and issue based)
criticism of what he calls nuclear opposition or antinuclearists at the very outset of his book. (Kn: xvi)
Once again, the unhappy choice forced upon the reader is to join Chaloupka in his total critique of all
sovereign truths or be trapped in obsolete essentialisms. This leads to a disastrous politics, pitting groups
that have the most in common (and need to unite on some basis to be effective) against each other. Both
Chaloupka and Der Derian thus reserve their most trenchant critique for political groups that should, in
any analysis, be regarded as the closest to them in terms of an oppositional politics and their desired
futures. Instead of finding ways to live with these differences and to (if fleetingly) coalesce against the
New Right, this fratricidal critique is politically suicidal. It obliterates the space for a political activism
based on provisional and contingent coalitions, for uniting behind a common cause even as one
recognizes that the coalition is comprised of groups that have very differing (and possibly unresolvable)
views of reality. Moreover, it fails to consider the possibility that there may have been other, more
compelling reasons for the failure of the Nuclear Freeze movement or anti-Gulf War movement. Like
many a worthwhile cause in our times, they failed to garner sufficient support to influence state policy.
The response to that need not be a totalizing critique that delegitimizes all narratives. The blackmail
inherent in the choice offered by Der Derian and Chaloupka, between total critique and ineffective
partial critique, ought to be transparent. Among other things, it effectively militates against the
construction of provisional or strategic essentialisms in our attempts to create space for activist politics. In
the next section, I focus more widely on the genre of critical international theory and its impact on such
an activist politics.

Page 91 of 1481
Strategic Essentialism Perm: 1AR

NEXT, EXTEND THE KRISHNA PERM. THE NEGS WITH US OR AGAINST US


MENTALITY FRACTURES EFFECTIVE SOCIAL ACTION INSTEAD OF FOCUSING ON
DIFFERENCES. WE SHOULD HIGHLIGHT OUR AGREEMENTS. THIS HAS 2
IMPLICATIONS:

IT FLIPS THE K SOLVENCY BECAUSE IT ENTRENCHING AN ALIENATING PRAXIS


IT PROVES THAT ONLY THE PERM, WHICH RECOGNIZES THE VALUE OF BOTH
ADVOCACIES, CAN LEAD TO EFFECTIVE POLITICAL ACTION DICHOTOMOUS
CHOICE COLLAPSES PRAXIS

Page 92 of 1481
Bleiker Perm: 2AC

VIEWING TWO COMPETING IDEOLOGIES TOGETHER AND CREATING


CONTRADICTIONS ALLOWS THE IDEOLOGIES TO COEXIST, OPENING MORE
AVENUES FOR POLITICAL THOUGHT

Bleiker 97
[Roland, PhD Cand @ Australian National U. of Political Sci, Alternatives 22, 57-85//uwyo]

No concept will ever be sufficient, will ever do justice to the object it is trying to capture. The objective then becomes to conceptualize thoughts so that they do not silence other
voices, but coexist and interact with them. Various authors have suggested methods for this purpose, methods that will always remain attempts without ever reaching the ideal

state that they aspire to. We know of Mikhail Bakhtins dialogism, a theory of knowledge and language that tries to avoid the excluding tendencies of monological thought
forms. Instead, he accepts the existence of multiple meanings, draws connections between differences, and searches
for possibilities to establish conceptual and linguistic dialogues among competing ideas, values, speech forms, texts, and validity claims, and the like. Jurgen
Habermas attempts to theorize the preconditions for ideal speech situations. Communication, in this case, should be as unrestrained as possible, such that claims to truth and
rightness can be discursively redeemed, albeit, one should add, though a rationalism and universalism that it violently anti-Bakhtinian and anti-Adornian. Closer to the familiar

empathetic cooperation, which aims at opening up questions of gender


terrain of IR we find Christine Sylvesters feminist method of

by a process of positional slippage that occurs when one listens seriously to the concerns, fears, and agendas of
those one is unaccustomed to heeding when building social theory. But how does one conceptualize such attempts if concepts can
ever do justice to the objects they are trying to capture?

The daring task is, as we know from Adorno, to open with concepts what does not fit into concepts, to resist the distorting power of reification and return the conceptual to the
nonconceptual. This disenchantment of the concept is the antidote of critical philosophy. It impedes the concept from developing its own dynamics and from becoming an
absolute in itself. The first step toward disenchanting the concept is simply refusing to define it monologically. Concepts should achieve meaning only gradually in relation to
each other. Adorno even intentionally uses the same concept in different way in order to liberate it from the harrow definition that language itself had already imposed on it. That

contradictions could arise out of this practice does not bother Adorno. Indeed, he considers them essential .

One cannot eliminate the contradictory, the fragmentary, and the discontinuous. Contradictions are only
contradictions if one assumes the existence of a prior universal standard of reference. What is different
appears as divergent, dissonant, and negative only as long as our consciousness strives for a totalizing standpoint,
which we must avoid if we are to escape the reifying and excluding dangers of identity thinking. Just as reality is
fragmented, we need to think in fragments. Unity then is not to be found be evening out discontinuities. Contradictions are to be

referred over artificially constructed meanings and the silencing of underlying conflicts. Thus, Adorno advocates
writing in fragments, such that the resulting text appears as if it always could be interrupted, cut off abruptly, any time, and place. He adheres to Nietzsches advice that one
should approach deep problems like taking a cold bath, quickly into them and quickly out again. The belief that one does not reach deep enough this way, he claims, is simply
the superstition of those who fear cold water. But Nietzsches bath has already catapulted us into the vortex of the next linguistic terrain of resistance the question of style.

Page 93 of 1481
Perm Solves: Coalitions Key

THE OPPRESSED SHOULD WELCOME THOSE FROM THE DOMINANT


GROUP COMMITTED TO FIGHTING AGAINST OPPRESSION

Ali Khan, Professor, Law, Washburn University. Lessons From Malcolm X: Freedom by Any Means
Necessary, HOWARD LAW JOURNAL v. 38 1994.

Yet, no concept of freedom requires that every member of the dominant group be dehumanized. Such
dehumanization is unnecessary, even counter-productive, in the fight against oppression. The oppressed
should welcome those among the dominant group who gather the moral courage to rebel against their
own kind and fight for the sake of justice. n60 [*95]

Page 94 of 1481
Perm Solves: Hybridization Effective

THE PERM FUNCTIONS AS A NEGOTIATION BETWEEN THE POLITICS OF THE 1AC


AND THE ALTERNATIVE, ALLOWING FOR MORE EFFECTIVE POLITICAL CHANGE
THAN EITHER THE ONE OR THE OTHER.

Homi K. Bhabha, Professor, University of Sussex, THE LOCATION OF CULTURE, 1994, p. 28.

My illustration attempts to display the importance of the hybrid moment of political change. Here the
transformational value of change lies in the rearticulation, or translation, of elements that are neither the
One (unitary working class) nor the Other (the politics of gender) but something else besides, which
contests the terms, and territories of both. There is a negotiation between gender and class, where each
formation encounters the displaced, differentiated boundaries of its group representation and
enunciative sites in which the limits and limitations of social power are encountered in an agonistic
relation. When it is suggested that the British Labour Party should seek to produce a socialist alliance
among progressive forces that are widely dispersed and distributed across a range of class, culture and
occupational forces - without a unifying sense of the class for itself - the kind of hybridity that I have
attempted to identify is being acknowledged as a historical necessity. We need a little less pietistic
articulation of political principle (around class and nation); a little more of the principle of political
negotiation.

Page 95 of 1481
Perm Solves: Multifaceted Resistance Best

THE PERM SOLVES BEST A MULTIFACETED APPROACH TO


COMBATING OPPRESSION IS KEY

Ali Khan, Professor, Law, Washburn University. Lessons From Malcolm X: Freedom by Any Means
Necessary, HOWARD LAW JOURNAL v. 38 1994.

It must be noted that Malcolm's concept of any means necessary includes, but is not limited to non-
violent civil disobedience. n29 If non-violent civil disobedience does not change the system, then any
means necessary allows the oppressed to consider armed resistance. The oppressed may use multiple
strategies. One group among the oppressed, for example, may use non-violent means to fight oppression;
another may advocate more radical methods to change the system. This multi-faceted approach creates
more pressure on the oppressor to lift oppression. In order for such a movement to be effective,
however, the oppressor must believe that those who are involved are serious about [*87] their cause.
Those who are oppressed must be willing to sacrifice their lives to abolish the state of subjugation. n30 It
is also important that the oppressed maintain their underlying solidarity because it is inevitable that they
will encounter efforts to divide them and turn them against each other.

Page 96 of 1481
Perm Solves: Radicalism Dooms the Movement

THEY WONT ACHIEVE THEIR MINDSET SHIFT, AND EVEN IF THEY DO


CONCRETE POLICIES OPTIONS WILL STILL BE KEY

Martin Lewis professor in the School of the Environment and the Center for International Studies at Duke
University. GREEN DELUSIONS, 1992 p 11-12.

Here I will argue that eco-radical political strategy, if one may call it that, is consummately self-defeating.
The theoretical and empirical rejection of green radicalism is thus bolstered by a series of purely
pragmatic objections. Many eco-radicals hope that a massive ideological campaign can transform popular
perceptions, leading both to a fundamental change in lifestyles and to large-scale social reconstruction.
Such a view is highly credulous. The notion that continued intellectual hectoring will eventually result in a
mass conversion to environmental monasticism (Roszak 1979:289)marked by vows of poverty and
nonprocreationis difficult to accept. While radical views have come to dominate many environmental
circles, their effect on the populace at large has been minimal. Despite the greening of European politics
that recently gave stalwarts considerable hope, the more recent green plunge suggests that even the
European electorate lacks commitment to environmental radicalism. In the United States several decades
of preaching the same ecoradical gospel have had little appreciable effect; the public remains, as before,
wedded to consumer culture and creature comforts. The stubborn hope that nonetheless continues to
inform green extremism stems from a pervasive philosophical error in radical environmentalism. As David
Pepper (1989) shows, most eco-radical thought is mired in idealism: in this case the belief that the roots
of the ecological crisis lie ultimately in ideas about nature and humanity As Dobson (1990:37) puts it:
Central to the theoretical canon of Green politics is the belief that our social, political, and economic
problems are substantially caused by our intellectual relationship with the world (see also Milbrath
1989:338). If only such ideas would change, many aver, all would be well. Such a belief has inspired the
writing of eloquent jeremiads; it is less conducive to designing concrete strategies for effective social and
economic change. It is certainly not my belief that ideas are insignificant or that attempting to change
others opinions is a futile endeavor. If that were true I would hardly feel compelled to write a polemic
work of this kind. But I am also convinced that changing ideas alone is insufficient. Widespread ideological
conversion, even if it were to occur, would hardly be adequate for genuine social transformation. Specific
policies must still be formulated, and specific political plans must be devised if those policies are ever to
be realized.

Page 97 of 1481
Perm Solves: Working within Institutions Key to
Change

ONLY WORKING WITHIN THE INSTITUTIONS OF POWER CAN CREATE CHANGE

Lawrence Grossburg, University of Illinois, WE GOTTA GET OUTTA THIS PLACE, 1992, p. 391-393

The Left needs institutions which can operate within the systems of governance, understanding that such
institutions are the mediating structures by which power is actively realized. It is often by directing
opposition against specific institutions that power can be challenged. The Left has assumed from some
time now that, since it has so little access to the apparatuses of agency, its only alternative is to seek a
public voice in the media through tactical protests. The Left does in fact need more visibility, but it also
needs greater access to the entire range of apparatuses of decision making and power. Otherwise, the
Left has nothing but its own self-righteousness. It is not individuals who have produced starvation and the
other social disgraces of our world, although it is individuals who must take responsibility for eliminating
them. But to do so, they must act within organizations, and within the system of organizations which in
fact have the capacity (as well as the moral responsibility) to fight them. Without such organizations, the
only models of political commitment are self-interest and charity. Charity suggests that we act on behalf
of others who cannot act on their own behalf. But we are all precariously caught in the circuits of global
capitalism, and everyones position is increasingly precarious and uncertain. It will not take much to
change the position of any individual in the United States, as the experience of many of the homeless, the
elderly and the fallen middle class demonstrates. Nor are there any guarantees about the future of any
single nation. We can imagine ourselves involved in a politics where acting for another is always acting for
oneself as well, a politics in which everyone struggles with the resources they have to make their lives
(and the world) better, since the two are so intimately tied together! For example, we need to think of
affirmation action as in everyones best interests, because of the possibilities it opens. We need to think
with what Axelos has described as a planetary thought which would be a coherent thoughtbut not a
rationalizing and rationalist inflection; it would be a fragmentary thought of the open totalityfor what
we can grasp are fragments unveiled on the horizon of the totality. Such a politics will not begin by
distinguishing between the local and the global (and certainly not by valorizing one over the other) for the
ways in which the former are incorporated into the latter preclude the luxury of such choices. Resistance
is always a local struggle, even when (as in parts of the ecology movement) it is imagined to connect into
its global structures of articulation: Think globally, act locally. Opposition is predicated precisely on
locating the points of articulation between them, the points at which the global becomes local, and the
local opens up onto the global. Since the meaning of these terms has to be understood in the context of
any particular struggle, one is always acting both globally and locally: Think globally, act appropriately!
Fight locally because that is the scene of action, but aim for the global because that is the scene of agency.
Local struggles directly target national and international axioms, at the precise point of their insertion
into the field of immanence. This requires the imagination and construction of forms of unity,
commonality and social agency which do not deny differences. Without such commonality, politics is too
easily reduced to a question of individual rights (i.e., in the terms of classical utility theory); difference
ends up trumping politics, bringing it to an end. The struggle against the disciplined mobilization of
everyday life can only be built on affective commonalities, a shared responsible yearning: a yearning out
towards something more and something better than this and this place now. The Left, after all, is defined
by its common commitment to principles of justice, equality and democracy (although these might
conflict) in economic, political and cultural life. It is based on the hope, perhaps even the illusion, that
such things are possible. The construction of an affective commonality attempts to mobilize people in a
common struggle, despite the fact that they have no common identity or character, recognizing that they
are the only force capable of providing a new historical and oppositional agency. It strives to organize
minorities into a new majority.

Page 98 of 1481
Page 99 of 1481
Page 100 of 1481
**Classic Turns**

Page 101 of 1481


Derrida Turn: 2AC
TURN CALL TO REJECT RE-INVENTS HIERARCHIES POLITICAL ACTION IS KEY TO
TRANSCEND THEIR FALSE BINARIES

Newman 2001
[Saul, Sociology @ Macquarie University, Philosophy & Social Criticism 27: 3, pp. 4-6//uwyo]

It must be made clear, however, that Derrida does not simply want to invert the terms of these binaries so that the
subordinated term becomes the privileged term. He does not want to put writing in the place of speech, for instance. Inversion in this
way leaves intact the hierarchical, authoritarian structure of the binary division. Such a strategy only re-
affirms the place of power in the very attempt to overthrow it. One could argue that Marxism fell victim to this logic by replacing the
bour- geois state with the equally authoritarian workers state. This is a logic that haunts our radical political imaginary. Revolutionary political theories

have often succeeded only in reinventing power and authority in their own image. However, Derrida also
recognizes the dangers of subversion that is, the radical strategy of overthrowing the hierarchy altogether,
rather than inverting its terms. For instance, the classical anarchists critique of Marxism went along the lines that Marxism neglected political power in particular the power of
the state for economic power, and this would mean a restoration of political power in a Marxist revolution. Rather, for anarchists, the state and all forms of political power must

Derrida believes that subversion and inversion both culminate in the same
be abolished as the first revolutionary act. However,

thing the reinvention of authority, in different guises. Thus, the anarchist critique is based on the Enlightenment idea of a rational and
moral human essence that power denies, and yet we know from Derrida that any essential identity involves a radical exclusion or sup- pression of other identities. Thus,

anarchism substituted political and economic authority for a rational authority founded on an Enlighten-
ment-humanist subjectivity. Both radical politico-theoretical strategies then the strategy of inversion, as
exemplified by Marxism, and the strategy of subversion, as exemplified by anarchism are two sides of
the same logic of logic of place. So for Derrida:
What must occur then is not merely a suppression of all hierarchy, for an- archy only consolidates just as surely the established order of a metaphys- ical hierarchy; nor is it a
simple change or reversal in the terms of any given hierarchy. Rather the Umdrehung must be a transformation of the hierar- chical structure itself.

to avoid the lure of authority one must go beyond both the anarchic desire to destroy hierarchy,
In other words,

and the mere reversal of terms. Rather, as Derrida suggests, if one wants to avoid this trap the hierar- chical structure itself must be transformed.
Political action must invoke a rethinking of revolution and authority in a way that traces a path between
these two terms, so that it does not merely reinvent the place of power. It could be argued that Derrida propounds an anarchism
of his own, if by anarchism one means a questioning of all authority, including textual and philosophical authority, as well as a desire to avoid the trap of reproducing authority
and hierarchy in ones attempt to destroy it.

This deconstructive attempt to transform the very structure of hier- archy and authority, to go beyond the binary opposition, is also found in Nietzsche. Nietzsche believes that

One must, he argues,


one cannot merely oppose auth- ority by affirming its opposite: this is only to react to and, thus, affirm the domination one is supposedly resisting.

tran- scend oppositional thinking altogether go beyond truth and error, beyond being and becoming, beyond good and evil.
For Nietzsche it is simply a moral prejudice to privilege truth over error. However, he does not try to counter
this by privileging error over truth, because this leaves the opposition intact. Rather, he refuses to confine his view of the
world to this opposition: Indeed what compels us to assume that there exists any essential antithesis between true and false? Is it not enough to suppose
grades of apparentness and as it were lighter and darker shades and tones of appearance? Nietzsche displaces, rather than replaces, these oppositional and authoritarian
structures of thought he displaces place. This strategy of displacement, similarly adopted by Derrida, provides certain clues to developing a non-essentialist theory of resist-

. Rather than reversing the terms of the binary opposition, one should perhaps question,
ance to power and authority

and try to make prob- lematic, its very structure.

Page 102 of 1481


Fear of Co-optation Turn: 2AC

FEAR OF CO-OPTATION LEADS TO PASSIVE ACCEPTANCE OF OPPRESSION THE


BETTER ALTERNATIVE IS TO ENGAGE IN POLITICS WHILE ACKNOWLEDGING THEIR
INCOMPLETION THAT VERY FAILURE SPURS MORE RADICAL TRANSFORMATION
OF THE SYSTEMS UNCONSCIOUS COORDINATES

Zizek 2004
[Slavoj, Ocean Rain, Liberation Hurts: An Interview with Slavoj Zizek, The Electronic Book Review, July 1, 2004,
www.electronicbookreview.com/v3/servlet/ebr?comman=view_essay&essay_id=rasmussen, Acc. 10-23-04//uwyo-ajl]

Zizek: Im trying to avoid two extremes. One extreme is the traditional pseudo-radical position
which says, If you engage in politics - helping trade unions or combating sexual harassment,
whatever - youve been co-opted and so on. Then you have the other extreme which says,
Ok, you have to do something. I think both are wrong. I hate those pseudo-radicals
who dismiss every concrete action by saying that This will all be co-opted. Of course,
everything can be co-opted [chuckles] but this is just a nice excuse to do absolutely
nothing. Of course, there is a danger that - to use the old Maoist term, popular in European
student movements thirty some years ago, the long march through institutions will last so
long that youll end up part of the institution. We need more than ever, a parallax view - a
double perspective. You engage in acts, being aware of their limitations. This does not
mean that you act with your fingers crossed. No, you fully engage, but with the
awareness that - the ultimate wager in the almost Pascalian sense - is not simply that
this act will succeed, but that the very failure of this act will trigger a much more radical
process.

Page 103 of 1481


Fear of Co-optation Turn: 1AR

EXTEND THE 2AC #__ ZIZEK 2004 EVIDENCE WHICH INDICATES THAT AVOIDING CO-
OPTATION CREATES PARALYZING POLITICS THAT ENABLE OPPRESSION TO FILL THE
VOID. ONLY THE AFF POLITICS OF INCOMPLETION SHATTERS STATUS QUO
POLITICS BY UNDERMINING THE ROOT CAUSE OF VIOLENCE

AND, FEAR OF CO-OPTATION FALLS INTO POWERLESSNESS

Pritchard 2000
[Elizabeth, Bowdoin College, Hypatia, Summer, CWI]

The third way in which a feminist reinscription of the development logic of mobility jeopardizes
women's well-being is that a fixation on development or liberty as escaping or exiting the
"closure" entailed in various locations reinscribes a utopianism that jeopardizes the
possibility of a politics directed toward constructing an alternative and liveable world. And
here again, some postmodern theorists betray the legacy of the Enlightenment. The dislocated
mobile subjects of the Enlightenment are "at home" in a utopianism that defers the
burden of the definitions, representations, and affiliations necessary for democratic political
action. Such burdensome tasks are seen to threaten closure--and hence are repudiated.

Reinhart Koselleck argues that a legacy of the Enlightenment is the persistence and pathology of
utopianism (Koselleck 1988). The tradition of Enlightenment critique arises in the context of
political absolutism that is instituted in the wake of religious wars. Setting themselves against
the constraining tendencies of absolutism, the Enlightenment thinkers, whose field of action is a
"single global world," engage in a "ceaseless movement" of depersonalized critique within the
horizon of an "open-ended future." This produces a utopian self-conception whereby "modern
man is destined to be at home everywhere and nowhere" (Koselleck 1988, 5). The error in this
legacy of modernity, according to Koselleck, is that an unpolitical position of utopianism is
mistaken as a political position. The Enlightenment thinkers were unwilling to take
responsibility for history by formulating concrete policies and goals and designing and
joining social and political institutions; instead they resorted to polar positions as persons who
negate present realities and dream of a future they are powerless to realize.

Page 104 of 1481


Page 105 of 1481
The Fetish: 2AC
DISAVOWAL OF THE VIOLENCE OF REPRESENTATION AND CALLS FOR INTERNAL
RETHINKING RELY ON ASSUMPTIONS OF METAPHYSICAL INNOCENCE, FETISHIZING
AN AUTHENTICITY THAT NEVER EXISTED

Bewes 97
[Timothy, doctorate in English Literature at the University of Sussex, Cynicism and Postmodernity, New
York City: Verso, 1997, 195-6//uwyo-ajl]

postmodernism has actually become something. Its


Despite the diligence and the sterling efforts of its best theoreti-cians, then, it seems that

principal characteristic is the retreat from and disavowal of the violence of representation - both political
and semiotic. There are three further aspects to this essentially ignominious cultural operation: (i) a cultivation of stupidity (what I
have called Kelvinism, or 'metaphysical innocence') as a means of circumventing the ideational 'brutality' of the

political life; (ii) a recourse to the idea of an internal or subjective 'truth of the soul' which transcends
political reality, along with the contingencies of representation. Both of these signal an attachment to a
surface/ depth model of subjectivity which in each case amounts to a fetishization of authenticity,
whether by opting to 'remain' on the surface, or by retreating 'inwards'; (iii) a collapse of faith by individuals and even politicians
themselves, not only in the political infrastructure but in the very' concept of political engagement - here it becomes apparent that Tony Blair, for example, is more 'postodern'
than any theoretician. .

these three responses stand in an approximately analogous relationship to the archetypal forms in which consciousness, in a
It should be clear that

state of anxiety, shrinks from the violence of determinate negation and 'strives to hold on to what it is in danger of losing'. 59 At various
points throughout the present work I have used the terms 'decadence', 'irony' and 'relativism' to refer to these instances of an epistemological loss of nerve, this

capitulation to 'things as they are'; it may be as well here to remind ourselves of the terms in which Hegel describes these manifestations of a retreat from
truth. Consciousness, he says, at the decisive moment in which it is required to go beyond its own limits, (i) 'wishes to remain in a state' of unthinking inertia'; (ii) gloats over its
own understanding, 'which knows how to dissolve every thought and always find the same barren Ego instead of any content'; (iii) 'entrenches itself in sentimentality, which

assures us that it finds everything to be good in its kind'. 60Postmodernism, an empirical social condition - by which I mean that a series of critical-theoretical
strategies has attained a certain concrete form - legitimizes these symptoms of cultural anxiety; postmodernism becomes synonymous,
therefore, with deceleration, with a sense of cultural and political conclusivity; postmodernism is the principal vehicle of what Baudrillard calls

'the illusion of the end'.

AUTHENTICITY FETISHIZATION AND ITS FEAR OF REASON AND VIOLENCE ALLOW


US TO SPEND HOURS DEBATING THE FINE POINTS OF BAUDRILLARIAN ETHICS
WHILE GAS CHAMBERS ARE BUILT

Bewes 97
[Timothy, doctorate in English Literature at the University of Sussex, Cynicism and Postmodernity, New
York City: Verso, 1997,146-7//uwyo-ajl]

If it is unreasonable to suppose that the Final Solution was potentiated or even necessarily facilitated by
Schmitt's theories, it is certainly the case that this metaphysical structure of domination in the Third
Reich, whereby the status of public citizens is reduced to a level determined entirely in the 'natural' or
biological realm of necessity, is foreshadowed in his 1927 essay. In an abstract and insidious way Schmitt
introduces the idea that the 'transcendent' realm of the political, as a matter of course, will not
accommodate a people with insufficient strength to ensure its own participation, and that such a fact is
ipso facto justification for its exclusion. 'If a people no longer possesses the energy or the will to maintain
itself in the sphere of politics, the latter will not thereby vanish from the world. Only a weak people will

Page 106 of 1481


disappear.'130 Schmitt's concept of the 'political', quite simply, is nothing of the sort - is instead weighed
down by necessity, in the form of what Marshall Berman calls German-Christian interiority - by its
preoccupation with authenticity, that is to say, and true political 'identity'. Auschwitz is a corollary not of
reason, understood as risk, but of the fear of reason, which paradoxically is a fear of violence. The stench
of burning bodies is haunted always by the sickly aroma of cheap metaphysics.

Page 107 of 1481


Page 108 of 1481
The Fetish: 1AR
THEIR ARGUMENT THAT WE SHOULD AVOID DISCURSIVE VIOLENCE IS
SYMPTOMATIC OF ANXIETY IN THE WAKE OF CONTEMPORARY
FRAGEMENTATION. THIS FEAR OF POLITICAL VIOLENCE ASSUMES THE EXISTENCE
OF A UTOPIAN VIOLENCE-FREE STATE OF METAPHYSICAL INNOCENCE, IGNORING
THE WAY THAT SUCH A STATE IS FORECLOSED BY OUR ENTRY INTO THE POLITICAL,
DESTROYING ALL CRITICAL SOLVENCY. CROSS-APPLY THE FIRST BEWES 97
EVIDENCE.

THIS MOURNING OF AUTHENTICITY IS DEPOLITICIZING. IT NECESSITATES A TURN


TOWARDS INTERNAL QUESTIONING AND A RETREAT FROM POLITICAL
ENGAGEMENT, ALLOWING US TO FOCUS ON THE TRUTH OF OUR INDIVIDUAL
IDENTITIES WHILE WERE COMPLICIT WITH ATROCITY, IN MUCH THE SAME WAY
THAT EICHMANN TOILED AWAY ENSURING THAT, UNDERNEATH IT ALL, HE WAS A
GOOD PERSON WHILE HE PARTICIPATED IN GENOCIDE. THATS THE SECOND
BEWES CARD

THIS PRECEDES ALL OF THEIR ARGUMENTS BECAUSE THE RESISTANCE ADVOCATED


BY THEIR ALTERNATIVE CANNOT OCCUR WITHOUT POLITICAL ENGAGEMENT

AUTHENTICITY CAUSES THE INTELLECTUAL PARALYSIS THAT ALLOWS ATROCITIES


TO OCCUR

Bewes 97
[Timothy, doctorate in English Literature at the University of Sussex, Cynicism and Postmodernity, New
York City: Verso, 1997,154-5//uwyo-ajl]

Thus Fackenheim's encounter with the horror and the 'obscene rationality' of Auschwitz, secondly, displays an anxiety concerning the perceived integrity of the Third Reich,
which is in fact an instinctive gesture of revulsion at the extremes which it is possible for man to justify. This revulsion, perfectly defensible in itself, is a prerequisite and an
It is this question of Hitler's 'integrity', perhaps more
important if unacknowledged constituent of the postmodern 'critique' of rationality.

than anything, which leads to the intellectual paralysis characteristic of postmodernity, of which the most
typical symptom is cynicism, in its various forms. On one level, of course, Hitler's programme was thoroughly 'integrated', if by this is meant 'internally
coherent'. Certainly the consistency with which both 'good' and 'bad' Jews were persecuted - and Eichmann's

diligence, it emerged, was exemplary in this regard - ensured that the Third Reich could indeed boast of a
mindless sort of integrity. It is this consistency, together with what he calls its 'cosmic scope', which for Fackenheim elevates Nazi ideology to the status of a
Weltanschauung, deserving of 'respect, even awe' .154 In this, how ever, Fackenheim's conception of what is or is not appropriate to the machinery of a political regime is
warped, his values infected by those of the very society he is attempting (or refusing) to analyse. Integrity, to begin with, is not a political virtue, since it is one of those
characteristics (like honesty, or moral scrupulousness) which cannot by their very nature appear intact in the public sphere.

Page 109 of 1481


integrity, particularly in this narrow sense of 'internal coherence' (and this is the third point), has no positive correlation with
Furthermore

rationality, and is in fact profoundly opposed to the processes of reason conceived, as Gillian Rose has defined it, in
terms of risk '1" as a continually hazardous endeavour of going beyond existing limits, a spirit directed
towards progress and the future, in which the "Hegelian moment of determinate negation is actively and recursively constitutive. The violence'
represented by determinate negation is in essence mobilized against integration, just as it is perpetrated by the
'disintegrated' figures of Rameau, Daisy Miller, or Walter Benjamin's 'destructive character' against the philosopher) Diderot-Moi, the dullard Winterbourne, and the 'etui-man' of

Benjamin's essay .

Page 110 of 1481


Page 111 of 1481
Authenticity Impossible: 1AR
1. THERE IS NO PURE RELATIONSHIP WITH ANYTHING BECAUSE EVERY
ENGAGEMENT IS TAINTED BY MISPERCEPTION, COGNITIVE MEDIATION, AND
VIOLENCE. CROSS-APPLY THE FIRST BEWES 97 CARD

2. EVERY ACT IS ALWAYS ALREADY INAUTHENTIC BECAUSE OF ITS MEDIATION BY


SIGNFICATION AND CULTURE. ONLY YOUR DEATH IS
AUTHENTIC

Bewes 97
[Timothy, doctorate in English Literature at the University of Sussex, Cynicism and Postmodernity, New
York City: Verso, 1997, 59//uwyo-ajl]

If the K Foundation sought by such an act to demonstrate their freedom from the incrimination of art by
capital, and thereby their own authenticity as artists, they inevitably failed. As perhaps they realize: 'I
don't think people should find out about it,' Cauty tells Reid. 'Nobody would understand. The shock value
would spoil ,it. Because it doesn't want to be a shocking thing; it just wants to be a fire.' Such an absence
of semiosis is unthinkable and unattainable: ,how could the incineration of one million pounds possibly
refuse to signify? As Reid observes 0 his article, 'this piece is the beginning of ithe art work. Without this
article none of it ever existed.'

Drummond and Caut are compromised from the outset, not only by money but by art itself, by
representation, by the 'passage' from idea to vehicle, from signified to signifier - precisely because such a
passage is neither linear nor free from diversion, is in fact

",eciprocal and systemically constituted: one begins at the signifier as often as one begins at the signified,
and at every place in the system, and at no place. The intention to demonstrate authenticity is impli-cated
in the demonstration itself. To have bothered to'destroy the money at all, even in complete privacy, is
already to determine their sabservience to it, to bow to its power. To make a statement, 'artis-tic' or
otherwise, is to concede at once to the violent demands of signification. Absolute authenticity
necessitates one's own extinc-tion; only in death does one accede to the immaculate. The business of
humanity, 'and thus of art, is precisely one of compromise, 'inau-thenticity' and fabication. In finding the
artistic institution phoney and depraved, the K Foundation confuse ethics with aesthetics. Their failure to
bring about the end of art dictates that Drummond and Cauty proceed logically to self-destruction; the
next bonfire must surely,be one intended for their own physical immolation.

Page 112 of 1481


Kulynych Turn: 2AC

DEBATE HAS VALUE DELIBERATIVE POLITICS AND PERFORMANCE RECAPTURE


DELIBERATIVE SPACE FROM OPPRESSIVE STRUCTURES

Kulynych 97
[Jessica, Asst. Prof. of Poli Sci @ Winthrop, Polity 30: 2, Winter//uwyo]

A performative perspective on participation enriches our understanding of deliberative democracy. This enlarged understanding can be demonstrated by considering the
examination of citizen politics in Germany presented in Carol Hager's Technological Democracy: Bureaucracy and Citizenry in the West German Energy Debate.(86) Her work
skillfully maps the precarious position of citizen groups as they enter into problemsolving in contemporary democracies. After detailing the German citizen foray into technical
debate and the subsequent creation of energy commissions to deliberate on the long-term goals of energy policy, she concludes that a dual standard of interpretation and
evaluation is required for full understanding of the prospects for citizen participation. Where traditional understandings of participation focus on the policy dimension and
concern themselves with the citizens' success or failure to attain policy preferences, she advocates focusing as well on the discursive, legitimation dimension of citizen action.
Hager follows Habermas in reconstituting participation discursively and asserts that the legitimation dimension offers an alternative reason for optimism about the efficacy of

, success is not defined in terms of getting, but rather in terms of


citizen action. In the discursive understanding of participation

solving through consensus. Deliberation is thus an end in itself, and citizens have succeeded whenever
they are able to secure a realm of deliberative politics where the aim is forging consensus among
participants, rather than achieving victory by some over others.
Through the creation of numerous networks of communication and the generation of publicity, citizen action furthers democracy by assuming a substantive role in governing
and by forcing participants in the policy process to legitimate their positions politically rather than technically. Hager maintains that a sense of political efficacy is enhanced by
this politically interactive role even though citizens were only minimally successful in influencing or controlling the outcome of the policy debate, and experienced a real lack of
autonomy as they were coerced into adopting the terms of the technical debate. She agrees with Alberto Melucci that the impact of [these] movements cannot.., be judged by
normal criteria of efficacy and success .... These groups offer a different way of perceiving and naming the world. They demonstrate that alternatives are possible, and they
expand the communicative as opposed to the bureaucratic or market realms of societal activity.(87)

Yet her analysis is incomplete. Like Habermas, Hager relies too heavily on a discursive reconstitution of political action. Though she recognized many of the limitations of
Habermas's theory discussed above, she insists on the :innovative and creative potential of citizen initiatives. She insists that deliberative politics can resist the tendency toward
authoritarianism common to even a communicative, deliberative search for objective truth, and that legitimation debates can avoid the tendency to devolve into the technical
search for the better argument. She bases her optimism on the non-hierarchical, sometimes even chaotic and incoherent, forms of decisionmaking practiced by citizen
initiatives, and on the diversity and spontaneity of citizen groups.

Unfortunately, it is precisely these elements of citizen action that cannot be explained by a theory of communicative action. It is here that a performative conception of political

action implicitly informs Hager's discussion. From a performative perspective , the goal of action is not only to secure a realm for deliberative politics, but to
disrupt and resist the norms and identities that structure such a realm and its participants. While Habermas
theorizes that political solutions will emerge from dialogue, a performative understanding of participation highlights the limits of dialogue and

the creative and often uncontrollable effect of unpremeditated action on the very foundations of
communication.

defiance and disruption that bring


When we look at the success of citizen initiatives from a performative perspective, we look precisely at those moments of

the invisible and unimaginable into view. Although citizens were minimally successful in influencing or controlling the out come of the policy debate and
experienced a considerable lack of autonomy in their coercion into the technical debate, the goal-oriented debate within the energy commissions could

be seen as a defiant moment of performative politics. The existence of a goal-oriented debate within a
technically dominated arena defied the normalizing separation between expert policymakers and
consuming citizens. Citizens momentarily recreated themselves as policymakers in a system that defined
citizens out of the policy process, thereby refusing their construction as passive clients. The disruptive
potential of the energy commissions continues to defy technical bureaucracy even while their decisions
are non-binding.

Page 113 of 1481


Kulynych Turn: 1AR

NEXT, EXTEND OUR KULYNYCH EVIDENCE:

DEBATE IS AN END UNTO ITSELF BECAUSE IT DISRUPTS NORMALIZING SYSTEMS BY


ELUCIDATING THE LIMITS AND CONSTRAINTS ON DIALOGUE THROUGH A
PERFORMATIVE ACT OF RESISTANCE WHAT WE DO IS NOT JUST CONSTITUTED BY
THE RATIONALITY OF OUR ARGUMENTS BUT BY THE TECHNIQUES WE USE
WHETHER OR NOT THIS PARTICULAR DISCUSSION CAUSES POLITICAL ACTION, OUR
ACT OF DEFIANCE EMPOWERS IDENTITIES AND MAKES DEBATE MEANINGFUL

Page 114 of 1481


Page 115 of 1481
Praxis Turn: 2AC
AND, THEORETICAL INTERVENTIONS EMPTY OF PRACTICE JUST COMMODIFY AND
DESTROY THE CRITICISM PERM SOLVES BEST

Routledge 96
[Paul, The Third Space as Critical Engagement, Antipode 28(4), October, 399//uwyo]

One of the problems of theory is that we attempt to understand processes, things, others, in a
moment of cultural petrification, where we objectify living cultural-political forms (Jeudy, 1994).
Such theory takes place at a distance. In the production of theory we are distanced from what
Bey (1994) terms immediatism direct, lived experience. Rather we become engaged in
representations of (an)others reality. As such, we are alienated form the lived moment,
enmeshed in the theory market, where the production of theory becomes another part of
spectacular production, another commodity. This commodification imples that a mediation has
occurred, and with every mediation so our alienation from live experience increases. As Mies
(1983) notes, we are too frequently engaged in uninvolved spectator knowledge, one separated
form active participation. As such, research and theory can remain analytical and disembodied.
It is not lived. To enact a third space within and between academia and activism is to attempt to
live theory in the immediate.

Page 116 of 1481


Praxis Turn:1AR

AND, EXTEND THE 2AC #__, ROUTLEDGE PRAXIS ARGUMENT. THEORETICAL


ENGAGEMENT REMOVES ITSELF FROM LIVED EXPERIENCE, RENDERING ITSELF
ANOTHER COMMODITY TO BE BOUGHT AND SOLD, PREVENTING
TRANSFORMATION

AND, THINKING ABOUT THINKING IS USELESS. THINKING ABOUT DOING IS KEY TO


CHANGING STRUCTURAL WRONGS

Booth 97
[Ken, Chair of Intl Pltcs @ Wales, Critical security studies, Ed. Krause & Williams, p. 114//uwyo]

study of
Security is concerned with how people live. An interest in practice (policy relevance) is surely part of what is involved in being a security specialist. The

security can beneft from a range of perspectives, but not from those who would refuse to engage with
the problems of those, at this minute, who are being starved, oppressed, or shot. It is therefore legitimate to ask what any theory that
purports to belong within world politics has to say about Bosnia or nuclear deterrence. Thinking about thinking is important, but, more

urgently, so is thinking about doing. For those who believe that we live in a humanly constituted world, the distinction between theory and practice
dissolves: theory is a form of practice, and practice is a form of theory. Abstract ideas about emancipation will not suffice: it is

important for critical security studies to engage with the real by suggesting policies, and sites of change,
to help humankind in whole or in part, to move away from its structural wrongs.

ALSO, MUST LINK PROTEST TO DEMANDS ON THE STATE OR WE LAPSE INTO


POLITICAL PARALYSIS IN THE FACE OF OPPRESSION

Foucault 82
[Michel, God, Politics and Ethics: An Interview, The Foucault Reader, Trans.
Catherine Porter, Ed. Paul Rabinow, 377//uwyo-ajl]

Q. And this is hard to situate within a struggle that is already under way, because the lines are drawn by others. . . .

ethics is a practice; ethos is a manner of being. Let's take an example that touches us
M.F. Yes, but I think that
all, that of Poland. If we raise the question of Poland in strictly political terms, it's clear that we quickly
reach the point of saying that there's nothing we can do. We can't dispatch a team of para- troopers, and
we can't send armored cars to liberate Warsaw. I think that, politically, we have to recognize this, but I think we
also agree that, for ethical reasons, we have to raise the problem of Poland in the form of a
nonacceptance of what is. happening there, and a nonacceptance of the passivity of our own
governments. I think this attitude is an ethical one, but it is also political; it does not consist in saying merely,
"I protest," but in making of that attitude a political phenomenon that is as substantial as
possible, and one which those who govern, here or there, will sooner or later be obliged to take
into account.

Page 117 of 1481


Page 118 of 1481
Praxis Turn: 2AR

NEXT, EXTEND THE 2AC #__, THE ROUTLEDGE PRAXIS ARGUMENT.

OUR POSITION IS THAT THE AFFS DEPLOYMENT OF THEORY IS NOTHING BUT AN


EMPTY GESTURE THAT FAILS BECAUSE ITS DEVOID OF PRACTICE

A PURELY ACADEMIC CRITICISM, LIKE THE NEGS, DIVORCES ITSELF FROM ANY
SENSE OF PRAXIS, INEXORABLY COMMODIFYING ARGUMENT, WHERE THEORY
BECOMES ANOTHER PRODUCT OF UNIVERSITY FACTORIES

NOT ONLY DOES THIS ARGUMENT PROVIDE SOLVENCY FOR OUR PWERM, WHICH
COMBINES THEORY AND PRACTICE, BUT IT SERVES AS A POWERFUL INDICTMENT
OF THE POTENTIAL FOR ANY POSITIVE CRITICAL IMPACT

Page 119 of 1481


Praxis Turn: Ext

REAL PROBLEMS DEMAND ACTION IVORY TOWER CRITICISMS CAUSE


IMMOBILIZATION

Booth 95
[Ken, Prof. of IR, Human wrongs and international relations, International Affaris, ASP//delizzozzle]

Philosophical sceptics, for whom nothing is certain, and so for whom the bases of action are
always problematic, are a familiar feature of academic life Tom Stoppard enjoyable caricatured
them in his clever comedy Jumpers, and in particular in the scene in which philosophical sceptics
were discussed whether the train for Bristol left yesterday from Paddington station. On what
basis could they ever know? Even if they were actually on the train that was supposed to leave
for Bristol, might not the happening be explained by Paddington leaving the train? We all know
such conundrums, and indeed such people Meanwhile, flesh is being fed or famished, and
people are being tortured and killed And even philospohical skeptics have to catch trains Some
of them do Unless acadmeics are merely to spread confusion, or snipe from the windows of
ivory towers, we must engage with the real. This means having the courage of our confusions
and thinking and acting without certainty.

In reply to those sensitive to post-colonial critiques of Western imperialism I would argue that
just because many Western ideas were spread by commerce and the Gatling gun, it does not
follow that every idea originating in the West, or backed by Western opinion, should therefore
simply be labelled imperialist and rejected. There are some ethnocentric ideas and individual
human rights is one of them for which we should not apologize. Furthermore, I do not see the
dissemination of powerful social and political ideas as necessarily occurring in one direction
only. As the economic and political power of Asia grows, for example, so will its cultural power.
World politics in the next century will be more Asian than the present one. What matters from a
cosmopolitan perspective is not the birthplace of an idea, but the meaning we give it.

Page 120 of 1481


Presymbolism Turn: 2AC

TURN GROUNDING RESISTANCE IN A BEFORE THE FALL IDENTITY RENDERS THE


COLONIZED PASSIVE VICTIMS WITHOUT AGENCY ACTIVISM WITHIN THE SYSTEM
USES ITS OWN EXCESSES TO DISMANTLE IT

Zizek '99
[Slavoj, Senior Researcher at Institute for Social Studies, Ljubliana and Badass, The Ticklish Subject: the
absent centre of political ontology, New York: Verso, 1999, 256-7//uwyo-ajl]

Against Butler, one is thus tempted to emphasize that Hegel was well aware of the retroactive
process by means of which oppressive power itself generates the form of resistance is not this
very paradox contained in Hegel's notion of positing the presuppositions, that is, of how the
activity of positing-mediating does not merely elaborate the presupposed immediate-natural
Ground, but thoroughly transforms the very core of its identity? The very In-itself to which
Chechens endeavour to return is already mediated-posited by the process of modernization,
which deprived them of their ethnic roots.

This argumentation may appear Eurocentrist, condemning the colonized to repeat the European
imperialist pattern by means of the very gesture of resisting it however, it is also possible to
give it precisely the opposite reading. That is to say: if we ground our resistance to imperialist
Eurocentrism in the reference to some kernel of previous ethnic identity, we automatically
adopt the position of a victim resisting modernization, of a passive object on which imperialist
procedures work. If, however, we conceive our resistance as an excess that results from the way
brutal imperialist intervention disturbed our previous self-enclosed identity, our position
becomes much stronger, since we can claim that our resistance is grounded in the inherent
dynamics of the imperialist system that the imperialist system itself, through its inherent
antagonism, activates the forces that will bring about its demise. (The situation here is strictly
homologous to that of how to ground feminine resistance: if woman is 'a symptom of man', the
locus at which the inherent antagonisms of the patriarchal symbolic order emerge, this in no
way constrains the scope of feminine resistance but provides it with an even stronger
detonating force.) Or to put it in yet another way the premise according to which resistance
to power is inherent and immanent to the power edifice (in the sense that it is generated by the
inherent dynamic of the power edifice) in no way obliges us to draw the conclusion that every
resistance is co-opted in advance, including in the eternal game Power plays with itself the key
point is that through the effect of proliferation, of producing an excess of resistance, the very
inherent antagonism of a system may well set in motion a process which leads to its own
ultimate downfall.

It seems that such a notion of antagonism is what Foucault lacks: from the fact that every
resistance is generated ('posited') by the Power edifice itself, from this absolute inherence of
resistance to Power, he seems to draw the conclusion that resistance is co-opted in advance,
that it cannot seriously undermine the system that is, he precludes the possibility that the
system itself, on account of its inherent inconsistency, may give birth to a force whose excess it
is no longer able to master and which thus detonates its unity, its capacity to reproduce itself. In
short, Foucault does not consider the possibility of an effect escaping, outgrowing its cause, so
that although it emerges as a form of resistance to power and is as such absolutely inherent to
it, it can outgrow and explode it. (the philosophical point to be made here is that this is the
fundamental feature of the dialectical-materialist notion of 'effect': the effect can 'outdo' its
cause; it can be ontologically 'higher' than its cause.) One is thus tempted to reverse the
Foucauldian notion of an all-encompassing power edifice which always-already contains its

Page 121 of 1481


transgression, that which allegedly eludes it: what if the price to be paid is that the power
mechanism cannot even control itself, but has to rely on an obscene protuberance at its very
heart? In other words: what effectively eludes the controlling grasp of Power is not so much the
external In-itself it tries to dominate but, rather, the obscene supplement which sustains its own
operation.

Page 122 of 1481


Presymbolism Turn: 1AR

AND, EXTEND THE 2AC # ___ ZIZEK 99 PRESYMBOLISM TURN. RESISTING


OPPRESSION CREATES A BEFORE THE FALL FANTASY, RENDERING US PASSIVE
VICTIMS. ONLY USING THE SYSTEMS OWN EXCESSES AGAINST ITSELF EXPLODES IT
FROM WITHIN, CAUSING ITS DOWNFALL

ALSO, POWER IS SPLIT FROM WITHIN BY ITS TRAUMATIC EXCESS USING THAT
DISAVOWED FOUNDATION DISMANTLES IT

Zizek '97
[Slavoj, The Game, The Plague Fantasies, NYC: Verso, 1997, 26-7//uwyo-ajl]

This last point must be further radicalized: the power edifice itself is split from within: in order
to reproduce itself and contain its Other, it has to rely on an inherent excess which grounds it -
to put it in the Hegelian terms of speculative identity, Power is always-already its own
transgression, if it is to function, it has to rely on a kind of obscene supplement. It is therefore
not enough to assert, in a Foucauldian way, that power is inextricably linked to counter-power,
generating it and being itself conditioned by it: in a self-reflective way, the split is always-
already mirrored back into the power edifice itself, splitting it from within, so that the gesture of
self-censorship is consubstantial with the exercise of power. Furthermore, it is not enough to
say that the `repression' of some libidinal content retroactively eroticizes the very gesture of
`repression' - this `eroticization' of power is not a secondary effect of its exertion on its object
but its very disavowed foundation, its `constitutive crime', its founding gesture which has to
remain invisible if power is to function normally. What we get in the kind of military drill
depicted in the first part of Full Metal Jacket, for example, is not a secondary eroticization of the
disciplinary procedure which creates military subjects, but the constitutive obscene supplement
of this pro- cedure which renders it operative. Judith Butler27 provides a perfect example of,
again, Jesse Helms who, in his very formulation of the text of the anti-pornography law~
displays the contours of a particular fantasy - an older man who engages in sadomasochistic
sexual activity with another, younger man, preferably a child - which bears witness to his own
perverted sexual desire. Helms thus unwittingly brings to light the obscene libidinal foundation
of his own crusade against pornography.

Page 123 of 1481


Rejection Bad Turn: 2AC

TURN - CALL TO REJECT IMPOVERISHES DISCOURSE PERM SOLVES BEST

Ashley 88
[Richard, Untying the Soveregin State: A Double Reading of the Anarchy Problematique, Millennium:
Journal of International Studies 17(2), June, 227-262//uwyo]

The monological reading of theoretical discourse of the anarchy problematique thus leaves the reader with
the dichotomous choice of positions mentioned earlier: the choice titled the blackmail of the heroic practice. One must be either
inside this discourse or outside, either for or against. On the one hand, in order to enter this discursive enclosure even if ones interest is criticism
or reform one must adopt a subjective standpoint that affirms the objective and original powers of the heroic practice and interpret everything in its terms. One
must resign oneself to complicity with the knowledgeable practices by which the anarchy problematique is constituted as a self-evident and objective condition

in order to stand outside this discursive enclosure thus to repudiate the hard core
of life. On the other hand,

one must condemn oneself to a position of practical futility, no


representations of the anarcy problematique

matter how self-righteous it may be. Saying no to a powerful discourse that participates in the
construction of the self-evIdent truth of the anarchy problematique, one is left to construct
subjective counter-truths that cannot be effective precisely because they remove themselves
from the workings of objective sources of power in history.

Page 124 of 1481


Rejection Bad Turn: 1AR

EXTEND THE 2AC #___ ASHLEY EVIDENCE. TOTAL REJECTION LOCKS US OUTSIDE OF
DISCURSIVE SYSTEMS, PREVENTING US FROM CHANGING THEM FROM WITHIN,
CONDEMNING US TO PASSIVE FUTILITY. THIS IS A NET BENEFIT TO THE PERM

ALSO, TOTAL DOGMATIC SYSTEMS WHERE ONE SIDE IS RIGHT AND THE OTHER
WRONG CREATE TOTALIZING POLITICS, RESULTING IN SLAUGHTER AND WAR

Said 94
[Edward W., Representations of the Intellectual: The 1993 Reich Lectures, Vintage,
1994, 113]

Such transfigurations sever the living connection between the intellectual and the movement or
process of which he or she is a part. Moreover there is the appalling danger of thinking of
oneself, ones views, ones rectittude, ones stated positions as all-important. To read over The
God That Failed testimonial is for me a depressing thing. I want to ask: Why as an intellectual
did you believe in a god anyway? And besides, who gave you the right to imagine that your early
belief and later disenchantment were so important? In and of itself religious belief is to me both
understandable and deeply personal: it is rather when a total dogmatic system in which one
side is innocently good, the other irreducibly evil is substituted for the process , the give and
take of vital interchange that the secular intellectual feels the unwelcome and inappropriate
enroachment of one realm on another. Politics becomes religious enthusiasm as it is the case
today in former Yugoslavia

with results in ethnic cleansing, mass slaughter and unending conflict that are horrible to
contemplate.

AND, FOREIGN POLICY CRITICISMS BECOME COMPLICIT WITH THE STRUCTURES


THEY OPPOSE

Ashley 96
[Richard, Erics Best Friend for Life & Prof. of Poli Sci @ ASU, The achievements of
post-structuralism, International Theory: Positivism and Beyond, New York:
Cambridge University Press, 1996, 247-8]

And to these four premises I might add just one more. Under these circumstances, it can make
little sense to rehearse all those strains of argument that have explored the limitations of the
model of critical activity I have been discussing this in the hope that I might thereby open up a
conversation that seems so disposed to closure. Call them post-structuralist or call them what
you will, these, once more, are strains of argument that have rigorously demonstrated how

Page 125 of 1481


very paradoxical is every attempt to cling fast to this model of criticism in the face of all
manner of excessive happenings that transgress or overflow the limits of every rendition of it;
how much every such attempt depends upon strategiems for disciplining excess whose
arbitrariness, whose violence, is right there on the surface for all to see; how much,
therefore, every such attempt must rely upon effecting a blindness to its own emergence;
and how readily, thanks to all of this,

these attempts can be drawn into a complicity (thought not a secret complicity) with those
very practices that would arrest ambiguity, discipline the proliferation of possibilities, tame
resistances, and sustain structures of domination ostensibly opposed.

Page 126 of 1481


Rejection Bad Turn: Ext

REJECTION OF SPECIFIC SOLUTIONS BECAUSE OF RADICAL DKEPTICISM IS MORE


DANGEROUS THAN PLAN

Fierlbeck 94
[Katherine, Prof. Poli Sci @ Dalhousie, Post-Modernism and the Social Sciences: Insights, Inroads, and
Intrusions, History & Theory, 33: 1, ASP//uwyo-ajl]

In many respects, even the dismally skeptical post-modernists are too optimistic in their allegiance to post-modern ideas. As many others have already pointed out, post-
modernism offers little constructive advice about how to reorganize and reinvigorate modern social relations. "The views of the post-modern individual," explains Rosenau, "are
likely neither to lead to a post-modern society of innovative production nor to engender sustained or contained economic growth." This is simply because "these are not post-
modern priorities"(55). Post-modernism offers no salient solutions; and, where it does, such ideas have usually been reconstituted from ideas presented in other times and

places.[9]What we need are specific solutions to specific problems: to trade disputes, to the redistribution of health care resources, to
unemployment, to spousal abuse . If one cannot prioritize public policy alternatives, or assign political responsibility to address such issues, or
even say without hesitation that wealthy nations that steadfastly ignore pockets of virulent poverty are immoral, then the worst nightmares of the most

cynical post-modernists will likely come to life. Such an overarching refusal to address these issues is at
least as dangerous as any overarching affirmation of beliefs regarding ways to go about solving them.

In order to achieve anything, constructive or otherwise, human beings


Post-modernism suffers from -- and is defined by -- too much indeterminacy.

must attempt to understand the nature of things, and to evaluate them. This can be done even if we
accept that we may never understand things completely, or evaluate them correctly. But if paralysis is the most obvious political
consequence of post-modernism, a graver danger lies in the rejection of the "Enlightenment ideals" of universality and impartiality. If the resounding end to the Cold War has
t the opposite of "universalism" is not invariably a coexistence of "little narratives": it can be, and frequently is,
taught us anything, it should be tha

some combination of intolerance, local prejudice, suspicion, bigotry, fear, brutality, and persecution. The
uncritical affiliation with the community of one's birth, as Martha Nussbaum notes, "while not without causal and formative power, is ethically arbitrary, and sometimes ethically
dangerous -- in that it encourages us to listen to our unexamined preferences as if they were ethical laws."[10]

Page 127 of 1481


Ricouer Turn: 2AC

TURN THE SEARCH FOR HIDDEN MOTIVES ENGAGES IN A HERMENEUTICS OF


SUSPICION, RISKING SPIRAL INTO PROFOUND SKEPTICISM

Berman 2001
[Paul Schiff, Assoc. Prof. Law @ U. of Connecticut, Yale Journal of Law and the Humanities, LN]

Ricoeur contrasts two different "poles" among hermeneutic styles. At one pole, "hermeneutics is understood as the manifestation and restoration of ...
meaning." 23 At the other pole, hermeneutics is "understood as a demystification, as a reduction of illusion." 24 It is not entirely clear to me precisely what
a hermeneutics of faith to be one that treats the object of
Ricoeur means by these two categories. Nevertheless, I understand

study as possessing inherent meaning on its own terms. In contrast, the hermeneutics of
suspicion seeks to expose societal practices as illusory edifices that mask underlying
contradictions or failures of meaning. I will return to the first pole in Part Four of this Essay, but for now I wish to focus on the
hermeneutics of demystification and suspicion.

t each of these
Ricoeur locates in the work of Nietzsche, Marx, and Freud the central hallmarks of this suspicious approach. He argues tha

thinkers makes "the decision to look upon the whole of consciousness primarily as
"false' consciousness." 25 Ricoeur sees this perspective as an extension of Descartes' fundamental position of doubt at the dawn of the
Enlightenment. According to Ricoeur, "The philosopher trained in the school of Descartes knows that things are doubtful, that they are not such as they appear;
but he does not doubt that consciousness is such as it appears to itself; in consciousness, meaning and consciousness of meaning coincide." 26

The hermeneutics of suspicion takes doubt one step farther, by distrusting even our
perceptions.
This suspicious position questions the so-called "correspondence [*104] theory" of truth. As we go through our lives, most of us generally assume that our
mental perceptions accord with reality because we believe we have direct access to reality through our senses or through reason. This is the legacy of the
Enlightenment, the "answer" to the fundamental Cartesian doubt. But the hermeneutics of suspicion maintains that human beings create false truths for
themselves.

Such false truths cannot be "objective" because they always serve some interest or
purpose.
By discovering and revealing those interests or purposes, suspicious analysis seeks to expose so-called "false consciousness" generated through social
ideology or self-deception. False consciousness may arise in many different ways. Nietzsche looked to people's self-deceit in the service of the "will to power."
Marx focused on the social being and the false consciousness that arises from ideology and economic alienation. Freud approached the problem of false
consciousness by examining dreams and neurotic symptoms in order to reveal hidden motivations and desires. Thus, "the Genealogy of Morals in Nietzsche's
sense, the theory of ideologies in the Marxist sense, and the theory of ideas and illusions in Freud's sense represent three convergent procedures of
demystification." 27

AND, SKEPTICISM STOPS SOCIAL CHANGE THEIR PARANOIA FORECLOSES UPON


REVOLUTION

Berman 2001
[Paul Schiff, Assoc. Prof. Law @ U. of Connecticut, Yale Journal of Law and the Humanities, LN]

, one might view this as a positive development. One might think people should
Of course

stop being lulled into a false sense of believing that the rhetoric of public life really
matters. If people began to view such rhetoric as a construction of entrenched power, so
the argument might go, they would form the nucleus of a truly revolutionary political
movement.

I doubt that such an eventuality is likely to occur. Moreover, I am not sure that a culture of
suspiciousness is the most effective way to seek political (or personal) change anyway. Suspicious

Page 128 of 1481


analysis seeks to expose the dangers of our enchantment with reason or truth or collectivity, but there are dangers that arise from relentless disenchantment as
well. As [*123] Richard K. Sherwin has observed,

Without the means of experiencing more profound enchantments , without communal rituals and social
dramas through which the culture's deepest beliefs and values may be brought to life and collectively reenacted, those beliefs ultimately
lose their meaning and die... . Forms of enchantment in the service of deceit, illicit desire, and
self-gratification alone must be separated out from forms of enchantment in the service of

feelings, beliefs, and values that we aspire to affirm in light of the self, social, and legal realities they help to construct and maintain. 112

Page 129 of 1481


Ricoeur Turn: 1AR

AND, EXTEND 2AC # ___, THE RICOUER TURN.

SUSPCION OF HIDDEN MOTIVATIONS BEHIND POLICYMAKING FORCES INFINITE


SKEPTICISM BECAUSE EVERY OUTCOME IS DETERMINATELY NEGATIVE. THE
IMPACT IS THE OTHER RICOUER CARD, WHICH SHOWS THAT SUCH PARANOIA
PREVENTS SOCIAL CHANGE, ALLOWING NIHILISM TO REPLACE REVOLUTIONARY
TRANSFORMATION

ALSO, THEIR HERMENEUTICS WORK AGAINST SOCIAL CHANGE AND KILL SOCIAL
MOVEMENTS

Berman 2001
[Paul Schiff, Assoc. Prof. Law @ U. of Connecticut, Yale Journal of Law and the Humanities, LN]

The second drawback of the hermeneutics of suspicion is perhaps even more important. As some scholars have noted, the
hermeneutics of suspicion can easily slip from healthy skepticism into a kind of rhetorical
paranoia. Paranoia, of course, is a loaded term, and probably a bit unfair. Nevertheless, because it is used frequently in
the academic literature about the hermeneutics of suspicion, I will use it as well - though I want to make clear that I believe
paranoia to be the hypothetical extreme in the movement toward skeptical scholarship. I do not mean to imply that any
actual scholars necessarily display such paranoid logic.

Critics of the hermeneutics of suspicion describe the "paranoid style of functioning" 104 as "an intense, sharply perceptive
but
narrowly focused mode of attention" that results in an attitude of "elaborate suspiciousness." 105 Paranoid individuals
constantly strive to demystify appearances; they take nothing at face value because "they regard reality as an obscure
dimension hidden from casual observation or participation." 106 On this vision,

The obvious is regarded as misleading and as something to be seen through. So, the paranoid style sees the world as
constructed of a web of hints to hidden meaning... . The way in which the paranoid protects fragile autonomy is by insuring,
or at least insisting, that the paranoid's interpretation of events is the interpretation. 107

Such a paranoid style may, over time, have a potentially corrosive effect on society. 108 Consider the long-term
consequences of repeated exposure to suspicious stories. An appeal to religious ideals is portrayed as an exercise of
political power or the result of deluded magical thinking. A [*122] canonical work of art is revealed to be the product of a
patriarchal "gaze." The programs of politicians are exposed as crass maneuverings for higher office or greater power. 109
The idealistic rhetoric of judicial opinions is depicted as an after-the-fact justification for the exercise of state-sanctioned
violence. And the life choices of individuals are shown to be responses to psychological neurosis, or social pathology.

All of these are exaggerations, but they increasingly represent the rhetoric that is used to describe human interaction both
in contemporary society and in the past. As Richard Rorty describes,

In this vision, the two-hundred-year history of the United States - indeed, the history of the European and American
peoples since the Enlightenment - has been pervaded by hypocrisy and self-deception. Readers
of Foucault often
come away believing that no shackles have been broken in the past two hundred years:
the harsh old chains have merely been replaced with slightly more comfortable ones. Heidegger describes America's
success in blanketing the world with modern technology as the spread of a wasteland . Those
who find Foucault
and Heidegger convincing often view the United States of America as ... something we
must hope will be replaced, as soon as possible, by something utterly different. 110

Page 130 of 1481


If that is one's viewpoint, it will inevitably be difficult to muster one's energy to believe in the
possibility of positive action in the world, short of revolution (and even revolution is
probably inevitably compromised). As Rorty points out, though the writers of supposedly "subversive" works
"honestly believe that they are serving human liberty," it may ultimately be "almost impossible to
clamber back down from [these works] to a level of abstraction on which one might
discuss the merits of a law, a treaty, a candidate, or a political strategy." 111

Page 131 of 1481


Page 132 of 1481
Ricoeur Turn: Ext
LAW CAN BE VIEWED AFFIRMATIVELY THE MULTIPLICITY OF STORES CAN PROVE
MORE HOPE FOR CHANGE AND MEANING NOT LESS

Berman 2001
[Paul Schiff, Assoc. Prof. of Law @ Connecticut, Yale Journal of Law and Humanities, LN//uwyo]

Recently, Richard K. Sherwin's When Law Goes Pop: The Vanishing Line Between Law and Popular Culture n127 has attempted a similar project. Sherwin argues (as I have

skeptical postmodernism
earlier in this Essay) against what he calls "skeptical postmodernism." Referring to Baudrillard, Sherwin observes that

"manifests a marked inclination toward pessimism and disenchantment." n128 If truth, meaning, and reality
are no longer discernible, and if any sense of the unified self or human agency is illusory, he argues, we risk
living in a world where "individuals can no longer be held accountable for having "authored' their acts or
caused an event to happen." n129 According to Sherwin, "In the end the skeptical postmodern is left with nothing more
than endless play and detached irony." n130

Postmodernism need not be skeptical... . A


Nevertheless, like me, Sherwin refuses to jettison postmodern theory altogether. Instead, he contends, "

story might concede the demise of the autonomous modern subject, but still find meaning through the
distributed self: an identity made up of multiple cultural and social constructs shared by others in
particular communities." n131 Similarly, taking Sherwin's [*129] "affirmative postmodern" view, we might recognize that concepts
such as truth and justice are contingent, but still see those ideas as coherent. "Abstraction may give way
to particularity, contextuality, multiplicity; judgment may turn toward characteristic voices and localized
accounts. But localization and contextualization are not fatal to meaning. It remains possible to seek
rather than abandon meaning for concepts like truth and justice - even in the face of contingency,
unpredictability, and spontaneity." n132
Following Sherwin's suggestion, I wish to pursue a story about law that makes no attempt to return to a formalist world where legal rules are "truths" to be "discovered" by
judges. Rather, I accept the idea that there is an infinite number of possible narratives for describing reality and that each narrative is inevitably a product of many cultural
forces. Further, I will accept that, at least within a certain range, none of these narratives necessarily has a stronger claim to truth than any other. In such a world, how might
one understand and justify law practice in America? n133

we might conceive of law as a site for encounter, contestation, and play among various
My suggestion is that

narratives. I draw on Hannah Arendt's conception of the "public" as a space of appearance where actors stand before others and are subject to mutual scrutiny and
judgment from a plurality of perspectives. n134 The public, on this view, "consists of multiple histories and perspectives relatively unfamiliar to one another, connected yet

By communicating about their differing perspectives on the social world in


distant and irreducible to one another." n135

which they dwell together, people and communities can collectively constitute an enlarged understanding
of the world. n136 In this Part, therefore, I will first outline a prominent conception of "communicative democracy" that builds on Arendt, offered by political theorist Iris
M. Young. Then, I will speculate about law's potential as a site for the type of idealized public discourse Young envisions. n137

Page 133 of 1481


Romanticization Turn: 2AC

TURN: APPROPRIATING THE OTHER VIOLENTLY SEIZES THE RIGHT TO SPEAK FOR
SELFISH ENDS

Routledge 96
[Paul, The Third Space as Critical Engagement, Antipode 28(4), October, 399//uwyo]

The issue of representation is a vexed one which has received much attention within the social
sciences. For example, in discussing the academic strategy of polyphony, Crang (1992) raises
issues of how the voices of others are (re)presented; the extent to which these voices are
interwoven with persona of narrator the degree of authorial power regarding who initiates
research, who decides on textual arrangements, and who decides which voices are heard; and
the power relations involved in the cultural capital conferred by specialist knowledge.
Moreover, Harrison (quoted in McLaren 1995 240) argues that polyphony can end up being
aform of romantic ventroloquism creating the magical notion of the Others coming to voice.
These questions have important political implications for research which must be negotiated
according to the specific circumstances of a particular project. It is all too easy for academics to
claim solidarity with the oppressed and act as relays for their voices within social scientific
discourse. This raises the danger of an uncritical alignment with resisters on the assumption that
they know all there is to know without the intervention of intellectuals; and hence an
academics role becomes that of helping them seize the right to speak.

Page 134 of 1481


Romanticization Turn: 1AR

AND, EXTEND THE 2AC #___ ROUTLEDGE 96 ROMANTICIZATION TURN. SPEAKING


ON BEHALF OF OTHERS USES THEIR SUFFERING FOR ONES OWN ENDS, SILENCING
THEM BY SEIZING THE RIGHT TO SPEAK, REINSCRIBING THE IMPACT

Page 135 of 1481


Page 136 of 1481
Romanticization Turn: 2AR
NEXT, EXTEND THE 2AC #__, THE ROUTLEDGE ROMANTICIZATION ARGUMENT.

OUR CLAIM IS THAT EFFORTS TO OPEN SPACE FOR THE OTHER WITHIN A
COMPETITIVE FRAMEWORK ARE PROBLEMATIC BECAUSE

THE WAY THAT THE VOICE IS PRESENTED IS NOT ONLY DETERMINED BY THE NEG,
BUT ITS RE-PRESENTATION LEGITIMIZES THE AUTHORIAL POWER OF ACADEMICS
TO SPEAK FOR OTHERS
THAT POWER USES THE GUISE OF POLYPHONY TO PROMULGATE A FORM OF
ROMANTIC VENTRILOQUISM THAT MASKS THE OPPRESSIVE NATURE OF THEIR
RESEARCH

THIS IS DEVASTATING TO THE NEG ON 2 LEVELS

FIRST, ITS AN ABSOLUTE TAKEOUT TO ANY POSITIVE IMLICATIONS OF THE


CRITICISM BECAUSE THE NEGS ALLEGED SPACE-CLEARING CAN NEVER LET OTHERS
SPEAK

SECOND, IT TURNS THE IMPLICATIONS BECAUSE THEIR PERFORMANCE ONY


FURTHER COMMODIFIES THE USE OF THE PAIN OF OTHERS FOR PERSONAL GAIN,
PLACING A WARM, FUZZY LEG WARMER OVER THE JACKBOOT OF DOMINATION

Page 137 of 1481


Page 138 of 1481
Said Turn: 2AC
THE ALTERNATIVE OPTS FOR INACTION IN THE FACE OF DOMINATION ONLY
POLICY DISCUSSIONS CAN REORIENT INTELLECTUALS TOWARDS FIGHTING
INJUSTICE

SAID (University Professor, Columbia University) 94


[Edward W., The Intellectuals and the War, The Politics of Dispossession: The Struggle

for Palestinian Self-Determination, 1969-1994, New York: Vintage, p. 316-19]

HARLOW: What are the political, intellectual, and cultural imperatives for combating this agenda? In 1967 Chomsky wrote the essay Responsibility of Intellectuals. What
would be the main component of such an essay today?

jargonistic postmodernisms that now dot the landscape. They are worse than useless.
SAID: One would have to pretty much scuttle all the jaw-shattering

are neither capable of understanding and analyzing the power structure of this country nor are they
They

capable of understanding the particular aesthetic merit of an individual work of art. Whether you call it deconstruction or
postmodernism or poststructuralism or post-anything, they all represent a sort of spectacle of giving back tickets that the entrance and saying, were really out of it. We want to
check into our private resort and be left alone. [317]

Reengagement with intellectual processes has very little to do with being politically correct, or citing
fashionable names, or striking acceptable poses, but rather having to do with a return in a way to a kind of old-
fashioned historical, literary, and above all, intellectual scholarship based upon the premise that human beings, men and
women, make their own history. And just as things are made, they can be unmade and re-re-remade. That sense of intellectual and political and citizenry
empowerment is what I think the intellectual class needs.

Theres only one way to anchor oneself, and that is by affiliation with a cause, with a political movement. There has to be some identification, not with the powers that

; there has to be an affiliation with matters involving


be, with the Secretary of State or the great leading philosopher of the time or sage

justice, principle, truth, conviction. Those dont occur in a laboratory or a library. For the American intellectual, that simply means, at bottom, in a
globalized environment, that there is today one superpower, and the relationship between the United States and

the rest of the world, based upon profit and power, has to be altered from an imperial one to one of
coexistence among human communities that can make and remake their own histories together. This
seems to me to be the number-one priority---theres nothing else.
An American has a particular role. If youre an anthropologist in America, its not the same thing as being an anthropologist in India or France; its a qualitatively
different thing.

HARLOW: Were both professors in English departments, despite the fact that the humanities have been quite irresponsible, unanswerable

SAID: Not the humanities. The professors of humanities.

HARLOW: Well, OK, the professors, but there is this question

SAID: I take the general view that, for all its inequity, for all its glaring faults and follies, the university in this society remains a relatively utopian place, a place of great privilege .
There needs to be some sense of the university as a place in which these issues are not, because it is that
kind of place, trivialized. Universities cannot afford to become just a platform for a certain kind of
narcissistic specialization and jargon. What you need is a regard for the product of the human mind. And
thats why Ive been very dispirited, I must tell you, but aspects of the great Western canon debate, which really suggest that the oppressed of the world, in wishing to be
heard, in wishing their work to be recognized, really wish to do dirt on everything else. Thats not the spirit of resistance. We come [318] back to Aime Cesaires line, There is
room for all that at the rendezvous of victory. Its not that some have to be pushed off and demeaned and denigrated. The question is not whether we should read more black
literature or less literature by white men. The issue is excellence---we need everything, as much as possible, for understanding the human adventure in its fullest, without
resorting to enormous abstractions and generalizations, without replacing Euro-centrism with other varieties of ethnocentrism, or say, Islamo-centrism or Afro-centrism or gyno-
centrism. Is it a game of substitutions? Thats where intellectuals have to clarify themselves.

HARLOW: I agree, but at least within certain university contexts there have been lately two major issues: the Gulf War and multiculturalism. I have not seen any linkage
between the two.

SAID: The epistemology and the ethic of specialization have been accepted by all. If youre a literature professor, thats what you talk about. And if youre an education
specialist, thats what you talk about. The whole idea of being in the university means not only respect for what others do, but respect for what you do. And the sense that they
all are part of a community. The main point is that we ascribe a utopian function to the intellectual. Even inside the university, the prevalence of norms based upon domination
and coercion is so strong because the idea of authority is so strong---whether its authority derived from the nation-state, from religion, from the ethnos, from tradition---is so
powerful that its gone relatively unchallenged, even in the very disciplines and studies that we are engaged in. Part of intellectual work is understanding how authority is

formed. Like everything else, authority is not God-given. Its secular. And if you can understand that, they your work is conducted in

Page 139 of 1481


such a way as to be able to provide alternatives to authoritative and coercive norms that dominate so
much of our intellectual life, our national and political life, and our international life above all.
HARLOW: What can alternative publications do to interrupt that particular way of presenting authority?

SAID: One is to remind readers that there are always other ways of looking at the issue---whatever it happens to be---than those that are officially credentialed. Second, one of
the things that one needs to do in intellectual enterprises is to---Whitehead says somewhere---always try to write about an author keeping in mind what he or she might say of
what youre writing. To adapt from that: some sense in which your constituency might be getting signals about what youre doing. The agenda isnt set only by you; its set by
others. You cant represent the others, but you can take them into account by soliciting their attention. Let such a publication be a place in which its pages that which is
occluded or suppressed or has disappeared from the consciousness of the West, of the intellectual, can be allowed to appear. Third, some awareness of the methodological
issues involved, and the gathering of information, the production of scholarship, the relationship between scholarship and knowledge. The great virtue of these journals is that
they are not guided by professional norms. Nobody is going to get tenure out of writing for these journals. And nobody is trying to advance in a career by what he or she does
there. So that means therefore that one can stand back and look at these things and take questions having to do with how people know things. In other words, a certain
emphasis on novelty is important and somewhat lacking. You dont want to feel too virtuous in what you are doing: that Im the only person doing this, therefore, I must continue
doing it. Wit is not such a bad thing.

Page 140 of 1481


Academic Work Spurs Activism: Ext (1/2)

INTELLECTUAL WORK SERVES AS A CRITICAL RESOURCE FOR ACTIVISTS

Milan Rai, independent peace researcher, CHOMSKYS POLITICS, 1995, p. 156.

Chomsky suggests that the intellecutal can make an important contribution to the struggle for peace and
justice by agreeing to serve as a resource, providing information and analysis to popular movements.
Intellectuals have the training, facilities, access to information and opportunity to organize and control
their own work, to enable them to make a very significant contribution to people who are trying to
escape the confines of indoctrination and to understand something about the real world in which they
live; in particular, to people who may be willing to act to change this world. For the same reasons,
intellectuals can be active and effective organizers. Furthermore, by virtue of their privilege, intellectuals
are also often visible and can exploit their privilege in valuable and important ways.

WORLDY ACADEMIC WORK IS DEMOCRATIZING AND SPURS ACTIVISM

Gordon R. Mitchell, Assistant Professor of Communication, University of Pittsburgh, ARGUMENTATION


AND ADVOCACY, Fall 1998, p. 47.

In basic terms the notion of argumentative agency involves the capacity to contextualize and employ the
skills and strategies of argumentative discourse in fields of social action, especially wider spheres of public
deliberation. Pursuit of argumentative agency charges academic work with democratic energy by linking
teachers and students with civic organizations, social movements, citizens and other actors engaged in
live public controversies beyond the schoolyard walls. As a bridging concept, argumentative agency links
decontextualized argumentation skills such as research, listening, analysis, refutation and presentation, to
the broader political telos of democratic empowerment. Argumentative agency fills gaps left in purely
simulation-based models of argumentation by focusing pedagogical energies on strategies for utilizing
argumentation as a driver of progressive social change. Moving beyond an exclusively skill-oriented
curriculum, teachers and students pursuing argumentative agency seek to put argumentative tools to the
test by employing them in situations beyond the space of the classroom. This approach draws from the
work of Kincheloe (1991), who suggests that through "critical constructivist action research," students and
teachers cultivate their own senses of agency and work to transform the world around them.

ACADEMICS FOSTER ACTIVISM BY LEGITIMATING DISSENT

Suzie Mackenzie, columnist, THE GUARDIAN, January 4, 2003, p. 20.

Page 141 of 1481


What does the intellectual have to offer that isn't already out there? "Dissent," Rose says. "It is the task of
the intellectual to think thoughts, to say things, that can't be said anywhere else. What I think goes most
frighteningly and disturbingly wrong in politics is that people hold intransigently to their ideals. They
admit no flaw, no break in (their own) system." You can't argue with this, it's what any good liberal
intellectual would say.

Page 142 of 1481


Page 143 of 1481
Academic Work Spurs Activism:
Ext (2/2)
WORLDLY ENGAGEMENT FOSTERS ACTIVISM WITHIN THE ACADEMY

Gordon R. Mitchell, Assistant Professor of Communication, University of Pittsburgh, ARGUMENTATION


AND ADVOCACY, Fall 1998, p. 47.

Encounters with broader public spheres beyond the realm of the academy can deliver unique pedagogical
possibilities and opportunities. By anchoring their work in public spaces, students and teachers can use
their talents to change the trajectory of events, while events are still unfolding. These experiences have
the potential to trigger significant shifts in political awareness on the part of participants. Academic
debaters nourished on an exclusive diet of competitive contest round experience often come to see
politics like a picturesque landscape whirring by through the window of a speeding train. They study this
political landscape in great detail, rarely (if ever) entertaining the idea of stopping the train and exiting to
alter the course of unfolding events. The resulting spectator mentality deflects attention away from roads
that could carry their arguments to wider spheres of public argumentation. However, on the occasions
when students and teachers set aside this spectator mentality by directly engaging broader public
audiences, key aspects of the political landscape change, because the point of reference for experiencing
the landscape shifts fundamentally.

Page 144 of 1481


Academics as Politics is Bad (1/2)

ACADEMICS AS POLITICS IS INEFFECTIVE AND CORRUPTS THE LEARNING PROCESS

Kwame Anthony Appiah, Professor, Princeton University, interviewed by Jenny Attiyeh, CHRISTIAN SCIENCE
MONITOR, August 22, 2002, p. 12.

No, not as an intellectual, because your responsibility as an intellectual is to deepen your understanding
and therefore our understanding.... I think our university life would be corrupted irremediably if you said
to everybody in the university, beyond understanding, you have an obligation to go out and change those
parts of the world that your understanding can help change. I don't think we're especially good at it -
practical wisdom doesn't come with theoretical understanding usually. Do we think Einstein would have
made a better leader for Britain ... than Winston Churchill? I don't think so!

ACADEMIC POLITICIZATION UNDERMINES UNIVERSITIES QUEST FOR KNOWLEDGE

Bradford P. Wilson, Executive Director of the National Assocation of Scholars and former Professor of
Political Science, Ashland University, NATIONAL FORUM PHI KAPPA PHI JOURNAL, Winter 1999, p. 18.

The culture wars in higher education are not between a political left and a political right, or between
liberals and conservatives. They are between those who wish to politicize academic life as part of a larger
agenda of social transformation, and those who see in the university the only institution in American life
where knowledge is valued for its own sake, where students can be forgiven a temporary lack of social
concern and engagement for the sake of remedying a more fundamental deprivation, their lack of self-
knowledge. The cure, insofar as there is one, is to be found in a liberal education, not in an identity-fix
offered by the latest multicultural initiative.

POLITICS AND ACADEMICS HAVE FUNDAMENTALLY CONTRADICTORY GOALS

Kwame Anthony Appiah, Professor, Princeton University, interviewed by Jenny Attiyeh, CHRISTIAN SCIENCE
MONITOR, August 22, 2002, p. 12.

The fundamental vocation of the intellectual is to figure things out, you know, intellego, to understand.
And politics isn't about understanding, politics is about getting things done. Understanding can be an
instrument of getting things done, but nuance and complexity of understanding can be an obstacle to
getting things done. Politics - it's the art of the possible, and sometimes in order to do the best that can
be done, you have to ride roughshod over what are, for an intellectual, important distinctions - for
example, between the truth and the untruth.

Page 145 of 1481


Page 146 of 1481
Academics as Politics is Bad
(2/2)
DEMANDS OF POLITICAL RELEVANCE DESTROY THE VERY FOUNDATION OF THE
ACADEMY

Wendy Brown, Professor of Womens Studies, University of California-Santa Cruz, THEORY AND EVENT 2:2,
1998, p. npg.

I think it is a terrible mistake to conflate or identify academic and political work. To see Left academics as
necessarily confining their intellectual endeavors, their theorizing, the texts they love, their reflections, to
that which is politically useful in an immediate way, is, I think, a serious error. It is a mistake just as it
would be a mistake to claim that Alan Sokal is no Leftist because he is a physicist and is poorly versed in
social theory, and I would never make such a silly claim. But I think it is equally silly to suggest that
everything any of us ever write or say must have immediate political cache. What we do in the academy is
think, and to constrain that thinking entirely to what is understandable and useful outside the academy is
basically to eliminate the point of the academy's existence. It is to constrain the space of imagination,
open-ended search, and inquiring into our own knowledge and beliefs, all of which are the life-blood of
intellectual work. For me, to stop calling into question that which I believed yesterday, to stop examining
ideas I have always been attached to, would literally be to stop thinking. It would be to go into a kind of
political automatic, as opposed to using the great privilege of being an intellectual, to keep digging up the
political ground we stand on. It would also be to constrain the space of original critique that has always
been so vital to Left projects

Page 147 of 1481


Page 148 of 1481
Criticism Destroys Agency
ACADEMIC CRITICISM BECOMES A REPLACEMENT FOR INDIVIDUAL ACTION. THE
CRITIC BECOMES SO COMMITTED TO REJECTION OF POWER STRUCTURES THAT
THEY FAIL TO CREATE A MIDDLE GROUND NECESSARY FOR CHANGE

Barber 92
Benjamin (prof o political science at Rutgers), An Aristocracy for Everyone, pg. 111-112

The questions this poses for pedagogy are drawn in the recondite language of literary
postmodernism and deconstruction, but are of the first importance for education. Does the art of
criticism doom the object of critical attention to displacement by the self-absorbed critic? In other
words, does criticizing books replace reading them? Can the art of questioning be made self -
limiting, or do critics always become skeptics? Are skeptics in turn doomed by their negative logic
to be relativists? Must relativists melt down into nihilists? Conservatives have worried that this
particularly slippery slope cannot be safely traversed at all, and thus have worried about a pedagogy
that relies on a too critical mode of radical questioning. They prefer to think of education as instilling
the right values and teaching authoritative bodies of knowledge to compliant students for whom
learning is primarily a matter of absorbing information. When these conservatives appeal to the
ancients, it is the rationalist Plato to whom they turn, rather than the subversive Socrates. Yet
pedagogical progressives actually confirm the conservatives' fears when they themselves tumble happily
down the slope, greasing it as they go with an epistemology that denies the possibility of any stopping
place, any objectivity, any rationality, any criterion of reasonableness or universalism whatsoever. Asked to
choose between dogma and nihilism, between affirming hegemonic authority and denying all authority,
including the authority of reason, of science, and of open debate, what choice does the concerned teacher
have but despair? Where she seeks a middling position, she is offered orthodoxy or nihilism. Where she
seeks moderation in her students-a respect for rationality but an unwillingness to confound it with or
measure it by somebody's power, or eloquence, or status-she is informed that all appeals to rationality
are pretense: Bertrand Russell's no less than Joseph Goebbels's, Hannah Arendt's no less than Catherine
the Great's, the rationality with which the skeptic skewers conventional reason no less than the
rationality the skeptic skewers.

Page 149 of 1481


Criticism is Nihilistic (1/4)

DECONSTRUCTION WITHOUT ACTION FOR MATERIAL JUSTICE BLOCKS POLITICAL


ESCAPE FROM OPPRESSION AND REINFORCES IVORY TOWER ELITISM

Anthony Cook, Associate Professor, Law, Georgetown University, NEW ENGLAND LAW REVIEW, Spring
1992, p. 761-762.

The effect of deconstructing the power of the author to impose a fixed meaning on the text or offer a
continuous narrative is both debilitating and liberating. It is debilitating in that any attempt to say what
should be done within even our insular Foucaultian preoccupations may be oppositionalized and
deconstructed as an illegitimate privileging of one term, value, perspective or narrative over another. The
struggle over meaning might continue ad infinitum. That is, if a deconstructionist is theoretically
consistent and sees deconstruction not as a political tool but as a philosophical orientation, political
action is impossible, because such action requires a degree of closure that deconstruction, as a theoretical
matter, does not permit. Moreover, the approach is debilitating because deconstruction without material
rootedness, without goals and vision, creates a political and spiritual void into which the socially real
power we theoretically deconstruct steps and steps on the disempowered and dispossessed. [*762] To
those dying from AIDS, stifled by poverty, dehumanized by sexism and racism, crippled by drugs and
brutalized by the many forms of physical, political and economic violence that characterizes our
narcissistic culture, power hardly seems a matter of illegitimate theoretical privileging. When vision, social
theory and political struggle do not accompany critique, the void will be filled by the rich, the powerful
and the charismatic, those who influence us through their eloquence, prestige, wealth and power.

Page 150 of 1481


Page 151 of 1481
Criticism is Nihilistic (2/4)
CRITICISM IS A SLIPPERY SLOPE THAT WILL EVENTUALLY LEAD TO THE REJECTION
OF EVERYTHINGWHAT BEGINS AS AN UNWILLINGNESS TO ACCEPT ON-FACE
OBJECTIVE KNOWLEDGE ENDS WITH A COMPLETE REJECTION OF ANY ATTEMPT TO
OBTAIN KNOWLEDGE (EXTINCTION)

Barber 92
Benjamin (prof o political science at Rutgers), An Aristocracy for Everyone, pg. 116-118

This cursory history of esoteric arguments about the nature of knowledge may seem far removed from
the educational controversies of our time. It is offered only as a reminder that such fashionable new
forms of radical criticism as deconstruction are but echoes of a very ancient skepticism and a very
well entrenched tradition of reductionism. It is for this reason that Allan Bloom pins the blame for
the changes in modern education on Heidegger, Nietzsche, Marx, and other maverick critics of reason
and reason's canon (see Chapter 5). It is for this same reason that conservatives who esteem the role
reason plays in grounding and justifying fundamental values view post-modern skepticism with alarm,
and that liberals who care about reform worry that reductive strategies are ill-suited to their purposes.
As Edmund Burke once noted, those who destroy everything are certain to remedy some grievance.
The annihilation of all values will undoubtedly rid us of hypocritical ones or the ones misused by
hypocrites. We can prevent the powerful from using reason to conceal their hegemony by burning the
cloak-extirpating reason from political and moral discourse. However, those who come after
can hardly complain that they feel naked or that their discourse, absent such terms as reason,
legitimacy, and justice, seems incapable of establishing an affirmative pedagogy or a just politics.

Just how crucially such seemingly abstruse issues impact on actual college curricula is unpleasantly
evident in this approving portrait of literature and culture in a recent issue of the Bulletin of the
American Association of University Professors:

Cultural studies moves away from "history of ideas" to a contested history of struggles for power
and authority, to complicated relations between "center" and "margin," between dominant and
minority positions. Literature is no longer investigated primarily as the masterworks of individual genius,
but as a way of designating specialized practices of reading and writing and cultural produc tion.... The
renaming of "literature" as "culture" is thus not j ust a shift in vocabulary. It marks a
rethinking of what is experienced as cultural materials ...[including] media, MTV, popular culture,
newspapers, magazines, advertising, textbooks, and advice materials. But the shift also marks the
movement away from the study of an "object" to the study of a practice, the practice called
"literary study" or "artistic production," the practice of criticism.'

How slippery this particular slope has become! What begins as a sound attempt to show that art is
produced by real men and women with agendas and interests attached to things like their gender,
race, and economic status ends as the nihilistic denial of art as object. What begins as a pedagogically
useful questioning of the power implications of truth ends as the cynical subverting of the very
possibility of truth. What begins as a prudent unwillingness to accept at face value "objective"
knowledge, which is understood to be, at least in part, socially constructed, ends as the absurd
insistence that knowledge is exclusively social and can be reduced entirely to the power of those who
produce it. What begins as an educationally provocative inquiry into the origins of literature in the
practice of literary production ends in the educationally insidious annihilation of literature and its
replacement by criticism-the practice, it turns out ever so conveniently, of those asking the questions!

Page 152 of 1481


Thus does the whirling blade of skepticism's latest reductive manifestations, post-modernism and de-
construction, cut and cut and go on cutting until there is nothing left. Thus does the amiable and
pedagogically essential art of criticism somehow pass into carnage.

Page 153 of 1481


Page 154 of 1481
Criticism is Nihilistic (3/4)
USING THE ACADEMIC AREA FOR CRITICIZING EXISTENT SYSTEMS RISKS HYPER
SKEPTICAL DISCUSSIONS. THE SELECTION OF THE MEDIAN FORCES RADICALISM
AND NIHILISM

Barber 92
Benjamin (prof o political science at Rutgers), An Aristocracy for Everyone

All thoughtful inquiry, and hence all useful education, starts with questioning. All usable knowledge, and
thus all practical science, starts with the provisional acceptance of answers. Education is a dialectic in
moderation in which probing and accepting, questioning and answering, must achieve a delicate
balance. Stories must be told, queried, retold, revised, questioned, and retold still again much as the
American story has been. In periods of rebellion, academic no less than social, when challenging
authority means questioning answers, there is an understandable tendency toward skepticism, even
cynicism. Michael Wood has characterized Jacques Derrida's approach to method as "a patient and
intelligent suspicion,' 3 which is a useful description of one moment in a student's democratic
education.

The methodologies deployed by critics of power and convention in the academy do not always find
the dialectical center, however, and are subject to distortion by hyperbole. Sometimes they seem
to call for all questions and no answers, all doubt and no provisional resting places. This radicalism
has many virtues as scholarship, but as pedagogy far fewer. In its postmodern phase, where the merely
modern is equated with something vaguely reactionary and post-modernism means a radical battering
down of all certainty, this hyperskeptical pedagogy can become self -defeating.

Skepticism is an essential but slippery and thus dangerously problematic teaching tool. It demystifies and
decodes; it denies absolutes; it cuts through rationalization and hypocrisy. Yet it is a whirling blade, an
obdurate reaper hard to switch off at will. It is not particularly discriminating. It doesn't necessarily under-
stand the difference between rationalization and reason, since its effectiveness depends precisely on
conflating them. It can lead to a refusal to judge or to take responsibility or to impose norms on conduct.
If, as Derrida has insisted, "the concept of making a charge itself belongs to the structure of
phallogocentrism" (the use of reason and language as forms of macho domination), there can be no
responsibility, no autonomy, no morals, no freedom. 4 Like a born killer who may be a hero in
wartime but, unable to discriminate between war and peace, becomes a homocidal maniac when the
war ends for everyone else, radical skepticism lacks a sense of time and place, a sense of elementary
propriety.

Page 155 of 1481


Page 156 of 1481
Criticism is Nihilistic (4/4)
THEIR PROJECT IS BANKRUPT. CONFRONTING POWER RELATIONSHIPS THROUGH
CYNICISM, SKEPTICISM, AND REJECTION WILL NOT CREATE PRAGMATIC
SOLUTIONS

Barber 92
Benjamin (prof o political science at Rutgers), An Aristocracy for Everyone, pg. 122-123

There can be no simple answer to such complex psycho -political questions, and I certainly do not
mean to challenge philosophical reductionism by psychoanalyzing philosophers and thereby replacing
one reductive logic with another. Nonetheless, as already suggested, Thrasymachus understood the
connection between his brand of reductive questioning and brute power perfectly well: his was the
cynicism of the power realist who wanted to convince Socrates' audience that power was all there
was. He wished not to legitimize and thus limit power, but to enthrone and sacralize it. This is clearly
not the goal of the far more naive advocates of the new hyperskepticism. They are genuine reformers
struggling against the dogmas of what they see as a hypocritical establishment. They seek more
equality, more justice, better education for all. They want not just to expose the hypocrisies of power,
but to tame and equalize it. They want to reclaim true justice from its hypocritical abusers. They chase
shadows in the valley of cynicism but trust they are on the path that leads to redemption.

Yet the instruments of revolution they have chosen are more suited to the philosophical terrorist than
the pedagogical reformer. Radical skepticism, reductionism, solipsism, nihilism, subjectivism, and
cynicism will not help American women gain a stronger voice in the classroom; will not lift Americans of
color from the prison of ignorance and despair to which centuries of oppression, broken families, and
ghettoized schools have relegated them; will not provide a firm value foundation for the
young in equality, citizenship, and justice. How can such reform -ers think they will empower the
voiceless by proving that voice is always a function of power? How can they believe the ignorant will
be rescued from illiteracy by showing that literacy is an arbitrary form of cultural imperialism? How do
they think the struggle for equality and justice can be waged with an epistemology that denies standing
to reasons and normative rational terms such as justice and equality?

Page 157 of 1481


**Postmodernism Bad**
Floating Subjectivity Bad (1/3)

POSTMODERN SUBJECTIVITY IS A SHELL GAME IT CAN EXIST ONLY BY


STRENGTHENING THE HOLD OF CAPITALISM

Laura Bartlett Snyder, Doctoral Fellow in the English Department at Louisville, Boundary Dissolution in
film, photography & advertising, 2000, http://athena.louisville.edu/a-
s/english/babo/snyder/bountexts.html, accessed 10/15/02

The argument I am making about the postmodern theories of subjectivity and global capitalism are similar
to arguments made about multiculturalism and global capitalism by David Rieff and Slavoj Zizek. Rieff
suggests that multiculturalism is a byproduct or corollary of a specific material integument (62). Rieffs
position is that although multiculturalists often regard their work as politically leftist: resulting in the
breakdown of patriarchal, European hegemony and the ascendancy of the previously marginalized, they
actually function as the silent partner of global capitalism. Additionally, Rieff points out how closely the
buzz words of multiculturalism--cultural diversity, difference, the need to do away with boundaries
resemble the stock phrases of the modern corporation: product diversification, the global marketplace,
and the boundary-less company (Rieff). Similarly, Zizek contends that postmodern identity politics
while ostensibly seeking to subvert capitalismare made possible only in the field of global capitalism.
He writes that cultural studies, is performing the ultimate service for the unrestrained development of
capitalism and that the ideal form of ideology of this global capitalism is multiculturalism (218; 216).My
argument is that postmodern theories and global capitalism dialectically influence one another.
Postmodern theory is generated by the material conditions of labor and production in late capitalism,
which needs consumers who will disregard national boundaries. By the logic that all products of the
system are necessary to the system, we assume that anything the system produces, it needs. Ideological
state apparatuses, like the university, do the work necessary to interpellating the ideal subject of global
capitalism. My thought is that global capitalism needs postmodern theories of subjectivity because they
produce subjects who are seamlessly articulated with the structures of global capitalism. While
postmodern subjectivity may seem wildly radical at firstbreaking down boundaries between genders,
between machines and humansthe similarities between its subjectivities and the structures of global
capitalism are eerily similar. Fluidity, flexibility, and boundary dissolution equally describe both. The
celebration of the loss of the unified, coherent subject of modernity and the new fluid, flexible,
fragmented subject of postmodernity is the stuff of Millenial Dreams, Paul Smiths term for the rhetoric
of globalization and the array of ideological forms which interpellate the desired subject of global
capitalism. Smith writes that the annunciation of globalization itself is part of the ideological battery
used to interpellate subjects in the current conjuncture . . . and attempt to regulate the moral and cultural
practices of subjects (46). I agree with Tereas Ebert that post-al theories are complicit with patriarchal
capitalism. Rather than seeking the liberation of the exploited workers of late capitalismprimarily third-
world, minority, poverty-stricken womenpostmodern theorists celebrate a liberatory freedom
experienced by a small percentage of the first world at the expense of the rest of the world.

Page 158 of 1481


Page 159 of 1481
Floating Subjectivity Bad (2/3)
FLOATING SUBJECTIVITY AND REBELLION AGAINST MODERNITY REINFORCES
PATTERNS OF DOMINATION

Kevin Cryderman, Jane and Louisa: The Tapestry Of Critical Paradigms: Hutcheon, Lyotard, Said, Dirlik,
And Brodber, 2000, http://65.107.211.206/post/caribbean/brodber/kcry1.html, accessed 11/7/01

In "Borderlands Radicalism," Dirlik is critical of the trends of postmodernism and postcolonialism in regard
to borders, subjectivity, and history. Dirlik claims that postmodernism and postcolonialism tend to simply
reinforce the reign of late capitalism: Post-modernism, articulating the condition of the globe in the age of
flexible production, has done great theoretical service by challenging the tyrannical unilinearity of
inherited conceptions of history and society. The political price paid for this achievement, however, has
been to abolish the subject in history, which destroys the possibility of political action, or to attach action
to one of another diffuse subject positions, which ends up in narcissistic preoccupations with self of one
kind of another. (89) Dirlik claims that the 'happy pluralism' of postcolonialism -- such as its emphasis on
flux, borderlands and liminal space -- does not so much oppose elite unified narratives of nations and
cultures as it does reinforce them. Dirlik also links this trend of "fluid subject positions" (98) in
postmodernism to postcolonialism and Global Capitalism: "in the age of flexible production, we all live in
the borderlands. Capital, deterritorialized and decentered, establishes borderlands where it can move
freely, away from the control of states and societies but in collusion with states against societies" (Dirlik
87). Moreover, the problem "presented by postcolonial discourse" is "a problem of liberating discourse
that divorces itself from the material conditions of life, in this case Global Capitalism as the foundational
principle of contemporary society globally" (99). Dirlik also links the intellectual class as a product of
global capitalism which, according to Dirlik, "has jumbled up notions of space and time" (100). Indeed,
both postmodernist and post-colonialist literature involve the fragmentation and rebellion against
modernist ideologies that impose essentializing identity, linear time schemes, and totalizing narratives.

FLOATING SUBJECTIVITY FACILITATES THE HEGEMONY OF TRANSNATIONAL


CAPITALISM

Laura Bartlett Snyder, Doctoral Fellow in the English Department at Louisville, Boundary Dissolution in
film, photography & advertising, 2000, http://athena.louisville.edu/a-
s/english/babo/snyder/bountexts.html, accessed 10/15/02

This web site explores the ways postmodern theories of subjectivity facilitate global capitalism. The seed
for this project was planted during Deconstructed Selves, Postmodern Narratives, a session at the 20th
Century Lit. Conference. I had just heard a paper on Crash so thoughts of cyborgs and strange
postmodern desires were already mingling with a project topic that was due in my Theories of
Interpretation seminar. While Silvio Gaggi flashed slides of Cindy Shermans photographythe pictures
of her well-groomed, appropriately feminized body, a 50s starlet in juxtaposition with images of
excrement, false eyelashes, cigarette butts--I discovered my topic: the ways that the postmodern notion
of subjectivity--fluid, unfixed, transgressed boundaries--and the modern notion of subjectivity-stable,

Page 160 of 1481


unified, coherent, preserved boundaries-are analogous to the evolution from classical to global/late
capitalism. My theory: While the dissolution of boundaries in postmodern subjectivity may at first seem
wildly radical, it actually facilitates the hegemony by interpellating the ideal subject of global capitalism,
one who can manipulate fluid capital, produce/consume intangible data, and accept the dissolution of
national boundaries for the purpose of exporting manufacturing work to 3rd world countries, for the
purpose of global e-commerce, and for the formation of multinational corporations.

Page 161 of 1481


Page 162 of 1481
Floating Subjectivity Bad (3/3)
FOCUSING ON TRANSITIONAL SUBJECTIVITIES CEMENTS OPPRESSION

Aihwa Ong, Professor of Anthropology at UC Berkeley, FLEXIBLE CITIZENSHIP: THE CULTURAL


LOGIC OF TRANSNATIONALITY, 1999, p. 13.

However, the influence on American cultural studies of the Center for Contemporary Cultural Studies in
Birmingham, England, with which Hall and Gilroy are associated, has generally been limited. American
studies of diasporan cultures have tended to uphold a more innocent concept of the essential diasporan
subject, one that celebrates hybridity, cultural border crossing, and the production of difference. In the
United States, the conjuncture of postcolonial theory and diaspora studies seems to produce a bifurcated
model of diasporan cultures. Some scholars dwell on narratives of sacrifice, which are associated with
enforced labor migrations, as well as on critiques of the immorality of development. Others, who write
about displacements in borderland areas, emphasize subjects who struggle against adversity and
violation by affirming their cultural hybridity and shifting positions in society. The unified moralism
attached to subaltern subjects now also clings to diasporan ones, who are invariably assumed to be
members of oppressed classes and therefore constitutionally opposed to capitalism and state power.
Furthermore, because of the exclusive focus on texts, narratives, and subiectivities, we are often left
wondering what are the particular local-global structural articulations that materially and symbolically
shape these dynamics of victimhood and ferment.

FRAGMENTARY IDENTITY IS CRUCIAL TO GLOBALIZING CAPITALISM

Laura Bartlett Snyder, Doctoral Fellow in the English Department at Louisville, Boundary
Dissolution in film, photography & advertising, 2000, http://athena.louisville.edu/a-
s/english/babo/snyder/bountexts.html, accessed 10/15/02

With its dependence on fluid capital and the production/consumption of intangible data, global capitalism
demands the dissolution of national boundaries for the purpose of exporting manufacturing work to 3rd
world countries, for the purpose of global e-commerce, and for the formation of multinational
corporations. Global capitalism makes similar demands on its ideal producing and consuming subject,
who is articulated as fluid, fragmented, and flexible. Clearly, this subject is a radical reconfiguration of the
unified, coherent subject of classical capitalism, who is articulated for the purposes of producing and
consuming solid material goods and preserving national boundaries.

Page 163 of 1481


Page 164 of 1481
**Pragmatism**

Page 165 of 1481


Pragmatism Good: 2AC (1/3)
VOTE AFF IN SOLIDARITY WITH OUR PROJECT TO RE-POLITICIZE THE ACADEMY

David E. McClean, New School University, The Cultural Left and the Limits of Social Hope, Presented at the 2001 Annual Conference of the Society for the Advancement of American
Philosophy, www.american-philosophy.org/archives/2001%20Conference/Discussion%20papers/david_mcclean.htm.

leftist critics continue to cite and


Yet for some reason, at least partially explicated in Richard Rorty's Achieving Our Country, a book that I think is long overdue,

refer to the eccentric and often a priori ruminations of people like those just mentioned, and a litany of others including Derrida, Deleuze, Lyotard,
Jameson, and Lacan, who are to me hugely more irrelevant than Habermas in their narrative attempts to
suggest policy prescriptions (when they actually do suggest them) aimed at curing the ills of
homelessness, poverty, market greed, national belligerence and racism. I would like to suggest that it is
time for American social critics who are enamored with this group, those who actually want to be
relevant, to recognize that they have a disease, and a disease regarding which I myself must remember to stay faithful to my own twelve step program of
recovery. The disease is the need for elaborate theoretical "remedies" wrapped in neological and multi-
syllabic jargon. These elaborate theoretical remedies are more "interesting," to be sure, than the
pragmatically settled questions about what shape democracy should take in various contexts, or whether private property should
be protected by the state, or regarding our basic human nature (described, if not defined (heaven forbid!), in such statements as "We don't like to starve" and "We like to speak our minds

"When one of today's academic leftists says that


without fear of death" and "We like to keep our children safe from poverty"). As Rorty puts it,

some topic has been 'inadequately theorized,' you can be pretty certain that he or she is going to drag in
either philosophy of language, or Lacanian psychoanalysis, or some neo-Marxist version of economic
determinism. . . . These futile attempts to philosophize one's way into political relevance are a symptom
of what happens when a Left retreats from activism and adopts a spectatorial approach to the problems
of its country. Disengagement from practice produces theoretical hallucinations"(italics mine).(1) Or as John Dewey put it in his
The Need for a Recovery of Philosophy, "I believe that philosophy in America will be lost between chewing a historical cud long since reduced to

woody fiber, or an apologetics for lost causes, . . . . or a scholastic, schematic formalism , unless it can somehow bring to

consciousness America's own needs and its own implicit principle of successful action."
Those who suffer or have suffered from this disease Rorty refers to as the Cultural Left, which left is juxtaposed to the Political Left that Rorty prefers and prefers for good reason. Another
attribute of the Cultural Left is that its members fancy themselves pure culture critics who view the successes of America and the West, rather than some of the barbarous methods for
achieving those successes, as mostly evil, and who view anything like national pride as equally evil even when that pride is tempered with the knowledge and admission of the nation's

, the Cultural Left, in this country, too often dismiss American society as beyond reform and
shortcomings. In other words

redemption. And Rorty correctly argues that this is a disastrous conclusion, i.e. disastrous for the Cultural Left. I think it
may also be disastrous for our social hopes, as I will explain.

Leftist American culture critics might put their considerable talents to better use if they bury some of their
cynicism about America's social and political prospects and help forge public and political possibilities in a
spirit of determination to, indeed, achieve our country - the country of Jefferson and King; the country of John Dewey and Malcom X; the country of Franklin Roosevelt and
Bayard Rustin, and of the later George Wallace and the later Barry Goldwater. To invoke the words of King, and with reference to the American society, the time is always

ripe to seize the opportunity to help create the "beloved community," one woven with the thread of agape into a conceptually single
yet diverse tapestry that shoots for nothing less than a true intra-American cosmopolitan ethos, one wherein both same sex
unions and faith-based initiatives will be able to be part of the same social reality, one wherein business interests and the university are not seen as belonging to two separate galaxies but as

. We who fancy ourselves philosophers would do well to create from


part of the same answer to the threat of social and ethical nihilism

a new kind of public intellectual who has both a hungry theoretical mind and who
within ourselves and from within our ranks

is yet capable of seeing the need to move past high theory to other important questions that are less
bedazzling and "interesting" but more important to the prospect of our flourishing - questions such as "How is it possible to
develop a citizenry that cherishes a certain hexis, one which prizes the character of the Samaritan on the road to Jericho almost more than any other?" or "How can we square the political
dogma that undergirds the fantasy of a missile defense system with the need to treat America as but one member in a community of nations under a "law of peoples?"

The new public philosopher might seek to understand labor law and military and trade theory and
doctrine as much as theories of surplus value; the logic of international markets and trade agreements as
much as critiques of commodification, and the politics of complexity as much as the politics of power (all of
which can still be done from our arm chairs.) This means going down deep into the guts of our quotidian social institutions,

into the grimy pragmatic details where intellectuals are loathe to dwell but where the officers and
bureaucrats of those institutions take difficult and often unpleasant, imperfect decisions that affect other

Page 166 of 1481


peoples' lives, and it means making honest attempts to truly understand how those institutions actually
function in the actual world before howling for their overthrow commences. This might help keep us from
being slapped down in debates by true policy pros who actually know what they are talking about but
who lack awareness of the dogmatic assumptions from which they proceed, and who have not yet found a good reason to listen
to jargon-riddled lectures from philosophers and culture critics with their snobish disrespect for the so-called "managerial class."

Page 167 of 1481


Page 168 of 1481
Pragmatism Good: 2AC (2/3)
SMACK TALKING ABOUT CHEATERS: READ LIBERALLY

David E. McClean, New School University, The Cultural Left and the Limits of Social Hope, Presented at the 2001 Annual Conference of the Society for the Advancement of American
Philosophy, www.american-philosophy.org/archives/2001%20Conference/Discussion%20papers/david_mcclean.htm.

There is a lot of philosophical prose on the general subject of social justice. Some of this is quite good, and some of it is quite bad. What distinguishes the good from the bad is not merely the

level of erudition . Displays of high erudition are gratuitously reflected in much of the writing by those, for
example, still clinging to Marxian ontology and is often just a useful smokescreen which shrouds a near
total disconnect from empirical reality. This kind of political writing likes to make a lot of references to
other obscure, jargon-laden essays and tedious books written by other true believers - the crowd that
takes the fusion of Marxian and Freudian private fantasies seriously. Nor is it the lack of scholarship that makes this prose bad. Much of
it is well "supported" by footnotes referencing a lode of other works, some of which are actually quite good. Rather , what makes this prose bad is its utter

lack of relevance to extant and critical policy debates, the passage of actual laws, and the amendment of
existing regulations that might actually do some good for someone else. The writers of this bad prose are
too interested in our arrival at some social place wherein we will finally emerge from our "inauthentic"
state into something called "reality." Most of this stuff, of course, comes from those steeped in the Continental tradition (particularly post-Kant). While that
tradition has much to offer and has helped shape my own philosophical sensibilities, it is anything but useful when it comes to truly relevant

philosophical analysis, and no self-respecting Pragmatist can really take seriously the strong poetry of formations like "authenticity looming on the ever remote horizons of
fetishization." What Pragmatists see instead is the hope that we can fix some of the social ills that face us if we

treat policy and reform as more important than Spirit and Utopia.

the substance of this prose dissipates before it can reach the


Like light rain released from pretty clouds too high in the atmosphere,

ground and be a useful component in a discussion of medicare reform or how to better regulate a pharmaceutical industry that
bankrupts senior citizens and condemns to death HIV patients unfortunate enough to have been born in Burkina Faso - and a regulatory regime that permits this. It is often too

drenched in abstractions and references to a narrow and not so merry band of other intellectuals
(Nietzsche, Bataille, Foucault, Lukcs, Benjamin) to be of much use to those who are the supposed subject matter of this preternatural
social justice literature. Since I have no particular allegiance to these other intellectuals, no particular impulse to carry their water or defend their reputations, I try and forget as much as I can
about their writings in order to make space for some new approaches and fresh thinking about that important question that always faces us - "What is to be done?" I am, I think, lucky to have
taken this decision before it had become too late.

One might argue with me that these other intellectuals are not looking to be taken seriously in the construction of solutions to specific socio-political problems. They are, after all,

they often write with


philosophers engaged in something called philosophizing. They are, after all, just trying to be good culture critics. Of course, that isn't quite true, for

specific reference to social issues and social justice in mind, even when they are fluttering about in the
ether of high theory (Lukcs, for example, was a government officer, albeit a minister of culture, which to me says a lot), and social justice is not a Platonic form but parses
into the specific quotidian acts of institutions and individuals. Social justice is but the genus heading which may be described better with reference to its species iterations- the various
conditions of cruelty and sadism which we wittingly or unwittingly permit. If we wanted to, we could reconcile the grand general theories of these thinkers to specific bureaucracies or social

such attempts, usually


problems and so try to increase their relevance. We could construct an account which acts as a bridge to relevant policy considerations. But

performed in the reams of secondary literature generated by their devotees, usually make things even
more bizarre. In any event, I don't think we owe them that amount of effort. After all, if they wanted to be
relevant they could have said so by writing in such a way that made it clear that relevance was a high
priority. For Marxians in general, everything tends to get reduced to class. For Lukcs everything tends to get reduced to "reification." But society and its social ills
are far too intricate to gloss in these ways, and the engines that drive competing interests are much more
easily explained with reference to animal drives and fears than by Absolute Spirit. That is to say, they are not easily explained
at all.

Page 169 of 1481


Page 170 of 1481
Pragmatism Good: 2AC (3/3)
INTELLECTUALS HAVE A RESPONSIBILITY TO ENGAGE WITH REAL PROBLEMS
CRITICAL TO MAKING THEIR CRITICISM RELEVANT

David E. McClean, New School University, The Cultural Left and the Limits of Social Hope, Presented at the 2001 Annual Conference of the Society for the Advancement of American
Philosophy, www.american-philosophy.org/archives/2001%20Conference/Discussion%20papers/david_mcclean.htm.

Is it really possible to philosophize by holding Foucault in one hand and the Code of Federal Regulation or the Congressional Record in the other? Given that whatever it has meant to be a

, I see no reason why referring to the way things are actually done in the
philosopher has been under siege at various levels

actual world (I mean really done, not done as we might imagine) as we think through issues of public morality and social issues
of justice shouldn't be considered a viable alternative to the way philosophy has proceeded in the past.
Instead of replacing epistemology with hermeneutics or God knows what else as the foundation of
philosophical practice, we should move social philosophers in the direction of becoming more like social
and cultural auditors rather than further in the direction of mere culture critics. We might be able to
recast philosophers who take-up questions of social justice in a serious way as the ones in society able to
traverse not only disciplines but the distances between the towers of the academy and the bastions of
bureaucracies seeking to honestly and sometimes dishonestly assess both their failings and achievements.
This we can do with a special advantage over economists, social scientists and policy specialists who are
apt to take the narrow view of most issues. We do have examples of such persons. John Dewey and Karl Popper come to mind as but two examples, but in
neither case was there enough grasp of the actual workings of social institutions that I believe will be called for in order to properly minister to a nation in need of helpful philosophical insights
in policy formation. Or it may just be that the real work will be performed by philosophically grounded and socially engaged practitioners rather than academics. People like George Soros
come to mind here.

But there are few people like George Soros around, and I think that the improbability of philosophers emerging as a special class of social auditor also marks the limits of social hope, inasmuch

asphilosophers are the class most likely to see the places at which bridges of true understanding can be
built not only between an inimical Right and Left, but between public policy and the deep and relevant
reflections upon our humanity in which philosophers routinely engage. If philosophers seek to remain what the public thinks we are
anyway, a class of persons of whom it can be said, as Orwell put it,

One has to belong to the intelligentsia to believe things like that; no ordinary man could be such a fool, then I do not know from what other class of persons to turn to navigate the

. For I do not see how policy wonks, political


complicated intellectual and emotional obstacles that prevent us from the achievement of our country

hacks, politicians, religious ideologues and special interests will do the work that needs to be done to
achieve the kind of civic consensus envisioned in our Constitution and Declaration of Independence.
Without a courageous new breed of public intellectual, one that is able to help articulate new visions for
community and social well being without fear of reaching out to others that may not share the narrow
views of the Cultural Left and Cultural Right, I do not see how America moves beyond a mere land of
toleration and oligarchy.

David E. McClean, New School University, The Cultural Left and the Limits of Social Hope, Presented at the 2001 Annual Conference of the Society for the Advancement of American
Philosophy, www.american-philosophy.org/archives/2001%20Conference/Discussion%20papers/david_mcclean.htm.

Our new president, possessing no towering intellect, talks of a people who share a continent, but are not a nation. He is right, of course. We are only beginning to learn to put tribal loyalties
aside and to let ourselves take seriously other more salutary possibilities, though we delude ourselves into believing that we have made great progress. Perhaps so-called "compassionate
conservatism," though a gimmick to win a political contest, will bear a small harvest of unintended and positive consequences, although I remain dubious about this if the task of thinking

if the not-too-Neanderthal-Right is finally willing to meet the


through what it might actually mean remains the chore of George W. Bush. But

not-too-wacky-Left at a place of dialogue somewhere in the "middle," then that is good news, provided
the Left does not miss the opportunity to rendevous. Yet, there is a problem here. Both the Cultural Left and
the Cultural Right tend to be self-righteous purists. The best chance, then, is for the emergence of Rorty's new
Political Left, in conjunction with a new Political Right. The new Political Left would be in the better position of the two to frame
the discourse since it probably has the better intellectual hardware (it tends to be more open-minded and
less dogmatic) to make a true dialogue work. They, unlike their Cultural Left peers, might find it more useful to be a
little less inimical and a little more sympathetic to what the other side might, in good faith, believe is at

Page 171 of 1481


stake. They might leave behind some of the baggage of the Cultural Left's endless ruminations (Dewey's
philosophical cud chewing) about commodity fetishization, or whether the Subject has really died, or where crack babies fit into neo-

capitalist hegemonies, and join the political fray by parsing and exposing the more basic idiotic claims and dogmas

of witless politicians and dangerous ideologues, while at the same time finding common ground, a larger
"We" perspective that includes Ronald Reagan and Angela Davis under the same tent rather than as
inhabitants of separate worlds. The operative spirit should be that of fraternal disagreement, rather than
self-righteous cold shoulders.

Page 172 of 1481


Plan focus good: Rorty (1/2)

SPECIFIC PROPOSALS PROVE THE ACTION IS THE SUPERIOR FORM ACTIVISM

Richard Rorty, philosopher, ACHIEVING OUR COUNTRY: LEFTIST THOUGHT IN TWENTIETH-


CENTURY AMERICA, 1998, p. 98-99

When we think about these latter questions, we begin to realize that one of the essential transformations
which the cultural Left will have to undergo is the shedding of its semi- conscious anti-Americanism, which
it carried over from the rage of the late Sixties. This Left will have to stop thinking up ever more abstract
and abusive names for "the system" and start trying to construct inspiring images of the country. Only by
doing so can it begin to form alliances with people outside the academyand, specifically, with the labor
unions. Outside the academy, Americans still want to feel patriotic. They still want to feel part of a nation
which can take control of its destiny and make itself a better place. If the Left forms no such alliances, it
will never have any effect on the laws of the United States. To form them will require the cultural Left to
forget about Baudrillard's account of America as Disneylandas a country of simulacraand to start
proposing changes in the laws of a real country, inhabited by real people who are enduring unnecessary
suffering, much of which can be cured by governmental action. Nothing would do more to resurrect the
American Left than agreement on a concrete political platform, a People's Charter, a list of specific
reforms. The existence of such a list endlessly reprinted and debated, equally familiar to professors and
production workers, imprinted on the memory both of professional people and of those who clean the
professionals' toiletsmight revitalize leftist politics.

THE FACT THAT SOMETHING IS PRODUCTIVE AND DESTRUCTIVE DOESNT


ELIMINATE THE NEED FOR CONCRETE POLICY ACTION

Richard Rorty, Professor, Humanities, University of Virginia, TRUTH, POLITICS, AND POSTMODERNISM:
SPINOZA LECTURES, 1997, p. 51-52.

Derrida, another writer who enjoys demonstrating that


This distinction between the theoretical and the practical point of view is often drawn by

something very important meaning, for example, or justice, or friendship is both necessary and impossible.
When asked about the implications of these paradoxical fact, Derrida usually replies that the paradox
doesn't matter when it comes to practice. More generally, a lot of the writers who are labeled `post-modernist;
and who talk a lot about impossibility, turn out to be good experimentalist social democrats when it comes to actual

political activity. I suspect, for example, that Gray, Zizek, Derrida and I, if we found ourselves citizens of the same
country, would all be voting for the same candidates, and supporting the same reforms. Post-modernist philosophers
have gotten a bad name because of their paradox-mongering habits, and their constant use of terms like `impossible; `self-contradictory' and `unrepresentable'. They have
helped create a cult of inscrutability, one which defines itself by opposition to the Enlightenment search for transparency - and more generally, to the `metaphysics of presence;

I am all for getting rid of the metaphysics


the idea that intellectual progress aims at getting things clearly illuminated, sharply delimited, wholly visible.

of presence, but I think that the rhetoric of impossibility and unrepresentability is counterproductive
overdramatization. It is one thing to say that we need to get rid of the metaphor of things being accurately represented, once and for all, as a result of being bathed
in the light of reason. This metaphor has created a lot of headaches for philosophers, and we would be better off without it. But that does not show that we are suddenly

Even if we agree that


surrounded by unrepresentables; it just shows that `more accurate representation' was never a fruitful way to describe intellectual progress.

we shall never have what Derrida calls "a full presence beyond the reach of play"; our sense of the possibilities open
to humanity will not have changed. We have learned nothing about the limits of human hope from metaphysics, or from the philosophy of history, or
from psychoanalysis. All that we have learned from `post-modern' philosophy is that we may need a different

Page 173 of 1481


gloss on the notion of `progress' than the rationalistic gloss which the Enlightenment offered. We have been given no reason to
abandon the belief that a lot of progress has been made by carrying out the Enlightenment's political
program. Since Darwin we have come to suspect that whether such progress is made will be largely a matter of luck. But we have been given no reason to stop hoping to
get lucky.

Page 174 of 1481


Page 175 of 1481
Plan focus good: Rorty (2/2)
FOCUS ON THE SPECIFIC, STATE-FOCUSED PLANS IS CRITICAL TO ALLIANCES AND
ACTIVISM

Richard Rorty, philosopher, ACHIEVING OUR COUNTRY: LEFTIST THOUGHT IN TWENTIETH-


CENTURY AMERICA, 1998, p. 98-99

The cultural Left often seems convinced that the nation-state is obsolete, and that there is therefore no point in attempting to revive national politics. The trouble with this claim

is thatthe government of our nation-state will be, for the foreseeable future, the only agent capable of making any real
difference in the amount of selfishness and sadism inflicted on Americans. It is no comfort to those in
danger of being immiserated by globalization to be told that, since national governments are now
irrelevant, we must think up a replacement for such governments. The cosmopolitan super-rich do not think any
replacements are needed, and they are likely to prevail. Bill Readings was right to say that the nation-state [has ceased] to be
the elemental unit of capitalism, but it remains the entity which makes decisions about social benefits, and thus about

social justice. The current leftist habit of taking the long view and looking beyond nationhood to a global polity is as useless as was faith in Marxs philosophy of history,
for which it has become a substitute. Both are equally irrelevant to the question of how to prevent the reemergence of hereditary castes, or of how to prevent right-wing
populists from taking advantage of resentment at that reemergence. When we think about these latter questions, we begin to realize that one of the essential transformations

This Left will


which the cultural Left will have to undergo is the shedding of its semiconscious anti-Americanism, which it carried over from the rage of the late Sixties.

have to stop thinking up ever more abstract and abusive names for the system and start trying to
construct inspiring images of the country. Only by doing so can it begin to form alliances with people
outside the academyand, specifically, with the labor unions. Outside the academy, Americans still want to feel patriotic. They still want to feel part of a nation
which can take control of its destiny and make itself a better place. If the Left forms no such alliances, it will never have any effect on

the laws of the United States. To form them will require the cultural Left to forget about Baudrillards account of
America as Disneylandas a country of simulacraand to start proposing changes in the laws of a real
country, inhabited by real people who are enduring unnecessary suffering, much of which can be cured by
governmental action. Nothing would do more to resurrect the American Left than agreement on a concrete
political platform, a Peoples Charter, a list of specific reforms. The existence of such a list endlessly reprinted
and debated, equally familiar to professors and production workers, imprinted on the memory both of professional people and of those who clean the professionals
toiletsmight revitalize leftist politics.

FOCUSING ON THE DETAILS OF POLICY IS CRITICAL TO POLITICAL EFFECTIVENESS

Richard Rorty, philosopher, ACHIEVING OUR COUNTRY: LEFTIST THOUGHT IN TWENTIETH-


CENTURY AMERICA, 1998, p. 103-104.

The Sixties did not ask how the various groups of stakeholders were to reach a consensus about when to remodel a factory rather than build a new one, what prices to pay for

. Sixties leftists skipped lightly over all the questions which had been raised by the experience of nonmarket economies in the so-called
raw materials, and the like

seemed to be suggesting that once we were rid of both bureaucrats and entrepreneurs,
socialist countries. They

the people would know how to handle competition from steel mills or textile factories in the developing
world, price hikes on imported oil, and so on. But they never told us how the people would learn how to
do this. The cultural Left still skips over such questions. Doing so is a consequence of its preference for
talking about the system rather than about specific social practices and specific changes in those
practices. The rhetoric of this Left remains revolutionary rather than reformist and pragmatic. Its insouciant use of terms like late
capitalism suggests that we can just wait for capitalism to collapse, rather than figuring out what, in the
absence of markets, will set prices and regulate distribution. The voting public, the public which must be won
over if the Left is to emerge from the academy into the public square, sensibly wants to be told the

Page 176 of 1481


details. It wants to know how things are going to work after markets are put behind us. It wants to know how participatory democracy is
supposed to function. The cultural Left offers no answers to such demands for further information , but until it confronts them

it will not be able to be a political Left. The public, sensibly, has no interest in getting rid of capitalism until it is offered

details about the alternatives. Nor should it be interested in participatory democracythe liberation of the people from the power of the technocratsuntil it
is told how deliberative assemblies will acquire the same know-how which only the technocrats presently possess. Even someone like myself, whose admiration for John

Dewey is almost unlimited, cannot take seriously his defense of participatory democracy against Walter Lippmanns insistence on the need for expertise .

Page 177 of 1481


Page 178 of 1481
**Realism**

Page 179 of 1481


Realism Good: 2AC (1/2)
FIRST, STATES INEVITABLY COMPETE WITH EACH OTHER FOR INTERNATIONAL
POWER ANY ATTEMPT TO DEVIATE FROM THIS STRUCTURE CAUSES VIOLENCE

Mearscheimer 2001
[John J., Prof. of Pol. Sci @ U. of Chicago, The Tragedy of Great Power Warfare]

Great powers fear each other. They regard each other with suspicion, and they worry that war
might be in the offing. They anticipate danger. There is little room for trust among states. For sure, the
level of fear varies across time and space, but it cannot be reduced to a trivial level. From the perspective of any one great
power, all other great powers are potential enemies. This point is illustrated by the reaction of the United
Kingdom and France to German reunification at the end of the Col War. Despite the fact that these three states had been
close allies for almost forty-five years, both the United Kingdom and France immediately began worrying about the
potential danger of a united Germany.

in a world where great powers have the capability to attack each other
The basis for this fear is that
and might have the motive to do so any state bent on survival must be at least suspicious of
other states and reluctant to trust them. Add to this the 911 problem the absence of a central authority to which a
threatened state can turn for help and states have even greater incentive to fear each other. Morever, there is no
mechanism, other than the possible self-interest of third parties, for punishing an aggressor. Because it is sometimes
difficult to deter potential aggressors, states have ample reason not to trust other states and to be prepared for war with
them.

The possible consequences of falling victim to aggression further amplify the importance of fear
as a motivating force in world politics. Great powers do not compete with each other as if international
marketplace. Political competition among states is a much more dangerous business than mere economic
intercourse, the former can lead to war, and war often means mass killing on the battlefield as well as
mass murder of civilians. In extreme cases, war can even lead to the destruction of states. The horrible
consequences of war sometimes cause states to view each other not just as competitors, but as
potentially deadly enemies. Political antagonism, in short, tends to be intense because the stakes are great.

States in the international system also aim to guarantee their own survival. Because other states are
potential threats, and because there is no higher authority to come to their rescue when they dial 911, states cannot
depend on others for their own security. Each state tends to see itself as vulnerable and alone, and
therefore it

aims to provide for its own survival. In international politics, God helps those who help themselves. This emphasis
on self-help does not preclude states from forming alliances. But alliances are only temporary marriages of convenience:
todays alliance partner might be tomorrows enemy, and todays enemy might be tomorrows alliance partner. For
example, the United States fought with China and the Soviet Union against Germany and Japan in World War II, but soon
thereafter flip-flopped enemies and partners and allied with West Germany and Japan against China and the Soviet Union
during the Cold War.

States operating in a self-help world almost always act according to their own self-interest and
do not subordinate their interests to the interests of other states, or the so-called international
community. The reason is simple: it pays to be selfish in a self-help world. This is true in the short term as
well as in the long term, because if a state loses in the short run, it might not be around for the
long haul.

Apprehensive about the ultimate intentions of other states, and a ware that they oeprate in a self-help system, states
quickly understand that the best way to ensure their survival is to be the most powerful state in
the system. The stronger a state is relative to its potential rivals, the less likely it is that
any of those rivals will attack it and threaten its survival. Weaker states will be reluctant to pick fights
with more powerful states because the weaker states are likely to suffer military defeat. Indeed, the bigger the gap in

Page 180 of 1481


power between any two states, the less likely it is that the weaker will attack the stronger.
Neither Canada nor Mexico, for example, would countenance attacking the United States, which is far more powerful than
its neighbors. The ideal situation is to be the hegemon in the system. As Immanuel Kant said, It is the desire of
every state, or of its ruler,

to arrive at a condition of perpetual peace by conquering the whole world, if that were possible. Survival
would then be almost guaranteed

Page 181 of 1481


Page 182 of 1481
Realism Good: 2AC (2/2)
SECOND, REALISM MUST BE USED STRATEGICALLY REJECTING IT RISKS WORSE
USES

Stefano Guzzini, Assistant Professor at Central European Univ., Realism in International Relations and International

Political Economy, 1998, p. 212

Therefore, in a third step, this chapter also claims that it is impossible just to heap realism onto the dustbin of history and start anew.
realist assumptions are well alive in the
This is a non-option. Although realism as a strictly causal theory has been a disappointment, various

minds of many practitioners and observers of international affairs. Although it does not correspond to a theory which helps us to understand a real world with
objective laws, it is a world-view which suggests thoughts about it, and which permeates our daily language for making sense of it. Realism has been a rich,
albeit very contestable, reservoir of lessons of the past, of metaphors and historical analogies, which, in the hands of its most gifted representatives, have been proposed, at
times imposed, and reproduced as guides to a common understanding of international affairs. Realism is alive in the collective memory and self-understanding of our (i.e.

, forgetting realism is also


Western) foreign policy elite and public, whether educated or not. Hence, we cannot but deal with it. For this reason

questionable. Of course, academic observers should not bow to the whims of daily politics. But staying at distance, or being critical, does not mean
that they should lose the capacity to understand the languages of those who make significant decisions, not

only in government, but also in firms, NGOs, and other institutions. To the contrary, this understanding, as increasingly varied as it may be, is a prerequisite for their very

profession. More particularly, it is a prerequisite for opposing the more irresponsible claims made in the name, although not

always necessarily in the spirit, of realism.

THIRD, THE PERM SOLVES BEST REALISM OPENS UP SPACE FOR ONGOING
CRITICISM, MAKING THE ALTERNATIVE POSSIBLE

Murray, Professor Politics at the University of Wales, 1997 (Alastair J.H., Reconstructing Realism:
Between Power Politics and Cosmopolitan Ethics, p. 193-6)

For realism man remains, in the final analysis, limited by himself. As such, it emphasizes caution, and focuses not merely upon the achievement of long-term objectives, but
also upon the resolution of more- immediate difficulties. Given that, in the absence of a resolution of such difficulties, longer-term
objectives are liable to be unachievable, realism would seem to offer a more effective strategy of
transition than relativism itself. Whereas, in constructivism, such strategies are divorced from an awareness of the immediate problems which obstruct such efforts, and,
in critical theoretical perspectives, they are divorced from the current realities of international politics altogether, realism's emphasis on first addressing the

immediate obstacles to development ensures that it at least generates strategies which offer us a tangible path to follow. If these
strategies perhaps lack the visionary appeal of reflectivist proposals, emphasizing simply the necessity of a restrained moderate diplomacy in order to ameliorate conflicts
between states, to foster a degree of mutual understanding in international relations, and, ultimately, to develop a sense of community which might underlie a more

, they at least seek to take advantage of the possibilities of reform in the current international system
comprehensive international society

without jeopardizing the possibilities of order. Realism's gradualist reformism, the careful tending of what it regards as an essentially organic
process, ultimately suggests the basis for a more sustainable strategy for reform than reflectivist perspectives, however dramatic, can offer. For the realist, then, if rationalist
theories prove so conservative as to make their adoption problematic, critical theories prove so progressive as to make their adoption unattractive. If the former can justifiably
be criticized for seeking to make a far from ideal order work more efficiently, thus perpetuating its existence and

legitimating its errors, reflectivist theory can equally be criticized for searching for a tomorrow which may never exist, thereby endangering the possibility of establishing any

form of stable order in the here and now. Realism's distinctive contribution thus lies in its attempt to drive a path between the two, a path
which, in the process, suggests the basis on which some form of synthesis between rationalism and relativism might be achieved. Oriented in its
genesis towards addressing the shortcomings in an idealist transformatory project, it is centrally motivated by concern to reconcile vision with practicality, to relate utopia and
reality. Unifying technical and a practical stance, it combines aspects of the positivist methodology employed by problem-solving theory with the interpretative stance adopted
by critical theory, avoiding the monism of perspective which leads to the self-destructive conflict between the two. Ultimately, it can simultaneously acknowledge the possibility
of change in the structure of the international system and the need to probe the limits of the possible, and yet also question the proximity of any international transformation,
emphasize the persistence of problems after such a transformation, and serve as a reminder of the need to grasp whatever semblance of order can be obtained in the mean
time. Indeed, it is possible to say that realism is uniquely suited to serve as such an orientation. Simultaneously to critique contemporary resolutions of the problem of political
authority as unsatisfactory and yet to support them as an attainable measure of order in an unstable world involves one in a contradiction which is difficult to accept. Yet,
because it grasps the essential ambiguity of the political, and adopts imperfectionism as its dominant motif, realism can relate these two tasks in a way which allows neither to
predominate, achieving, if not a reconciliation, then at least a viable synthesis. Perhaps the most famous realist refrain is that all politics are power politics. It is the all that is
important here. Realism lays claim to a relevance across systems, and because it relies on a conception of human nature, rather than a historically specific structure of world

Page 183 of 1481


politics, it can make good on this claim. If its observations about human nature are even remotely accurate, the problems that it addresses will transcend contingent
formulations of the problem of political order. Even in a genuine cosmopolis, conflict might become technical, but it would not be eliminated altogether.67 The primary
manifestations of power might become more economic or institutional rather than (para)military but, where disagreements occur and power exists, the employment of the one to
ensure the satisfactory resolution of the other is inevitable short of a wholesale transformation of human behaviour. Power is ultimately of the essence of politics; it is not

, realism achieves a universal relevance to the problem of political action


something which can be banished, only tamed and restrained. As a result

which allows it to relate the reformist zeal of critical theory, without which advance would be impossible, with the problem-solver's sensible caution
that before reform is attempted, whatever measure of security is possible under contemporary conditions must first be ensured

Page 184 of 1481


Page 185 of 1481
#1 Mearsheimer: 1AR
EXTEND THE 2AC #___ MEARSCHEIMER 2001 EVIDENCE. THE SELF-HELP
INTERNATIONAL SYSTEM MAKES REALISM INEVITABLE BECAUSE OF STATE
COMPETITION AND THE DESIRE FOR SURVIVAL. TRYING TO BREAK DOWN THAT
SYSTEM CAUSES POWER DIFFERENTIALS THAT RESULT IN MASS WAR AND DEATH

THAT MAKES THEIR ARGUMENT TERMINALLY NOT UNIQUE, BECAUSE STATES WILL
STILL COMPETE AND FILL THE VOID AND YOU VOTE ON ANY RISK OF WAR

ALSO, STATES ALWAYS ACT TO INCREASE THEIR RELATIVE POWER, MAKING


SECURITY COMPETITION INEVITABLE

Mearscheimer 2001
[John J., Prof. of Pol. Sci @ U. of Chicago, The Tragedy of Great Power Warfare]

Given the difficulty of determing how much power is enough for today and tomorrow, great
powers recognize that the best way to ensure their security is to achieve hegemony now, thus
eliminating any possibility of a challenge by another great power. Only a misguided state would
pass up an opportunity to be the hegemon in the system because it already had sufficient
power to survive. But even if a great power does not have the wherewithal to achieve
hegemony (and that is usually the case), it will still act offensively to amass as much power as it
can, because states are always better off with more rather than less power. In short, states do
not become status quo powers until they completely dominate the system.

All states are influence by this logic, which means htat not only do they look for opportunities to
take advantage of one another, they also work to ensure that other states do not take
advantage of them. After all, rival states are driven by the same logic, and most states are likely
to recognize their own motives at play in the actions of other states. In short, states ultimately
pay attention to defense as well as offense. They think about conquest themselves, and they
work to check aggressor states from gaining power at their expense. This inexorably leads to a
world of constant security competition, hwere states are wiling to lie, cheat, and use brute force
if it helps them gain advantage over their rivals. Peace, if one defines that concept as a state of
tranquility or mutual concord, is nt liekly to break out in this world.

Page 186 of 1481


Page 187 of 1481
#1 Mearsheimer: Ext
THEIR CRITICISM DOESNT PROVIDE US WITH A ROADMAP WHICH ENSURES
VIOLENCE REALISM IS NEEDED TO KEEP THE BALANCE OF POWER STABLE IT IS
ON BALANCE BETTER

Murray, Professor Politics at the University of Wales, 1997 (Alastair J.H., Reconstructing Realism:
Between Power Politics and Cosmopolitan Ethics, p. 188-9)

His disagreement with realism depends on a highly contestable claim - based on Herz's argument that, with the development of global threats, the conditions which might
produce some universal consensus have arisen - that its 'impossibility theorem' is empirically problematic, that a universal consensus is achievable, and that its practical
strategy is obstructing its realisation. In much the same way, in `The poverty of neorealism', realism's practical strategy is illegitimate only because Ashley's agenda is
inclusionary. His central disagreement with realism arises out of his belief that its strategy reproduces a world order organised around sovereign states, preventing exploration
of the indeterminate number of - potentially less exclusionary - alternative world orders. Realists, however, would be unlikely to be troubled by such charges. Ashley needs to
do rather more than merely assert that the development of global threats will produce some universal consensus, or that any number of less exclusionary world orders are
possible, to convince them. A universal threat does not imply a universal consensus, merely the existence of a universal threat faced by particularistic actors. And the assertion
that indeterminate numbers of potentially less exclusionary orders exist carries little weight unless we can specify exactly what these alternatives are and just how they might be
achieved. As such, realists would seem to be justified in regarding such potentialities as currently unrealizable ideals and in seeking a more proximate good in the fostering of

Despite the adverse side-effects that such a balance of power implies,


mutual understanding and, in particular. of a stable balance of power.

it at least offers us something tangible rather than ephemeral promises lacking a shred of support. Ultimately,
Ashley's demand that a new, critical approach be adopted in order to free us from the grip of such 'false conceptions depends upon ideas about the prospects for the
development of a universal consensus which are little more than wishful thinking, and ideas about the existence of potentially less exclusionary orders which are little more than
mere assertion. Hence his attempts, in 'Political realism and human interests', to conceal these ideas from view by claiming that the technical base of realism serves only to
identify, and yet not to reform, the practical, and then, in 'The poverty of neorealism', by removing the technical from investigation altogether by an exclusive reliance on a

problem of hermeneutic circularity. In the final analysis, then, Ashley's post-structuralist approach boils down to little more than a critique which fails. It
is predicated on the assumption that the constraints upon us are simply restrictive knowledge practices, such that it presumes that the entirety of the solution to our problems is

little more than the removal of such false ways of thinking. It offers nothing by way, of alternative - no strategies, no proximate goals, indeed, little by
way of goals at all. If, in constructivism, the progressive purpose leads to strategies divorced from an awareness of the problems confronting transformatory efforts, and, in
critical theoretical perspectives, it produces strategies divorced from international politics in their entirety, in post-structuralism it generates a complete absence of strategies

altogether. Critique serves to fill the void, yet this critique ultimately proves unsustainable. With its defeat, post-structuralism is left with nothing.
If realism is, as Ashley puts it, 'a tradition forever immersed in the
Once one peels away the layers of misconstruction, it simply fades away.

expectation of political tragedy'. it at least offers us a concrete vision of objectives and ways in which to
achieve them which his own position. forever immersed in the expectation of deliverance- is manifestly unable to provide."

AND, COMPETITION AMONG STATES IS INEVITABLE 3 REASONS:


1) NO CENTRAL AUTHORITY
2) STATES HAVE OFFENSIVE CAPABILITIES
3) VAGUE INTENTIONS

MEARSHEIMER 2001
[John, Co-Director of IR Policy at University of Chicago and Former research fellow at
the Brookings institute, The Tragedy of Great Power Politics, pg 3. )

Why do great powers behave this way? My answer is that the structure of the international
system forces states which seek only to be secure nonetheless to act aggressively toward each
other. Three features of the international system combine to cause states to fear one another:
1) the absence of a central authority that sits above states and can protect them from each

Page 188 of 1481


other. 2) the fact that states always have some offensive mili- tary capability, and 3) the fact
that states can never be certain about other states' intentions. Given this fear-which can never
be wholly eliminat- ed-states recognize that the more powerful they are relative to their rivals,
the better their chances of survival. Indeed, the best guarantee of survival is to be a hegemon,
because no other state can seriously threaten such a mighty power.

Page 189 of 1481


Page 190 of 1481
#2 Guzzini: 1AR
REALISM MUST BE USED STRATEGICALLY BECAUSE REAL-WORLD ACTORS RELY ON
IT

Stefano Guzzini, Assistant Professor at Central European Univ., Realism in International Relations and International

Political Economy, 1998, p. 235

Third, this last chapter has argued that although the evolution of realism has been mainly a disappointment as a general causal theory, we have to deal with it. On the one
hand, realist assumptions and insights are used and merged in nearly all frameworks of analysis offered in International Relations or International Political Economy. One of the

book's purposes was to show realism as a varied and variably rich theory, so heterogeneous that it would be better to refer to it only in plural terms. On the other hand ,

to dispose of realism because some of its versions have been proven empirically wrong, ahistorical, or logically incoherent, does not necessarily touch
its role in the shared understandings of observers and practitioners of international affairs. Realist theories have a persisting
power for constructing our understanding of the present. Their assumptions, both as theoretical constructs, and as particular lessons of the past translated from one generation

of decision-makers to another, help mobilizing certain understandings and dispositions to action. They also provide them with legitimacy. Despite realism's several deaths
is hence reflected in the actions, of many practitioners.
as a general causal theory, it can still powerfully enframe action. It exists in the minds, and

Whether or not the world realism depicts is out there, realism is. Realism is not a causal theory that explains International Relations,
but, as long as realism continues to be a powerful mind-set, we need to understand realism to make sense of International Relations. In other words, realism is a still

necessary hermeneutical bridge to the understanding of world politics. Getting rid of realism without having a deep understanding of it, not
only risks unwarranted dismissal of some valuable theoretical insights that I have tried to gather in this book; it would also be futile. Indeed, it might be the best

way to tacitly and uncritically reproduce it.

REJECTION FAILS IT REPRODUCES SOVEREIGNTY AND PERPETUATES


EXPLOITATION ACTION MUST BE TAKEN

Agathangelou, Director of the Global Change Institute, 1997 (Anna M., Studies in Political
Economy, v. 54, p. 7-8)

dissident IR also paralyzes itself into non-action. While it


Yet, ironically if not tragically,

challenges the status quo, dissident IR fails to transform it. Indeed, dissident IR claims that a
coherent paradigm or research program even an alternative one reproduces the stifling parochialism and hidden powermongering of sovereign
scholarship. Any agenda of global politics informed by critical social theory perspectives, writes Jim George must forgo the simple, albeit self-gratifying,
options inherent in readymade alternative Realisms and confront the dangers, closures, paradoxes, and complicities associated with them. Even references to
a real world, dissidents argue, repudiate the very meaning of dissidence given their sovereign presumption of a universalizable, testable Reality. What

dissident scholarship opts for, instead, is a sense of disciplinary crisis that resonates
with the effects of marginal and dissident movements in all sorts of other localities. Despite its
emancipatory intentions, this approach effectively leaves the prevailing prison
of sovereignty intact. It doubly incarcerates when dissident IR highlights the
layers of power that oppress without offering a heuristic, not to mention a program, for
emancipatory action. Merely politicizing the supposedly non-political neither
guides emancipatory action nor guards it against demagoguery. At best, dissident IR
sanctions a detached criticality rooted (ironically) in Western modernity. Michael Shapiro, for
instance, advises the dissident theorist to take a critical distance or position offshore from which to see the possibility of change. But what

Page 191 of 1481


becomes of those who know they are burning in the hells of exploitation, racism,
sexism, starvation, civil war, and the like while the esoteric dissident observes critically

from offshore? What hope do they have of overthrowing these shackles of sovereignty? In not answering these
questions, dissident IR ends up reproducing despite avowals to the contrary, the sovereign
outcome of discourse divorced from practice, analysis from policy, deconstruction from reconstruction, particulars
from universals, and critical theory from problem-solving.

Page 192 of 1481


Page 193 of 1481
#2 Guzzini: Ext
BALANCE OF POWERS REMAINS A TOP PRIORITY- STATES WILL STILL FEAR EACH
OTHER POST THE ALT

Mearsheimer, Professor of Pol Sci at University of Chicago, 01, The Tragedy of Great Power
Politics

The optimists' claim that security competition and war among the great powers has been burned out of
the system is wrong. In fact, all of the major states around the globe still care deeply about the balance of
power and are destined to compete for power among themselves for the foreseeable future.
Consequently, realism will offer the most powerful explanations of international politics over the next
century, and this will be true even if the debates among academic and policy elites are dominated by non-
realist theories. In short, the real world remains a realist world. States still fear each other and seek to
gain power at each other's expense, because international anarchythe driving force behind great-power
behaviordid not change with the end of the Cold War, and there are few signs that such change is likely
any time soon. States remain the principal actors in world politics and there is still no night watchman
standing above them. For sure, the collapse of the Soviet Union caused a major shift in the global
distribution of power. But it did not give rise to a change in the anarchic structure of the system, and
without that kind of profound change, there is no reason to expect the great powers to behave much
differently in the new century than they did in previous centuries.

OTHERS WONT FOLLOW OUR LEAD MAKES REALISM NECESSARY

Murray, Professor Politics at the University of Wales, 1997 (Alastair J.H., Reconstructing Realism:
Between Power Politics and Cosmopolitan Ethics, p. 181-2)

This highlights the central difficulty with Wendt's constructivism. It is not any form of unfounded idealism about the
possibility of effecting a change in international politics. Wendt accepts that the
intersubjective character
of international institutions such as self-help render them relatively hard social facts.
Rather, What is problematic is his faith that such chance, if it could be achieved, implies
progress. Wendt's entire approach is governed by the belief that the problematic elements of international politics can
be transcended, that the competitive identities which create these elements can be reconditioned, and that the predatory
Everything in his account, is up for gabs:
policies which underlie these identities can be eliminated.
there is no core of recalcitrance to human conduct which cannot bereformed, unlearnt, disposed of.
This venerates a stance that so privileges the possibility of a systemic
transformation that it simply puts aside the difficulties which it recognises to be
inherent in its achievement. Thus, even though Wendt acknowledges that the intersubjective basis of the self-
help system makes its reform difficult, this does not dissuade him. He simply demands that states adopt a strategy of
'altercasting', a strategy which 'tries to induce alter to take on a new identity (and thereby enlist alter in ego's effort to change
itself) by treating alter as if it already had that identity'. Wendt's
position effectively culminates in a
demand that the state undertake nothing less than a giant leap of faith. The fact that

Page 194 of 1481


its opponent might not take its overtures seriously. might not be interested in
reformulating its own construction of the world. or might simply see such an opening as a
weakness to be exploited. are completely discounted. The prospect of achieving a systemic
transformation simply outweighs any adverse consequences which might arise from the effort to achieve it. Wendt
ultimately appears, in the final analysis, to have overdosed on 'Gorbimania'.

Page 195 of 1481


Page 196 of 1481
#3 Murray: 1AR
REALISM IS THE BEST MIDDLE GROUND IT SYNTHESISES CRITICAL THEORIES IN
ORDER TO PROVIDE THE REAL POSSIBILITY FOR TRANSFORMATION

Murray, Professor Politics at the University of Wales, 1997 (Alastair J.H., Reconstructing Realism:
Between Power Politics and Cosmopolitan Ethics, p. 178-9)

n Wendt's constructivism, the argument appears in its most basic version, presenting an analysis of realist assumptions which
associate it with a conservative account of human nature. In Linklater's critical theory it moves a stage farther,
presenting an analysis of realist theory which locates it within a conservative discourse of state-centrism. In Ashley's post-
structuralism it reaches its highest form, presenting an analysis of realist strategy which locates it not merely within a
conservative statist order, but, moreover, within an active conspiracy of silence to reproduce it. Finally, in Tickner's
feminism, realism becomes all three simultaneously and more besides, a vital player in a greater, overarching, masculine
conspiracy against femininity. Realism thus appears, first, as a doctrine providing the grounds for a relentless
pessimism, second, as a theory which provides an active justification for such pessimism, and, third, as a strategy which
proactively seeks to enforce this pessimism, before it becomes the vital foundation underlying all such pessimism in
international theory. Yet, an examination of the arguments put forward from each of these perspectives suggests not only that
the effort to locate realism within a conservative. rationalist camp is untenable but, beyond this, that realism is able to
provide reformist strategies which are superior to those that they can generate themselves. The
progressive
purpose which motivates the critique of realism in these perspectives ultimately generates
a bias which undermines their own ability to generate effective strategies of
transition. In constructivism, this bias appears in its most limited version, producing strategies so
divorced from the obstacles presented by the current structure of international
politics that they threaten to become counter-productive. In critical theory it moves a stage
further producing strategies so abstract that one is at a loss to determine what they actually imply in terms of the
current structure of international politics. And, in
post-modernism, it reaches its highest form,
producing an absence of such strategies altogether, until we reach the point at
which we are left with nothing but critique. Against this failure, realism contains
the potential to act as the basis of a more constructive approach to international relations,
incorporating many of the strengths of reflectivism and yet avoiding its
weaknesses. It appears, in the final analysis, as an opening within which some synthesis
of rationalism and reflectivism. of conservatism and progressivism might be built.

Page 197 of 1481


Page 198 of 1481
#3 Murray: Ext
REALISM BRIDGES THE GAP BETWEEN CRITIQUE AND THE NEED FOR POLITICAL
ACTION IT CAN ENCORPORATE ALL OF THEIR ARGUMENTS WHILE STILL
RECOGNIZING THAT TEHRE ARE PROBLEMS THAT HAVE TO BE DEALT WITH IN THE
WORLD TODAY

Murray, Professor Politics at the University of Wales, 1997 (Alastair J.H., Reconstructing Realism:
Between Power Politics and Cosmopolitan Ethics, p. 202-3)

Ultimately, the only result of the post-positivist movement's self-styled 'alternative' status is the generation
of an unproductive opposition; between a seemingly mutually exclusive rationalism and reflectivism. Realism
would seem to hold out the possibility of a more constructive path for international relations theory. The fact
that it is engaged in a normative enquiry is not to say that it abandons a concern for the practical realities of international
politics, only that it is concerned to bridge the gap between cosmopolitan moral and power political logics. Its approach
ultimately provides an overarching framework which can draw on many different strands of thought, the
'spokes' which can be said to be attached to its central hub, to enable it to relate empirical concerns to a normative
agenda. It can incorporate the lessons that geopolitics yields, the insights that neorealism might achieve, and all
the other information that the approaches which effectively serve to articulate the specifics of its orientation generate,
and. once incorporated within its theoretical framework, relate them both to one another and to the
requirements of the ideal, in order to support an analysis of the conditions which characterise contemporary
international politics and help it to achieve a viable political ethic. Against critical theories which are
incomprehensible to any but their authors and their acolytes and which prove incapable of relating their
categories to the issues which provide the substance of international affairs, and against rationalist, and
especially neorealist, perspectives which prove unconcerned for matters of values and which simply ignore the relevance of
ethical questions to political action, realism is capable of formulating a position which brings ethics and politics
into a viable relationship. It would ultimately seem to offer us a course which navigates between the Scylla
of defending our values so badly that we end up threatening their very existence, and the Charybdis of
defending them so efficiently that we become everything that they militate against. Under its auspices.
we can perhaps succeed in reconciling our ideals with our pragmatism.

Page 199 of 1481


Page 200 of 1481
Democratic Realism Solves the
Links
DEMOCRATIC REALISM RESPONDS TO THE CRITIQUES CONCERNS, PROMOTING
THE NATIONAL INTEREST AT THE SAME TIME AS WORLD PEACE AND PROSPERITY.

Will Marshall, President of the Progressive Policy Institute, Democratic Realism: The Third Way,
BLUEPRINT, Winter 2000, http://www.ndol.org/blueprint/winter2000/marshall.html.

Democratic Realism seeks a new balance of American ideals and interests. It builds on the time-honored
principles of liberal internationalism: At the core of the post-Cold War world is a growing zone of
democracies committed to relatively open markets and free trade, political relations based on agreed-
upon rules and norms of behavior, and institutions to cooperatively manage and enforce those standards.
Protecting and extending that democratic community serves our security and economic interests while
also expressing Americans' ingrained belief in our country's historic mission. Deftly executed, policies
based on Democratic Realism can not only underpin America's vital interests and continued global
success, but help ensure a safer, more prosperous, and more democratic world.

Page 201 of 1481


Page 202 of 1481
Violence is Endemic
POLITICS MUST INCORPORATE THE EXISTENCE OF ENDEMIC VIOLENCE. WE CAN
INCORPORATE THIS WITHOUT BUYING INTO EVERY REALIST PREMISE

Stefano Guzzini, Assistant Professor at Central European University, The enduring dilemmas of realism in

International Relations, Copenhagen Peace Research Institute, December 2001,


http://www.ciaonet.org/wps/gus02/gus02.pdf, accessed 8/13/02

Until now, the purpose of this article might have appeared to be just another, perhaps more
systematically grounded, critique of the difficulties realist theories of International Relations have been
facing. By drawing on the lessons one can learn from these dilemmas, this conclusion wants to suggest a
way forward. Once we know where realism gets stuck in its analytical justification, the study of its
dilemmas should open a more reflexive way to re-apprehend Realism as a double negation and the trap of
the realism-idealism debate In what follows, I argue that the underlying reason why realists are not facing
up the implications of the identity (distinctiveness/determinacy) and the conservative (science/tradition)
dilemma consists in the terms of the first debate in which many realists feel compelled to justify realism.
According to this self-understanding, realists are there to remind us about the fearful, the cruel side of
world politics which lurks behind. This distinct face of international politics inevitably shows when the
masquerade is over. In the Venetian carnival of international diplomacy, only the experienced will be
prepared when the curtain falls and world history picks up its circular course. By trying to occupy a
vantage point of (superior) historical experience, science came then as an offer, IR realism could not
refuse. IR Realism has repeatedly thought to have no other choice but to justify this pessimism with a
need to distance itself from other positions, to be nonsubsumable. It needed to show that whatever else
might temporarily be true, there is an unflinching reality which cannot be avoided. Realism needed to
point to a reality which cannot be eventually overcome by politics, to an attitude which would similarly
rebuff the embrace by any other intellectual tradition. The first debate is usually presented as the place
in which this negative attitude has been played out, indeed mythically enshrined. It is to this
metaphorical foundation to which many self-identified realists return. Yet, I think that the first debate is
a place where the thoughts not only of so-called idealist scholars, but also of self-stylised realists look
unduly impoverished exactly because it is couched in terms of an opposition. When scholars more
carefully study the type of opposition, however, they quickly find out that many so-called realist scholars
have been not only critical of utopian thought and social engineering, but also of Realpolitik. In other
words, if one concentrates on scholars and their work, and not on labels, one sees realism not simply as
an attitude of negation which it is but as an attitude of double negation: in the words of R.N. Berki,
realism must oppose both the conservative idealism of nostalgia and the revolutionist idealism of
imagination. Norberto Bobbio has developed this double negation in his usually lucid style as both a
conservative realism which opposes the ideal, and a critical realism which opposes the apparent, a
difference too few realists have been able to disentangle. For this double heritage of political realism is
full of tensions. Realism as anti-idealism is status-quo oriented. It relies on the entire panoply of
arguments so beautifully summarised by Alfred Hirschman. According to the futility thesis, any attempt at
change is condemned to be without any real effect. The perversity thesis would argue that far from
changing for the better, such policies only add new problems to the already existing ones. And the central
jeopardy thesis says that purposeful attempts at social change will only undermine the already achieved.
The best is the enemy of the good, and so on. Anti-apparent realism, however, is an attitude more akin to
the political theories of suspicion. It looks at what is hidden behind the smokescreen of current ideologies,
putting the allegedly self-evident into the limelight of criticism. With the other form of realism , it shares a
reluctance to treat beautiful ideas as what they claim to be. But it is much more sensible to their
ideological use, revolutionary as well as conservative. Whereas anti-ideal realism defends the status quo,
anti-apparent realism questions it. It wants to unmask existing power relations.

Page 203 of 1481


Page 204 of 1481
Page 205 of 1481
Realism Inevitable
WE MUST USE REALISM BECAUSE OTHERS RELY ON IT

Stefano Guzzini, Assistant Professor at Central European Univ., Realism in International Relations and International
Political Economy, 1998, p. 227

The main line of critique can be summarized as follows: realism does not take its central concepts
seriously enough. To start with, its critiques claim that realism is a sceptical practice which however, stops
short of problematizing the inherent theory of the state. It is, second, a practice which informs an
international community. Third, international politics is not power politics because it resembles realist
precepts, but because the international community which holds a realist world-view acts in such a way as
to produce power politics: it is a social construction. Realist expectations might hold, not because they
objectively correspond to something out there, but because agents make them the maxims that guide
their actions. Finally, this can have very significant policy effects: even at the end of the Cold War which
might have shattered realist world-views, realist practices could mobilize old codes, such as to belittle the
potential historical break of the post-Berlin wall system. Realism still underlies major re-conceptualization
of the present international system, from Huntington's geocultural reification to `neomedievalism' - and
justifies the foreign policies which can be derived from them.

Page 206 of 1481


Realism Good: Prevents Nuclear War

REALISM KEY TO STOPPING NUCLEAR WAR.

Hans Morgenthau, University of Chicago, Realism in International Politics, 1958, Published in NAVAL
WAR COLLEGE REVIEW, Winter 1998.

It seems to me that to a great extent the future peace of the world-and the future peace of the world
means under present conditions the future existence of the world-will depend upon the restoration of the
original, the traditional, the realistic concepts of foreign policy: of a foreign policy which was regarded and
practiced as what you might call the "mundane business" of accommodating divergent interests, defining
seemingly incompatible interests, and then redefining them until finally they became compatible. For it
seems to me to be very unlikely that the "cold war," as it has been practiced in the last ten years, will
continue indefinitely. About five or six years ago Sir Winston Churchill said in a speech in the House of
Commons exactly this: "Things as they are cannot last; either they will get better, or they will get worse."
If the present trend continues, I think, in spite of what has been said about the desirability and possibility
of limited war, the danger of an all-out atomic war will increase. One of the instruments to avoid this
universal catastrophe lies in the restoration of those processes of a realistic foreign policy to which I have
referred.

Page 207 of 1481


Realism Good: Prevents War (1/3)

REALISM IS KEY TO INTERNATIONAL PEACE THE CRITIQUE ATTACKS THE WORST


ASPECTS OF REALIST POLITICS, THE PLAN EMBODIES THE BEST.

Robert Jervis, President, American Political Science Association, INTERNATIONAL ORGANIZATION, Autumn
1998, ASP.

Realism can also speak to the conditions under which states are most likely to cooperate and the
strategies that actors can employ to foster cooperation. This line of theorizing is sometimes associated with neoliberalism, but the two are
hard to distinguish in this area. Making a distinction would be easy if realism believed that conflict was zero-sum, that actors were always on the Pareto frontier. This conclusion
perhaps flows from the view of neoclassical economics that all arrangements have evolved to be maximally efficient, but realists see that politics is often tragic in the sense of

. Although offensive realists who see aggression and expansionism as


actors being unable to realize their common interests

omnipresent (or who believe that security requires expansion) stress the prevalence of extreme conflict of
interest, defensive realists believe that much of international politics is a Prisoners dilemma or a more
complex security dilemma. The desire to gain mixes with the need for protection; much of statecraft
consists of structuring situations so that states can maximize their common interests. The ever-present
fear that others will take advantage of the state and the knowledge that others have reciprocal worries
leads diplomats to seek arrangements that will reduce if not neutralize these concerns. Even if international politics
must remain a Prisoners Dilemma, it can often be made into one that is more benign by altering the pay-offs to encourage cooperation, for example, by enhancing each states
ability to protect itself should the other seek to exploit it and increasing the transparency that allows each to see what the other side is doing and understand why it is doing it.
The knowledge that even if others are benign today, they may become hostile in the future due to
changes of mind, circumstances, and regimes can similarly lead decision makers to create arrangements
that bind others and themselves, as previously noted.

REALISM KEY TO DIPLOMACY AND PREVENTING CONFLICT.

Robert Jervis, President, American Political Science Association, INTERNATIONAL ORGANIZATION, Autumn
1998, ASP.

Just as understanding the limits of the states power can reduce conflict, so in protecting what is most
important to them states must avoid the destructive disputes that will result from failing to respect the
vital interests of others. Realists have long argued that diplomacy and empathy are vital tools of
statecraft: conceptions of the national interest that leave no room for the aspirations and values of others
will bring ruin to the state as well as to its neighbors.

WAR AND VIOLENCE ARE ENDEMIC TO IR POLITICS, MOVING AWAY WILL


INEVITABLY RESULT IN GREAT POWER WARS

MEARSHEIMER 2001

Page 208 of 1481


[John, Co-Director of IR Policy at University of Chicago and Former research fellow at the Brookings
institute, The Tragedy of Great Power Politics, pg xi-xii. )

The twentieth century was a period of great international violence.In World War I (1914-18), roughly nine million
people died on European battlefields. About fifty million people were killed duringWorld War 11(1939-45), well over half of them civilians. Soon after the end of
World War II, the Cold War engulfed the globe. During this con-frontation, the Soviet Union and its Warsaw Pact allies never directly fought the United States
and its North Atlantic Treaty Organization allies,but many millions died in proxy wars in Korea, Vietnam, Afghanistan, Nicaragua, Angola, El Salvador, and
elsewhere. Millions also died in the century's lesser, yet still fierce, wars, including the Russo-Japanese con-flicts of 1904-5 and 1939, the Allied intervention in
the Russian Civil War from 1918 to 1920, the Russo-Polish War of 1920-21, the various Arab-Israeli wars, and the han-Iraq War of 1980-88. This cycle of

Hopes for peace will probably not be realized, because the great
violence will continue far into the new millennium.

powers that shape the international system fear each other and compete for power as a result.
Indeed, their ultimate aim is to gain a position of dominant power over others, because having
dominant power is the best means to ensure one's own survival. Strength ensures safety, and
the greatest strength is the greatest insurance of safety. States facing this incentive are fated to clash as each competes
for advantage over the others. This is a tragic situation, but there is no escaping it unless the states that make up the system

agree to form a world government. Such a vast transformation is hardly a realistic prospect, however, so conflict and war are bound to

continue as large and enduring features of world politics.

Page 209 of 1481


Page 210 of 1481
Realism Good: Prevents War
(2/3)
ANY SHIFT AWAY FROM REALISM WILL CAUSE A POWER VACUUM RESULTING IN
GREAT POWER WARS

MEARSHEIMER 2001
[John, Co-Director of IR Policy at University of Chicago and Former research fellow at
the Brookings institute, The Tragedy of Great Power Politics, pg 3. )

Alas, the claim that security competition and war between the great powers have been purged
from the international system is wrong. Indeed, there is much evidence that the promise of
everlasting peace among the great powers was stillborn. Consider, for example, that even
though the soviet threat has disappeared, the United States still maintains about one hundred
thousand troops in Europe and roughly the same number in Northeast Asia. It does so because
it recognizes that dangerous rivalries would probably emerge among the major powers in these
regions if U.S. troops were withdrawn. Moreover, almost every European state, includ- ing the
United Kingdom and France, still harbors deep-seated, albeit muted, fears that a Germany
unchecked by American power might behave aggressively; fear of Japan in Northeast Asia is
probably even more profound, and it is certainly more frequently expressed. Finally, the possi-
bility of a clash between China and the United States over Taiwan is hard- ly remote. This is not
to say that such a war is likely, but the possibility reminds us that the threat of great-power war
has not disappeared. The sad fact is that international politics has always been a ruthless and
dangerous business, and it is likely to remain that way. Although the intensity of their
competition waxes and wanes, great powers fear each other and always compete with each
other for power. The overriding goal of each state is to maximize its share of world power,
which means gain- ing power at the expense of other states. But great powers do not merely
strive to be the strongest of all the great powers, although that is a wel- come outcome. Their
ultimate aim is to be the hegemon--that is, the only great power in the system.

(NEXT PAGE)

Page 211 of 1481


Page 212 of 1481
Realism Good: Prevents War
(3/3)
(PREVIOUS PAGE)

There are no status quo powers in the international system, save for the occasional hegemon
that wants to maintain its dominating position over potential rivals. Great powers are rarely
content with the current dis- tribution of power; on the contrary, they face a constant incentive
to change it in their favor. They almost always have revisionist intentions, and they will use
force to alter the balance of power if they think it can be done at a reasonable price.3 At times,
the costs and risks of trying to shift the balance of power are too great, forcing great powers to
wait for more favorable circumstances. But the desire for more power does not go away, unless
a state achieves the ultimate goal of hegemony. Since no state is likely to achieve global
hegemony, however, the world is condemned to perpetual great-power competition. This
unrelenting pursuit of power means that great powers are Inclined to look for opportunities to
alter the distribution of world power in their favor. They will seize these opportunities if they
have the necessary capa- bility. Simply put, great powers are primed for offense. But not only
does a great power seek to gain power at the expense of other states, it also tries to thwart
rivals bent on gaining power at its expense. Thus, a great power will defend the balance of
power when looming change favors another state, and it will try to undermine the balance
when the direction of change is in its own favor.

SURVIVAL IS CONTIGENT ON OFFENSIVE MILITARY POWER

MEARSHEIMER 2001
[John, Co-Director of IR Policy at University of Chicago and Former research fellow at
the Brookings institute, The Tragedy of Great Power Politics, pg 36-7. )

The security dilemma," whith is one of the most well-known concepts in the international
relations literature, reflects the basic logic of offensive realism. The essence of the dilemma is
that the measures a state takes to increase its own security usually decrease the security of
other states. Thus, it is difficult for a state to increase its own chances of survival with- out
threatening the survival of other states. John Hen first introduced the security dilemma in a
1950 article in the journal World Politkc.'7 After dis- cussing the anarchic nature of international
politics. he writes, "Striving to attain security from . . . attack, [states] are driven to acquire more
and more power in order to escape the impact of the power of others. This, in turn, renders the
others more insecure and compels them to prepare for the worst. Since none can ever feel
entirely secure in such a world of competing units, power competition ensues, and the vicious
circle of secu- rity and power accumulation is on."8 The implication of Herz's analysis is clear:
the best way for a state to survive in anarchy is to take advantage of other states and gain
power at their expense. The best defense is a good offense. Since this message is widely
understood, ceaseless security com- petition ensues. Unfortunately, little can be done to
ameliorate the securi- ty dilemma as long as states operate in anarchy.

Page 213 of 1481


It should be apparent from this discussion that saying that states are power maximizers is
tantamount to saying that they care about relative power, not absolute power. There is an
important distinction here, because states concerned about relative power behave differently
than do states interested in absolute power.'9 States that maximize relative power are
concerned primarily with the distribution of material capabilities. In particular, they try to gain
as large a power advantage as possible over potential rivals, because power is the best means to
survival in a danger- ous world. Thus, states motivated by relative power concerns are likely to
forgo large gains in their own power, if such gains give rival states even greater power, for
smaller national gains that nevertheless provide them with a power advantage over their
rivals.20 States that maximize absolute power, on the other hand, care only about the size of
their own gains, not those of other states. They are not motivated by balance-of-power logic but
instead are concerned with amassing power without regard to how much power other states
control. They would jump at the opportunity for large gains, even if a rival gained more in the
deal. Power, according to this logic, is not a means to an end (survival), but an end in itself.2'

Page 214 of 1481


Realism Good: Militarism Solves War (1/2)

U.S. MILITARISM IS CRITICAL TO WORLD PEACE

Kagan, Hillhouse Professor of History at Yale, 1997 (Donald, Roles and Missions. Orbis, Spring,
Volume 41)

the keystone of American


Few, if any, nations in the history of the world have ever enjoyed such a favorable situation. It stands to reason that

strategy should be an effort to preserve and sustain the situation as well and as long as possible. America's
most vital interest, therefore, is maintaining the general peace, for war has been the swiftest, most expensive,
and most devastating means of changing the balance of international power. But peace does not keep itself, although one of the most
common errors in modern thinking about international relations is the assumption that peace is natural and can be preserved merely by having peace-seeking nations avoid

major war is more likely to


provocative actions. The last three-quarters of the twentieth century strongly suggests the opposite conclusion:

come when satisfied states neglect their defenses and fail to take an active part
in the preservation of peace. It is vital to understand that the current relatively peaceful and
secure situation is neither inevitable nor immutable. It reflects two conditions
built up with tremendous effort and expense during the last half century: the
great power of the United States and the general expectation that Americans
will be willing to use that power when necessary. The diminution of U.S. power
and credibility, which would follow on a policy of reduced responsibility, would thus not be a neutral act that
would leave the situation as it stands. Instead, it would be a critical step in
undermining the stability of the international situation. Calculations based on
the absence of visible potential enemies would immediately be made invalid by
America's withdrawal from its current position as the major bulwark supporting
the world order. The cost of the resulting upheaval in wealth, instability, and
the likelihood of war would be infinitely greater than the cost of continuing to
uphold the existing international structure.

AND, NON-VIOLENCE DOESNT SOLVE ITS JUST WISHFUL THINKING

Regan, Political Science Professor at Fordham, 1996 (Richard, Just War: Principles and Causes, p. 6)

Pacifists generally argue that nonviolence and nonresistance will ultimately win the minds
and hearts of aggressors and oppressors, but that argument is neither convincing nor
dispositive. The success of Gandhi or King may have been due (at least in part) to the appeal of their nonviolent campaigns to the conscience of
their oppressors. But if that is true, it is because Gandhi could appeal to the moral conscience of a free

British electorate over the heads of colonial administrators, and King could appeal to the moral
conscience of the national American electorate over the heads of regional southern officials. There is

Page 215 of 1481


no reason to believe that such campaigns would have been successful against
the rulers of Nazi Germany. Second, the argument rests on an extremely optimistic
view about the reformability of human behavior. Hobbes was surely correct in describing a persistent
conflictual pattern of human behavior. To imagine that every or even most human beings will

behave like saints seems to be wishful thinking. And even were human beings
to be so transformed at some indefinite future point of time, why should
innocent human beings suffer oppression in the intervening short run?

Page 216 of 1481


Page 217 of 1481
Realism Good: Militarism Solves
War (2/2)
AND, THEIR STRATEGY IS IMMORAL AND INCITES MORE VIOLENCE

Coates, Politics Lecturer at Reading, 1997 (A.J., The Ethics of War, p. 115-6)

Doubts arise not just about the utility or efficacy of the pacifist strategy, but also about its moral
consistency. The moral claim of the strategy rests on the assumption that non-violent resistance is
noncoercive, that here is a morally superior form of action that is not part of a culture or cycle of violence.
That assumption seems unfounded. As one critic argues: Even though your action is non-violent, its first
consequence must be to place you and your opponents in a state of war. For your opponents now have
only the same sort of choice that an army has: that of allowing you to continue occupying the heights you
have moved on to, or of applying force dynamic, active, violent force to throw you back off them. Your
opponents cannot now uphold the laws which they value without the use of such violence. And to fail to
uphold them is to capitulate to you In terms of its practical impact, therefore, your tactic is basically a
military one rather than a morally persuasive one or even a political one. (Prosch 1965, pp. 104-5) Not
only does non-violent resistance invite a violent response from an opponent; it also produces in some
cases even deliberately engineers circumstances in which those of a more militant and less sensitive
disposition can realize their violent ambitions. In such circumstances it seems either nave or hypocritical
to parade ones pacific and non-violent credentials while ignoring the key role that has been played in the
unleashing of the cycle of violence.

Page 218 of 1481


Page 219 of 1481
Realism Good: Militarism Solves
Genocide
U.S. MILITARISM IS CRITICAL TO PREVENTING GENOCIDE

Diamond, Senior Research Fellow at the Hoover Institution, 1996 (Larry, Why the United States
must remain engaged, Orbis, Summer, Volume 40, Number 3)

Much in Nordlinger's book is wise, prudent, and morally responsible. Let us hope that we never again so demonize a
global challenger that our officials are tempted to vitiate our constitution and values, or make the mistake, so tragically
common in the cold war, of embracing any ethically repugnant regime that happens to be on "our side." Let us have a
serious debate on our national interests and the military means we need to defend them. If we can pare our defense
spending further by eliminating expensive weapons programs that are not needed or not likely to work (or even in
some cases not wanted by the armed forces themselves), by all means let us do so. But let
us not make the
mistake - the core mistake of isolationists then and now - of assuming that a world without
effective rules and the power to enforce them would be any more benign than Hobbes
imagined it would be, or that a world full of escalating rivalries, arms buildups,
aggression, repression, genocide, and war would not ultimately threaten our
values, our security, and our way of life. Especially now, in a turbulent era of power instabilities and
rapidly resurgent nationalisms, world order will depend heavily on preeminent American
military power, selectively but strategically engaged around the world in the service of liberal
principles. In the necessary task of reconfiguring U.S. foreign policy for a new century, liberal internationalism offers
the best, wisest, most secure, and most humane foundation on which to build.

EVEN IF THEY WIN THAT THE PLAN DOESNT PASS WELL WIN THAT THE KRITIK
SANCTIONS GENOCIDE

Willis 12-19-95 (Ellen, The Village Voice)

If intellectuals are more inclined to rise to the discrete domestic issue than the historic
international moment, this may have less to do with the decay of the notion of international solidarity than with
the decay of confidence in their ability to change the world, not to mention the
decay of anything resembling a coherent framework of ideas within which to
understand it. Certainly the received ideas of the left, to the extent that a left can still be said to exist, have been less than helpful as a framework
for understanding the Bosnian crisis or organizing a response to it. Although the idea of American imperialism explains less and less

in a world where the locus of power is rapidly shifting to a network of transnational corporations, it still fuels a strain of reflexive

anti-interventionist sentiment whose practical result is paralyzed dithering in


the face of genocide. Floating around "progressive" circles and reinforcing the

Page 220 of 1481


dithering is a brand of vulgar pacifism whose defining characteristic is not
principled rejection of violence but squeamish aversion to dealing with it. In the
academy in particular, entrenched assumptions about identity politics and cultural relativism
promote a view of the Balkan conflict as too complicated and ambiguous to allow for
choosing sides. If there is no such thing as universality, if multiethnic democracy is not intrinsically preferable to ethnic separatism, if there are no clear-
cut aggressors and victims but merely clashing cultures, perhaps ethnic partition is simply the most practical way of resolving those "implacable ancient rivalries."\

Page 221 of 1481


Page 222 of 1481
Realism Good: Militarism Solves
Democracy
U.S. MILITARISM IS CRITICAL TO THE SPREAD OF DEMOCRACY

Diamond, Senior Research Fellow at the Hoover Institution, 1996 (Larry, Why the United States
must remain engaged, Orbis, Summer, Volume 40, Number 3)

In the past, global power has been an important reason why certain countries have become models for
emulation by others. The global power of the United States, and of its Western democratic allies, has been a
factor in the diffusion of democracy around the world, and certainly is crucial to our ability to help popular,
legitimate democratic forces deter armed threats to their overthrow, or to return to power (as in Haiti) when
they have been overthrown. Given the linkages among democracy, peace, and human rights - as well as the
recent finding of Professor Adam Przeworski (New York University) that democracy is more likely to survive in a country
when it is more widely present in the region - we should not surrender our capacity to diffuse and defend
democracy. It is not only intrinsic to our ideals but important to our national security that we remain globally
powerful and engaged - and that a dictatorship does not rise to hegemonic power within any major
region.

LITTLE B: DEMOCRACY PREVENTS WAR, MASS DEATH, AND GENOCIDE

Rummel, Professor of Political Science at the University of Hawaii & Director of the Haiku Institute of
Peace Research, 1994 (Rudolph J., Power, Genocide and Mass Murder, Journal of Peace Research,
February, Volume 31, Nubmer 1)

The principal empirical and theoretical conclusion emerging from this project confirms previous work on the causes of
war: Powerkills, absolute power kills absolutely. The more power a regime has,
the more it can act arbitrarily according to the whims and desires of the elite .
The more freely a political elite can control the power of the state apparatus, the more thoroughly it can repress and
murder its subjects and the more insistently it can declare war on domestic and foreign enemies. By contrast, the more
it will make war on others and murder its foreign and domestic subjects, the more constrained the power of a regime -
the more political power is diffused, checked, and balanced - the less it will aggress on
others and commit democide. This finding holds up through a variety of multivariate analyses
comprising over a hundred different kinds of political, cultural, social, and economic variables. All considered, including
the partial correlations, regression analysis, and the independent dimensions defined through factor analysis, a
measure of democracy versus totalitarian regimes and measures of war and rebellion are the best independent
predictors of democide (Rummel, 1995). At the extremes of power, the totalitarian regimes murdered
their people by the tens of millions, while many democracies can barely bring themselves to execute
even serial murderers. The way to virtually eliminate genocide and mass murder
appears to be through restricting and checking power. This means to foster democratic freedom.
This is the ultimate conclusion of this project.

Page 223 of 1481


Page 224 of 1481
Alt Bad: Could Make Things Worse

THE ALTERNATIVE MAY MAKE THINGS WORSE, WILL ELIMINATE BENEFITS OF THE
CURRENT ORDER

Alastair J.H. Murray, RECONSTRUCTING REALISM: BETWEEN POWER POLITICS AND COSMOPOLITAN
ETHICS, Keele University Press: Edinburgh, 1997, p. 182.

This is not merely to indulge in yet another interminable discourse on the lessons of Munich,
rejecting all strategies of assurance for more familiar policies of deterrence. A realist
perspective does not, as Wendt seems to assume, require worst-case forecasting, nor does it
adopt an ethic of sauve qui peut. But it is to suggest that, when realism emphasizes the need
for a cautious, gradual approach to attempts to transform the nature of the system, it had a
point. In Wendts analysis, change ultimately becomes as privileged as the status quo in
rationalist perspectives. If he does not hold that history is progressive, he does hold that
change is. If he is not idealistic about the possibilities of effecting a transformation of the
system, he is with regard to the way in which it might be accomplished. Yet, even if we
acknowledge that a transformation in the structure of international politics would be
beneficial, this does not imply the acceptance of a desperate gamble to accomplish it. And, at
the end of the day, if we can accept that the current structure of international politics contains
many injustices, there is no guarantee that its transformation would remove such iniquities
anyway. The only thing that the quest to overthrow the status quo does not guarantee to do is
to undermine those fragments of order that we currently possess. Ultimately, constructivism
can be seen to rest upon a value of judgment which sacrifices the safe option of remaining
within the current situation for the attempt to explore its possibilities. It can be seen to rest on
a progressive philosophy which privileges the possible over the extant and sacrifices stability on
the altar of transformation.

Page 225 of 1481


Alt Fails: Realism Inevitable (1/2)

REALISM IS INEVITABLE

John Mearsheimer, Professor, University of Chicago, THE TRAGEDY OF GREAT POWER POLITICS,

2001, p. 2.

The sad fact is that international politics has always been a ruthless and dangerous business, and it is
likely to remain that way. Although the intensity of their competition waxes and wanes, great powers
fear each other and always compete with each other for power. The overriding goal of each state is to
maximize its share of world power, which means gaining power at the expense of other states. But great
powers do not merely strive to be the strongest of all the great powers, although that is a welcome
outcome. Their ultimate aim is to be the hegemon-that is, the only great power in the system.

REALISM IS A FACT OF INTERNATIONAL POLITICS EVEN IF WE DONT LIKE IT

John Mearsheimer, Professor, University of Chicago, THE TRAGEDY OF GREAT POWER POLITICS,

2001, p. 3-4.

This situation, which no one consciously designed or intended, is genuinely tragic. Great powers that
have no reason to fight each other- that are merely concerned with their own survival- nevertheless have
little choice but to pursue power and to seek to dominate the other states in the system. This dilemma is
captured in brutally frank comments that Prussian statesman Otto von Bismarck made during the early
1860s, when it appeared that Poland, which was not an independent state at the time, might regain its
sovereignty. Restoring the Kingdom of Poland in any shape or form is tantamount to creating an ally for
any enemy that chooses to attack us, he believed, and therefore he advocated that Prussia should
smash those Poles till, losing all hope, they lie down and die; I have every sympathy for their situation,
but if we wish to survive we have no choice but to wipe them out.

Although it is depressing to realize that great powers might think and act this way, it behooves us to see
the world as it is, not as we would like it to be. For example, one of the key foreign policy issues facing
the United States is the question of how China will behave if its rapid economic growth continues and
effectively turns China into a giant Hong Kong. Many Americans believe that if China is democratic and
enmeshed in the global capitalist system, it will not act aggressively; instead it will be content with the
status quo in Northeast Asia. According to this logic, the United States should engage China in order to
promote the latters integration into the world economy, a policy that also seeks to encourage Chinas
transition to democracy. If engagement succeeds, the United States can work with a wealthy and
democratic China to promote peace around the globe. Unfortunately, a policy of engagement is doomed
to fail.

Page 226 of 1481


Page 227 of 1481
Alt Fails: Realism Inevitable
(2/2)
STATES COMPETE WITH EACHOTHER TO SURVIVE; ANY LOSS OF POWER IS ZERO
SUM, MAKING REALIST AN INEVITABILITY

MEARSHEIMER 2001
[John, Co-Director of IR Policy at University of Chicago and Former research fellow at
the Brookings institute, The Tragedy of Great Power Politics, pg 32-33 )

. There is
Great powers fear each other, They regard each other with suspicion, and they worry that war might be in the offing. They anticipate danger

little room for trust among states. For sure, the level of fear varies across time and space, but it cannot be reduced to a trivial level.
From the per- spective of any one great power, all other great powers are potential ene- mies.
This point is illustrated by the reaction of the United Kingdom and France to German reunification at the end of the Cold War. Despite the fact that these three
states had been close allies for almost forty-five years, both the United Kingdom and France immediately began worrying about the potential dangers of a

united Germany.' The basis of this fear is tha t in a world where great powers have the capability to attack each
other and might have the motive to do so, any state bent on survival must be at least suspicious of other states and reluctant
to trust them. Add to this the "911" problem-the absence of a cen- tral authority to which a threatened state can turn for help-and

states have even greater incentive to fear each other. Moreover, there is no mechanism, other than
the possible self-interest of third parties, for pun- ishing an aggressor. Because it is sometimes difficult to deter potential aggressors, states

have ample reason not to trust other states and to be prepared for war with them. The possible consequences of falling victim to

aggression further amplIfy the importance of fear as a motivating force in world politics. Great pow- ers do not compete with
each other as if international politics were merely an economic marketplace. Political competition among states is a much

more dangerous business than mere economic intercourse; the former can lead to war, and war
often means mass killing on the battlefield as well as mass murder of civilians. In extreme cases, war can even lead to the destruction of states. The

horrible consequences of war sometimes cause states to view each other not just as competitors, but as potentially deadly enemies. Political

antagonism, in short, tends to be intense, because the stakes are great. States in the international
system also aim to guarantee their own sur- vival. Because other states are potential threats, and because there is no higher
authority to come to their rescue when they dial 911, states can- not depend on others for their own security. Each state tends to see itself

as vulnerable and alone, and therefore it aims to provide for its own sur- vival. In international politics, God helps those who help themselves.
This emphasis on self-help does not preclude states from forming alliances." But alliances are only temporary marriages of convenience: today's affiance
partner might be tomorrow's enemy, and today's enemy might be tomorrow's alliance partner. For example, the United States fought with China and the Soviet
Union against Germany and Japan in World War H, but soon thereafter flip-flopped enemies and partners and allied with West Germany and Japan against

China and the Soviet Union during the Cold War. States operating in a self-help world almost always act according
to their own sell-interest and do not subordinate their interests to the inter- ests of other states,
or to the interests of the so-called international com- munity. The reason is simple: it pays to be selfish in a self-help world. This is true in the short
term as weli as in the long term, because if a state loses in the short run, it might not be around for the long haul. Apprehensive about the ultimate intentions

states quickly understand that the best way to ensure


of other states, and aware that they operate in a self-help system,

their survival is to be the most powerful state in the system. The stronger a state is relative to its potential rivals, the
less likely it is that any of those rivals will attack it and threaten its survival. Weaker states will be reluctant to pick fights with more powerful states because the
weaker states are likely to suffer military defeat. Indeed, the bigger the gap in power between any two states, the less likely it is that the weaker will attack the
stronger. Neither Canada nor Mexico, for example, would countenance attacking the United States, which is far more powerful than its neighbors. The ideal
situation is to be the hegemon in the system. As Immanuel Kant said, "It is the desire of every state, or of its ruler, to arrive at a condition of perpetual peace by
conquering the whole world, if that were possible."12 Survival would then be almost guaranteed." Consequently, states pay close attention to how power is
distributed among them, and they make a special effort to maximize their share of world power. Specifically, they look for opportunities to alter the balance of
power by acquiring additional increments of power at the expense of potential rivals. States employ a variety of means-economic, diplomatic, and military-to

Because one state's gain in


shift the balance of power in their favor, even if doing so makes other states suspicious or even hostile.

power is another state's loss, great powers tend to have a zero-sum mentality when dealing
with each other. The trick, of course, is to be the winner in this competition and to dominate the other states in the system. Thus, the claim that
states maximize relative power is tantamount to arguing that states are disposed to think offensively toward other

states, even though their ultimate motive is simply to survive. In short, great powers have aggressive intentions.'4

Page 228 of 1481


Alt Fails: Realism Will Reasset Itself

RELYING ON A STRUCTURAL APPROACH TO REFORMING INTERNATIONAL


RELATIONS FAILS, NEW PROBLEMS WILL ALWAYS DEMAND SPECIFIC REALISTIC
SOLUTIONS.

Hans Morgenthau, University of Chicago, Realism in International Politics, 1958, Published in NAVAL
WAR COLLEGE REVIEW, Winter 1998.

I could go on and on to give you examples. I'll give you another one which just comes to my mind: the
expectation (which was very prevalent in the last year or so of the Second World War) that at the end of
that war, with the enemies defeated, we would enter into a kind of millennium from which, again, power
politics with all of its manifestations would be dispelled. Secretary of State Cordell Hull, when he came
back from the Moscow Conference of 1943, at which the establishment of the United Nations had been
agreed upon, said that the United Nations would usher in a new era in foreign policy by doing away with
power politics, with alliances, with the armaments race, with spheres of influence, and so forth. And he
repeated this utopian expectation much later, in his memoirs. This is another example of the belief that
the difficulties which confront us, the risks which threaten us, the liabilities which we must face in
international affairs are the result of some kind of ephemeral, unique configuration; that if you do away
with the latter you will have done away with the liabilities, the risks, and the difficulties as well. This belief
is mistaken; for it is the very essence of historic experience that whenever you have disposed of one
danger in foreign policy another one is going to raise its head. Once we had disposed of the Axis as a
threat to American security, we were right away confronted with a new threat: the threat of the Soviet
Union. I daresay if we could, by some kind of miracle, do away tomorrow with the threat which emanates
from the Soviet Union, we would very soon be confronted again with a new threat-and perhaps from a
very unexpected quarter.

Page 229 of 1481


IR is Realist Now (1/2)

REALPOLITIK DOMINATES THE IR (5 REASONS):


1. NO CENTRAL AUTHORITY OVER STATES
2. STATES HAVE OFFENSIVE MILITARY CAPABILTIES
3. STATES INTENTIONS ARE AMBIGUOUS
4. LONG TERM SURVIVAL IS A STATES PRIMARY GOAL
5. STATES ARE RATIONAL ACTORS

MEARSHEIMER 2001
[John, Co-Director of IR Policy at University of Chicago and Former research fellow at
the Brookings institute, The Tragedy of Great Power Politics, pg 31-2 )

The first assumption is that the international system is anarchic, which does not mean that it is
chaotic or riven by disorder. It is easy to thaw that conclusion, since realism depicts a world
characterized by security compe- tition and war. By itself, however, the realist notion of anarchy
has noth- ing to do with conflict; it is an ordering principle, which says that the system
comprises independent states that have no central authority above them.4 Sovereignty, in other
words, inheres in states because there is no higher ruling body in the international system.'
There is no "government over governments. "~ The second assumption is that great powers
inherently possess some offensive military capability, which gives them the wherewithal to hurt
and possibly destroy each other. States are potentially dangerous to each other, although some
states have more military might than others and are therefore more dangerous. A state's
military power is usually identified with the particular weaponry at its disposal, although even if
there were no weapons. the Individuals in those states could still use their feet and hands to
attack the population of another state. After all, for every neck, there are two hands to choke it.
The third assumption is that states can never be certain about other states' intentions.
Specifically, no state can be sure that another state will not use its offensive military capability
to attack the first state. This is not to say that states necessarily have hostile intentions. Indeed,
all of the states in the system may be reliably benign, but it is impossible to be sure of that
judgment because intentions are impossible to divine with 100 percent cer- tainty.7 There are
many possible causes of aggression, and no state can be sure that another state is not
motivated by one of them.8 Furthermore, intentions can change quickly, so a state's intentions
can be benign one day and hostile the next. Uncertainty about intentions is unavoidable, which
means that states can never be sure that other states do not have offensive intentions to go
along with their offensive capabilities. The fourth assumption is that survival is the primary
goal of great pow- ers. Specifically, states seek to maintain their territorial integrity and the
autonomy of their domestic political order. Survival dominates other motives because, once a
state is conquered, it is unlikely to be in a posi- tion to pursue other aims. Soviet leader Josef
Stalin put the point well during a war scare in 1927: "We can and must build socialism in the
[Soviet Union]. But in order to do so we first of all have to exist."9 States can and do pursue
other goals, of course, but security is their most impor- tant objective. The fifth assumption is
that great powers are rational actors. They are aware of their external environment and they
think strategically about how to survive in it. In particular, they consider the preferences of
other states and how their own behavior is likely to affect the behavior of those other states,

Page 230 of 1481


and how the behavior of those other states is likely to affect their own strategy for survival.
Moreover, states pay attention to the long term as well as the immediate consequences of their
actions.

Page 231 of 1481


Page 232 of 1481
IR is Realist Now (2/2)
STATES VIEW POWER IS AN END IN ITSELF THIS HAS TWO IMPLICATIONS:
1. MAKES THEIR LINKS NON-UNIQUE AND INEVITABLE
2. TAKES OUT SOLVENCY AS THEIR ALTERNATIVE IS UNREALISABLE

MEARSHEIMER 2001
[John, Co-Director of IR Policy at University of Chicago and Former research fellow at
the Brookings institute, The Tragedy of Great Power Politics, pg 36 )

It should be apparent from this discussion that saying that states are power maximizers is
tantamount to saying that they care about relative power, not absolute power. There is an
important distinction here, because states concerned about relative power behave differently
than do states interested in absolute power.'~ States that maximize relative power are
concerned primarily with the distribution of material capabilities. In particular, they try to gain
as large a power advantage as possible over potential rivals, because power is the best means to
survival in a danger- ous world. Thus, states motivated by relative power concerns are likely to
forgo large gains in their own power, if such gains give rival states even greater power, for
smaller national gains that nevertheless provide them with a power advantage over their
rivals.2U States that maximize absolute power, on the other hand, care only about the size of
their own gains, not those of other states. They are not motivated by balance-of-power logic but
instead are concerned with amassing power without regard to how much power other states
control. They would jump at the opportunity for large gains, even if a rival gained more in the
deal. Power, according to this logic, is not a means to an end (survival), but an end in itself.2'

Page 233 of 1481


Miscalculation Inevitable

POWER MISCALCULATION IS INEVITABLE


1. STATES LIE
2. THEY MAKE MISTAKES IN CALCULATED STRENGTHS AND WEAKNESSES

MEARSHEIMER 2001
[John, Co-Director of IR Policy at University of Chicago and Former research fellow at
the Brookings institute, The Tragedy of Great Power Politics, pg 38. )

Nevertheless, great powers miscalculate from time to time because they invariably make important

decisions on the basis of imperfect informa- tion. States hardly ever have complete information about any situation they
confront. There are two dimensions to this problem. Potential adver- saries have incentives to misrepresent their own

strength or weakness, and to conceal their true aims.24 For example, a weaker state trying to deter a stronger state is likely
to exaggerate its own power to discourage the potential aggressor from attacking. On the other hand, a state bent on aggression is likely

to emphasize its peaceful goals while exaggerating its military weakness, so that the potential
victim does not build up its own arms and thus leaves itself vulnerable to attack. Probably no national
leader was better at practicing this kind of deception than Adolf Hitler. But even if disinformation was not a problem, great powers are often unsure about how
their own military forces, as well as the adversary's, will perform on the battlefield. For example, it is sometimes difficult to determine in advance how new
weapons and untested combat units will perform in the face of enemy fire. Peacetime maneuvers and war games are helpful but imperfect indicators of what is
likely to happen in actual combat. Fighting wars is a complicated business in which it is often diffi- cult to predict outcomes. Remember that although the United
States and its allies scored a stunning and remarkably easy victory against Iraq in early 1991, most experts at the time believed that Iraq's military would be a
formidable foe and put up stubborn resistance before finally succumbing to American military might.25

Great powers are also sometimes unsure about the resolve of opposing states as well as allies.
For example, Germany believed that if it went to war against France and Russia in the summer of 1914,
the United Kingdom would probably stay out of the fight. Saddam Hussein expected the United
States to stand aside when he invaded Kuwait in August 1990. Both aggressors guessed wrong, but each had good reason to
think that its initial judgment was correct. In the 1930s, Adolf Hitler believed that his great-power rivals would be easy to exploit and isolate because each had

, great powers
little interest in fighting Germany and instead was determined to get someone else to assume that burden. He guessed right. In short

constantly find themselves confronting situations in which they have to make important
decisions with incomplete information. Not surprisingly, they sometimes make faulty judgments
and end up doing themselves serious harm. Some defensive realists go so far as to suggest that the constraints of the international
system are so powerful that offense rarely succeeds, and that aggressive great powers invariably end up being punished.2' As noted, they emphasize that 1)
threatened states balance against aggressors and ultimately crush them, and 2) there is an offense-defense balance that is usually heavily tilted toward the
defense, thus making conquest especially difficult. Great powers, therefore, should be content with the existing balance of power and not try to change it by
force. After all, it makes little sense for a state to initiate a war that it is likely to lose; that would be self- defeating behavior. It is better to concentrate instead on
preserving the balance of power.27 Moreover, because aggressors seldom succeed, states should understand that security is abundant, and thus there is no
good strategic reason for wanting more power in the first place. In a world where conquest seldom pays, states should have relatively benign inten- tions
toward each other. If they do not, these defensive realists argue, the reason is probably poisonous domestic politics, not smart calculations about how to
guarantee one's security in an anarchic world.

ITS IMPOSSIBLE FOR STATES TO ADEQUATELY PERCIEVE FUTURE POWER


RELATIONMISCALCULATION IS INEVITABLE

MEARSHEIMER 2001
[John, Co-Director of IR Policy at University of Chicago and Former research fellow at
the Brookings institute, The Tragedy of Great Power Politics, pg 35. )

Page 234 of 1481


Second, determining how much power is enough becomes even more

complicated when great powers contemplate how power wifi be distrib-

uted among them ten or twenty years down the road. The capabilities of

individual states vary over time, sometimes markedly, and it is often diffi-

cult to predict the direction and scope of change in the balance of power.

Remembet few in the West antidpated the collapse of the Soviet Union

before it happened. In fact, during the first hail of the Cold War, many in

the West feared that the Soviet economy would eventually generate

greater wealth than the American economy, which would cause a marked

power shift against the United States and its allies. What the future holds

for China and Russia and what the balance of power will look like in 2020

is difficult to foresee.

Page 235 of 1481


Page 236 of 1481
Perm Solves: Realism Necessary
to Understand Parts of IR
PERM: COMBINE THE ALTERNATE APPROACH TO IR WITH THE REALIST STANCE OF
THE 1AC THIS PROVIDES THE BEST POSSIBLE SOLVENCY FOR DECREASING
VIOLENCE AND WAR.

Robert Jervis, President, American Political Science Association, INTERNATIONAL ORGANIZATION, Autumn
1998, ASP.

The popularity of alternative approaches to international politics cannot be explained entirely by their
scholarly virtues. Among the other factors at work are fashions and normative and political preferences.
This in part explains the increasing role of rationalism and constructivism. Important as they are, these
approaches are necessarily less complete than liberalism, Marxism and realism. Indeed, they fit better
with the latter than is often realized. Realism, then, continues to play a major role in IR scholarship. It can
elucidate the conditions and strategies that are conducive to cooperation and can account for significant
international change, including a greatly decreased tolerance for force among developed countries, which
appears to be currently the case.

Page 237 of 1481


A2 9/11 Disproves Realism

EVEN IN THE POST 9/11 WORLD, WE STILL LIVE IN AN INTENSELY REALIST WORLD
THE UN IS IN THE GUTTER, COUNTRIES DO NOT WANT TO ENGAGE IN A
COMMUNITY, AND THE US STILL REMAINS DIVIDED WITH EUROPE

Rieff, Member of the Council on Foreign Relations, 2003 (David, Mother Jones, Goodbye, New World Order,
July-August, http://www.motherjones.com/news/feature/2003/07/ma_442_01.html)

<Yes, many people still want to believe in the United Nations -- though they're becoming fewer and fewer in number. There is even the fantasy that some institutional or policy
silver bullet -- the International Criminal Court, say, or the Kyoto Protocol -- will provide an Archimedean lever for solving the world's woes. Were it not for the machinations of

the United States, which refused to sign on to either Kyoto or the international court, the argument goes, we would be well on our way to a better world; even so, America
stands only as an obstacle that will be overcome on the road to inevitable progress.

European governments
Such claims have all the ingredients of a fine press release, but the reality is more depressing. It is true, for example, that

increasingly subscribe to the ideology -- some would say the secular religion -- of human rights. But then so does the
United States; after all, the official position of the U.S. government is that the intervention in Iraq was
undertaken at least in part in the name of human rights. Now a doctrine that can be claimed by the United States of America as well as the
still social democratic nations of Western Europe, and the nongovernmental organizations that view the United States as little more than a rogue state -- not to mention major
transnational corporations that have signed on to a U.N. "compact with business" -- has become elastic to the point of fatuousness. If we all claim to be pledged to the cause of
human rights (and who, it seems, does not?), then it is hard not to think of Dr. Johnson's remark about patriotism, that it is the last refuge of a scoundrel.

As far as the international system is concerned, what are the most striking aspects of the current situation? There is the United Nations sunk in
irrelevancy, except as the world's leading humanitarian relief organization. There is a landscape of
international relations that seems far more to resemble the bellicose world of pre-1914 Europe than the
interdependent, responsible world imagined by the framers of the U.N. Charter. There is an entire
continent, sub-Saharan Africa, mired in an economic calamity largely not of its own making. There is a
Europe that pays lip service to human rights, but remains intransigent where its own real interests -- such as
farm subsidies that effectively condemn sub-Saharan Africa to grinding poverty by limiting its agricultural exports -- are concerned. And then there is

the United States, seemingly bent on empire.


Where was the good news again? That Augusto Pinochet was briefly detained in London, or that Slobodan Milosevic will likely spend the rest of his life in a U.N. jail? This,
, much
while somewhere between 2 and 4 million Congolese die in the first general war in Africa since decolonization? The truth is that, outside the developed countries

of the world is actually in worse shape than it was just a few decades ago. Where there has been
progress, if that term is even appropriate in so apocalyptic a context, it has been in the realm of norms -- that is, the laws that
nations try to evade and ignore, and in which many of the most decent people on this slaughterhouse of a planet continue to believe. But we are deep in
loaves-and-fishes land here. To believe that states will suddenly come to their senses and behave as responsible

members of an "international community," when few states have ever done this, is, indeed, to believe in
miracles.
There is unquestionably a globalized world economy, which remains largely dominated by the United States and is administered through central banks, the International

there is no such thing as an international community, at least not one worthy of the name --
Monetary Fund, and the World Bank. But

even the old, Cold


assuming, that is, we mean a community of shared values and interests, not just shared membership in the United Nations. For that matter,

War-era blocs are disintegrating: The G-77, the major international organization representing the
developing world, now has trouble agreeing on anything beyond the most generic recommendations. The
run-up to the Iraq war showed the depth of the divisions within the so-called transatlantic family, and equally
sharp splits were evident within Europe during the same period. Never mind community; how can there
be any international system when what we have actually witnessed in the period since 9/11 has been the
steady erosion of the very idea of consensus in international relations?

Page 238 of 1481


A2 Cold War Disproves Realism (1/2)

REALISM ACCURATELY DESCRIBES THE WORLD POST-COLD WAR U.S.


INTERVENTIONISM PROVES

Miller, IR at the Hebrew University of Jerusalem, 2003 (Benjamin, Integrated Realism and Hegemonic Military
Intervention in Unipolarity, Hanami, Associate Professor of Political Science at San Francisco State University,
Perspectives on Structural Realism, p. 34-35)

Since the end of the Cold War, the U.S. has undertaken several military interventions abroad, fluctuating
widely in scope from the massive intervention in the Gulf War through medium-scale intervention in
Panama and Haiti to the limited and abruptly terminated engagement in Somalia. Similarly another
regional crisis (Bosnia) was the occasion for great fluctuations of policy. The U.S. response to the crisis
shifted from military disengagement in the first four years of the crisis to a considerable intervention on
the ground in the last three years. It has also refrained from intervention on other occasions, notably in
post-Soviet and African crises.

Is there a coherent logic behind these wide-ranging variations in post-Cold War U.S. intervention
behavior? Numerous critics have argued that there is not, and that this erratic behavior reflects a lack of
focus in U.S. foreign policy since the end of the Cold War and the collapse of the Soviet Union, the former
archenemy. For example, in a recent comprehensive treatment Gholz, Press and Sapolsky characterize
U.S. behavior this way: "the U.S. intervenes often in the conflicts of others, but without a consistent
rationale, without a clear sense of how to advance U.S. interests, and sometimes with unintended and
expensive consequences" (1997, 5).

In the following discussion I will challenge the conventional wisdom about the illogic and incoherence of
recent U.S. military interventions. I will argue that in contrast to widespread opinion, there is a clear logic
to postCold War interventions, even if it does not amount to a preconceived and purposive grand
strategy. Indeed, the U.S. has followed, whether consciously or not, the logic of costs and benefits,
namely different combinations of incentives and constraints in different regions. More specifically, the
intensity of U.S. interests at stake and the intensity of the regional constraints on intervention (as
reflected by the estimated costs of intervention, especially in terms of casualties) best account for the
scope of U.S. military interventions in the postCold War era. My argument suggests that different types
of regions are prone to specific levels of intervention or nonintervention because of the different
combinations of U.S. interests and constraints in each region. Thus, this logic accounts for the variations in
the scope of interventions and predicts different patterns of U.S. intervention in different regions. The
realist explanation presented here integrates the classical realist focus on state interests with the
structural realist emphasis on constraints on state action in order to provide a theoretical model of
hegemonic military intervention in unipolarity. To illustrate this model, this study will outline briefly the
variations in the scope of U.S. military engagement in all the major post-Cold War regional crises, notably
the Persian Gulf (1990-1991, Fall 1994), Panama (1989), Somalia (1992-1994), Bosnia (since 1995), Kosovo
(since 1999), Haiti (1994-1996) and also the cases of nonintervention in post-Soviet and African crises. The
proposed explanation will demonstrate the continuing relevance of realism to major issues of postCold
War U.S. foreign policy.

Page 239 of 1481


Page 240 of 1481
A2 Cold War Disproves
Realism (2/2)
REALISM IS MORE APPLICABLE IN THE POST COLD WAR ERA UNIPOLARITY
MAKES ALL STATES MORE VULNERABLE TO FOREIGN AGGRESSION

Hanami, Associate Professor of Political Science at San Francisco State University, 2003 (Andrew, December,
Structural Realism and Interconnectivity, Perspectives on Structural Realism, p. 200-201)

As a theory, now decades old , it has been said that structural realism has run its course in explanations of
international relations in the post-Cold War era. Presumably this is because since the end of the Cold War, there is now as expected the long-
term absence of a major war between the major states. For some, it was the high-conflict era of bipolarity in which structural realism had its greatest explanatory power. But

the occurrence of war was never the sole reason why structural realism explained international behavior . It
was only its most dramatic, and in some ways, its most important. Structural realism today can be expected to endure as long as state

preeminence endures and states remain the most important actors in the international system, even in
peace, for in peace one finds the rudiments of war. In recent years, non-state and near-state actors have been put forth as decisive new units in a world now focused on
economics, limited campaigns or on terrorism. The state therefore is said to have declined in relative importance. But one needs to identify the impact of such non-state actors

Interconnectivity is
in the world before we can make an assessment about the significance of the new relations they create, and the theory that explains them.

the relationship between states as conditioned by structure and state motive. Interconnectivity, as a feature of the prevailing
international structure, allows that significant internal or even multilateral actors can forge relations across borders. The
inside-out and outside-in perspectives can be seen to combine when individual personalities of key leaders, for example, may be pushed by internal, historical or group

personalities and
dynamics to act outwardly. An international organization may decide on an agenda simply from the internal inertia of its members. But

organizations are important, in part, because they represent a state's power, and to be effective they must push with
that state and act with one eye on their external environment. Personalities and organizations may initiate foreign policy, bin
foreign policy action that stems from internal drives but which goes against the grain of structure is risking
failure, and over time, successful leadership will see that.1 The disappearance of the Soviet Union from the center stage for some seems
to mean that suddenly unit-level explanations have replaced structure. But in reality the unipolarity that was created when the Soviet Union slid away merely gives unit-level
actors like personalities the appearance of .1 greater relative profile because they stand on a narrower stage. They went there before. Systemic dynamics that operated then
We should not be repulsed by
continue to persist. A change in history does not necessarily require a change in the general theory that explains history.

the continuation of the familiar just because it did not explain all actions in the past. As the simplest structure,
unipolarity may not seem as threatening to all states as bipolarity had been. If, however implausible, under bipolarity then-was a direct U.S.Soviet conflict of any proportion,
the results would have significant systemic effects. But since the onset of unipolarity if the U.S. and any other power engaged in a conflict, there would be much less system it
impact. Thus all states feel the release of dread that accompanied the prospect of superpower confrontation in which they as smaller states could only watch, wait and weather

The change from bipolarity to unipolarity is forcing most states to learn more about
as best they can.

themselves, and their world. Structure still instructs. With a lone superpower, the challenge today is not only what the U.S.
might do to second states, and they may feel the U.S. has less urgency to shape some of them as formerly was the case, but what other
second states could do to them, directly or indirectly. Whether it was true or not, states believed that strong bipolar confrontations would have negative
consequences sooner or later. Unipolarity, whether it is a moment or a few decades in length, has ushered in a more variegated and self-

help environment and has thus caused states to focus on their most likely or immediate problems. Neither
Asia nor a united Europe, as David Rieff believes, is likely to successfully challenge U.S. hegemony in the twenty-first century. In pan, this is
because European armies are shrinking both in "size and in capability. The only threats to U.S. leadershipterrorism, failed states, Saddam Hussein, Slobodan Milosivic or
even the heirs to Osama bin Laden are limited." In bipolarity, major confrontations being rare and their prevention by the action of lesser states was not possible, the
international system below the level of the superpowers was, in a sense, frozen in time. Their maneuvers mattered less because it was the potential top tier movement that held

. With the
the greatest leverage. Thus the orbit of state actions took place within a relatively immobile, stable and patterned bipolar world, as structuralists have predicted

erosion to unipolarity, the calculus has changed considerably. Now more states must watch more states. There are
not just two sides, therefore there is no "protection," sociology or structure of belonging to East or West. There is a sense of

greater anarchy, or at least, greater uncertainty as to both the movement and consequences of the actions of
states in an unbalanced world. This is worrisome particularly to smaller states because the prospect of rescue in unipolarity is
reduced as the U.S. has greater choices of how and if to prop up second states in proportion to their value
in a less bifurcated world. Both Africa and Latin America have received less attention and aid from the U.S. since 1990. This has caused Kenneth Jowitt to
remark that large parts of the world today are now "disconnected" from the main states of the world. Therefore, many things suddenly become or

appear to become important to smaller states: their economies, militaries, allies, rivals, relations with the U.S. and even their relations with
bigger states like Russia, China or other regional powers. Everything matters more because the importance of margins has

increased in a unipolar world as small gains or losses tilt states no longer buoyed by a superpower

Page 241 of 1481


sponsorship. Indeed, the fact that the U.S. remains the only important superpower may have led Osama bin Laden to target the "World Trade Center and Pentagon on
September 11, 2001, as he and his al Qaida group tried to "balance" or, in their minds, punish or alter U.S. behavior in the Middle East

Page 242 of 1481


Page 243 of 1481
A2 Cold War End Proves
Liberalism
REMOVING US HEGEMONY WOULD BE CATASTROPHIC IN A POST COLD-WAR
WORLD

MEARSHEIMER 2001
[John, Co-Director of IR Policy at University of Chicago and Former research fellow at
the Brookings institute, The Tragedy of Great Power Politics, pg 2-3. )

Alas, the claim that security competition and war between the great powers have been purged
from the international system is wrong. Indeed, there is much evidence that the promise of
everlasting peace among the great powers was stillborn. Consider, for example, that even
though the soviet threat has disappeared, the United States still maintains about one hundred
thousand troops in Europe and roughly the same number in Northeast Asia. It does so because
it recognizes that dangerous rivalries would probably emerge among the major powers in these
regions if U.S. troops were withdrawn. Moreover, almost every European state, includ- ing the
United Kingdom and France, still harbors deep-seated, albeit muted, fears that a Germany
unchecked by American power might behave aggressively; fear of Japan in Northeast Asia is
probably even more profound, and it is certainly more frequently expressed. Finally, the possi-
bility of a clash between China and the United States over Taiwan is hard- ly remote. This is not
to say that such a war is likely, but the possibility reminds us that the threat of great-power war
has not disappeared. The sad fact is that international politics has always been a ruthless and
dangerous business, and it is likely to remain that way. Although the intensity of their
competition waxes and wanes, great powers fear each other and always compete with each
other for power. The overriding goal of each state is to maximize its share of world power,
which means gain- ing power at the expense of other states. But great powers do not merely
strive to be the strongest of all the great powers, although that is a wel- come outcome. Their
ultimate aim is to be the hegemon--that is, the only great power in the system.

Page 244 of 1481


A2 Cooperation Good (1/2)

PEACE IS IMPOSSIBLESTATES WILL CHEAT

MEARSHEIMER 2001
[John, Co-Director of IR Policy at University of Chicago and Former research fellow at
the Brookings institute, The Tragedy of Great Power Politics, pg 35 )

All states are Influenced by this logic, which means that not only do

they look for opportunities to take advantage of one another, they also

work to ensure that other states do not take advantage of them. After all,

rival states are driven by the same logic, and most states are likely to rec-

ognize their own motives at play in the actions of other states. In short,

states ultimately pay attention to defense as well as offense. They think

about conquest themselves, and they work to check aggressor states from

gaining power at their expense. This inexorably leads to a world of con-

stant security competition, where states are willing to lie, cheat, and use

brute force if it helps them gain advantage over their rivals. Peace, if one

defines that concept as a state of tranquility or mutual concord, is not like-

ly to break out in this world.

STATES COOPERATE TO GAIN POWER OVER POTENTIAL RIVALSEVERY


COOPERATION IS NEARLY IMPOSSIBLE TO SUSTAIN

MEARSHEIMER 2001
[John, Co-Director of IR Policy at University of Chicago and Former research fellow at
the Brookings institute, The Tragedy of Great Power Politics, pg 48ish]

One might conclude from the preceding discussion that my theory does not allow for any
cooperation among the great powers. But this Conclusion would be wrong. States can
cooperate, although cooperation is sometimes difficult to achieve and always difficult to
sustain. Two factors inhibit cooperation: considerations about relative gains and concern about
cheating.'3 Ultimately, great powers live in a fundamentally competitive world where they view
each other as real, or at least potential, enemies, and they therefore look to gain power at each
other's expense. Any two states contemplating cooperation must consider how profits or gains
will be distributed between them. They can think about the division in terms of either absolute
or relative gains (recall the distinction made earlier between pursuing either absolute power or

Page 245 of 1481


relative power; the concept here is the same). With absolute gains, each side is concerned with
maximizing its own profits and cares little about how much the other side gains or loses in the
deal. Each side cares about the other only to the extent that the other side's behavior affects its
own prospects for achieving maximum profits. With relative gains, on the other hand, each side
considers not only its own individual gain, but also how well it fares compared to the other side.
Because great powers care deeply about the balance of power, their thinking focuses on relative
gains when they consider cooperating with other states. For sure, each state tries to maximize
its absolute gains; still, it is more important for a state to make sure that it does no worse, and
perhaps better, than the other state in any agreement. Cooperation is more difficult to achieve,
however, when states are attuned to relative gains rather than absolute gains.~' This is because
states concerned about absolute gains have to make sure that if the pie is expanding, they are
get- ting at least some portion of the increase, whereas states that worry about relative gains
must pay careful attention to how the pie is divided, which complicates cooperative efforts.
Concerns about cheating also hinder cooperation. Great powers are often reluctant to enter
into cooperative agreements for fear that the other side will cheat on the agreement and gain a
significant advantage. This concern is especially acute in the military realm, causing a "special
peril of defection." because the nature of military weaponry allows for rapid shifts in the
balance of power.5' Such a development could create a window of opportunity for the state
that cheats to inflict a decisive defeat on its victim. These barriers to cooperation
notwithstanding, great powers do cooper- ate in a realist world. Balance-of-power logic often
causes great powers to

Page 246 of 1481


Page 247 of 1481
A2 Cooperation Good (2/2)
ALLIANCES ARE TEMPORARY AND UNRELIABLE

MEARSHEIMER 2001
[John, Co-Director of IR Policy at University of Chicago and Former research fellow at
the Brookings institute, The Tragedy of Great Power Politics, pg 33-4 )

States in the international system also aim to guarantee their own sur-

vival. Because other states are potential threats, and because there is no

higher authority to come to their rescue when they dial 911, states can-

not depend on others for their own security. Each state tends to see itself

as vulnerable and alone, and therefore it aims to provide for its own sur-

vival. In international politics, God helps those who help themselves.

This emphasis on self-help does not preclude states from forming

alliances." But alliances are only temporary marriages of convenience:

today's affiance partner might be tomorrow's enemy, and today's enemy

might be tomorrow's alliance partner. For example, the United States

fought with China and the Soviet Union against Germany and Japan in

World War I, but soon thereafter flip-flopped enemies and partners and

allied with West Germany and Japan against China and the Soviet Union

during the Cold War.

Page 248 of 1481


A2 Democracy Solves War

DEMOCRACIES STILL ENGAGE IN REALIST MINDSET

MEARSHEIMER 2001
[John, Co-Director of IR Policy at University of Chicago and Former research fellow at
the Brookings institute, The Tragedy of Great Power Politics, pg 5. )

Unfortunately, a policy of engagement is doomed to fail. If China becomes an economic


powerhouse it will almost certainly translate its economic might into military might and make a
rim at dominating Northeast Asia. Whether China is democratic and deeply enmeshed in the
global economy or autocratic and autarkic will have little effect on its behavior, because
democracies care about security as much as non- democracies do, and hegemony is the best
way for any state to guarantee its own survival. Of course, neither its neighbors nor the United
States would stand idly by while China gained increasing increments of power. Instead, they
would seek to contain China, probably by trying to form a balancing coalition. The result would
be an intense security competition between China and its rivals, with the ever-present danger of
great-power war hanging over them. In short, China and the United States are des- tined to be
adversaries as China's power grows.

Page 249 of 1481


A2 Defense Solves

OFFENSE IS THE BEST DEFENSEWHOEVER COMMITS THE FIRST STRIKE WINS 60%
OF WARS

MEARSHEIMER 2001
[John, Co-Director of IR Policy at University of Chicago and Former research fellow at
the Brookings institute, The Tragedy of Great Power Politics, pg 38. )

There is no question that systemic factors constrain aggression, especially balancing by


threatened states. But defensive realists exaggerate those restraining forces.28 Indeed, the
historical record provides little support for their claim that offense rarely succeeds. One
study estimates that there were 63 wars between 1815 and 1980, and the initiator won
39 times, which translates into about a 60 percent success rate. Turning to specific cases, Otto
von Bismarck unified Germany by winning military victories against Denmark in 1864, Austria in
1866, and France in 1870, and the United States as we know it today was created in good part
by conquest in the nineteenth century. Conquest certainly paid big dividends in these cases.
Nazi Germany won wars against Poland in 1939 and France `0 1940, but lost to the Soviet Union
between 1941 and 1945. Conquest ultimately did not pay for the Third Reich, but if Hitler had
restrained himself after the fall of France and had not invaded the Soviet Union, conquest
probably would have paid handsomely for the Nazis, In short, the historical record shows that
offense sometimes succeeds and some- times does not. The trick for a sophisticated power
maximizer is to figure out when to raise and when to fold.

Page 250 of 1481


A2 Human Nature

THE ANARCHIC SYSTEM OF IR IS THE REASON WHY OFFENSIVE REALISM IS


CORRECTWE NEVER MAKE CLAIMS ABOUT HUMAN NATURE

MEARSHEIMER 2001
[John, Co-Director of IR Policy at University of Chicago and Former research fellow at
the Brookings institute, The Tragedy of Great Power Politics, pg 56-7]

In sum, my argument is that the structure of the international system. not the particular
characteristics of individual great powers, causes them to thinic and act offensively and to seek
hegemony.6C I do not adopt Morgenthau's claim that states invariably behave aggressively
because they have a will to power hardwired into them. Instead, I assume that the prin- cipal
motive behind great-power behavior is survival. In anarchy, however, the desire to survive
encourages states to behave aggressively Nor does my theory classify states as more or less
aggressive on the basis of their eco- nomic or political systems. Offensive realism makes only a
handful of assumptions about great powers, and these assumptions apply equally to all great
powers. Except for differences in how much power each state con- trols, the theory treats all
states alike. I have now laid out the logic explaining why states seek to gain as much power as
possible over their rivals. I have said little, however, about the object of that pursuit: power
itself. The next two chapters provide a detailed discussion of this important subject.

Page 251 of 1481


Page 252 of 1481
A2 Mindset Shift
INEVITABLY PARANOIA AND DISAGREEMENTS OVER COOPERATION MAKES
REALIST IDEOLOGY INEVITABLEMOVING AWAY RISKS A DECAPITATING BLOW BY
AN INVADING NATION

MEARSHEIMER 2001
[John, Co-Director of IR Policy at University of Chicago and Former research fellow at the Brookings
institute, The Tragedy of Great Power Politics, pg 40. )

The claim Is sometimes made that great powers can transcend realist logic by working
together to build an international order that fosters peace and justice. World peace, it would appear, can only
enhance a state's pros- perity and security. America's political leaders paid considerable lip service to this line of argument over the course of the twentieth
century. President Clinton, for example, told an audience at the United Nations in September 1993 that "at the birth of this organization 48 years ago a
generation of gifted leaders from many nations stepped forward to organize the world's efforts on behalf of security and prosperity . . . Now history has granted
to us a moment of even greater opportunity . . Let us resolve that we will dream larger. . . . Let us ensure that the world we pass to our children is healthier,
safer and more abundant than the one we inhabit today.""

great powers do not work together to promote world order for its own sake.
This rhetoric notwithstanding,

, each seeks to maximize its own share of world power, which is likely to clash with the goal
Instead

of creat- ing and sustaining stable international orders. This is not to say that great powers never aim to prevent wars and
keep the peace. On the con- trary, they work hard to deter wars in which they would be the likely vic tim. In such cases, however, state behavior is driven
largely by narrow calculations about relative power, not by a commitment to build a world order independent of a state's own interests. The United States, for
exam- ple, devoted enormous resources to deterring the Soviet Union from start- ing a war in Europe during the Cold War, not because of some deep-seated
commitment to promoting peace around the world, but because American leaders feared that a Soviet victory would lead to a dangerous shift in the balance of
power.46

The particular international order that obtains at any time is mainly a by-product of the self-interested behavior of the system's great powers. The configuration
of the system, in other words, is the unintended conse- quence of great-power security competition, not the result of states acting together to organize peace.
The establishment of the Cold War order in Europe illustrates this point. Neither the Soviet Union nor the United States intended to establish it, nor did they
work together to create it. In fact, each superpower worked hard in the early years of the Cold War to gain power at the expense of the other, while preventing
the other from doing likewise.47 The system that emerged in Europe in the aftermath of World War II was the unplanned consequence of intense security
compe- tition between the superpowers.

Although that intense superpower rivalry ended along with the Cold War in 1990. Russia and the United States have not worked together to create the present
order in Europe. The United States, for example, has rejected out of hand various Russian proposals to make the Organization for Security and Cooperation in
Europe the central organizing pillar of European security (repladng the U.S.-dominated NATO). Furthermore,

Russia was deeply opposed to NATO expansion, which It viewed as a serious threat to Russian security. Recognizing that Russia's weakness would pre- clude
any retaliation, however, the United States ignored Russia's concerns and pushed NATO to accept the Czech Republic, Hungary, and Poland as new
members. Russia has also opposed u.S. policy in the Balkans over the past decade, especially NATO's 1999 war against Yugoslavia. Again, the United States
has paid little attention to Russia's concerns and has taken the steps it deems necessary to bring peace to that volatile region. Finally, it is worth noting that
although Russia is dead set against allowing the

United States to deploy ballistic missile defenses, it is highly likely that Washington will deploy such a system if it is judged to be technologically feasible. For
sure, great-power rivalry will sometimes produce a stable interna- tional order, as happened during the Cold War. Nevertheless, the great powers will continue
looking for opportunities to increase their share of world power, and if a favorable situation arises, they will move to under- mine that stable order. Consider how
hard the United States worked dur- ing the late 1980s to weaken the Soviet Union and bring down the stable order that had emerged in Europe during the latter
part of the Cold War.48 Of course, the states that stand to lose power will work to deter aggression and preserve the existing order. But their motives will be
selfish, revolving around balance-of-power logic, not some commitment to world peace. Great powers cannot commit themselves to the pursuit of a peaceful

states are unlikely to agree on a general formula for bolstering peace.


world order for two reasons. First,

, international relations scholars have never reached a consensus on what the blueprint
Certainly

should look like. In fact, it seems there are about as many theories on the causes of war and peace as there are scholars studying the subject. But
more important, poll- cymakers are unable to agree on how to create a stable world. For exam- ple, at the
Paris Peace Conference after World War I, important differences over how to create stability in Europe divided Georges Clemenceau, David Lloyd George, and
Woodrow Wilson.49 In particular, Clemenceau was determined to impose harsher terms on Gennany over the Rhineland than was either Lloyd George or

Wilson, while Lloyd George stood out as the hard-liner on German reparations . The Treaty of Versailles, not sur- prisingly, did
little to promote European stability.
Furthermore, consider American thinking on how to achieve stability in Europe in the early days of the Cold War.' The key elements for a sta- ble and durable
system were in place by the early 1950s. They included the division of Germany, the positioning of American ground forces in Western Europe to deter a Soviet
attack, and ensuring that West Germany would not seek to develop nuclear weapons. Officials in the Truman administration, however, disagreed about whether
a divided Germany would be a source of peace or war. For example, George Kennan and Paul Nitze, who held important positions in the State Department,
believed that a divided Germany would be a source of instability whereas Secretary of State Dean Acheson disagreed with them. In the 1950s, President
Eisenhower sought to end the American commitment to defend Western Europe and to provide West Germany with its owr~ nuclear deterrent. This policy,
which was never fully adopted, nevertheless caused significant instability in Europe. as it led directly to the Berlin crises of 1958-59 and 196l.~'

Second, great powers cannot put aside power considerations and work to promote international

peace because they cannot be sure that their efforts will succeed. If their attempt fails, they are
likely to pay a steep price for having neglected the balance of power, because if an aggressor

Page 253 of 1481


appears at the door there will be no answer when they dial 911. That is a risk few states are
willing to run. Therefore, prudence dictates that they behave according to realist logic. This line
of reasoning accounts for why collective security schemes, which call for states to put aside
narrow con- cerns about the balance of power and instead act in accordance with the broader
interests of the international community, invariably die at birth.

Page 254 of 1481


Page 255 of 1481
A2 Realism Assumes States
Rational
FIRST, HISTORY PROVES THAT ONLY STATES THAT ACT THROUGH SELF-INTEREST
WILL SURVIVE ONLY THE LONG RUN, ENSURING RATIONAL BEHAVIOR. CROSS-
APPLY MEARSHEIMER

SECOND REALISM DOES NOT POSIT RATIONALITY OR CONSTANCY BY STATES. WE


ONLY POINT OUT THAT SELF-HELP SYSTEMS REINFORCE THOSE TENDENCIES

Kenneth Waltz, Crams BFF, Neorealism and its Critics, ed. by Robert Keohane, 1986, p. 117-118

Most of the confusions in balance-of-power theory and criticisms of it, derive from misunderstanding these three points. A balance-of-power theory, properly stated, begins with

assumptions about states: They are unitary actors who, at a minimum, seek their own preservation and, at a maximum, drive for
universal domination. States, or those who act for them, try in more or less sensible ways to use the means available in order to achieve the ends in view. Those means fall into
two categories: internal efforts (moves to increase economic capability, to increase military strength, to develop clever strategies) and external efforts (moves to strengthen and
enlarge ones own alliance or to weaken and shrink an opposing one). The external game of alignment and realignment requires three or more players, and it is usually said
that balance-of-power systems require at least that number. The statement is false, for in a two-power system the politics of balance continue, but the way to compensate for an
incipient external disequilibrium is primarily by intensifying ones internal efforts. To the assumptions of the theory we then add the condition for its operation: that two or more
states coexist in a se1f-help system, one with no superior agent to come to the aid of states that may be weakening or to deny to any of them the use of whatever instruments
they think will serve their purposes. The theory, then, is built up from the assumed motivations of states and the actions that correspond to them. It describes the constraints
that arise from the system that those actions produce, and it indicates the expected outcome: namely, the formation of balances of power. Balance-of-power theory is

microtheory precisely in the economists sense . The system, like a market in economics, is made by the actions and interactions of its
units, and the theory is based on assumptions about their behavior. A self-help system is one in which those who do not help themselves, or who do
so less effectively than others, will fail to prosper, will lay themselves open to dangers, will suffer. Fear of such unwanted consequences

stimulates states to behave in ways that tend toward the creation of balances of power. Notice that the theory
requires no assumptions of rationality or of constancy of will on the part of all of the actors. The theory says simply that if some
do relatively well, others will emulate them or fall by the wayside. Obviously, the system wont work if all
states lose interest in preserving themselves. It will, however, continue to work if some states do, while others do not, choose
to lose their political identities, say, through amalgamation. Nor need it be assumed that all of the competing states are striving relentlessly to increase their power. The

possibility that force may be used by some states to weaken or destroy others does, however, make it difficult for them to
break out of the competitive system.

THIRD, STATES RATIONALLY CALCULATE OFFENSIVE MEASURES BEFORE TAKING


RISKS

MEARSHEIMER 2001
[John, Co-Director of IR Policy at University of Chicago and Former research fellow at
the Brookings institute, The Tragedy of Great Power Politics, pg 38. )

Nevertheless, great powers miscalculate from time to time because they invariably make
important decisions on the basis of imperfect informa- tion. States hardly ever have complete
information about any situation they confront. There are two dimensions to this problem.

Page 256 of 1481


Potential adver- saries have incentives to misrepresent their own strength or weakness, and to
conceal thek true aims.24 For example, a weaker state trying to deter a stronger state is likely to
exaggerate its own power to discourage the potential aggressor from attacking. On the other
hand, a state bent on aggression is likely to emphasize its peaceful goals while exaggerating its
military weakness, so that the potential victim does not build up its own arms and thus leaves
itself vulnerable to attack. Probably no national leader was better at practicing this kind of
deception than Adolf Hitler.

Page 257 of 1481


A2 Realism Constructs Threats

REALISM DOESNT REQUIRE WORST CASE FORECASTING OR THREAT


CONSTRUCTION. THE CRITIQUE SACRIFICES STABILITY ON THE ALTER OF
UNCERTAIN TRANSFORMATION.

Alastair Murray, Politics Department, University of Wales Swansea, Reconstructing Realism, 1997, p. 182

This is not merely to indulge in yet another interminable discourse on the `lessons of Munich', rejecting all strategies of assurance for more familiar policies of deterrence .

A realist perspective does not, as Wendt seems to assume, require worst-case forecasting, nor does it adopt an ethic of `sauve qui
peut'. But it is to suggest that, when realism emphasises the need for a cautious, gradual approach to attempts to

transform the nature of the system, it has a point. In Wendt's analysis, change ultimately becomes as privileged as the status quo in rationalist
perspectives. If he does not hold that history is progressive, he does hold that change is. If he is not idealistic about the possibilities of effecting a transformation of the system,
he is with regard to the way in which it might be accomplished. Yet, even if we acknowledge that a transformation in the structure of international politics would be beneficial,

at the end of the day, if we can accept that the current


this does not imply the acceptance of a desperate gamble to accomplish it. And,

structure of international politics contains many injustices, there is no guarantee that its transformation
would remove such iniquities anyway. The only thing that the quest to overthrow the status quo does guarantee to do is to undermine those fragments of
order that we currently possess. Ultimately, constructivism can be seen to rest upon a value judgment which sacrifices the safe option of remaining within the current

situation for the attempt to explore its possibilities. It can be seen to rest on a progressive philosophy which privileges the

possible over the extant and sacrifices stability on the altar of transformation. This is not to attempt to level a charge of
utopianism, as Wendt complains that Mearsheimer does, by emphasising constructivism's normative rather than explanatory commitment. As Wendt responds: `Constructivists
have a normative interest in promoting social change, but they pursue this by trying to explain how seemingly natural social structures, like self-help or the Cold War, are effects
of practice ... If critical theorists fail, this will be because they do not explain how the world works, not because of their values."' All theories ultimately have normative
commitments; the fact of their existence does not allow us to question the validity of constructivism's explanatory power. What does, however, is the impact of these normative

Just as reflectivists argue that the implicit conservatism of neorealism


assumptions on its account of international politics.

generates its ahistoricism, the implicit progressivism of constructivism generates its unwillingness to
acknowledge even the possibility of elements of permanency. And, just as reflectivists argue that the implicit conservatism of
neorealism generates strategies which threaten to become self-perpetuating, so the implicit progressivism of constructivism generates strategies

which threaten to become counter-productive.

REALISM IS NOT A SELF-FULFILLING PROPHECY.

Alastair Murray, Politics Department, University of Wales Swansea, Reconstructing Realism, 1997, p. 184-5

Now, if this is directed at realism, as it would seem to be, it seriously misinterprets its approach. First, as
we have seen, the `logic of anarchy' that realism portrays is not a material phenomenon, but the
intersubjective emanation of cumulative past choices, albeit choices rooted in a material account of
human nature. If realism maintains that this logic represents a relatively entrenched structure, it
nevertheless holds that it is, potentially at least, malleable by judicious statecraft. If it takes the state to
be the principal focus of this logic in contemporary world politics, there is no sense that this is permanent
or final - indeed, no sense that it is even unproblematic. Second, the notion that realism ignores the clash
between the individual's simultaneous identification as both man and citizen mistakes the entire thrust of
its work. If realism is concerned with the duties owed to the state, it is only for the conflict that this
produces with the cosmopolitan moral obligations which fall upon men. Third, if realism insisted that
change must be compatible with the national interests of the state, it also recognised that, particularly in
an age of interdependence and nuclear weapons, a stable international order could ultimately only be
built on some broader sense of community than that which existed in states alone, and was thus centrally
concerned with the extension of community in international relations.

Page 258 of 1481


A2 Realism is Amoral

THEY DONT UNDERSTAND REALISMIT IS AN EFFORT TO NEGOTIATE BETWEEN


THE INTERESTS OF MORAL AGENTS

Alastair J.H. Murray, RECONSTRUCTING REALISM: BETWEEN POWER POLITICS AND COSMOPOLITAN
ETHICS, Keele University Press: Edinburgh, 1997, p. 2.

Consequently, realism is portrayed by its opponents not only as being silent in the
contemporary normative debate, but as being incapable of saying anything. Such a conception
of realism is, however, fundamentally erroneous. Realism arose in opposition to idealism; and,
given that the locus of idealism was a concern with the moral, realisms genesis was oriented
towards normative issues. Of course, it never sought to engage in the type of abstract moral
principles, and to introduce an awareness of the pervasive influence of power in the
determination of political outcomes. Yet, whilst this presupposed an intimate involvement with
the facts as they really are, the realist concern with the real was not exclusive, but rather a
function of its desire to juxtapose it to the ideal. It sought to interrelate morality and power in
a viable synthesis, to generate a practical ethic which might prove more realistic, and more
productive, than those which ignored the rules of international politics. Realism ultimately
represented a fundamentally practical tradition of thought, centrally concerned with the moral
understandings of participants, with the productive application of these understandings, and
with the task of generating some form of moral consensus in international relations which
might support a stable international order. Whatever the merits of its solutions to these issues,
it clearly was not a positivist, explanatory theory; it was profoundly concerned for normative
issues, and, in particular, for the articulation of a self-consciously political ethic.

Page 259 of 1481


Page 260 of 1481
A2 Realism is a Self-Fulfilling
Prophecy (1/2)
THEYVE GOT IT BACKWARDS FAILURE TO PLAN FOR CATASTROPHES CAUSES
THEM

Macy General Systems Scholar and deep ecologist, 1995 (Joanna, Ecopsychology)

There is also the superstition that negative thoughts are self-fulfilling. This is of a piece
with the notion, popular in New Age circles, that we create our own reality I have had people tell me that to speak of catastrophe will just make it more likely

the contrary is nearer to the truth. Psychoanalytic theory and personal


to happen. Actually,

experience show us that it is precisely what we repress that eludes our conscious

control and tends to erupt into behavior. As Carl Jung observed, When an inner situation is
not made conscious, it happens outside as fate. But ironically, in our current
situation, the person who gives warning of a likely ecological holocaust is often
made to feel guilty of contributing to that very fate.

REALISM DOES NOT REQUIRE WORST CASE FORECASTING- IT SIMPLY DOES NOT
SACRIFICE STABILITY FOR UTOPIANISM

Murray, Professor of Politics at the University of Wales, 1997 (Alastair J.H., Reconstructing Realism: Between
Power Politics and Cosmopolitan Ethics, p. 192)

This is not merely to indulge in yet another interminable discourse on the "lessons of Munich', rejecting all
strategies of assurance for more familiar policies of deterrence. A realist perspective does not, as Wendt
seems to assume, require worst-case forecasting, nor does it adopt an ethic of "sauve qui peut'. But it is
to suggest that, when realism emphasizes the need for a cautious, gradual approach to attempts to
transform the nature of the system, it has a point. In Wendt's analysis, change ultimately becomes as
privileged as the status quo in rationalist perspectives. If he does not hold that history is progressive, he
does hold that change is. If he is not idealistic about the possibilities of effecting a transformation of the
system, he is with regard to the way in which it might be accomplished. Yet, even if we acknowledge that
a transformation in the structure of international politics would be beneficial, this does not imply the
acceptance of a desperate gamble to accomplish it. And, at the end of the day, if we can accept that the
current structure of international politics contains many injustices, there is no guarantee that its
transformation would remove such iniquities anyway. The only thing that the quest to overthrow the
status quo does guarantee to do is to undermine those fragments of order that we currently possess.
Ultimately, constructivism can be seen to rest upon a value judgment which sacrifices the safe option of
remaining within the current situation for the attempt to explore its possibilities. It can be seen to rest on
a progressive philosophy which privileges the possible over the extant and sacrifices stability on the altar

Page 261 of 1481


of transformation. This is not to attempt to level a charge of utopianism, as Wendt complains that
Mearsheimer does, by emphasizing constructivism's normative rather than explanatory commitment. As
Wendt responds: "Constructivists have a normative interest in promoting social change, but they pursue
this by trying to explain how seemingly natural social structures, like self-help or the Cold War, are effects
of practice... If critical theorists fail, this will be because they do not explain how the world works, not
because of their values."1 All theories ultimately have normative commitments; the fact of their existence
does not allow us to question the validity of constructivism's explanatory power. What does, however, is
the impact of these normative assumptions on its account of international politics. Just as reflectivists
argue that the implicit conservatism of neo-realism generates its ahistoricism the implicit progressivism of
constructivism generates its unwillingness to acknowledge even the possibility of elements of
permanency. And, just as reflectivists argue that the implicit conservatism of neorealism generates
strategies which threaten to become self-perpetuating, so the implicit progressivism of constructivism
generates strategies which threaten to become counter-productive.

Page 262 of 1481


Page 263 of 1481
A2 Realism is a Self-Fulfilling
Prophecy (2/2)
REALISM IS NOT A SELF-FULFILLING PROPHECY- IT ACCURATELY DESCRIBES THE
WORLD

Murray, 1997 [Alastair, Politics at the University of Wales Swansea, Reconstructing Realism, 1997 pg. 184-185]

Now, if this is directed at realism, as it would seem to be, it seriously misinterprets its approach. First, as
we have seen, the 'logic of anarchy' that realism portrays is not a material phenomenon, but the
intersubjective emanation of cumulative past choices, albeit choices rooted in a material account of
human nature. If realism maintains that this logic represents a relatively entrenched structure, it
nevertheless holds that it is, potentially at least, malleable by judicious statecraft. If it takes the state to
be the principal focus of this logic in contemporary world politics, there is no sense that this is permanent
or final - indeed, no sense that it is even unproblematic. Second, the notion that realism ignores the clash
between the individual's simultaneous identification as both man and citizen mistakes the entire thrust of
its work. If realism is concerned with the duties owed to the state, it is only for the conflict that this
produces with the cosmopolitan moral obligations which fall upon men. Third, if realism insisted that
change must be compatible with the national interests of the state, it also recognized that, particularly in
an age of interdependence and nuclear weapons, a stable international order could ultimately only be
built on some broader sense of community than that which existed in states alone, and was thus centrally
concerned with the extension of

Page 264 of 1481


Page 265 of 1481
A2 Social Constructivism (1/3)
CHANGING REPRESENTATIONAL PRACTICES DOESNT ALTER THE MATERIAL
REALITY OF STATE PRACTICES OR HELP CREATE BETTER POLICY FOR THE
OPPRESSED

Jarvis 2k [DSL, lecturer in the Dept. of Gov and International Relations, Faculty of Economics, Politics and Business
at U. of Sydney International Relations and the Challenge of Post Modernism, University of South Carolina Press, pg
128-30]

Perhaps more alarming though is the outright violence Ashley recom-mends in response to what at best seem trite, if not imagined, injustices. Inculpating modernity, positivism,
technical rationality, or realism with violence, racism, war, and countless other crimes not only smacks of anthropomorphism but, as demonstrated by Ashley's torturous prose
and reasoning, requires a dubious logic to malce such connections in the first place. Are we really to believe that ethereal entities like positivism, mod-ernism, or realism
emanate a "violence" that marginalizes dissidents? Indeed, where is this violence, repression, and marginalization? As self- professed dissidents supposedly exiled from the
discipline, Ashley and Walker appear remarkably well integrated into the academy-vocal, pub-lished, and at the center of the Third Debate and the forefront of theo-retical
research. Likewise, is Ashley seriously suggesting that, on the basis of this largely imagined violence, global transformation (perhaps even rev-olutionary violence) is a
necessary, let alone desirable, response? Has the rationale for emancipation or the fight for justice been reduced to such vacuous revolutionary slogans as "Down with
positivism and rationality"? The point is surely trite. Apart from members of the academy, who has heard of positivism and who for a moment imagines that they need to be

In an era of unprecedented change and turmoil, of new political and


emancipated from it, or from modernity, rationality, or realism for that matter?

military configurations, of war in the Balkans and ethnic cleansing, is Ashley really suggesting that some of the greatest
threats facing humankind or some of the great moments of history rest on such innocu-ous and largely unknown nonrealities like
positivism and realism? These are imagined and fictitious enemies, theoretical fabrications that represent

arcane, self-serving debates superfluous to the lives of most people and, arguably, to most issues of importance in international
relations. More is the pity that such irrational and obviously abstruse debate should so occupy us at a time of great global turmoil. That it does

and continues to do so reflects our lack of judicious criteria for evaluating the-ory and, more importantly, the lack of attachment theorists have

to the real world. Certainly it is right and proper that we ponder the depths of our theoretical imaginations, engage in epistemological and ontological debate, and
analyze the sociology of our lmowledge.37 But to suppose that this is the only task of international theory, let alone the most important one, smacks of intellectual elitism and

does Ashley's project, his


displays a certain contempt for those who search for guidance in their daily struggles as actors in international politics. What

it help solve the plight of the


deconstructive efforts, or valiant fight against positivism say to the truly marginalized, oppressed, and des-titute? How does

poor, the displaced refugees, the casualties of war, or the emigres of death squads? Does it in any way
speak to those whose actions and thoughts comprise the policy and practice of international relations? On
all these questions one must answer no. This is not to say, of course, that all theory should be judged by its technical rationality and problem-solving capacity as

Ashley forcefully argues. But to suppose that problem-solving technical theory is not necessary-or is in some way bad-is a

contemptuous position that abrogates any hope of solving some of the nightmarish realities that millions
confront daily. As Holsti argues, we need ask of these theorists and their theories the ultimate question, "So what?" To what purpose do they deconstruct,
problematize, destabilize, undermine, ridicule, and belittle modernist and rationalist approaches? Does this get us any further, make the world any better, or enhance the
human condition? In what sense can this "debate toward [a] bottomless pit of epistemology and metaphysics" be judged pertinent, relevant, help-ful, or cogent to anyone other

poststructural approach
than those foolish enough to be scholasti-cally excited by abstract and recondite debate.38 Contrary to Ashley's assertions, then, a

fails to empower the marginalized and, in fact, abandons them. Rather than ana-lyze the political economy of power, wealth, oppression,
production, or international relations and render an intelligible understanding of these processes, Ashley succeeds in ostracizing those he

portends to represent by delivering an obscure and highly convoluted discourse. If Ashley wishes to chastise structural
realism for its abstractness and detachment, he must be prepared also to face similar criticism, especially when he so adamantly intends his work to address the real life plight

of those who struggle at marginal places. If the relevance of Ashley's project is questionable, so too is its logic and cogency. First , we might ask to what
extent the postmodern "empha-sis on the textual, constructed nature of the world" represents "an
unwarranted extension of approaches appropriate for literature to other areas of human practice that are
more constrained by an objective reality. " All theory is socially constructed and realities like the nation-state, domestic and
international politics, regimes, or transnational agencies are obviously social fabrications. But to what extent is this observation of any real use? Just because we

acknowledge that the state is a socially fabricated entity, or that the division between domestic and international society is arbitrar-ily
inscribed does not make the reality of the state disappear or render invisible international politics. Whether socially

constructed or objectively given, the argument over the ontological status of the state is of no particular moment. Does
this change our experience of the state or somehow diminish the political-economic-juridical-military functions of the state? To recognize that states are not naturally inscribed
but dynamic entities continually in the process of being made and reimposed and are therefore culturally dissimilar, economically different, and politically atypical, while
perspicacious to our historical and theoretical understanding of the state, in no way detracts from its reality, practices, and consequences. Similarly, few would object to
Ashley's hermeneutic interpretivist understanding of the international sphere as an artificially inscribed demarcation. But, to paraphrase Holsti again, so what? This does not
That international politics and states
malce its effects any less real, diminish its importance in our lives, or excuse us from paying serious attention to it.

would not exist with-out subjectivities is a banal tautology. The point, surely, is to move beyond this and study these processes. Thus,
while intellectually interesting, con-structivist theory is not an end point as Ashley seems to think, where we all throw up our

Page 266 of 1481


hands and announce there are no foundations and all reality is an arbitrary social construction. Rather, it should
be a means of rec-ognizing the structurated nature of our being and the reciprocity between subjects and structures through history. Ashley, however, seems not to

want to do this, but only to deconstruct the state, international politics, and international theory on the basis that none of these is
objectively given but fictitious entities that arise out of modernist practices of representation. While an interesting
theoretical enterprise, it is of no great conse- quence to the study of international politics. Indeed, structuration theory has long talcen care of these ontological dilemmas that
otherwise seem to preoccupy Ashley.40

Page 267 of 1481


Page 268 of 1481
A2 Social Constructivism (2/3)
SOCIAL CONSTRUCTIVISM IS FLAWED IT FAILS TO ACKNOWLEDGE THE VALUES
THAT WE HAVE THAT HAVE CREATED PROSPERITY, FOR EXAMPLE BY STOPPING
SLAVERY

Kors, Professor of History at University of Pennsylvania and Senior Fellow at the Foreign Policy
Research Institute, 2001 (Alan Charles, Triumph without Self-Belief, Orbis, Summer, ebsco)

. It is a dangerous
What often denies us both optimism and pride, however, is the very stringency of our self-judgment untempered by historical realism

intellectual error to imagine that goodness, wisdom, order, justice, peace, freedom, legal equality, mutual forbearance, and
kindness are the "default mode" in human affairs, and that it is malice, folly, disorder, war, coercion, legal inequality,
murderous intolerance, and cruelty that stand in need of historical explanation. The West, in theory, always has understood that man has

a lower side to which he is drawn, that man is a wolf to man, and that we are governed more by prejudice and passion than by the

rational capacity of our minds. If that is so, however, then we err grievously in our assumptions of what it is that requires particular explanation in the world.
We understand the defaults; what should astonish us is the ability to change them. Rousseau and the postmodernists have it all backward in this domain. It is not aversion to

aversion to difference is the human condition; rather, it is the West's


difference, for example, that requires historical explanation, for

partial but breathtaking ability to overcome tribalism and exclusion that demands explanation, above all
in the singular American accomplishment. Anti-Semitism is not surprising; the opening of Christian America to Jews is what should amaze. Racial
aversion and injustice are not sources of wonderment; the Fourteenth Amendment and its gradual implementation are what should astonish. It is not the abuse of

power that requires explanation--that is the human condition--but the Western rule of law. Similarly, it is not
coerced religious conformity that should leave us groping for understanding, but the forging of values and institutions of religious toleration. It is not slavery that requires

slavery is one of the most universal of all human institutions; rather, it is the values and agency
explanation, for

by which the West identified slavery as an evil and, astonishment of astonishments, abolished it. Finally, it is not relative pockets of
poverty in the West that should occasion our wonder, because we used to term almost infinitely worse absolute levels of poverty simply "the human condition." Instead, what is
extraordinary are the values, institutions, knowledge, risk, ethics, and liberties that created such prosperity that we even notice that poverty at all, yet alone believe that it is

we lose our wonder at the accomplishments


eradicable. We are surprised, in a failure of intellectual analysis, by all of the wrong things, and

and aspirations of our civilization as a tragic result. Depravity should never startle us; rather, the identification and naming of depravity should
amaze us, and the attempt, frequently successful, to contain it should fill us with awe. Indeed, that attempt has been so successful in the West, relative to the human condition,
that the other world fantasized by the multiculturalists seeks entrance, again and again, at our doors, and the multiculturalists are not riding leaky boats to the otherness of the

the multiculturalists' ostensible rejection of the West's philosophical realism--their


Third World. Most obviously,

vaunted "social constructionism"-does not stay with them past their medical doctor's door. In the final analysis, it is
that last trait, the West's commitment to a logically ordered philosophical realism, that undergirds its ways of thinking, valuing, and, indeed, worshiping. Such philosophical
realism was defended by Augustine, Aquinas, and almost all fathers and doctors of the Church. While various extreme epistemological and ontological skepticisms and radical

Western civilization has always had at its core. a belief


irrationalisms have flourished, sometimes with brilliance and profundity in our history,

that there is a reality independent of our wishes for and ideas of it; that natural knowledge of that reality
is possible and indeed indispensable to human dignity; that such knowledge must be acquired through a
discipline of the will and mind; and that central to that discipline is a compact with reason. The West has willed, in
theory at least, to reduce the chaos of the world to natural coherence by the powers of the mind. Indeed, the belief that truth is independent of a

particular time and place is precisely what has led the West to borrow so much from other cultures, such that,
ironically, whole schools of tendentious thought decry Western "thefts," as if the recognition of compelling example and argument in others were a weakness, not a strength.
The West recognized and adopted Eastern systems of numbers superior to that of the Romans; it took the Aristotelianism of the High Middle Ages from the Islamic scholars
who had preserved and interpreted it in manners superior to the schools of the West; it took music, art, forms of expression, and new foods from around the earth that, in large
part out of restless curiosity about realities beyond its own, it had explored. The West has always renewed and revitalized itself by means Of recognizing superior ways to its
own. It did so, however, with a commitment to being a rational culture. The Greek principle of self-contradiction as the touchstone of error, and thus its avoidance as a
touchstone of truth, is the formal expression of a commitment to reason that the Christian West always understood to separate us from beasts and madmen. To live with self-
contradiction was not merely to fail an introduction to philosophy, it was to be less than human. Induction from experience always had a logic, and the exploration of that logic
was one of the great and ultimately triumphant pursuits of the Western mind. To live with error was to deny oneself the fruits of human light. Again, the core philosophical
assumption of Western civilization is that there is a reality that exists independently of our will and wish, and that this reality can be known by human inquiry and reason. There
were many radical ruptures in the history of certain disciplines in the West; there were no radical ruptures with the Western compact with reality and reason. It is that compact
that led to a civilization of self-scrutiny and honest borrowings; to a civilization in which self-criticism gave rise to a critical scholarship that could question and either strengthen
or repair the West's received beliefs themselves; to a civilization in which the mind could appeal, with ultimate success, against the irrational to the rational; to a way of
understanding that led to the sciences that have changed both the entire human relationship to nature and our sense of human possibilities, always tempered by our knowledge
of human nature.

Page 269 of 1481


Page 270 of 1481
A2 Social Constructivism (3/3)
SOCIAL CONSTRUCTIVISM IS THE ROOT CAUSE OF THEIR HARMS- FAILURE TO TAKE
REALIST ACTION ENSURES SYSTEMIC OPPRESSION

Kors, Professor of History at University of Pennsylvania and Senior Fellow at the Foreign Policy Research Institute,
2001 (Alan Charles, Triumph without Self-Belief, Orbis, Summer, ebsco)

The fruits of that civilization have been an unprecedented ability to modify the remediable causes of
human suffering, to give great agency to utility and charity alike; to give to each individual a degree of choice and freedom
unparalleled in all of human history; to offer a means of overcoming the station in life to which one was born by the effort of one's labor, mind, and will.
A failure to understand and to teach that accomplishment would be its very betrayal . To the extent that Western civilization survives, then,

the hope of the world survives to eradicate unnecessary suffering; to speak a language of human dignity,
responsibility, and rights linked to a common reality; to minimize the depredations of the irrational, the
unexamined, the merely prejudicial in our lives; to understand the world in which we find ourselves, and, moved by interest and charity, to apply
that knowledge for good. The contest, then, is between the realists and the antirealists, and the triumph of the West

ultimately depends on its outcome. The failure to assess the stakes of the struggle between the West and its communist adversary always came from
either a pathological self-hatred of one's own world or, at the least, from a gross undervaluation of what the West truly represented in the history of mankind. The West

has altered the human relationship to nature from one of fatalistic helplessness to one of hopeful
mastery. It has made possible a human life in which biological atavism might be replaced by cultural
value, the rule of law, individuation, and growing tolerance. It also created an intellectual class irrationally devoted to an adversarial
stance. That adversarial view of the West, in the past generation at least, had become a neo-Gramscian and thus neo-Marxist one in which the West was seen as an
unparalleled source of the arbitrary assignment of restrictive and life-stultifying roles. The enemies of the West--for some, in practice; for others, increasingly in the ideal
represented a fictive make-believe that supposedly cast grave doubt upon the West's claim of enhancing freedom, dignity, and opportunity. With the triumph of the West in

reality, and with the celebration of Marxism and the Third World shown more and more to have been truly delusional, the adversarial intellectual class
appears to be retreating into ideologies and philosophies that deny the very concept of reality itself. One
sees this in the growing strength in the humanities and social sciences of critical theories that view all
representations of the world as mere text and fiction. When the world of fact can be twisted to support
this or that side of delusion (as in astrology or parapsychology), pathology tries to appropriate what it can of the
empirical. When the world of fact manifestly vitiates the very foundations of pathological delusion, then it is the claim of facticity or reality per se that must be denied. This
is what we now may expect: the world having spoken, the intellectual class, the left academic wing of it above all, may appropriate a little postcommunist chaos to show how

In Orwell's 1984, it was the


merely relative a moral good the defeat of Stalin's heirs has been. If it does so, however, it will assail the notion of reality itself.

mark of realistic, totalitarian power to make its subjects say that all truth was not objective but political--
"a social construction," as intellectuals would say now--and that, in the specific case, 2 + 2 = 5. By 2004, making students in the
humanities and social sciences grant the equivalent of 2 + 2 = 5 will be the goal of adversarial culture. They will urge that all logical--and, one should add, inferential--inductive
truths from experience are arbitrary, mere social constructions. The West Has Indeed Survived,, So Far The ramifications of that effort will dominate the central debates of the
humanities in the generation to come. Until there is a celebration and moral accounting of the historical reality of "The Triumph of the West," that "triumph" will be

ephemeral indeed. Academic culture has replaced the simplistic model that all culture was functional, a model that indeed could not account for massive discontents or

Whole disciplines
revolutionary change, let alone for moral categories, by the yet more astonishing and absurd model that virtually all culture is dysfunctional.

now teach that propositions are to be judged by their therapeutic value rather than by their inductive link
to evidence until, in the final analysis, feeling good about saying something determines the truth-value of
what is said. Understanding human weakness, however, the West has always believed that it is precisely when we want to believe something self-gratifying that we
must erect barriers of experiment, rigor, and analysis against our self-indulgence and our propensity for self-serving error. The human ability to learn from

experience and nature, so slighted in current humanistic theory, is not merely an object of cultural transmission, let alone of
social control, but an evolutionary triumph of the species, indeed, a triumph on which our future ultimately

depends. There is nothing more desperate than helplessness, and there is no more inveterate cause of helplessness than the inability to affect and mitigate the traumas
of our lives. If the role of both acquired knowledge and the transmission and emendation of the means of

acquiring knowledge is only a "Western" concern, then it is a Western concern upon which human fate
depends. In the current academic climate of indoctrination, tendentiousness, and fantasy, the independence of critical intellect and the willingness to learn
open-mindedly from experience of a reality independent of the human will are the greatest hopes of our
civilization. Has Western civilization survived? That is, has a human relationship to the world based upon the assumption of a knowable reality, reason, and a
transcendent value of human dignity and responsibility survived? Has a will to know oneself and the world objectively survived? Has a recognition of human depravity and the
need to limit the power of men over men survived? I do not think that free men and women will abandon that hard-won shelter from chaos, ignorance, parochial tribalism,
irrationalism, and, ultimately, helplessness. Has Western civilization survived, its principle of reality justified and intact? Yes, indeed, though it requires constant defense. The
demand for perfection is antinomian, illogical, and empirically absurd. The triumph of the West is flawed but real. While everyone else around you weeps, recall Alexander

Page 271 of 1481


Ushakov, and celebrate the fall of the Soviet threat as he celebrated the fall of Grenada. Then recall how everything depends on realism in our understanding, and rejoin the
intellectual struggle.

Page 272 of 1481


A2 State/Sovereignty Bad

INTERNATIONAL GOALS CAN ONLY BE ACHIEVED BY STATES. ONLY REALISM


ESCAPES THE TYRANNY OF SMALL DECISIONS

Kenneth Waltz, Travis BFF, Neorealism and its Critics, ed. by Robert Keohane, 1986, p. 105-108

We may well notice that our behavior produces unwanted outcomes, but we are also likely to see that such instances as these are examples of what Alfred E. Kahn describes

people are victims of the tyranny of small


as large changes that are brought about by the accumulation of small decisions. In such situations

decisions, a phrase suggesting that if one hundred consumers choose option x, and this causes the market to
make decision X (where X equals 100x), it is not necessarily true that those same consumers would have voted for
that outcome if that large decision had ever been presented for their explicit consideration (Kahn 1966:523). If the
market does not present the large question for decision, then individuals are doomed to making decisions that are sensible within their narrow

contexts even though they know all the while that in making such decisions they are bringing about a result that most of them

do not want. Either that or they organize to overcome some of the effects of the market by changing its structurefor example, by bringing consumer units roughly up
to the size of the units that are making producers decisions. This nicely makes the point: So long as one leaves the structure unaffected it is

not possible for change in the intentions and the actions of particular actors to produce desirable outcomes or to avoid
undesirable ones. Structures may be changed, as just mentioned, by changing the distribution of capabilities across units. Structures may also be changed by imposing
requirements where previously people had to decide for themselves. If some merchants sell on Sunday, others may have to do so in order to remain competitive even though
most prefer a six-day week. Most are able to do as they please only if all are required to keep comparable hours. The only remedies for strong structural effects are structural
changes. Structural constraints cannot be wished away, although many fail to understand this. In every age and place, the units of self-help systems nations, corporations, or
whateverare told that the greater good, along with their own, requires them to act for the sake of the system and not for their own narrowly defined advantage. In the 1950s,
as fear of the worlds destruction in nuclear war grew, some concluded that the alternative to world destruction was world disarmament. In the 1970s, with the rapid growth of
population, poverty, and pollution, some concluded, as one political scientist put it, that states must meet the needs of the political ecosystem in its global dimensions or court
annihilation (Sterling 1974:336). The international interest must be served; and if that means anything at all, it means that national interests are subordinate to it. The problems

are found at the global level. Solutions to the problems continue to depend on national policies. What are the conditions that would
make nations more or less willing to obey the injunctions that are so often laid on them? How can they resolve the tension between pursuing their own interests and acting for
the sake of the system? No one has shown how that can be done, although many wring their hands and plead for rational behavior. The very problem, however, is that rational
behavior, given structural constraints, does not lead to the wanted results. With each country constrained to take care of itself, no one can take care of the system. A strong
sense of peril and doom may lead to a clear definition of ends that must be achieved. Their achievement is not thereby made possible. The possibility of effective action
depends on the ability to provide necessary means. It depends even more so on the existence of conditions that permit nations and other organizations to follow appropriate

. World-shaking problems cry for global solutions, but there is no global agency to provide
policies and strategies

them. Necessities do not create possibilities. Wishing that final causes were efficient ones does not make them so. Great tasks can be accomplished
only by agents of great capability. That is why states, and especially the major ones, are called on to do what is
necessary for the worlds survival. But states have to do whatever they think necessary for their own
preservation, since no one can be relied on to do it for them. Why the advice to place the international
interest above national interests is meaningless can be explained precisely in terms of the distinction between micro- and macrotheories. Among
economists the distinction is well understood. Among political scientists it is not. As I have explained, a microeconomic theory is a theory of the market built up from
assumptions about the behavior of individuals. The theory shows how the actions and interactions of the units form and affect the market and how the market in turn affects
them. A macro-theory is a theory about the national economy built on supply; income, and demand as systemwide aggregates. The theory shows how these and other
aggregates are interconnected and indicates how changes in one or some of them affect others and the performance of the economy. In economics, both micro- and
macrotheories deal with large realms. The difference between them is found not in

A macrotheory of
the size of the objects of study; hut in the way the objects of study are approached and the theory to explain them is constructed.

international politics would show how the international system is moved by system-wide aggregates. One
can imagine what some of them might beamount of world GNP, amount of world imports and exports, of deaths in war, of everybodys defense spending, and of migration,
for example. The theory would look something like a macroeconomic theory in the style of John Maynard Keynes, although it is hard to see how the international aggregates
would make much sense and how changes in one or some of them would produce changes in others. I am not saying that such a theory cannot be constructed, but only that I
cannot see how to do it in any way that might be useful. The decisive point, anyway, is that a macrotheory of international politics would lack
the practical implications of macroeconomic theory. National governments can manipulate system-wide
economic variables. No agencies with comparable capabilities exist internationally. Who would act on the possibilities of
adjustment that a macrotheory of international politics might reveal? Even were such a theory available, we would still be stuck with

nations as the only agents capable of acting to solve global problems. We would still have to revert to a micropolitical approach in
order to examine the conditions that make benign and effective action by states separately and collectively more or less likely. Some have hoped that

changes in the awareness and purpose, in the organization and ideology of states would change the quality of inter-
national life. Over the centuries states have changed in many ways, but the quality of international life has
remained much the same. States seek reasonable and worthy ends, but they cannot figure out how to reach them. The problem is not in their stupidity or ill
will, although one does not want to claim that those qualities are lacking. The depth of the difficulty is not understood until one realizes that intelligence and

goodwill cannot discover and act on adequate programs. Early in this century Winston Churchill observed that the British-German naval race promised
disaster and that Britain had no realistic choice other than to run it. States facing global problems are like individual consumers

trapped by the tyranny of small decisions.

Page 273 of 1481


Page 274 of 1481
**Calculability/Util**

Page 275 of 1481


Utilitarianism Good: 2AC (1/2)
FIRST, EXTINCTION OF THE SPECIES IS THE MOST HORRIBLE IMPACT IMAGINEABLE,
PUTTING RIGHTS FIRST IS PUTTING A PART OF SOCIETY BEFORE THE WHOLE

Schell 1982
(Jonathan, Professor at Wesleyan University, The Fate of the Earth, pages 136-137 uw//wej)

Implicit in everything that I have said so far about the nuclear predicament there has been a perplexity that I would now like to take up explicitly, for it leads, I
predicament. I have pointed out that our
believe, into the very heart of our response-or, rather, our lack of response-to the

species is the most important of all the things that, as inhabitants of a common world, we
inherit from the past generations, but it does not go far enough to point out this superior
importance, as though in making our decision about ex- tinction we were being asked to
choose between, say, liberty, on the one hand, and the survival of the species, on the other.
For the species not only overarches but contains all the benefits of life in the common world , and

to speak of sacrificing the species for the sake of one of these benefits involves one in the
absurdity of wanting to de- stroy something in order to preserve one of its parts, as if one were
to burn down a house in an attempt to redecorate the living room, or to kill someone to improve his character. ,but
even to point out this absurdity fails to take the full measure of the peril of extinction, for mankind is not some invaluable object that lies outside us and that we
must protect so that we can go on benefiting from it; rather, it is we ourselves, without whom everything there is loses its value. To say this is another way of
saying that extinction is unique not because it destroys mankind as an object but because it destroys mankind as the source of all possible human subjects,
and this, in turn, is another way of saying that extinction is a second death, for one's own individual death is the end not of any object in life but of the subject
that experiences all objects. Death, how- ever, places the mind in a quandary. One of-the confounding char- acteristics of death-"tomorrow's zero," in
Dostoevski's phrase-is that, precisely because it removes the person himself rather than something in his life, it seems to offer the mind nothing to take hold of.
One even feels it inappropriate, in a way, to try to speak "about" death at all, as. though death were a thing situated some- where outside us and available for
objective inspection, when the fact is that it is within us-is, indeed, an essential part of what we are. It would be more appropriate, perhaps, to say that death,
as a fundamental element of our being, "thinks" in us and through us about whatever we think about, coloring our thoughts and moods with its presence

throughout our lives.

SECOND, SURVIVAL OF POLITICAL ORDER KEY TO ETHICS

Stenlisli, 2003 (Pace nr.1 accessed onlinehttp://www.pacem.no/2003/1/debatt/stensli/ )

The debate on political realism, a set of ontological assumptions about international politics, has been a central theme in international relations over the past 40 years. Many
scholars and politicians have wrestled over the question of the limitations and insights of realism. Still, realism seems very much alive today, one reason perhaps being that the
value of realism as an analytical tool seems to become more relevant to policymakers in times of crises. In turn, such changes cause further debate among realists and their
critics. In PACEM 5:2 (2002), Commander Raag Rolfsen(1) in practise argues that we are in need of a new framework for analysing international politics. According to Rolfsen,
A situation characterized by globalisation, democratisation and a new sense of shared vulnerability demands a novel theoretical framework for world politics. Rolfsen`s aim is
indeed ambitious, but his state of departure is surprising: political realism cannot provide this framework because, again according to Rolfsen, it was developed in an
undemocratic environment.(2) Thus, we are not far from concluding that realism is corrupted and that realists are conspicuous people.(3) This bold proclamation illuminates the
front between idealism and realism in a manner that is not typical of Norwegian academic discourses on international relations. Rolfsen has delivered a substantial and
refreshing article. It is of such originality and importance that it deserves to be debated and criticised, which is no evident feature in contributions on world politics in Norway.
Having said that, my motivation to engage in such a debate does not spring from a wholehearted embracement of realism. Rather, its source is the belief that a theory of
foreign policy cannot do without significant elements of realism. Traditional security policy can never remove our vulnerability. At this point there simply is no disagreement
between realists and idealists. However, security has an instrumental value in ensuring other ends. Thus, acknowledging our vulnerability does not remove the value and
importance of security as phenomenon and concept.(4) In this article, I will discuss whether the effort to construct a new security concept possibly can succeed when it
simultaneously becomes an attack on political realism (PR). Rolfsen undoubtedly deals some blows against Hans Morgenthaus Theory of International Politics, although the
same points have been made by others before him.(5) Indeed, political realism has to be anchored to ideals and visions of desired end states beyond its basic assumptions,(6)
but my main line of argument is that any attempt at establishing a basis for ethical conduct in politics is bound to remain a purely theoretical construction without empirical

since the existence of a polity is a


relevance if it is not mixed with a sound and thorough understanding of PR. The reason simply is, that

precondition for thinking about, implementing and evaluating policies in other areas, politics based on
realism is required in the first place in order to secure the polity. There can be no democracy without a
modern state, and no state without a minimum level of security through a monopoly of violence. Herein lies a significant aspect of what makes the state legitimate to
its citizens. In this way, one can even claim that all normative evaluations and - theories implicitly rest on minimum requirements both

to the practises and theoretical considerations of realism.(7) Indeed, one should at least question whether attempts at denying the
empirical relevance of PR could lead us into paralysis or hypocrisy. The latter can even serve, unintentionally to be sure, as a basis for demonising opponents, thus functioning
as a (moral) sentiment that forms the basis of a more hawkish or brutal conduct in international crisis than is necessary. The prudence found in Morgenthau should not be seen
as cynical or a-ethical, but rather as a configuration of thought that should balance our aspirations to fulfil what Morgenthau calls the ultimate aims of politics. The central
political problem is exactly how to translate these aspirations (like democracy and human rights) into feasible and efficient decisions. But in order to pursue these important
goals, the ability to use power, be it hard or soft, is required.

Page 276 of 1481


Page 277 of 1481
Page 278 of 1481
Utilitarianism Good: 2AC (2/2)
DEONTOLOGY LOCKS US INTO A DEADLOCK WHEN VALUES CONFLICT, ONLY WAY
TO RESOLVE THAT IS BY USING CONSEQUENTIALISM

Person, 1997
(lngmar. Lund University. Three Methods of Ethics: a debate. Eds. Baron, Marcia, Philip Petit, and Michael
Stole. Pg 13-14. uw//wej)

Now the natural rights theorist maintains, of course, that. the presence of a right is such a relevant factor, or reason, that may justify departing from the goal
of fulfilment maximization. In Ronald Dwor. kin's phrase, rights could in this way `trump' the pursuit of maximal fulfilment. A right to M provides a reason for
holding that one morally should have M even if this is at odds with the goal mentioned. I do not say that it ensures that one should have M because the rights
theorist may like to impose a limit on the weight of rights, on how great the loss of fulfilment overall may be if a right is not to be outweighed. Suppose that my
hair has a unique healing quality: thousands of terminally ill patients could be saved if a couple of strands are removed and made into a medicine. What should
the rights theorist say if I none the less refuse to have these strands removed? Surely, something like this: the suffering caused by respecting my right to my

there is a limit on the weight of my right, on its


strands of hair is so great that we are morally justified in violating the right. But then

capacity to restrain maximiza- tion; a right provides a moral reason that can be outweighed. As an aside, note that, like
the limit on the extension of rights, this limit would seem to have to be based on consequentialist considera-

tions, on weighing the frustration and confusion occasioned by infring- ing our deep-seated intuitions about rights against the frustration and suffering
caused by respecting them. Thus, when It comes to the precise weight of rights, no less than their extension,

we see that it cannot be fixed unless we transcend the natural rights framework in favour of a
consequentialist one.

UTILITY CALCULUS ALLOWS ACTION, MORAL DOGMATISM FREEZES US INTO


INACTION

Smart, 1973
(J.J.C prof. of philosophy, Australian riatibual university. Utilitarianism: For and

Against uw//wej)

lf we are able to take account of probabilities in our ordinary prudential decisions it seems idle to say that in the field of ethics, the
field of our universal and humane atti- tudes, we cannot do the same thing, but must rely on some dogmatic morality, in short on

some set of rules or rigid criteria, Maybe sometimes we just will be unable to say whether we prefer for

humanity an improbable great advantage or a probable small advantage, and in these cases perhaps we shall
have to toss a penny to decide what to do. Maybe we have not any precise methods for deciding what to do, but then our imprecise

methods must just serve their turn. We need not on that account be driven into authori.-
tarianism, dogmatism or romanticism.

Page 279 of 1481


Utilitarianism Good: 1AR

First, extend our Jonathan Schell evidence, he explains that accepting extinction to
uphold rights is like burning down a house to remodel the living room, rights are a
result of human society and accepting the destruction of that society to uphold a
right is going too far and ultimately self-defeating.

Second, Stenlisli indicates that survival of the political order is a precondition of all
other values. The alternative is impossible without a stable security framework.

Third, LIFE IS KEY TO ETHICS

Diana Meyers, prof of Philosophy @ Connecticut University, 1985 Inalienable Rights, p. 54

The right to life prohibits other persons from killing the person who possesses the right and allows this person to defend himself if he is
attacked. It is obvious that a person cannot be a moral agent unless he is alive (at least, not within the moral sphere in which we presently find

ourselves), and so it is also obvious that this right protects something essential to moral agency. But it is doubtful that it is always
supererogatory when it is appropriate for a person to sacrifice his life for the benefit of others. Two representative cases can be adduced to call this claim into question: I) a
soldier has a duty to follow orders to participate in battles if her army is involved in a just war, and 2) a citizen may have a duty to join her countrys army in wartime.

Fourth, Ingmar Person explains even rights must be weighed against each other,
but that deontology doesnt allow preferential treatment of one right over
another without resorting to consequentialism, making consequentialism
inevitable, or action impossible.

Fifth, Smart in 73 illustrates how consequentialism avoids dogmatic action, making


it flexible in dealing with different situations

UTILITARIANISM IS THE ONLY ALTERNATIVE TO EXTINCTION, OUTWEIGHING


RIGHTS

Ratner 84
[Leonard G., Legion Lex Prof. Law @ USC, The Utilitarian Imperative: Autonomy, Reciprocity, and
Evolution, 12 Hofstra L. Rev. 723, Spring, LN//uwyo-ajl]

Page 280 of 1481


The search for the ought is a search for the goals of human behavior. Underlying the ought of every goal is an implicit description of reality that predicts the consequences for
humans of compliance or noncompliance with the ought. n49 Humans choose the goals. n50 And the perceived accuracy of the description, along with the perceived value of
the consequences predicted by the description, influence the choice. Ought and is thus coalesce.

The goal of enhanced human need/want fulfillment implies that such enhanced fulfillment is possible and
will facilitate long-run human existence.Goals that facilitate human existence are persistently chosen by
most humans, because human structure and function have evolved and are evolving to facilitate such
existence. The decisionmaking organism is structured to generally prefer survival, although some may trade long-term
existence for short-term pleasure, and physiological malfunction or traumatic experience may induce the preference of a few for personal nonsurvival. Intermediate

human goals change with human structure and function; long-run human survival remains the ultimate
human goal as long as there are humans.

Page 281 of 1481


Page 282 of 1481
Calculability Good: 2AC (1/2)
FIRST, FAILURE TO CALCULATE ALLOWS TOTALITARIANISM BY DENYING
INSTITUTIONAL RESPONSIBILITY

Campbell 98
[David, Intl Relations Prof @ UM, National Deconstruction: Violence, Identity, and Justice in Bosnia,
Minneapolis: University of Minnesota Press, 1998, 186]

The undecidable within the decision does not, however, prevent the decision nor avoid its urgency. As Derrida observes, a just decision is always required
the pursuit of infinite information and
immediately, right away. This necessary haste has unavoidable consequences because

the unlimited knowledge of conditions, rules or hypothetical imperatives that could justify it are unavailable in the
crush of time. Nor can the crush of time be avoided, even by unlimited time, because the moment of decision as such always remains a finite moment
of urgency and precipitation. The decision is always structurally finite, it aalways marks the interruption of the juridico- or ethico- or politico-cognitive
deliberation that precedes it, that must precede it. That is why, invoking Kierkegaard, Derrida, declares that the instant of decision is a madness.

The finite nature of the decision may be a madness in the way it renders possible the impossible, the infinite character of justice, but Derrida argues for the
necessity of this madness. Most importantly, Derrida argues for the necessity of this madness. Most importantly, although Derridas argument concerning the
decision has, to this pint, been concerned with an account of the procedure by which a decision is possible, it is with respect to the ncessity of the decision that
Derrida begins to formulate an account of the decision that bears upon the content of the decision. In so doing, Derridas argument addresses more directly
more directly, I would argue than is acknowledged by Critchley the concern that for politics (at least for a progressive politics) one must provide an account of
the decision to combat domination.

That undecidability resides within the decision, Derrida argues, that justice exceeds law and calculation, that the unpresentable exceeds the
determinalbe cannot and should not serve as alibi for staying out of juridico-political battles, within an
institution or a state, or between institutions or states and others. Indeed, incalculable justice requires us to
calculate. From where do these insistences come? What is behind, what is animating, these imperatives? It is both the character of infinite justice as a
heteronomic relationship to the other, a relationship that because of its undecidability multiplies responsibility, and the fact that left to itself, the

incalculable and given (donatrice) idea of justice is always very close to the bad, even to the worst, for it can always be
reappropriated by the most perverse calculation. The necessity of calculating the incalculable thus responds to a duty
a duty that inhabits the instant of madness and compels the decision to avoid the bad, the perverse calculation, even the worst. This is the duty that also

dwells with deconstructive thought and makes it the starting point, the at least necessary condition, for

the organization of resistance to totalitarianism in all its forms. And it is a duty that responds to practical political concerns
when we recognize that Derrida names the bad, the perverse, and the worst as those violences we recognize all too well without yet having thought them
through, the crimes of xenophobia, racism, anti-Semitism, religious or nationalist fanaticism.

SECOND, EVEN IF WE OBSCURE THE INCALCULABLE, WE HAVE AN ETHICAL


RESPONSIBILITY TO CALCULATE DEATH BECAUSE ITS OUR ONLY MEANS OF
FIGHTING INJUSTICE

Santilli 2003
[Paul C., Siena College, Radical Evil, Subjection, and Alain Badious Ethic of the Truth Event, World Congress of the International Society
for Universal Dialogue, May 18-22, www.isud.org/papers/pdfs/Santilli.pdf, acc. 9-24-06//uwyo-ajl]

From the standpoint of an ethics of subjection there is even something unnecessary or superfluous about the void of suffering in the subject bearers of evil. For Levinas, the
return to being from the ethical encounter with the face and its infinite depths is fraught with the danger the subject will reduce the other to a "like-me," totalizing and violating
the space of absolute alterity. As Chalier puts it, "Levinas conceives of the moral subject's awakening, or the emergence of the human in being, as a response to that pre-
originary subjection which is not a happenstance of being." But if there really is something inaccessible about suffering itself, about the 'other' side of what is manifestly finite,
subjected, and damaged, then to a certain extent it is irrelevant to ethics, as irrelevant as the judgment of moral progress in the subject-agent. Let me take the parent-child
relation again as an example. Suppose the child to exhibit the symptoms of an illness. Are not the proper "ethical" questions for the parent to ask questions of measure and
mathematical multiples: How high is the fever? How long has it lasted? How far is the hospital? Can she get out of bed? Has this happened before? These are the questions of

Ethically our response to the needs of


the doctor, the rescue squads and the police. They are questions about being, about detail, causes and effects.

must be reduced to a positivity simply because we have access to nothing but the symptoms, which are like mine.

Page 283 of 1481


Our primary moral responsibility is to treat the symptoms that show up in being, not the radically other
with whom I cannot identify. Say we observe someone whose hands have been chopped off with a machete. How would we characterize this? Would it not
be slightly absurd to say, "He had his limbs severed and he suffered," as though the cruel amputation were not horror enough. Think of the idiocy in the common platitude: "She
died of cancer, but thank God, she did not suffer", as though the devastating annihilation of the human by a tumor were not evil itself. For ethics, then, the only suffering that
matters are the visible effects of the onslaught of the world. All other suffering is excessive and inaccessible. Therefore, it is in being, indeed in the midst of the most elemental
facts about ourselves and other people, that we ethically encounter others by responding to their needs and helping them as best we can

by identifying being and not pretending that we know any thing about suffering, other than it is a
It is precisely

hollow in the midst of being, that we can act responsibly. What worries me about Levinas is that by going beyond being to what he
regards as the ethics of absolute alterity, he risks allowing the sheer, almost banal facticity of suffering to be swallowed in the infinite depths of transcendence. Indeed, it seems
to me that Levinas too often over emphasizes the importance of the emergence of the subject and the inner good in the ethical encounter, as though the point of meeting the
suffering human being was to come to an awareness of the good within oneself and not to heal and repair. I agree with Chalier's observation that Levinas's "analyses adopt the
point of view of the moral subject, not that of a person who might be the object of its solicitude." Ethics has limits; there are situations like the Holocaust where to speak of a

an ethics that would be oriented to the vulnerabilities of the subjected


moral responsibility to heal and repair seems pathetic. But

needs to address the mutilation, dismemberment, the chronology of torture,


(which are others, of course, but also myself)

the numbers incarcerated, the look of the bodies, the narratives, the blood counts, the mines knives, machetes, and poisons. Evil really is all that. When
the mind does its work, it plunges into being, into mathematical multiples and starts counting the cells, the
graveyards, and bullet wounds. Rational practical deliberation is always about the facts that encircle the void inaccessible to deliberation and practical reason.

Page 284 of 1481


Page 285 of 1481
Calculability Good: 2AC (2/2)
THIRD, INFINITE JUSTICE REQUIRES CALCULATION

Jacques Derrida, in Deconstruction and the Possibility of Justice, Drucilla Cornell, ed, 92, p. 28-9.

, that the unpresentable exceeds the determinable cannot and should not serve as an
That justice exceeds law and calculation

alibi for staying out of juridico-political battles, within an institution or a state or between institutions or states and others. Left to
itself, the incalculable and giving (donatrice) idea of justice is always very close to the bad, even to the worst for it can
always be reappropriated by the most perverse calculation. It's always possible. And so incalculable justice requires
us to calculate. And first, closest to what we associate with justice, namely, law, the juridical field that one cannot isolate within sure frontiers, but also in all the fields
from which we cannot separate it, which intervene in it and are no longer simply fields: ethics, politics, economics, psycho-sociology, philosophy, literature, etc. Not only

must we calculate, negotiate the relation between the calculable and the incalculable, and negotiate without the sort of rule that wouldn't have to be reinvented there
where we are cast, there where we find ourselves; but we must take it as far as possible, beyond the place we find ourselves and beyond the -already
identifiable zones of morality or politics or law, beyond the distinction between national and international, public and private, and so on. This requirement does not properly
belong either to justice or law. It only belongs to either of these two domains by exceeding each one in the direction of the other. Politicization, for example, is interminable even
if it cannot and should not ever be total. To keep this from being a truism or a triviality, we must recognize in it the following consequence: each advance in politicization obliges

This was true for example in the


one to reconsider, and so to reinterpret the very 4bundations of law such as they had previously been calculated or delimited.

Declaration of the Rights of Man, in the abolition of slavery, in all the emancipatory battles that remain and will
have to remain in progress, everywhere in the world, for men and for women. Nothing seems to me less outdated than the

classical emancipatory ideal. We cannot attempt to disqualify it today, whether crudely or with sophistication, at least not without
treating it too lightly and forming the worst complicities. But beyond these identified territories of juridico-politicization on the grand geopolitical scale,
beyond all self-serving interpretations, beyond all determined and particular reappropriations of international law, other areas must constantly open up that at first can seem like
secondary or marginal areas. This marginality also signifies that a violence, indeed a terrorism and other forms of hostage-taking are at work (the examples closest to us would
be found in the area of laws on the teaching and practice of languages, the legitimization of canons, the military use of scientific research, abortion, euthanasia, problems of
organ transplant, extra-uterine conception; bio-engineering, medical experimentation, the social treatment of AIDS, the macro- or micro-politics of drugs, the homeless, and so
on, without forgetting, of course, the treatment of what we call animal life, animality. On this last problem, the Benjamin text that I'm coming to now shows that its author was not
deaf or insensitive to it, even if his propositions on this subject remain quite obscure, if not quite traditional).

FOURTH, FOCUS ON THE INCALCULABLE IS PARALYZING

Mithcell Stephens, chairman of the journalism and mass-communication department at NYU, New York Times
Magazine, January 23, 1994, http://www.nyu.edu/classes/stephens/Jacques%20Derrida%20-%20NYT%20-
%20page.htm, accessed 11/7/02

Deconstruction had another problem: the widely held belief that reading in search of contradictions and
misunderstandings is foolish, if not insidious. John Updike has attacked what he has called
"deconstruction's fatiguing premise that art has no health in it." Critics on the right are outraged by the
implication that there is something tangled or "impossible" about such important concepts as "reality"
and "truth," which they are committed to extricating from the grip of quotation marks. "Derrida's
influence has been disastrous," Roger Kimball, a conservative critic and author of "Tenured Radicals,"
proclaims. "He has helped foster a sort of anemic nihilism, which has given imprimaturs to squads of
imitators who no longer feel that what they are engaged in is a search for truth, who would find that
notion risible." Though Derrida considers himself a member of the democratic left, critics on the left
haven't necessarily been any kinder. Some have charged that all this emphasis on the "impossible," on
what we can't know, threatens to leave us paralyzed, "standing" -- like poor Bartleby -- "mute and
solitary" before the world's injustices.

Page 286 of 1481


Page 287 of 1481
A2 Tyranny of Survival (1/2)
FIRST, WE OUTWEIGH EVEN IF SURVIVAL RHETORIC CAUSES TYRANNY, THEY
HAVENT DISPROVEN OUR TRUTH CLAIMS. WE STILL PREVENT EXTINCTION

SECOND, NO LINK THE NAZIS ALSO WORE T-SHIRTS, THAT DOESNT PROVE OUR
USE OF SURVIVAL CAUSES OPPRESSION

THIRD, IRREVERSIBLE CHANGE JUSTIFIES SURVIVAL RHETORIC

Daniel Callahan, Institute of Society, Ethics, and the Life Sciences, Hastings-on-Hudson, New York, The Tyranny of

Survival, 1973, p. 106-7

But let us assume that the stage of a dark cloud on some distant horizon has been passed, and the
evidence is good that serious deterioration has already set in. At what point in the deterioration should
survival become a priority? Observe that I said a priority; it should never become the priority if that means
the sacrifice of all other values. But there are surely conditions under which it could become a priority,
and a very high one. The most important of those conditions would be the existence of evidence that
irreversibility was beginning to set in, making it increasingly impossible to return to the original
conditions. That situation, combined with visible evidence of serious present deterioration-for instance,
an urgent need to develop compensatory technologies-would warrant a focus on survival; for that is just
what would be at stake.

FOURTH, EXTINCTION OF THE SPECIES IS THE MOST HORRIBLE IMPACT


IMAGINEABLE, PUTTING RIGHTS FIRST IS PUTTING A PART OF SOCIETY BEFORE THE
WHOLE

Schell 1982
(Jonathan, Professor at Wesleyan University, The Fate of the Earth, pages 136-137 uw//wej)

Implicit in everything that I have said so far about the nuclear predicament there has been a perplexity that I would now like to take up explicitly, for it leads, I
predicament. I have pointed out that our
believe, into the very heart of our response-or, rather, our lack of response-to the

species is the most important of all the things that, as inhabitants of a common world, we
inherit from the past generations, but it does not go far enough to point out this superior
importance, as though in making our decision about ex- tinction we were being asked to
choose between, say, liberty, on the one hand, and the survival of the species, on the other.
For the species not only overarches but contains all the benefits of life in the common world , and

to speak of sacrificing the species for the sake of one of these benefits involves one in the
absurdity of wanting to de- stroy something in order to preserve one of its parts, as if one were
to burn down a house in an attempt to redecorate the living room, or to kill someone to improve his character. ,but

Page 288 of 1481


even to point out this absurdity fails to take the full measure of the peril of extinction, for mankind is not some invaluable object that lies outside us and that we
must protect so that we can go on benefiting from it; rather, it is we ourselves, without whom everything there is loses its value. To say this is another way of
saying that extinction is unique not because it destroys mankind as an object but because it destroys mankind as the source of all possible human subjects,
and this, in turn, is another way of saying that extinction is a second death, for one's own individual death is the end not of any object in life but of the subject
that experiences all objects. Death, how- ever, places the mind in a quandary. One of-the confounding char- acteristics of death-"tomorrow's zero," in
Dostoevski's phrase-is that, precisely because it removes the person himself rather than something in his life, it seems to offer the mind nothing to take hold of.
One even feels it inappropriate, in a way, to try to speak "about" death at all, as. though death were a thing situated some- where outside us and available for
objective inspection, when the fact is that it is within us-is, indeed, an essential part of what we are. It would be more appropriate, perhaps, to say that death,
as a fundamental element of our being, "thinks" in us and through us about whatever we think about, coloring our thoughts and moods with its presence

throughout our lives.

Page 289 of 1481


Page 290 of 1481
A2 Tyranny of Survival (2/2)
FIFTH, *INDIVIDUALISM IS ALSO TYRANNY: CALLAHAN ARGUES AGAINST
ABSOLUTISM, NOT FOR CATEGORICAL REJECTION OF ARGUMENTS APPEALING TO
SURVIVAL.

Daniel Callahan, Institute of Society, Ethics, and the Life Sciences, Hastings-on-Hudson, New York, The Tyranny of

Survival, 1973, p. 134-5

The irony with which Rieff analyzes psychological man makes evident his distrust and final rejection. But
Rieff offers little to put in its place, in great part because he does not offer a positive view of culture which
would strike a good bargain between the demands of the individual and of the culture. No more than
Freud can he offer the foundation for a social ethic which would integrate a range of values in a way that
would enable the individual and civilization to mutually behave toward each other in ways which
respected the requirements of each. What Rieff has done is to lay bare the hubris and folly of an
individualism run amuck, seeking a final break from all cultural restraints. But having rejected that form of
individualism, what are the alternatives? Not an ethic of survival, which would manage to keep the
individual in line at the price of a final victory of the community over the individual, resolving all tensions,
ending the possibility of a mutual respect. If the tyranny of individualism, inherent in the mode of life of
psychological man, presents only the prospect of a culture of self-contained human monads occasionally
jostling each other, the tyranny of survival projects a world where the individual is effaced altogether.
Both tyrannies are proof against any kind of social ethic, for both dissolve that necessary dialectic
between individual and community which is the prime requirement of such an ethic. A failure in the first
place to posit the validity of both individual and community will make it impossible in the end to combat
the virulence of individualism and survivalism, a virulence which not paradoxically draws them closer
together with every advance in technology and affluence.

The first step, then, in constructing a social ethic for technological societies is to reject the polarities of the
analytic attitude, on the one hand, and the species attitude, on the other. The analytic attitude dissolves
all of life into a cunning detachment of individual from community, providing the former with the
psychological weapons to keep other human beings at bay. The species attitude, seeking only survival and
perpetuation, provides no less effective weapons for keeping human beings at bay, only this time in the
name of a future made safe for the future. The great threat to the possibility of a social ethic for a
technological society is less the absence of all values than the triumph of one value over all others. Both
individualism and survival are struggling to achieve that position, with a striking degree of success.
Nothing is more important than to deny both the triumph they seek.

SIXTH, SURVIVAL AS THE HIGHEST VALUE CAN'T JUST BE REPLACED WITH


UNCRITICAL INDIVIDUAL FREEDOM AS THE HIGHEST VALUE.

Daniel Callahan, Institute of Society, Ethics, and the Life Sciences, Hastings-on-Hudson, New York, The Tyranny of

Survival, 1973, p. 57-8

Page 291 of 1481


Moreover, as I will develop more fully in later chapters, technological societies impose both a tyranny of
survival and a tyranny of individualism. They impose the former because, in times of stress, their extreme
fragility (stemming from the high base of expectation they engender and the high degree of total control
their complexity demands) is instantly and terrorizingly apparent, creating a natural environment for an
obsessive fear of annihilation, i.e., a tyranny of survival. They impose the latter-monomaniacal
individualism-because only the privatized life seems viable or endurable in the midst of a system which
presents itself as impersonal and uncontrollable. Thus is intensified the tyranny of individualism, which
demands that each person create his or her own world ex nihilo: self-direction, self-realization,
self-fulfillment-self, self, self.

Page 292 of 1481


Page 293 of 1481
A2 Ontology First: 2AC
PREVENTING VIOLENCE COMES BEFORE ONTOLOGY

Arnold Davidson, 1989


Critical Inquiry, Winter, p. 424

I understand Levinas work to suggest another path to the recovery of the human, one that leads through
or toward other human beings:

The dimension of the divine opens forth froni the human face. Hence metaphysics is enacted where the
social relation is enacted in our relations with men. . . . The Other is not the incarnation of God, but
precisely by his face, in which he is disincarnate, is the manifestation of the height in which God is
revealed. It is our relations with men .. . that give to theological concepts the sole signification they admit
of.35

Levinas places ethics before ontology by beginning with our experience of the human face; and, in a clear
reference to Heideggers idolatry of the village life of peasants, he associates himself with Socrates, who
preferred the city where he encountered men to the country with its trees. In his discussion of skepticism
and the problem of others, Cavell also aligns himself with this path of thought, with the recovery of the
finite human self through the acknowledgment of others:

As long as God exists, I am not alone. And couldnt the other suffer the fate of God? ... I wish to
understand how the other now bears the weight of God, shows me that I am not alone in the universe.
This requires understanding the philosophical problem of the other as the trace or scar of the departure
of God. [CR, p. 47Oj

The suppression of the other, the human, in Heideggers thought accounts, I believe, for the absence, in
his writing after the war, of the experience of horror. Horror is always directed toward the human; every
object of horror bears the imprint of the human will.38 So Levinas can see in Heideggers silence about
the gas chambers and death camps a kind of consent to the horror.39 And Cavell can characterize Nazis
as those who have lost the capacity for being horrified by what they do.4 Where was Heideggers
horror? How could he have failed to know what he had consented to?

Hannah Arendt associates Heidegger with Paul Valerys aphorism, Les evenments ne sont que lcume
des choses (Events are but the foam of things).4 I think one understands the source of her intuition.
The mass extermination of human beings, however, does not produce foam, but dust and ashes; and it is
here that questioning must stop.

Page 294 of 1481


Page 295 of 1481
A2 Your Impact is Inevitable:
2AC
AND, ALL OF THEIR INEVITABIILTY ARGUMENTS ARE QUALITATIVELY DIFFERENT
THAN OUR 1AC SCENARIOS. THEY REFER TO EXTREMELY LOW LEVEL WARS THAT
DONT CAUSE ANNIHILATION. ANY BIGGER IMPACT IS PURE RHETORIC, WHEREAS
WE HAVE EV THAT A BREAKDOWN OF THE REALIST BALANCE CAUSES GREAT
POWER WARS

ALSO, WARFARE IS AT ITS LOWEST EBB IN HUMAN HISTORY

Gregg Easterbrook, journalist, The End of War? THE NEW REPUBLIC, May 30, 2005, p. 18.

War has entered a cycle of decline. Combat in Iraq and in a


But here is something you would never guess from watching the news:

few other places is an exception to a significant global trend that has gone nearly unnoticed--namely that, for about 15 years, there
have been steadily fewer armed conflicts worldwide. In fact, it is possible that a person's chance of dying because of
war has, in the last decade or more, become the lowest in human history. Five years ago, two academics--Monty Marshall, research director at the
Center for Global Policy at George Mason University, and Ted Robert Gurr, a professor of government at the University of Maryland--spent months compiling all available data on the
frequency and death toll of twentieth-century combat, expecting to find an ever-worsening ledger of blood and destruction. Instead, they found, after the terrible years of World Wars I and II,
a global increase in war from the 1960s through the mid-'80s. But this was followed by a steady, nearly uninterrupted decline beginning in 1991. They also found a steady global rise since the
mid-'80s in factors that reduce armed conflict--economic prosperity, free elections, stable central governments, better communication, more "peacemaking institutions," and increased
international engagement. Marshall and Gurr, along with Deepa Khosla, published their results as a 2001 report, Peace and Conflict, for the Center for International Development and Conflict
Management at the University of Maryland. At the time, I remember reading that report and thinking, "Wow, this is one of the hottest things I have ever held in my hands." I expected that
evidence of a decline in war would trigger a sensation. Instead it received almost no notice.

AND, CURRENT GLOBAL TRENDS ARE AGAINST WARFARE

Gregg Easterbrook, journalist, The End of War? THE NEW REPUBLIC, May 30, 2005, p. 18.

. War "may well


In his 1993 book, A History of Warfare, the military historian John Keegan recognized the early signs that combat and armed conflict had entered a cycle of decline

be ceasing to commend itself to human beings as a desirable or productive, let alone rational, means of
reconciling their discontents," Keegan wrote. Now there are 15 years of positive developments supporting the
idea. Fifteen years is not all that long. Many things could still go badly wrong; there could be ghastly surprises in store. But, for the
moment, the trends have never been more auspicious: Swords really are being beaten into plowshares and spears into pruning hooks. The
world ought to take notice.

Page 296 of 1481


Page 297 of 1481
A2 Your Impact is Inevitable:
1AR
AND, EXTEND THE 2AC ANSWERS TO THE INEVITABILITY DEBATE.

FIRST, THEIR EV ONLY SHOWS THAT LOW SCALE, REGIONAL SKIRMISHES ARE
INEVITABLE, NOT THE GREAT POWER WARS OF THE 1AC. THEIR TRANSITION IS THE
ONLY RISK OF AN IMPACT

SECOND, WERE RUNNING CIRCLES AROUND THEM ON THE UNIQUENESS


QUESTION. EASTERBOOK 2005 SHOWS THAT GLOBAL CONFLICT IS AT ITS LOWEST
IN HISTORY

THIRD, YOU PUT EXTINCTION FIRST. THE RISK OF A NUCLEAR WAR, WHICH
SHATTERS THE MORAL FRAME. CROSS-APPLY SCHELL 82

FOURTH, WAR IS DOWN

Gregg Easterbrook, journalist, The End of War? THE NEW REPUBLIC, May 30, 2005, p. 18.

Of course, 2001 was the year of September 11. But, despite the battles in Afghanistan, the Philippines, and elsewhere that were ignited by Islamist terrorism and the West's response, a
second edition of Peace and Conflict, published in 2003, showed the total number of wars and armed conflicts continued to decline. A third edition of the study, published last week, shows

despite the invasion of Iraq and other outbreaks of fighting, the overall decline of war continues. This
that,

even as the global population keeps rising, which might be expected to lead to more war, not less.

Page 298 of 1481


A2 Your Impact = Bare Life: 2AC (1/3)

FIRST, NO LINK We dont ascribe a quantitative value to someones life, but only
say that we shouldnt forcibly allow them to die in a horrific way, allowing them
the option to find their own value.

SECOND, VALUE TO LIFE IS SUBJECTIVE MUST ALLOW PEOPLE THE CHOICE TO


FIND THEIR OWN VALUE AT ALL COSTS AND RESIST EXTERNAL ATTEMPTS TO
DESTROY IT

Schwartz 2004
[A Value to Life: Who Decides and How? www.fleshandbones.com/readingroom/pdf/399.pdf]

Those who choose to reason on this basis hope that if the quality of a life can be measured then the answer to whether that life has value to the
individual can be determined easily. This raises special problems, however, because the idea of quality involves a value judgement, and value
judgements are, by their essence, subject to indeterminate relative factors such as preferences and dislikes. Hence, quality of life is difficult to
measure and will vary according to individual tastes, preferences and aspirations. As a result,

no general rules or principles can be asserted that would simplify decisions about the
value of a life based on its quality. Nevertheless, quality is still an essential criterion in
making such decisions because it gives legitimacy to the possibility that rational, autonomous
persons can decide for themselves that their own lives either are worth, or are no longer worth,
living. To disregard this possibility would be to imply that no individuals can legitimately make such value judgements about their own lives and,
if nothing else, that would be counterintuitive. 2 In our case, Katherine Lewis had spent 10 months considering her decision before concluding
that her life was no longer of a tolerable quality. She put a great deal of effort into the decision and she was competent when she made it. Who
would be better placed to make this judgement for her than Katherine herself? And yet, a doctor faced with her request would most likely be
uncertain about whether Katherines choice is truly in her best interest, and feel trepidation about assisting her. We need to know which
considerations can be used to protect the patients interests. The quality of life criterion asserts that there is a difference between the type of
life and the fact of life. This is the primary difference between it and the sanctity criterion discussed on page 115. Among quality of life
considerations rest three assertions: 1. there is relative value to life 2. the value of a life is determined subjectively 3. not all lives are of equal
value. Relative value The first assertion, that life is of relative value, could be taken in two ways. In one sense, it could mean that the value of a
given life can be placed on a scale and measured against other lives. The scale could be a social scale, for example, where the contributions or
potential for contribution of individuals are measured against those of fellow citizens. Critics of quality of life criteria frequently name this as a
potential slippery slope where lives would be deemed worthy of saving, or even not saving, based on the relative social value of the individual
concerned. So, for example, a mother of four children who is a practising doctor could be regarded of greater value to the community than an
unmarried accountant. The concern is that the potential for discrimination is too high. Because of the possibility of prejudice and injustice,
supporters of the quality of life criterion reject this interpersonal construction in favour of a second, more personalized, option. According to
this interpretation, the notion of relative value is relevant not between individuals but within the context of one persons life and is measured
against that persons needs and aspirations. So Katherine would base her decision on a comparison between her life before and after her illness.
The value placed on the quality of a life would be determined by the individual depending on whether he or she believes the current state to be
relatively preferable to previous or future states and whether he or she can foresee controlling the circumstances that make it that way. Thus,
the life of an athlete who aspires to participate in the Olympics can be changed in relative value by an accident that leaves that person a
quadriplegic. The athlete might decide that the relative value of her life is diminished after the accident, because she perceives her desires and
aspirations to be reduced or beyond her capacity to control. However, if she receives treatment and counselling her aspirations could change
and, with the adjustment, she could learn to value her life as a quadriplegic as much or more than her previous life. This illustrates how it is
possible for a person to adjust the values by which they appraise their lives. For Katherine Lewis, the decision went the opposite way and she
decided that a life of incapacity and constant pain was of relatively low value to her. It is not surprising that the most vociferous protesters
against permitting people in Katherines position to be assisted in terminating their lives are people who themselves are disabled. Organizations
run by, and that represent, persons with disabilities make two assertions in this light. First, they claim that accepting that Katherine Lewis has a
right to die based on her determination that her life is of relatively little value is demeaning to all disabled people, and implies that any life with
a severe disability is not worth Write a list of three things that make living. Their second assertion is that with proper help, over time Katherine
would be able to transform her personal outlook and find satisfaction in her life that would increase its relative value for her. The first assertion
can be addressed by clarifying that the case of Katherine Lewis must not be taken as a general rule. Deontologists, who are interested in
knowing general principles and duties that can be applied across all cases would not be very satisfied with this; they would prefer to be able to
a case-based, context-sensitive approach is better suited.
look to duties that would apply in all cases. Here,
Contextualizing would permit freedom to act within a particular context , without the implication that
the decision must hold in general. So, in this case, Katherine might decide that her life is relatively valueless. In another case, for example that of
actor Christopher Reeve,

CONTINUED

Page 299 of 1481


Page 300 of 1481
A2 Your Impact = Bare Life:
2AC (2/3)
CONTINUED

the decision to seek other ways of valuing this major life change led to him perceiving his life as highly valuable, even if different in value from
before the accident that made him a paraplegic. This invokes the second assertion, that Katherine could change her view over time. Although we
recognize this is possible in some cases, it is not clear how it applies to Katherine. Here we have a case in which a rational and competent person
has had time to consider her options and has chosen to end her life of suffering beyond what she believes she can endure. Ten months is a long
time and it will have given her plenty of opportunity to consult with family and professionals about the possibilities open to her in the future.
Given all this, it is reasonable to assume that Katherine has made a well-reasoned decision. It might not be a decision that everyone can agree
with but if her reasoning process can be called into question then at what point can we say that a decision is sound? She meets all the criteria
for competence and she is aware of the consequences of her decision. It would be very difficult to determine what arguments could truly justify
interfering with her choice. The second assertion made by supporters of the quality of life as a criterion for decisionmaking is closely related to
the value of the quality of a given
the first, but with an added dimension. This assertion suggests that the determination of
life is a subjective determination to be made by the person experiencing that life. The
important addition here is that the decision is a personal one that, ideally, ought not to be made
externally by another person but internally by the individual involved. Katherine Lewis made this decision for herself based on
a comparison between two stages of her life. So did James Brady. Without this element, decisions based on quality of life criteria lack salient
information and the patients concerned cannot give informed consent. Patients must be given the opportunity to decide for themselves
To ignore or overlook patients judgement in this matter
whether they think their lives are worth living or not .
is to violate their autonomy and their freedom to decide for themselves on the basis of relevant
information about their future, and comparative consideration of their past. As the deontological position puts it so well, to do so is to violate
the imperative that we must treat persons as rational and as ends in themselves .

THIRD, REFUSAL TO ASSIGN A VALUE TO LIFE RENDERS LIFE VALUELESS

Phera.com 2005
[www.phera.com/value_of_life]

Refusal to assign any value to life often leads, ironically, to ''no'' value being attached to
life. So, treating an endangered human life, or even the value of Earth itself, in economics
formally as a commodity can be morally justified, in that risks of failure to protect it, thus
become costs.

FOURTH, NUCLEAR WEAPONS USE IS A HORROR ON PAR WITH GENOCIDE


BECAUSE OF HOW IT INDISCRIMINATELY AND ABSOLUTELY DESTROYS INNOCENT
LIFE

Evans 95
[Gareth, Ministor of Foreign Affairs, Australia, On the Legality of the Threat or Use of Nuclear Weapons,
Verbatim Excerpts of Oral Statements to the International Court of Justice, October 30,
disarm.igc.org/oldwebpages/icjquote.html, acc. 8-24-05//uwyo-ajl]

Page 301 of 1481


The right to self-defence is not unlimited. It is subject to fundamental principles of humanity.
Self-defence is not a justification for genocide, for ordering that there shall be no enemy
survivors in combat or for indiscriminate attacks on the civilian population. Nor is it a
justification for the use of nuclear weapons.

The fact remains that the existence of nuclear


weapons as a class of weapons threatens the whole
of civilization. This is not the case with respect to any class or classes of conventional
weapons. It cannot be consistent with humanity to permit the existence of a weapon
which threatens the very survival of humanity.
There are some weapons the very existence of which is inconsistent with fundamental general principles of humanity. In
the case of weapons of this type, international law does not merely prohibit their threat or use. It prohibits even their
acquisition or manufacture, and by extension their possession. Such an attitude has been manifested in the case of other
types of weapons of mass destruction. Both the 1972 Biological Weapons Convention and the 1992 Chemical Weapons
Convention do not merely prohibit the use of biological and chemical weapons of mass destruction, but prevent their very
existence.

As was hideously demonstrated at Hiroshima, where a relatively minuscule atomic bomb


was detonated, and as the release of radiation by the Chernobyl disaster showed to our
horror, any use of nuclear weapons, anywhere at any time, would be devastating and in
no way comparable to any use, in whatever magnitude, of conventional weapons

Page 302 of 1481


Page 303 of 1481
A2 Your Impact = Bare Life:
2AC (3/3)
FIFTH, FAILURE TO ACT IN THE FACE OF ANNIHILATION RISKS TOTALITARIANISM BY
DENYING INSTITUTIONAL RESPONSIBILITY

Campbell 98
[David, Intl Relations Prof @ UM, National Deconstruction: Violence, Identity, and Justice in Bosnia,
Minneapolis: University of Minnesota Press, 1998, 186]

The undecidable within the decision does not, however, prevent the decision nor avoid its urgency. As Derrida observes, a
just decision is always required immediately, right away. This necessary haste has unavoidable consequences because
the pursuit of infinite information and the unlimited knowledge of conditions, rules or hypothetical
imperatives that could justify it are unavailable in the crush of time. Nor can the crush of time be avoided, even
by unlimited time, because the moment of decision as such always remains a finite moment of urgency and precipitation.
The decision is always structurally finite, it aalways marks the interruption of the juridico- or ethico- or politico-cognitive
deliberation that precedes it, that must precede it. That is why, invoking Kierkegaard, Derrida, declares that the instant of
decision is a madness.

The finite nature of the decision may be a madness in the way it renders possible the impossible, the infinite character of
justice, but Derrida argues for the necessity of this madness. Most importantly, Derrida argues for the necessity of this
madness. Most importantly, although Derridas argument concerning the decision has, to this pint, been concerned with an
account of the procedure by which a decision is possible, it is with respect to the ncessity of the decision that Derrida
begins to formulate an account of the decision that bears upon the content of the decision. In so doing, Derridas argument
addresses more directly more directly, I would argue than is acknowledged by Critchley the concern that for politics (at
least for a progressive politics) one must provide an account of the decision to combat domination.

That undecidability resides within the decision, Derrida argues, that justice exceeds law and calculation, that the
unpresentable exceeds the determinalbe cannot and should
not serve as alibi for staying out of juridico-
political battles, within an institution or a state, or between institutions or states and others. Indeed,
incalculable justice requires us to calculate. From where do these insistences come? What is behind, what is
animating, these imperatives? It is both the character of infinite justice as a heteronomic relationship to the other, a
relationship that because of its undecidability multiplies responsibility, and the fact that left
to itself, the
incalculable and given (donatrice) idea of justice is always very close to the bad, even to the worst, for it can
always be reappropriated by the most perverse calculation. The necessity of calculating the
incalculable thus responds to a duty a duty that inhabits the instant of madness and compels the decision to avoid
the duty that also dwells with deconstructive
the bad, the perverse calculation, even the worst. This is
thought and makes it the starting point, the at least necessary condition, for the
organization of resistance to totalitarianism in all its forms. And it is a duty that responds to practical
political concerns when we recognize that Derrida names the bad, the perverse, and the worst as those violences
we recognize all too well without yet having thought them through, the crimes of xenophobia, racism, anti-
Semitism, religious or nationalist fanaticism.

Page 304 of 1481


Page 305 of 1481
A2 No Value to Life: 2AC (1/3)
FIRST, THIS ARGUMENT IS REPULSIVE People ascribe their own value to life.
Violently taking it from them is the worst form of atrocity

SECOND, THERES ALWAYS A VALUE TO LIFE Even people in the worst conditions
find was of living beautifully

THIRD, THIS ISNT OFFENSE If someone finds their life valueless, they can commit
suicide. We at least give people who want to live the choice

FOURTH, LIFE ONLY BECOMES VALUELESS WHEN IT IS DECLARED AS SUCH [author is


describing specific men who were in Auschwitz with him]

Victor Frankl, Professor of Neurology and Psychiatry at the University of Vienna, Mans Search for
Meaning, 1946, p. 90-93

We have stated that that which was ultimately responsible for the state of the prisoners inner self was not so much the enumerated psychophysical causes as it was the result
only the men who allowed their inner hold on their moral
of a free decision. Psychological observations of the prisoners have shown that

and spiritual selves to subside eventually fell victim to the camps degenerating influences. The question now arises, what
could, or should, have constituted this inner hold? Former prisoners, when writing or relating their experiences, agree that the most depressing influence of all was that a
prisoner could not know how long his term of imprisonment would be. He had been given no date for his release. (In our camp it was pointless even to talk about it.) Actually a
prison term was not only uncertain but unlimited. A well-known research psychologist has pointed out that life in a concentration camp could be called a provisional existence.
We can add to this by defining it as a provisional existence of unknown limit. New arrivals usually knew nothing about the conditions at a camp. Those who had come back
from other camps were obliged to keep silent, and from some camps no one had returned. On entering camp a change took place in the minds of the men. With the end of
uncertainty there came the uncertainty of the end. It was impossible to foresee whether or when, if at all, this form of existence would end. The latin word finis has two
A man who could not see the end of his provisional existence was not
meanings: the end or the finish, and a goal to reach.

able to aim at an ultimate goal in life. He ceased living for the future, in contrast to a man in normal life. Therefore the whole
structure of his inner life changed; signs of decay set in which we know from other areas of life. The unemployed worker, for example, is in a
similar position. His existence has become provisional and in a certain sense he cannot live for the future or aim at a goal. Research work done on unemployed miners has
shown that they suffer from a peculiar sort of deformed timeinner time-which is a result of their unemployed state. Prisoners, too, suffered from this strange time-experience.
In camp, a small time unit, a day, for example, filled with hourly tortures and fatigue, appeared endless. A larger time unit, perhaps a week, seemed to pass very quickly. My
comrades agreed when I said that in camp a day lasted longer than a week. How paradoxical was our time-experience! In this connection we are reminded of Thomas Manns
The Magic Mountain, which contains some very pointed psychological remarks. Mann studies the spiritual development of people who are in an analogous psychological
position, i.e., tuberculosis patients in a sanatorium who also know no date for their release. They experience a similar existencewithout a future and without a goal. One of the
prisoners, who on his arrival marched with a long column of new inmates from the station to the camp, told me later that he had felt as though he were marching at his own
funeral. His life had seemed to him absolutely without future. He regarded it as over and done, as if he had already died. This feeling of lifelessness was intensified by other
causes: in time, it was the limitlessness of the term of imprisonment which was most acutely felt; in space, the narrow limits of the prison. Anything outside the barbed wire
became remoteout of reach and, in a way, unreal. The events and the people outside, all the normal life there, had a ghostly aspect for the prisoner. The outside life, that is,
as much as he could see of it, appeared to him almost as it might have to a dead man who looked at it from another world. A man who let himself decline because he could not
see any future goal found himself occupied with retrospective thoughts. In a different connection, we have already spoken of the tendency there was to look into the past, to

danger. It became easy to overlook the


help make the present, with all its horrors, less real. But in robbing the present of its reality there lay a certain

opportunities to make something positive of camp life, opportunities which really did exist. Regarding our provisional
existence as unreal was in itself an important factor in causing the prisoners to lose their hold on life;
everything in a way became pointless. Such people forget that often it is just such an exceptionally difficult external situation which gives man the
opportunity to grow spiritually beyond himself. Instead of taking the camps difficulties as a test of their inner strength, they did not take their life seriously and despised it as

something of no consequence. They preferred to close their eyes and to live in the past. Life for such people became meaningless.

Page 306 of 1481


Page 307 of 1481
A2 No Value to Life: 2AC (2/3)
FIFTH, VALUE TO LIFE IS SUBJECTIVE MUST ALLOW PEOPLE THE CHOICE TO FIND
THEIR OWN VALUE AT ALL COSTS AND RESIST EXTERNAL ATTEMPTS TO DESTROY
IT

Schwartz 2004
[A Value to Life: Who Decides and How? www.fleshandbones.com/readingroom/pdf/399.pdf]

Those who choose to reason on this basis hope that if the quality of a life can be measured then the answer to whether that life has value to the individual can
be determined easily. This raises special problems, however, because the idea of quality involves a value judgement, and value judgements are, by their
essence, subject to indeterminate relative factors such as preferences and dislikes. Hence, quality of life is difficult to measure and will vary according to
individual tastes, preferences and aspirations. As a result,

no general rules or principles can be asserted that would simplify decisions about the value of a life based on its quality. Nevertheless, quality is still an

persons can decide for


essential criterion in making such decisions because it gives legitimacy to the possibility that rational, autonomous

themselves that their own lives either are worth, or are no longer worth, living. To disregard this possibility would be to
imply that no individuals can legitimately make such value judgements about their own lives and, if nothing else, that would be counterintuitive. 2 In our case,
Katherine Lewis had spent 10 months considering her decision before concluding that her life was no longer of a tolerable quality. She put a great deal of effort
into the decision and she was competent when she made it. Who would be better placed to make this judgement for her than Katherine herself? And yet, a
doctor faced with her request would most likely be uncertain about whether Katherines choice is truly in her best interest, and feel trepidation about assisting
her. We need to know which considerations can be used to protect the patients interests. The quality of life criterion asserts that there is a difference between
the type of life and the fact of life. This is the primary difference between it and the sanctity criterion discussed on page 115. Among quality of life
considerations rest three assertions: 1. there is relative value to life 2. the value of a life is determined subjectively 3. not all lives are of equal value. Relative
value The first assertion, that life is of relative value, could be taken in two ways. In one sense, it could mean that the value of a given life can be placed on a
scale and measured against other lives. The scale could be a social scale, for example, where the contributions or potential for contribution of individuals are
measured against those of fellow citizens. Critics of quality of life criteria frequently name this as a potential slippery slope where lives would be deemed worthy
of saving, or even not saving, based on the relative social value of the individual concerned. So, for example, a mother of four children who is a practising
doctor could be regarded of greater value to the community than an unmarried accountant. The concern is that the potential for discrimination is too high.
Because of the possibility of prejudice and injustice, supporters of the quality of life criterion reject this interpersonal construction in favour of a second, more
personalized, option. According to this interpretation, the notion of relative value is relevant not between individuals but within the context of one persons life
and is measured against that persons needs and aspirations. So Katherine would base her decision on a comparison between her life before and after her
illness. The value placed on the quality of a life would be determined by the individual depending on whether he or she believes the current state to be relatively
preferable to previous or future states and whether he or she can foresee controlling the circumstances that make it that way. Thus, the life of an athlete who
aspires to participate in the Olympics can be changed in relative value by an accident that leaves that person a quadriplegic. The athlete might decide that the
relative value of her life is diminished after the accident, because she perceives her desires and aspirations to be reduced or beyond her capacity to control.
However, if she receives treatment and counselling her aspirations could change and, with the adjustment, she could learn to value her life as a quadriplegic as
much or more than her previous life. This illustrates how it is possible for a person to adjust the values by which they appraise their lives. For Katherine Lewis,
the decision went the opposite way and she decided that a life of incapacity and constant pain was of relatively low value to her. It is not surprising that the
most vociferous protesters against permitting people in Katherines position to be assisted in terminating their lives are people who themselves are disabled.
Organizations run by, and that represent, persons with disabilities make two assertions in this light. First, they claim that accepting that Katherine Lewis has a
right to die based on her determination that her life is of relatively little value is demeaning to all disabled people, and implies that any life with a severe
disability is not worth Write a list of three things that make living. Their second assertion is that with proper help, over time Katherine would be able to transform
her personal outlook and find satisfaction in her life that would increase its relative value for her. The first assertion can be addressed by clarifying that the case
of Katherine Lewis must not be taken as a general rule. Deontologists, who are interested in knowing general principles and duties that can be applied across
all cases would not be very satisfied with this; they would prefer to be able to look to duties that would apply in all cases. Here, a case-based, context-sensitive
approach is better suited. Contextualizing would permit freedom to act within a particular context, without the implication that the decision must hold in general.
So, in this case, Katherine might decide that her life is relatively valueless. In another case, for example that of actor Christopher Reeve, the decision to seek
other ways of valuing this major life change led to him perceiving his life as highly valuable, even if different in value from before the accident that made him a
paraplegic. This invokes the second assertion, that Katherine could change her view over time. Although we recognize this is possible in some cases, it is not
clear how it applies to Katherine. Here we have a case in which a rational and competent person has had time to consider her options and has chosen to end
her life of suffering beyond what she believes she can endure. Ten months is a long time and it will have given her plenty of opportunity to consult with family
and professionals about the possibilities open to her in the future. Given all this, it is reasonable to assume that Katherine has made a well-reasoned decision.
It might not be a decision that everyone can agree with but if her reasoning process can be called into question then at what point can we say that a decision is
sound? She meets all the criteria for competence and she is aware of the consequences of her decision. It would be very difficult to determine what arguments
could truly justify interfering with her choice. The second assertion made by supporters of the quality of life as a criterion for decisionmaking is closely related to
the value of the quality of a given life is
the first, but with an added dimension. This assertion suggests that the determination of

a subjective determination to be made by the person experiencing that life. The important addition
here is that the decision is a personal one that, ideally, ought not to be made externally by

another person but internally by the individual involved. Katherine Lewis made this decision for herself based on a comparison between two stages
of her life. So did James Brady. Without this element, decisions based on quality of life criteria lack salient information and the patients concerned cannot give

To ignore or
informed consent. Patients must be given the opportunity to decide for themselves whether they think their lives are worth living or not.

overlook patients judgement in this matter is to violate their autonomy and their freedom
to decide for themselves on the basis of relevant information about their future, and comparative consideration of their past. As the deontological
position puts it so well, to do so is to violate the imperative that we must treat persons as rational and as ends in themselves.

Page 308 of 1481


Page 309 of 1481
A2 No Value to Life: 2AC (3/3)
SIXTH, NO VALUE TO LIFE RHETORIC UNDERMINES HOPE FOR THE FUTURE. IT
CREATES FALSE HOPE OF LIBERATION FROM MEANINGLESSNESS WITHOUT
ADDRESSING WHAT WE ARE LIVING FOR. VOTE TO AFFIRM INTRINSIC VALUE TO
EXISTENCE [THIS EVIDENCE IS GENDER PARAPHRASED]

Victor Frankl, Professor of Neurology and Psychiatry at the University of Vienna, Mans Search
for Meaning, 1946, p. 96-98

I once had a dramatic demonstration of the close link between the loss of faith in the future and this dangerous giving up. F, my senior block warden, a fairly well-known
composer and librettist, confided in me one day: I would like to tell you something, Doctor. I have had a strange dream. A voice told me that I could wish for something, that I
should only say what I wanted to know, and all my questions would be answered. What do you think I asked? That I would like to know when the war would be over for me. You
know what I mean, Doctorfor me! I wanted to know when we, when our camp, would be liberated and our sufferings come to an end. And when did you have this dream? I
asked. In February, 1945, he answered. It was then the beginning of March. What did your dream voice answer? Furtively he whispered to me, March thirtieth. When F
told me about his dream, he was still full of hope and convinced that the voice of his dream would be right. But as the promised day drew nearer, the war news which reached
our camp made it appear very unlikely that we would be free on the promised date. On March twenty-ninth, F suddenly became ill and ran a high temperature. On March
thirtieth, the day his prophecy had told him that the war and suffering would be over for him, he became delirious and lost consciousness. On March thirty-first, he was dead. To
all outward appearances, he had died of typhus. Those who know how close the connection is between the state of mind of a manhis courage and hope, or lack of them

sudden loss of hope and courage can have a deadly effect. The ultimate
and the state of immunity of his body will understand that the

cause of my friends death was that the expected liberation did not come and he was severely disappointed. This
suddenly lowered his bodys resistance against the latent typhus infection. His faith in the future and his will to live had become
paralyzed and his body fell victim to illnessand thus the voice of his dream was right after all. The observations of this one case and the conclusion drawn from them are in
accordance with something that was drawn to my attention by the chief doctor of our concentration camp. The death rate in the week between Christmas, 1944, and New
Years, 1945, increased in camp beyond all previous experience. In his opinion, the explanation for this increase did not lie in the harder working conditions or the deterioration
of our food supplies or a change of weather or new epidemics. It was simply that the majority of the prisoners had lived in the naive hope that they would be home again by
Christmas. As the time drew near and there was no encouraging news, the prisoners lost courage and disappointment overcame them. This had a dangerous influence on their

any attempt to restore a mans inner strength in the camp had first
powers of resistance and a great number of them died. As we said before,

to succeed in showing him some future goal. Nietzsches words, [One] He who has a why to live for can bear with
almost any how, could be the guiding motto for all psychotherapeutic and psychohygienic efforts regarding prisoners. Whenever there was an opportunity for it, one
had to give them a whyan aimfor their lives, in order to strengthen them to bear the terrible how of their existence. Woe to him who saw no more sense in his life, no aim,
no purpose, and therefore no point in carrying on. He was soon lost. The typical reply with which such a man rejected all encouraging arguments was, I have nothing to expect
from life any more. What sort of answer can one give to that? What was really needed was a fundamental change in our attitude toward life. We had to learn ourselves and,
We needed to stop
furthermore, we had to teach the despairing men, that it did not really matter what we expected from life, but rather what life expected from us.

asking about the meaning of life, and instead to thisnk of ourselves as those who were being questioned
by lifedaily and hourly. Our answer must consist, not in talk and meditation, but in right action and in right conduct. Life ultimately means taking the
responsibility to find the right answer to its problems and to fulfill the tasks which it constantly sets for each individual.

SEVENTH, EXTINCTION OF THE SPECIES IS THE MOST HORRIBLE IMPACT


IMAGINEABLE, PUTTING RIGHTS FIRST IS PUTTING A PART OF SOCIETY BEFORE THE
WHOLE

Schell 1982
(Jonathan, Professor at Wesleyan University, The Fate of the Earth, pages 136-137 uw//wej)

Implicit in everything that I have said so far about the nuclear predicament there has been a perplexity that I would now like to take up explicitly, for it leads, I
believe, into the very heart of our response-or, rather, our lack of response-to the predicament. I have pointed out that our species is the most important of all

it does not go far enough to point out


the things that, as inhabitants of a common world, we inherit from the past generations, but

this superior importance, as though in making our decision about ex- tinction we were being
asked to choose between, say, liberty, on the one hand, and the survival of the species, on the other. For the
species not only overarches but contains all the benefits of life in the common world, and to

Page 310 of 1481


speak of sacrificing the species for the sake of one of these benefits involves one in the absurdity
of wanting to de- stroy something in order to preserve one of its parts, as if one were to burn
down a house in an attempt to redecorate the living room, or to kill someone to improve his character. ,but even to point
out this absurdity fails to take the full measure of the peril of extinction, for mankind is not some invaluable object that lies outside us and that we must protect
so that we can go on benefiting from it; rather, it is we ourselves, without whom everything there is loses its value. To say this is another way of saying that
extinction is unique not because it destroys mankind as an object but because it destroys mankind as the source of all possible human subjects, and this, in
turn, is another way of saying that extinction is a second death, for one's own individual death is the end not of any object in life but of the subject that
experiences all objects. Death, how- ever, places the mind in a quandary. One of-the confounding char- acteristics of death-"tomorrow's zero," in Dostoevski's
phrase-is that, precisely because it removes the person himself rather than something in his life, it seems to offer the mind nothing to take hold of. One even
feels it inappropriate, in a way, to try to speak "about" death at all, as. though death were a thing situated some- where outside us and available for objective
inspection, when the fact is that it is within us-is, indeed, an essential part of what we are. It would be more appropriate, perhaps, to say that death, as a
fundamental element of our being, "thinks" in us and through us about whatever we think about, coloring our thoughts and moods with its presence throughout

.
our lives

Page 311 of 1481


No Value To Life Justifies Genocide

EUTHANASIA AND GENOCIDE IS JUSTIFIED BY THE DEPLOYMENT OF THE RHETORIC


OF NO VALUE TO LIFE

Richard Coleson, M.A.R., J.D., ISSUES IN LAW & MEDICINE, Summer, 1997

Euthanasia also was advocated in Germany. As early as 1895, a widely-used German medical textbook
made a claim for "the right to death." Michael Berenbaum, The World Must Know: The History of the
Holocaust as Told in the United States Holocaust Memorial Museum 64 (1993). Immediately following
World War I, the notion took greater root in the German medical and legal professions, instigated largely
by a publication by Professors Karl Binding and Alfred Hoche of Die Freigabe der Vernichtung
lebensunwertens Leben (Permitting the Destruction of Unworthy Life) (1920). See 8 Issues in Law & Med.
221 (1992) (Patrick Derr and Walter Wright, trans.) (copies of which have been lodged with the Court).
What transpired in Germany in the late 1930s and 1940s would unalterably change the debate over the
ethics and legality of physicians participating in ending the lives of their patients. In that period, the lives
of hundreds of thousands of terminally ill, incurably sick, and mentally incompetent patients were
terminated by German doctors--the elite of the profession in Europe--in a program of "euthanasia"
propagated both by acceptance of the " unworthy life" thesis and by the imposition of National Socialist
theories of eugenics derived from earlier concepts developed by the German medical profession and
intelligentsia. Michael Burleigh, Death and Deliverance: 'Euthanasia' in Germany 1900-1945 93-97, 273-
277, 284-285 (1994); Robert Jay Lifton, The Nazi Doctors: Medical Killing and the Psychology of Genocide
44-79 (1986); Gallagher, By Trust Betrayed, supra at 74-95. In the ensuing decades, the connection of
medical killing in Nazi Germany to contemporary debates regarding the legalization of assisted suicide and
euthanasia has been a matter of great controversy. Burleigh, Death and Deliverance, supra at 291-98.
[Footnote omitted] It is clear, however, that those closest to these events saw some connection. The
condemnation of the "Nazi doctors" was universal and prompted great reflection on the question of
ensuring that their actions never be repeated. As one step, the world's physicians reaffirmed the
foundational ethical principle of their profession: that doctors must not kill. [Footnote omitted] The cases
before this Court are the most important juridical test since that time of the meaning of that principle. For
this reason alone, the experience which influenced so much of what the world thinks today of the issue of
euthanasia is relevant to the deliberations of this Court. The acceptance by physicians of the notion of a
"life not worthy to be lived" under the "euthanasia" program was a cornerstone of the horror that was to
follow. Leo Alexander, Medical Science Under Dictatorship, 241 New Eng. J. Med. 39, 44 (1949). Without
the willingness of doctors to participate, the euthanasia program would not have occurred. Patrick Derr,
Hadamar, Hippocrates, and the Future of Medicine: Reflections on Euthanasia and the History of German
Medicine, 4 Issues in Law & Med. 487 (1989). This "cornerstone" principle persists today. The experience
of the Netherlands (described in the Brief of Amicus Curiae the American Suicide Foundation in No. 96-
110) establishes that the participation of physicians in killing their patients invariably rests upon, and
propagates, the notion of life unworthy of life. The writings of pro-euthanasia philosophers James
Rachels, Peter Singer, and John Harris [Footnote omitted] confirm this fact. While social and political
conditions in Western democracies obviously differ from those of post-World War I and Nazi Germany,
the consequences of legalizing physician-assisted suicide and euthanasia will be no less dire.

Page 312 of 1481


No Value To Life Justifies Nazism

ALSO, THE ARGUMENT THAT CERTAIN CONDITIONS MAKE LIFE NOT WORTH
LIVING ACCEPTS THE PHILOSOPHICAL PREMISE OF NAZI GERMANY STYLE
MURDERS AND CONCENTRATION CAMPS THAT RESPECT FOR LIFE DOES NOT
ENTAIL PRESERVING LIFE

Steven Neeley, Assistant Professor at Saint Francis, AKRON LAW REVIEW v. 28, Summer, 1994.

The final solution in the United States and other western societies will be unlike the final solution in Nazi
Germany in its details, but not unlike it in its horror. And I fear that some who now live will experience
this final solution. They will live to see the day they will be killed. Variations of the "slippery-slope"
argument as applied to suicide and euthanasia are abundant. Beauchamp has argued, for example, that at
least from the perspective of rule utilitarianism, the wedge argument against euthanasia should be taken
seriously. Accordingly, although a "restricted-active-euthanasia rule would have some utility value" since
some intense and uncontrollable suffering would be eliminated, "it may not have the highest utility value
in the structure of our present code or in any imaginable code which could be made current, and
therefore may not be a component in the ideal code for our society . . . . For the disutility of introducing
legitimate killing into one's moral code (in the form of active euthanasia rules) may, in the long run,
outweigh the utility of doing so, as a result of the eroding effect such a relaxation would have on rules in
the code which demand respect for human life. " Beauchamp then continues down a now-familiar path:
If, for example, rules permitting active killing were introduced, it is not implausible to suppose that
destroying defective newborns (a form of involuntary euthanasia) would become an accepted and
common practice, that as population increases occur the aged will be even more neglectable and
neglected than they now are, that capital punishment for a wide variety of crimes would be increasingly
tempting, that some doctors would have appreciably reduced fears of actively injecting fatal doses
whenever it seemed to them propitious to do so . . . . A hundred such possible consequences might easily
be imagined. But these few are sufficient to make the larger point that such rules permitting killing could
lead to a general reduction of respect for human life.

Page 313 of 1481


Theres Always Value To Life

THERES ALWAYS VALUE TO LIFE

Victor Frankl, Professor of Neurology and Psychiatry at the University of Vienna, Mans Search for
Meaning, 1946, p. 104

But I did not only talk of the future and the veil which was drawn over it. I also mentioned the past; all its
joys, and how its light shone even in the present darkness. Again I quoted a poetto avoid sounding like a
preacher myselfwho had written, Was Dii erlebst, k,ann keme Macht der Welt Dir rauben. (What you
have experienced, no power on earth can take from you.) Not only our experiences, but all we have done,
whatever great thoughts we may have had, and all we have suffered, all this is not lost, though it is past;
we have brought it into being. Having been is also a kind of being, and perhaps the surest kind. Then I
spoke of the many opportunities of giving life a meaning. I told my comrades (who lay motionless,
although occasionally a sigh could be heard) that human life, under any circumstances, never ceases to
have a meaning, and that this infinite meaning of life includes suffering and dying, privation and death. I
asked the poor creatures who listened to me attentively in the darkness of the hut to face up to the
seriousness of our position. They must not lose hope but should keep their courage in the certainty that
the hopelessness of our struggle did not detract from its dignity and its meaning. I said that someone
looks down on each of us in difficult hoursa friend, a wife, somebody alive or dead, or a Godand he
would not expect us to disappoint him. He would hope to find us suffering proudlynot miserably
knowing how to die.

THERES ALWAYS VALUE TO LIFE, EVEN WITH TREMENDOUS SUFFERING

Victor Frankl, Professor of Neurology and Psychiatry at the University of Vienna, Mans Search for
Meaning, 1946, p. 99-100

When a man finds that it is his destiny to suffer, he will have to accept his suffering as his task; his single
and unique task. He will have to acknowledge the fact that even in suffering he is unique and alone in the
universe. No one can relieve him of his suffering or suffer in his place. His unique opportunity lies in the
way in which he bears his burden. For us, as prisoners, these thoughts were not speculations far removed
from reality. They were the only thoughts that could be of help to us. They kept us from despair, even
when there seemed to be no chance of coming out of it alive. Long ago we had passed the stage of asking
what was the meaning of life, a naive query which understands life as the attaining of some aim through
the active creation of something of value. For us, the meaning of life embraced the wider cycles of life and
death, of suffering and of dying. Once the meaning of suffering had been revealed to us, we refused to
minimize or alleviate the camps tortures by ignoring them or harboring false illusions and entertaining
artificial optimism. Suffering had become a task on which we did not want to turn our backs. We had
realized its hidden opportunities for achievement, the opportunities which caused the poet Rilke to write,
Wie viel ist aufzuleiden! (How much suffering there is to get through!) Rilke spoke of getting through
suffering as others would talk of getting through work. There was plenty of suffering for us to get
through. Therefore, it was necessary to face up to the full amount of suffering, trying to keep moments of
weakness and furtive tears to a minimum. But there was no need to be ashamed of tears, for tears bore
witness that a man had the greatest of courage, the courage to suffer. Only very few realized that.

Page 314 of 1481


Shamefacedly some confessed occasionally that they had wept, like the comrade who answered my
question of how he had gotten over his edema, by confessing, I have wept it out of my system.

Page 315 of 1481


Page 316 of 1481
A2 Communication Scholar
Framework: 2AC
MCCHESNEY CONCEDES THAT UNANTICIPATED CONSEQUENCES MUST BE TAKEN
INTO ACCOUNT

McChesney 96
[Robert W., U. of Wisconsin-Madison, The Internet and U.S. Communication Policy-Making in Historical
and Critical Perspective, Journal of Communication 46 (1), Winter,
http://jcmc.indiana.edu/vol1/issue4/mcchesney.html, acc. 9-30-06//uwyo-ajl]

All communication technologies have unanticipated and unintended effects, and one function of policy-
making is to understand them so we may avoid or minimize the undesirable ones. The digitalization and
computerization of our society are going to transform us radically, yet even those closely associated with
these developments express concern about the possibility of a severe deterioration of the human
experience as a result of the information revolution (Deitch, 1994; Stoll, 1995; Talbott, 1995). As one
observer notes, "Very few of us-only the high priests-really understand the new technologies, and these
are surely the people least qualified to make policy decisions about them" (Charbeneau, 1994, pp. 28-29).
For every argument extolling the "virtual community" and the liberatory aspects of cyberspace, it seems
every bit as plausible to reach dystopian conclusions. Why not look at the information highway as a
process that encourages the isolation, atomization, and marginalization of people in society? In fact,
cannot the ability of people to create their own community in cyberspace have the effect of terminating a
community in the general sense? In a class-stratified, commercially oriented society like the United States,
cannot the information highway have the effect of simply making it possible for the well-to-do to bypass
any contact with the balance of society altogether? These are precisely the types of questions that need
to be addressed and answered in communication policy-making and precisely the types of questions in
which the market has no interest (Chapman, 1995). At any rate, a healthy skepticism toward technology
should be the order of the day.

COMMUNICATION SCHOLARS HAVE TO CONSIDER POLITICAL CONSEQUENCES

Sandgathe 2001
[Sharon, Engl Dept. @ Arizona, The Culture of Agriculture, February 27,
darkwing.uoregon.edu/~tns/session_6.htm, acc 9-30-06]

As a scholar of rhetoric, much of my work examines discourse in public arenas. I find public constructions
of agriculture to be a fascinating site for study because in agriculture people must explicitly engage the
interpenetration of nature and culture. Currently, a common way to validate a particular vision of that
interpenetration is to label a favored version of agriculture with the highly prized signifier sustainable.
In this discussion I will argue that the shifting use of the term sustainable agriculture in public discourse
reflects political conflict over social identities, cultural values, and material practices. I will also examine
how discourses about nature, especially highly valued scientific discourses, are used to legitimate the

Page 317 of 1481


social agendas represented by sustainable agriculture, and what the political consequences of that
legitimization might be.

Page 318 of 1481


Page 319 of 1481
**Democratic Talk**

Page 320 of 1481


Democratic Talk Turn: 2AC (1/2)
TURN: DEMOCRATIC TALK

A. REFUSING TO ACT AS IF WERE THE GOVERNMENT DESTROYS THE DELIBERATIVE


DEMOCRATIC POTENTIAL OF DEBATE

Barber, Professor of Political Science at Rutgers, 1984 (Benjamin, Strong Democracy: Participatory Politics for a
New Age)

Agenda-Setting. In liberal democracies, agendas are typically regarded as the province of elites -- of
committees, or executive officers, or (even) pollsters. This is so not simply because representative systems
delegate the agenda-setting function or because they slight citizen participation, but because they conceive of
agendas as fixed and self-evident, almost natural, and in this sense incidental to such vital democratic
processes as deliberation and decision-making. Yet a people that does not set its own agenda, by means
of talk and direct political exchange, not only relinquishes a vital power of government but also exposes its
remaining powers of deliberation and decision to ongoing subversion. What counts as an "issue" or a
"problem" and how such issues or problems are formulated may to a large extent predetermine what
decisions are reached. For example, the choice between building a small freeway and a twelve-lane interstate
highway in lower Manhattan may seem of little moment to those who prefer to solve the problems of urban
transportation with mass rail transit. Or the right to choose among six mildly right-of-center candidates may fail to
exercise the civic imagination of socialists. Nor is it sufficient to offer a wide variety of options, for what
constitutes an option-how a question is formulated-is as controversial as the range of choices offered.
Abortion is clearly an issue that arouses intense public concern at present, but to say that it belongs on the public
agenda says too little. The vital question remains: How is it presented? In this form: "Do you believe there should be an
amendment to the Constitution protecting the life of the unborn child?" Or in this form: "Do you believe there should
be an amendment to the Constitution prohibiting abortions?" When asked the first question by a New York Times-CBS
poll, over one-half responded "yes," whereas when asked the second question only 29 percent said "yes .,,25 He who
controls the agenda-if only its wording-controls the outcome. The battle for the Equal Rights Amendment was probably
lost because its enemies managed to place it on the public agenda as calling for "the destruction of the family, the
legitimization of homosexuality, and the compulsory use of coed toilets." The ERA's supporters never succeeded in
getting Americans to see it as "the simple extension of the Constitution's guarantees of rights to women"-a goal that
most citizens would probably endorse. The ordering of alternatives can affect the patterns of choice as decisively as
their formulation. A compromise presented after positions have been polarized may fail; a constitutional amendment
presented at the tail end of the period of change that occasioned it may not survive in a new climate of opinion. A
proposal paired with a less attractive alternative may succeed where the same proposal paired with some third option
would fail. What these realities suggest is that in a genuine democracy agenda-setting cannot precede talk or
deliberation, and decision but must be approached as a permanent function of talk itself. Relegating
agenda-setting to elites or to some putatively "natural" process is an abdication of rights and responsibilities.
Unless the debate about Manhattan's interstate freeway permits people to discuss their fundamental priorities for
mass transportation, energy, and ecology, it is a sham. Unless the debate over abortion permits people to discuss the
social conditions of pregnancy, the practical alternatives available to the poor, and the moral dilemmas of a woman
torn between her obligations to her own body and life and to an embryo, such debate will treat neither pregnant
women nor unborn babies with a reasonable approximation of justice. For these reasons, strong democratic talk
places its agenda at the center rather than at the beginning of its politics. It subjects every pressing issue
to continuous examination and possible reformulation. Its agenda is, before anything else, its agenda. It
thus scrutinizes what remains unspoken, looking into the crevices of silence for signs of an unarticulated
problem, a speechless victim, or a mute protester. The agenda of a community tells a community where
and what it is. It defines that community's mutualism and the limits of mutualism and draws up plans for
pasts to be institutionalized or overcome and for futures to be avoided or achieved. Far from being a
mere preliminary of democracy, agenda-setting becomes one of its pervasive, defining functions. 180-182

Page 321 of 1481


Page 322 of 1481
Page 323 of 1481
Democratic Talk Turn: 2AC (2/2)
B. THE IMPACT IS SLAVERY [THIS EV HAS BEEN GENDER MODIFIED]

Barber, Professor of Political Science at Rutgers, 1984 (Benjamin, Strong Democracy: Participatory Politics for a
New Age)

Political animals interact socially in ways that abstract morals and metaphysics cannot account for. Their virtue is of
another order, although few theorists who have defended this claim have been called everything from m realists to
immoralists for their trouble. Yet Montaigne caught the very spirit of social man when he wrote, "the virtue assigned to
the affairs of the world is a virtue with many bends, angles, and elbows, so as to join and adapt itself to human
weakness; mixed and artificial, not straight, clean, constant or purely innocent." If the human essence is social, then
men and women have to choose not between independence or dependence
but between citizenship or slavery. Without citizens, Rousseau warns, there will be
neither free natural men nor satisfied solitaries-there will be "nothing but debased
slaves, from the rulers of the state downwards." To a strong democrat, Rousseau's assertion at
the opening of his Social Contract that [an individual] is born free yet is everywhere in chains
does not mean that [an individual] is free by nature but society enchains him
[or her]. It means rather that natural freedom is an abstraction, whereas
dependency is the concrete human reality, and that the aim of-politics must
therefore be not to rescue natural freedom from politics, but, to invent and
pursue artificial freedom within and through politics. Strong democracy aims
not to disenthrall [individuals] but to legitimate their dependency by means of
citizenship and to establish their political freedom by means of the democratic
community. 216

Page 324 of 1481


Page 325 of 1481
Democratic Talk Turn: 1AR (1/3)
OUR TURNS ARE IMPORTANT BECAUSE THE KRITIK IS UNLIKELY TO BRING ABOUT
AN ENTIRELY CHANGED WORLD THE PROCESS OF DEMOCRATIC TALK BRINGS US
TOGETHER AS A POLITICAL COMMUNITY WHERE WE CAN ENVISION ALTERNATIVE
FUTURES (RE)CREATING OUR POWER AS POLITICALLY ACTIVE PARTICIPATING
CITIZENS

MORE EV

Barber, Professor of Political Science at Rutgers, 1984 (Benjamin, Strong Democracy: Participatory Politics for a
New Age)

Liberal critics of participation, imbued with the priorities of privatism, will continue to believe
that the neighborhood-assembly idea will falter for lack of popular response. "Voters," writes
Gerald Pomper, "have too many pressing tasks, from making money to making love, to follow the arcane procedures of
government." If the successful and industrious will not participate because they are too busy, and the poor and
victimized will not participate because they are too apathetic, who will people the assemblies and who will give to talk
a new democratic life? But of course people refuse to participate only where politics does
not count-or counts less than rival forms o private activity. They are apathetic
because they are powerless, not powerless because they are apathetic. There is
no evidence to suggest that once empowered, a people will refuse to
participate. The historical evidence of New England towns, community school boards, neighborhood
associations, and other local bodies is that participation fosters more participation. 272

Page 326 of 1481


Page 327 of 1481
Democratic Talk Turn: 1AR (2/3)
RIGHT HERE RIGHT NOW WE HAVE TO SET THE AGENDA LEAVING THESE DUTIES
UP TO THE ELITES AND THOSE IN CONTROL ENSURES THAT WE WILL ALL LOSE OUR
SOVEREIGNTY WE HAVE TO DETERMINE WHAT QUESTIONS ARE GOING TO BE
ASKED AND WHAT FORM THOSE QUESTIONS TAKE TAKING PROACTIVE ACTION
EVEN IF IT IS JUST COMMON DELIBERATION IN THIS ROOM IS WHAT IS TRULY
CRITICAL TO OUR OWN POLITICAL EFFICACY AND PREVENTING THOSE IN POWER
FROM SETTING THE AGENDA FOR US

Barber, Professor of Political Science at Rutgers, 1984 (Benjamin, Strong Democracy: Participatory Politics for a New
Age)

If talk can give the dead back their voices, it can also challenge
the paradigms of the living
and bring fundamental changes in the meaning or valuation of words. Major
shifts in ideology and political power are always accompanied by such
paradigmatic-shifts in language usage-so much so that historians have begun to map the
former by charting the latter. The largely pejorative meaning that the classical and early Christian periods
gave to such terms as individual and privacy was transformed during the Renaissance in a fashion that
eventually produced the Protestant Reformation and the ethics of commercial society. Eighteenth-
century capitalism effected a transvaluation of the traditional vocabulary of virtue in a manner that put
selfishness and avarice to work in the name of public goods. (George Gilder's Wealth and Poverty is
merely the last and least in a long line of efforts to invert moral categories.) The history of democracy
itself is contained in the history of the word democracy. The battle for self-government has been fought
over and over again as pejorative valuations of the term have competed with affirmative ones (pitting
Plato or Ortega or Lippmann or modern political science against Machiavelli or Rousseau or Jefferson).
The terms ochlocracy, mob rule, tyranny of the majority, and rule-of the masses all reflect hostile
constructions of democracy; communitarianism, participationism, egalitarianism, and -it must be
admitted-strong democracy suggest more favorable-constructions. Poverty was once a sign of moral
weakness; now it is a badge of environmental victimization. Crime once proceeded from original now it is
an escape from poverty. States' rights once bore the stigma of dishonor, then signified vigorous
sectionalism, then was a code word for racism, and has now become a byword for the new decentralized
federalism. Busing was once an instrument of equal educational opportunity; now it is a means of
destroying communities. The shifts in the meaning of these and dozens of other key words
mirror fundamental national shifts in power and ideology. The clash of competing visions-of social
Darwinism versus collective responsibility and political mutualism, of original sin and innate ideas versus
environmentalism, of anarchism versus collectivism ultimately plays
itself out on the field of
everyday language, and the winner in the daily struggle for meaning may
emerge as the winner in the clash of visions, with the future itself as the spoils of victory.
An ostensibly free citizenry that leaves this battle to elites, thinking that it
makes a sufficient display of its freedom by deliberating and voting on issues
already formulated in concepts and terms over which it has exercised no
control, has in fact already given away the greater part of its sovereignty. How
can such a citizenry -help but oppose busing if busing means the wrecking of communities and only the

Page 328 of 1481


wrecking of communities? How can it support the right to abortion if abortion means murder, period? To
participate in a meaningful process of decision on these questions, self-governing
citizens must participate in the talk through which the questions are
formulated and given a

Page 329 of 1481


Page 330 of 1481
Democratic Talk Turn: 1AR (3/3)
(Barber continues)

decisive political conception. The anti-Vietnam War movement of the 1960s did
just this, of course; it won no elections, it participated in no votes, and it
contributed to no legislative debates. But it radically altered how most
Americans saw the war and so helped bring it to an end . If language as a living,
changing expression of an evolving community can both encapsulate and challenge the past, it also
provides a vehicle for exploring the future. Language's
flexibility and its susceptibility to
innovation permit [people] to construct their visions of the future first in the
realm of words, within whose confines a community can safely conduct its
deliberations. Language can offer new solutions to old problems by altering,
how we perceive these problems and can make new visions accessible to
traditional communities by the imaginative use (and transvaluation) of familiar
language. This-is the essence of public thinking." The process moves us perforce from
particularistic and immediate considerations of our own and our groups' interests, examined in a narrow
temporal framework ("Will there be enough gasoline for my summer vacation trip?" for example), to
general and long-term considerations of the nature of the communities we live in and of how well our life
plans fit in with that nature ("Is dependence on oil a symbol of an overly materialistic, insufficiently self-
sufficient society?" for example). In sum, what we call things affects how we do things; and
despite the lesson of Genesis, for mortals at least the future must be named before it can
be created. Language is thus always the crucial battlefield; it conserves or
liquidates tradition, it challenges or, champions established power paradigms
and it is the looking glass of all future vision. If language is alive, society can grow; if it is
dialectical, society can reconcile its parts-past and future no less than interest and interest or class and
class. As Jurgen Habermas has understood, democracy
means above all equal access to
language, and strong democracy means widespread and ongoing participation
in talk by the entire citizenry. Left to the media, the bureaucrats, the
professors, and the managers, language quickly degenerates into one more
weapon in the armory of elite rule. The professoriate and the literary establishment are all
too willing to capture the public with, catch phrases and portentous titles. How often in the past several
decades have Americans been made to see themselves, and thus their futures, through the lens of a
writer's book title? Recall The True Be liever, The Managerial Society, The End of Ideology, The Other
America, The Culture of Narcissism, The Greening of America, The Totalitarian Temptation, The
Technological Society, The Two Cultures, The Zero-Sum Society, Future Shock. We are branded by
words and our future is held hostage to bestseller lists'.195-197

Page 331 of 1481


Page 332 of 1481
Debate Solves Democratic Talk:
Ext
DEBATE SERVES AS A FORUM THROUGH WHICH WE CAN ENGAGE IN THE
DEMOCRATIC PROCESS

Watson, 04 (J.B. Watson, Assistant Professor of sociology and gerontology coordinator @ Stephen F. Austin State
University, A Justification of the Civic Engagement Model, p. 73-74, Service Learning: History, Theory, and Issues)

The civic engagement of ordinary citizens with voluntary associations, social institutions, and government
in local communities is a central feature of strong democracies. Further, a fundamental feature of
democratic governmental structure is its relationship to civil society, defined as "voluntary social activity
not compelled by the state" (Bahlmueller, 1997, p. 3). Through voluntary participation in civil society
associations at the local and regional level, citizens pursue activities that potentially serve the public good.
Through this rudimentary civic engagement, citizens learn the attitudes, habits, skills, and knowledge
foundational to the democratic process-(Patrick, 1998). Unfortunately, in 1998 the National Commission
on Civic Renewal (NCCR) highlighted the declining quantity and quality of civic engagement at all levels of
American life. A number of other studies concur on the decline of involvement in civic activities
(Bahlmueller, 1997; McGrath, 2001; Putnam, 1995). This concern about the nature and extent of civic
engagement in the United States has impacted the debate on the proper role of higher education in a
democracy. Higher education institutions, as transmitters of essential elements of the dominant culture,
struggle with the development of mechanisms to socialize the next generation about democratic values. A
national debate has emerged on the higher education response to this perceived need for revitalizing
constructive democratic engagement, building civil society, and increasing citizen participation in
government at all levels. Colleges and universities have responded with a number of civic engagement
initiatives, including university-community partnerships, empirical studies of political engagement,
community-based (collaborative) research, and the development of new (or expanded) service-learning
programs (Jacoby 2003).

A RENEWAL OF DEMOCRATIC TALK VIA COMMUNITY BASED ORGANIZATIONS IS


KEY TO CREATING A FOUNDATION FOR DEMOCRACY- ALLOWING US TO
INFLUENCE THE POLITICAL REALM

Cohen 03--Professor of Political Science at Columbia University( Jean L., Civic Innovation in America: Towards a
Reflexive Politics, The Good Society 12.1 (2003) 56-62,
http://muse.jhu.edu/journals/good_society/v012/12.1cohen.html)

Civic Innovation in America is a refreshing addition to what has become a growth industry of writing on
American civil society. Unlike the influential approach of Robert Putnam, this is not a backward-looking
lament about the decline of associational life, although Sirianni and Friedland are aware of the worrisome
signs of civic disaffection and citizen passivity in the U.S. 1Yet they don't join neo-communitarian efforts to
revive traditionalistic types of "mediating institutions" in order to secure social integration. 2Although not

Page 333 of 1481


adverse to mobilizing old forms of social capitalsuch as congregation-based community organizations
within and across denominational linesthey are primarily interested in networks that expand local
organizing capacities for new purposes and with fresh democratic methods. 3 Indeed, the focus of Civic
Innovation is on significant recent attempts "from below" to reinvent and revitalize American democracy.
Accordingly, the book points the reader to the ongoing public work of citizens and the actual processes of
civic innovation that have sprung up in recent years. The authors maintain that: "Over the past several
decades American society has displayed a substantial capacity for civic innovation, and the future of our
democracy will depend on whether we can deepen and extend such innovation to solve major public
problems, and transform the way we do politics." 4Theirs is a forward-looking approach: it highlights new
forms of cooperative civic participation in civil society and discusses the new modes of governance
needed to support them.

Page 334 of 1481


Page 335 of 1481
Democratic Talk Key to
Autonomy: Ext
THE DEMOCRATIC TALK THAT WE ARE CONDUCTING IS A NECESSARY CONDITION
FOR AUTONOMY GIVING UP POLITICAL TALK OF WHAT SHOULD BE DONE
ENSURES THAT VALUES AND BELIEFS WILL BECOME OSSIFIED

Barber, Professor of Political Science at Rutgers, 1984 (Benjamin, Strong Democracy: Participatory Politics for a
New Age)

6. Maintaining Autonomy. Talk helps us overcome narrow selfinterest, but it plays an equally significant role in
buttressing the autonomy of individual wills that is essential to democracy. It is through talk that we
constantly reencounter, reevaluate, and repossess the beliefs, principles, and maxims on the basis of
which we exert our will in the political realm. To be free, it is not enough for us simply to will what we
choose to will. We must will what we possess, what truly belongs to us. John Stuart Mill commented on the
"fatal tendency of mankind to leave all thinking about a thing when it is no longer doubtful." He ascribed to this
tendency "the cause of about half [men's] errors." Mindless convictions not only spawn errors, they turn those
who hold them into charlatans of liberty. Today's autonomously held belief is tomorrow's heteronomous
orthodoxy unless, tomorrow, it is reexamined and repossessed. Talk is the principal mechanism by which
we can retest and thus repossess our convictions, which means that a democracy that does not
institutionalize talk will soon be without autonomous citizens, though men and women who call themselves
citizens may from time to time deliberate, choose, and vote. Talk immunizes values from ossification and
protects the political process from rigidity, orthodoxy, and the yoke of the dead past. This, among all the
functions of talk, is the least liable to representation, since only the presence of our own wills working on a value can
endow that value with legitimacy and us with our autonomy. Subjecting a value to the test of repossession is a
measure of legitimacy as well as of autonomy: forced knowingly to embrace their prejudices, many men falter.
Prejudice is best practiced in the dark by dint of habit or passion. Mobs are expert executors of bigotry
because they assimilate individual wills into a group will and relieve individuals of any responsibility for
their actions. It is above all the imagination that dies when will is subordinated to instinct, and as we have seen, it is
the imagination that fires empathy. Values will, naturally, conflict even where they are thoughtfully embraced
and willed; and men's souls are sufficiently complex for error or even evil to dwell comfortably in the autonomous
man's breast. Autonomy is no guarantee against moral turpitude; indeed, it is its necessary condition. But in the social
setting, it seems evident that maxims that are continuously reevaluated and repossessed are preferable to
maxims that are embraced once and obeyed blindly thereafter. At a minimum, convictions that are
reexamined are more likely to change, to adapt themselves to altered circumstances and to evolve to
meet the challenges offered by competing views. Political willing is thus never a one-time or sometime
thing (which is the great misconception of the social-contract tradition), but an ongoing shaping and reshaping of
our common world that is as endless and exhausting as our making and remaking of our personal lives. A
moment's complacency may mean the death of liberty; a break in political concentration may spell the
atrophy of an important value; a pleasant spell of privatism may yield irreversible value ossification. Democratic
politics is a demanding business. Perhaps this is why common memory is even more important for democracy than for
other forms of political culture. Not every principle of conduct can be tested at every moment; not every conviction can
be exercised on every occasion; not every value can be regarded as truly ours at a given instant. Thus remembrance
and imagination must act sometimes as surrogates for the actual testing of maxims. Founding myths and the rituals
associated with them (July 14 in France or August 1 in Switzerland), representative political heroes who embody
admired convictions (Martin Luther King or Charles de Gaulle), and popular oral traditions can all revivify citizens'
common beliefs and their sense of place in the political culture. These symbols are no substitute for the citizenry's
active reexamination of values through participation in political talk, but they can and do supplement such talk through
the imaginative reconstruction of the past in live images and through the cultivation of beliefs that are not necessarily
involved in a given year's political business. 190-191

Page 336 of 1481


Page 337 of 1481
Democratic Talk Key to Checking
Right: Ext
FAILURE TO ENGAGE IN DEMOCRATIC TALK MEANS THE POLITICAL REALM WILL BE
DOMINATED BY THE FAR-RIGHT AND COLLAPSE INTO FASCISM, CAUSING WARS
AND TYRANNY

Rorty 98 (Richard, Stanford Philosophy Professor, Achieving Our Country, pp. 87-94)

if the pressures of globalization create such castes not only in the United States
If the formation of hereditary castes continues unimpeded, and

but in all the old democracies, we shall end up in an Orwellian world. In such a world, there may be no supernational analogue of Big
Brother, or any official creed analogous to Ingsoc. But there will be an analogue of the Inner Partynamely, the international, cosmopolitan super-rich. They will make all the
important decisions. The analogue of Orwells Outer Party will be educated, comfortably off, cosmopolitan professionalsLinds overclass, the people like you and me. The
job of people like us will be to make sure that the decisions made by the Inner Party are carried out smoothly and efficiently. It will be in the interest of the international super-
rich to keep our class relatively prosperous and happy. For they need people who can pretend to be the political class of each of the individual nation-states. For the sake of
keeping the proles quiet, the super-rich will have to keep up the pretense that national politics might someday make a difference. Since economic decisions are their
prerogative, they will encourage politicians, of both the Left and the Right, to specialize in cultural issues.7 The aim will be to keep the minds of the proles elsewhereto keep
the bottom 75 percent of Americans and the bottom 95 percent of the worlds population busy with ethnic and religious hostilities, and with debates about sexual mores. If the
proles can be distracted from their own despair by media-created psuedo-events, including the occasional brief and bloody war, the super-rich will have little to fear.
Contemplation of this possible world invites two responses from the Left. The first is to insist that the inequalities between nations need to be mitigatedand, in particular, that
to insist that the primary responsibility of each
the Northern Hemisphere must share its wealth with the Southern. The second is

democratic nation-state is to its own least advantaged citizens. These two responses obviously conflict with each other. In particular,
the first response suggests that the old democracies should open their borders, whereas the second suggests that they should close them.8 The first response comes naturally

comes naturally to members of trade unions, and


to academic leftists, who have always been internationally minded. The second response

to the marginally employed people who can most easily be recruited into right-wing populist movements.
Union members in the United States have watched factory after factory close, only to reopen in Slovenia, Thailand, or Mexico. It is no wonder that they see the result of
international free trade as prosperity for managers and stockholders, a better standard of living for workers in developing countries, and a very much worse standard of living for
American workers. It would be no wonder if they saw the American leftist intelligentsia as on the same side of the managers and stockholdersas sharing the same class
interests. For we intellectuals, who are mostly academics, are ourselves quite well insulated, at least in the short run, from the effects of globalization. To make things worse,
we often seem more interested in the workers of the developing world than in the fate of our fellow citizens. Many writers on socioeconomic policy have warned that the old

democracies are heading into a Weimar-like period, one in which populist movements are likely to
industrialized

overturn constitutional governments. Edward Luttwak, for example, has suggested that fascism may be the American future.
The point of his book The Endangered American Dream is that members of labor unions, and unorganized unskilled workers, will sooner or later realize that their government is
not even trying to prevent wages from sinking or to prevent jobs from being exported. Around the same time, they will realize that suburban white-collar workersthemselves

something will crack.


desperately afraid of being downsizedare not going to let themselves be taxed to provide social benefits for anyone else. At that point,

The nonsuburban electorate will decide that the system has failed and start looking around for a strongman to vote
forsomeone will assure them that, once he is elected, the smug bureaucrats, tricky lawyers, overpaid bond salemen, and postmodernist professors will no longer be
calling the shots. A scenario like that of Sinclair Lewis novel It Cant Happen Here may then be played out. For once such a strongman takes office, nobody can predict what
will happen. In 1932, most of the predictions made about what would happen if Hindenburg named Hitler chancellor were wildly overoptimistic. One thing that is very likely to

the gains made in the past forty years by black and brown Americans, and by homosexuals, will
happen is that

be wiped out. Jocular contempt for women will come back into fashion. The words nigger and kike will once again be heard in the workplace. All the
sadism which the academic Left has tried to make unacceptable to its students will come flooding back. All
the resentment which badly educated Americans feel about having their manners dictated to them by college graduates will find an outlet. But such a renewal of sadism will not

my imagined strongman takes charge, he will quickly make peace with the
alter the effects of selfishness. For after

international superrich, just as Hitler made with the German industrialists. He will invoke the glorious memory of the Gulf War to provoke military
adventures which will generate short-term prosperity. He will be a disaster for the country and the world.
People will wonder why there was so little resistance to his evitable rise. Where, they will ask, was the American Left? Why was it only rightists
like Buchanan who spoke to the workers about the consequences of globalization? Why could not the Left channel the mounting rage of the newly dispossesed? It is often

said that we Americans, at the end of the twentieth century, no longer have a Left. Since nobody denies the existence of what I have called the cultural Left, this
amounts to an admission that that Left is unable to engage in national politics. It is not the sort of the Left which can be asked to deal with the
consequences of globalization. To get the country to deal with those consequences, the present cultural Left would have to transform itself by opening relations with the
residue of the old reformist Left, and in particular with the labor unions. It would have to talk much more about money, even at the cost of talking less about stigma. I have two

suggestions about how to effect this transition. The first is that the Left should put a moratorium on theory. It should try to kick its philosophy
habit. The second is that the Left should try to mobilize what remains of our pride in being Americans. It should ask the public to consider how the country of Lincoln and
Whitman might be achieved. In support of my first suggestion, let me cite a passage from Deweys Reconstruction in Philosophy in which he expresses his exasperation with
the sort of sterile debate now going on under the rubric of individualism versus communitarianism. Dewey thought that all discussions which took this dichotomy seriously
suffer from a common defect. They are all committed to the logic of general notions under which specific situations are to be brought. What we want is light upon this or that
group of individuals, this or that concrete human being, this or that special institution or social arrangement. For such a logic of inquiry, the traditionally accepted logic
substitutes discussion of the meaning of concepts and their dialectical relationships with one another. Dewey was right to be exasperated by sociopolitical theory conducted at
this level of abstraction. He was wrong when he went on to say that ascending to this level is typically a rightist maneuver, one which supplies the apparatus for intellectual
justifications of the established order.9 For such ascents are now more common on the Left than on the Right. The contemporary academic Left seems to think that the higher
your level of abstraction, the more subversive of the established order you can be. The more sweeping and novel your conceptual apparatus, the more radical your critique.
When one of todays academic leftists says that some topic has been inadequately theorized, you can be pretty certain that he or she is going to drag in either philosophy of
language, or Lacanian psychoanalysis, or some neo-Marxist version of economic determinism. Theorists of the Left think that dissolving political agents into plays of differential
subjectivity, or political initiatives into pursuits of Lacans impossible object of desire, helps to subvert the established order. Such subversion, they say, is accomplished by

Page 338 of 1481


problematizing familiar concepts. Recent attempts to subvert social institutitons by problematizing concepts have produced a few very good books. They have also produced
many thousands of books which represent scholastic philosophizing at its worts. The authors of these purportedly subversive books honestly believe that the are serving
human liberty. But it is almost impossible to clamber back down from their books to a level of abstraction on
which one might discuss the merits of a law, a treaty, a candidate or a political strategy. Even though what these authors theorize
is often something very concrete and near at handa curent TV show, a media celebrity, a recent scandalthey offer the most absract and barren explanations imaginable.

These futile attempts to philosophize ones way into political relevance are a symptom of what happens
when a Left retreats from activism and adopts a spectatorial approach to the problems of its country.
Disengagement from practice produces theoretical hallucinations. These result in an intellec- tual environment which is, as Mark
Edmundson says in his book Nightmare on Main Street, Gothic. The cultural Left is haunted by ubiquitous specters, the most frightening of which is called "power." This is the
name of what Edmund- son calls Foucault's "haunting agency, which is everywhere and nowhere, as evanescent and insistent as a resourceful spook."10

Page 339 of 1481


Page 340 of 1481
Restoring Public Sphere Solves
Oppression: Ext
RESTORING THE PUBLIC SPHERE FACILITATES AN EMANCIPATORY PRAXIS OF OPEN
COMMUNITY

Lakeland 93 (Paul, professor of religious studies at Fairfield University, Preserving The Lifeworld, Restoring the
Public Sphere, Renewing Higher Education, Cross Currents, Winter, Vol. 43 Issue 4, p488, 15p
http://www.crosscurrents.org/lakeland2)

Habermas, then, is our third ally and resource. He describes the pathology of life in late capitalist societies
as the "colonization of the lifeworld by the system,"[4] and vests the hope of movement toward a newly
humane and democratic society in the "transformation of the public sphere."[5] The former phrase
expresses the conviction that distinctly human patterns of communication and interaction, which are in
principle open and even emancipatory, are under threat, progressively squeezed to the margins of
communal life by the more instrumental or manipulative model of interactions appropriate to technology
or to impersonal systems. By "the public sphere," Habermas means first the empirically discerned
historical phenomenon of a community of discourse in which rational discussion of matters of social and
political import took place, and influenced the formation of public policy. Secondly, he uses the term to
point toward the (perhaps counterfactual) possibility of creating something today that would serve to
protect the lifeworld from the depredations of the system or, more simply expressed, to preserve
democracy in late capitalist society. Habermas's view is not dissimilar to Frankl's. What Frankl saw
epitomized by the Nazi "final solution," namely, the systematic application of technology to eradicate the
sense of personal identity, Habermas sees as the logic of late capitalist, national security, consumerist
society. But where Frankl looks to inner spiritual resources to defeat these annihilating pressures,
Habermas turns to the dynamics of the speech-act. By so doing, incidentally, he strengthens Freire's
somewhat unfocused appeal to the "dialogical method" and shows why it is so potentially revolutionary.
For Habermas, the attempt to communicate directly with other human beings rests on a set of mutual
assumptions: there is something comprehensible to be heard; the speaker is sincere; the speaker seeks
truth; the hearer will listen; and so on. Even someone who attempts to deceive another can only hope to
do so because the hearer will assume the speaker is acting according to the rules of open communication.
Thus, the communication community is oriented in principle towards the "ideal speech situation," that is,
a context of distortion-free discourse in which all have equal access to the conversation, and all seek
consensus on norms for action. Though such an ideal speech situation may never exist, it operates
regulatively to draw communication onward. And what is assumed about the importance of truthfulness
and sincerity, and about the dignity of other speakers and hearers, makes communication, which is after
all the fundamental structure of human sociality, intrinsically emancipatory. The pathologies of personal,
communal, and political life become interpretable in terms of "systematically distorted communication,"
and overcoming them becomes a matter of restoring the contexts in which communicative praxis can
occur.

Page 341 of 1481


Page 342 of 1481
Talk is Action: Ext
TALK IS ACTION IT MAKES AND REMAKES THE WORLD IT DEFINES WHAT WE
ARE AS A COMMUNITY, WHAT WE WANT AND WHAT WE NEED

Barber, Professor of Political Science at Rutgers, 1984 (Benjamin, Strong Democracy: Participatory Politics for a
New Age)

Stripped of such artificial disciplines, however, talk


appears as a mediator of affection and
affiliation as well as of interest and identity, of patriotism as well as of
individuality. It can build community as well as maintain rights and seek
consensus as well as resolve conflict. It offers, along with meanings and significations, silences,
rituals, symbols, myths, expressions and solicitations, and a hundred other quiet and noisy manifestations of our
common humanity. Strong democracy seeks institutions that can give these things a voice-and an ear. The third
issue that liberal theorists have underappreciated is the complicity of talk in
action. With talk we can invent alternative futures, create mutual purposes,
and construct competing visions of community. Its potentialities thrust talk into
the realm of intentions and consequences and render it simultaneously more
provisional and more concrete than philosophers are wont to recognize. Their
failure of imagination stems in part from the passivity of thin democratic
politics and in part from the impatience of speculative philosophy with
contingency, which entails possibility as well as indeterminateness. But
significant political effects and actions are possible only to the extent that
politics is embedded in a world of fortune, uncertainty, and contingency.
Political talk is not talk about the world; it is talk that makes and remakes the world. The
posture of the strong democrat is thus "pragmatic" in the sense of William James's definition of pragmatism as "the
attitude of looking away from first things, principles, 'categories,' supposed necessities; and of looking toward last
things, fruits, consequences, facts." James's pragmatist "turns toward concreteness and adequacy, toward facts,
toward action, and toward power.... [Pragmatism thus] means the open air and possibilities of nature, as against
dogma, artificiality and the pretense of finality in truth." Strong
democracy is pragmatism
translated into politics in the participatory mode. Although James did not pursue the powerful
political implications of his position, he was moved to write: "See already how democratic [pragmatism] is. Her
manners are as various and flexible, her resources as rich and endless, and her conclusions as friendly as those of
mother nature." The active, future-oriented disposition of strong democratic talk
embodies James's instinctive sense of pragmatism's political implications. Future action,
not a priori principle, constitutes such talk's principal (but not principled) concern. 177-178

Page 343 of 1481


Page 344 of 1481
**Performance**

Page 345 of 1481


A2 Performativity (1/2)
THE PERFORMANCE IS ALWAYS ALREADY TAKING PLACE. THE EXISTENCE OF THE
ROUND IS THE PERFORMANCE, NOT SPECIFIC SPEECHES

Jessica Kulynych, Asst Professor of Political Science at Winthrop University, Polity, Winter, 1997, n2 p315(32)

We bring normativity to our performances as ethical principles that are themselves subject to resistance. By unearthing the contingency of the "self-evident," performative

, the question is not should we resist (since resistance is always, already present), but
resistance enables politics. Thus

rather what and how we should resist. This notion of performativity is also important for understanding the possibilities for innovation in Habermasian
deliberative participation. Just as a protestor exposes the contingency of concepts like justice, a dialogue exposes the limits and contingency of rational argumentation. Once
we are sensitive to the performative nature of speech, language and discourse, then we can see that deliberative politics cannot be confined to the rational statement of validity

claims.Deliberation must be theatrical: it is in the performance of deliberation that that which cannot be argued for finds expression. Indeed it is
precisely the non-rational aspects of deliberation that carry the potential for innovation. In his description of the
poignant reminders of demonstration Chaloupka recognizes that it is at the margins that the actual force of the

demonstration resides, no matter what happens at the microphone. The oral histories of demonstrations
(the next day over coffee) linger over the jokes and funny signs and slogans, the outrages and
improprieties, more than the speeches and carefully coherent position papers. (68)

PERFORMANCE IS ALWAYS CONTEXT-DEPENDENT. OUR CRITICISM CAN ONLY BE


EVALUATED IN THE CONTEXT OF DEBATE

Jessica Kulynych, Asst Professor of Political Science at Winthrop University, Polity, Winter, 1997, n2 p315(32)

Consequently, a performative concept of political participation changes debates within the traditional
participation literature over the inclusion of protest activities and community decisionmaking in the
definition of political participation. While these debates have generally been conducted on familiar
terrain, justifying the inclusion of such activity by delineating its impact on the distribution of goods,
services, or political power by the government, a performative concept of participation breaks down this
distinction altogether.(75) Because performative participation is defined by its relation to a set of
normalizing disciplinary rules and its confrontation with those rules, nothing can be categorically excluded
from the category of political participation. As Honig eloquently puts it, "not everything is political on this
(amended) account; it is simply the case that nothing is ontologically protected from politicization, that
nothing is necessarily or naturally or ontologically not political."(76) Therefore, the definition of political
participation is always context dependent; it depends upon the character of the power network in which
it is taken. Political participation is not categorically distinguished from protest or resistance, but rather
the focus is on the disruptive potential of an action in a particular network of power relations. To say that
participation is context dependent means not only that any action is potentially participation, but also
that no particular action is necessarily a participatory act. Housecleaning is a good example. The character
of the power network in which one exists defines housecleaning as a potential act of political
participation. In her description of the defensive strategies of Black women household workers, Bonnie
Thorton Dill argues that the refusal to mop the floor on hands and knees, or the refusal to serve an extra
dinner, constitutes an effective act of resistance.(77) It is not the act itself that is politically definitive, but
rather the context. Black domestic laborers, who in this context are constructed as desperate, willing to
do any type of work, and always immediately available for service, resist that construction by acting as if
they have other choices. Thus it is the context of the domestic labor relationship that defines the
repertoire of political actions. Similarly, Jonathan Kozol describes poor welfare mothers living in the
degrading conditions of the South Bronx whose homes "no matter how besieged, are nonetheless kept
spotless and sometimes even look cheerful."(78) For women who are constructed as thoroughly

Page 346 of 1481


dependent, irresponsible, unfit, and unclean, cleaning the house takes on the character of resistance; it
becomes a political act. Housecleaning itself is not necessarily political, rather, the disciplinary context of
a gendered social welfare state gives political import to seemingly banal, everyday activities.

Page 347 of 1481


Page 348 of 1481
A2 Performativity (2/2)
COALITIONS MUST PRECEDE VICTORY THROUGH PERFORMANCE

Jessica Kulynych, Asst Professor of Political Science at Winthrop University, Polity, Winter, 1997, n2 p315(32)

A performative perspective on participation enriches our understanding of deliberative democracy. This


enlarged understanding can be demonstrated by considering the examination of citizen politics in
Germany presented in Carol Hager's Technological Democracy: Bureaucracy and Citizenry in the West
German Energy Debate.(86) Her work skillfully maps the precarious position of citizen groups as they
enter into problemsolving in contemporary democracies. After detailing the German citizen foray into
technical debate and the subsequent creation of energy commissions to deliberate on the long-term goals
of energy policy, she concludes that a dual standard of interpretation and evaluation is required for full
understanding of the prospects for citizen participation. Where traditional understandings of participation
focus on the policy dimension and concern themselves with the citizens' success or failure to attain policy
preferences, she advocates focusing as well on the discursive, legitimation dimension of citizen action.
Hager follows Habermas in reconstituting participation discursively and asserts that the legitimation
dimension offers an alternative reason for optimism about the efficacy of citizen action. In the discursive
understanding of participation, success is not defined in terms of getting, but rather in terms of solving
through consensus. Deliberation is thus an end in itself, and citizens have succeeded whenever they are
able to secure a realm of deliberative politics where the aim is forging consensus among participants,
rather than achieving victory by some over others.

Page 349 of 1481


Performance is Commodified (1/2)

THEIR POETRY SUPPORTS THE CULTURE INDUSTRY. IT IS MANUFACTURED DISSENT

Dr. Lee Spinks lectures in English Literature at the University of Edinburgh in Scotland, Writing, Politics, and the
Limit: Reading J. H. Prynne's "The Ideal Star-Fighter," Intertexts, Fall 2000 v4 i2 p144(23)

It would be easy to conclude from passages like this that avant-garde styles of writing which foreground
the production of subject positions within the discursive configuration of a text are necessarily subversive
of established political order because they forestall the "reconciliation of the general and particular, of
the rule and the specific demands of the subject matter" that underpins the systematic totality of the
culture industry. This belief in the inherently subversive effect of textual polyphony and difference
underscores Easthope's reading of modernist poetics. But the matter is not so simple. For as Adorno and
Horkenheimer demonstrate, incommensurable or "refractory material" is always and everywhere
implicated in a dialectical relationship with the "total process of production" that it opposes (Adorno and
Horkheimer xii). One of their more melancholy insights is that the culture industry actively produces
different images and styles in order to reassert the absolute uniformity of its own authority. Novelty is all
around us, from the "standardized jazz improvisation to the exceptional film star whose hair curls over
her eye to demonstrate her originality" but what is individual here "is no more than the generality's
power to stamp the accidental detail so firmly that it is accepted as such" (Adorno and Horkheimer 154).
The "accidental" or incommensurable detail is "accepted as such" because it can be endlessly reproduced
as a "house style" or "lifestyle practice" and, paradoxically, it is the capacity of the culture industry to
transform difference into a set of uniform discriminations that allows a social body to be demarcated
according to the sectional logic of politicians, advertisers and marketing executives. Fredric Jameson
makes exactly the same point when he observes that what has happened in the contemporary or
postmodern phase of monopoly capitalism is "that aesthetic production today has become integrated into
commodity production generally: the frantic economic urgency of producing fresh waves of producing
ever more novel-seeming goods (from clothing to airplanes), at ever greater rates of turnover, now
assigns an increasingly essential structural function and position to aesthetic innovation and
experimentation" (Jameson 4-5). It is therefore inadequate to proclaim the ineluctable emancipatory
promise of incommensurable or refractory material because "capitalism also produces difference or
differentiation as a function of its own internal logic" (Jameson 406).

CHALLENGES TO CONFORMITY ONLY CEMENT THE OVER-ARCHING CONTROL OF


THE DOMINANT LANGUAGE

Dr. Lee Spinks lectures in English Literature at the University of Edinburgh in Scotland, Writing, Politics, and the
Limit: Reading J. H. Prynne's "The Ideal Star-Fighter," Intertexts, Fall 2000 v4 i2 p144(23)

The central claim of this essay is that these critical debates concerning the dialectic between totality and
difference in modern cultural production provide the most rewarding context within which to discuss the
relationship between textuality and politics in Prynne's poetry. For Prynne's work takes as its subject the
very status of writing, and the epistemological practices writing both produces and brings into question, in
a cultural sphere dominated by the power of instrumental reason to enforce a principle of "equivalence"
where "whatever does not conform to the rule of computation and utility is suspect" (Adorno and
Horkheimer 6). The importance of style, or the mode of relation between thought and its representation,

Page 350 of 1481


to this question becomes apparent when we consider that the failure to challenge this universal principle
of equivalence means to accept that the "identity of everything with everything else is paid for in that
nothing may at the same time be identical with itself" (Adorno and Horkheimer 12). Yet any challenge to
this process of abstraction and exchange based upon the formal autonomy or "difference" of style is
vulnerable to Adorno's charge that it is through difference and exchange "that non-identical individuals
and performances become commensurable and identical" (Adorno, Negative Dialectics 146-47).

Page 351 of 1481


Page 352 of 1481
Performance is Commodified
(2/2)
POETIC RESISTANCE IS DIRECTED BY THE CULTURE INDUSTRY

Dr. Lee Spinks lectures in English Literature at the University of Edinburgh in Scotland, Writing, Politics, and the
Limit: Reading J. H. Prynne's "The Ideal Star-Fighter," Intertexts, Fall 2000 v4 i2 p144(23)

Prynne's difficult and dialectical style in fact proposes two points of resistance to the principle of
equivalence enforced by instrumental rationality and the culture industry. Both may be explicated by
reference to Adorno's assertion that the work of art is a "fetish against commodity fetishism" (Adorno,
Aesthetic Theory 227). The fetishistic element within art, according to Adorno, lies in its illusory claim that
its value is integral to itself rather than an effect of consumption and exchange. This insistence of the
artwork upon its autonomy as a source of value, and the cultivation of styles and modes of reference that
place it at one remove from the world around it, is often identified as the origin of the 'elitism' of
modernist art. But if we reconsider the entire question of modernist style in the context of the
remorseless conversion of use or human labor value into exchange value effected by late capitalism, then
the conviction of the modernist artwork that it conceals an autonomous and non-exchang- eable source
of value offers a challenge to prevailing political and cultural conditions. For it is only by "persisting with
its illusory claim to a non-exchangeable dignity" argues Simon Jarvis, that "art resists the notion that the
qualitatively incommensurable can be made qualitatively commensurable" (Jarvis 117). This is the
artwork's first point of resistance to the principle of equivalence within commodity production. Yet it
might still be objected that far from challenging the commodification of culture, the autonomous
character of the artwork is instead produced by capitalism, which enables both art and artistic labor to be
alienated from any broader social or cultural purpose.

Page 353 of 1481


Performance Fails

FAITH IN PERFORMANCE IS NAVE AND FAILS TO CHANGE POLITICS

Rothenberg & Valente 97


[Molly Anne, Assoc. Prof. English @ Tulane, & Joseph, Prof. @ Illinois, Performative Chic: The Fantasy of a
Performative Politics, College Literature 24: 1, February, ASP]

The recent vogue for performativity, particularly in gender and postcolonial studies, suggests that the
desire for political potency has displaced the demand for critical rigor.[1] Because Judith Butler bears the
primary responsibility for investing performativity with its present critical cachet, her work furnishes a
convenient site for exposing the flawed theoretical formulations and the hollow political claims advanced
under the banner of performativity. We have undertaken this critique not solely in the interests of
clarifying performativity's theoretical stakes: in our view, the appropriation of performativity for purposes
to which it is completely unsuited has misdirected crucial activist energies, not only squandering
resources but even endangering those naive enough to act on performativity's (false) political promise.

It is reasonable to expect any practical political discourse to essay an analysis which links its proposed
actions with their supposed effects, appraising the fruits of specific political labors before their seeds are
sown. Only by means of such an assessment can any political program persuade us to undertake some
tasks and forgo others. Butler proceeds accordingly: "The task is not whether to repeat, but how to repeat
or, indeed to repeat, and through a radical proliferation of gender, to displace the very gender norms that
enable repetition itself" (Gender Trouble 148). Here, at the conclusion to Gender Trouble, she makes
good her promise that subjects can intervene meaningfully, politically, in the signification system which
iteratively constitutes them. The political "task" we face requires that we choose "how to repeat" gender
norms in such a way as to displace them. According to her final chapter, "The Politics of Parody," the way
to displace gender norms is through the deliberate performance of drag as gender parody.

Page 354 of 1481


**Link Answers: General**
A2 The Case is Apolitical/Has No Theory

THE DISTINCTION BETWEEN THEORY AND PRACTICE IS FALSE BOTH FORMS OF


POLITICAL ACTION INVOLVE AND DEPEND ON THE OTHER

Homi K. Bhabha, Professor, University of Sussex, THE LOCATION OF CULTURE, 1994, p. 21-22.

Committed to what? At this stage in the argument, I do not want to identify any specific
'object' of political allegiance - the Third World, the working class, the feminist struggle.
Although such an objectification Of Political activity is crucial and must significantly inform
political debate, it is not the only option for those critics or intellectuals who are committed to
progressive political change in the direction of a socialist society. It is a sign of political maturity
to accept that there are many forms of Political writing whose different effects are obscured
when they are divided between the 'theoretical' and the 'activist'. It is not as if the leaflet
involved in the organization of a strike is short on theory, while a speculative article on the
theory of ideology ought to have more practical examples or applications. They are both forms
of discourse and to that extent they produce rather than reflect their objects of reference. The
difference between them lies in their operational qualities. The leaflet has a specific expository
and organizational purpose, temporally bound to the event; the theory of ideology makes its
contribution to those embedded political ideas and principles that inform the right to strike.
The latter does not justify the former; nor does it necessarily precede it. It exists side by side
with it - the one as an enabling part of the other - like the recto and verso of a sheet of paper,
to use a common serniotic analogy in the uncommon context of politics.

Page 355 of 1481


**Alternative Answers: General**
Individual Action Fails

THE ALTERNATIVE ALONE WILL FAIL. THE NATURE OF DISCOURSE AND DOMINANT
RECONTEXTUALIZATION PREVENTS INDIVIDUALS FROM SOLVING

D. Franklin Ayers 2005


The Review of Higher Education, 28.4, Neoliberal Ideology in Community College Mission Statements: A
Critical Discourse Analysis

Because discourses are determined by higher levels of social structuring, textssuch as community
college mission statementsand the discourses they represent are not created entirely by individuals.
Instead, individual producers of text can only choose among the discursive options available at higher
levels of social structuring. Because no ideology is monolithic, multiple discourses exist and are available
to producers of text, although hegemonic discourses may make alternatives nearly imperceptible.
Because discourses reflect ideologies of groups with unequal power resources and because the producer
of text must choose among these discourses, he or she engages in a negotiation of power relations. [End
Page 534]

To the degree that powerful groups act upon discourses at various levels of social structuring, their
ideologies and world views gain authority. Dominant discourses consequently determine the meanings
assigned to social and material processes, and they do this in ways that reinforce power inequities. One
way that meanings may be determined is through recontextualization (Fairclough, 1995).
Recontextualization is a process in which the discourse related to one social process dominates or
colonizes the discourse related to another social process.

Page 356 of 1481


Page 357 of 1481
Mann
THE CONTEXT OF DEBATE COOPTS THE CRITICISM SINCE IT IS ANTICIPATED AND
FOOTNOTED ALTERNATIVE TACTICS WOULD BE NECESSARY FOR IT TO HAVE AN
EFFECT

Paul Mann, professor of comparative literature at Pomona college, Masocriticism, 1999, pg. 106-
107.

Without exception, all positions are oriented toward the institutional apparatus. Marginality here is only
relative and temporary: the moment black studies or womens studies or queer theory conceives of itself
as a discipline, its primary orientation is toward the institution. The fact that the institution might treat it
badly hardly constitutes an ethical privilege. Any intellectual who holds a position is a function of this
apparatus; his or her marginality is, for the most part, only an operational device. It is a critical
commonplace that the state is not a monolithic hegemony but rather a constellation of disorganized and
fragmentary agencies of production. This is often taken as a validation for the political potential of
marginal critical movements: inside-outside relations can be facilely deconstructed, and critics can still
congratulate themselves on their resistance, but the contrary is clearly the case. The most profitable
intellectual production does not take place at the center (e.g., romance philology), where mostly obsolete
weapons are produced; the real growth industries are located precisely on the self-proclaimed margins. It
will be argued that resistance is still possible, and nothing I propose here argues against such a possibility.
I wish only to insist that effective resistance will never be located in the position, however oppositional it
imagines itself to be. Resistance is first of all a function of the apparatus itself. What would seem to be the
transgressive potential of such institutional agencies as certain orders of gender criticism might
demonstrate the entropy of the institution, but it does nothing to prove the counterpolitical claims of the
position. Fantasies of resistance most often serve as mere alibis for collusion. Any position is a state
agency, and its relative marginality is a mode of orientation, not an exception. Effective resistance must
be located in other tactical forms

CRITICISM CAN NEVER BE MAINTAINED AND IS IGNORED BECAUSE OF ITS PROLIFIC


NATURE

Paul Mann, professor of comparative literature at Pomona college, Masocriticism, 1999, pg. 16-
17.

The avant-garde, which always began in brilliant refusals and destructions, must in the end abandon those
economies that, with frightening efficiency, have put it to use, made it instrumental, profited from it,
developed ways to get a return even from negation, even from the death drive itself. In the light of the sun of
expenditure, such a culture seems the narrowest of misconceptions. Imagine instead that the vast proliferation of writing,
drawing, painting, performancenot just what cultures have preserved for us through the filtration systems of their own values, but all
writing, all music, and so onis the actual, lived field of culture; that culture is waste, expenditure: productivity
and destruction without any exclusion or discrimination; that all of these works have been produced not
so that a few precious articles of value, the best that has been known and thought, can, through a sort
of reasoned brokerage, be conserved as culture per se, but so that they would be destroyed; that what is
most important about all of those poems and paintings and constructions is precisely that the vast

Page 358 of 1481


majority of them disappear even as they are born, that they dismember and consume themselves without
our ever knowing them, vanish in the air, into the death they most desired, never to be remembered
again. Imagine a writing that saw itself in this light, a light that never shines on most of what we call culture, that never consigns itself to productive
discourse but always escapes, that is valuable only because it escapes, because it is elsewhere, nowhere. Or imagine a certain book: it arrives uncalled for, un-
predicted, perhaps in the mail, perhaps fallen from the sky, unmarked by a publishers apparatus, by advertising, even by an authors name; a book made of
white noise that erases itself as it goes along and everything you say for weeks is stolen from it; a book that you cut into pieces and disseminate at random (on
the street, on walls, through the mail) or that you burn without having read it and scatter the ashes to the four winds; or imagine such a book that you never
receive in the first place. Perhaps that is the useless book one must learn to write, that is the only book one ever writes. Or perhaps it is precisely a book one
cannot write, but only imagine, and in imagining it call it down upon ones writing, to tear ones own writing apart. As this talk, this argument that began at
cross-purposes and went nowhere, unravelling itself as it proceeded, even now beginning to cease vibrating in the air, will soon vanish,
leaving nothing
but a fading imprint on your memories, soon to be effaced as you turn toward more productive labors,
and itself only the trace of an expenditure whose disappearance it briefly betrayed

Page 359 of 1481


Page 360 of 1481
Power Vaccuum
POWER IS ZERO SUM THE ALTERNATIVE ONLY SHIFTS POWER ELSEWHERE

John Mearsheimer, Professor at University of Chicago, 2001 (The Tragedy of Great Power Politics p. 34)

Consequently, states pay close attention to how power is distributed among them, and they make a special effort to maximize
their share of world power. Specifically, they look for opportunities to alter the balance of power by acquiring
additional increments of power at the expense of potential rivals. States employ a variety of meanseconomic,
diplomatic, and militaryto shift the balance of power in their favor, even if doing so makes other states suspicious or even
hostile. Because one states gain in power is another states loss, great powers tend to have a zero-sum
mentality when dealing with each other. The trick, of course, is to be the winner in this competition and to dominate
the other states in the system. Thus, the claim that states maximize relative power is tantamount to arguing that states are
disposed to think offensively toward other states, even though their ultimate motive is simply to survive. In short, great
powers have aggressive intentions.

Page 361 of 1481


Page 362 of 1481
Page 363 of 1481
**SPECIFIC K ANSWERS**

Page 364 of 1481


**Apocalyptic Rhetoric**

Page 365 of 1481


Perm Solvency
PERM: DO BOTH EVEN YOUR AUTHOR CONCEEDS THAT APOCALYPTIC RHETORIC
USED AWAY FROM RELIGIOUS FORM IS KEY TO SPUR ACTIVISM AND SOCIAL
CHANGE- ITS KEY TO AVOIDING TYRANNY

QUINBY in 1994
[Lee, Anti-Apocalypse, http://www.dhushara.com/book/renewal/voices2/quin/quinby.htm //wyo-pinto]

I am not saying that this is all bad. Precisely because it is on tap in the United States, it is
possible for apocalyptic ideas to aid struggles for democracy by exciting people toward activism.
This is the force of Cornet West's warning about ,this country's failures in creating a multiracial
democracy: "Either we learn a r;ew language of empathy and compassion, or the fire this time
will consume us all. , But even when apocalyptic imagery is used to fight racist suppressions of
freedom, as with West's allusion to James Baldwin's warning, it runs the risk of displacing
concrete political analysis. While advocating a new kind of leadership "grounded in grass-roots
organizing that highlights democratic accountability," West's insistence that if we don't learn
this lesson the fire will consume us all is the kind of hyperbole that undermines his own earlier
analysis of local devastation. People in positions of privilege can, and clearly do, dismiss the
threat to their own way of life as by and large inaccurate.

At stake here are the relationships between power, truth, ethics, and apoca@pse. In attempting to
represent the unrepresentable, the unknowable-the End, or death par excellence -apocalyptic writings
are a quintessential technology of power/knowledge. They promise the defeat of death, at least for the
obedient who deserve everlasting life, and the prolonged agony of destruction for those who have not
obeyed the Law of the Father. One does not have to succumb to apocalyptic eschatology to understand
why end-time propensities imperil democracy: the apocalyptic tenet of preordained history disavows
questionings of received truth, discredits skepticism, and disarms challengers of the status quo. Appeals
to the Day of judgment, the dawn of a New Age, even the dream of a cryogenic "return" to life, put off the
kinds of immediate political and ethical judgments that need to be made in order to resist both overt
domination and the more seductive forms of disciplinary power operative in the United States today and
fostered by the United States in other countries.

Page 366 of 1481


Apocalyptic Rhetoric Good (1/3)

ONLY BY CONFRONTING THE APOCALYPSE CAN WE EXPOSE THE CONTRADICTIONS


WITHIN THE SYSTEM OF THE BOMB, OUR APOCALYPTIC RHETORIC IS KEY

MODERN AMERICAN POETRY NO DATE


[from Thomas McClanahan's "Gregory Corso", http://www.english.uiuc.edu/maps/poets/a_f/corso/bomb.htm
//wyo-pinto]

Although it can be read as a polemic against nuclear war, "Bomb" is also an examination of the
loss of humanistic virtue. Additionally, it is a vehicle for expressing Corso's developing
epistemology. To know the world, for the younger poet, is to recognize it as a Heraclitean
continuum, an alteration of consciousness that prefigures the way man understands himself and
the world about him. Like the bomb, powerful forces--whether they are generated by great
religious prophets or authentic poetic statement--provide the elemental energy that transforms
human consciousness. So Corso's poem is a paradoxical rendering of two points of view: on the
one hand it is about the destructive power of a weapon that can annihilate mankind, while at
the other extreme it concerns the positive force of man's own potential to see the world from a
new perspective.

CONFRONTING THE APOCALYPSE CAUSES SOCIAL TRANCENDENCE- ITS THE ONLY


WAY TO RESCUE PEOPLE

WINK in 2001
[Walter, nqa, Apocalypse Now? Christian Century, Oct 17,
http://www.findarticles.com/p/articles/mi_m1058/is_28_118/ai_79514992 //wyo-pinto]

If that were the whole story about apocalyptic, many of us would want nothing to do with it. That is not
the whole story, however. There is a positive role for apocalyptic as well as its better-known negative. The
positive power of apocalyptic lies in its capacity to force humanity to face threats of unimaginable
proportions in order to galvanize efforts at self and social transcendence. Only such Herculean responses
can actually rescue people from the threat and make possible the continuation of humanity on the other
side. Paradoxically, the apocalyptic warning is intended to remove the apocalyptic threat by acts of
apocalyptic transcendence.

Page 367 of 1481


Page 368 of 1481
Apocalyptic Rhetoric Good (2/3)
CONFRONTING THE APOCALYPSE CREATES A FEARLESS FEAR THAT INCITES ACTION
AGAINST WHAT IS SAID AS INEVITABLE- THIS FEARLESS FEAR IS KEY TO ACTION AS
OPPOSED TO THE INACTION OF THE CURRENT SYSTEM- A CALL FOR INACTION
PARALYZES*****

WINK in 2001 [Walter, nqa, Apocalypse Now? Christian Century, Oct 17,
http://www.findarticles.com/p/articles/mi_m1058/is_28_118/ai_79514992 //wyo-pinto]

Positive apocalyptic, by contrast, calls on our every power to avert what seems inevitable.
"Nothing can save us that is possible," the poet W. H. Auden intoned over the madness of the
nuclear crisis; "we who must die demand a miracle." And the miracle we got came about
because people like the physician Helen Caldicott refused to accept nuclear annihilation. But she
did it by forcing her hearers to visualize the consequences of their inaction.

Imagination, says Anders, is the sole organ capable of conveying a truth so overwhelming that
we cannot take it in. Hence the bizarre imagery that always accompanies apocalyptic. Optimists
want to believe that reason will save us. They want to prevent us from becoming really afraid.
The anti-apocalyptist, on the contrary, insists that it is our capacity to fear which is too small and
which does not correspond to the magnitude of the present danger. Therefore, says Anders, the
anti-apocalyptist attempts to increase our capacity to fear. "Don't fear fear, have the courage to
be frightened, and to frighten others too. Frighten thy neighbor as thyself." This is no ordinary
fear, however; it is a fearless fear, since it dares at last to face the real magnitude of the danger.
And it is a loving fear, since it embraces fear in order to save the generations to come. That is
why everything the anti-apocalyptist says is said in order not to become true.

If we do not stubbornly keep in mind how probable the disaster is and if we do not act
accordingly, we will not be able to prevent the warnings from becoming true. There is nothing
more frightening than to be right. And if some amongst you, paralyzed by the gloomy likelihood
of the catastrophe, should already have lost their courage, they, too, still have the chance to
prove their love of man by heeding the cynical maxim: "Let's go on working as though we had
the right to hope. Our despair is none of our business."

Page 369 of 1481


Page 370 of 1481
Apocalyptic Rhetoric Good (3/3)
WE MUST TAKE ACTION IN THE FACE OF THE REAL APOCALYPSES- GLOBAL
WARMING, THE OZONE HOLE, WAR, POLLUTION, NUCLEAR WAR- THE THREATS
WONT GO AWAY ******

WINK in 2001 [Walter, nqa, Apocalypse Now? Christian Century, Oct 17,
http://www.findarticles.com/p/articles/mi_m1058/is_28_118/ai_79514992 //wyo-pinto]

It is not difficult to see in that warning perils that threaten the very viability of life on earth today. Global
warming, the ozone hole, overpopulation, starvation and malnutrition, war, unemployment, the
destruction of species and the rain forests, pollution of water and air, pesticide and herbicide poisoning,
errors in genetic engineering, erosion of topsoil, overfishing, anarchy and crime, the possibility of a
nuclear mishap, chemical warfare or all-out nuclear war: together, or in some cases singly, these dangers
threaten to "catch us unexpectedly, like a trap." Our inability thus far to measure ourselves against these
threats is an ominous portent that apocalypse has already rendered us powerless.

Terrible as it was, the destruction of the World Trade Center was not an apocalypse. That horror will
slowly recede. Other acts of infamy may take place. But we can anticipate a time when terrorism will
decline. Nor are we helpless. We have the means to stop at least many, perhaps even most, of the
terrorist attacks hurled at us. But we can see the other side of this catastrophe, when life feels normal
again.

The threats to our very survival that I listed above, however, will not go away. They could well spell the
end of humanity, and even of most sentient life. This is the awful truth that we have yet to recognize: We
are living in an apocalyptic time disguised as normal, and that is why we have not responded
appropriately. If we are in the midst of the sixth great extinction, as scientists tell us we are, our response
has in no way been commensurate with the danger. We Homo sapiens are witnessing the greatest
annihilation of species in the last 65 million years, and our children may live to witness ecocide with their
own eyes. So while we are understandably preoccupied with terrorism, and must do everything necessary
to stamp it out, we must at the same time wake up to these more serious threats that could effectively
end life on this planet.

SOUTH AFRICA PROVES THAT OUR MODEL OF APOCALYPSE WORKS- WE MUST


INCITE ACTION

WINK in 2001 [Walter, nqa, Apocalypse Now? Christian Century, Oct 17,
http://www.findarticles.com/p/articles/mi_m1058/is_28_118/ai_79514992 //wyo-pinto]

BUT THE VERDICT is not yet in. It is late, but a positive response to the real apocalypse of our
time is still possible. Consider South Africa. When I was there in the 1980s, it appeared that
armed revolution was inevitable. Blacks were becoming more desperate by the day. Teenage
boys were confronting the police and army without concern for their safety. Chaos was
beginning to overtake the townships, as children, outraged by the timorousness of their

Page 371 of 1481


parents, seized the initiative themselves. Whites were taking an increasingly hard line. It was a
recipe for disaster. The whole scene reeked of an apocalypse of the negative sort.

Then the most unexpected thing happened. The white government chose, under intense internal and
international pressure, to relinquish power, and negotiated with its former black enemies a process that
led to the election of a black president, a model constitution, and relatively low casualties, considering the
alternatives. No one to my knowledge anticipated this turn of events. What had appeared as an inevitable
(negative) apocalyptic bloodbath turned out to have been a (positive) apocalyptic situation instead,
thanks to the "anti-apoca

Page 372 of 1481


Page 373 of 1481
**Badiou**

Page 374 of 1481


A2 Badiou: 2AC
EVERY AFFIRMATIVE ETHICAL STANCE REQUIRES A REPRESSED ELEMENT OF
NEGATION, MEANING THAT EVERY AFFIRMATION OF LIFE OCCURS AGAINS THE
BACKGROUND OF HUMN DEATH AND FINITUDE

Zizek '99
[Slavoj, Senior Researcher at Institute for Social Studies, Ljubliana and Badass, The Ticklish Subject: the
absent centre of political ontology, New York: Verso, 1999, 153-4//uwyo-ajl]

It would therefore be tempting to risk a Badiouian-Pauline reading of the end of psychoanalysis, determining it as a
New Beginning, a symbolic 'rebirth' - the radical restructuring of the analysand's subjectivity in such a way that the
vicious cycle of the superego is suspended, left behind. Does not Lacan himself provide a number of hints that the end
of analysis opens up the domain of Love beyond Law, using the very Pauline terms to which Badiou refers?
Nevertheless, Lacan's way is not that of St Paul or Badiou: psychoanalysis is not 'psychosynthesis'; it does not
already posit a 'new harmony', a new Truth-Event; it - as it were - merely wipes the slate clean for one.
However, this 'merely' should be put in quotation marks, because it is Lacan's contention that, in this
negative gesture of 'wiping the slate clean', something (a void) is confronted which is already 'sutured'
with the arrival of a new Truth-Event. For Lacan, negativity, a negative gesture of withdrawal, precedes
any positive gesture of enthusiastic identifiction with a Cause: negativity functions as the condition of
(im)possibility of the enthusiastic identification - that is to say, it lays the ground, opens up space for it,
but is simultaneously obfuscated by it and undermines it. For this reason, Lacan implicitly changes the balance between Death and
Resurrection in favour of Death: what 'Death' stands for at its most radical is not merely the passing of earthly life, but
the 'night of the world', the self-withdrawal, the absolute contraction of subjectivity, the severing of its
links with 'reality' - this is the 'wiping the slate clean' that opens up the domain of the symbolic New
Beginning, of the emergence of the 'New Harmony' sustained by a newly emerged Master-Signifier. Here, Lacan parts company with St Paul and Badiou:
God not only is but always-already was dead - that is to say, after Freud, one cannot directly have faith in a Truth-Event; every such Event
ultimately remains a semblance obfuscating a preceding Void whose Freudian name is death drive. So Lacan
differs from Badiou in the determination of the exact status of this domain beyond the rule of the Law. That is to say: like Lacan, Badiou
delineates the contours of a domain beyond the Order of Being, beyond the politics of service des biens, beyond the 'morbid' super ego
connection between Law and its transgressive desire. For Lacan, however, the Freudian topic of the death drive cannot be accounted for
in the terms of this connection: the 'death drive'
is not the outcome of the morbid confusion of Life and Death
caused by the intervention of the symbolic Law. For Lacan, the uncanny domain beyond the Order of
Being is what he calls the domain 'between the two deaths', the pre-ontologicalf domain of monstrous
spectral apparitions, the domain that is 'immortal', yet not in the Badiouian sense of the immortality of
participating in Truth, but in the sense of what Lacan calls lamella, of the monstrous 'undead' object-
libido.18

Page 375 of 1481


Page 376 of 1481
Perm Solvency (1/3)
WE SHOULD COMBINE THE PLAN AND THE ALTERNATIVE THIS IS THE ONLY WAY
TO SOLVE THE CASE WHILE MAINTAINING AN AFFIRMATIVE CONCEPTION OF
ETHICS OUTSIDE THE BOUNDS OF THE STATE

Hallward, Lecturer in the French department @ Kings College, 2K2 (Peter BADIOU'S POLITICS: EQUALITY
AND JUSTICE, http://culturemachine.tees.ac.uk/Cmach/Backissues/j004/Articles/hallward.htm)

At this point, the reader has to wonder if the OPs policy of strict non-participation in the state really
stands up. The OP declares with some pride that we never vote, just as in the factories, we keep our
distance from trade unionism (LDP, 12.02.95: 1).26 The OP consistently maintains that its politics of
prescription requires a politics of non-vote. But why, now, this either/or? Once the state has been
acknowledged as a possible figure of the general interest, then surely it matters who governs that figure.
Regarding the central public issues of health and education, the OP maintains, like most mainstream
socialists, that the positive tasks on behalf of all are incumbent upon the state (LDP, 10.11.94: 1).27 That
participation in the state should not replace a prescriptive externality to the state is obvious enough, but
the stern either/or so often proclaimed in the pages of La Distance politique reads today like a displaced
trace of the days when the choice of state or revolution still figured as a genuine alternative.

WE SHOULD COMBINE BADIOUS GENERIC CONCEPTION OF BEING WITH OUR


DESCRIPTION OF THE SPECIFIC, WHICH DOESNT RESULT IN DEPICTION OF THE
SINGULAR

Hallward, Lecturer in the French Department @ Kings College, 2K3 (Peter Badiou: A Subject to Truth,
P. 274)

At each point, the alternative to Badious strictly generic conception of things is a


more properly specific understanding of individuals and situations as conditioned
by the relations that both enable and constrain their existence. In order to develop this
alternative, it is essential to distinguish scrupulously between the specific and what might be called the
specified (Badious objectified).5 Actors are specific to a situation even though their
actions are not specified by it, just as a historical account is specific to the facts it describes even
though its assessment is not specified by them. The specific is a purely relational subjective domain. The
specified, by contrast, is defined by positive, intrinsic characteristics or essences (physical, cultural,
personal, and so on). The specified is a matter of inherited instincts as much as of acquired habits. We
might say that the most general effort of philosophy or critique should be to move
from the specified to the specificwithout succumbing to the temptations of
the purely singular. Badiou certainly provides a most compelling critique of the specified. But he
hasat least thus far inadequate means of distinguishing specified from specific.

Page 377 of 1481


The result, in my view, is an ultimately unconvincing theoretical basis for his
celebration of an extreme particularity as such.

Page 378 of 1481


Page 379 of 1481
Perm Solvency (2/3)
BADIOUS OWN WRITING CONCEDES THE NECESSITY OF INCLUDING THE STATE
WITHIN OUR POLITICAL FOCUS. WHEN SOMETHING MUST BE DONE THAT ONLY
THE STATE CAN DO LIKE THE PLAN BADIOUS ETHICS FORCE US TO DEMAND
THE PLAN FROM THE STATE WHILE MAINTAINING A PROPER DISTANCE TOWARDS
IT THIS ALLOWS THE PLAN TO FUNCTION AS A TRULY ETHICAL COMMITMENT

Hallward, Lecturer in the French department @ Kings College, 2K2 (Peter BADIOU'S POLITICS: EQUALITY
AND JUSTICE, http://culturemachine.tees.ac.uk/Cmach/Backissues/j004/Articles/hallward.htm)

We know that Badious early and unequivocally hostile attitude to the state has considerably evolved . Just how far
it has evolved remains a little unclear. His conception of politics remains resolutely anti-consensual, anti-re-presentative, and thus anti-democratic (in the
ordinary sense of the word). A philosophy today is above all something that enables people to have done with the "democratic" submission to the world as it
is (Entretien avec Alain Badiou, 1999: 2). But he seems more willing, now, to engage with this submission on its own terms. La Distance politique again offers
the most precise points de repre. On the one hand, the OP remains suspicious of any political campaign for instance, electoral contests or petition
movements that operates as a prisoner of the parliamentary space (LDP, 19-20.04.96: 2). It remains an absolute necessity [of politics] not to have the state
their separation
as norm. The separation of politics and state is foundational of politics. On the other hand, however, it is now equally clear that
need not lead to the banishment of the state from the field of political thought (LDP, 6.05.93: 1).24 The OP now
conceives itself in a tense, non-dialectical vis--vis with the state, a stance that rejects an intimate
cooperation (in the interests of capital) as much as it refuses any antagonistic conception of their operation, any
conception that smacks of classism. There is to no more choice to be made between the state or revolution; the vis--vis
demands the presence of the two terms and not the annihilation of one of the two (LDP, 11.01.95: 3-4). Indeed, at the height of the December 95 strikes, the
OP recognised that the only contemporary movement of dstatisation with any real power was the corporate-driven movement of partial de-statification in
the interests of commercial flexibility and financial mobility. Unsurprisingly, we are against this withdrawal of the state to the profit of capital, through
general, systematic and brutal privatisation. The
state is what can sometimes take account of people and their situations
in other registers and by other modalities than those of profit. The state assures from this point of view
the public space and the general interest. And capital does not incarnate the general interest (LDP, 15.12.96:
11). Coming from the author of Thorie de la contradiction, these are remarkable words.

Page 380 of 1481


Page 381 of 1481
Perm Solvency (3/3)
BADIOUS ETHICAL PROJECT NECESSITATES ENDLESSLY RECONSTITUTING THE
SOCIAL REALM TO OPEN IT UP TO THE TRUTH-EVENT THE SPECIFIC DEMAND OF
THE PLAN CAN HAVE UNIVERSAL ETHICAL RESONANCE AND CAN FORM THE BASIS
OF A POLITICS OF TRUTH

Barker, Lecturer in Communications and a Doctoral Candidate in the Department of Philosophy @ Cardiff U, 2K2
(Jason, Alain Badiou: A Critical Introduction, P. 146-48)

How does Balibars theory of the State constitution stand alongside Badious, and can we find any key areas of mutual agreement between these two ex-
Althusserians? The most general area of difference involves Balibars aporetic approach to the question of the masses. Balibar refuses to see any principle
underlying the masses conduct, since the latter are synonymous with the power of the State. Badiou, on the other hand, regards the masses (ideally) as the
bearers of the category of justice, to which the State remains indifferent (AM, 114). Two divergent theories of the State, then, each of which is placed in the
service of a distinctive ethics. With Balibar we have an ethics or ethic in the sense of praxis of communication which encourages a dynamic and

With Badiou we have


expanding equilibrium of desires where every opinion has an equal chance of counting in the democratic sphere.

an ethics of truths which hunts down those exceptional political statements in


order to subtract from them their egalitarian core, thereby striking a blow for
justice against the passive democracy of the State. Overall we might say that the general area of agreement
lies in the fact that, in each case, democracy remains a rational possibility. In particular, for both Balibar and Badiou, it is love as an amorous feeling towards
or encounter with ones fellow man a recognition that the fraternal part that is held in common between human beings is somehow greater than the
whole of their differences which forges the social bond. However, on the precise nature of the ratio of this bond their respective paths diverge somewhat. In
Balibars case we are dealing with an objective illusion wherein one imagines that the love one feels for an object (an abstract egalitarian ideal, say) is shared
by others. Crucially, love in this sense is wholly ambivalent, wildly vacillating between itself and its inherent opposite, hate.18 On this evidence we might say
that a communist peace would be really indistinct from a fascist one. Therefore, the challenge for Balibar is to construct a prescriptive political framework
capable of operating without repression in a utilitarian public sphere where the free exchange of opinions is more likely than not to result in the self-limitation

of extreme views. In Badious case what we are dealing with, on the other hand and what we have been dealing with
more or less consistently throughout this book is a subjective reality. The social contract is forever

being conditioned, worked on practically from within by the political militants, in


readiness for the occurrence of the truth-event. This is the unforeseen moment
of an amorous encounter between two natural adversaries (a group of students mounting a
boycott of university fees, for instance) which retrieves the latent communist axiom of equality from

within the social process. Here we have a particular call for social justice (free education
for all!) which strikes a chord with the whole people (students and non-students alike). Crucially, love in

this sense is infinite, de-finite, in seizing back (at least a part of) the State power directly into the
hands of the people. Moreover, in this encounter between students and the university authorities there is an invariant connection (of
communist hope) which is shared by all, and where any difference of opinion is purely incidental. Momentarily, at least. For Badiou, the challenge

is to develop and deepen an ethical practice, not in any utilitarian or


communitarian sense since the latter would merely risk forcing a political
manifesto prematurely, perhaps giving rise to various brands of State-sponsored
populism9 but in the sense of a politics capable of combating repression; a politics
which, in its extreme singularity, holds itself open to seizure by Truth.

Page 382 of 1481


Page 383 of 1481
Human Rights Solve
BADIOU IS WRONG ABOUT HUMAN RIGHTS THEYRE A CRUCIAL RALLYING POINT
FOR ACTIVISTS AGAINST OPPRESSION

Dews, Prof of Philosophy @ U of Essex, 2K4 (Peter, Think Again: Alain Badiou and the Future of
Philosophy, P. 109)

Badiou is not mistaken, of course, in suggesting that the discourse of human rights has come to provide a
crucial ideological cover for economic and cultural imperialism, not to mention outright military intervention. No one doubts
the murderous hypocrisy with which the Western powers, led by the US, have invoked the language of human rights in recent years. But 'human
rights' have also been a rallying call for many activists around the globe. In the form of the Helsinki Accords, they were a
major focus for the East European opposition in the years leading up to 1989- They were equally important tactically for Latin
America's struggle against the dictatorships, and continue to provide a vital political point of leverage for
many indigenous populations, not to mention the Tibetans, the Burmese, the Palestinians. The United States, as is
well known, continues to refuse recognition to the recently established International Criminal Court, fearful, no doubt, that members of its own armed forces,
and perhaps of former administrations, could be amongst those arraigned before it.

Page 384 of 1481


Page 385 of 1481
Double Bind
BADIOU IS IN A DOUBLE-BIND: EITHER THERES NO WAY TO DISTINGUISH
BETWEEN TRUE AND FALSE EVENTS WHICH MEANS THE ALTERNATIVE CANT
SOLVE, OR SUBJECTS OF THE EVENT GO INTO IT WITH A PRECONCEIVED NOTION
OF THE EVENT, WHICH MAKES TRUE FIDELITY IMPOSSIBLE

Hallward, Professor of French at Kings College, London, 2K4 (Peter, Think Again: Alain Badiou and the
Future of Philosophy, P. 15-16)

Badiou insists on the rare and unpredictable


One implication of this last point is easily generalized.

character of every truth. On the other hand, we know that every truth, as it composes a generic or
egalitarian sampling of the situation, will proceed in such a way as to suspend the normal grip of the state of

its situation by eroding the distinctions used to classify and order parts of the
situation. Is this then a criterion that subjects must presume in advance or one that they come to discover in each case? If not the former, if
truth is entirely a matter of post-evental implication or consequence, then
there can be no clear way of distinguishing, before it is too late, a genuine
event (which relates only to the void of the situation, i.e. to the way inconsistency might appear within a situation) from a false
event (one that, like September 11th or the triumph of National Socialism, reinforces the basic distinctions governing the situation). But if
there is always an initial hunch which guides the composition of a generic set,
a sort of preliminary or prophetic commitment to the generic just as there is, incidentally,
in Cohens own account of generic sets, insofar as this account seeks to demonstrate a possibility implicit in the ordinary extensional definition of set25

then it seems difficult to sustain a fully post-evental conception of truth. In short: is


the initial decision to affirm an event unequivocally free, a matter of consequence alone? Or is it tacitly guided by the criteria of the generic at every step,
and thereby susceptible to a kind of anticipation?

Page 386 of 1481


Page 387 of 1481
Alternative Fractures Coalitions
BADIOUS ALTERNATIVE IS A DISASTROUS FORM OF POLITICS BECAUSE THE
SUBJECTS OF A TRUTH CAN NEVER TRANSLATE THAT TRUTH TO THOSE HOSTILE TO
THEIR AGENDA, AND THUS CAN NEVER MAKE POLITICAL COALITIONS

Hallward, Professor of French at Kings College, London, 2K4 (Peter, Think Again: Alain Badiou and the
Future of Philosophy, P. 17)

6. In a related sense, is it enough to explain the process of subjectivation, the


transformation of an ordinary individual into the militant subject of a uni-
versalizable cause, or truth, mainly through analogies with the process of
conversion? It is certainly essential to maintain (after Saint Paul) that anyone can become the militant of a truth, that truth is not primarily a
matter of background or disposition. If it exists at all, truth must be equally indifferent to both

nature and nurture, and it is surely one of the great virtues of Badious account of the subject that it, like Zizeks or Lacans, remains
irreducible to all the forces (historical, social, cultural, genetic .. .) that shape the individual or ego in the ordinary sense. On the other hand, the lack

of any substantial explanation of subjective empowerment, of the process that


enables or inspires an individual to become a subject, again serves only to
make the account of subjectivation unhelpfully abrupt and abstract. Isnt there
a danger that by disregarding issues of motivation and resolve at play in any
subjective decision, the militants of a truth will preach only to the converted?
Doesnt the real problem of any political organization begin where Badious
analyses tend to leave off, i.e. with the task of finding ways whereby a truth will
begin to ring true for those initially indifferent or hostile to its implications?

Page 388 of 1481


Page 389 of 1481
Divorcing Politics from State Bad
BADIOUS DESIRE TO SEPARATE POLITICS FROM THE STATE MAKES POLITICS ITSELF
IMPOSSIBLE

Bensaid, Prof @ the U of Paris VIII and leading member of the Ligue Commiuniste Revolutionnaire, 2K4 (Daniel
Think Again: Alain Badiou and the Future of Philosophy, P. 99-100)

Yet in
Badiou, the intermittence of event and subject renders the very idea of politics problematic . According to
him, politics defines itself via fidelity to the event whereby the victims of oppression declare themselves. His
determination to prise politics
free from the state in order to subjecrivize it, to deliver it from history in order to hand it over to the event, is part of a tentative
search for an autonomous politics of the oppressed. The alternative effort, to subordinate politics to some putative meaning of
history, which has ominous echoes in recent history, is he suggests to incorporate it within the process of general technicization and to reduce it to the
management of state affairs. One must have the courage to declare that, from the point of view of politics, history as meaning or direction does not exist: all
that exists is the periodic occurrence of the a priori conditions of chance. However,
this divorce between event and history
(between the event and its historically determined conditions) tends to render politics if not unthinkable
then at least impracticable (PP 18).

BADIOUS ALTERNATIVE FAILS BECAUSE HES BLIND TO POLITICAL POWER


STRUCTURES HIS DEMAND TO DIVORCE POLITICS FROM THE STATE MEANS IT
CANT DEAL WITH TODAYS MOST PRESSING PROBLEMS

Hallward, Professor of French at Kings College, London, 2K4 (Peter, Think Again: Alain Badiou and the
Future of Philosophy, P. 18-19)

to what extent can we abstract an exclusively political truth from matters relating to society,
Most obviously,
history and the state? Take those most familiar topics of cultural politics: gender, sexuality and race. No doubt the greater part of the still
incomplete transformation here is due to militant subjective mobilizations that include the anti-colonial wars of liberation, the civil rights movement, the
feminist movements, Stonewall, and so on. But has cumulative, institutional change played no role in the slow movement towards racial or sexual indistinction,
precisely? More importantly: since under the current state of things political authority is firmly vested in the hands
of those with economic power, can a political prescription have any enduring effect if it manages only to
distance or suspend the operation of such power? If a contemporary political sequence is to last (if at least it is
to avoid the usual consequences of capital flight and economic sabotage) must it not also directly entail a genuine transformation
of the economy itself, i.e. enable popular participation in economic decisions, community or workers control over resources and production, and so
on? In todays circumstances, if a political prescription is to have any widespread consequence, isnt it
essential that it find some way of bridging the gap between the political and the economic? Even Badious own
privileged example indicates the uncertain purity of politics. The declaration of 18 March 1871 (which he quotes as the inaugural affirmation of a proletarian
political capacity) commits the Communards to taking in hand the running of public affairs,3 and throughout its short existence the Commune busies itself as
much with matters of education, employment and administration as with issues of equality and power. Is
a sharp distinction between
politics and the state helpful in such circumstances? Do forms of discipline subtracted from the state,
from the party, apply in fact to anything other than the beginning of relatively limited political sequences?
Does the abstract ethical imperative, continue!, coupled with a classical appeal to moderation and
restraint,38 suffice to safeguard the long-term persistence of political sequences from the altogether
necessary return of state-like functions (military, bureaucratic, institutional . . .)? To what extent, in short, does Badious position, which he
presents in anticipation of an as yet obscure step beyond the more state-centred conceptions of Lenin and Mao, rather return him instead to the familiar
objections levelled at earlier theories of anarchism?

Page 390 of 1481


Page 391 of 1481
Page 392 of 1481
**Baudrillard**

Page 393 of 1481


Baudrillard Destroys Social
Change (1/2)
BAUDRILLARDS ALTERNATIVE ALLOWS CONSERVATIVE IDEOLOGICAL DISTORTION

Christopher Norris, Distinguished Research Professor in Philosophy at the University of Cardiff, Wales, Whats
Wrong with Postmodernism, 1990, p. 190-191. *Gender modified

Baudrillards alternative is stated clearly enough: a hyperreal henceforth sheltered from the imaginary,
and from any distinction between the real and the imaginary, leaving room only for the orbital recurrence
of models and the simulated generation of difference (p. 167). It is a vision which should bring great
comfort to government advisers, PR experts, campaign managers, opinion-pollsters, media watch-dogs,
Pentagon [spokespeople] spokesmen and others with an interest in maintaining this state of affairs.
Baudrillards imagery of orbital recurrence and the simulated generation of difference should commend
itself to advocates of a Star Wars program whose only conceivable purpose is to escalate EastWest
tensions and divert more funds to the military-industrial complex. There is no denying the extent to which
this and similar strategies of disinformation have set the agenda for public debate across a range of
crucial policy issues. But the fact remains (and this phrase carries more than just a suasive or rhetorical
force) that there is a difference between what we are given to believe and what emerges from the
process of subjecting such beliefs to an informed critique of their content and modes of propagation. This
process may amount to a straightforward demand that politicians tell the truth and be held to account for
their failing to do so. Of course there are cases like the IrangateContra affair or Thatchers role in
events leading up to the Falklands war where a correspondence-theory might seem to break down
since the facts are buried away in Cabinet papers, the evidence concealed by some piece of high-level
chicanery (Official Secrets, security interests, reasons of state, etc.), or the documents conveniently
shredded in time to forestall investigation of their content. But there is no reason to think as with
Baudrillards decidedly Orwellian prognosis that this puts the truth forever beyond reach, thus
heralding an age of out-and-out hyperreality. For one can still apply other criteria of truth and falsehood,
among them a fairly basic coherence-theory that would point out the various lapses, inconsistencies, non-
sequiturs, downright contradictions and so forth which suffice to undermine the official version of events.
(Margaret Thatchers various statements on the Malvinas conflict especially the sinking of the General
Beigrano would provide a good example here.)29 It may be argued that the truth-conditions will vary
from one specific context to another; that such episodes involve very different criteria according to the
kinds of evidence available; and therefore that it is no use expecting any form of generalised theory to
establish the facts of this or that case. But this ignores the extent to which theories (and truth-claims)
inform our every act of rational appraisal, from commonsense decisions of a day-to-day, practical kind to
the most advanced levels of speculative thought. And it also ignores the main lesson to be learnt from
Baudrillards texts: that any politics which goes along with the current postmodernist drift will end up by
effectively endorsing and promoting the work of ideological mystification.

Page 394 of 1481


Page 395 of 1481
Baudrillard Destroys Social
Change (2/2)
RELEGATING HUMAN SUFFERING TO THE REALM OF THE SIGN AND SIMULATION IS
JUST DISGUISED NIHILISM, WHICH CRUSHES THE POSSIBILITY FOR EFFECTIVE
POLITICS

Kellner, Philosophy Chair @ UCLA, 89 (Douglas, Jean Baudrillard, P. 107-8)

Yet does the sort of symbolic exchange which Baudrillard advocates really provide a solution to the question of death? Baudrillards notion of symbolic
exchange between life and death and his ultimate embrace of nihilism (see 4.4) is probably his most un-Nietzschean moment, the instant in which his thought
radically devalues life and focuses with a fascinated gaze on that which is most terrible death. In a
popular French reading of Nietzsche, his transvaluation of values demanded negation of all repressive
and life- negating values in favor of affirmation of life, joy and happiness. This philosophy of value
valorized life over death and derived its values from phenomena which enhanced, refined and nurtured
human life. In Baudrillard, by contrast, life does not exist as an autonomous source of value, and the body
exists only as the caarnality of signs, as a mode of display of signification. His sign fetishism erases all
materialjty from the body and social life, and makes possible a fascinated aestheticized fetishism of signs
as the primary ontological reality. This way of seeing erases suffering, disease, pain and the horror of
death from the body and social life and replaces it with the play of signs Baudrillards alternative.
Politics too is reduced to a play of signs, and the ways in which different politics alleviate or intensify
human suffering disappears from the Baudrillardian universe. Consequently Baudrillards theory spirals into a
fascination with signs which leads him to embrace certain privileged forms of sign culture and to reject
others (that is, the theoretical signs of modernity such as meaning, truth, the social, power and so on) and to pay less and less attention
to materiality (that is, to needs, desire, suffering and so on) a trajectory will ultimately lead him to embrace nihilism (see
4.4). Thus Baudrillards interpretation of the body, his refusal of theories of sexuality which link it with desire and pleasure, and his valorization of death as a
mode of symbolic exchange which valorizes sacrifice, suicide and other symbolic modes of death are all part and parcel of a fetishizing of signs, of a
valorization of sign culture over all other modes of social life. Such fetishizing of sign culture finds its natural (and more harmless) home in the fascination with
the realm of sign culture which we call art. I shall argue that Baudrillards trajectory exhibits an ever more intense aestheticizing of social theory and
philosophy, in which the values of the representation of social reality, political struggle and change and so on are displaced in favor of a (typically French) sign
fetishism. On this view, Baudrillards trajectory is best interpreted as an increasingly aggressive and extreme fetishizing of signs, which began in his early works
in the late 1 960s and which he was only gradually to exhibit in its full and perverse splendor as aristocratic aestheticism from the mid-1970s to the present. Let
us now trace the evolution of his fascination with art, a form of sign culture which Baudrillard increasingly privileges and one which provides an important
feature attraction of the postmodern carnival.

Page 396 of 1481


Page 397 of 1481
Alternative Masks Violence
FOCUS ON THE HYPER-REAL PRIVILEGES THE SIGNIFIER OVER THE SIGNIFIED,
NUMBING US TO ACTUAL VIOLENCE

Krishna 93
[Snakaran, Dept. Poli Sci @ Hawaii, Alternatives 18, 399]

By emphasizing the technology and speed in the Gulf War, endlessly analyzing the representation of the
war itself, without a simultaneous exposition of the ground realities, postmodernist analyses wind up,
unwittingly, echoing the Pentagon and the White House in their claims that this was a clean war with
smart bombs that take out only defense installations with minimal collateral damage. One needs to
reflesh the Gulf War dead through our postmortems instead of merely echoing, with virilio and others,
the disappearance of territory or the modern warrior with the new technologies; or the intertext
connecting the war and television; or the displacement of the spectacle.

Second, the emphasis on speed with which the annihilation proceeded once the war began tends to
obfuscate the long build-up to the conflict and US complicity in Iraqi foreign and defense policy in prior
times. Third, as the details provided above show, if there was anything to highlight about the war, it was
not so much its manner of representation as the incredible levels of annihilation that have been
perfected. To summarize: I am not suggesting that postmodern analysts of the war are in agreement with
the Pentagons claims regarding a clean war; I am suggesting that their preoccupation with
representation, sign systems, and with the signifier over the signified, leaves one with little sense of the
annihilation visited upon the people and land of Iraq. And, as the Vietnam War proved and Schwartzkopf
well realized, without that physicalist sense of violence war can be more effectively sold to a jingoistic
public.

Page 398 of 1481


Page 399 of 1481
Our Representations Solve
TURNMEDIA IMAGES REVEAL THEIR OWN ILLUSIONS

Jean Baudrillard, professor of philosophy of culture and media at Univ. or Paris, 1994, Illusion of the End,
pg. 60-61

And yet there will, nonetheless, have been a kind of verdict in this Romanian affair, and the artificial
heaps of corpses will have been of some use, all the same. One might ask whether the Romanians, by the
very excessiveness of this staged event and the simulacrum of their revolution, have not served as
demystifiers of news and its guiding principle. For, if the media image has put an end to the credibility of
the event, the event will, in its turn, have put an end to the credibility of the image. Never again shall we
be able to look at a television picture in good faith, and this is the finest collective demystification we
have ever known. The finest revenge over this new arrogant power, this power to blackmail by events.
Who can say what responsibility attaches to the televisual production of a false massacre (Timisoara), as
compared with the perpetrating of a true massacre? This is another kind of crime against humanity, a
hijacking of fantasies, affects and the credulity of hundreds of millions of people by means of television
a crime of blackmail and simulation. What penalty is laid down for such a hijacking? There is no way to
rectify this situation and we must have no illusions: there is no perverse effect, nor even anything
scandalous in the Timisoara syndrome. It is simply the (immoral) truth of news, the secret purpose
[destination] of which is to deceive us about the real, but also to undeceive us about the real. There is no
worse mistake than taking the real for the real and, in that sense, the very excess of media illusion plays a
vital disillusioning role. In this way, news could be said to undo its own spell by its effects and the violence
of information to be avenged by the repudiation and indifference it engenders. Just as we should be
unreservedly thankful for the existence of politicians, who take on themselves the responsibility for that
wearisome function, so we should be grateful to the media for existing and taking on themselves the
triumphant illusionism of the world of communications, the whole ambiguity of mass culture, the
confusion of ideologies, the stereotypes, the spectacle, the banality soaking up all these things in their
operation. While, at the same time, constituting a permanent test of intelligence, for where better than
on television can one learn to question every picture, every word, every commentary? Television
inculcates indifference distance, scepticism and unconditional apathy. Through the worlds becoming-
image, it anaesthetizes the imagination, provokes a sickened abreaction, together with a surge of
adrenalin which induces total disillusionment. Television and the media would render reality [le reel]
dissuasive, were it not already so. And this represents an absolute advance in the consciousness or the
cynical unconscious of our age.

Page 400 of 1481


Page 401 of 1481
Baudrillard is Wrong (1/2)
BAUDRILLARDS CRITIQUE IS EMPIRICALLY DENIED BY THE GULF WAR

Christopher Norris, Distinguished Research Professor in Philosophy at the University of Cardiff, Wales, Uncritical
Theory: Postmodernism, Intellectuals, and the Gulf War, 1992, p. 11.

How far wrong can a thinker go and still lay claim to serious attention? One useful test-case is Jean
Baudrillard, a cult figure on the current postmodernist scene, and purveyor of some of the silliest ideas
yet to gain a hearing among disciples of French intellectual fashion. Just a couple of days before war broke
out in the Gulf, one could find Baudrillard regaling readers of The Guardian newspaper with an article
which declared that this war would never happen, existing as it did only as a figment of mass-media
simulation, war-games rhetoric or imaginary scenarios which exceeded all the limits of real-world, factual
possibility.1 Deterrence had worked for the past forty years in the sense that war had become strictly
unthinkable except as a rhetorical phenomenon, an exchange of ever-escalating threats and counter-
threats whose exorbitant character was enough to guarantee that no such event would ever take place.
What remained was a kind of endless charade, a phoney war in which the stakes had to do with the
management of so-called public opinion, itself nothing more than a reflex response to the images, the
rhetoric and PR machinery which create the illusion of consensus support by supplying all the right
answers and attitudes in advance. There would be no war, Baudrillard solemnly opined, because talk of
war had now become a substitute for the event, the occurrence or moment of outbreak which the term
war had once signified. Quite simply, we had lost all sense of the difference or the point of transition
between a war of words, a mass-media simulation conducted (supposedly) by way of preparing us for
the real thing, and the thing itself which would likewise take place only in the minds and imaginations
of a captive TV audience, bombarded with the same sorts of video-game imagery that had filled their
screens during the build-up campaign.

Page 402 of 1481


Page 403 of 1481
Baudrillard is Wrong (2/2)
BAUDRILLARDS CRITIQUE IS NAVE AND CONTRADICTORY, DOES NOT
CORRESPOND WITH REALITY, AND IS NORMATIVELY USELESS

James Marsh, Professor of Philosophy, Fordham University, 1995, Critique, Action, and Liberation, pp. 292-293

Such an account, however, is as one-sided or perhaps even more one-sided than that of naive modernism.
We note a residual idealism that does not take into account socioeconomic realities already pointed out
such as the corporate nature of media, their role in achieving and legitimating profit, and their function of
manufacturing consent. In such a postmodernist account is a reduction of everything to image or symbol
that misses the relationship of these to realities such as corporations seeking profit, impoverished
workers in these corporations, or peasants in Third-World countries trying to conduct elections.
Postmodernism does not adequately distinguish here between a reduction of reality to image and a
mediation of reality by image. A media idealism exists rooted in the influence of structuralism and
poststructuralism and doing insufficient justice to concrete human experience, judgment, and free
interaction in the world.4 It is also paradoxical or contradictory to say it really is true that nothing is really
true, that everything is illusory or imaginary. Postmodemism makes judgments that implicitly deny the
reduction of reality to image. For example, Poster and Baudrillard do want to say that we really are in a
new age that is informational and postindustrial. Again, to say that everything is imploded into media
images is akin logically to the Cartesian claim that everything is or might be a dream. What happens is
that dream or image is absolutized or generalized to the point that its original meaning lying in its contrast
to natural, human, and social reality is lost. We can discuss Disneyland as reprehensible because we know
the difference between Disneyland and the larger, enveloping reality of Southern California and the
United States.5 We can note also that postmodernism misses the reality of the accumulation-legitimation
tension in late capitalism in general and in communicative media in particular. This tension takes different
forms in different times. In the United States in the 1960s and 1970s, for example, social, economic, and
political reality occasionally manifested itself in the media in such a way that the electorate responded
critically to corporate and political policies. Coverage of the Vietnam war, for example, did help turn
people against the war. In the 1980s, by contrast, the emphasis shifted more toward accumulation in the
decade dominated by the great communicator. Even here, however, the majority remained opposed to
Reagans policies while voting for Reagan. Human and social reality, while being influenced by and
represented by the media, transcended them and remained resistant to them.6 To the extent that
postmodernists are critical of the role media play, we can ask the question about the normative adequacy
of such a critique. Why, in the absence of normative conceptions of rationality and freedom, should
media dominance be taken as bad rather than good? Also, the most relevant contrasting, normatively
structured alternative to the media is that of the public sphere, in which the imperatives of free,
democratic, nonmanipulable communicative action are institutionalized. Such a public sphere has been
present in western democracies since the nineteenth century but has suffered erosion in the twentieth
century as capitalism has more and more taken over the media and commercialized them. Even now the
public sphere remains normatively binding and really operative through institutionalizing the ideals of
free, full, public expression and discussion; ideal, legal requirements taking such forms as public service
programs, public broadcasting, and provision for alternative media; and social movements acting and
discoursing in and outside of universities in print, in demonstrations and forms of resistance, and on
media such as movies, television, and radio.7

Page 404 of 1481


Page 405 of 1481
A2 Disaster Porn (1/3)
TURN: VIOLENCE IS INESCAPABLE. OUR VIOLENCE ENABLES UNDERSTANDING
MORE THAN IT INHIBITS. REMEMBERING AND REPRESENTING VIOLENCE IS
ESSENTIAL TO AVERT THE DESTRUCTION OF THE OTHER. REJECT THE CRITIQUES
SILENCE.

Michael Eskin, Research Fellow and Lecturer, European Literature, Cambridge University, Dialectical
Anthropology, 24: 407-450, 1999, p. 391-6

Derrida allows nothing prior to language; since, in Derrida's s philosophy, everything is inscribed in language, he places
speech and language prior to ethics, prior to any possible ethical injunction. Derrida's formulations owe a tremendous
debt to several major epistemological shifts. of the early twentieth century: Sapir's and Whorf's notion that language
conditions thought, for example, or Lacan's claims that both conscious and unconscious thought processes (and thus
the subject) are structured by language. Because for Derrida ethics is inscribed, along with everything else, in language,
and because for Derrida language is inherently violent in that it is always a reduction, a totalization, he
reaches the conclusion that even a Levinasian ethics cannot ever avoid violence: "One never escapes the
economy of war." The origin of this violence inherent in discourse is the act of inscribing the other in the
definitions and terms of the same: Predication is the first violence. Since the verb to be and the predicative act are
implied in every other verb, and in every common noun, nonviolent language, in the last analysis, would be a language
of pure invocation . . .purified of all rhetoric [in Levinas' terms] . . . . Is a language free from all rhetoric possible?
Derrida answers his own question in the negative, affirming that "there is no phrase which is indeterminate, that is,
which does not pass through the violence of the concept. Violence appears with articulation." Foucault has expressed
this same sentiment, maintaining that "We must conceive discourse as a violence we do to things, or, at all events, as a
practice we impose upon them." Naming and predication-two acts essential to language-confine what is
being described, and fix it in one's own terms. As we shall see from an examination of Hiroshima non amour,
memory works the same way, attempting to enclose the past within determinate parameters, employing
the same brand of totalization to whose presence in language Derrida has gestured. Concern over the necessary
violence of memory as representation to the consciousness, as willed inscription in one's own terms of what is other
because past, is perhaps the most obvious point at which Derrida, Levinas, Duras, and Resnais converge, for the
impossibility of remembering an historical event as it was-of actually arriving at a clear understanding of a past event
by imaging it through memory, by re-presenting it to our memory-is a chronic preoccupation of Hiroshima mon amour.
Resnais confronted this dilemma as well in the process of constructing Nuit et brouillard. Claiming historical
authority over Auschwitz, or giving the illusion that it is comprehensible, would only, in Resnais' opinion,
"humaniz[e] the incomprehensible terror," thereby "diminishing it," perhaps even romanticizing it; so,
unable to describe the violence, and unwilling to inscribe it, Resnais opted instead to document our
memory of it. Resnais carries no illusions that the past can be duplicated to any significant degree, rendered for us
now as it was then. Given the accepted generic constraints of a film, he says, "it is absolutely absurd to think that
in that space of time one can properly present the historical reality of such a complex event. [Historical
facts] were the bases for our `fiction,' points of departure rather than ends in themselves." This explains what Leo
Bersani has described as Resnais' clear favoring of the word "imagination" over the word "memory" when referring to
his own films." However, in the case of Hiroshima mon amour, instead of filling in with imagination the details between
the historical "facts," the film throws its hands up at any effort to "remember" or "see" the tragedy at Hiroshima. Thus,
Hiroshima mon amour, in the words of one critic, turns out "to be a film about the impossibility of making a
documentary about Hiroshima"1' or, in Armes' more broadly epistemologically oriented phrase, "a documentary on the
impossibility of comprehending." Duras reminds us of this in her synopsis of the screenplay: "Impossible de parler de
HIROSHIMA. Tout ce qu'on peut faire c'est de parler de l'impossibilite de parler de HIROSHIMA (Impossible to speak
of HIROSHIMA. All one can do is speak of the impossibility of speaking of HIROSHIMA)." She then drives the
point home in Hiroshima mon amour's unforgettable opening sequence, as Okada incessantly reminds Riva that
she can never know Hiroshima's tragedy. Riva knows, for example, that there were two hundred thousand
dead and eighty thousand wounded, in nine seconds; she can rattle off the names of every flower that
bloomed at ground zero two weeks after the bombing; she has been to the museum four times, seen the
pictures, watched the films. As if to accentuate the veracity of' Riva's learned data, Duras alerts the reader in a

Page 406 of 1481


footnote to the origin of the details, and there is hardly a more famous or traditionally reputable source on the
immediate aftermath of the bombing than John Mersey's Hiroshima. And yet, as one critic has commented, "les images
collees aux murs . . . sont incapables de faire revivre completement la realite du fait (images pasted to walls . . . are
incapabale of completely restoring the reality of the fact)." Despite Riva's wealth of statistical (read:
historically trustworthy) data, Okada is able to refute her with confidence, "Tu n'as rien vu a Hiroshima (You
saw nothing at Hiroshima)," and the almost incantatory

continued

Page 407 of 1481


Page 408 of 1481
A2 Disaster Porn (2/3)
continued

repetition of this phrase strengthens its punch. Duras increases the effect by reminding us that the day of the bombing of Hiroshima, while a tragedy for Okada, coincides with
Riva's liberation from her horrifying wartime experience in Nevers, France. This fact forces the question: How can Riva ever understand as a tragedy an event that
corresponded with her own emotional rebirth and reclaiming of some measure of normalcy?

The effect is even stronger on what Duras must have assumed would be a predominantly Western audience, when Okada points out that the entire
world was celebrating while Hiroshima smouldered in ashes. This fact forces another, similar question, one that I myself must confront
on reading or watching Hiroshima mon amour: How could the Westerners in the audience ever expect to grasp the tragedy

that they originally celebrated as the end of the war? These reminders have their own Verfremdungseffekt further alienating the
audience/reader from the history of Hiroshima, dispelling any lingering notion that historical tragedy can ever be fully comprehended. Riva's optimism is almost infectious,

though, and she indeed believes that she can master the history behind the leveling of Hiroshima. She claims to know everything, and she is

once again swiftly negated by the Japanese. She contents herself by concluding that, even if she does not know yet, ca s'apprend (one learns).""
She is not gifted with memory, though, as Okada reminds her and thus all she can claim to know about Hiroshima is what she has "invente." This particular verbal exchange is
highlighted by the fact that it is for the first time in the text Riva's turn to use the word "rien," until this point a word uttered frequently and only by Okada: ELLS: Je n'ai rien
invente. (SHE: I invented nothing.) LUI: Tu as tout invente. (HE: You invented everything.) Proof of her inability to approach comprehension of Hiroshima arrives in the form of a
laugh, when Riva asks her lover if he was at Hiroshima the day of the bombing and he laughs as one would laugh at a child. She shows herself further distanced from the
historical event by the manner in which she sounds out the name of the city, "Hi-ro-shi-ma," as if it were-or rather because it is-radically foreign to her. (Later, in the same
manner, Okada sounds out Riva's youth, the story of which will always be unknown and incomprehensible to him: "Jeune-a-Ne-vers [ Young-in-Nevers].") Her memory of
Hiroshima, created by herself and inscribed in terms that she can understand from photographs taken by other people, is mere "illusion," truth several times removed. She

remembers, though, and almost obsessively, because she knows that it is worse to forget . Historical memory must be reductive, sometimes violently
so, according to a Derridean understanding of it, because it is always a form of representation and thus of predication. A less diplomatic statement made by Okada goes so far

as to suggest that one's memory only ever serves one's own purposes: "Est-ce que to avais remarque," he asks, "que c'est toujours dans le
meme sens que l'on remarque les chows? (Did you ever notice that one always notices things in the same way?)." We notice what suits us, in the direction and sense which we
However, just as language-the system of representation par excellence-carries in
prefer, and we notice it in the manner in which we can best use it.

its every use the violence inherent in its reductiveness, we use it anyway, as it enables far more than
inhibits. In Levinas's formulation, not only is discourse our primary means of relating to and maintaining the other,
but the absence of it, silence, "is the inverse of language . . . a laughter that seeks to destroy language. "
Derrida accords with Levinas: "denying discourse" is "the worst violence," "the violence of the night which precedes

or represses discourse." Despite the violence that Riva's impulse toward memory commits against any
ideal or "objective" history, absolute forgetting is far more dangerous; by any account, remembering and
representing past violence must be seen as a necessary evil, as a sort of metaphysically violent means
of averting future real, physical violence. Still, the partial forgetting of the unforgettable tragedy is inevitable, as John Ward points out in his
treatment of Resnais' films: "With the passage of time we become so insensitive to other people's suffering that we

can lie in the disused ovens of Auschwitz and have our photographs taken as souvenirs." Duras' text also renders
disturbing images of forgetting, of loubli. Riva confesses to her own struggle against ignorance: "mei aussi, j'ai essaye de lutter de toutes mes forces contre l'oubli . . . . Comme
toi, j'ai oublie (me too, I've tried to struggle with all my strength against forgetting . . . . Like you, I've forgotten). "During the third part of Duras' script, at the staged
demonstration against nuclear armaments, Okada seems far too preoccupied with taking Riva back to his family's house to care about the demonstration, even if it is only a
performance for a film. Immediately after explaining the appearance of the charred skin of Hiroshima's surviving children, he informs her, "Tu vas venir avec moi encore une fois
(You will come with me once again)." Remembering the bombing is quite obviously not a first priority for him. There are other grim reminders of the forgetting in the
reconstruction of Hiroshima and the importation of American culture. At one point, Riva and Okada enter a nightclub called "Casablanca" -a strange immortalization of American
pop culture in a city leveled by an American bomb less than two decades earlier. Moreover, the Japanese man who tries to converse with Riva in the Casablanca gladly (and
proudly, it seems) speaks the language of the conquerors, the bomb-droppers. The attitude on display in this scene is reminiscent of one in John Hersey's account of the
months following the bombing, in Hiroshima: [Dr. Fujiil bought [the vacant clinic] at once, moved there, and hung out a sign inscribed in English, in honor of the conquerors: M.
MUJII, M.D. MEDICAL & VENEREAL Quite recovered from his wounds, he soon built up a strong practice, and he was delighted, in the evenings, to receive members of the

While there is certainly something to be said for not bearing a


occupying forces, on whom he lavished whiskey and practiced English.

grudge, the speed of the forgetting and forgiving seems unbelievable. Memory represents historical tragedy insufficiently, in
violently subjective reductions; we are never able to experience being there and can never know the event, can never have witnessed it firsthand. Thus, we forget. Duras' script

clearly stresses both the necessity and difficulty of remembering, but demonstrates, perhaps pessimistically, that we will veer slightly but inexorably toward l'oubli. And once
we forget, violence will erupt again.

Page 409 of 1481


Page 410 of 1481
A2 Disaster Porn (3/3)
THE CRITIQUE IS REDUCTIVE. THEY FORECLOSE THE ESSENTIAL ABILITY TO
MOBILIZE VIOLENCE AGAINST VIOLENCE.

Michael Eskin, Research Fellow and Lecturer, European Literature, Cambridge University, Dialectical Anthropology,

24: 407-450, 1999, p. 403-4

I have tried to demonstrate through this reading of Hiroshima mon amour that Resnais' and Duras' text
falls prey to the violence of historical memory and to the worse violence of absolute oblivion. Strictly
following a theoretical apparatus reconstructed from the thought of Levinas and Derrida, Hiroshima mon
amour seems to participate, through the apparently deliberate reduction to race and place and event of
two already allegorical and emblematic characters, in the very violence which Resnais and Duras set out
initially to document, the most reductive of predications. The script trades in an economy of violence,
dealing out the abstractions and totalizations that are the seed of every Holocaust, that mark every
uninhabitable corner of Hiroshima and Nagasaki. This conclusion seems to me, though, far too conclusive,
far too reductively critical and discomforting, far too dependant on a great deal of interpretive faith, not
unmerited but certainly not absolute, in the debate between and formulations of Levinas and Derrida
What I am trying gingerly to say is that our reading should remain sensitive, attentive and open enough to
discover those points at which the theoretical scaffolding may fail us, points at which a
Levinasian/Derridean reading seems to stall; I believe a conclusive dismissal of Hiroshima mon amour as a
text governed and permeated by violence is probably one such moment. I would propose instead a
different, and hopefully more useful, reading of my reading of this well-intentioned script and film. For,
while Hiroshima mon amour is certainly guilty of the very violence it claims as its object, it is likely from
this portrayal and mobilizing of violence that the film sees its greatest anti-violent gesture; all that is
required is a return to Duras' stated desire to avoid the banal describing of "l'horreur par l'horreur."
Instead of horrifying us with horror, as she refused to do, Duras' screenplay has shown us the humble
beginnings of horror: the total forgetting of past horrors, and the blatant inscribing of infinite Others
within the finitudes of the language of the Same. And in this, Duras and Resnais may have succeeded,
ultimately, in their declared mission to bring the horrifying tragedy of Hiroshima back to life, to see it
reborn, out of the ashes.

Page 411 of 1481


Page 412 of 1481
**Butler**

Page 413 of 1481


Butler Answers: 2AC (1/2)

Page 414 of 1481


Page 415 of 1481
Butler Answers: 2AC (2/2)

Page 416 of 1481


Page 417 of 1481
A2 Legal Categories Bad
BAILING ON LEGAL CHANGE FOR PARODIC PERFORMANCE FAILS TO BREAK DOWN
GENDER CATEGORIES AND COLLAPSES INTO QUIETISM

Nussbaum 99 (Martha, Feb. 22, Professor of Parody, New Republic, Lexis)

What precisely does Butler offer when she counsels subversion? She tells us to engage in parodic performances, but she
warns us that the dream of escaping altogether from the oppressive structures is just a dream: it is within
the oppressive structures that we must find little spaces for resistance, and this resistance cannot hope to change the overall situation. And here lies a
dangerous quietism. If Butler means only to warn us against the dangers of fantasizing an idyllic world in which sex raises no serious problems,
she is wise to do so. Yet frequently she goes much further. She suggests that the institutional structures that ensure the marginalization of lesbians and gay
men in our society, and the continued inequality of women, will never be changed in a deep way; and so our best hope is to thumb our noses at them, and
to find pockets of personal freedom within them. "Called by an injurious name, I come into social being, and because I have a certain inevitable attachment
to my existence, because a certain narcissism takes hold of any term that confers existence, I am led to embrace the terms that injure me because they
constitute me socially." In other words: I cannot escape the humiliating structures without ceasing to be, so the best I can do is mock, and use the language
of subordination stingingly. In
Butler, resistance is always imagined as personal, more or less private, involving
no unironic, organized public action for legal or institutional change. Isn't this like saying to a slave that
the institution of slavery will never change, but you can find ways of mocking it and subverting it,
finding your personal freedom within those acts of carefully limited defiance? Yet it is a fact that the
institution of slavery can be changed, and was changed-- but not by people who took a Butler-like view
of the possibilities. It was changed because people did not rest content with parodic performance: they
demanded, and to some extent they got, social upheaval. It is also a fact that the institutional structures that shape women's lives have
changed. The law of rape, still defective, has at least improved; the law of sexual harassment exists, where it did not exist before; marriage is no longer
regarded as giving men monarchical control over women's bodies. These things were changed by feminists who would not take parodic performance as
Butler not only eschews such a hope, she
their answer, who thought that power, where bad, should, and would, yield before justice.
takes pleasure in its impossibility. She finds it exciting to contemplate the alleged immovability of power, and to envisage the ritual
subversions of the slave who is convinced that she must remain such. She tells us--this is the central thesis of The Psychic Life of Power-- that we all
eroticize the power structures that oppress us, and can thus find sexual pleasure only within their confines. It seems to be for that reason that she prefers
the sexy acts of parodic subversion to any lasting material or institutional change. Real change would so uproot our psyches that it would make sexual
parodic
satisfaction impossible. Our libidos are the creation of the bad enslaving forces, and thus necessarily sadomasochistic in structure. Well,
performance is not so bad when you are a powerful tenured academic in a liberal university. But here is
where Butler's focus on the symbolic, her proud neglect of the material side of life, becomes a fatal blindness. For
women who are hungry, illiterate, disenfranchised, beaten, raped, it is not sexy or liberating to reenact,
however parodically, the conditions of hunger, illiteracy, disenfranchisement, beating, and rape. Such
women prefer food, schools, votes, and the integrity of their bodies. I see no reason to believe that they long
sadomasochistically for a return to the bad state. If some individuals cannot live without the sexiness of domination, that seems sad, but it is not really our
when a major theorist tells women in desperate conditions that life offers them only bondage,
business. But
she purveys a cruel lie, and a lie that flatters evil by giving it much more power than it actually has.

Page 418 of 1481


Page 419 of 1481
**Biopolitics**

Page 420 of 1481


Agamben Answers: 2AC (1/6)
FIRST, NO LINK PLAN DOESNT TAKE A STANCE ON THE BODILY SITUATION OF
DETAINEES. IT ONLY STRIPS THE EXECUTIVE OF ONE SOURCE OF CONTROL

SECOND, AGAMBENS ALTERNATIVE TO PLAN IS PARALYZING AND DELINKS THE


LAW AND JUSTICE, ENABLING TOTALITARIANISM

Kohn 2006
[Margaret, Asst. Prof. Poli Sci @ Florida, Bare Life and the Limits of the Law,.Theory and Event, 9:2,
http://muse.jhu.edu/journals/theory_and_event/v009/9.2kohn.html, Retrieved 9-26-06//uwyo-ajl]

Is there an alternative to this nexus of anomie and nomos produced by the state of exception? Agamben invokes genealogy and politics as two interrelated avenues of struggle.
According to Agamben, "To show law in its nonrelation to life and life in its nonrelation to law means to open a space between them for human action, which once claimed for
itself the name of 'politics'." (88) In a move reminiscent of Foucault, Agamben suggests that breaking the discursive lock on dominant ways of seeing, or more precisely not
seeing, sovereign power is the only way to disrupt its hegemonic effects . Agamben clearly hopes that his theoretical analysis could
contribute to the political struggle against authoritarianism, yet he only offers tantalizingly abstract hints
about how this might work. Beyond the typical academic conceit that theoretical work is a decisive element of political struggle, Agamben seems
to embrace a utopianism that provides little guidance for political action. He imagines, "One day humanity will play with law just
as children play with disused objects, not in order to restore them to their canonical use but to free them from it for good." (64) More troubling is his

messianic suggestion that "this studious play" will usher in a form of justice that cannot be made juridical.
Agamben might do well to consider Hannah Arendt's warning that the belief in justice unmediated by law
was one of the characteristics of totalitarianism.
It might seem unfair to focus too much attention on Agamben's fairly brief discussion of alternatives to the sovereignty-exception-law nexus, but it is precisely those sections
that reveal the flaws in his analysis. It also brings us back to our original question about how to resist the authoritarian implications of the state of exception without falling into

the liberal trap of calling for more law . For Agamben, the problem with the "rule of law" response to the war on terrorism
is that it ignores the way that the law is fundamentally implicated in the project of sovereignty with its corollary
logic of exception. Yet the solution that he endorses reflects a similar blindness. Writing in his utopian-mystical mode, he insists, "the

only truly political action, however, is that which severs the nexus between violence and law."(88) Thus Agamben, in spite of all of his theoretical sophistication,

ultimately falls into the trap of hoping that politics can be liberated from law, at least the law tied to violence

and the demarcating project of sovereignty.

THIRD, PLAN IS NECESSARY FOR THE ALTERNATIVE BECAUSE THE EXECUTIVE WILL
STILL VIOLENTLY DETAIN. THIS CREATES A DOUBLE BIND: EITHER THE END RESULT
OF THE ALT IS PLAN AND THERES NO LINK DIFFERENTIAL OR IT DOES THE STATUS
QUO AND DOESNT SOLVE

FOURTH, PERM RECOGNIZE THE TENSION BETWEEN DEMOCRATIC INCLUSION


AND EXCLUSION AND ENGAGE IN THE RESISTANCE OF THE 1AC

Page 421 of 1481


Page 422 of 1481
Agamben Answers: 2AC (2/6)
FIFTH, PERM SOLVES BEST ACKNOWLEDGING THE TENSION OF MODERNITY
WHILE ENGAGING IN DEMOCRATIC STRUGGLE ALLOWS POLITICS BEYOND THE
POLICE STATE IN OPPOSITION TO SOVEREIGNTY AND EXCEPTION

Deranty 2004
[Jean-Philippe, Macquarie University, Agambens challenge to normative theories of modern rights, borderlands e-journal, Vol. 3, No.
1, www.borderlandsejournal.adelaide.edu.au/vol3no1_2004/deranty_agambnschall.htm, acc 1-7-05//uwyo-ajl]

47. If, with Rancire, we define politics not through the institution of sovereignty, but as a continual
struggle for the recognition of basic equality, and thereby strongly distinguish politics from the
police order viewed as the functional management of communities (Rancire 1999), then it is
possible to acknowledge the normative break introduced by the democratic revolutions of the
modern age without falling into a one-sided view of modernity as a neat process of
rationalisation. What should be stressed about modernity is not primarily the list of substantive inalienable and imprescriptible human rights,
but the equal entitlement of all to claim any rights at all. This definition of politics must be accompanied by the parallel
acknowledgment that the times that saw the recognition of the fundamental equality of all also produced the total negation of this principle. But this parallel
claim does not necessarily render the first invalid. Rather it points to a tension inherent in modern communities,
between the political demands of equality and the systemic tendencies that structurally produce stigmatisation and exclusion.

One can acknowledge the descriptive appeal of the biopower hypothesis without renouncing
48.

the antagonistic definition of politics. As Rancire remarks, Foucaults late hypothesis is more about power than it is about politics
(Rancire 2002). This is quite clear in the 1976 lectures (Society must be defended) where the term that is mostly used is that of "biopower". As Rancire
suggests, when the "biopower" hypothesis is transformed into a "biopolitical" thesis, the very possibility of politics becomes problematic. There is a way of
The power that subjects and excludes
articulating modern disciplinary power and the imperative of politics that is not disjunctive.

socially can also empower politically simply because the exclusion is already a form of
address which unwittingly provides implicit recognition. Power includes by excluding, but in
a way that might be different from a ban. This insight is precisely the one that Foucault was developing in his last writings, in his
definition of freedom as "agonism" (Foucault 1983: 208-228): "Power is exercised only over free subjects, and only insofar as they are free" (221). The

exclusionary essence of social structures demands as a condition of its possibility an equivalent


hierarchical,

implicit recognition of all, even in the mode of exclusion. It is on the basis of this recognition that politics
can sometimes arise as the vindication of equality and the challenge to exclusion.

SIXTH, NO ALTERNATIVE AGAMBEN ISOLATES SOVEREIGNTY AS INEVITABLY


EXCLUSIONARY OF NON-POLITICAL LIFE, MEANING THERES NO WAY TO ESCAPE
THAT SYSTEM, RENDERING THEIR OFFENSE INEVITABLE

Page 423 of 1481


Page 424 of 1481
Agamben Answers: 2AC (3/6)
SEVENTH, OUR SPECIFIC USE OF BIOPOLITICS IS GOOD, LEADING TO LIBERAL
DEMOCRACY THAT SOLVES THEIR VIOLENCE AND OPPRESSION CLAIMS

Dickinson, Prof @ University of Cincinnati, 2K4 (Edward Ross, Biopolitics, Fascism,


Democracy: Some Reflections on Our Discourse About Modernity, Central European History,
vol. 37, no. 1, March)

the continuities between early twentieth-century biopolitical discourse and the practices of the
In short,
welfare state in our own time are unmistakasble. Both are instances of the disciplinary society and of biopolitical, regulatory, social-
engineering modernity, and they share that genealogy with more authoritarian states, including the National Socialist state, but also fascist Italy, for example.
analysis can easily become superficial and
And it is certainly fruitful to view them from this very broad perspective. But that
misleading, because it obfuscates the profoundly different strategic and local dynamics of power in the
two kinds of regimes. Clearly the democratic welfare state is not only formally but also substantively quite
different from totalitarianism. Above all, again, it has nowhere developed the fateful, radicalizing dynamic
that characterized National Socialism (or for that matter Stalinism), the psychotic logic that leads from
economistic population management to mass murder. Again, there is always the potential for such a discursive
regime to generate coercive policies. In those cases in which the regime of rights does not successfully produce health, such a system can
and historically does create compulsory programs to enforce it. But again, there are political and policy potentials and
constraints in such a structuring of biopolitics that are very different from those of National Socialist
Germany. Democratic biopolitical regimes require, enable, and incite a degree of self-direction and
participation that is functionally incompatible with authoritarian or totalitarian structures. And this
pursuit of biopolitical ends through a regime of democratic citizenship does appear, historically, to have
imposed increasingly narrow limits on coercive policies, and to have generated a logic or imperative of
increasing liberalization. Despite limitations imposed by political context and the slow pace of discursive change, I think this is the
unmistakable message of the really very impressive waves of legislative and welfare reforms in the 1920s
or the 1970s in Germany.90 Of course it is not yet clear whether this is an irreversible dynamic of such systems.
Nevertheless, such regimes are characterized by sufficient degrees of autonomy (and of the potential for
its expansion) for sufficient numbers of people that I think it becomes useful to conceive of them as
productive of a strategic configuration of power relations that might fruitfully be analyzed as a condition
of liberty, just as much as they are productive of constraint, oppression, or manipulation. At the very least, totalitarianism cannot
be the sole orientation point for our understanding of biopolitics, the only end point of the logic of social
engineering. This notion is not at all at odds with the core of Foucauldian (and Peukertian) theory. Democratic
welfare states are regimes of power/knowledge no less than early twentieth-century totalitarian states;
these systems are not opposites, in the sense that they are two alternative ways of organizing the same thing. But they are two
very different ways of organizing it. The concept power should not be read as a universal stifling night of
oppression, manipulation, and entrapment, in which all political and social orders are grey, are essentially
or effectively the same. Power is a set of social relations, in which individuals and groups have varying
degrees of autonomy and effective subjectivity. And discourse is, as Foucault argued, tactically polyvalent.
Discursive elements (like the various elements of biopolitics) can be combined in different ways to form
parts of quite different strategies (like totalitarianism or the democratic welfare state); they cannot be
assigned to one place in a structure, but rather circulate. The varying possible constellations of power in
modern societies create multiple modernities, modern societies with quite radically differing potentials.

EIGHTH, POWER IS ZERO SUM THE ALTERNATIVE ONLY SHIFTS POWER


ELSEWHERE

Page 425 of 1481


John Mearsheimer, Professor at University of Chicago, 2001 (The Tragedy of Great Power Politics p. 34)

Consequently, states pay close attention to how power is distributed among them, and they make a special effort to maximize their share of world power. Specifically, they
look for opportunities to alter the balance of power by acquiring additional increments of power at the
expense of potential rivals. States employ a variety of meanseconomic, diplomatic, and militaryto shift the balance of power in their favor, even if doing so
makes other states suspicious or even hostile. Because one states gain in power is another states loss, great powers tend

to have a zero-sum mentality when dealing with each other. The trick, of course, is to be the winner in this competition and to dominate
the other states in the system. Thus, the claim that states maximize relative power is tantamount to arguing that states are disposed to think offensively toward other states,

even though their ultimate motive is simply to survive. In short, great powers have aggressive intentions.

Page 426 of 1481


Page 427 of 1481
Agamben Answers: 2AC (4/6)
NINTH, AGAMBEN ESSENTIALIZES THE STATE, IGNORING THE DIFFERENCES
BETWEEN LIBERAL DEMOCRACY AND TOTALITARIANISM

Heins, Vis Prof Poli Sci @ Concordia U and Senior Fellow at the Institute for Social Research in Frankfurt, 2K5 (Volker, Giorgio
Agamben and the Current State of Affairs in Humanitarian Law and Human Rights Policy, 6 German Law Journal No. 5, May,
http://www.germanlawjournal.com/article.php?id=598)

Agamben is not interested in such weighing of costs and benefits because he assumes from the outset
that taking care of the survival needs of people in distress is simply the reverse side of the modern
inclination to ignore precisely those needs and turn life itself into a tool and object of power politics. By way of
conclusion, I will indicate briefly how his view differs from two other, often no less shattering critiques of modern humanitarianism. Martti Koskenniemi warned that humanitarian demands
and human rights are in danger of degenerating into "mere talk."[47] The recent crisis in Darfur, Sudan, can be cited as an example for a situation in which the repeated invocation of human
rights standards and jus cogens norms, like those articulated in the Genocide Convention, might ultimately damage those norms themselves if states are unwilling to act on them.[48] This
criticism implies that human rights should be taken seriously and applied in a reasonable manner. Both David Kennedy and Oona Hathaway have gone one step further by taking issue even
with those who proved to be serious by joining treaties or engaging in advocacy. In a controversial quantitative study, Hathaway contended that the ratification of human rights treaties by
sets of given countries not only did not improve human rights conditions on the ground, but actually correlated with increasing violations.[49] In a similar vein, David Kennedy radicalized
Koskenniemi's point by arguing that human rights regimes and humanitarian law are rather part of the problem than part of solution, because they "justify" and "excuse" too much.[50] To
some extent, this is an effect of the logic of legal reasoning: marking a line between noncombatants and combatants increases the legitimacy of attacking the latter, granting privileges to
lawful combatants delegitimizes unlawful belligerents and dramatically worsens their status. On the whole, Kennedy is more concerned about the dangers of leaving human rights to
international legal elites and a professional culture which is blind for the mismatch between lofty ideals and textual articulations on the one side, and real people and problems on the other
side.[51] Whereas these authors reveal the "dark sides" of overly relying on human rights talk and treaties, the moral fervor of activists or the routines of the legal profession, Agamben claims
that something is wrong with human rights as such, and that recent history has demonstrated a deep affinity between the protection and the infringement of these rights. Considered in this
light, the effort of the British aid organization Save the Children, for instance, to help children in need both in Britain and abroad after World War I faithful to George Bernard Shaw's saying,
"I have no enemies under seven"is only the flip side of a trend to declare total war on others regardless of their age and situation. This assertion clearly goes far beyond the voices of other
pessimists. Agamben's work is understandable only against the backdrop of an entirely familiar mistrust of liberal democracy and its ability to cultivate nonpartisan moral and legal

According to Agamben, democracy does not threaten to turn into totalitarianism, but rather both
perspectives.

regimes smoothly cross over into one another since they ultimately rest on the same foundation of a
political interpretation of life itself.[52] Like Carl Schmitt, Agamben sees the invocation of human rights by democratic governments as well as the "humanitarian
concept of humanity"[53] as deceptive manouvers or, at least, as acts of self-deception on the part of the liberal bourgeois subject. The difference between Agamben and Schmitt lies in the
fact that Schmitt fought liberal democracy in the name of the authoritarian state, while Agamben sees democracy and dictatorship as two equally unappealing twins. Very much unlike

confronts us with a mode of thinking in vaguely felt resemblances in lieu of distinctly


Schmitt, the Italian philosopher

perceived differences. Ultimately, he offers a version of Schmitt's theory of sovereignty that changes its political valence and downplays the
difference between liberal democracy and totalitarian dictatorshipa difference about which Adorno
once said that it "is a total difference. And I would say," he added, "that it would be abstract and in a problematic way
fanatical if one were to ignore this difference."[54]

TENTH, DESIRE IS TOO DYNAMIC TO BE CONTAINED BY THE SOVEREIGN ITS


FLUDITY ENABLES BIOPOWER THAT TRANSCENDS THE STATE OF EXCEPTION BY
CREATING NEW FORMS OF LIFE OUTSIDE THE SYSTEM ***

Neilson 2004
[Brett, University of Western Sydney, Potenza Nuda? Sovereignty, Biopolitics, Capitalism, Contretemps 5, December 2004,
www.usyd.edu.au/contretemps/5december2004/neilson.pdf, acc 1-7-04//uwyo-ajl]

Like Agamben, Hardt and Negri take as a point of departure the Foucauldian account of biopolitics as a system of rule that emerges at the beginning of the modern era with the
exercise of power over life itself. Importantly, however, they extend Foucaults argument by drawing on Gilles Deleuzes Postscript on the Society of Control. Foucault
describes the modern system of disciplinary rule that fixes individuals within institutions (hospitals, schools, prisons, factories, and so on) but does not succeed in consuming

, Hardt and Negri trace the emergence of a new


them completely in the rhythm of productive practices or productive socialization. By contrast

mode of power that is expressed as a control that extends throughout the consciousness and bodies of
the populationand at the same time across the entirety of social relations.9 In so doing, they combine the Deleuzian emphasis on free-floating and mobile logics
of control (data banking, risk management, electronic tagging, and so on) with an attention to the productive dimension of biopower (living labour) derived from the work of
exponents of Italian operaismo like Paolo Virno and Christian Marazzi. While Hardt and Negri question the tendency of these thinkers to understand all contemporary forms of
production on the horizon of communication and language, they are clearly indebted to their notions of immaterial labour and general intellect (which in turn derive from a

productive aspect of biopower that places Hardt and Negri


reading of the famous Fragment on Machines from Marxs Grundrisse). It is this emphasis on the

at odds with Agamben on bare lifea concept that, for them, excludes the question of labour from the field of theoretical observation. Thus,

Page 428 of 1481


in a footnote, they comment critically on a line of Benjamin-inspired interpretations of Foucault (from Derridas Force of Law to Homo Sacer itself): It seems fundamental to us,
however, that all of these discussions be brought back to the question of the productive dimension of the bios, identifying in other words the materialist dimension of the
concept beyond any conception that is purely naturalistic (life as zo) or simply anthropological (as Agamben in particular has a tendency to do, making the concept in effect
indifferent).10 With this identification of what Agamben calls indistinction as indifference (indifference to productive power of cooperation between human minds and bodies),

, Agambens philosophical specification of the


Hardt and Negri voice their most severe reservations about the concept of bare life. For them

negative limit of humanity displays behind the political abysses that modern totalitarianism has
constructed the (more or less heroic) conditions of human passivity.11 The apparatus of the sovereign ban condemns humanity to
inactivity and despair. By contrast, Hardt and Negri claim that bare life must be raised up to the dignity of productive
power. Rather than reducing humanity to mere living matter, the exceptional power of the modern state becomes
effective at precisely the moment when social cooperation is seen no longer the result of the investment of capital but an autonomous
power, the a priori of every act of production.12 Try as it may to relegate humanity to minimal naked life (or zo), the modern constituted order
cannot destroy the enormous creativity of living labour or expunge its powers of cooperative production.

Page 429 of 1481


Page 430 of 1481
Agamben Answers: 2AC (5/6)
ELEVENTH, AGAMBEN MISUNDERSTANDS THE SHIFTS IN SOVEREIGNTY, PAPERING
OVER INSIDIOUS VIOLENCE

Hardt & Dumm 2000


[Michael & Thomas, Sovereignty, Multitudes, Absolute Democracy: A Discussion between Michael Hardt and Thomas
Dumm about Hardt and Negri's Empire, Theory & Event 4:3, Muse//uwyo-ajl]

The most significant difference between our projects, though, is that Agamben dwells on modern
sovereignty whereas we claim that modern sovereignty has now come to an end and transformed into a
new kind of sovereignty, what we call imperial sovereignty. Imperial sovereignty has nothing to do with
the concentration camp. It no longer takes the form of a dialectic between Self and Other and does not
function through any such absolute exclusion, but rules rather through mechanisms of differential
inclusion, making hierarchies of hybrid identities. This description may not immediately give you the same
sense of horror that you get from Auschwitz and the Nazi Lager, but imperial sovereignty is certainly just
as brutal as modern sovereignty was, and it has its own subtle and not so subtle horrors.

TWELFTH, AGAMBENS USE OF THE CAMP CONFLATES VICTIM WITH OPPRESSOR,


PREVENTING US FROM HOLDING PERPETRATORS RESPONSIBLE AND DESTROYING
ANY ETHICAL OBLIGATION TO ACT SINCE WE POSIT EVERYONE AS THE VICTIM

Sanyal, Assist Prof of French @ UC Berkeley, 2K2 (Debarati, A Soccer Match in Auschwitz: Passing Culpability in
Holocaust Criticism, Representations, Issue 79, Caliber)

Agambens radicalization of Levis gray zone has


Beyond the problems inherent in a transhistorical treatment of shame and complicity,
even more disturbing consequences for understanding the relations of power within the camps. The
unstable boundary between oppressor and oppressed in the gray zone is radicalized in Agambens
account such that the two positions appear to be reciprocal and convertible: It seems, in fact, that the only thing that
interests him [Levi] is what makes judgement impossible: the gray zone in which victims become executioners and executioners become victims (Remnants,
his emphasis on the camps as sites for
17).18 While Agamben nowhere suggests that perpetrators and victims truly did exchange positions,
a potentially endless circulation of guilt nevertheless takes the convertibility of victims and executioners
as a structural given. Primo Levi, however, was at pains to emphasize that this convertibility was a
politically expedient fiction designed to erase the difference between victim and executioner by forcing Jews to
participate in the murder and cremation of their own. He also stressed the singular, unimaginable strain such a predicament must have exerted upon the SK.
To transform such a charged, ambiguous lived reality into a formal conception of convertibility has
disturbing ethical consequences. It suggests that the perpetrators too, by virtue of occupying this zone of
radical inversion and participating in the traumatic conditions of camp life, could be perceived as victims.
The fallacy of this structural reciprocity, however, is refuted by Levi in a cautionary preface to his discussion of the Sonderkommando: This mimesis, this
identification or imitation or exchange of roles between oppressor and victim, has provoked much discussion. . . . I do not know, and it does not much interest
me to know, whether in my depths there lurks a murderer, but I do know that the murderers existed, not only in Germany, and still exist, retired or on active
duty, and that to confuse them with their victims is a moral disease or an aesthetic affectation or a sinister sign of complicity; above all, it is a precious service
The conceptualization of the gray zone as a transhistorical
rendered (intentionally or not) to the negators of truth. (Drowned, 50)
and trans-subjective site of culpability, in which victims become executioners and executioners become
victims, thus conflates the positions of Muslims, Prominents, Kapos, and SS in a gesture that reaches beyond the
concentration camp experience to include us in a general condition of traumatic culpability. This

Page 431 of 1481


blurring of subject positions leads to a vision of inescapable guilt, in which we are always already
collectively steeped in the eliminationist logic that led to the concentration camp and continue
unknowingly to perpetuate its violence. But just as this vision posits an ever-encroaching web of complicity, it also, paradoxically,
proposes an infinitely elastic notion of victimhood. If we are obscurely complicit with the logic of the soccer match, the
irrealization of violence in daily life, we are also comparably violated by the historical trauma of the camps. The
generalization of complicity and victimization not only dismantles the historical specificity of the camps
and the survivors testimonies. It also, more disturbingly, coopts the figure of the victim as an other
who is but an avatar of ourselves, a point I will address in a moment.

Page 432 of 1481


Page 433 of 1481
Agamben Answers: 2AC (6/6)
THIRTEENTH, THEORY IS IRRELEVENT ABSENT SPECIFIC APPLICATION MUST
COMBINE THEORY AND PRACTICE FOR A PHILOSOPHY AS LIFE

Foucault 82
[Michel, God, Politics and Ethics: An Interview, The Foucault Reader, Trans. Catherine Porter, Ed. Paul Rabinow,
373-4//uwyo-ajl]

M.F. That's right. When Habermas was in Paris, we talked at some length, and in fact I was quite struck by his observation of the extent to which the problem of
Heidegger and of the political implications of Heidegger's thought was quite a pressing and important one for him. One thing he said to me has left me musing,

After explaining how Heidegger's thought indeed constituted a


and it's something I'd like to mull over further.

political disaster, he mentioned one of his professors who was a great Kantian, very well-known in the '30s,
and he explained how astonished and disappointed he had been when, while looking through card catalogues one day, he found some texts from around 1934

by this illustrious Kantian that were thoroughly Nazi in orientation.


I have just recently had the same experience with Max Pohlenz, who heralded the universal values of Stoicism all his life. I came across a text of his from 1934
devoted to Fiihrertum in Stoicism. You should reread the introductory page and the book's closing remarks on the Fuhrersideal and on the true humanism
constituted by the Volk under the inspiration of the leader's direction-Heidegger never wrote anything more disturbing. Nothing in this condemns Stoicism or
Kantianism, needless to say.

there is a very tenuous "analytic" link between a philosophical


But I think that we must reckon with several facts:

conception and the concrete political attitude of someone who is appealing to it; the "best" theories do not
constitute a very effective protection against disastrous political choices; certain great themes
such as "humanism" can be used to any end whatever-for example, to show with what gratitude Pohlenz would have greeted Hitler.

a demanding, prudent,
I do not conclude from this that one may say just anything within the order of theory, but, on the contrary, that

"experimental" attitude is necesary; at every moment, step by step, one must confront what one is
thinking and saying with what one is doing, with what one is. I have never been too concerned about people who say: "You are
bor-rowing ideas from Nietzsche; well, Nietzsche was used by the Nazis, therefore. . ."; but, on the other hand, I have always been

concerned with linking together as tightly as possible the historical and theoretical analysis of
power relations, institu-tions, and knowledge, to the movements, critiques, and experiences that call them
into question in reality. If I have insisted on all this "practice," it has not been in order to "apply" ideas, but in order to put them to the test and
modify them. The key to the Personal poetic attitude of a philosopher is not to be sought in his ideas, as if

it could be deduced from them, but rather in his philosophy-as-life, in his philosophicallife, his ethos.

Among the French philosophers who participated in the Resistance during the war, one was Cavailles, a historian of mathematics who was interested in the
development of its internal structures. None of the philosophers of engagement-Sartre, Simone de Beauvoir, Merleau-Ponty-none
of them did a thing.

FOURTEENTH, EVEN IF THE LAW WAS ORIGINALLY FOUNDED ON VIOLENCE, IT


NOW OPERATES IN A NON-VIOLENT WAY

Deranty 2004
[Jean-Philippe, Macquarie University, Agambens challenge to normative theories of modern rights, borderlands e-journal, Vol. 3,
No. 1, www.borderlandsejournal.adelaide.edu.au/vol3no1_2004/deranty_agambnschall.htm, acc 1-7-05//uwyo-ajl]

29. The problem with this strategic use of the decisionistic tradition is that it does not do justice to the complex
relationship that these authors establish between violence and normativity, that is, in the end the very normative nature of
their theories. In brief, they are not saying that all law is violent, in essence or in its core, rather

that law is dependent upon a form of violence for its foundation. Violence can found the
law, without the law itself being violent. In Hobbes, the social contract, despite the absolute

Page 434 of 1481


nature of the sovereign it creates, also enables individual rights to flourish on the basis
of the inalienable right to life (see Barret-Kriegel 2003: 86).
30. In Schmitt, the decision over the exception is indeed "more interesting than the regular case", but only because it makes the regular case possible. The
"normal situation" matters more than the power to create it since it is its end (Schmitt 1985: 13). What Schmitt has in mind is not the indistinction between fact
and law, or their intimate cohesion, to wit, their secrete indistinguishability, but the origin of the law, in the name of the law. This explains why the primacy given
by Schmitt to the decision is accompanied by the recognition of popular sovereignty, since the decision is only the expression of an organic community.
Decisionism for Schmitt is only a way of asserting the political value of the community as homogeneous whole, against liberal parliamentarianism. Also, the
evolution of Schmitts thought is marked by the retreat of the decisionistic element, in favour of a strong form of institutionalism. This is because, if indeed the
juridical order is totally dependent on the sovereign decision, then the latter can revoke it at any moment. Decisionism, as a theory about the origin of the law,
leads to its own contradiction unless it is reintegrated in a theory of institutions (Kervgan 1992).

Agamben sees these authors as establishing a circularity of law and violence,


31. In other words,

when they want to emphasise the extra-juridical origin of the law, for the laws sake. Equally, Savignys
polemic against rationalism in legal theory, against Thibaut and his philosophical ally Hegel, does not amount to a recognition of the capture of life by the law,
For Agamben, it seems, the origin and the
but aims at grounding the legal order in the very life of a people (Agamben 1998: 27).

essence of the law are synonymous, whereas the authors he relies on thought rather
that the two were fundamentally different.

32. Agamben obviously knows all this. He argues that it is precisely this inability of the decisionists to hold on to their key insight,
. But this reading does not meet
the anomic core of norms, which gives them the sad distinction of accurately describing an evil order

the objection to his problematic use of that tradition.

Page 435 of 1481


Page 436 of 1481
#2 Alternative Kills Liberation:
1AR (1/2)
EXTEND THE 2AC KOHN 2006 EV

First Agambens alternative is so abstract that it offers no mean of liberation.


Delinking the law and justice enables unchecked power that allows totalitarian
violence, flipping their argument.

SECOND, RIGHTS ARE CRITICAL TO HUMYN DIGNITY--AGAMBENS ALTERNATIVE


FAILS BECAUSE:
1. IGNORES THE VALUE OF RIGHTS IN RESISTING EXPLOITATION
2. FOSTERS GESTURAL POLITICS THAT CANNOT ADDRESS THE PROBLEMS OF THE
OPPRESSED

Frances Daly, Research Fellow, Philosophy Department, Australian National University, The non-citizen

and the concept of human rights, BORDERLANDS E-JOURNAL v. 3 n. 1, 2004,


www.borderlandsejournal.adelaide.edu.au/vol3no1_2004/daly_noncitizen.htm .

27. Certainly , Agamben calls for making all residents of extraterritorial space (which would include both citizen and non-citizen) as existing within a position of exodus or refuge, and
in this we can perhaps see some basis for resistance. A position of refuge, he argues, would be able to "act back onto" territories as states and 'perforate' and alter' them such that "the citizen
would be able to recognize the refugee that he or she is" (Agamben, 2000: 26). In this Agamben directs our attention usefully to the importance of the refugee today both in terms of the
plight of refugees and their presence in questioning any assumption about citizen rights, and also in placing the refugee, or "denizen" as he says using Tomas Hammar's term, as the central

reduces the concepts of right and the values they involve to forms of
figure of a potential politics (Agamben, 2000: 23). But he also

State control, eliding all difference within right and thereby terminating an understanding of the reasons
for a disjuncture between legality and morality and of an existing separation of rights from the ideal of
ethicality, in which liberation and dignity exist to be realized beyond any form of contract.
28. It is always possible to suppose that a self-fashioned potentiality is simply available to us, and in some senses it is, but not because a type of theory merely posits the social and the
historical as completely open to our manipulation or 'perforation'. Likewise, we cannot merely assume that changing 'forms of life' necessarily amount to types of refusal. Such a claim would
only make sense if it were put forward on the basis of an appreciation of an impulse to freedom from particular types of constraint and oppression. It would also require a sense of how this
impulse takes place within a variety of conditions, some of which might be easily altered and some of which might not. In the absence of an engaged sense of what this impulse means, and of
the context in which elements of freedom and unfreedom do battle, it is impossible to speculate on the nature of the subjectivity or potentiality which might be emerging or which might be in

Agamben merely presumes that a strategy by which we all identify as refugees will renew a
stages of decomposition.

politics and thereby end the current plight of the refugee, as if no other reality impinges on this
identification. This is also assumed on the basis that the State in Agamben's theorizing, the abstraction of an all-encompassing, leviathan
State is equally, readily and easily liable to perforation. This contradiction is indicative of a wider problem

where what we encounter is a form of critique that is oddly inappropriate to the type of issue it
addresses.

29. Much can be said in criticism of the doctrine of right, of the limited nature of the understanding of freedom and rights in documents on rights,
of the assumption of the place of citizen rights as the locus of the fundamental rights of the human, and most significantly, the absence of any sense of the undetermined nature of what being

But what must be stated, I feel, is that it would be a serious impoverishment of the ethical problem
might mean.

that we currently face to deny any potential value of rights in carrying forth traces of an impetus towards
human dignity, of the ideals of freedom and equality, and to thus reduce rights to what might be termed

Page 437 of 1481


an absolute politics. Rights cannot be reduced to citizenship rights as if the ideas of rights and citizenship
are coterminus. What most critically needs to be understood is, firstly, why values of freedom and
equality have such a limited and fragile place within conditions of such inordinate legalism, and, secondly, what the absence
of freedom, which the cause of human rights inevitably suggests, means for the installation of any such
rights. Without such an understanding we are left with a gestural politics that contains a posture of
radicalism but one which fails to connect the aspirations of those who are struggling to achieve
elementary rights with a vision of a world that could accord them a degree of dignity. To acknowledge this
is not to be seduced by concepts of right or law, but is rather to refuse the denial of a radical questioning
of the possibilities with which a discourse presents us. Benjamin's understanding of a genuinely messianic
idea is something that is "not the final end of historical progress, but rather its often failed and finally
accomplished interruption" (Benjamin, 1974: 1231). We find this in values that resist exploitation and assaults upon
human dignity. And it is this realm that currently requires urgent, emphatic and significant renewal.

Page 438 of 1481


Page 439 of 1481
#2 Alternative Kills Liberation:
1AR (2/2)
RATIONAL, INDIVIDUAL RIGHTS ARE GOOD ONLY WAY TO PREVENT FUTURE
HOLOCAUSTS

Robert Tracinski, Received his undergraduate degree in Philosophy from the University of Chicago and studied
with the Objectivist Graduate Center and Editorial Director of the Ayn Rand Institute, Why It Can Happen Again, Ayn
Rand Institute, April 22, 2003, http://www.aynrand.org/site/News2?page=NewsArticle&id=7888&news_iv_ctrl=1021,
UK: Fisher

Most people avoid these stark implications by retreating to a compromise between self-sacrifice and self-
interest. Calls for sacrifice are proper, they say, but should not be taken "too far." The Fascists condemned
this approach as hypocrisy. They took the morality of sacrifice to its logical conclusion. They insisted, in
the words of Italian Fascist Alfredo Rocco, on "the necessity, for which the older doctrines make little
allowance, of sacrifice, even up to the total immolation of individuals." And the Nazis certainly practiced
what Rocco preached. A central goal of the concentration camps, wrote survivor Bruno Bettelheim, was
"to break the prisoners as individuals, and to change them into a docile mass." "There are to be no more
private Germans," one Nazi writer declared; "each is to attain significance only by his service to the
state." The goal of National Socialism was the relentless sacrifice of the individual: the sacrifice of his
mind, his independence, and ultimately his person. A free country is based on precisely the opposite
principle. To protect against what they called the "tyranny of the majority," America's Founding Fathers
upheld the individual's right to "life, liberty, and the pursuit of happiness." The implicit basis of American
government was an ethics of individualism--the view that the individual is not subordinate to the
collective, that he has a moral right to his own interests, and that all rational people benefit under such a
system. Today, however, self-sacrifice is regarded as self-evidently good. True, most people do not want
a pure, consistent system of sacrifice, as practiced by the Nazis. But once the principle is accepted, no
amount of this "virtue" can ever be condemned as "too much." We will not have learned the lessons of
the Holocaust until we completely reject this sacrifice-worship and rediscover the morality of
individualism.

Page 440 of 1481


Page 441 of 1481
#5 Perm: 1AR
EXTEND THE PERM. RECOGNIZING MODERNITYS PROBLEM WITH EXCLUSION
WHILE USING DEMOCRATIC STRUGGLE ENABLES A CONTESTATION OF DIGNITY
THAT CHALLENGES THE EXCEPTION, AS SHOWN BY DERANTY 2004

ALSO, SOVEREIGNTY MUST BE USED STRATEGICALLY CRITIQUE CAN BE


SIMULTANEOUS

Lombardi, Assoc Prof of Political Science @ Tampa, 96 (Mark Owen, Perspectives on Third-World Sovereignty, P.
161)

Sovereignty is in our collective minds. What we look at, the way we look at it and what we expect to see must be altered. This is the call for
international scholars and actors. The assumptions of the paradigm will dictate the solution and approaches
considered. Yet, a mere call to change this structure of the system does little except activate reactionary
impulses and intellectual retrenchment. Questioning the very precepts of sovereignty, as has been done in many
instances, does not in and of itself address the problems and issues so critical to transnational relations. That
is why theoretical changes and paradigm shifts must be coterminous with applicative studies. One does
not and should not precede the other. We cannot wait until we have a neat self-contained and accurate
theory of transnational relations before we launch into studies of Third-World issues and problem-solving. If we
wait we will never address the latter and arguably most important issue-area: the welfare and quality of
life for the human race.

THE PERM USES POLITICAL ENGAGEMENT TO AVOID THE ESSENTIALISM OF THE


SOVEREIGN AND AGAMBENS ALTERNATIVE BY USING CONTINGENCY TO
CHALLENGE THE ATROCITY THAT BOTH MAKE INEVITABLE

Deranty 2004
[Jean-Philippe, Macquarie University, Agambens challenge to normative theories of modern rights, borderlands e-journal, Vol. 3,
No. 1, www.borderlandsejournal.adelaide.edu.au/vol3no1_2004/deranty_agambnschall.htm, acc 1-7-05//uwyo-ajl]

49. This proposal rests on a logic that challenges Agambens reduction of the
overcoming of the classical conceptualisation of potentiality and actuality to the single
Heideggerian alternative. Instead of collapsing or dualistically separating potentiality and
actuality, one would find in Hegels modal logic a way to articulate their negative, or
reflexive, unity, in the notion of contingency. Contingency is precisely the potential as existing,
a potential that exists yet does not exclude the possibility of its opposite (Hegel 1969: 541-
554). Hegel can lead the way towards an ontology of contingency that recognises the place of
contingency at the core of necessity, instead of opposing them. The fact that the impossible
became real vindicates Hegels claim that the impossible should not be opposed to the actual.

Page 442 of 1481


Instead, the possible and the impossible are only reflected images of each other and, as
actual, are both simply the contingent. Auschwitz should not be called absolute necessity
(Agamben 1999a: 148), but absolute contingency. The absolute historical necessity of
Auschwitz is not "the radical negation" of contingency, which, if true, would indeed necessitate
a flight out of history to conjure up its threat. Its absolute necessity in fact harbours an indelible
core of contingency, the locus where political intervention could have changed things,
where politics can happen. Zygmunt Baumans theory of modernity and his theory about the
place and relevance of the Holocaust in modernity have given sociological and contemporary
relevance to this alternative historical-political logic of contingency (Bauman 1989).

Page 443 of 1481


Page 444 of 1481
#5 Perm: Ext
AMBIGUOUS MODERNITY THAT ACKNOWLEDGES INCOMPLETION PROVIDES THE
TOOLS FOR RESISTING OPPRESSION

Deranty 2004
[Jean-Philippe, Macquarie University, Agambens challenge to normative theories of modern rights, borderlands e-journal, Vol. 3,
No. 1, www.borderlandsejournal.adelaide.edu.au/vol3no1_2004/deranty_agambnschall.htm, acc 1-7-05//uwyo-ajl]

50. In the social and historical fields, politics is only the name of the contingency that strikes
at the heart of systemic necessity. An ontology of contingency provides the model with
which to think together both the possibility, and the possibility of the repetition of,
catastrophe, as the one heritage of modernity, and the contingency of catastrophe as logically
entailing the possibility of its opposite. Modernity is ambiguous because it provides the
normative resources to combat the apparent necessity of possible systemic
catastrophes. Politics is the name of the struggle drawing on those resources.

51. This ontology enables us also to rethink the relationship of modern subjects to rights.
Modern subjects are able to consider themselves autonomous subjects because legal
recognition signals to them that they are recognised as full members of the community,
endowed with the full capacity to judge. This account of rights in modernity is precious
because it provides an adequate framework to understand real political struggles, as fights
for rights. We can see now how this account needs to be complemented by the notion of
contingency that undermines the apparent necessity of the progress of modernity. Modern
subjects know that their rights are granted only contingently, that the possibility of the
impossible is always actual. This is why rights should not be taken for granted. But this does
not imply that they should be rejected as illusion, on the grounds that they were disclosed as
contingent in the horrors of the 20th century. Instead, their contingency should be the
reason for constant political vigilance.

Page 445 of 1481


Page 446 of 1481
#7 Good Biopower: 1AR (1/2)
AGAMBEN IS WRONG BIOPOWER DOESNT CAUSE EXCEPTION OR VIOLENCE,
BUT MAINTAINS LIFE

Ojakangas 2005
[Mike, Helsinki Collegium for Advanced Studies, Impossible Dialogues on Bio-Power: Agamben and
Foucault, Foucault Studies 2 (5-28), www.foucault-studies.com/no2/ojakangas1.pdf, acc. 9-24-06//uwyo-
ajl]

In fact, the history of modern Western societies would be quite incomprehensible without taking into account that there exists a form of power which refrains from

The effectiveness of bio-power can be seen lying


killing but which nevertheless is capable of directing peoples lives.

precisely in that it refrains and withdraws before every demand of killing, even though these demands would derive from the demand of
justice. In biopolitical societies, according to Foucault, capital punishment could not be maintained except by invoking less the enormity of the crime itself than the monstrosity
of the criminal: One had the right to kill those who represented a kind of biological danger to others. However, given that the right to kill is precisely a sovereign right, it
can be argued that the bio-political societies analyzed by Foucault were not entirely bio-political. Perhaps, thereneither has been nor can be a society that is entirely
bio-political. Nevertheless, the fact is that present-

day European societies have abolished capital punishment. In them, there are no longer exceptions. It is th
e very right to kill that has been called into question. However, it is not called into question because of enlightened moral sentiments, but
rather because of the deployment of bio-political thinking and practice.

For all these reasons, Agambens thesis, according to which the concentration camp is the fundamental bio-political paradigm of the West, has to be corrected. The
bio-political paradigm of the West is not the concentration camp, but, rather, the present-
day welfare society and, instead of homo sacer, the paradigmatic figure of the bio-
political society can be seen, for example, in the middle-class Swedish social-democrat. Although
this figure is an object and a product of the huge bio-
political machinery, it does not mean that he is permitted to kill without committing homicide. Actually, the fact that he eventually dies, seems to be his greatest crime against t
he machinery. (In bio-political societies, death is not only something to be hidden away, but, also, as Foucault stresses, the most shameful thing of all.
) Therefore, he is not exposed to an unconditional threat of death, but rather to an unconditional retreat of all dying. In fact, the bio-
political machinery does not want to threaten him, but to encourage him, with all its material and spiritual capacities, to live healthily, to live long and to live happily
even when, in biological terms, he should have been dead longago.

This is because biopower is not bloody power over bare life for its own sake but pure power over all life for the

sake of the living. It is not power but the living, the condition of all life individual as well as collective
that is the measure of the success of bio-power.

BIOPOLITICS IS NOT THE PROBLEM IN AND OF ITSELF ITS BIOPOLITICS DEPLOYED


IN TOTALITARIANS SOCIETIES WHICH IS BAD OUR STRENGTHENING OF
DEMOCRATIC STRUCTURES SOLVES THEIR IMPACT

Dickinson, Prof @ University of Cincinnati, 2K4 (Edward Ross, Biopolitics, Fascism,


Democracy: Some Reflections on Our Discourse About Modernity, Central European History,
vol. 37, no. 1, March)

In an important programmatic statement of 1996 Geoff Eley celebrated the fact that Foucaults ideas have fundamentally directed attention away from
institutionally centered conceptions of government and the state . . . and toward a dispersed and decentered notion of power and its microphysics.48 The
broader, deeper, and less visible ideological consensus on technocratic reason and the ethical unboundedness of science was the focus of his interest.49
But the power-producing effects in Foucaults microphysical sense (Eley) of the construction of social bureaucracies and
social knowledge, of an entire institutional apparatus and system of practice ( Jean Quataert), simply do not explain Nazi policy.50 The
destructive dynamic of Nazism was a product not so much of a particular modern set of ideas as of a
particular modern political structure, one that could realize the disastrous potential of those ideas. What

Page 447 of 1481


was critical was not the expansion of the instruments and disciplines of biopolitics, which occurred everywhere in Europe.
Instead, it was the principles that guided how those instruments and disciplines were organized and used, and the external
constraints on them. In National Socialism, biopolitics was shaped by a totalitarian conception of social
management focused on the power and ubiquity of the vlkisch state. In democratic societies, biopolitics
has historically been constrained by a rights-based strategy of social management. This is a point to which I will return
shortly. For now, the point is that what was decisive was actually politics at the level of the state. A comparative framework can help us to clarify this point.
Other states passed compulsory sterilization laws in the 1930s indeed, individual states in the United States had already
begun doing so in 1907. Yet they did not proceed to the next steps adopted by National Socialism mass sterilization,
mass eugenic abortion and murder of the defective. Individual figures in, for example, the U.S. did make such suggestions. But neither the
political structures of democratic states nor their legal and political principles permitted such policies
actually being enacted. Nor did the scale of forcible sterilization in other countries match that of the Nazi program. I do not mean to
suggest that such programs were not horrible; but in a democratic political context they did not develop
the dynamic of constant radicalization and escalation that characterized Nazi policies.

Page 448 of 1481


Page 449 of 1481
#7 Good Biopower: 1AR (2/2)
BIOPOLITICS DOESNT CAUSE ATROCITY

Ojakangas 2005
[Mike, Helsinki Collegium for Advanced Studies, Impossible Dialogues on Bio-Power: Agamben and
Foucault, Foucault Studies 2 (5-28), www.foucault-studies.com/no2/ojakangas1.pdf, acc. 9-24-06//uwyo-
ajl]

For Foucault, the coexistence in political structures of large destructive

mechanisms and institutions oriented toward the care of individual life was

something puzzling: It is one of the central antinomies of our political

reason. However, it was an antinomy precisely because in principle the

sovereign power and bio-power are mutually exclusive. How is it possible

that the care of individual life paves the way for mass slaughters? Although

Foucault could never give a satisfactory answer to this question, he was

convinced that mass slaughters are not the effect or the logical conclusion of

bio-political rationality. I am also convinced about that. To be sure, it can be

argued that sovereign power and bio-power are reconciled within the modern

state, which legitimates killing by bio-political arguments. Especially, it can be

argued that these powers are reconciled in the Third Reich in which they

seemed to coincide exactly. To my mind, however, neither the modern

state nor the Third Reich in which the monstrosity of the modern state is

crystallized are the syntheses of the sovereign power and bio-power, but,

rather, the institutional loci of their irreconcilable tension. This is, I believe,

what Foucault meant when he wrote about their demonic combination.

Page 450 of 1481


Page 451 of 1481
#9 Essentialism: 1AR (1/2)
EXTEND 2AC NUMBER 3, HEINS 2005 EVIDENCE. GROUP IT. THE CRITICISM
ESSENTIALIZES OPPRESSION BY COLLAPSING DEMOCRACY AND TOTALITARIANISM
INTO A SINGLE TRANSCENDENT ENTITY, DESTROYING CRTICISIM OF DIFFERENT
FORMS OF OPPRESSION

ALSO, THAT TAKES OUT THEIR IMPACT BECAUSE AGAMBENS TRANSHITORICAL


ARGUMENT CONFLATES DIFFERENT HISTORICAL ERAS. GLOBAL CAPITAL IS MORE
DECENTRALIZED THAN FASCISM, MAKING THEIR TERMINAL OFFENSE IMPOSSIBLE.

IT ALSO PROVES THAT THE PERM SOLVES BEST BECAUSE WE CAN ENGAGE IN
CRITICISM OF THE SHORTCOMINGS OF RIGHTS, WHILE STILL PROVIDING THE
MECHANISMS NECESSARY TO PREVENT FULL SCALE FASCISM

AGAMBEN ESSENTIALIZES INTERNMENT INTO A TRANSHISTORICAL ENTITY,


PREVENTING TESTIMONY NECESSARY TO MOBILIZE AGAINST DIVERSE FORMS OF
OPPRESSION AND TO CRITICIZE THE SHORTCOMINGS OF WESTERN RIGHTS
DISCOURSE FROM WITHIN ***

Deranty 2004
[Jean-Philippe, Macquarie University, Agambens challenge to normative theories of modern rights, borderlands e-journal, Vol. 3, No.
1, www.borderlandsejournal.adelaide.edu.au/vol3no1_2004/deranty_agambnschall.htm, acc 1-7-05//uwyo-ajl]

11. In the case of empirical examples, the erasure of difference between phenomena seems
particularly counter-intuitive in the case of dissimilar modes of internment. From a
practical point of view, it seems counter-productive to claim that there is no substantial
difference between archaic communities and modern communities provided with the
language of rights, between the lawlessness of war times and democratic discourse. There
must be a way of problematising the ideological mantra of Western freedom, of
modernitys moral superiority, that does not simply equate it with Nazi propaganda (Ogilvie
2001). Habermas and Honneth probably have a point when they highlight the advances
made by modernity in the entrenchment of rights. If the ethical task is that of testimony,
then our testimony should go also to all the individual lives that were freed from
alienation by the establishment of legal barriers against arbitrariness and exclusion. We
should heed Honneths reminder that struggles for social and political emancipation have often
privileged the language of rights over any other discourse (Fraser, Honneth 2003). To reject the
language of human rights altogether could be a costly gesture in understanding past
political struggles in their relevance for future ones, and a serious strategic, political loss for

Page 452 of 1481


accompanying present struggles. We want to criticise the ideology of human rights, but
not at the cost of renouncing the resources that rights provide. Otherwise, critical
theory would be in the odd position of casting aspersions upon the very people it
purports to speak for, and of depriving itself of a major weapon in the struggle against
oppression.

Page 453 of 1481


Page 454 of 1481
#9 Essentialism: 1AR (2/2)
AND, AGAMBENS FOCUS ON LANGUAGE IGNORES HOW HISTORICAL CONDITIONS
HAVE CHANGED, PREVENTING RESISTANCE TO OPPRESSION

Wark 2004
[McKenzie, Re: <nettime> Agamben: No to Bio-Political Tattooing, posted to
nettime mailing list, January 27, amsterdam.nettime.org/Lists-Archives/nettime-l-
0401/msg00092.html, acc 1-7-2004//uwyo-ajl]

What never occurs to Agamben is to inquire into the historical rather than philological --
conditions of existence of this most radical challenge to the state. Agamben reduces everything
to power and the body. Like the Althusserians, he too has dispensed with problem of relating
together the complex of historical forces. In moving so quickly from the commodity form to the
state form, the question of the historical process of the production of the abstraction and the
abstraction of production disappears, and with it the development of class struggle.

AGAMBENS TRANSHISTORICAL MODEL OF BIOPOWER COLLAPSES HISTORY,


IGNORING ITS CONTEXTUAL FUNCTION

Panagia 99
[Davide, The Sacredness of Life and Death: Giorgio Agamben's Homo Sacer and the Tasks of Political
Thinking, Theory & Event 3:1, Muse//uwyo-ajl]

What emerges through the logic of the paradox of sovereignty is an event Agamben calls the zone of
indistinction. In the suspension of the rule through the state of exception, what we are presented with is a
complex plateau where such philosophically distinct categories as state of nature and law, outside and
inside, exception and rule flow through one another to the point of literal indistinction. On Agamben's
account, the operation of sovereignty abandons individuals whenever they are placed outside the law and
in so doing, exposes and threatens them to a sphere where there is no possibility of appeal. (Agamben, p.
29) What is crucial for Agamben's entire project, then, is to point out how the zone of indistinction
collapses the possibility of making distinctions - which is to say further, to point out how political
philosophy finds the limit of thinking in the paradox of sovereignty. In the sphere of indistinction, we
cannot think as if distinctions operated as they might in everyday life.6.

The political point here is, I think, insightful and worth pursuing. What makes this insight problematic,
however, is Agamben's treatment of history and the status of homo sacer therein. Part of the task of this
book is to ascertain how the category of homo sacer is a specifically historical category. This is evident in
Agamben's constant referral to ancient Roman legal documents as well as his exploration of the
reappearance of homo sacer throughout history. But it is precisely the possibility that homo sacer is
something that occurs 'throughout history' that makes Agamben's analysis at times difficult to swallow. At
the purely conceptual level, one might be willing to accept the meta claim that Agamben seems to be
making. But Agamben does not want to limit himself to the conceptual level. He wants to insist on the

Page 455 of 1481


material dimension of homo sacer and the actuality of this category in contemporary life. There is thus a
substantial tension between the particularity of homo sacer as a material instance of modern politics and
the trans-historical category of homo sacer as a category constituted by the paradox of sovereignty and
the state of indistinction.

Page 456 of 1481


Page 457 of 1481
#9 Essentialism: Ext
AGAMBEN CONFLATES DIFFERENT HISTORICAL PERIODS INTO A SINGULAR AND
STABLE TRANSHISTORICAL BIOPOLITICS THAT NEVER EXISTED, MEANING NONE OF
THEIR HISTORICAL IMPACTS APPLY

Wark 2004
[McKenzie, Re: <nettime> Agamben: No to Bio-Political Tattooing, posted to
nettime mailing list, January 27, amsterdam.nettime.org/Lists-Archives/nettime-l-
0401/msg00092.html, acc 1-7-2004//uwyo-ajl]

Eugene asks about Georgio Agamben. Below is a short note on him. I find his writings on the state les interesting and useful
On the
than his return to the question of commodity fetishism, which is a refreshing revisiting of a neglected concept.
state, his approach seems more philological than historical. By not bringing his thinking on the
commodity and on the state more closely together, one is not really given much of a handle on
how developments in the commodity form may have transformed the state. 'Biopower'
becomes a vague, transhistorical notion in Agamben. Agamben is one of the few contemporary thinkers to try
to think *past* Debord's Society of the Spectacle, which I think is still an untranscended horizon in its matching of political
and theoretical intransigence. And so in the note below I concentrate on his handling of Debord.

AND, NAZISM AND CONTEMPORARY DECENTRALIZED CONTROL FUNCTION


DIFFERENTLY

Neilson 2004
[Brett, University of Western Sydney, Potenza Nuda? Sovereignty, Biopolitics,
Capitalism, Contretemps 5, December 2004,
www.usyd.edu.au/contretemps/5december2004/neilson.pdf, acc 1-7-04//uwyo-ajl]

Negris ruse in this review is to suggest that the permanent state of exception specified by the first Agamben describes the
that it is inaccurate to fix
new condition of global Empire. But he counters Agamben on his own terms, charging
everything that happens in the world today onto a static and totalitarian horizon, as under
Nazism. Such an equation, for Negri, is anachronistic and inaccurate, since it conflates the
fascist rule of the twentieth century with contemporary modes of decentralized global control.
With implicit reference to the first chapter of Stato di Eccezione, where Agamben describes the current world situation as
global civil war (a term initially used by both Carl Schmitt and Hannah Arendt), Negri questions the notion of a sovereign
ban that renders constituent and constituted power indistinct:

But things are differentif we live in a state of exception it is because we live through a
ferocious and permanent civil war, where the positive and negative clash: their antagonistic power can
in no way be flattened onto indifference.18 There can be no doubt that Stato di Eccezione finds Agamben
writing of a positive counterpower that breaks the connection of violence to law posited by Schmitts exceptionalist model
of sovereignty. For Schmitt, the state of exception exists only as a means of maintaining and restoring the constituted
sovereign order. By contrast, Agamben follows the argument of Benjamins Critique of Violence, which posits a divine or
revolutionary violence that intercedes upon the struggle of constituent and constituted power, breaking the connection of
violence to law that, in the final instance, undergirds their interrelation. By opening the possibility of a power that operates

Page 458 of 1481


in complete independence from the law, Agamben claims, Benjamin specifies the nature of the violence that pertains in the
permanent state of exception. Furthermore, by virtue of the influence of his essay, Benjamin provokes the negative
reaction of Schmitt, whose entire political theory can be read as a fearful response to the prospect of an exception that
does not return to the norm. This is not to claim, however, that Stato di Eccezione affirms Negris equation of constituent
violence with living counterpower. Rather the Benjaminian violence celebrated by Agamben remains separate from the
whole complex of constituent and constituted power, both interceding upon them with an energy that makes the paradigm
of modern sovereignty obsolete and, in so doing, maintaining them in indistinction.

Page 459 of 1481


Page 460 of 1481
#10 Criticism Causes
Powerlessness: 1AR (1/2)
EXTEND 2AC NEILSON 2004 EV. GROUP IT.

FIRST, THE NEG POSITS BIOPOWER AS AN ALL ENCOMPASSING NEGATIVE


STRUCTURE THAT CO-OPTS ALL RESISTANCE, WHICH RENDERS US UNABLE TO
INTERVENE BECAUSE EVERY MOVE IS SHUT OFF IN ADVANCE, DOOMING US TO
ENDLESS ATROCITY. THE BETTER ALTERNATIVE IS TO USE BIOPOWER AGAINST
ITSELF. PURE DESIRE EXPLODES THE SYSTEMS COORDINATES, UNDERMINING ITS
FOUNDATIONS FROM WITHIN

SECOND, THIS TAKES OUT ALL OF THE INTERNALS TO THEIR OFFENSE BECAUSE THE
1AC USES A DIFFERENT KIND OF BIOPOWER THAN AGAMBEN IS CRITICIZING BY
APPROPRIATING IT AGAINST ITSELF, RATHER THAN USING IT TO EXCLUDE NON-
POLITICAL LIFE

THIRD, AGAMBENS MODEL OF BIOPOLITICS CREATES POWERLESSNESS,


SUBVERTING RESISTANCE

Hardt & Dumm 2000


[Michael & Thomas, Sovereignty, Multitudes, Absolute Democracy: A Discussion between Michael Hardt and Thomas
Dumm about Hardt and Negri's Empire, Theory & Event 4:3, Muse//uwyo-ajl]

But still none of that addresses the passivity you refer to. For that we have to look instead at Agamben's
notions of life and biopower. Agamben uses the term "naked life" to name that limit of humanity, the
bare minimum of existence that is exposed in the concentration camp. In the final analysis, he explains,
modern sovereignty rules over naked life and biopower is this power to rule over life itself. What results
from this analysis is not so much passivity, I would say, but powerlessness. There is no figure that can
challenge and contest sovereignty. Our critique of Agamben's (and also Foucault's) notion of biopower is
that it is conceived only from above and we attempt to formulate instead a notion of biopower from
below, that is, a power by which the multitude itself rules over life. (In this sense, the notion of biopower
one finds in some veins of ecofeminism such as the work of Vandana Shiva, although cast on a very
different register, is closer to our notion of a biopower from below.) What we are interested in finally is a
new biopolitics that reveals the struggles over forms of life.

Page 461 of 1481


Page 462 of 1481
#10 Criticism Causes
Powerlessness: 1AR (2/2)
FOURTH, AGAMBENS CONCEPTION OF POWER IS POLITICALLY DISABLING
BECAUSE IT REDUCES EVERY RESISTANCE TO AN ALL PERVASIVE POWER
STRUCTURE ONLY VIEWING IT AS AN EXPLOSION OF DESIRE ALLOWS US TO
SUBVERT THE SOVEREIGN BY ALLOWING BIOPOWERS OWN PRODUCTIVITY TO
DESTROY ITSELF

Neilson 2004
[Brett, University of Western Sydney, Potenza Nuda? Sovereignty, Biopolitics, Capitalism, Contretemps
5, December 2004, www.usyd.edu.au/contretemps/5december2004/neilson.pdf, acc 1-7-04//uwyo-ajl]

How then can Negri maintain that constituent power and sovereignty are opposites, separate even in the absoluteness to
which both lay claim? Already in Il potere constituente, three years before the publication of Homo Sacer, Negri fends off
the argument that reduces constituent power to an infinite void of possibilities or the presence of negative possibilities. For
him, the crucial question is the relation between potentiality (potenza) and power (potere). He recognizes

in the definition of potentiality that runs from Aristotle and the Renaissance and from Schelling to Nietzsche

a metaphysical alternative between absence and power, between desire and possession, between refusal
and domination.8

Far from opening a zone of indistinction, Negri believes this alternative to open a choice, at least
when it is not closed off by the dogma that reduces power to a pre-existing physical fact,
finalized order, or dialectical result. And the philosophical conduit of this opening is the great current of modern political
power as an overflowing
thought, from Machiavelli to Spinoza to Marx, which understands constituent
expression of desire, an absence of determinations, and a truly positive concept of freedom and
democracy.

For Negri, the


danger of Agambens thought lies not in its Aristotelian rigour or formal elegance but in its
inability to open a panorama of revolutionary struggle that can oppose the modern order of
sovereignty and the transcendental ideal of power that backs it up. As long as constituent power remains
caught in the paradox of sovereignty and the constituted order produces bare life as the limit
condition of an exception that has become the rule, there can be no hope of questioning the
transcendentalism of sovereign power or imagining a form of political conduct that remains
free of the impositions of the modern state. Thus it is the concept of bare life that becomes the primary object
of Negris critique of Agambens understanding of sovereignty. This much is clear in Empire, where Negri and his co-author
Michael Hardt distance themselves from the notion of bare life.

Page 463 of 1481


Page 464 of 1481
#10 Criticism Causes
Powerlessness: Ext (1/3)
CRITICISM OF BIOPOLITICS OBSCURES THE CONTROL OF LIFE, JUSTIFYING THE
STATUS QUO

Virno 2002
[Paolo, Paolo Virnos criticism of Agamben, www.generation-online.org/p/fpagamben1.htm, acc. 9-24-
06//uwyo-ajl]

when Agamben speaks of the biopolitical he


Agamben is a thinker of great value but also, in my opinion, a thinker with no political vocation. Then,

has the tendency to transform it into an ontological category with value already since the archaic Roman
right. And, in this, in my opinion, he is very wrong-headed. The problem is, I believe, that the biopolitical is only an effect derived from the
concept of labor-power. When there is a commodity that is called labor-power it is already implicitly
government over life. Agamben says, on the other hand, that labor-power is only one of the aspects of the biopolitical; I say the contrary: over all because labor
power is a paradoxical commodity, because it is not a real commodity like a book or a bottle of water, but rather is simply the potential to produce. As soon as this potential is
transformed into a commodity, then, it is necessary to govern the living body that maintains this potential, that contains this potential. Toni (Negri) and Michael (Hardt), on the
other hand, use biopolitics in a historically determined sense, basing it on Foucault, but Foucault spoke in few pages of the biopolitical - in relation to the birth of liberalism - that

biopolitical can be transformed


Foucault is not a sufficient base for founding a discourse over the biopolitical and my apprehension, my fear, is that the

into a word that hides, covers problems instead of being an instrument for confronting them. A fetish
word, an "open doors" word, a word with an exclamation point, a word that carries the risk of blocking critical thought instead of helping it.
Then, my fear is of fetish words in politics because it seems like the cries of a child that is afraid of the

dark..., the child that says "mama, mama!", "biopolitics, biopolitics!". I don't negate that there can be a serious content in the term, however I see
that the use of the term biopolitics sometimes is a consolatory use, like the cry of a child, when what serves us are, in all cases, instruments of work
and not propaganda words.

THEIR ALTERNATIVE ENSURES THE PERPETUAL REPLICATION OF SOVEREIGNTY


ONLY WORKING THROUGH THE SPECIFIC PRACTICES OF SOVEREIGNTY CAN
SUCCEED ATTEMPTS TO MOVE AWAY FROM IT OUTSIDE OF THE STATE
REPRODUCE SOVEREIGN POWER

Walker, Prof of International Relations @ Arizona State U, 2K2 (RBJ, Reframing the International, P.
3-5)

Page 465 of 1481


Page 466 of 1481
Page 467 of 1481
#10 Criticism Causes
Powerlessness: Ext (2/3)
AND, THE NEGATIVITY OF BARE LIFE NEUTRALIZES REVOLUTIONARY POTENTIAL
WHICH IS TOO DYNAMIC TO BE CONSTRAINED BY POWER, AS IS PROVEN BY
HISTORICAL STRUGGLES

Neilson 2004
[Brett, University of Western Sydney, Potenza Nuda? Sovereignty, Biopolitics, Capitalism, Contretemps
5, December 2004, www.usyd.edu.au/contretemps/5december2004/neilson.pdf, acc 1-7-04//uwyo-ajl]

In these articulations with Hardt, Negris disagreement with Agamben stems from

an equation of constituent power with living labour and a refusal to ground ontology

in the condition of bare life. If, in Empire, this quarrel with Agamben is relatively

marginal (confined to footnotes and passing comments), it assumes prominence in a

subsequent essay, Il mostro politico. Nuda vita e potenza. In this piece, which traces

the philosophical and historical consequences of eugenics (from classical Greece to

contemporary biotechnology), the concept of bare life is understood as an ideological

device for neutralizing the transgressive potentiality of human existence. Here Negris

criticism of Agamben is more rhetorical and direct:

Were the Vietnamese combatants or the blacks who revolted in the ghettos naked?

Were the workers or the students of the 1970s naked? It doesnt seem so if you look

at photos. At least if the Vietnamese werent denuded by napalm or the students

hadnt decided to give witness naked as a sign of their freedom.13

Human struggle, by this account, cannot be held ransom to the biopolitical machine that

produces bare life. Even in the case of the Nazi camps, Negri contends, it is mistaken to

equate bare life with powerlessness. The mussulmani (or denuded concentration camp

victims) of whom Agamben writes in Remnants of Auschwitz (1999) are humans before

they are naked. And to make bare life an absolute and assimilate it to the horrors of

Nazism is a ruse of ideology:

Life and death in the camps represents nothing more than life and death in the

campsan episode of the civil war of the twentieth century, a horrific spectacle of

the destiny of capitalism and the ideological masking of its will, of the capitalist

Page 468 of 1481


motive against every instance of liberty.14

For Negri, the concept of bare life denies the potentiality of being. Like Hobbess

Leviathan, which promotes a vision of life as subjugated and unable to resist, the theory of

bare life represents a kind of foundation myth for the capitalist state. It is a cry of weakness

that constructs the body as a negative limit and licenses a nihilistic view of history. More

pointedly, bare life is the opposite of Spinozan potential and corporeal joy.15 With this

statement, Negri reaches the nub of his disagreement with Agamben. As an alternative

to the Aristotelian notion of potentiality (as intrinsically and paradoxically connected to

the act), he poses the Spinozan vision of potentiality (potenza) as the unstoppable and

progressive expansion of desire (cupiditas). By this view, fully developed by Negri in

The Savage Anomaly, the construction of politics is a process of permanent innovation.

Desire is the determinant force of the constitution of the sociala creative project that

is continually reopened and defined as absolute in this reopening. At once conflictual

and constituent, desire in this analysis functions without lack and provides the basis for

an absolute democracy that reaches beyond modern political representation.

Page 469 of 1481


Page 470 of 1481
#10 Criticism Causes
Powerlessness: Ext (3/3)
EACH EXERCISE OF POWER IS CO-PRODUCTIVE WITH ITS OWN IMMANENT
RESISTANCE THAT USES IT AS ITS TARGET, ALLOWING BETTER SUBVERSION THAN
AN ISOLATED REJECTION FROM THE OUTSIDE

Foucault 78
[Michel, God, The History of Sexuality: An Introduction Volume I, trans. Robert
Hurley, New York City: Random House, Vintage Books Edition, 95-6//uwyo-ajl]

-Where there is power, there is resistance, and yet, or rather consequently, this resistance is
never in a position of exteriority in relation to power. Should it be said that one is always
"inside" power, there is no "escaping" it, there is no absolute outside where it is concerned,
because one is subject to the law in any case? Or that, history being the ruse of reason, power is
the ruse of history, always emerging the winner? This would be to misunderstand the strictly
relational character of power relationships. Their existence depends on a multiplicity of points
of resistance:

these play the role of adversary, target, support, or handle in power relations. These points of
resistance are present everywhere in the power network. Hence there is no single locus of great
Refusal, no soul of revolt, source of all rebellions, or pure law of the revolutionary. Instead there
is a plurality of resistances, each of them a special case: resistances that are possible, necessary,
improbable; others that are spontaneous, savage, solitary, concerted, ram-pant, or violent; still
others that are quick to compromise, interested, or sacrificial; by definition, they can only exist
in the strategic field of power relations. But this does not mean that they are only a reaction or
rebound, forming with respect to the basic domination an underside that is in the end always
passive, doomed to perpetual defeat. Resistances do not derive from a few heterogeneous
prin-ciples; but neither are they a lure or a promise that is of necessity betrayed. They are the
odd term in relations of power; they are inscribed in the latter as an irreducible opposite. Hence
they too are distributed in irregular fash-ion: the points, knots, or focuses of resistance are
spread over time and space at varying densities, at times mobiliz-ing groups or individuals in a
definitive way, inflaming certain points of the body, certain moments in life, certain types of
behavior. Are there no great radical ruptures, massive binary divisions, then? Occasionally, yes.
But more often one is dealing with mobile and transitory points of resistance, producing
cleavages in a society that shift about, fracturing unities and effecting regroupings, furrowing
across individuals themselves, cutting them up and remolding them, marking off irreducible
regions in them, in their bodies and minds. Just as the network of power relations ends by
forming a dense web that passes through apparatuses and institutions, without being exactly
localized in them, so too the swarm of points of resistance traverses social stratifications and
individual unities. And it is doubtless the strategic codification of these points of resistance that
makes a revolution possible, somewhat similar to the way in which the state relies on the
institutional integration of power relationships.

Page 471 of 1481


A2 Neilson Conclude Negative: 1AR

FIRST, NO HE DOESNT. HE ONLY SAYS THAT NEITHER AUTHOR TAKES THE OTHER
SERIOUSLY ON CERTAIN POINTS, WHICH IS NON-RESPONSIVE TO THE ARGUMENT
THAT WERE MAKING

SECOND, EVEN IF AGAMBEN AVOIDS OUR ARGUMENT, THE NEGATIVE CRITICISM


DOESNT BECAUSE IT STILL POSITS POWER AS BEING SO TOTAL THAT EVERY
ACTION GETS CO-OPTED, PREVENTING PRODUCTIVE RESISTANCE. CROSS-APPLY
NEILSON

Page 472 of 1481


Page 473 of 1481
#11 Agamben Misunderstands
Sovereignty: 1AR
THEIR PICTURE OF THE CAMP OBSCURES THE DAILY VIOLENCE OF SOVEREIGNTY

Hardt & Dumm 2000


[Michael & Thomas, Sovereignty, Multitudes, Absolute Democracy: A Discussion between Michael Hardt and Thomas
Dumm about Hardt and Negri's Empire, Theory & Event 4:3, Muse//uwyo-ajl]

TD: In that regard, my sense is that you both recognize the power of Giorgio Agamben's argument in
Homo Sacer concerning the extraordinary violence of sovereignty at the end of modernity and yet you
seek to overcome what may (not too unjustly) be thought of as a terrifying passivity that his position
could result in.14.

MH: Our argument in Empire does share some central concerns with Agamben's Homo Sacer, particularly
surrounding the notions of sovereignty and biopower. Agamben brilliantly elaborates a conception of
modern sovereignty based on Carl Schmitt's notions of the decision on the exception and the state of
emergency, in which the modern functioning of rule becomes a permanent state of exception. He then
links this conception to the figure of the banned or excluded person back as far as ancient Roman law
with his usual spectacular erudition. The pinnacle and full realization of modern sovereignty thus becomes
the Nazi concentration camp: the zone of exclusion and exception is the heart of modern sovereignty and
grounds the rule of law. My hesitation with this view is that by posing the extreme case of the
concentration camp as the heart of sovereignty it tends to obscure the daily violence of modern
sovereignty in all its forms. It implies, in other words, that if we could do away with the camp then all the
violence of sovereignty would also disappear.

BIOPOWER DOESNT EMERGE FROM THE SOVEREIGN, BUT FROM SOCIAL


RELATIONS THAT ARE BEYOND PLAN

Lazzarato no date
[Maurizio, From Biopower to Biopolitics, Trans. Ivan A. Ramirez,
www.goldsmiths.ac.uk/csisp/papers/lazzarato_biopolitics.pdf, acc 1-7-05//uwyo-ajl]

Foucault needs a new political theory and a new ontology to describe

the new power relations expressed in the political economy of forces. In

effect, biopolitics are grafted and anchored upon a multiplicity of

disciplinary [de commandemant et d'obissance] relations between forces,

those which power coordinates, institutionalizes, stratifies and targets, but

that are not purely and simply projected upon individuals. The fundamental

Page 474 of 1481


political problem of modernity is not that of a single source of sovereign

power, but that of a multitude of forces that act and react amongst each

other according to relations of command and obedience. The relations

between man and woman, master and student, doctor and patient, employer

and worker, that Foucault uses to illustrate the dynamics of the social body

are relations between forces that always involve a power relation. If power,

in keeping with this description, is constituted from below, then we need an

ascending analysis of the constitution of power dispositifs, one that begins

with infinitesimal mechanisms that are subsequently invested, colonized,

utilized, involuted, transformed and institutionalized by ever more general

mechanisms, and by forms of global domination.

Consequently, biopolitics is the strategic coordination of these power

relations in order to extract a surplus of power from living beings. Biopolitics

is a strategic relation; it is not the pure and simple capacity to legislate or

legitimize sovereignty. According to Foucault the biopolitical functions of

coordination and determination concede that biopower, from the moment

it begins to operate in this particular manner, is not the true source of

power. Biopower coordinates and targets a power that does not properly

belong to it, that comes from the outside. Biopower is always born of

something other than itself.

Page 475 of 1481


Page 476 of 1481
#11 Agamben Misunderstands
Sovereignty: Ext (1/2)
AGAMBEN IS WRONG. BIOPOWER IS DISPERSED THROUGH SOCIETY, MAKING
RESISTANCE POSSIBLE AND UNDERMINING SOVEREIGN POWER

Lazzarato no date
[Maurizio, From Biopower to Biopolitics, Trans. Ivan A. Ramirez,
www.goldsmiths.ac.uk/csisp/papers/lazzarato_biopolitics.pdf, acc 1-7-05//uwyo-ajl]

2. Giorgio Agamben , recently, in a book inscribed explicitly within the research being undertaken on the concept of biopolitics, insisted that the
theoretical and political distinction established in antiquity between zoe and bios, between natural life and political life, between man as a living being [simple
vivant] whose sphere of influence is in the home and man as a political subject whose sphere of influence is in the polis, is now nearly unknown to us. The
introduction of the zoe into the sphere of the polis is, for both Agamben and Foucault, the decisive event of modernity; it marks a radical transformation of the

is this impossibility of distinguishing between zoe and


political and philosophical categories of classical thought. But

bios, between man as a living being and man as a political subject, the product of the action of sovereign power or
the result of the action of new forces over which power has no control? Agambens
response is very ambiguous and it oscillates continuously between these two
alternatives. Foucaults response is entirely different: biopolitics is the form of
government taken by a new dynamic of forces that, in conjunction, express power
relations that the classical world could not have known. Foucault described this dynamic,
in keeping with the progress of his research, as the emergence of a multiple and heterogeneous power of

resistance and creation that calls every organization that is transcendental, and every regulatory
mechanism that is extraneous, to its constitution radically into question. The birth of biopower
and the redefinition of the problem of sovereignty are only comprehensible to us on this basis.
Foucaults entire work leads toward this conclusion even if he did not coherently explain the dynamic of this power,
founded on the freedom of subjects and their capacity to act upon the conduct of others, until the end of his life.

POWER ISNT STATE-CENTERED OR INSTITUTIONAL BUT RATHER, A MULTIPLICITY


OF DISPERSED SOCIAL FORCES

Foucault 78
[Michel, God, The History of Sexuality: An Introduction Volume I, trans. Robert
Hurley, New York City: Random House, Vintage Books Edition, 92-3//uwyo-ajl]

word
Hence the objective is to analyze a certain form of knowl-edge regarding sex, not in terms of repression or law, but in terms of power. But the

power is apt to lead to a number of misunderstandings-misunderstandings with re-spect to its nature, its form, and its
unity. By power, I do not mean "Power" as a group of institutions and mechanisms that ensure the

subservience of the citizens of a given state. By power, I do not mean, either, a mode of subjugation
which, in contrast to violence, has the form of the rule. Finally, I do not have in mind a general
system of domi-nation exerted by one group over another, a system whose effects, through successive derivations,
pervade the entire social body. The analysis, made in terms of power, must not assume that the sovereignty

of the state, the form of the law, or the over-all unity of a domination are given at the outset;
rather, these are only the terminal forms power takes. It seems to me that power must be understood

Page 477 of 1481


in the first instance as the multiplicity of force relations immanent in the sphere in which they
operate and which constitute their own organization; as the process which, through ceaseless strug-gles and confrontations, transforms, strengthens, or
reverses them; as the support which these force relations find in one another, thus forming a chain or a system, or on the con-trary, the disjunctions and
contradictions which isolate them from one another; and lastly, as the strategies in which they take effect, whose general design or institutional crystalliza-tion is

embodied in the state apparatus, in the formulation of the law, in the various social hegemonies . Power's condi-tion of possibility, or in
any case the viewpoint which permits one to understand its exercise, even in its more "peripheral" effects, and which also makes it possible to use its

must not be sought in the primary existence of a central


mech-anisms as a grid of intelligibility of the social order,

point, in a unique source of sovereignty from which secondary and de-scendent forms would
emanate; it is the moving substrate of force relations which, by virtue of their inequality,
constantly engender states of power, but the latter are always local and unstable. The omnipresence of power: not because it has the
privilege of consolidating everything under its invincible unity, but because it is produced from one moment to the next, at

every point, or rather in every relation from one point to another. Power is everywhere; not because it em-braces everything, but because it comes from
everywhere. and "Power," insofar as it is permanent, repetitious, inert, and self-reproducing, is simply the over-all effect that emerges from all these mobilities,

power is not an
the concatenation that I;ests on each of them and seeks in turn to arrest their move-ment. One needs to be nominalistic, 110 doubt:

institution, and not a structure; neither is it a certain strength we are endowed with; it is the name that one attrib-utes to a complex
strategical situation in a particular society.

Page 478 of 1481


Page 479 of 1481
#11 Agamben Misunderstands
Sovereignty: Ext (2/2)
BIOPOWER OCCURS IN THE SHIFT TO POPULAR ADMINISTRATION AND ISNT
LOCATED IN THE SOVEREIGN

Foucault 78
[Michel, God, The History of Sexuality: An Introduction Volume I, trans. Robert
Hurley, New York City: Random House, Vintage Books Edition, 135-7//uwyo-ajl]

For a long time, one of the characteristic privileges of sovereign power was the right to decide life and death. In a
formal sense, it derived no doubt from the ancient patria potestas that granted the father of the Roman family the right to
"dispose" of the life of his children and his slaves; just as he had given them life, so he could take it away. By the time the
right of life and death was framed by the classi-cal theoreticians, it was in a considerably diminished form. It was no longer
considered that this power of the sovereign over his subjects could be exercised in an absolute and un-conditional way, but
only in cases where the sovereign's very existence was in jeopardy: a sort of right of rejoinder. If he were threatened by
external enemies who sought to over-throw him or contest his rights, he could then legitimately wage war, and require his
subjects to take part in the defense of the state; without "directly proposing their death," he was empowered to "expose
their life": in this sense, he wielded an "indirect" power over them of life and death. I But if someone dared to rise up
against him and transgress his laws, then he could exercise a
direct power over the offender's life: as
punishment, the latter would be put to death. Viewed in this way, the power of life and death was
not an absolute privilege: it was conditioned by the defense of the sovereign, and his own survival.
Must we follow Hobbes in seeing it as the transfer to the prince of the natural right possessed by every individual to defend
his life even if this meant the death of others? Or should it be regarded as a specific right that was manifested with the
formation of that new juridical being, the sovereign?2 ln any case, in its modern form-relative and limited-as in its ancient
and absolute form, the right of life and death is a dlissymmetrical one.

The sovereigm exercised his right of life only by exercising his right to kill, or by refraining from killing; he
evidenced his power over life only through the death he was capable of requiring. The right which was formulated
as the "power of life and death" was in reality the right to take life or let live. Its symbol, after
all, was the sword. Perhaps this juridical form must be re-ferred to a historical type of society in
which Power was exercised mainly as a means of deduction (prelewement), a subtraction meclhanism, a
right to appropriate a portion of the wealth, a tax: of products, goods and services, labor and blood, levied on. the subjects.
Power in this instance was essentially a riglht of seizure: of things, time, bodies, and ultimately life itself; it culminated in
the privilege to seize hold of life in order to suppress it.

Since the classical age the West has undergome a very profound transformation of these
mechanisms of power. "Deduction" hasl tended to be no longer the major form of power but merelly one element
among others, wlorking to incite, reinforce, control" monitor, optimize, and organize the forces under it: a power bent
on generating forces, making them grow, and ordering them, rather than one Idedicated to
impeding them, making them submit, or destroying them. There has been a Parallel shift in the right of
death, (or at least a tendency to align itself with the exigencies of a life-adminis-tering power and to define itself
This death that was based on the right of the sovereign is now mamifested as simply
accordingly.
the reverse of the right of the social body to ensure, maintain, or deveIop its life. Yet wars were never as bloody
as they have been since the nineteenth century, and, all things being equal, never before did regimes visit such holocausts
on their own populations. But this formidable power of death -and this is perhaps what accounts for part of its
force and the cynicisom with which it has so greatly expanded its limits -now presents itself as the counterpart of
a power that exerts a positive influence on life, that endeawors to administer, optimize, and
multiply it, subjecting it to Iprecise controls and comprehensive regulations. Wars are no Ronger waged in the name of a
sovereign who must be defended; they are waged on 1behalf of the existence of everyone:; entire popula-tions are
mobilized for the purpose of wholes:ale slaughter in the name of life necessity: massacres have become vital. It is as
manage:rs of life and survival, of bodies amd the race, that so many regimes have been able to wage so many wars, causing
so' many men to be killed. And through a turn that closes the circle, as the technology of wars bias caused them to tend

Page 480 of 1481


increasingly toward all-out destruction, the decision that initiattes them and the one that terminaltes them are in fact
increa:singly informed by the naked questtion of survival. The atomilc situation is now at the end point of this process: the
power to expose a whole population to death is the underside of the power to guarantee an irudividual's con-tinued
existence. The principle underlying tbie tactics of bat-tle-that one has to be capable of killing in order to go on living-has
become the principle that defines the strategy of states. But the existence
in question is no longer the
juridical existence of sovereignty; at stake is the biological existence of a population. If genocide is
indeed the dream of modern powers, this is not because of a recent returm of the ancient right to kill; it is because power is
situated and exercised at the level of life, the species, the race, and the large-scale phenomema of population.

Page 481 of 1481


Page 482 of 1481
#13 Praxis: 1AR
PROTEST ISNT ENOUGH MUST LINK IT TO PRACTICE AND DEMANDS ON THE
STATE OR WE LAPSE INTO POLITICAL PARALYSIS IN THE FACE OF OPPRESSION

Foucault 82
[Michel, God, Politics and Ethics: An Interview, The Foucault Reader, Trans.
Catherine Porter, Ed. Paul Rabinow, 377//uwyo-ajl]

Q. And this is hard to situate within a struggle that is already under way, because the lines are
drawn by others. . . . M.F. Yes, but I think that ethics is a practice; ethos is a manner of being.
Let's take an example that touches us all, that of Poland. If we raise the question of Poland in
strictly political terms, it's clear that we quickly reach the point of saying that there's nothing
we can do. We can't dispatch a team of para- troopers, and we can't send armored cars to
liberate Warsaw. I think that, politically, we have to recognize this, but I think we also agree
that, for ethical reasons, we have to raise the problem of Poland in the form of a nonacceptance
of what is. happening there, and a nonacceptance of the passivity of our own governments. I
think this attitude is an ethical one, but it is also political; it does not consist in saying merely, "I
protest," but in making of that attitude a political phenomenon that is as substantial as possible,
and one which those who govern, here or there, will sooner or later be obliged to take into
account.

Page 483 of 1481


Page 484 of 1481
#14 Liberalism Doesnt Cause
Exception: 1AR
AGAMBEN HAS NO JUSTIFICATION FOR THE INDISTINCTION BETWEEN THE
FOUNDING AND CONTINUED OPERATION OF THE LAW

Deranty 2004
[Jean-Philippe, Macquarie University, Agambens challenge to normative theories of modern rights, borderlands e-journal, Vol. 3, No.
1, www.borderlandsejournal.adelaide.edu.au/vol3no1_2004/deranty_agambnschall.htm, acc 1-7-05//uwyo-ajl]

35. But this grounding in the political is just the result of a theoretical decision, and the alternatives should
Agamben
be confronted more explicitly. This lack of a substantial engagement with other legal alternatives becomes obvious a few pages later, when
analyses once more the specific problem of the application of the law. When he writes that "in the case of the juridical norm, the reference to the concrete case
supposes a "process" that always implies a plurality of subjects, and that culminates in the last instance in the enunciation of a sentence, that is to say, a

simply formulates a
statement whose operative reference to reality is guaranteed by institutional powers" (Agamben 2003: 69), he

classical distinction that can receive an entirely different treatment with no less
plausibility. A recent philosophical solution to the gap between justification and
application has been famously given by Habermas (1990 and 1996). Chapters 5 and 6 of Between facts and norms in particular
provide an excellent overview of plausible alternatives to Schmitts decisionistic theory of adjudication, from Kelsen to Critical Legal Studies.

Agamben cannot simply use the fact that "the application of a norm is not
36. But then

contained in it" as leading directly to the theory of the state of exception, since from the
very same premise another form of political grounding of the legal could be advanced,
one, for instance, that focuses on intersubjectivity and the institutionalisation of dissensus. The
"violence" that realizes the statement is not necessarily "without logos". For Schmitt, it draws its authority from the political, that is, the logos of the polis as

ethnos; for another tradition, it would do so from the logos of intersubjectively constituted and essentially contested institutions .

RIGHTS ONLY JUSTIFY EXCLUSION IF THEYRE ABSTRACT MODERNITY DISTILLS


THEM INTO UNIVERSAL CITIZENSHIP PREVENTING THE STATE OF EXCEPTION

Deranty 2004
[Jean-Philippe, Macquarie University, Agambens challenge to normative theories of modern rights, borderlands e-journal, Vol. 3, No.
1, www.borderlandsejournal.adelaide.edu.au/vol3no1_2004/deranty_agambnschall.htm, acc 1-7-05//uwyo-ajl]

17. Agamben quotes Arendts critical conclusion: the conception of human rights, based upon the assumed existence of a human being as such,
broke down at the very moment when those who professed to believe in it were for the first time confronted with people who had indeed lost all other qualities

fails to quote the very next


and specific relationships except that they were still human (Arendt 1966: 299; Agamben 1998: 126). But he

line, which makes all the difference: "The world found nothing sacred in the abstract
nakedness of the human being" (Arendt 1966: 299).

only when they are realised in a political "commonwealth" do human


18. What Arendt means is that

rights have any meaning. They are an abstraction otherwise. More important than the right to
freedom or the right to justice is "the right to have rights", that is, to be the member of a
political community. Arendt therefore asserts the opposite of what Agamben wants to
say: she believes that the political solution lies in what he considers to be a fiction, namely the citizen. Her point is that when

Page 485 of 1481


never really existed as a subject of rights. This is the exact
man and citizen come apart, we realise that man

opposite of Agamben for whom the citizen is just a travesty.


19. Despite this opposition, Agamben borrows Arendts critical interpretation of the French revolution and modernity in general, even though this interpretation

human rights lose all significance if they are not


itself is not beyond doubt. The French declaration makes it clear that

reinscribed within a political community that transforms them into constitutional


principles, and the American constitution also defines a clear link between individual freedom and a political order whose goal is freedoms protection.
Yet, Agamben reads the first article of the Declaration of 1789, "all men are born and remain free and equal in rights" as proof that modern sovereign power

applies to bare life, here in the form of birth (Agamben 1995: 128). But this seems disingenuous . Birth here refers not to nationality,
but simply to the fundamental fact of the equality of all human beings in right. The term
effectuates the radical break with ancient and absolutist natural law, a break that is synonymous with
legal modernity. In ancient natural law, rights were associated with the social position or the notion of a perfect cosmic order underpinned by God.

Page 486 of 1481


Page 487 of 1481
Agamben Collapses the State
AGAMBENS ALTERNATIVE MAKES NO SENSE ON A PUBLIC LEVEL THE NET
RESULT IS COMMUNITIES AT WAR WITH THE STATE WHICH WOULD COLLAPSE THE
STATE

Cmiel, Prof of Cultural History @ Iowa, 96 (Kenneth, The Fate of the Nation and the Withering of the State,
American Literary History, Spring, P. 196, http://alh.oxfordjournals.org/cgi/reprint/8/1/184)

If community cannot be a closed thing, if it is forever open to the potentially new, then the dream of a
national community is simply impossible. In Agamben's community, the idea of something being "un-
American" makes no sense, for there is no defining essence in a "whatever singularity." Yet Agamben is
also aware that capitalism and the state will continue. Indeed, he recognizes that after the fall of Communism, they are sweeping
the globe. Politics, in the future, Agamben argues, will not be community building but the perpetual project of communities against the state, "a struggle
between the State and the non-State (humanity), an insurmountable disjunction between whatever singularity and the State organization" (84). I
doubt
Agamben's new community is actually coming. It remains far from clear that communities without
identities are emerging anywhere except in the febrile imaginations of a few philosophers. It is not that I dislike
the dream. It is for me the most attractive dream there is. It is that I am skeptical that such "whatever singularities" are possible
on more than the level of personal behavior. Politics is too clunky for such subtlety. Even the new social
movements seem far more down-to-earth and prone to defining themselves than Agamben's theorizing.
Politics, alas, xdemands more leaden language. Still, the image of the state fighting communities is one worth pondering. Its distance from
earlier welfare state thinking could not be more dramatic. Instead of the state embodying the will of the nation, we have a
picture of numerous communities at war with the state. It is, and I say this with no relish, a far more plausible picture of our
emerging politics than Walzer's happy pluralism. Just think of insurance companies, Perotistas, and gay and lesbian
activistsall communities distrustful of the state, all committed to struggling with the state. Agamben does
not ask what this perpetual warfare will do to government. Like Walzer, he assumes that the state will trudge on as before. Yet if
this warfare between humanity and the state is constant, is it not plausible to surmise that hostility to the
state will become permanent? With the fiction that the state embodies the nation's will dying, who will
defend the state? Who will keep it from becoming the recipient of increasing rancor and from being
permanently wobbly? Isn't that a good way of understanding recent politics in the US? And as for Agamben's own
Italy the past decade has revealed a public far more disgusted with the state than even in America.

Page 488 of 1481


Page 489 of 1481
**Foucault**

Page 490 of 1481


Foucault Answers: 2AC (1/3)
FIRST, PLAN IS NECESSARY FOR THE ALTERNATIVE BECAUSE IT CHALLENGES A
MORE VIOLENT FORM OF UNILATERAL BIOPOWER. THIS CREATES A DOUBLE BIND:
EITHER THE END RESULT OF THE ALT IS PLAN AND THERES NO LINK DIFFERENTIAL
OR IT DOES THE STATUS QUO AND DOESNT SOLVE

SECOND, PERM: DO PLAN AND THE ALTERNATIVE OUR ADVOCACY IS THE FIRST
TEMPORARY EXPRESSION OF THE CRITIQUE ALTERNATIVE. REFORM IS NECESSARY
TO ENGAGE THE PUBLIC SPHERE

Foucault, French Sociologist, 1988


(Michel, On Criticism in Michel Foucault: Politics Philosophy Culture Interviews and other writings 1977-
1984)

D.E. You mean it will be possible to work with this government?

FOUCAULT: We must escape from the dilemma of being either for or against. After all, it is possible to face up to

a government and remain standing. To work with a government implies neither subjection nor total
acceptance. One may work with it and yet be restive. I even believe that the two things go together.
D.E. After Michel Foucault the critic, are we now going to see Michel Foucault the reformist? After all, the reproach was often made that the criticism made by intellectuals leads
to nothing.

FOUCAULT First Ill answer the point about that leads to nothing. There are hundreds and thousands of people who have worked for the emergence of a number of problems
that are now on the agenda. To say that this work produced nothing is quite wrong. Do you think that twenty years ago people were considering the problems of the relationship
between mental illness and psychological normality, the problem of prison, the problem of medical power, the problem of the relationship between the sexes, and so on, as they
are doing today?

Furthermore, there are no reforms as such. Reforms are not produced in the air, independently of those who carry them out. One cannot not take account of those who will
have the job of carrying out this transformation.

And, then, above all, I believe that an opposition can be made between critique and transformation, ideal critique and real transformation.

A critique is not a matter of saying that things are not right as they are. It is a matter of pointing out on what kinds of assumptions, what kinds of familiar, unchallenged, uncon-
sidered modes of thought the practices that we accept rest.

We must free ourselves from the sacrilization of the social as the only reality and stop regarding as superfluous something so essential in human life and in human relations as
thought. Thought exists independently of systems and structures of discourse. It is something that is often hidden, but which always animates everyday behavior. There is
always a little thought even in the most stupid institutions; there is always thought even in silent habits.

Criticism is a matter of flushing out that thought and trying to change it: to show that things are not as
self-evident as one believed, to see that what is accepted as self-evident will no longer be accepted as such. Practicing criticism is a matter of
making facile gestures difficult.
In these circumstances, criticism (and radical criticism) is absolutely indispensable for any transformation. A transformation that remains within the same mode of thought, a
transformation that is only a way of adjusting the same thought more closely to the reality of things can merely be a superficial transformation.

as soon as one can no longer think things as one formerly thought them, transformation
On the other hand,

becomes both very urgent, very difficult, and quite possible.


It is not therefore a question of there being a time for criticism and a time for transformation, nor people who do the criticism and others who do the transforming, those who are

the work of deep


enclosed in an inaccessible radicalism and those who are forced to make the necessary concessions to reality. In fact I think

transformation can only be carried out in a free atmosphere, one constantly agitated by a permanent
criticism.
D.E. But do you think the intellectual must have a programmatic role in this transformation?

FOUCAULT A reform is never only the result of a process in which there is conflict, confrontation, struggle, resistance

Page 491 of 1481


To say to oneself at the outset: what reform will I be able to carry out? That is not, I believe, an aim for the intellectual to pursue. His role, since he works specifically in the
realm of thought, is to see how far the liberation of thought can make those transformations urgent enough for people to want to carry them out and difficult enough to carry out
for them to be profoundly rooted in reality.

It is a question of making conflicts more visible, of making them more essential than mere confrontations
of interests or mere institutional immobility. Out of these conflicts, these confrontations, a new power relation must emerge,
whose first, temporary expression will be a reform. If at the base there has not been the work of thought upon itself and if, in fact, modes of
thought, that is to say modes of action, have not been altered, whatever the project for reform, we know that it will be swamped, digested by modes of behavior and institutions
that will always be the same.

THIRD, NO LINK PLAN DOESNT EXERCISE POWER OVER THE BODIES AT


GUANTANAMO. IT ONLY OVERRULES ONE ASPECT OF DETAINMENT

Page 492 of 1481


Page 493 of 1481
Foucault Answers: 2AC (2/3)
FOURTH, NO IMPACT FOUCAULT DOESNT SAY THAT BIOPOWER IS NECESSARILY
BAD, BUT THAT ITS DANGEROUS. PLAN IS AN INSANTIATION OF POWER
CREATING ITS OWN RESISTENCE, CHALLENGING VIOLENCE

FIFTH, DEMANDS ON THE STATE ARE MORE EFFECTIVE THAN RADICAL REJECTION
THEIR ALTERNATIVES FEAR OF COOPTION PARALYZES POLITICAL PRAXIS ONLY
THROUGH THE DEMANDS OF THE PLAN CAN WE CHANGE THE SYSTEM

Zizek, Senior Researcher @ Libjulian, Slovenia, 98

The dialectical tension between the vulnerability and invulnerability of the system also enables us to denounce the ultimate racist and/or sexist trick, that of
'two birds in the bush instead of a bird in hand": when women demand dimple equality, quasi -"feminists" often pretend to offer them "much more" (the role of
the warm and wise "conscience of society/'elevated above the vulgar everyday competition and struggle for domination...)- the only proper answer to this offer,
of course. Is "no, thanks! Better is the enemy of the good! We do not want more, just equality!" Here, at least, the last lines in Now Voyager ("why reach for the
moon. When we. Can have the stars?") Are wrong. It is homologous with the Native American who wants to become integrated into the predominant "white"
society, and a politically correct progressive liberal endeavors to convince him that he is thereby renouncing his very unique prerogative, the authentic native
culture and tradition- no thanks, simple equality is enough, I also wouldn't mind my part of consumerist alienation! ... A
modest demand of the
excluded group for the full participation at the society's universal rights is much more threatening for the system
than the apparently much more "radical" rejection of the predominant social values" and the assertion of the
superiority of one's own culture. For a true feminist, Otto Weininger's assertion that, although women are "ontologically false." lacking the proper ethical
stature, they should be acknowledged the same rights as men in public life, is infinitely more acceptable than the false elevation of' women that makes them
is not that every opposition, every attemot at
'too good" for the banality of men's rights. Finally, the point about inherent transgression
subversion is automatically "co-opted." On the contrary, the very fear of being co-opted that makes us search
for more and more radical, "pure" attitudes, is the supreme strategy of suspension or marginalization. The point is
rather that true subversion is not always where it seems to be sometimes. A small distance is much more explosive for the system
that an ineffective radical rejection. In religion. A small heresy can be more threatening than an outright
atheism or passage to another religion; for a hardline Stalinist, a Trotskyite is infinitely more threatening
than a bourgeois liberal or social democrat. As Le Carre put it, one true revisionist in the Central Committees is
worth more than thousand dissidents outside it. It was easy to dismiss Gorbachev for a imin g only at
improving the system, making it more efficient - he nonetheless set in motion its disintegration. So one should also
bear in mind the obverse of the inherent transgression: one is tempted to paraphrase Freuds claim from the Ego and the Id that man is not only much more
immoral than he believes, but also much more moral than he knows - the
system is not only infinitely more resistant and invulnerable than it may
infinitely more vulnerable (a small revision
appear (it can co-opt apparently subversive strategies, they can serve as its support), it is also
etc. Can have large unforeseen catastrophic consequences).

SIXTH, FOUCAULDIAN CRITIQUE DENIES AGENCY BY IGNORING ANY SOCIAL


JUSTICE OR USEFUL HUMAN ACTION

Anthony Cook, Associate Professor at Georgetown Law, NEW ENGLAND LAW REVIEW, Spring, 1992

Unless we are to be trapped in this Foucaultian moment of postmodern insularity, we must resist the temptation to sever description from explanation. Instead, our objective

should be to explain what we describe in light of a vision embracing values that we make explicit in struggle. These values should act as magnets that

Page 494 of 1481


link our particularized struggles to other struggles and more global critiques of power. In other words, we must
not, as Foucault seems all too willing to do, forsake the possibility of more universal narratives that, while tempered by postmodern
insights, attempt to say and do something about the oppressive world in which we live. Second, Foucault's emphasis

on the techniques and discourses of knowledge that constitute the human subject often diminishes, if not
abrogates, the role of human agency. Agency is of tremendous importance in any theory of oppression,
because individuals are not simply constituted by systems of knowledge but also constitute hegemonic
and counter-hegemonic systems of knowledge as well. Critical theory must pay attention to the ways in
which oppressed people not only are victimized by ideologies of oppression but the ways they craft from
these ideologies and discourses counter-hegemonic weapons of liberation.

Page 495 of 1481


Page 496 of 1481
Foucault Answers: 2AC (3/3)
SEVENTH, NO ALTERNATIVE FOUCAULDIAN POWER IS SO ALL ENCOMPASSING
THAT NO BREAK FROM CO-OPTATION IS POSSIBLE

EIGHTH, FOUCAULT MISUNDERSTANDS POWER LIBERAL SOCIETY IS


SUBSTANTIVELY DIFFERENT FROM INTERNMENT

Walzer, Professor of Social Sciences at the Institute for Advanced Studies & Former Professor at Harvard, 1983
(Michael, The Politics of Michel Foucault, Dissent, Fall)

For it is Foucault's claim, and I think he is partly right, that the discipline of a prison, say, represents a
continuation and intensification of what goes on in more ordinary places-and wouldn't be possible if it didn't.
So we all live to a time schedule, get up to an alarm, work to a rigid routine, live in the eye of authority, are
periodically subject to examination and inspection. No one is entirely free from these new forms of social
control. It has to be added, however, that subjection to these new forms is not the same thing as being in
prison: Foucault tends systematically to underestimate the difference, and this criticism, which I shall want
to develop, goes to the heart of his politics.

NINTH, THEIR TOTALIZING CRITICISM OF POWER PREVENTS REFORMWE MUST


USE THE STATE FOR INCREMENTAL ENDS.

James D. Faubian, professor of anthro @ Rice University, Michel Foucault: Power, Essential Works
of Foucault 1954-1984 Volume 3, 1994, p. xxxi-xxxii

Foucault wanted, then, to move both the descriptive and prescriptive functions of political analysis away
from the juridico-discursive language of legitimation. To try to put the matter as simply as possible: he
does not think that all power is evil or all government unacceptable, but does think that theorems
claiming to confer legitimacy on power or government are fictions; in a lecture of 1979, he expresses
sympathy with the view of earlier political skeptics that civil society is a bluff and the social contract a
fairy tale. This does not mean that the subject matter of political philosophy is evacuated, for doctrines
of legitimation have been and may still act as political forces in history. But his analytic quarrel with
legitimation theory is that it can divert us from considering the terms in which modern government
confers rationality, and thus possible acceptability, on its activity and practice. This is the main reason why
he argues political analysis is still immature, having still not cut off the kings head.1o The deployment and
application of law is, for Foucault, like everything else, not good or evil in itself, capable of acting in the
framework of liberalism as an instrument for economizing and moderating the interventions of
governmental power, necessary as an indispensable restraint on power in some contexts, uses, and
guises; it is to be resisted as an encroaching menace in others. In his governmentality lectures, Foucault
investigates the evolution, from the era of the police states through the development of parliamentary
liberal government, of the ambiguous and dangerous hybridization of law with a rationality of security
and with new theories of social solidarity and social defense. This historical analysis and diagnosis informs

Page 497 of 1481


Foucaults commentary on the civil liberties politics of seventies France, with its distinctive contemporary
recrudescence of raison detat and the police state. But at the same time, in a way we tend not to think of
as typically French, he dryly mocked and debunked the excesses of what he called state phobiathe
image of the contemporary state as an agency of essential evil and limitless despotism. The state, he said,
does not have a unitary essence or indeed the importance commonly ascribed to it: what are important to
study are the multiple governmental practices that are exercised through its institutions and elsewhere.
(In a lecture describing the seventeenth-century theory of raison detat, Foucault characterized it as a
doctrine of the permanent coup detata piquant choice of phrase, because it had been the title of a
polemical book written against de Gaulle by Francois Mitterrand. We know that Foucault did not share
the view, common in the French Left, of de Gaulles government as an antidemocratic putsch with crypto-
fascistic tendencies. The Left, he also suggested, should expect to win elected power not by demonizing
the state (never a very convincing platform for a socialist party) but by showing it possessed its own
conception of how to govern.

Page 498 of 1481


Page 499 of 1481
#2 Perm: 1AR
PERM SOLVES BEST - MICROPOLITICS AND LARGER STRUGGLES AGAINST
OPPRESSION SHOULD BE COMBINED, CREATING A RADICAL REFORMISM IN
OPPOSITION TO TOTALIZING POLITICS

May 93
[Todd, Between Genealogy and Epistemology: Psychology, Politics, and Knowledge in the Thought of
Michel Foucault, Pennsylvania: The Pennsylvania State University Press, 1993, 118//wfi-ajl]

The risk of a totalizing theory of politics is that it will unsuspectingly promote what it struggles
against, because it is ignorant of oppressions at the micropolitical level. The alternative to this,
though, is not a bourgeois reformism but what one critic has called a "radical reformism"
(Gandal 1986, p. 122). This radical reformism recognizes both that a change of power which
comes solely at the top hazards a repetition of the old forms of domination and that not just any
small reform will change micropolitical domination. Instead, what the radical reformist seeks
are changes at the micropolitical level which actually change the relations of power between
groups. Those changes involve very different types of struggle, depending upon the situation of
the groups involved. They cannot be cast in a common form or be reduced to a common goal.
But they possess a solidarity that derives from a complementarity investing all struggles against
domination under capitalism. I , Micropolitical struggles do not replace the struggle against
exploitation, and no one of them can be substituted for the others. What binds them is the
recognition that in the modern epoch power operates in many and diffuse ways, and that to
end the domination of such power is a matter of many independent but mutually reinforcing
struggles both at the micropolitical and the macropoliticallevel. And thus, there is a need for the
kinds of analyses which are situated not in the region of general political theory, but in the
domains of struggles which occur both beneath and across that region. "I am attempting. . .
apart from any totalization-which would be at once abstract and limiting-to open up problems
that are as concrete and general as possible, problems that approach politics from behind and
cut across societies on the diagonal, problems that are at once constituents of our history and
constituted by that history" (Foucault 1984b, pp. 375-76).

Page 500 of 1481


Juxtaposition Solves: 1AR (1/2)

WE SHOULD JUXTAPOSE FOUCAULDIAN CRITICISM IN OPPOSITION TO THE IDEAS


HE CRITICIZES

Cook 92
[Anthony E., prof at Georgetown School of Law, New England Law Review, 1992, LN//wfi-ajl]

Thus, Foucault has prompted an entirely different approach to social criticism. Rejecting
modernist attempts to develop master narratives in the fashion of Hegel, Marx, and Kant,
Foucault instructs us to "develop action, thought, and desires by proliferation, juxtaposition,
and disjunction, and to prefer what is positive and multiple, difference over uniformity, flows
over unities, mobile arrangements over systems." n14 "Believe," he advises us, "that what is
productive is not sedentary but nomadic." n15

JUXTAPOSITION OF INCOMPATIBLE IDEAS AVOIDS THE PROBLEMS OF


TRADITIONAL THEORY AND ENABLES A PROCESS OF CONSTANT CRITICISM

Marcus '98
[George E., Professor of Anthro at Rice University, Ethnography through Thick and
Thin, Princeton: Princeton University Press, 1998, 186-7//uwyo-ajl]

The postmodern notions of heterotopia (Foucault), juxtapositions, and the blocking together
of incommensurables (Lyotard) have served to renew the long-neglected practice of
comparison in anthropology, but in altered ways. Juxtapositions do not have the obvious
meta-logic of older styles of comparison in anthropology (e.g., controlled comparisons
within a cultural area or "natural" geographical region); rather, they emerge from putting
questions to an emergent object of study whose controus are not known beforehand, but
are themselves a contribution of making an account which has different, complexly
connected real-world sites of investigation. The postmodern object of study is ultimately
mobile and multiply situated, so any ethnography of such an object will have a
comparative dimension that is integral to it, in the form of juxtapositions of seeming
incommensurables or phenomena that might conventionally have appeared to be "world
apart." Comparison reenters the very act of ethnographic specificity by a postmodern
vision of seemingly improbably juxtapositions, the global collapsed into and made and
integral part of a parallel, related local situations rather than something monolithic and
external to them. This move toward comparison as heterotopia firmly deterritorializes culture
in ethnographic writing and simulates accounts of cultures composed in a landscape for which
there is as yet no developed theoretical comparison

Page 501 of 1481


Page 502 of 1481
Juxtaposition Solves: 1AR (2/2)
JUXTAPOSING FOUCAULDIAN ACCOUNTS OF POWER WITH TRADITIONAL
SOVEREIGN MODELS EXPOSES DISCIPLINARY RELATIONS

Boyle 97
[James, Prof. Law at Washington College of Law, Foucault in Cyberspace:
Surveillance, Sovereignty, and Hardwired Censors, University of Cincinatti Law
Review, Fall, LN//wfi-ajl]

From the point of view of this Article, one of Foucault's most interesting contributions was to
challenge a particular notion of power, power-as-sovereignty, and to juxtapose against it a
vision of "surveillance" and "discipline." n21 At the heart of this project was a belief that both
our analyses of the operation of political power and our strategies for its restraint or limitation
were inaccurate or misguided. In a series of essays and books Foucault argued that, rather than
the public and formal triangle of sovereign, citizen, and right, we should focus on a series of
subtler private, informal, and material forms of coercion organized around the concepts of
surveillance and discipline. The paradigm for the idea of surveillance was the Panopticon,
Bentham's plan for a prison constructed in the shape of a wheel around the hub of an observing
warden. At any moment the warden might have the prisoner under observation through a
nineteenth century version of the closed-circuit TV. n22 Unsure when authority might in fact be
watching, the prisoner would strive always to conform his behavior to its presumed desires.
Bentham had hit upon a behavioralist equivalent of the superego, formed from uncertainty
about when one was being observed by the powers that be. The echo of contemporary laments
about the "privacy-free state" is striking. To this, Foucault added the notion of discipline-crudely
put, the multitudinous private methods of regulation of individual behavior ranging from
workplace time-and-motion efficiency directives to psychiatric evaluation. n23

Page 503 of 1481


Page 504 of 1481
#5 Demands on the State Good:
1AR (1/4)
OUR DEMAND TURNS THE TABLES ON THE BIOPOLITICAL APPARATUS. WE UTILIZE
THE TENSION BETWEEN FREEDOM AND CONTROL TO ARTICULATE A SERIES OF
DEMANDS WHICH ARE A STRATEGIC REVERSAL OF POWER RELATIONS

Campbell, Prof of IR @ Newcastle U, 98 (David, Writing Security, September 1, P. 203-5)

The answer to that question is an unequivocal yes. I suggested above in a tentative way how we might think differently about some issues pertinent to United
Were those possibilities explored, the boundaries of American identity and the realm of
States Foreign Policy.
the political would be very different from that which currently predominates, for the distinction
between what counts as normal and what is thus pathological would have been refigured. Besides, the
evident differences in emergent discourse of danger demonstrates how even those articulations with the most affinity do not mechanically reproduce a
monolithic identity. Of course, the pursuit of new possibilities through different interpretations is often strongly contested. Even recommendations to redirect
political practices so as to confront new challenges sometimes do not escape old logic. For example, the effort to address environmental issues within the
parameters of international relations and nation security often involves simply extending the old registry of security to cover his new domain. Usually signified
by the appropriation of the metaphor of war to a new problem, this is evident in some of the literature that advocates the importance of global cooperation
and management to counter environmental degradation, where ecological danger often replaces fading military threats as the basis of an interpretation
designed to sustain sovereignty. 35 Yet as I noted in Chapter 7, environmental danger can also be figured in a manner that challenges traditional forms of
As a danger that can be articulated in terms of security strategies that are de-
American and western identity.
territorialized, involve communal cooperation, and refigure economic relationships, the environment can
serve to enframe a different rendering of the political. Recognizing the possibility of rearticulating danger leads us to a final
question: what modes of being and forms of life could we or should we adopt? To be sure, a comprehensive attempt to answer such a question is beyond the
ambit of this book. But it is important to note that asking the question in this way mistakenly implies that such possibilities exist only in the future. Indeed,
the extensive and intensive nature of the relations of power associated with the society of security means
that there has been and remains a not inconsiderable freedom to explore alternative possibilities. While
traditional analyses of power are often economistic and negative, Foucaults understanding of power
emphasizes its productive and enabling nature. 36 Even more important, his understanding of power
emphasizes the ontology of freedom presupposed by the existence of disciplinary and normalizing
practices. Put simply, there cannot be relations of power unless subjects are in the first instance free: the need to institute negative and constraining
power practices comes about only because without them freedom would abound. Were there no possibility of freedom, subjects would not act in a way that
because it is only through
required containment so as to effect order. 37 Freedom, though, is not the absence of power. ON the contrary,
power that subjects exercise their agency, freedom and power cannot be separated. As Foucault maintains: At the
very heart of the power relationship, and constantly provoking it, are the recalcitrance of the will and the intransigence of freedom. Rather than speaking of an
essential freedom, it would be better to speak of an agonism of a relationship which is at the same time reciprocal incitation and struggle: less of a face-to-
face confrontation which paralyzes both sides than a permanent provocation. 38 The political possibilities enable by permanent provocation of power and
freedom can be specified in more detail by thinking in terms of the predominance of the bio-power discussed above. In this sense, because the governmental
practices of biopolitics in western nations have been increasingly directed towards modes of being and forms of life such that sexual conduct has become an
object of concern, individual health has been figured as a domain of discipline, and the family has been transformed into an instrument of government the
ongoing agonism between those practices and the freedom of the counter demands drawn from those new fields of concern. For example, as the state
continues to prosecute people according to sexual orientation, human rights activist have proclaimed the right of gays to enter into formal marriages, adopt
children, and receive the same health and insurance benefits granted to their straight counterparts. These
claims are a consequence of the
permanent provocation of power and freedom in biopolitics, and stand as testament to the strategic
reversibility of power relations: if the terms of governmental practices can be made into focal points for
resistances, then the history of government as the conduct of conduct is interwoven with the history of
dissenting counter-conducts Indeed, the emergence of the state as the major articulation of the political has involved an unceasing agonism
between those in office and those they rule. State intervention in everyday life has long incited popular collective action, the result of which has been both
resistance to the state and new claims upon the state. In particular, the core of what we now call citizenship consists of multiple bargains hammered out by
rulers and ruled in the course of there struggle over means of state action, especially in the making of war. In more recent times, constituencies associated
with womens, youth, ecological, and peace movements (among others) have also issued claims on society. These resistances are evidence that the break with
the discursive / non discursive dichotomy central to the logic of interpretation underlining this analysis is (to put in conventional terms) not only theoretically
licensed; it is empirically warranted.. Indeed, expanding the interpretive imagination so as to enlarge the categories through which we understand the
constitution of the political has been a necessary precondition for making sense of Foreign Policys concern for the ethical borders of identity in America.
Accordingly, there are manifest political implications that flow from theorizing identity. As Judith Butler concluded: The deconstruction of identity is not the
deconstruction of politics; rather it establishes as political the very terms through which identity is articulated.

Page 505 of 1481


Page 506 of 1481
Page 507 of 1481
#5 Demands on the State Good:
1AR (2/4)
FOUCAULT'S MODEL OF POWER DOOMS EVERY RESISTANCE TO INEVITABLE CO-
OPTATION BECAUSE OF A LACK OF SUBJECTIVITY ANTAGONISM EXCEEDS ITS
POSITIVE HISTORICAL ANTECEDENTS AND CAN BREAK THE POWER CYCLE BY
USING THE EDIFICE'S EXCESS AGAINST ITSELF

Zizek '99
[Slavoj, Senior Researcher at Institute for Social Studies, Ljubliana and Badass, The
Ticklish Subject: the absent centre of political ontology, New York: Verso, 1999, 256-
7//uwyo-ajl]

Against Butler, one is thus tempted to emphasize that Hegel was well aware of the retroactive process by means of which
oppressive power itself generates the form of resistance is not this very paradox contained in Hegel's
notion of positing the presuppositions, that is, of how the activity of positing-mediating does not merely elaborate the
presupposed immediate-natural Ground, but thoroughly transforms the very core of its identity? The very In-itself to which
Chechens endeavour to return is already mediated-posited by the process of modernization, which deprived them of their
ethnic roots.

This argumentation may appear Eurocentrist, condemning the colonized to repeat the European
imperialist pattern by means of the very gesture of resisting it however, it is also possible to give it
precisely the opposite reading. That is to say: if we ground our resistance to imperialist Eurocentrism in the
reference to some kernel of previous ethnic identity, we automatically adopt the position of a
victim resisting modernization, of a passive object on which imperialist procedures work. If, however, we conceive
our resistance as an excess that results from the way brutal imperialist intervention disturbed our previous self-
enclosed identity, our position becomes much stronger, since we can claim that our resistance is
grounded in the inherent dynamics of the imperialist system that the imperialist system itself,
through its inherent antagonism, activates the forces that will bring about its demise. (The
situation here is strictly homologous to that of how to ground feminine resistance: if woman is 'a symptom of man', the
locus at which the inherent antagonisms of the patriarchal symbolic order emerge, this in no way constrains the scope of
feminine resistance but provides it with an even stronger detonating force.) Or to put it in yet another way the
premise according to which resistance to power is inherent and immanent to the power edifice
(in the sense that it is generated by the inherent dynamic of the power edifice) in no way obliges us to draw the
conclusion that every resistance is co-opted in advance, including in the eternal game Power plays with
itself the key point is that through the effect of proliferation, of producing an excess of resistance, the very inherent
antagonism of a system may well set in motion a process which leads to its own ultimate
downfall.

It seems that such a notion of antagonism is what Foucault lacks: from the fact that every resistance is
generated ('posited') by the Power edifice itself, from this absolute inherence of resistance to Power, he seems to draw the
conclusion that resistance is co-opted in advance, that it cannot seriously undermine the system that is, he precludes
the possibility that the system itself, on account of its inherent inconsistency, may give birth to a
force whose excess it is no longer able to master and which thus detonates its unity, its capacity to
reproduce itself. In short, Foucault does not consider the possibility of an effect escaping, outgrowing its cause, so that
although it emerges as a form of resistance to power and is as such absolutely inherent to it, it
can outgrow and explode it. (the philosophical point to be made here is that this is the fundamental feature of the
dialectical-materialist notion of 'effect': the effect can 'outdo' its cause; it can be ontologically 'higher' than its cause.) One

Page 508 of 1481


is thus tempted to reverse the Foucauldian notion of an all-encompassing power edifice which always-already contains its
transgression, that which allegedly eludes it: what if the price to be paid is that the power mechanism cannot even control
itself, but has to rely on an obscene protuberance at its very heart? In other words: what effectively
eludes the
controlling grasp of Power is not so much the external In-itself it tries to dominate but, rather,
the obscene supplement which sustains its own operation.

And this is why Foucault lacks the appropriate


notion of the subject: the subject is by definition in
excess over its cause, and as such it emerges with the reversal of the repression of sexuality into
the sexualization of the repressive measures themselves. This insufficiency of Foucault's theoretical edifice
can be discerned in the way, in his early History of Madness, he is already oscillating between two radically opposed views:
the view that madness is not simply a phenomenon that exists in itself and is only secondarily the object of discourses, but
is itself the product of a multitude of (medical, legal, biological...) discourses about itself; and the opposite view, according
to which one should 'liberate' madness from the hold exerted over it by these discourses , and 'let madness itself speak'.

Page 509 of 1481


Page 510 of 1481
#5 Demands on the State Good:
1AR (3/4)
FOUCAULT IGNORES THE WAY THAT THE POWER EDIFICE IS SPLIT FROM WITHIN
AND HOW THE ITS DISAVOWED FOUNDATION CAN UNDERMINE IT

Zizek '97
[Slavoj, The Game, The Plague Fantasies, NYC: Verso, 1997, 26-7//uwyo-ajl]

We are now in a position to specify the distinction between the Foucauldian interconnection
between Power and resistance, and our notion of `inherent transgression'. Let us begin via the
matrix of the possible relations between Law and its transgression. The most elementary is the
simple relation of externality, of external opposition, in which transgression is directly opposed
to legal Power, and poses a threat to it. The next step is to claim that transgression hinges on
the obstacle it violates: without Law there is no transgression; transgression needs an obstacle
in order to assert itself. Foucault, of course, in Volume I of The History of Sexuality, rejects both
these versions, and asserts the absolute immanence of resistance to Power. However, the point
of `inherent transgression' is not only that resistance is immanent to Power, that power and
counter-power generate each other; it is not only that Power itself generates the excess of
resistance which it can no longer dominate; it is also not only that - in the case of sexuality - the
disciplinary `repression' of a libidinal investment eroticizes this gesture of repression itself, as in
the case of the obsessional neurotic who derives libidinal satisfaction from the very compulsive
rituals destined to keep the traumatic jouissance at bay.

This last point must be further radicalized: the power edifice itself is split from within: in order
to reproduce itself and contain its Other, it has to rely on an inherent excess which grounds it -
to put it in the Hegelian terms of speculative identity, Power is always-already its own
transgression, if it is to function, it has to rely on a kind of obscene supplement. It is therefore
not enough to assert, in a Foucauldian way, that power is inextricably linked to counter-power,
generating it and being itself conditioned by it: in a self-reflective way, the split is always-
already mirrored back into the power edifice itself, splitting it from within, so that the gesture of
self-censorship is consubstantial with the exercise of power. Furthermore, it is not enough to
say that the `repression' of some libidinal content retroactively eroticizes the very gesture of
`repression' - this `eroticization' of power is not a secondary effect of its exertion on its object
but its very disavowed foundation, its `constitutive crime', its founding gesture which has to
remain invisible if power is to function normally. What we get in the kind of military drill
depicted in the first part of Full Metal Jacket, for example, is not a secondary eroticization of the
disciplinary procedure which creates military subjects, but the constitutive obscene supplement
of this procedure which renders it operative. Judith Butler27 provides a perfect example of,
again, Jesse Helms who, in his very formulation of the text of the anti-pornography law~
displays the contours of a particular fantasy - an older man who engages in sadomasochistic
sexual activity with another, younger man, preferably a child - which bears witness to his own
perverted sexual desire. Helms thus unwittingly brings to light the obscene libidinal foundation
of his own crusade against pornography.

Page 511 of 1481


Page 512 of 1481
#5 Demands on the State Good:
1AR (4/4)
THE INNER LAW OF THE SUBJECT EMERGES FROM THE FAILURE OF THE EXTERNAL
LAW, ALLOWING THE SUBJECT TO DISRUPT DISCIPLINARY POWER

Zizek '99
[Slavoj, Senior Researcher at Institute for Social Studies, Ljubliana and Badass, The
Ticklish Subject: the absent centre of political ontology, New York: Verso, 1999, 279-
80//uwyo-ajl]

Butler's elaboration of the logic of melancholic identification with the lost object in fact provides
a theoretical model which allows us to avoid the ill-fated notion of the 'internalization' of
externally imposed social forms: what this simplistic notion of 'internalization' misses is the
reflexive turn by means of which, in the emergence of the subject, external power (the pressure
it exerts on the subject) is not simply internalized but vanishes, is lost: and this loss is
internalized in the guise of the 'voice of conscience', the internalization which gives birth to the
internal space itself:

In the absence of explicit regulation, the subject emerges as one for whom power has become
voice, and voice, the regulatory instrument of the psyche . . . the subject is produced,
paradoxically, through this withdrawal of power, its dissimulation and fabulation of the psyche
as a speaking topos.

This reversal is embodied in Kant, the philosopher of moral autonomy, who identifies this
autonomy with a certain mode of subjection, namely, the subjection to even the humiliation in
the face of the universal moral Law. The key point here is to bear in mind the tension between
the two forms of this Law: far from being a mere extension or internalization of the external
law, the inner Law (Call of Conscience) emerges when the external law fails to appear, in order
to compensate for its absence. In this perspective, liberation from the external pressure of
norms embodied in one's social conditioning (in the Enlightenment vein) is strictly identical to
submission to the unconditional inner Call of Conscience. That is to say: the opposition between
external social regulations and internal moral Law is that between reality and the Real: social
regulations can still be justifted (or pretend to be justified) by objective requirements of social
coexistence (they belong to the domain of the 'reality principle'); while the demand of the moral
Law is unconditional, brooking no excuse 'You can, because you must!', as Kant put it. For that
reason, social regulations make peaceful coexistence possible, while moral Law is a traumatic
injunction that disrupts it.. One is thus tempted to go a step further and to invert once more the
relationship between 'external' social norms and the inner moral Law: what if the subject
invents external social norms precisely in order to escape the unbearable pressure of the moral
Law? Isn't it much easier to have an external Master who can be duped, towards whom one can
maintain a minimal distance and private space, than to have an ex-timate Master, a stranger, a
foreign body in the very heart of one's being? Doesn't the minimal definition of Power (the
agency experienced by the subject as the force that exerts its pressure on him from the Outside,
opposing his inclinations, thwarting his goals) rely precisely on this externalization of the
extimate inherent compulsion of the Law, of that which is 'in you more than yourself? This
tension between external norms and the inner Law, which can also give rise to subversive

Page 513 of 1481


effects (say, of opposing public authority on behalf of one's inner moral stance), is neglected by
Foucault.

Page 514 of 1481


#6 Nihilism (Cook): 1AR (1/2)

THEY ARE IN A DOUBLE BIND EITHER FOUCAULT IS A NIHILIST OR THE


ALTERNATIVE DOESNT SOLVE

Hicks, Prof and Chair of Philosophy at Queens College of the CUNY, 2K3 (Steven V., Nietzsche, Heidegger, and
Foucault: Nihilism and Beyond, Foucault and Heidegger: Critical Encounters, Ed. Alan Milchman and Alan Rosenberg, p.
109, Questia)

Here Foucault seems less interested in defining a purpose for incitation and struggle than underscoring its potential creativity: bringing into the struggle as much gaiety, lucidity and

Given his belief that even our modern discourses of liberation, rights, and humanism are
determination as possible. 76

all deeply entangled in the inarticulable and inescapable background web of power practices, Foucault's
only option to passive nihilism seems to be the perpetuation and amelioration of the conditions that
make struggle itself possible 77 And this political task of promoting the pathos of struggle functions as an
alternative to the ascetic ideal: creating and maintaining many sites of resistance to the numerous
forms of domination, exploitation, and subjectification present in the social and political body . 78 Admittedly,
the pathos of struggle has a strong (and from a Nietzschean perspective, a possibly suspect) negative component: struggling
against any system of constraints or technologies of power that prevent individuals (affected by the
systems) from having the possibility of altering them or the means of modifying them. 79 As an ethico-political
ideal, the pathos of struggle would call for the negation of all political, social, and cultural conditions that

preclude the possibility of struggling to change these conditions. As Foucault writes, perhaps one must not be for consensuality, but one
must be against nonconsensuality. 80 But it would also contain an affirmative component as well, a struggle for something: Minimally, it will be a struggle for the establishing of conditions
in which self-creation is made possible, in which the assertion of individuality and otherness is viable. 81 As with Nietzsche's alternative ideals (of recurrence and will to power), the final
trajectory of the pathos of struggle remains undetermined. It can't tell us beforehand what our goals should be, only that (a) the conditions of their conception and articulation must remain
polymorphous and unhierarchical, and that (b) whatever they are, they should remain rooted in gratitude and service to life a joyful creative, and self-constituting engagement
rather than resentment against it. 82 But as with Nietzsche's nonascetic ideals, the pathos of struggle might also supply some affirmative content as well: the doing of what is necessary to
affirm your creative freedom and enhance the ongoing process of self-definition and social definition (within the constraints of not excluding or disempowering the viable other). For
example, overcome the oppression of your present situation if it prevents you from getting a sufficient sense of power and effectiveness in relation to life except by devaluing life. 83 In a

we might view Foucault as


manner somewhat reminiscent of Schiller's attempt to instill an aesthetic education in humanity to promote political freedom,

attempting to instill an agonistic education a will to struggle within an overarching aesthetics of


lifeto prepare the ground for, and manifest, our creative freedom. 84 According to Foucault, glimpses of freedom
and creation of the self as a work of art are prompted by continuous acts of resistance and political
struggle that serve to loosen the hold of those vast matrices of disciplinary power and technologies of the
body that threaten to overwhelm and homogenize us (cf. HS, 2,:io-n). 85 As Foucault sees it, then, a will to struggle,
an aesthetic agonism, becomes the defining characteristic and alternate (nonascetic) ideal that allows us to
best live out our unresolved existencesurrounded by ubiquitous, inescapable power arrangements
and tottering on the abyss of nihilism.

FOUCAULT IS FASCINATING, AND IRRELEVANT TO PUBLIC POLICY

David E. McClean, New School University, The Cultural Left and the Limits of Social Hope, Presented at the 2001 Annual Conference of the Society for the Advancement of American
Philosophy, www.american-philosophy.org/archives/2001%20Conference/Discussion%20papers/david_mcclean.htm.

Or we might take Foucault who, at best, has provided us with what may reasonably be described as a very
long and eccentric footnote to Nietzsche (I have once been accused, by a Foucaltian true believer, of "gelding" Foucault with other similar remarks).
Foucault, who has provided the Left of the late 1960s through the present with such notions as "governmentality," "Limit," "archeology," "discourse" "power" and "ethics," creating or
redefining their meanings, has made it overabundantly clear that all of our moralities and practices are the

successors of previous ones which derive from certain configurations of savoir and connaisance arising
from or created by, respectively, the discourses of the various scientific schools. But I have not yet found
in anything Foucault wrote or said how such observations may be translated into a political movement or

Page 515 of 1481


hammered into a political document or theory (let alone public policies) that can be justified or founded
on more than an arbitrary aesthetic experimentalism. In fact, Foucault would have shuddered if any one
ever did, since he thought that anything as grand as a movement went far beyond what he thought
appropriate. This leads me to mildly rehabilitate Habermas, for at least he has been useful in exposing
Foucault's shortcomings in this regard, just as he has been useful in exposing the shortcomings of others enamored with the abstractions of various Marxian-
Freudian social critiques.

Page 516 of 1481


Page 517 of 1481
#6 Nihilism (Cook): 1AR (2/2)
RESISTANCE DOESNT REQUIRE REJECTION OF DISCIPLINARY PRACTICES, ONLY
THEIR INTERROGATON

May 93
[Todd, Between Genealogy and Epistemology: Psychology, Politics, and Knowledge in
the Thought of Michel Foucault, Pennsylvania: The Pennsylvania State University
Press, 1993, 125//wfi-ajl]

Resistance in contemporary society does not require the complete abandonment of psychology.
What it does require is an understanding of the ways in which psychology has contributed to
our present, particularly the dangers it poses and the damages it has fostered in that present. It
is indeed important for us to get free of psychology. But to get free of psychology is not
necessarily to abandon it. It is to understand its hold on us, theoretically and practically, and to
be able to make choices about what place, if any, we want it to have in our future. If Foucault's
last works on Greek and Roman sexuality were not written in order to offer concrete
alternatives to contemporary methods of self-formation, neither is the idea of experimentation
which motivated them an implicit advocacy of the complete abandonment of psychology. They
are an attempt to understand who we are and what our present is like, by reference to histories
of practices rather than to the unfolding of truths or falsehoods.

REJECTING DISCIPLINE CREATES NEW FORMS OF UTOPIAN DOMINATION ONLY


ANALYZING HOW POWER CONSTITUTES KNOWLEDGE ALLOWS RESISTANCE

Cook 92
[Anthony, Associate Professor of Law @ Georgetown, Hangs out with Gingrich, New
England Law Review, LN//wfi]

Third, Foucault's intervention at these localized sites of domination is not a mere seizing of
power that replaces one utopian vision with another that is likely to be as dominating as its
predecessor when based on the same techniques and knowledge systems embedded in the
displaced system. Instead, Foucault's intervention has a theoretical dimension that is of primary
importance. He

wants first and foremost to challenge the specific ways in which knowledge is produced and
constituted. That is, he wants to explore the ways in which we are socialized into seeing the
world and its possibilities in a certain way and dismissing other visions as "unreasonable" or
"impossible." We must understand the extent to which we all carry around in our heads fascist,
[*759] racist, homophobic, and sexist constructs that are produced and reproduced by received
discourses of knowledge that are inextricably connected to the exercise of power and

Page 518 of 1481


domination of certain groups. When this is realized, the possibility of building around rather
than on these constructs is enhanced. All of this, I believe, is good.

Page 519 of 1481


#10 Reformism Good: 1AR

FOUCAULT IGNORES JURIDICAL POWER AS A KEY SOURCE OF VIOLENCE FOR THE


CONSTITUTIONAL STATE. WE CAN STRATEGICALLY REFORM THE LAW AND USE
THE EXTENSION OF RIGHTS TO HEDGE AGAINST POWER FOUCAULT HIMSELF
WAS ENGAGED IN THESE VERY SAME POLITICAL LIKE THE AFF

Habermas, Permanent Visiting Prof @ Northwestern U, 87 (Jrgen, The Philosophical Discourse of


Modernity, P. 289-291)

Foucault begins by analyzing the normative language game of rational natural law in connection with the latent functions that the discourse on authority has in
the age of Classicism for the establishment and the exercise of absolutist state power. The sovereignty of the state that has a monopoly on violence is also
expressed in the demonstrative forms of punishment that Foucault depicts in connection with the procedures of torture and ordeal. From the same
functionalist perspective, he then describes the advances made by the Classical language game during the reform era of the Enlightenment. They culminate, on
the one hand, in the Kantian theory of morality and law and, on the other hand, in utilitarianism. Interestingly enough, Foucault docs not go into the fact that
these in turn serve the revolutionary establishment of a constitutionalized slate power, which is to say, of a political order transferred ideologically from the
sovereignty of the prince to the sovereignty of the people. This kind of regime is, after all, correlated with those normalizing forms of punishment that
Because Foucault filters out the internal aspects of the development
constitute the proper theme of Discipline and Punish.
of law, he can inconspicuously take a third and decisive step: Whereas the sovereign power of Classical
formations of power is constituted in concepts of right and law, this normative language game is
supposed to be inapplicable to the disciplinary power of the modern age; the latter is suited only to
empirical, at least nonjuridical, concepts having to do with the factual steering and organization of the behavioral
modes and the motives of a population rendered increasingly manipulable by science: "The procedures of normalization
come to be ever more constantly engaged in the colonization of those of the law. I believe that all this can explain the global functioning of what I would call a
the complex life-context of
society of normalization."33 As the transition from doctrines of natural law to those of natural societies shows,34
modern societies as a whole can as a matter of fact be less and less construed in the natural-law
categories of contractual relationships. However, this circumstance cannot justify the strategic decision (so
full of consequences for Foucault's theory) to neglect the development of normative structures in connection with the

modern formation of power. As soon as Foucault takes up the threads of the biopolitical establishment of disciplinary power, he lets
drop the threads of the legal organization of the exercise of power and of the legitimation of the order of
domination. Because of this, the ungrounded impression arises that the bourgeois constitutional state is a
dysfunctional relic from the period of absolutism. This uncircumspect leveling of culture and politics to immediate substrates of the
application of violence explains the ostensible gaps in his presentation. That his history of modern penal justice is detached from
the development of the constitutional state might be defended on methodological grounds. The
theoretical narrowing down to the system of carrying out punishment is more questionable. As soon as he
passes from the Classical to the modern age, Foucault pays no attention whatsoever to penal law and to the law
governing penal process. Otherwise, he would have had to submit the unmistakable gains in liberality and
legal security, and the expansion of civil-rights guarantees even in this area, to an exact interpretation in
terms of the theory of power. However, his presentation is utterly distorted by the fact that he also filters out of the history of penal
practices itself all aspects of legal regulation. In prisons, indeed, just as in clinics, schools, and military
installations, there do exist those "special power relationships" that have by no means remained
undisturbed by an energetically advancing enactment of legal rights Foucault himself has been politi-
cally engaged for this cause. This selectivity does not take anything away, from the importance of his fascinating unmasking of the capillary
effects of power. But his generalization, in terms of the theory of power, of such a selective reading hinders Foucault from
perceiving the phenomenon actually in need of explanation: In the welfare-state democracies of the West, the
spread of legal regulation has the structure dilemma, because it is the legal means for securing freedom that themselves endanger
the freedom of their presumptive beneficiaries. Under the premises of his theory of power, Foucault so levels down the complexity of
societal modernization that the disturbing paradoxes of this process cannot even become apparent to
him.

Page 520 of 1481


Page 521 of 1481
Alt Fails: Body Cannot Be a Site
of Resistance
FOUCAULT PLACES AGENCY WITHIN THE BODY WHICH OFFERS LITTLE CHANCE FOR
RESISTANCE.

Kenneth Rufo, Rhetoric and Power: Rethinking and Re-linking, ARGUMENTATION AND ADVOCACY v. 40 n.
2, Fall 2003, ASP.

The grounds on which Foucault believed such a liberation to be possible are problematic given that power
is always already a relational domination. Therein lies his emancipatory failure; as Murphy (1995, p. 7)
notes: "The oxymoron of an 'active subject' has been the Achilles' heel of any project, such as critical
rhetoric, influenced by Foucault." If all are produced as subjects, then to whom and from whom can we
speak? Certainly, Foucault's body is not the only agency within the "body of discourse" or the "body
politic." The placement of agency within the body offers little chance of resistance, for even the body is
constituted within the discursive realm. As Kevin Olson (1996, p. 32) explains, speaking of punk rockers'
attempt to cast off the social norm:

We cannot claim ... that such power is mobilized from the body or that the source of resistance arises
from some innate corporeal rebellion. It is difficult to imagine a property of the body that could constitute
an alternative to the structuring force of power, since there is no sense in which we can say that bodies
are 'elastic' or that they can resist the impression of power ... the idea of resistance to the effects of
power is incoherent in the terms Foucault uses to discuss it. His analysis fails to explain how the body can
resist power or take on a structure not completely determined by disciplinary regimes.

Page 522 of 1481


Alt Fails: Cannot Escape Subjectivity

YOUR METHODOLOGY IS BANKRUPT BECAUSE IT STILL PRIVILEGES THE NOTION OF


A SUBJECT AS A UNITARY ACTOR

Jon Simons, professor of political philosophy and feminist theory @ the Hebrew University of Jerusalem,
FOUCAULT AND THE POLITICAL, 1995, p. 25-26

Moreover, Foucaults analysis of the systematic arrangement of the elements of discourse3 leads him to
conclude that the figure of Man was the effect of a change in the fundamental arrangements of
knowledge. The existence of Man is contingent on the rules of regulation and systematic relations that
constitute the modern episteme. Humanism presupposes the existence of Man, who for Foucault is a
figure of discourse which appeared only at the end of the eighteenth century. The startling implication of
this is that [i]f those arrangements were to disappear .. . then one can certainly wager that Man would be
erased, like a face drawn in sand at the edge of the sea (1973b: 387). Indeed, Foucault suggests that the
modern episteme is coming to an end, having exhausted the possible constellations of theory available
between the three sets of doubles (l972a: 70). Humanism is a failed philosophical project because it takes
Man to be its foundation for knowledge, whereas he is one of its effects. Foucault not only declares the
demise of the modern episteme but aims to contribute to it. What Foucault was trying to achieve in his
archaeological discourse was his (in)famous decentring that leaves no privilege to any centre, especially
the subject (1972a: 205). Foucault argues that Man, the subject or the author cannot be considered as the
foundation, origin or condition of possibility of discourse. Rather, the subject, and especially the author,
can be defined as an element within a discursive field, a particular space from which it is possible to speak
or write and which must be filled if the discourse is to exist (1972a: 956). For example, the subject of a
discourse such as medicine is a function of legal rights, criteria of competence, institutional relations and
professional hierarchy. Doctors can only operate as the subjects of medical discourse if they speak from
the correct institutional sites: the hospital, laboratory, the professional journal. They also have different
roles depending on the object of discourse they speak about, sometimes observing, sometimes
questioning, listening or seeing, which also vary with the institutional site they are in. Since, in relation to
medical discourse, we find a variety of subject roles in different positions, it is concluded that discourse is
not the majestically unfolding manifestation of a thinking, knowing, speaking subject, but ... a totality, in
which the dispersion of the subject and his discontinuity with himself may be determined (1972a: 545).
Discourses of knowledge should not be analysed as unities by reference to psychological individuality or
to the opinions of a particular person (63, 70).

Page 523 of 1481


Alt Fails: Geneologies Dont Produce Change

GENEALOGIES, ALTHOUGH INTERESTING, DONT GENERATE POLITICAL CHANGE


THEY JUST LEAD US DOWN AN ENDLESS PATH OF QUESTIONS

Michel Foucault, SOCIETY MUST BE DEFENDED: LECTURES AT THE COLLEGE DE FRANCE 1975-1976,
2003, p. 3-4.

So what was I going to say to you this year? That Ive just about had enough; in other words, Id like to
bring to a close, to put an end to, up to a point, the series of research projectswell, yes, researchwe
all talk about it, but what does it actually mean?that weve been working on for four or five years, or
practically ever since Ive been here, and I realize that there were more and more drawbacks, for both you
and me. Lines of research that were very closely interrelated but that never added up to a coherent body
of work, that had no continuity. Fragments of research, none of which was completed, and none of which
was followed through; bits and pieces of research, and at the same time it was getting very repetitive,
always falling into the same rut, the same themes, the same concepts. A few remarks on the history of
penal procedure; a few chapters on the evolution, the institutionalization of psychiatry in the nineteenth
century; considerations on sophistry or Greek coins; an outline history of sexuality, or at least a history of
knowledge about sexuality based upon seventeenth-century confessional practices, or controls on
infantile sexuality in the eighteenth and nineteenth centuries; pinpointing the genesis of a theory and
knowledge of anomalies, and of all the related techniques. We are making no progress, and its all leading
nowhere. Its all repetitive, and it doesnt add up. Basically, we keep saying the same thing, and there
again, perhaps were not saying anything at all. Its all getting into something of an inextricable tangle,
and its getting us nowhere, as they say. I could tell you that these things were trails to be followed, that it
didnt matter where they led, or even that the one thing that did matter was that they didnt lead
anywhere, or at least not in some predetermined direction. I could say they were like an outline for
something. Its up to you to go on with them or to go off on a tangent; and its up to me to pursue them
or give them a different configuration. And then, weyou or Icould see what could be done with these
fragments. I felt a bit like a sperm whale that breaks the surface of the water, makes a little splash, and
lets you believe, makes you believe, or want to believe, that down there where it cant be seen, down
there where it is neither seen nor monitored by anyone, it is following a deep, coherent, and
premeditated trajectory.

Page 524 of 1481


Alt Fails: Remains Enmeshed in Power

EVEN IN SELF-EXAMINATION, WE ARE STILL ENSNARED BY THE WEB OF


CONSTITUTIVE POWER

Jon Simons, professor of political philosophy and feminist theory @ the Hebrew University of Jerusalem,
FOUCAULT AND THE POLITICAL, 1995, p. 36

Analysis at self-formation contributes to a broader social critique. Modern subjection of the insane took
the form of an ethical self-recognition. In Tukes asylum, the inmates were made to feel guilty for the
negligence which led to their loss of reason. They became aware of themselves as guilty, as objects of
punishment and therapy and as unequal to their keepers, who had not exceeded their liberty but
submitted it to the reason of morality and reality. It was through awareness of themselves as objects that
the mad were restored to awareness of themselves as responsible subjects, capable of restraining their
own behaviour rather than being restrained by the paternal authority of the asylum. The asylum. . .
organized. . . guilt. . . for the madman as a consciousness of himself (1965: 24750). On a grander scale,
the definition of European Man, identified with his reason, can be drawn by its opposition to the
experience of madness, now understood as mental illness. That form of human self-recognition and type
of subjecting thought puts in question . . . the limits rather than the identity of a culture (xiii). We are
limited to the identities in which we recognize ourselves as ethical as well as scientific beings.

Page 525 of 1481


Alt Fails: Praxis

THEORY IS IRRELEVENT ABSENT SPECIFIC APPLICATION MUST COMBINE THEORY


AND PRACTICE FOR A PHILOSOPHY AS LIFE

Foucault 82
[Michel, Politics and Ethics: An Interview, The Foucault Reader, Trans. Catherine
Porter, Ed. Paul Rabinow, 373-4//wfi-ajl]

Q. There is much talk in America these days comparing your work to that of Jurgen Habermas. It
has been suggested that your work is more concerned with ethics and his with politics.
Habermas, for example, grew up reading Heidegger as a politically disastrous heir of Nietzsche.
He associates Heidegger with German neo-conservatism. He thinks of these people as the
conservative heirs of Nietzsche and of you as the anarchistic heir. You don't read the
philosophical tradition this way at all, do you?

M.F. That's right. When Habermas was in Paris, we talked at some length, and in fact I was quite
struck by his observation of the extent to which the problem of Heidegger and of the political
implications of Heidegger's thought was quite a pressing and important one for him. One thing
he said to me has left me musing, and it's something I'd like to mull over further. After
explaining how Heidegger's thought indeed constituted a political disaster, he mentioned one of
his professors who was a great Kantian, very well-known in the '30s, and he explained how
astonished and disappointed he had been when, while looking through card catalogues one day,
he found some texts from around 1934 by this illustrious Kantian that were thoroughly Nazi in
orientation.

I have just recently had the same experience with Max Pohlenz, who heralded the universal
values of Stoicism all his life. I came across a text of his from 1934 devoted to Fiihrertum in
Stoicism. You should reread the introductory page and the book's closing remarks on the
Fuhrersideal and on the true humanism constituted by the Volk under the inspiration of the
leader's direction-Heidegger never wrote anything more disturbing. Nothing in this condemns
Stoicism or Kantianism, needless to say.

But I think that we must reckon with several facts: there is a very tenuous "analytic" link
between a philosophical conception and the concrete political attitude of someone who is
appealing to it; the "best" theories do not constitute a very effective protection against
disastrous political choices; certain great themes such as "humanism" can be used to any end
whatever-for example, to show with what gratitude Pohlenz would have greeted Hitler.

I do not conclude from this that one may say just anything within the order of theory, but, on
the contrary, that a demanding, prudent, "experimental" attitude is necesary; at every moment,
step by step, one must confront what one is thinking and saying with what one is doing, with
what one is. I have never been too concerned about people who say: "You are bor-rowing ideas
from Nietzsche; well, Nietzsche was used by the Nazis, therefore. . ."; but, on the other hand, I
have always been concerned with linking together as tightly as possible the historical and
theoretical analysis of power relations, institu-tions, and knowledge, to the movements,
critiques, and experiences that call them into question in reality. If I have insisted on all this
"practice," it has not been in order to "apply" ideas, but in order to put them to the test and
modify them. The key to the Personal poetic attitude of a philosopher is not to be sought in his
ideas, as if it could be deduced from them, but rather in his philosophy-as-life, in his
philosophicallife, his ethos.

Page 526 of 1481


Among the French philosophers who participated in the Resistance during the war, one was
Cavailles, a historian of mathematics who was interested in the development of its internal
structures. None of the philosophers of engagement-Sartre, Simone de Beauvoir, Merleau-
Ponty-none of them did a thing.

Page 527 of 1481


Page 528 of 1481
Alt Fails: Praxis
GIVING UP ON RESISTANCE THROUGH AGENCY ALLOWS OPPRESSION TO REMAIN
DOMINANT ONLY THE PERM SOLVES

Cook 92
[Anthony, Associate Professor of Law @ Georgetown, Hangs out with Gingrich, New
England Law Review, LN//wfi]

Several things trouble me about Foucault's approach. First, he nurtures in many ways an
unhealthy insularity that fails to connect localized struggle to other localized struggles and to
modes of oppression like classism, racism, sexism, and homophobia that transcend their
localized articulation within this particular law school, that particular law firm, within this
particular church or that particular factory.

I note among some followers of Foucault an unhealthy propensity to rely on rich, thick,
ethnographic type descriptions of power relations playing themselves out in these localized
laboratories of social conflict. This reliance on detailed description and its concomitant
deemphasis of explanation begins, ironically, to look like a regressive positivism which purports
to sever the descriptive from the normative, the is from the ought and law from morality and
politics.

Unless we are to be trapped in this Foucaultian moment of postmodern insularity, we must


resist the temptation to sever description from explanation. Instead, our objective should be to
explain what we describe in light of a vision embracing values that we make explicit in struggle.
These values should act as magnets that link our particularized struggles to other struggles and
more global critiques of power. In other words, we must not, as Foucault seems all too willing to
do, forsake the possibility of more universal narratives that, while tempered by postmodern
insights, attempt to say and do something about the oppressive world in which we live.

Second, Foucault's emphasis on the techniques and discourses of knowledge that constitute the
human subject often diminishes, if not abrogates, the role of human agency. Agency is of
tremendous importance in any theory of oppression, because individuals are not simply
constituted by systems of knowledge but also constitute hegemonic and counter-hegemonic
systems of knowledge as well. Critical theory must pay attention to the ways in which oppressed
people not only are victimized by ideologies of oppression but the ways they craft from these
ideologies and discourses counter-hegemonic weapons of liberation.

PROTEST ISNT ENOUGH MUST LINK IT TO PRACTICE AND DEMANDS ON THE


STATE OR WE LAPSE INTO POLITICAL PARALYSIS IN THE FACE OF OPPRESSION

Foucault 82
[Michel, God, Politics and Ethics: An Interview, The Foucault Reader, Trans.
Catherine Porter, Ed. Paul Rabinow, 377//wfi-ajl]

Page 529 of 1481


Q. And this is hard to situate within a struggle that is already under way, because the lines are
drawn by others. . . .

M.F. Yes, but I think that ethics is a practice; ethos is a manner of being. Let's take an example
that touches us all, that of Poland. If we raise the question of Poland in strictly political terms,
it's clear that we quickly reach the point of saying that there's nothing we can do. We can't
dispatch a team of para- troopers, and we can't send armored cars to liberate Warsaw. I think
that, politically, we have to recognize this, but I think we also agree that, for ethical reasons, we
have to raise the problem of Poland in the form of a nonacceptance of what is. happening
there, and a nonacceptance of the passivity of our own governments. I think this attitude is an
ethical one, but it is also political; it does not consist in saying merely, "I protest," but in making
of that attitude a political phenomenon that is as substantial as possible, and one which those
who govern, here or there, will sooner or later be obliged to take into account.

Page 530 of 1481


Alt Fails: Suspicion

ALTS SUSPICION FORECLOSES UPON PRODUCTIVE ACTION

James D. Faubian, Professor, Anthropology, Rice University, MICHEL FOUCAULT: POWER, ESSENTIAL
WORKS OF FOUCAULT 1954-1984 Volume 3, 1994, p. xviii-xix

One of the key clarifying points Foucault makes is that what is most interesting about links between
power and knowledge is not the detection of false or spurious knowledge at work in human affairs but,
rather, the role of knowledges that are valued and effective because of their reliable instrumental
efficacy. Foucault often uses the French word savoira term for knowledge with connotations of know-
how (a way to make a problem tractable or a material manageable)for this middle sort of knowledges,
which may fall short of rigorous scientificity but command some degree of ratification within a social
group and confer some recognized instrumental benefit. The reason the combining of power and
knowledge in society is a redoubtable thing is not that power is apt to promote and exploit spurious
knowledges (as the Marxist theory of ideology has argued) but, rather, that the rational exercise of power
tends to make the fullest use of knowledges capable of the maximum instrumental efficacy. What is
wrong or alarming about the use of power is not, for Foucault, primarily or especially the fact that a
wrong or false knowledge is being used. Conversely, power and the use of knowledge by power are not
guaranteed to be safe, legitimate, or salutory because (as an optimistic rationalist tradition extending
from the Enlightenment to Marxism has inclined some to hope) the knowledge that guides or
instrumentalizes the exercise of power is valid and scientific. Nothing, including the exercise of power, is
evil in itselfbut everything is dangerous. To be able to detect and diagnose real dangers, we need to
avoid equally the twin seductions of paranoia and universal suspicion, on the one hand, and the
compulsive quest for foundationalist certainties and guarantees, on the otherboth of which serve to
impede or dispense us from the rational and responsible work of careful and specific investigation.

Page 531 of 1481


Page 532 of 1481
**Benjamin**

Page 533 of 1481


Benjamin Answers: 2AC
BENJAMIN IS GOOD FOR AESTHETICS, BAD FOR POLICY

David E. McClean, New School University, The Cultural Left and the Limits of Social Hope, Presented at the 2001 Annual Conference of the Society for the Advancement of American
Philosophy, www.american-philosophy.org/archives/2001%20Conference/Discussion%20papers/david_mcclean.htm.

Cavell meant this reflection to be taken non-pejoratively because he seems to take Benjamin more
seriously as an aesthetician and literary metaphysician (in Rorty-speak, as a "strong poet") than as a
serious, social commentator with good ideas. Keeping Benjamin and his cohorts in the box of aesthetics
and metaphysics is, I believe, good intellectual policy for social critics seeking to be relevant. They should
be cited for seasoning and not for meat.

I am convinced that the


Yet I am not at all convinced that anything I have described is about to happen, though this essay is written to help force the issue, if only a little bit.

modern Cultural Left is far from ready to actually run the risks that come with being taken seriously and
held accountable for actual policy-relevant prescriptions. Why should it? It is a hell of a lot more fun and a
lot more safe pondering the intricacies of high theory, patching together the world a priori (which means
without any real consideration of those officers and bureaucrats I mentioned who are actually on the
front lines of policy formation and regulation). However the risk in this apriorism is that both the
conclusions and the criticisms will miss the mark, regardless of how great the minds that are engaged.
Intellectual rigor and complexity do not make silly ideas politically salient, or less pernicious, to paraphrase Rorty. This
is not to say that air-headed jingoism and conservative rants about republican virtue aren't equally silly and pernicious. But it seems to me that the new public philosopher of the Political Left
will want to pick better yardsticks with which to measure herself.

Page 534 of 1481


Page 535 of 1481
**Chaloupka**

Page 536 of 1481


Chaloupka Answers: 2AC (1/3)
FIRST, TURN EVEN IF NUCLEAR WEAPONS ARENT CONTROLLABLE, PLAN SOLVES
SOCIALLY CONSTRUCTED ACTIONS THAT CAUSE THEIR USE

SECOND, CHALOUPKA DOESNT UNDERSTAND IR. NUCLEAR WEAPONS ONLY


REMAIN TEXTUAL BECAUSE DETERRENCE WORKS. OUR SCENARIOS INDICATE A
BREAKDOWN ON MAD THAT ACTUALIZES NUCLEAR WAR.

THIRD, PERM: TO PLAN AND THE ALTERNATIVE. THE CRITIQUE ALONE IS A FALSE
CHOICE THAT DOOMS ACTIVISM

Sankaran Krishna, Professor of Political Science, U of Hawaii, Alternatives 1993, v. 18. p. 400-1

The dichotomous choice presented in this excerpt is straightforward: one either indulges in total critique,
delegitimizing all sovereign truths, or one is committed to nostalgic, essentialist unities that have
become obsolete and have been the grounds for all our oppressions.

In offering this dichotomous choice, Der Derian replicates a move made by Chaloupka in his equally
dismissive critique of the move mainstream nuclear opposition, the Nuclear Freeze movement of the
early 1980s, that, according to him, was operating along obsolete lines, emphasizing facts and
realities, while a postmodern President Reagan easily outflanked them through an illusory Star Wars
program (See KN: chapter 4)

Chaloupka centers this difference between his own supposedly total critique of all sovereign truths (which
he describes as nuclear criticism in an echo of literary criticism) and the more partial (and issue based)
criticism of what he calls nuclear opposition or antinuclearists at the very outset of his book. (Kn: xvi)
Once again, the unhappy choice forced upon the reader is to join Chaloupka in his total critique of all
sovereign truths or be trapped in obsolete essentialisms.

This leads to a disastrous politics, pitting groups that have the most in common (and need to unite on
some basis to be effective) against each other. Both Chaloupka and Der Derian thus reserve their most
trenchant critique for political groups that should, in any analysis, be regarded as the closest to them in
terms of an oppositional politics and their desired futures. Instead of finding ways to live with these
differences and to (if fleetingly) coalesce against the New Right, this fratricidal critique is politically
suicidal. It obliterates the space for a political activism based on provisional and contingent coalitions, for
uniting behind a common cause even as one recognizes that the coalition is comprised of groups that
have very differing (and possibly unresolvable) views of reality. Moreover, it fails to consider the
possibility that there may have been other, more compelling reasons for the failure of the Nuclear
Freeze movement or anti-Gulf War movement. Like many a worthwhile cause in our times, they failed to
garner sufficient support to influence state policy. The response to that need not be a totalizing critique
that delegitimizes all narratives.

The blackmail inherent in the choice offered by Der Derian and Chaloupka, between total critique and
ineffective partial critique, ought to be transparent. Among other things, it effectively militates against
the construction of provisional or strategic essentialisms in our attempts to create space for activist

Page 537 of 1481


politics. In the next section, I focus more widely on the genre of critical international theory and its impact
on such an activist politics.

FOURTH, TURN DEBATE ISNT A TRAGIC PERSPECTIVE ON NUCLEAR WAR ITS A


COMICAL GAME IN WHICH WE THROW AROUND SCENARIOS THAT WE TAKE WITH
A GRAIN OF SALT, OVERCOMING THE PERSPECTIVE CHALOUPKA CRITICIZES

Page 538 of 1481


Page 539 of 1481
Chaloupka Answers: 2AC (2/3)
FIFTH, NO LINK CHALOUPKA IS CRITICIZING ANTI-NUCLEARISTS WHO DEFEND
UNSPEAKABILITY. THE 1AC IS AN EXPLICITY ACKNOWLEDGEMENT OF THE TEXTUAL
SPEAKABILITY OF NUKES

SIXTH, CLAIMING THAT NUKES ARE ONLY TEXTUAL ERASES THE HISTORY OF
FOURTH WORLD NUCLEAR VIOLENCE

Masahide Kato, professor of political science at the University of Hawaii, 1993, Alternatives vol. 18, p. 339

, from the perspectives of the Fourth World and Indigenous Nations, the nuclear catastrophe has never
Thus

been the unthinkable single catastrophe but the real catastrophe of reptetitive and ongoing nuclear
explosions and exposure to radioactivity. Nevertheless, ongoing nuclear wars have been subordinated to the imaginary
grand catastrophe by rendering them as mere preludes to the apocalypse. As a consequence, the history and
ongoing processes of nuclear explosions as war have been totally wiped out from the history and
consciousness of the First World community. Such a discursive strategy that aims to mask the real of nuclear warfare in the domain of imagery of nuclear
catastrophe can be observed even in Stewart Fiths Nuclear Playground, which extensively covers the history of nuclear testing in the Pacific:

Nuclear explosions in the atmosphere were global in effect. The winds and seas carried radioactive contamination over vast areas of the fragile ecosphere on which we all
depend for our survival and which we call the earth. In preparing for war, we were poisoning our planet and going to battle against nature itself.

AND, THAT LEGITIMIZES NUCLEAR VIOLENCE

Masahide Kato, professor of political science at the University of Hawaii, 1993, Alternatives vol. 18, p. 339

, the problematic division/distinction between


Although Firths book is definitely a remarkable study of the history of nuclear testing in the Pacific

the nuclear explosions and the nuclear war is kept intact. The imagery of final nuclear war narrated with
the problematic use of the subject (we) is located higher than the real of nuclear warfare in terms of
discursive value. This ideological division/heirarchization is the very vehicle through which the history and the ongoing
processes of the destruction of the Fourth World and Indigenous Nations by means of nuclear violence are

obliterated and hence legitimatized.

SEVENTH, IMAGINING NUCLEAR ANNIHILATION IS A PROJECT OF SURVIVAL


THEIR ALTERNATIVE CREATES REPRESSION AND DENIAL WHICH MAKES NUCLEAR
WAR MORE LIKELY

Lenz, Science and Policy Professor at SUNY, 90 (Nuclear Age Literature For Youth, p. 9-10)

Page 540 of 1481


A summary of Franks thought in Psychological Determinants of the Nuclear Arms Race notes how all
people have difficulty grasping the
magnitude and immediacy of the threat of nuclear arms and this psychological unreality is a basic obstacle to eliminating
that threat. Only events that people have actually experienced can have true emotional impact. Since Americans have escaped the devastation of nuclear
weapons on their own soil and nuclear weapons poised for annihilation in distant countries cannot be seen, heard, smelled, tasted, or touched, we find
it easy to imagine ourselves immune to the threat. Albert Camus had the same phenomenon in mind when he wrote in his essay
Neither Victims nor Executioners of the inability of most people really to imagine other peoples death (he might have added or their own). Commenting on
Camus, David P. Barash and Judith Eve Lipton observed that this distancing from deaths reality is yet another aspect of our
insulation from lifes most basic realities. We make love by telephone, we work not on matter but on machines, and we kill and are killed
by proxy. We gain in cleanliness, but lose in understanding. If we are to heed Camuss call to refuse to be either the victims of
violence like the Jews of the Holocaust, or the perpetrators of it like the Nazi executioners of the death camps, we must revivify the
imagination of what violence really entails. It is here, of course, that the literature of nuclear holocaust can
play a significant role. Without either firsthand experience or vivid imagining, it is natural, as Frank points out, to deny the
existence of death machines and their consequences. In psychiatric usage denial means to exclude from
awareness, because letting [the instruments of destruction] enter consciousness would create too strong a level of anxiety or other painful emotions. In
most life-threatening situations, an organisms adaptation increases chances of survival, but ironically, adapting ourselves to nuclear fear is
counterproductive. We only seal our doom more certainly. The repressed fear, moreover, takes a psychic toll.

Page 541 of 1481


Page 542 of 1481
Chaloupka Answers: 2AC (3/3)
EIGHTH, CRITICIZING REPRESENTATIONS OF NUCLEAR PRESENCE DOESNT
PRECLUDE THE NEED FOR CONCRETE ACTION

Richard Rorty, Professor of Humanities, University of Virginia, Truth, Politics, and Postmodernism, Spinoza Lectures,

1997, p. 51-2

This distinction between the theoretical and the practical point of view is often drawn by Derrida, another writer who enjoys demonstrating that something very important
meaning, for example, or justice, or friendship is both necessary and impossible. When asked about the implications of these paradoxical fact, Derrida usually replies that

the paradox doesn't matter when it comes to practice. More generally, a lot of the writers who are labeled
`post-modernist; and who talk a lot about impossibility, turn out to be good experimentalist social democrats when it
comes to actual political activity. I suspect, for example, that Gray, Zizek, Derrida and I, if we found ourselves citizens of the same country, would all be
voting for the same candidates, and supporting the same reforms. Post-modernist philosophers have gotten a bad name because of their paradox-mongering habits, and their
constant use of terms like `impossible; `self-contradictory' and `unrepresentable'. They have helped create a cult of inscrutability, one which defines itself by opposition to the
Enlightenment search for transparency - and more generally, to the `metaphysics of presence; the idea that intellectual progress aims at getting things clearly illuminated,

. I am all for getting rid of the metaphysics of presence, but I think that the rhetoric of
sharply delimited, wholly visible

impossibility and unrepresentability is counterproductive overdramatization. It is one thing to say that we need to get rid of the
metaphor of things being accurately represented, once and for all, as a result of being bathed in the light of reason. This metaphor has created a lot of headaches for
philosophers, and we would be better off without it. But that does not show that we are suddenly surrounded by unrepresentables; it just shows that `more accurate

Even if we agree that we shall never have what Derrida calls "a full
representation' was never a fruitful way to describe intellectual progress.

presence beyond the reach of play"; our sense of the possibilities open to humanity will not have
changed. We have learned nothing about the limits of human hope from metaphysics, or from the philosophy of history, or from psychoanalysis. All that we have learned
from `post-modern' philosophy is that we may need a different gloss on the notion of `progress' than the rationalistic gloss which the Enlightenment offered. We have

been given no reason to abandon the belief that a lot of progress has been made by carrying out the
Enlightenment's political program. Since Darwin we have come to suspect that whether such progress is made will be largely a matter of luck. But we
have been given no reason to stop hoping to get lucky.

NINTH, MEDIA IMAGES PLAY THE CRUCIAL ROLE OF REVEALING THEIR OWN
ILLUSIONS

Jean Baudrillard, professor of philosophy of culture and media at Univ. or Paris, 1994, Illusion of the End,
pg. 60-61

And yet there will, nonetheless, have been a kind of verdict in this Romanian affair, and the artificial
heaps of corpses will have been of some use, all the same. One might ask whether the Romanians, by the
very excessiveness of this staged event and the simulacrum of their revolution, have not served as
demystifiers of news and its guiding principle. For, if the media image has put an end to the credibility of
the event, the event will, in its turn, have put an end to the credibility of the image. Never again shall we
be able to look at a television picture in good faith, and this is the finest collective demystification we
have ever known. The finest revenge over this new arrogant power, this power to blackmail by events.
Who can say what responsibility attaches to the televisual production of a false massacre (Timisoara), as
compared with the perpetrating of a true massacre? This is another kind of crime against humanity, a
hijacking of fantasies, affects and the credulity of hundreds of millions of people by means of television
a crime of blackmail and simulation. What penalty is laid down for such a hijacking? There is no way to
rectify this situation and we must have no illusions: there is no perverse effect, nor even anything
scandalous in the Timisoara syndrome. It is simply the (immoral) truth of news, the secret purpose
[destination] of which is to deceive us about the real, but also to undeceive us about the real. There is no

Page 543 of 1481


worse mistake than taking the real for the real and, in that sense, the very excess of media illusion plays a
vital disillusioning role. In this way, news could be said to undo its own spell by its effects and the violence
of information to be avenged by the repudiation and indifference it engenders. Just as we should be
unreservedly thankful for the existence of politicians, who take on themselves the responsibility for that
wearisome function, so we should be grateful to the media for existing and taking on themselves the
triumphant illusionism of the world of communications, the whole ambiguity of mass culture, the
confusion of ideologies, the stereotypes, the spectacle, the banality soaking up all these things in their
operation. While, at the same time, constituting a permanent test of intelligence, for where better than
on television can one learn to question every picture, every word, every commentary? Television
inculcates indifference distance, scepticism and unconditional apathy. Through the worlds becoming-
image, it anaesthetizes the imagination, provokes a sickened abreaction, together with a surge of
adrenalin which induces total disillusionment. Television and the media would render reality [le reel]
dissuasive, were it not already so. And this represents an absolute advance in the consciousness or the
cynical unconscious of our age.

Page 544 of 1481


Page 545 of 1481
Page 546 of 1481
CLS Answers: 2AC (1/4)
FIRST, TURN WE EXPOSE THE FLAWS IN EX PARTE QUIRIN, SOLVING BETTER
THROUGH HISTORICAL ANALYSIS

SECOND, CRITIQUE DOESNT SOLVE THERES NO REASON POINTING OUT FLAWS


IN THE SYSTEM WILL LEAD TO A HUGE MINDSET SHIFT. THE LAW WILL STILL
UNILATERALLY DETAIN ENEMY COMBATANTS. PREFER OUR SPECIFIC TRIBE AND
KATYAL EV

THIRD, TURN- UPHOLDING LEGAL PRINCIPLES PROVES THE LAWS FRAUDULENCE


AND HOLDS IT ACCOUNTABLE

Vclav Havel, playwright, political prisoner, and president elect of Czechoslovakia, 1986 (Living in Truth, p. 137-
38)

A persistent and never-ending appeal to the laws not just to the laws concerning human rights, but to all
laws does not mean at all that those who do so have succumbed to the illusion that in our system the
law is anything other than what it is. They are well aware of the role it plays. But precisely because they
know how desperately the system depends on it on the noble version of the law, that is they also
know how enormously significant such appeals are. Because the system cannot do without the law,
because it is hopelessly tied down by the necessity of pretending the laws are observed, it is compelled to
react in some way to such appeals. Demanding that the laws be upheld is thus an act of living within the
truth that threatens the whole mendacious structure at its point of maximum mendacity. Over and over
again, such appeals make the purely ritualistic nature of the law clear to society and to those who inhabit
its power structures. They draw attention to its real material substance and thus, indirectly, compel all
those who take refuge behind the law to affirm and make credible this agency of excuses, this means of
communication, this reinforcement of the social arteries outside of which their will could not be made to
circulate through society. They are compelled to do so for the sake of their own consciences, for the
impression they make on outsiders, to maintain themselves in power (as part of the systems own
mechanism of self-preservation and its principles of cohesion), or simply out of fear that they will be
reproached for being clumsy in handling the ritual. They have no other choice: because they cannot
discard the rules of their own game, they can only attend more carefully to those rules. Not to react to
challenges means to undermine their own excuse and lose control of their mutual communications
system. To assume that the laws are a mere facade, that they have no validity and that therefore it is
pointless to appeal to them would mean to go on reinforcing those aspects of the law that create the
facade and the ritual. It would mean confirming the law as an aspect of the world of appearances and
enabling those who exploit it to rest easy with the cheapest (and therefore the most mendacious) form of
their excuse. I have frequently witnessed policemen, prosecutors or judges if they were dealing with an
experienced Chartist or a courageous lawyer, and if they were exposed to public attention (as individuals
with a name, no longer protected by the anonymity of the apparatus) suddenly and anxiously begin to
take particular care that no cracks appear in the ritual. This does not alter the fact that a despotic power is
hiding behind that ritual, but the very existence of the officials anxiety necessarily regulates, limits and
slows down the operation of that despotism.

Page 547 of 1481


Page 548 of 1481
Page 549 of 1481
CLS Answers: 2AC (2/4)
FOURTH, PERM DO BOTH. FIGHTING WITHIN THE SYSTEM BY PRETENDING THAT
WE CAN CHANGE IT IN SPITE OF ITS LIMITATIONS PRODUCES A MORE EFFECTIVE
CLS THAT ENGAGES IN PRAXIS

Sparer 84
[Ed, Prof. Law and Soc Welfare @ Pennsylvania, Fundamental Human Rights, Legal Entitlements, and the
Social Struggle: A Friendly Critique of the Critical Legal Studies Movement, 36 Stan. L. Rev. 509, January,
ln//uwyo-ajl]

From this background, Gordon traces an emerging "interpretative" Critical legal theory that emphasizes the role of legal doctrine in "belief-systems that people have
externalized and allowed to rule their lives." n121 It is "belief systems" that count, even though "many constraints on human social activity," such as finite resources, do exist.
Given these belief systems, not even the "organization of the working class or capture of the state apparatus will automatically" produce conditions which lead to "the utopian
possibilities of social life." He then concludes:

Of course, this does not mean that people should stop trying to organize the working class or to influence the
exercise of state power; it means only that they have to do so pragmatically and experimentally, with full
knowledge that there are no deeper logics of historical necessity. . . . Yet, if the real enemy is us -- all of us, the structures we carry
around in our heads, the limits on our imagination -- where can we even begin? Things seem to change in history when people break out

of their accustomed ways of responding to domination, by acting as if the constraints on their improving
their lives were not real and that they could change things; and sometimes they can, though not always in
the way they had hoped or intended; but they never knew they could change them at all until they tried. n122

Gordon's conclusion is profound. But it contradicts the view that a negative attack on liberal legal doctrine is the
key path to a liberated future. n123 People break out of their accustomed ways of responding to [*558]
domination by acting as if they could change things. "Acting as if they could change things" does not mean confining scholarly endeavor to
negative doctrinal analysis, even though negative doctrinal analysis may be one helpful step towards acting. Acting means struggling for and living a

different way, even if only "experimentally," and this requires praxis, theory which guides and is in turn influenced by action. n124 Yet the whole of
Gordon's piece, until his conclusion, is an exposition which becomes a polemic -- almost an apology -- for the negative Critical analysis which constitutes virtually the sole

response to the practitioners' yearning for helpful theory .

FIFTH, SPECIFIC SOLVENCY TRUMPS PREFER OUR TRIBE AND KATYAL EV


SHOWING THAT OVERRULING QUIRIN CREATES EFFECTIVE DUE PROCESS RIGHTS

SIXTH, HERES MORE EV INDETERMINACY MEANS YOU HAVE TO EVALUATE THE


EMPIRICAL JUSTIFICATION OF OUR SOLVENCY CLAIMS

Hasnas 95
[John, JD & PhD Phil @ duke, Asst. prof. Bus Ethics @ Georgetown, Back to the Future, 45 Duke L.j. 84,
October, LN//uwyo-ajl]

I have suggested that this greatly overstates what the indeterminacy argument actually implies. Rather,
the proper inference to draw from a demonstration that the law is indistinguishable from politics is that

Page 550 of 1481


the cases in which the law should be employed to reform society are limited to those in which the desired
reforms can be effectively realized through political action. The insight the legal realists provided long ago
was that to identify these cases, one must undertake the pragmatic examination of how the law works in
practice relative to alternative methods of social control. Thus, there is a need for empirical investigation
to determine how the expected outcomes of collective political action compare with those of politically
unrestrained individuals functioning in a market environment. Further, to be valid, this investigation must
compare like with like; it must compare what can reasonably be achieved

[*131] through real-world political processes staffed by less than perfect human beings with what is
likely to result from unrestrained human interaction in the flawed markets that actually exist, not the
utopian results of an ideal political system with those of imperfect, real-world markets. Because this is the
case and because the Crits have resisted undertaking such investigations, I have argued that they have
missed the point of the indeterminacy argument, and that if this argument is in fact correct, the way
forward into our jurisprudential future lies in a return to the uncompleted project of the realists.

Page 551 of 1481


Page 552 of 1481
CLS Answers: 2AC (3/4)
SEVENTH, EXPERIENTIAL DECONSTRUCTION:
ORGANIC INTELLECTUALS MUST CONTEXTUALIZE CRITICISM IN THE CONTEXT OF
SPECIFIC OPPRESION, STRATEGICALLY USING HEGEMONIC NORMS TO CREATE
THEIR ALTERNATIVE ****

Cook 90
[Anthony E., Assoc Prof. Law @ Florida, Beyond Critical Legal Studies, 103 Harv. L. Rev. 985, March, LN//uwyo-ajl]

Because he appreciated the dialectic of theory and the broad-based confrontational strategies of socially
transformative action, King stands as the paradigmatic organic intellectual of twentieth-century American life. King's method and practice offer
direction to progressive scholars concerned about the exclusionary, repressive, and non-communal dimensions of American life.

[*1013] Gramsci's conception of the organic intellectual provides a useful framework for understanding the thought of King and what it has to offer CLS. The organic
intellectual brings philosophy to the masses, not for the merely instrumental purposes of unifying them, "but precisely in order to construct an intellectual-moral bloc which can
Gramsci's organic intellectual struggles
make politically possible the intellectual progress of the mass and not only of small intellectual groups." n78

to transform those who are oppressed as a means of transforming the conditions under which they are
oppressed. n79 Gramsci understands domination in terms of both coercion and consent, the latter constituting what he refers to as hegemony. Under his formulation,
hegemony consists, then, of "[t]he 'spontaneous' consent given by the great masses of the population to the general direction imposed on social life by the dominant
fundamental group." n80 Gramsci argues that "this consent is 'historically' caused by the prestige (and consequent confidence) which the dominant group enjoys because of its
position and function in the world of production." n81 Thus, oppression is not only physical and psychological but also cultural. n82

King, like Gramsci's organic intellectual, empowered his community through a practical effort to bridge the gap
between theory and lived experience. King's work consisted of four interrelated activities. First, he used theoretical
deconstruction to free the mind to envision alternative conceptions of community. Second, he employed
experiential deconstruction to understand the liberating dimensions of legitimating ideologies like liberalism and
Christianity, dimensions easily ignored by the abstract, ahistorical, and potentially misleading critiques that

rely exclusively on theoretical deconstruction. Third, he used the insights gleaned from the first two
activities to postulate an [*1014] alternative social vision intended to transform the conditions of oppression
under which people struggle. Drawing from the best of liberalism and the best of Christianity, King forged
a vision of community that transcended the limitations of each and built upon the accomplishments of
both. Finally, he created and implemented strategies to mobilize people to secure that alternative vision. I
refer to this multidimensional critical activity as "philosophical praxis."

Although many critical theorists engage primarily in theoretical deconstruction, and some appreciate certain forms of experiential deconstruction, n83 few have embraced either

reconstructive theorizing and socially


a full experiential deconstruction or the third and fourth dimensions of philosophical praxis --

transformative struggle. n84 These dimensions of critical activity directly confront the material conditions of oppression
whereas the preoccupation with deconstructing theory does not. King went further than these critical
theorists by examining the subtle and complex ways in which consent was shaped, while fully appreciating
the role of state and private coercion in legitimating authority in the lives of the oppressed.

King filtered his theoretical deconstruction of hegemonic theologies through his


This Part examines how

knowledge of the history and experience of oppression, and thereby made that theoretical deconstruction
richer, more contextual, and ready to engage the existential realities of oppression. The interplay between King's
theoretical and experiential deconstruction is best illustrated by reference to the African-American Church -- the institution providing the organic link between philosophy and the
masses, theory and praxis. n85

My analysis proceeds in four steps. First, I examine how African-American religion served at once to legitimate slave society, delegitimize that society, and inform alternative
visions of community. Second, I examine King's use of theoretical deconstruction and illustrate its dependence on the historic mission of the African-American Church. Like a
true organic intellectual engaged in a philosophical praxis, King used theoretical deconstruction to illustrate the possibilities [*1015] of his reconstructive vision and the
centrality of social struggle in realizing that vision. Third, I discuss King's experiential deconstruction, his unwillingness to be distracted by the reified abstractions of theoretical

the combination of theoretical and experiential deconstruction results in a more


deconstruction. Finally, I show how

contextual framework -- one more appreciative of the conditions of choice within which authority is
legitimated and challenged through reconstructive vision and struggle.

Page 553 of 1481


Page 554 of 1481
CLS Answers: 2AC (4/4)
EIGHTH, LIBERALISM IS INEVITABLE AND NECESSARY TO ACCOMPLISH CLSS
LIBERATORY GOALS

Sparer 84
[Ed, Prof. Law and Soc Welfare @ Pennsylvania, Fundamental Human Rights, Legal Entitlements, and the
Social Struggle: A Friendly Critique of the Critical Legal Studies Movement, 36 Stan. L. Rev. 509, January,
ln//uwyo-ajl]

The thrust of CLS critique is devoted, in turn, to the exposure of the contradictions in liberal philosophy and law. This strand of the Critical legal critique is quite powerful and

, the critique lends itself to exaggeration. This


makes a much-needed contribution. In my view, however, it suffers from two general problems. First

when Critical legal theorists themselves make tentative gestures at the


observation may be appreciated by considering what happens

social direction in which we should move. Such gestures, even from the most vigorous critics of liberalism,
do not escape from liberalism and, indeed, liberal rights theory. Nevertheless, those gestures have great merit,
particularly because of their use of liberal rights. For example, Frug, while expounding his vision of the city as a site of localized power and
participatory democracy, attacks liberal theory and its dualities as an obstacle to his vision. n19 At the same time, without [*518] acknowledging the significance of what he is

doing, Frug relies on the liberal image of law and rights to defend the potential of his vision. He writes:

participatory democracy on the local level need not mean the tyranny of the majority
It should be emphasized that

over the minority. Cities are units within states, not the state itself; cities, like all individuals and entities within the state, could be subject to state-created legal
restraints that protect individual rights. Nor does participatory democracy necessitate the frustration of national political objectives by local protectionism; participatory

. The liberal image of law as mediating


institutions, like others in society, could still remain subject to general regulation to achieve national goals

between the need to protect the individual from communal coercion and the need to achieve communal
goals could thus be retained even in the model of participatory democracy. n20

Page 555 of 1481


Page 556 of 1481
#4 Permutation: 1AR (1/2)
WE MUST RECOGNIZE THE LIMITATIONS OF LAW WHILE USING IT AS A STRATEGY
FOR SURVIVAL.

Ruthann Robson, Professor of Law, CUNY Law School, New York, Lesbian (Out)law, 1992, p. 89-90

Yet these legal strategies can also afford concrete improvements as we live our lives within the dominant
culture. They can even make us validate our own experiences because they have been recognized by the
law. Within our own communities, theories, and relationships, the implementation of equality in the form
of antidiscrimination rules of law would bring out change. Gone would be the Latina Lesbian Caucus,
womenonly space, sliding scales, anthologies of older lesbians. If we accepted the rule of law as the rule
of lesbianism, we would not discriminate between lesbians and nonlesbians. For many of us, this is
unacceptable. I am not proposing that we must either totally adopt antidiscrimination discourse into all
facets of our lives, or we must totally abandon it as a legal strategy. Such a duality is a false one. We are
not hypocritical, inconsistent, or contradictory if we recognize antidiscrimination as a potential strategy
for legal change, yet recognize its limitations. Our desires are as complex as we are. Concepts such as
equality and antidiscrimination cannot fulfill our desires. Yet we can use these legal notions to effect the
type of legal change that can facilitate our survival. Our formidable task is strategizing, theorizing, and
actualizing our own desires against a legal background of discrimination, all the while resisting our own
domestication.

REJECTION FAILS- MUST COMBINE THE PLAN AND THE ALTERNATIVE

HUTCHINSON AND MONAHAN 84


(Allan and Patrick, Asst Prof @ NYU and Asst Prof @ Ottawa U, January, 36 Stan. L. Rev. 199, CRITICAL
LEGAL STUDIES SYMPOSIUM: Law, Politics, and the Critical Legal Scholars: The Unfolding Drama of
American Legal Thought, MosE)

The development and implementation of such an enlarged notion of legal doctrine would require a
complete restructuring of the existing order. Unger, of course, is not blind to this. With a truly grand
sweep, he drafts the essential framework of such a society; he substantiates and formalizes the "structure
of no-structure." He envisages the establishment of a "rotating capital fund" n150 to finance individual
projects and to effect a decentralization of production and exchange. The legal counterpart of this notion
would be "the disaggregation of the consolidated property right." n151 Yet Unger recognizes that some
regime of rights would be necessary for his proposals to succeed. n152 He therefore suggests the creation
of four kinds of rights: immunity rights which give individuals the power to resist interference and
domination by any other individual or organization, including the state; destabilization rights which entitle
individuals to demand the disruption of established institutions and forms of social practice; market rights
which give a conditional claim to divisible portions of social capital, in place of the existing absolute
property rights; and solidarity rights which foster mutual reliance, loyalty, and communal responsibility.
Such arrangements, according to Unger, need not be established all at once, but can be introduced
gradually. n153 Unger finds this scheme attractive because it accommodates continuing conflict between
transitory factions of society; it allows [*233] "history itself [to] become a source of moral insight." n154

Page 557 of 1481


Page 558 of 1481
#4 Permutation: 1AR (2/2)
REFORM INSTITUTIONS FROM THE INSIDE. ITS THE ONLY WAY TO SOLVE AND
PRESERVE DEMOCRACY

Thomas F. McInerney III, Associate, Dorsey & Whitney LLP, New York, Creighton Law Review, 31 Creighton L. Rev.

805, May, 1998

Herein lies the normative turn in Habermas' thought. He claims that not only has this new paradigm in law
emerged, but also that such new understanding of law and politics must be guided by an understanding of
the limits of human reason, of which an intersubjective, or discourse-oriented approach to rationality,
entails. n240 Communicative, and hence intersubjective, rationality provides the means of reassessing
modernist legal institutions in light of a proceduralist reconstruction of law and democracy. n241 As such,
his discourse theory provides a critical tool to evaluate existing political, legal, and social institutions. Such
a critical program need not advocate the elimination of current institutions, but can build on the
principles on which such institutions are based. It may thus be used to reinterpret existing traditions and
institutions to realize a certain kinetic power for reinvigorating democracy. n242 This rather thin
normative argument requires only the critical reappraisal of legal and political institutions in accordance
with the discourse principle in an attempt to implement the principle in existing practice. After mapping
the earlier paradigms, Habermas makes the descriptive claim that a new paradigm has emerged to
replace the traditional liberal-bourgeois paradigm and welfare-bureaucratic [*832] paradigm. n238 This
new paradigm attempted to overcome the inadequacy of the previous orders. It represents a departure
from modernism and can be termed a post-modern paradigm. Unlike modernist ideologies, the post-
modern paradigm arises from an intersubjective n239 notion of rationality. No longer can political and
legal decisions be considered the product of a singular will within this paradigm but, instead, must be
viewed as a consensus-oriented process of decision-making involving communication by and among all
concerned participants. Under this paradigm, law must be understood procedurally. This normative
stance may at first appear inconsistent. On the one hand, Habermas asks that we accept his descriptive
claims that a new post-modern paradigm has emerged. On the other hand, he claims that we must adopt
a proceduralist view of law and an intersubjective notion of reason. Because our current political and legal
systems are not to be abandoned completely, Habermas intends his communicative sense of rationality to
be more completely realized in the existing legal order. n243 Habermas does not conceive the possibility
of realizing such changes in existing institutions as problematic. He [*834] argues that through a process
of reification, we have come to believe, incorrectly in his view, that existing social and political institutions
are fixed entities which cannot undergo change. n244 Rights, such as freedom of speech, although
justified by appeal to modernist ideals when implemented originally, have taken on new meaning within
this new paradigm. As such, these rights become essential to the more complete realization of
intersubjective rationality and communicative decision-making. n245 Having established the
methodological basis upon which Habermas's theory is founded, attention may be given to more
foundational aspects to the theory beginning with his under standing of communicative action. n243. Put
in critical theoretical terms, The critical enterprise must now be a critique of the inherent potential for
reaction within the existing power structure - i.e., the question is not one of dismantling the structure and
replacing it by another, but rather one of buttressing the existing power structure against the threat
looming from the right - whether the political, the economic, or the religious right.

Page 559 of 1481


Page 560 of 1481
#7 Experiential Deconstruction
Turn: 1AR
BLACK CHRISTIANITY PROVES OUR ARG READING THE INSTITUTION AGAINST
ITSELF ALLOWS COUNTERHEGEMONIC FREEDOM

Cook 90
[Anthony E., Assoc Prof. Law @ Florida, Beyond Critical Legal Studies, 103 Harv. L. Rev. 985, March, LN//uwyo-ajl]

2. The Role of Religion in the Delegitimation of Authority. -- Although the use of religion as an instrument
of social control often necessitated oversight by white masters, n101 strict enforcement was not
maintained, and slaves often met separately for religious services, including weekly and Sunday evening
services. n102 It was within the freedom provided for religious worship that Africans began to assert
some control over how the void created by the disintegration of their historical identity and community
would be filled. In this small space of freedom, an alternative conception of community was defined and
the history of a new American people began to emerge. African-American religion and its primary vehicle
of expression, the African-American Church, supplied the needed catalyst for the reconstruction of
community destroyed by slavery. n103

To the surprise and fear of many whites, slaves transformed an ideology intended to reconcile them to a
subordinate status into a manifesto of their God-given equality. n104 This deconstruction was both
revolutionary and pragmatic in nature. The Africans' appropriation of conservative evangelicalism as a
bulwark against the degradation and countless microaggressions of slavery proved that there were
alternate interpretations of the text that supposedly justified their subjugation. Slaves demonstrated that
scripture was subject to an alternative interpretation that called for the eradication of the very social
structure evangelicals sought to legitimate. n105 In short, slaves deconstructed ideology through their
struggles against oppression.

Although slavemasters and evangelicals attempted to limit the transmission of counter-hegemonic


interpretations of scripture, their [*1019] efforts met with limited success. African gospel preachers and
slaves who learned to read against their masters' wishes (and, many times, against state law as well) were
determined to read the Bible in light of their own experiences. Many slaves realized that the message of
submission, docility, and absolute obedience to the master was a distorted picture of the Bible's eternal
truths. n106

STRUGGLE IS A CATALYST FOR MAKING RIGHTS DETERMINATE, DISMANTLING


OPPRESSION

Cook 90
[Anthony E., Assoc Prof. Law @ Florida, Beyond Critical Legal Studies, 103 Harv. L. Rev. 985, March, LN//uwyo-ajl]

Unlike some CLS scholars, King understood the importance of a system of individual rights. CLS
proponents have urged that rights are incoherent and indeterminate reifications of concrete experiences;

Page 561 of 1481


they obfuscate, through the manipulation of abstract categories, disempowering social relations. n158
King, on the other hand, understood that the oppressed could make rights determinate in practice;
although "law tends to declare rights -- it does not deliver them. A catalyst is needed to breathe life
experience into a judicial decision." n159 For King, the catalyst was persistent social struggle to transform
the oppressiveness of one's existential condition into ever closer approximations of the ideal. The
hierarchies of race, gender, and class define those conditions, and the struggle for substantive rights
closes the gap between the latter and the ideal of the Beloved Community. Under the pressures of social
struggle, the oppressed can alter rights to better reflect the exigencies of social reality -- a reality itself
more fully understood by those engaged in transformative struggle.

King's Beloved Community accepted and expanded the liberal tradition of rights. King realized that
notwithstanding its limits, the liberal vision contained important insights into the human condition. For
those deprived of basic freedoms and subjected to arbitrary acts of state authority, the enforcement of
formal rights was revolutionary. African-Americans understood the importance of formal liberal rights and
demanded the full enforcement of such rights in order to challenge and rectify historical practices that
had objectified and subsumed their existence.

Page 562 of 1481


Page 563 of 1481
A2 Religious Institution
Rationalized Oppression: 1AR
FIRST, OUR 2AC COOK EV PRE-EMPTS THIS. INSTITUTIONS MAINTAINED
HEGEMONY BY NOT CONTEXTUALIZING THEMSELVES IN TERMS OF ACTION
AGAINST OPPRESSION. PLAN SOLVE BY ENGAGING SUBORDINATION

SECOND, THIS IS A DISAD TO THE ALT. PRAXIS IS NECESSARY TO AVOID CO-


OPTATION

Cook 90
[Anthony E., Assoc Prof. Law @ Florida, Beyond Critical Legal Studies, 103 Harv. L. Rev. 985, March, LN//uwyo-ajl]

King's synthesis of pragmatic and revolutionary evangelicalism was most powerfully expressed in his
"Letter from Birmingham City Jail." n151 Conservative evangelicalism's dichotomy between the spiritual
and the secular caused many religious leaders, just as in the days of slavery, to continue to oppose any
interpretation of Christianity demanding that equality before God in the spiritual realm also be embodied
in the legal and social relations defining the secular realm. These leaders still offered patience as a
panacea for the pain of persecution and the joys of an afterlife as an answer for the sufferings of this life.
If integration was the will of God, He and not humans would change people's hearts in His own way and
time. Be patient, they urged, and wait on the Lord. n152 King discerned the hegemonic role of this
theology and boldly challenged the injustice to which it gave rise wherever he encountered it. To those
who urged that nonviolent, [*1033] direct action was "unwise and untimely," King sharply retorted:

We know through painful experience that freedom is never voluntarily given by the oppressor; it must be
demanded by the oppressed. Frankly, I have never yet engaged in a direct action movement that was
"well-timed," according to the timetable of those who have not suffered unduly from the disease of
segregation. For years now I have heard the words "Wait!" It rings in the ear of every Negro with a
piercing familiarity. This "Wait" has almost always meant "Never." . . . We must come to see with the
distinguished jurist of yesterday, that "justice too long delayed is justice denied." n153

King expressed his great disappointment with this otherworldly orientation of the white Church:

In the midst of blatant injustices inflicted upon the Negro, I have watched white churches stand on the
sideline and merely mouth pious irrelevancies and sanctimonious trivialities. In the midst of a mighty
struggle to rid our nation of racial and economic injustice, I have heard so many ministers say, "Those are
social issues with which the gospel has no real concern," and I have watched so many churches commit
themselves to a completely otherworldly religion which made a strange distinction between body and
soul, the sacred and the secular. n154

Thus, King spent his life leading African-Americans into direct confrontation with oppressive institutions
and practices. Through direct action the African-American community exposed the contradictions and
violence endemic to American society. In this way, the civil rights movement King led was itself a powerful
form of experiential deconstruction, one that provided fertile ground for a new vision of community in
America.

Page 564 of 1481


Page 565 of 1481
#8 Liberalism Good Turn: 1AR
CLS FORECLOSES STRATEGIC LIBERALISM, DESTROYING LIBERATORY MOVEMENTS
AND REINFORCING OPPRESSION

Cook 90
[Anthony E., Assoc Prof. Law @ Florida, Beyond Critical Legal Studies, 103 Harv. L. Rev. 985, March, LN//uwyo-ajl]

ther are some liberating as well


Second, when we adopt this more contextual and experiential approach to understanding oppression, we will realize that

as legitimating aspects of the line-drawing or boundary-setting enterprise we critique. Democratic socialism, the American
Revolution, the African-American civil rights movement, and other social movements were based, in part, on
the liberating dimensions of liberal theory. Failing to recognize this, some scholars unwittingly fall into too
simplistic an analysis of the problem and its possible solutions. When we appreciate the liberating dimension of ideology, revealed by
experiential deconstruction, we might conclude that there are many dimensions of the present system that are good and quite enabling.

Thus, although I share critical methods, I question the conclusions of CLS. The CLS critique rightly points out that we need not accept oppressive institutions and practices as
unalterable expressions of truth, because the premises on which they are based are contradictory and indeterminate at best. The critique suggests, therefore, that we are free
to envision and construct alternative forms of community that represent a more accurate or at least more plausible conception of human nature -- one believed to be
fundamentally good, which may replace "our pervasive alienation and fear of one another with something more like mutual trust." n74 But should we be so certain that this

From this optimistic view, one might


optimistic view of human nature is clearly more liberating than the insights provided by Hobbes or Locke?

envision emerging a quite oppressive community in which groups, behind the guise of love and mutual
dependency, legitimate [*1011] behavior that is more oppressive than anything imagined by Hobbes'
sovereign. When, therefore, CLS proponents argue that liberalism's public-private dichotomy undermines a society's transformative potential, we should also ask how
and when does it advance those efforts. Indeed, if CLS' primary concern is one of legitimation and power, it is important to

ask under what conditions the liberal discourse of rights may be strategically delegitimizing and
substantively empowering.

EVEN IF THEYRE RIGHT, WE SHOULD STILL FIGHT FOR RIGHTS TO MAKE A


HUMANE SOCIETY THE ALTERNATIVE IS ETHICAL ABDICATION

Sparer 84
[Ed, Prof. Law and Soc Welfare @ Pennsylvania, Fundamental Human Rights, Legal Entitlements, and the
Social Struggle: A Friendly Critique of the Critical Legal Studies Movement, 36 Stan. L. Rev. 509, January,
ln//uwyo-ajl]

My point is that both liberal and radical theory (including Critical legal theory) must balance competing values. Of course, the same problem affects any statement of "rights" as

. There is no way of generalizing a resolution of all potentially contradictory values in all situations. This
well

impossibility, however, does not necessarily implicate the virtues of and need for rights themselves. Nor does
it mean that we should not struggle to alter the political and social context in which rights operate or to win
preference for certain rights over others.

The significance of the CLS overemphasis on and exaggeration of contradictions is that it increases the tendency on the part of
some Critical legal theorists to emphasize negative critique because they are overwhelmed by the very
deficiencies they criticize in liberal legal theory. n21 At the same time, some Critical legal theorists lose an
appreciation [*519] of the potential contribution of rights, a potential contribution which coexists with their negative potential. Exaggeration
thereby promotes an "undialectical" approach despite Critical theory's emphasis on dialectics.

Page 566 of 1481


CLS DISEMPOWERS BY IGNORING THE LIBERATORY POTENTIAL OF LIBERALISM

Kimberl Crenshaw , et al, Professor of Law, Columbia University, Critical Race Theory, Ed. Kimberl Crenshaw
et al, 1995, p. 110-111

Finally, in addition to exaggerating the role of liberal legal consciousness and underestimating that of
coercion, CLS scholars also disregard the transformative potential that liberalism offers. Although liberal
legal ideology may indeed function to mystify, it remains receptive to some aspirations that are central to
black demands; it may also perform an important function in combating the experience of being excluded
and oppressed. This receptivity to black aspirations is crucial, given the hostile social world that racism
creates. The most troubling aspect of the critical program, therefore, is that trashing rights consciousness
may have the unintended consequences of disempowering the racially oppressed while leaving white
supremacy basically untouched.

Page 567 of 1481


Page 568 of 1481
No Links (1/2)
YOUR ARGUMENT ASSUMES THE WRONG LIBERAL LEGALISM- THE AFF HAS A
COMMITMENT TO NEUTRALITY- WE CAN NEVER ACHIEVE WHAT YOUR
ALTERANTIVE CALLS FOR

ALTMAN 90
(Andrew, Prof of Philosophy @ Georgia State, 1990, Critical Legal Studies: A Liberal Critique, Pg. 102-103)
PHM

This chapter has examined three important lines of argument in the CLS literature. All three attempt to
establish that liberal theory is internally inconsistent, and all three claim that the inconsistency arises
from the liberal embrace of pluralism, neutrality, and the rule of law. The central contention of these
arguments is that it is impossible to satisfy both the demands of legality and those of neutrality in a
context of moral, religious, and political pluralism. I found that the three main lines of argument deployed
to support such a contention are all wanting. The arguments rest to a large degree on a confused
understanding of the liberal commitment to neutrality. In addition, the more radical CLS arguments rest
on a seriously inadequate understanding of linguistic meaning. Once those confusions and inadequacies
are remedied, it becomes clear that the requirements of legality and neutrality can be met in a pluralist
context.

NO LINK- SOCIAL REALITY IS NOT CONSTITUED BY LAW- MULTIPLE ALTERNATE


FACTORS

ALTMAN 90
(Andrew, Prof of Philosophy @ Georgia State, 1990, Critical Legal Studies: A Liberal Critique, Pg. 151) PHM

To join the issue with the rule conception, one must deny the claim that socially meaningful behavior
must be explained by reference to social rules. This denial became more and more frequent in the 1960s
and 1970s. The view became widespread that social rules must be explained by reference to individual
presocial motivation. According to this view, rules do not constrain and channel individual behavior at all
or do so only in sporadic and marginal ways. By and large, rules are resources and instruments that
individuals manipulate to get what they want or think good, and what they want or think good, at the
most fundamental level, is not determined by social rules. Rules exert no power (or little power) of their
own over individual thought, desire, and action; they are mere words. Nonetheless, rules can be invoked
by those who wield power to rationalize their actions and even to convince those over whom they
exercise power that their subordination is right and proper. Let us call this the instrumentalist view of
social rules. Edgerton summarizes the influence of this view on contemporary thinking:

In most social theory today, rules are seen as ambiguous, flexible, contradictory, and inconsistent; they
are said seldom to govern the actions of people, much less to mold these people by being internalized by
them. Instead, they serve as resources for human strategies.4

Page 569 of 1481


Page 570 of 1481
No Links (2/2)
NO LINK- LIBERAL LEGAL PHILOSOPHY DOESNT SAY THAT LAW SOLVES ALL OUR
PROBLEMS, BUT THAT IT IS BETTER THAN DOING NOTHING- YOU MUST WIN
EVERY INSTANCE OF LAW IS BAD

ALTMAN 90
(Andrew, Prof of Philosophy @ Georgia State, 1990, Critical Legal Studies: A Liberal Critique, Pg. 200) PHM

In the course of criticizing liberal legal philosophy, Robert Gordon has argued against "the kind of rule
fetishism that supposes salvation comes through rules, rather than through the social practices that the
rule makers try to symbolize and crystallize."65 It should now be apparent that Gordon's criticism of
liberalism in this regard rests on several misconceptions. First, liberal theory does not promise salvation
through legal rules; what it promises is a society that does a better job of protecting people from
intolerance, prejudice, and oppression than it would if law was dispensed with. Second, Gordon poses a
false dichotomy: Protection must be attempted either through rules (presumably he has legal rules in
mind) or through the nonlegal practices of society. The soundest version of liberal theory will reject this
dichotomy and argue that protection from intolerance, prejudice, and oppression requires both legal rules
and at least some complementary social practice.

Page 571 of 1481


Page 572 of 1481
Turns: Ricoeur
CLS CREATES AN EXTREME LEGAL HERMEUTICS OF SUSPICION, PREVENTING ANY
LEGAL REFORM

Hasnas 95
[John, JD & PhD Phil @ duke, Asst. prof. Bus Ethics @ Georgetown, Back to the Future, 45 Duke L.j. 84,
October, LN//uwyo-ajl]

Unlike the mainstream Crits, the irrationalists offer no specific program for legal reform. n83 This is because, as their designation
[*104] suggests, they believe that reason is impotent to resolve legal and moral issues. Heavily influenced by the philosophy of Richard Rorty n84 and the deconstructionist
school of literary criticism associated with Jacques Derrida, n85 the irrationalists believe that objective knowledge is impossible. Following Rorty, they reject the correspondence
theory of truth that holds that a statement is true when it is an accurate representation of an underlying reality. n86 They assert that since it is impossible "to step outside our
skins--the traditions, linguistic and other, within which we do our thinking and self-criticism--and compare ourselves with something absolute," n87 reality is socially constructed,
i.e., the result of social practices that "embody contingent choices concerning how to organize the thick texture of the world in consciousness." n88 Thus, the irrationalists adopt
the coherence theory in which "the meaning of words are not determined by external referents, but instead by their coherence with other words or judgments within our total

This, however, implies that "the attempt to fix the meaning of an expression leads to an
body of knowledge." n89

infinite regress," n90 and hence, that "meaning is ultimately indeterminate." n91 Since this is true generally, it obviously must be true within the legal realm as well.
n92 Therefore, for the irrationalists, the indeter- [*105] minacy of the law is merely a consequence of the inherent indeterminacy of human language. n93

This philosophical position, which has been described as radical subjective idealism, n94 leads the irrationalists to embrace an
extreme form of epistemic skepticism in which "it is impossible to say anththing true about the world." n95
This, of course, entails a commitment to ethical relativism such that "any action may be described as right
or wrong, good or bad." n96 Thus, for the irrationalists, reason is irrelevant to our normative pursuits. Since there are no objective moral
or legal truths, reason cannot help us find them: "Legal and moral questions are matters to be answered by experience, emotion, introspection,
and conversation, rather than by logical proof." n97 Hence,

when judges decide cases, they should do what we all do when we face a moral decision. We identify a limited set of alternatives; we predict the most likely consequences of
following different courses of action; we articulate the values that are important in the context of the decision and the ways in which they conflict

[*106] with each other; we see what relevant people (judges, scholars) have said about similar issues; we talk with our friends; we drink enormous amounts of coffee; we
choose what to do. n98

SKEPTICISM STOPS SOCIAL CHANGE THEIR PARANOIA FORECLOSES UPON


REVOLUTION

Berman 2001
[Paul Schiff, Assoc. Prof. Law @ U. of Connecticut, Yale Journal of Law and the Humanities, LN]

, one might view this as a positive development. One might think people should
Of course

stop being lulled into a false sense of believing that the rhetoric of public life really
matters. If people began to view such rhetoric as a construction of entrenched power, so
the argument might go, they would form the nucleus of a truly revolutionary political
movement.

I doubt that such an eventuality is likely to occur. Moreover, I am not sure that a culture of
suspiciousness is the most effective way to seek political (or personal) change anyway. Suspicious
analysis seeks to expose the dangers of our enchantment with reason or truth or collectivity, but there are dangers that arise from relentless disenchantment as
well. As [*123] Richard K. Sherwin has observed,

Without the means of experiencing more profound enchantments, without communal rituals and social
those beliefs ultimately
dramas through which the culture's deepest beliefs and values may be brought to life and collectively reenacted,

Page 573 of 1481


lose their meaning and die... . Forms of enchantment in the service of deceit, illicit desire, and
self-gratification alone must be separated out from forms of enchantment in the service of

feelings, beliefs, and values that we aspire to affirm in light of the self, social, and legal realities they help to construct and maintain. 112

Page 574 of 1481


Page 575 of 1481
Turns: Judicial Oppression
THE ALTERNATIVE FREES JUDGES FROM LEGAL RULES, ALLOWING UNCHECKED
OPPRESSION

Solum 87
[Lawrence B., Assoc. Prof. Law @ Loyola, On the Indeterminacy Crisis: Critiquing Critical Dogma, 54 U.
Chi. L. Rev. 462, Spring, ln//uwyo-ajl]

This is not the place for extended consideration of this conception of freedom. I do wish, however, to
make an observation [*500] about its implications: the sort of freedom brought about by acceptance of
the strong indeterminacy thesis disassociates internal critique from programmatic social change. This
radical sort of freedom might enable individual legal adjudicators, practitioners, and scholars to undergo
"conversions," liberating them from the constraints of doctrine. But the nature of such a liberation is
ambiguous. It is hardly clear that liberating those who wield legal power from the "mistaken" belief that
legal doctrine constrains their actions will have a progressive effect. If the mystification thesis is correct,
then acceptance of the indeterminacy thesis also will awaken those in power to the fact that legality is no
barrier to repression. n111

Singer recognizes the argument that "if we let judges do just what they want, they would inevitably
exercise judicial power in oppressive ways," and responds:

But people do not want just to be beastly to each other. To suppose so is to ignore facts. People want
freedom to pursue happiness. But they also want not to harm others or be harmed themselves. The
evidence is all around us that people are often caring, supportive, loving, and altruistic, both in their
family lives and in their relations with strangers.

It is also not true that, if left to do "just what they like," government officials will necessarily harm us or
oppress us. They may do these things if that is what they want to do. But it is simply not the case that all
government officials admire Hitler and Stalin and use them as role models. n112

It is possible that all that stands between us and a progressive system of justice is the elimination of the
myth that legal rules constrain judges, but the violent lessons of human history place a heavy burden of
persuasion on those who make that claim. Singer's view is profoundly optimistic.

Page 576 of 1481


Page 577 of 1481
Turns: Criticism Perpetuates
Capitalism
THEIR CRITIQUE OF THE LAW PREVENTS SOCIAL ORGANIZATION NECESSARY TO
CREATE SOCIALISM

Johnson 84
[Phillip E., Prof. Law @ Berkeley, Critical Legal Studies Symposium: Do You Sincerely Want To Be
Radical? 36 Stan. L. Rev. 247, January, ln//uwyo-ajl]

I have already described Critical legal scholarship as ambivalent in its diagnosis of our social and personal
ills, and, of course, uncertainty of diagnosis leads to uncertainty in the prescription of a remedy. There is a
further problem with the remedy itself, and the vacuity of Critical scholarship when faced with the task of
proposing remedies stems partly from a reluctance to come to grips with this problem. The issue can be
put quite simply: If we assume that leftist political movements aim to create "socialism," and that
"socialism" means something like individual self-determination within an ethic of cooperation, does this
goal imply a centralization or a decentralization of power?

Not many years ago socialism meant nationalization -- ownership and control of the economy by the
national government. But government management of industry and agriculture has been tried on a large
scale in many countries, and the results have not been exactly what the pioneers of socialism had hoped.
Inequalities of wealth have no doubt been reduced, but the bureaucratic state provides no cure for
alienation, competitive individualism, greed, power-seeking, or other ills previously associated with
capitalism. Furthermore, bureaucracies operate "by the book," and therefore even the most benign
bureaucracy is inherently hostile to individual selfdetermination. Anyone with experience in public
employment cannot fail to be aware of this fact.

To escape the rigidity of bureaucracy, socialists must reduce the scale of economic and political
organizations. Hence, they have been interested in worker control of individual factories, in small-scale
cooperatives, and in semi-independent local geographical units where social cooperation might flourish.
But how is a decentralized socialist [*285] society to prevent those small-scale units from adopting
antisocialist policies? Some localities are sure to set up new hierarchies, or to refuse to share the wealth
with the disadvantaged or the unproductive.If local units are permitted to trade with each other, market
forces will again begin to operate. Unless there is pervasive control by a national bureaucracy, what is to
prevent self-governing economic units from turning capitalist and attracting most of the movable capital
and the most ambitious people?

It is not for me to say whether socialists should prefer the rigidities of bureaucracy or the risks of
autonomy, but any socialist or "radical" author who evades the dilemma or attempts to straddle it is
peddling sheer fantasy. Neither will it do to propose that a "balance" be struck between national and local
authority. Power will inevitably gravitate to the authority that does the balancing.

Page 578 of 1481


Page 579 of 1481
Turns: Law Key to Solving
Atrocity
TRASHING THE LAW DESTROYS OUR BEST MEANS OF PROTECTING THE WEAK
AGAINST THE STRONG, ALLOWING FOR ENDLESS OPPRESSION AND ATROCITY

Hegland 85
[Kenney, Prof. of Law @ Arizona, Goodbye to Deconstruction, 58 S. Cal. L. Rev. 1203, July, ln//uwyo-ajl]

I fear deconstruction because people might come to believe in it, come to believe that the Rule of
So, what are these chips?

Law is a hoax masking illegitimate power. I believe this would be a bad thing. I offer, in support, one war story and one literary quote.
In the summer of 1965, I went south as a member of the Law Student Civil Rights Research Council. I worked with attorney C.B. King in Albany, Georgia. That summer there
were many civil rights marches, and the police often refused to protect the demonstrators. I recall sitting in a Federal District Court with C.B. King and listening to the judge tell a
rural sheriff, "The law requires you to protect the demonstrators. If you don't, I have no choice but to hold you in contempt." Be this illusion, I would not blithely dispel it.

Law can protect the weak from the strong. Economic and racial minorities would be in a worse condition
in a deconstructed world, for our southern sheriff would argue, "The only reason I must protect them
folks is to protect their first amendment rights, and the only reason they have first amendment rights is to get their voices heard, and, what with
television being what it is, I can assure them of a much larger audience by turning my dogs on them ." Right on, Sheriff! n38

[*1220] A character in Robert Bolt's play, A Man for All Seasons, argues that he would "cut down every law in England" to get the Devil. n39 Sir Thomas More responds:

And when the last law was down, and the Devil turned around on you -- where would you hide, the laws all being flat? This country's planted thick with laws from coast to coast
. . . and if you cut them down . . . d'you really think you could stand upright in the winds that would blow then? n40

I realize that one war story and one literary quote will not prove the need for the Rule of Law. I realize there are counter examples and, indeed, conflicting images: "In Heaven
there will be no law, and the lion will lie down with the lamb," Grant Gilmore assures us, while "[i]n Hell there will be nothing but law, and due process will be meticulously

observed." n41 No doubt , the issue of the importance of the Rule of Law ultimately resolves itself into a vision of
human nature. Morton Horwitz has written that to see the Rule of Law as an "unqualified human good" is to "succumb to Hobbesian pessimism" and to embrace a
"conservative doctrine." n42 One hates to admit to being suspicious, fearful and, perhaps, even mean-spirited. Yet,

we live in a century that has produced Hitler and Stalin.

Perhaps now is not the time to dump the Rule of Law.


"Goodbye to Deconstruction" -- I stole the title. In 1936 Fred Rodell of Yale wrote a delightful essay, "Goodbye to Law Reviews." n43 Mostly he pokes fun at the pomposity of
law reviews.

There are two things wrong with almost all legal writing. One is style. The other is content. That, I think, about covers the ground. [*1221] . . . [I]t seems to be a cardinal
principle of law review writing and editing that nothing may be said forcefully and nothing may be said amusingly. This, I take it, is in the interest of something called dignity. n44

Rodell's ultimate point goes, however, to content. And in this he is quite serious.

. With law as the only alternative to force as a means


I do not wish to labor the point but perhaps it had best be stated once in dead earnest

of solving the myriad problems of the world, it seems to me that the articulate among the clan of lawyers
might, in their writings, be more pointedly aware of those problems, might recognize that the use of law to help
toward their solution is the only excuse for the law's existence, instead of blithely continuing to make
mountain after mountain out of tiresome technical molehills. n45

Articulate deconstructionists, instead of blithely denying the existence of the mountain with tiresome
epistemology, might better devote their obvious talents to making it more habitable. n46

THE ALTERNATIVE TO LAW IS A WORLD WHERE THERE IS NO ORDER AND PEOPLE


DO WHAT THEY WANT- JUSTIFIES EVEN WORSE ATROCITIES THAN YOUR IMPACT

Page 580 of 1481


ALTMAN 90
(Andrew, Prof of Philosophy @ Georgia State, 1990, Critical Legal Studies: A Liberal Critique, Pg. 128) PHM

Consider the legal duty to aid a person to whom one owes no contractual or statutory obligation. The
traditional common law rule is that there is no legal duty to aid such a person (a "stranger"). But there are
a series of rules that qualify and carve out exceptions to the traditional rule. Thus, there is a rule that if
the actions of the defendant helped to create the dangerous situation in which the plaintiff found himself,
the defendant may have had a duty to render aid.32 There is a rule that if the plaintiff and the defendant
stand in some "special relationship," there may be a duty to render aid, even if there is no statute or valid
contract between the two requiring the aid.33

Page 581 of 1481


Page 582 of 1481
Turns: Law Key to Solving
Exploitation
LAW ALLOWS A COLLECTION OF RULES TO SETTLE SOCIETAL PROBLEMS LIKE
VIOLENCE- THE ALTERNATIVE HOMOGONIZES IDENTITY AND ELIMINATES
PROTECTION FROM THE ADVANTAGED

ALTMAN 90
(Andrew, Prof of Philosophy @ Georgia State, 1990, Critical Legal Studies: A Liberal Critique, Pg. 192-193)
PHM

"Let it be conceded that law typically operates at a higher level of abstraction than other social rules, at
least in a liberal society that exhibits moral, religious, and political pluralism. The law there will often
exclude considerations that would be viewed as relevant from the perspective of a certain ethical system,
religious doctrine, or political morality. If it did not, legal reasoning could not be clearly distinguished from
unconstrained moral inquiry and political choice. Moreover, the liberal conception of the rule of law
requires that public and private power be regulated by norms that are generalizable across situations and
can be applied in a regularized, predictable manner. This again requires that certain aspects of a case be
deliberately disregarded in the name of predictability. Where institutions cannot presuppose that all
officials share the same set of background moral, religious, and political ideas, the authoritative norms
that they lay down cannot regularly call for highly context-sensitive judgments without threatening the
regularity, predictability, and perhaps even the stability of the system.

Liberal law, then, does require a high level of abstraction, in the sense that it sometimes prescribes a
deliberate disregard for certain particulars of a case that could be quite relevant to a decision if one were
involved in more context-sensitive moral or political deliberation. Thus, the liberal should concede that
legal reasoning will often take place at a significantly higher level of abstraction than context-sensitive
normative deliberation. But he will contend that there are two very good arguments for institutions that
regulate power in accordance with reasoning that proceeds at a relatively high level of abstraction, given
a context of moral, religious, and political pluralism. First, it settles the terms of social life in a way that
allows us to avoid reopening fundamental questions about society and human life every time a conflict or
dispute breaks out. This liberates our energies from constant moral and ideological battles and enables us
to pursue vigorously other aims: commercial, scientific, artistic, and so forth. Second, legal abstraction can
materially assist in protecting people from intolerance and prejudice: When the Jew, the black, or the
homosexual is regarded as "just anybody" by the existing system of legal rules, he or she is protected from
the inclinations of intolerance and prejudice that could well playa role in more context-sensitive modes
for regulating public and private power. Let us examine the CLS response to each of these liberal
arguments.

RULE OF LAW IS THE ONLY OPTION IN A WORLD OF THE NATION-STATE- THE


ALTERNATIVE ALLOWS THE PRIVELEDGED TO EXPLOIT THE DISADVANTAGED

ALTMAN 90

Page 583 of 1481


(Andrew, Prof of Philosophy @ Georgia State, 1990, Critical Legal Studies: A Liberal Critique, Pg. 200-201)
PHM

Morton Horwitz has correctly pointed out that the rule of law can constrain not only oppressive and
misguided uses of power but also benevolent and beneficial ones.66 Whether the rule of law is to be
prized, then, hinges on the question of whether there is a greater need to confine through the rule of law
the intolerant and oppressive impulses of humans or to liberate the tolerant and benevolent impulses
from the constraints of legality. I do not believe that there is an a priori answer to this question. To that
extent, Horwitz is quite right to say that it is a mistake to characterize the rule of law as an "unqualified
human good," a characterization made by E. P. Thompson.67 However, the sorry human history of
persecution, prejudice, and intolerance over the past several centuries makes one conclusion inescapable:
Within the context of the nation-state and over the foreseeable future, the need to confine the impulses
of intolerance and oppression with the requirements of legality will continue to be far greater than the
need to liberate the impulses of of tolerance and benevolence from the restrictions of the rule of law. 68

Page 584 of 1481


Page 585 of 1481
Turns: Rights Good (1/4)
RIGHTS ALLOW RESISTANCE, EMPOWERMENT AND RECONFIGURING OF LAW
OUTSIDE THE LEGAL SYSTEM

Martha Minow, Professor of Law, Harvard University, Yale Law Journal, Interpreting Rights: An Essay for Robert

Cover, pg L/N 1987

Before drawing on these interpretive themes, I should try to clarify what I mean by "rights," an overused
word in legal, philosophical, and political debates. Defining "rights" is a difficult task because there is
considerable ambiguity in the meanings invoked in the debates about rights, and because much ink has
been spilled by legal and political theorists on this subject. One meaning is the formally announced legal
rules that concern relationships among individuals, groups, and the official state. "Rights" typically are the
articulation of such rules in a form that describes the enforceable claims of individuals or groups against
the state. n25 [*1867] Yet a second meaning will become important in this essay. "Rights" can give rise to
"rights consciousness" so that individuals and groups may imagine and act in light of rights that have not
been formally recognized or enforced. Rights, in this sense, are neither limited to nor co-extensive with
precisely those rules formally announced and enforced by public authorities. Instead, rights represent
articulations -- public or private, formal or informal -- of claims that people use to persuade others (and
themselves) about how they should be treated and about what they should be granted. I mean, then, to
include within the ambit of rights discourse all efforts to claim new rights, to resist and alter official state
action that fails to acknowledge such rights, and to construct communities apart from the state to nurture
new conceptions of rights. Rights here encompass even those claims that lose, or have lost in the past, if
they continue to represent claims that muster people's hopes and articulate their continuing efforts to
persuade. Consciousness, or cognizance, of rights, then, is not simply awareness of those rights that have
been granted in the past, but also knowledge of the process by which hurts that once were whispered or
unheard have become claims, and claims that once were unsuccessful, have persuaded others and
transformed social life. The connections between past and future claims of rights are voiced through
interpretations of inherited understandings of rights. Interpretation engages lawyers and nonlawyers in
composing new meanings inside and outside of legal institutions. Charges against new rights express
opposition to this interpretive process.

Page 586 of 1481


Page 587 of 1481
Turns: Rights Good (2/4)
RIGHTS ARE PART OF THE DECONSTRUCTIVE ENTERPRISE THEY OVERLOOK THE
FACT THAT RIGHTS DEMANDS ARE MADE BY SPECIFIC OPPRESSED GROUPS THAT
USE THEIR DEMANDS TO CALL INTO QUESTION SOCIETYS DOMINANT IDEOLOGIES

Goldfarb, Associate Law Professor at Boston College, 92 (Phyllis, A DIVERSITY OF INFLUENCE: From the Worlds
of "Others": Minority and Feminist Responses to Critical Legal Studies, New England Law Review, Spring, 26 New Eng.L.
Rev. 683)

Because some CLS scholars have focused narrowly on legal consciousness as the predominant ideological support of civil society,
they view appeals to legal consciousness -- through rights rhetoric, for example -- as ultimately legitimating the prevailing social
conditions and as fundamentally counterproductive to meaningful social change. n23 This argument is not without merit in the terms in which it is phrased. Nevertheless,
it overlooks the fact that challenges captured in abstract rights language are presented , not by generic groups, but by

specific groups with identifiable histories whose relationship to the social order may influence the way in
which others perceive their rights claims. African-Americans acquired a place in American society through chattel slavery which persists in the form of an
entrenched race hierarchy that denies recognition of African-Americans' full humanity. Against this backdrop, African-Americans' assertions of rights have

been a radical challenge to social arrangements, a challenge containing sufficient threat at various historical moments to provoke violent resistance.
n24 One must first appreciate the central ideological importance of racism in American society in order to fully comprehend the radical nature of nonwhites' claims to equal rights in a context

of deeply-felt white supremacy. n25 Civil rights claimants, who understood experientially the intransigent daily realities of their own race domination, were not
likely to underestimate the challenge posed to the traditional social order by their assertion of
mainstream equality. n26 As Crenshaw suggests, [*692] people of color knew that when powerful elements in society had defined
particular racial characteristics as conclusive proof of inferiority, an equality claim was a potent assault
on these collective psychological structures. n27 By proclaiming the unthinkable -- that people understood
to be inferior were entitled to equality -- the civil rights movement, through simple assertion of rights
routinely granted to whites, began delegitimating the ideology of race consciousness. In a powerful
deconstructive move, the reified abstractions harbored by masses of white Americans concerning the
characteristics attributed to African-Americans were thrown into question by African-Americans'
assertion of mainstream equality . n28 Crenshaw and others suggest that the feature of this story that African-Americans continue to need most to deconstruct is
the racist imagery, not the rights imagery. n29 Liberal legal notions, such as rights, represent strategies to be deployed in this

deconstructive enterprise. n30 The recognition of African-Americans as rights-bearers, as members of the


American community, transformed the experience of race oppression . In Patricia Williams' words: [*693] Rights imply a
respect which places one within the referential range of self and others, which elevates one's status
from human body to social being. For blacks, then, the attainment of rights signifies the due, the respectful
behavior, the collective responsibility properly owed by a society to one of its own . n31 The civil rights
movement reinforced one ideological support of American society -- legal consciousness -- to undermine another
ideological support of American society -- race consciousness. As Crenshaw explains, the effect of the latter ideology had been to isolate African-Americans so
effectively that no other route to social power was available. Only by playing the logic of the two prevailing ideologies against one

another, applying the language of rights to the situation of African-Americans, could the movement
hope to achieve any progress at all. The contradiction between American legal mythology and the
systemic treatment of African-Americans created the only room within which the racially subordinated
could maneuver. n32 The weight of daily oppression created an urgency that impelled African-Americans
to seize the only viable opportunity for change that presented itself . n33

Page 588 of 1481


Page 589 of 1481
Turns: Rights Good (3/4)
RIGHTS EXPOSE OPPRESSION AND GIVE SILENCED VOICES A FORUM FOR
RECOGNITION

Martha Minow, Professor of Law, Harvard University, Yale Law Journal, Interpreting Rights: An Essay for Robert

Cover, pg L/N 1987

What, then, is the equality signaled by rights discourse? The equality registered by rights claims is an
equality of attention. The rights tradition in this country sustains the call that makes those in power at
least listen. Rights -- as words and as forms -- structure attention even for the claimant who is much less
powerful than the authorities, and for individuals and groups treated throughout the community as less
than equal. n70 The interpretive [*1880] approach construes a claim of right, made before a judge, as a
plea for recognition of membership in a community shared by applicant and judge, much as reader and
author share the world of the text. n71 The language of rights voices an individual's desire to be
recognized in tones that demand recognition. n72 Rights discourse implicates those who use it in a form
of life, a pattern of social and political commitment. n73 Which claims will persuade, and how? With what
consequences for prior and subsequent claims? Which claims, indeed, will be recognized as even
deserving communal attention? n74 These are difficult and persistent questions in a community
committed to rights discourse. There is a risk that those points of view that have been silenced in the past
will continue to go unheard, and will be least adaptable to the vocabulary of preexisting claims. These are
issues for struggle, and some struggles may well take place beyond rights discourse, beyond language.
Some people may feel so shut out that the appeal to a communal commitment to rights makes no sense
to them. Nonetheless, an interpretive conception of rights is a way to take the aspirational language of
the society seriously n75 and to promote change by reliance on inherited traditions. It is a way to
challenge those who want to close the doors now that some of the previously excluded have fought and
found their way in. n76 [*1881] The metaphors of interpretation and conversation enable a conception of
community connections forged through the exchange of words in the struggle for meaning. n77 In a
powerful novel about contemporary South Africa, Nadine Gordimer's Rosa Burger responds to a critic of
liberalism by saying: I'm not offering a theory. I'm talking about people who need to have rights -- there -
- in a statute book, so that they can move about in their own country, decide what work they'll do and
what their children will learn at school. . . . People must be able to create institutions -- institutions must
evolve that will make it possible in practice. That utopia, it's inside . . . without it, how can you . . . act?
n78 The use of rights discourse affirms community, but it affirms a particular kind of community: a
community dedicated to invigorating words with power to restrain, so that even the powerless can appeal
to those words. It is a community that acknowledges and admits historic uses of power to exclude, deny,
and silence -- and commits itself to enabling suppressed points of view to be heard, to make covert
conflict overt. n79 Committed to making available a rhetoric of rights where it has not been heard before,
this community uses rights rhetoric to make conflict audible and unavoidable, even if limited to words, or
to certain forms of words. n80 If there is [*1882] conflict experienced in the introduction of rights rhetoric
to a new area, it is over this issue: Should the normative commitment to restrain power with communal
dedication reach this new area? The power in question may be public or private. For example, with
children's rights, large disagreements persist over whether and how communal limits should constrain the
exercise of private, especially parental, power. n81 Children's rights may enlarge state power over both
children and adults, not simply recognize children's pre-existing autonomy. n82 But it is the meaning of
autonomy, and its relation to rights, that claims attention next.

Page 590 of 1481


Page 591 of 1481
Turns: Rights Good (4/4)
DEMANDS OUTSIDE OF RIGHTS RHETORIC FAIL DEMANDING RIGHTS MAY REIFY
THE DOMINANT SYSTEM BUT ARE THE ONLY TO PROTECT THE LIVES AND LIBERTY
OF THE OPPRESSED

Crenshaw, Law Professor at UCLA, 88 (Kimberle Williams, RACE, REFORM, AND RETRENCHMENT:
TRANSFORMATION AND LEGITIMATION IN ANTIDISCRIMINATION LAW, Harvard Law Review, May, 101
Harv. L. Rev. 1331)

Rights discourse provided the ideological mechanisms through which the conflicts of federalism, the power of the
Presidency, and the legitimacy of the courts could be orchestrated against Jim Crow. Movement leaders used these tactics to force open a conflict between

whites that eventually benefited Black people. Casting racial issues in the moral and legal rights rhetoric of the prevailing ideology

helped create the political controversy without which the state's coercive function would not have been
enlisted to aid Blacks. Simply critiquing the ideology from without or making demands in language outside the
rights discourse would have accomplished little. Rather, Blacks gained by using a powerful combination of
direct action, mass protest, and individual acts of resistance, along with appeals to public opinion and the courts couched in
the language of the prevailing legal consciousness. The result was a series of ideological and political
crises. In these crises, civil rights activists and lawyers induced the federal government to aid Blacks and
triggered efforts to legitimate and reinforce the authority of the law in ways that benefited Blacks. Simply
insisting that Blacks be integrated or speaking in the language of "needs" would have endangered the lives of those who were already taking risks -- and with no reasonable chance of success.

Eisenhower, for example, would not have sent federal troops to Little Rock simply at the behest of protesters
President

demanding that Black schoolchildren receive an equal education. Instead, the successful manipulation of legal
rhetoric led to a crisis of federal power that ultimately benefited Blacks. n192 Some critics of legal reform
movements seem to overlook the fact that state power has made a significant difference -- sometimes
between life and death -- in the efforts of Black people to transform their world. Attempts to harness the
power of the state through the appropriate rhetorical/legal incantations should be appreciated as
intensely powerful and calculated political acts. In the context of white supremacy, engaging in rights discourse should be
seen as an act of self-defense. This was particularly true because the state could not assume a position of neutrality regarding Black people once the movement had
mobilized people to challenge the system of oppression: either the coercive mechanism of the state had to be used to support white supremacy, or it had to be used to dismantle it. We know
now, with hindsight, that it did both. n193

LIBERAL LEGAL THEORY INTEGRATES NON-LEGAL SOLUTIONS AS A COMPANION


TO LAW- THERE IS NO NORMATIVE VIEW INHERENT IN OUR REPRESENTATIONS
AND ONLY THE AFF CAN CREATE A FRAMEWORK FOR RIGHTS

ALTMAN 90
(Andrew, Prof of Philosophy @ Georgia State, 1990, Critical Legal Studies: A Liberal Critique, Pg. 101-102)
PHM

In addition, it would be a distortion of liberal theory to suggest that it has no place for nonlegal modes of
social regulation, such as mediation. Liberals can and do acknowledge the value of such nonlegal
mechanisms in certain social contexts and can consistently allow a place for them in liberal society. And
those who reject the rule of law can argue in the political arena for extending the role of such informal

Page 592 of 1481


mechanisms. Of course, a liberal state could not allow the antinomians to eradicate legal institutions; in
that sense, one might say that the liberal rule of law is not neutral. But the kind of political neutrality
which the liberal defends does not aim to guarantee that any normative view has an opportunity to
remake society wholly in its vision. It does guarantee an opportunity to negotiate and compromise within
a framework of individual rights, and there is no reason why those who defend nonlegal modes of social
regulation cannot seize the opportunity under a liberal regime to carve out a significant role for nonlegal
modes of social regulation within the liberal state. The liberal version of political neutrality demands that
antinomians have such an opportunity, but there is nothing remotely inconsistent in liberal thought in
making that demand or prohibiting anti legalism from going so far as to destroy all legal institutions.

Page 593 of 1481


Page 594 of 1481
Turns: Alternative Causes Rights
Rollback
THE ALTERNATIVES DIALECTICAL CONCEPTION OF RIGHTS PUTS ALL GUARANTEES
AT RISK, JEOPARDIZING LIBERATION

Sparer 84
[Ed, Prof. Law and Soc Welfare @ Pennsylvania, Fundamental Human Rights, Legal Entitlements, and the
Social Struggle: A Friendly Critique of the Critical Legal Studies Movement, 36 Stan. L. Rev. 509, January,
ln//uwyo-ajl]

Kennedy, however, adds a second reason for agreeing with Klare [*525] rather than Lynd: "[T]he left
doesn't need a counter-theory that ends with rights" because "our program for the future must emerge
dialectically from our past, rather than as a deduction from it." n38 This point causes me some concern.
Kennedy is no longer talking about rights theory but about rights themselves. His refusal to develop a
counter-theory which "ends" with rights is due not merely to the inadequacy of rights alone to protect
and imprve the workers' situation; that could be achieved by making clear that much more is needed,
even for the adequate functioning of the rights themselves. Rather, his refusal is based on a disavowal of
an ongoing (one might say "principled") commitment to rights. n39

What "our program for the future" is must emerge "dialectically" (rather than as a "deduction" from our
past). Does this mean that Kennedy's "future program" may not include the right of working people to
organize? This very possibility is why -- given the deductions from past history and present experience
discussed later in this essay -- some of us feel it is appropriate to make a principled commitment to the
legal right of working people to organize and engage in concerted activities, just as we would make a
commitment to the right to dissent. We cannot trust future programs that emerge "dialectically," but
which are not based on at least limited deductions from our past.

Page 595 of 1481


Page 596 of 1481
Turns: Minorities
CLS DISEMPOWERS MARGINALIZED GROUPS WHO USE LEGAL DISCOURSE IN
TRANSFORMATIVE WAYS

Phyllis Goldfarb, Associate Professor, Boston College Law School, New England Law Review, Spring, 1992, 26
New Eng.L. Rev. 683

Viewed through Minow's eyes, rights talk represents a demand for public airing that makes pre-existing
conflicts "audible and unavoidable." It is a "process by which hurts that once were whispered or unheard
have become claims, and claims that once were unsuccessful, have persuaded others and transformed
social life." Rights, Minow argues, can remake relationships; in relating her view, Minow helps us remake
our relationship to rights. This transformative approach to rights, adopted by movements of the
disempowered, is a view that feminist scholars and scholars of color have urged proponents of Critical
Legal Studies to embrace. The foregoing descriptions comprise content-oriented critiques of certain CLS
theories. Feminists and minorities would offer a methodological critique as well, a critique rooted in
sensitivity to the methods by which one builds theory. Each has implicitly and explicitly criticized certain
CLS literature for its contextual failures, its inattention to the specific ways that diverse groups of people
experience society and feel its impact in their everyday lives. Each would contribute to CLS a theory-
building epistemology grounded in political struggle, attentive to the conditions in which people live, and
inclusive of the perspectives they express. The infusion of these diverse perspectives, especially from the
voices of the disempowered, and attention to political practice are likely to affect CLS theories. For
feminists and critical race scholars, this infusion of voices and involvement in practice represent a moral
and epistemological imperative for a transformative project aimed at reducing hierarchy.

Page 597 of 1481


Page 598 of 1481
Turn: Working in System Good
(1/2)
THERES NO ALTERNATIVE TO RIGHTS STRATEGIES THAT WILL SOLVE DEMANDS
CAN ONLY BE MADE USING THE INSTITUTIONAL LOGIC OF THE PRESENT SYSTEM
DEMANDS FOR RIGHTS CREATE INSTITUTIONAL CRISES THAT CAUSE REAL
REFORMS

Crenshaw, Law Professor at UCLA, 88 (Kimberle Williams, RACE, REFORM, AND RETRENCHMENT:
TRANSFORMATION AND LEGITIMATION IN ANTIDISCRIMINATION LAW, Harvard Law Review, May, 101 Harv. L. Rev.
1331)

The Critics' product is of limited utility to Blacks in its present form. The implications for Blacks
of trashing liberal legal ideology are troubling , even though it may be proper to assail belief structures that obscure liberating
possibilities. Trashing legal ideology seems to tell us repeatedly what has already been established -- that legal discourse is unstable and relatively indeterminate. Furthermore,

trashing offers no idea of how to avoid the negative consequences of engaging


in reformist discourse or how to work around such consequences. Even if we imagine the wrong world when we think in terms of legal discourse, we
must nevertheless exist in a present world where legal protection has at times been a
blessing -- albeit a mixed one. The fundamental problem is that, although Critics criticize law because it
functions to legitimate existing institutional arrangements, it is precisely this
legitimating function that has made law receptive to certain demands in this area.
The Critical emphasis on deconstruction as the vehicle for liberation leads to the conclusion that engaging in legal discourse should be avoided because it reinforces not only
the discourse itself but also the society and the world that it embodies. Yet Critics offer little beyond this observation. Their focus on delegitimating

rights rhetoric seems to suggest that, once rights rhetoric has been discarded,
there exists a more productive strategy for change , one which does not reinforce existing patterns of domination.
Unfortunately, no such strategy has yet been articulated, and it is difficult to imagine that racial minorities will ever
be able to discover one. As Frances Fox Piven and Richard Cloward point out in their [*1367] excellent account of the civil rights movement, popular struggles

are a reflection of institutionally determined logic and a challenge to that logic.


n137 People can only demand change in ways that reflect the logic of the

institutions that they are challenging. n138 Demands for change that do not
reflect the institutional logic -- that is, demands that do not engage and subsequently reinforce the
dominant ideology -- will probably be ineffective. n139 The possibility for
ideological change is created through the very process of legitimation , which is triggered by
crisis. Powerless people can sometimes trigger such a crisis by challenging an institution

internally, that is, by using its own logic against it. n140 Such crisis occurs when
powerless people force open and politicize a contradiction between the
dominant ideology and their reality. The political consequences [*1368] of maintaining the contradictions may sometimes force an
adjustment -- an attempt to close the gap or to make things appear fair. n141 Yet, because the adjustment is triggered by the political consequences of the contradiction, circumstances will be
adjusted only to the extent necessary to close the apparent contradiction. This approach to understanding legitimation and change is applicable to the civil rights movement. Because Blacks
were challenging their exclusion from political society, the only claims that were likely to achieve recognition were those that reflected American society's institutional logic: legal rights

ideology. Articulating their formal demands through legal rights ideology, civil rights protestors exposed

Page 599 of 1481


a series of contradictions -- the most important being the promised privileges of American citizenship and the practice of absolute racial subordination.
Rather than using the contradictions to suggest that American citizenship was itself illegitimate or false, civil rights protestors proceeded as if American citizenship were real, and demanded to

By seeking to restructure reality to reflect American


exercise the "rights" that citizenship entailed.

mythology, Blacks relied upon and ultimately benefited from politically inspired efforts to
resolve the contradictions by granting formal rights . Although it is the need to maintain legitimacy that presents
powerless groups with the opportunity to wres

Page 600 of 1481


Page 601 of 1481
Turn: Working in System Good
(2/2)
WEVE GOT TO WORK THROUGH THE SYSTEM TO CHANGE IT.

Andrew Sullivan, Editor of the New Republic, Virtually Normal, 1995, p. 88-91

Moreover, a cultural strategy as a political strategy is a dangerous one for a minority-and a small minority
at that. Inevitably, the vast majority of the culture will be at best uninterested. In a society where the
market rules the culture, majorities win the culture wars. And in a society where the state, pace Foucault
actually does exist, where laws are passed according to rules by which the society operates, culture, in
any case, is not enough. It may be necessary, but it is not sufficient. To achieve actual results, to end
persecution of homosexuals in the military, to allow gay parents to keep their children, to provide basic
education about homosexuality in high schools, to prevent murderers of homosexuals from getting
lenient treatment, it is necessary to work through the, very channels Foucault and his followers revile. It is
necessary to conform to certain disciplines in order to reform them, necessary to speak a certain language
before it can say something different, necessary to abandon the anarchy of random resistance if actual
homosexuals are to be protected. As Michael Walzer has written of Foucault, he 11 stands nowhere and
finds no reasons, Angrily he rattles the bars of the iron cage. But he has no plans or projects for turning
the cage into something more like a human home." The difficult and compromising task of interpreting
one world for another, of reforming an imperfect and unjust society from a criterion of truth or reasoning,
is not available to the liberationists. Into Foucault's philosophical anarchy they hurl a political cri de coeur.
When it eventually goes unheard, when its impact fades, when its internal nihilism blows itself out, they
have nothing left to offer. Other homosexuals, whose lives are no better for queer revolt, remain the
objects of a political system which the liberationists do not deign to engage. The liberationists prefer to
concentrate-for where else can they go?-on those instruments of power which require no broader
conversation, no process of dialogue, no moment of compromise, no act of engagement. So they focus on
outing, on speech codes, on punitive measures against opponents on campuses, on the enforcement of
new forms of language, by censorship and by intimidation. Insofar, then, as liberationist politics is cultural,
it is extremely vulnerable; and insofar as it is really political, it is almost always authoritarian. Which is to
say it isn't really a politics at all. It's a strange confluence of political abdication and psychological violence.

Page 602 of 1481


Page 603 of 1481
Indeterminacy False (1/4)
THE LAW REASONABLY GUIDES IMPLIMENTATION EVEN IN HARD CASES

Solum 87
[Lawrence B., Assoc. Prof. Law @ Loyola, On the Indeterminacy Crisis: Critiquing Critical Dogma, 54 U.
Chi. L. Rev. 462, Spring, ln//uwyo-ajl]

What then is the truth about indeterminacy? There is certainly room for dispute, but as practical
boundaries for the debate, three conclusions are firm. First, legal doctrine underdetermines the results in
many, but not all, actual cases. That is to say that aside from the easiest cases, aspects of the outcome are
rule-guided but not rule-bound. For example, in the most routine cases, the amount of a traffic fine or of
a damage award may vary within some range. Second, although there may be some cases in which the
result is radically underdeterminate, in the sense that any party could "win" under some valid
interpretation of legal doctrine, it does not follow that the doctrine itself is indeterminate over all cases.
For example, the three-pronged test for impermissible state establishment of religion, articulated in
Lemon v. Kurtzman, n97 is often criticized as highly underdeterminate. But, in spite of any uncertainty
about some applications of the Lemon test, we can be quite sure that a court applying the Lemon test
would strike down any law giving parochial school teachers a pay raise out of state funds. n98 Third, it is
pure nonsense to say that legal doctrine is completely indeterminate even with respect to very [*495]
hard cases. Even in the hardest hard case, legal doctrine limits the court's options. One of the parties will
receive a judgment, not some unexpected stranger; the relief will be related to the dispute at hand and
will not be a declaration that Mickey Mouse is the President of the United States.

EVIDENCE OF INDETERMINACY IS FLAWED: SELECTION BIAS

Solum 87
[Lawrence B., Assoc. Prof. Law @ Loyola, On the Indeterminacy Crisis: Critiquing Critical Dogma, 54 U.
Chi. L. Rev. 462, Spring, ln//uwyo-ajl]

Furthermore, one of the primary criteria for inclusion in a casebook may be indeterminacy itself:
practically indeterminate cases may be useful pedagogically because they can be used to illustrate both
the methods and limits of formal legal reasoning as well as the role of principle and policy. The
generalization that the law is practically indeterminate may thus stem from the predominance of such
examples in the materials with which legal scholars work on a daily basis. n104

Finally, critical legal scholars have a strong practical motive for belief in the indeterminacy thesis. If one
believes that the rules are strongly determinate, but fundamentally wrong, one is left with very little room
to maneuver within the limited horizons of legal scholarship. The notion that it is possible to achieve
radical results working with the existing body of legal doctrine -- because the seeming constraints are
illusory -- has powerful attraction for those committed to social change, but whose professional lives are
confined to the academy and not the capitol buildings.

Page 604 of 1481


Page 605 of 1481
Indeterminacy False (2/4)
INDETERMINACY IS AN UNPROVABLE FARCE THE LAW IS ONLY
UNDERDETERMINED AND USUALLY WORKS

Solum 87
[Lawrence B., Assoc. Prof. Law @ Loyola, On the Indeterminacy Crisis: Critiquing Critical Dogma, 54 U.
Chi. L. Rev. 462, Spring, ln//uwyo-ajl]

[*475] This confusion between indeterminacy and underdeterminacy is also reflected in Duncan Kennedy's definition of formalism in his early essay, Legal Formality: "The
essence of rule application, as I have defined it above, is that it is mechanical. The decision process is called rule application only if the actor resolutely limits himself to

Kennedy has defined rule application in such a way that


identifying those aspects of the situation which, per se, trigger his response." n47

only a completely determined decision will count as a decision that is not indeterminate. The difficulty with this
definition is that legal rules (or, more broadly, doctrines) can significantly constrain outcomes even if they do not

mechanically determine them.

My general argument against the internal skeptic's defense is that underdeterminacy is not the same as indeterminacy and that a case need
not be indeterminate to be hard. With all this in mind, I can agree with critical scholars that there are some cases that appear easy on their surface but are actually hard. But the
internal skeptics believe that by demonstrating that easy cases are hard cases, they have also demonstrated that the law is indeterminate. At this stage in the argument, I part
company with these advocates of indeterminacy.

, the internal skeptic cannot demonstrate that all law is indeterminate through
For two principal reasons

conventional legal argumentation. The first reason is conceptual: if a decision is not determinate, it does not follow
that it is also not underdeterminate and, therefore, indeterminate. Neither does it follow that because a case is hard, it is
indeterminate. Even if all seemingly easy cases were actually hard cases, it would not follow that the law is indeterminate with respect to all these cases -- although it would
follow that the law is less determinate than we might have thought. Hard cases can be very hard, even if their results are not completely indeterminate. I submit further that
even the hardest of hard cases are merely underdetermined by the law, not indeterminate. But I defer discussion of this point until later in this essay.

there are at
The second reason internal skepticism cannot prove complete indeterminacy is rooted in the standards implicit in the practice of acceptable legal argument:

least some very easy cases that are completely determinate. For example, if I were sued by Gore Vidal for slander on the basis of the
first paragraph of this [*476] article, the only possible outcome would be a verdict for me. A skeptic might respond that it is possible to think

of an argument suggesting that I should lose the case, or that the judge could simply rule against me
without explanation. But it is simply incredible to say that any such argument or arbitrary ruling would be
considered acceptable by the legal profession. That is, this sort of defense of indeterminacy is not internal to the law. It may, however, have some
critical bite -- a matter I turn to in the following discussions of external skepticism and the epiphenomenalist defense.

THE VAST MAJORITY OF CASES ARE DETERMINATE YOU JUST DONT HEAR
ABOUT THEM

Hegland 85
[Kenney, Prof. of Law @ Arizona, Goodbye to Deconstruction, 58 S. Cal. L. Rev. 1203, July, ln//uwyo-ajl]

Let us return to the mundane -- can legal doctrines determine the outcome of specific legal controversies?
I think the acne case establishes, at least in theory, that legal rules and doctrines can determine outcomes
and that they can constrain judicial discretion and immunize decisions from subjective preference. But
even if I have won my quarrel theoretically, I have not done much to save the legal order if all I have
shown is that legal doctrine determines outcome only in what I must now concede to be the most
ridiculous of hypotheticals. What of the real world of judges, lawyers and clients? Does doctrine
determine outcome there?

Page 606 of 1481


My sense is that legal doctrines determine the outcomes in most cases. I do not believe this is due to the
litigants' lack of imagination or resources. It is because doctrines are not mirages; they have real
substance and are what they appear to be.

Law professors teach the difficult cases of the casebooks, read the novel cases of the advance sheets, and
fret over "major" Supreme Court decisions. Law professors overestimate the degree of legal uncertainty. I
teach a course in contracts, and last summer I took a week to read every appellate decision in my home
state dealing with that subject over the last several years. It is, by and large, boring stuff: "The rule is X,
the facts are Y, and therefore we hold for the plaintiff." I realize that in the process of writing an opinion
an uncertain case may become certain. Nonetheless, in most of the opinions I read, there was simply no
sign of doctrinal uncertainty: seldom were there dissenting opinions, seldom were cases distinguished,
and seldom did the court discuss "social policy" to convince the reader that the legal doctrine should
apply. Typically, the doctrine was recited and then applied. It was a long week.

Now it may be that, for some dark or benign purpose, the judges of Arizona are out to hoodwink us, or for
perhaps some climatic reason, Arizona lawyers have been made dumb and their clients poor. But, if my
reading of the cases is fair, I think that as an empirical matter the deconstructionists have some explaining
to do -- and it will not do to simply assert, rather than prove, that Sun-Belt lawyers lack imagination and
resolve.

Page 607 of 1481


Page 608 of 1481
Indeterminacy False (3/4)
INDETERMINACY DOES NOT MEAN WE CANNOT MAKE REASONABLE PREDICTIONS

Robert Gordon, Professor of Law, Stanford University, Stanford Law Review, January, 1984, 36 Stan. L. Rev.
57,

The other argument rests, I think, on a misunderstanding of what the Critics mean by indeterminancy.
They don't mean -- although sometimes they sound as if they do -- that there are never any predictable
causal relations between legal forms and anything else. As argued earlier in this essay, there are plenty of
short- and medium-run stable regularities in social life, including regularities in the interpretation and
application, in given contexts, of legal rules. Lawyers, in fact, are constantly making predictions for their
clients on the basis of these regularities. The Critical claim of indeterminacy is simply that none of these
regularities are necessary consequences of the adoption of a given regime of rules. The rule-system could
also have generated a different set of stabilizing conventions leading to exactly the opposite results and
may, upon a shift in the direction of political winds, switch to those opposing conventions at any time.

YOUR INDETERMINACY ARGUMENT ASSUMES THE PAST DONT MANIFEST IN


FUTURE DECISIONS- PAST MISTAKES GUIDE FUTURE- ADDITIONALLY, YOUR
ARGUMENT THAT LAW CANNOT HAVE A POSITIVE EFFECT IS WRONG- POWER
DISTRIBUTION PROVES

ALTMAN 90
(Andrew, Prof of Philosophy @ Georgia State, 1990, Critical Legal Studies: A Liberal Critique, Pg. 177-178)
PHM

Ultra-theory relies, in fact, upon a seriously flawed conception of social reality and rests upon several
fallacious inferences. We may begin the criticism of it with a point to which I have already alluded,
concerning the issue of whether the social past can control the social future. The CLS ultra-theorist
correctly believes that the social past can never guarantee the character of the social future. It is never a
necessary truth that the social world will continue to turn in the way it has been turning up to now.
However, ultra-theorists fallaciously infer from this that the social past cannot control the social future,
that social rules cannot constrain and channel human social behavior and thought. This inference is a
fallacy because control is always a matter of degree; it may never reach the point of constituting a
necessary connection between past and future, but it does not follow that there is no control. 35

CLS ultra-theorists have been led astray here by an ill-conceived reliance on the metaphysical categories
of contingency and necessity. They reason that the social future is contingent, that it does not have to be
a certain way; in particular, it does not have to be a repetition of the social past. They fallaciously
conclude that the social past can exert no control over the social future. Underlying this fallacious
inference is the mistaken belief that there can be a relation of control between x and y only if x's
prescription that y behave in a certain way necessarily leads to y behaving in that way.

Page 609 of 1481


Moreover, the ultra-theorist's view that control requires necessary connections contradicts his own view
that one individual can control another. Recall that the CLS ultra-theorist denies that social rules have the
power to control the behavior and thought of individuals but that he simultaneously affirms that
individuals (e.g., slaveowners) can control other individuals (e.g., their slaves). Yet the ultra-theorist
argument explaining why rules cannot control individuals also defeats the possibility of individuals
controlling other individuals. Nothing makes it impossible for slaves to revolt, for workers to rebel, for the
oppressed to rise up. The ultra-theory argument would force one to conclude that masters exert no
control over slaves, bosses no control over workers, the oppressors no control over the oppressed. These
conclusions are flatly inconsistent with the claims of CLS ultra-theorists, in addition to being wholly
implausible. The conclusion to draw from the fact that the oppressed can revolt at any time is not that the
oppressors do not exert control over them but that the control is not total. And exactly the same
conclusion should be drawn about social rules: The fact that such rules can be trashed at any moment
does not show that they exert no control, only that the control is not total.

Page 610 of 1481


Page 611 of 1481
Indeterminacy False (4/4)
EVEN IF THEYRE RIGHT, THAT ONLY MEANS THAT JUSTIFICATIONS ARE
DISPARATE LEGAL OUTCOMES ARE STILL DETERMINATE

Solum 87
[Lawrence B., Assoc. Prof. Law @ Loyola, On the Indeterminacy Crisis: Critiquing Critical Dogma, 54 U.
Chi. L. Rev. 462, Spring, ln//uwyo-ajl]

I pause now to examine this argument in some detail. It is easy to agree that existing legal rules are not fully determined by any unified and consistent social theory. Even if we
had a fully satisfactory theory justifying the broad outlines of the modern state, it would be hard to argue that any such theory required a particular set of legal rules, much less

it does not follow from this admission that critical scholars have made out a
the precise set of rules we have now. However,

case for complete indeterminacy of justification. Some specific legal rules may necessarily follow from a broad
social theory; many legal rules may be incompatible with a given theory.

[*467] Moreover, indeterminacy of justification does not entail indeterminacy in a set of legal rules. n16 A number

of competing theories could be used to justify or critique a wide range of legal doctrines, while the legal
doctrines themselves nonetheless would constrain the outcome of particular cases. n17 For example, one could make
consequentialist arguments for and against the doctrine of promissory estoppel, while the doctrine itself remained determinate in application. Of course, if (as is often the case)
the justification for a rule is used to guide its application, indeterminacy of justification will lead to greater indeterminacy of legal outcomes. n18

Page 612 of 1481


Page 613 of 1481
A2 Language Makes Law
Indeterminate: 2AC
LINGUISTIC INDETERMINACY CAN GO EITHER WAY, CANCELLING OUT ANY EFFECT
ON THE LAW

Solum 87
[Lawrence B., Assoc. Prof. Law @ Loyola, On the Indeterminacy Crisis: Critiquing Critical Dogma, 54 U.
Chi. L. Rev. 462, Spring, ln//uwyo-ajl]

But the difficulty with appealing to Wittgenstein's skeptical paradox is that it costs the indeterminacy
thesis its critical bite. Wittgenstein makes the following observation:

This was our paradox: no course of action could be determined by a rule, because every course of action
can be made out to accord with the rule. The answer was: if everything can be made out to accord with
the rule, then it can also be made out to conflict with it. And so there would be neither accord nor conflict
here. n56

Thus, we may admit the paradox but reject its significance because it has no consequences for human
conduct. Like other skeptical paradoxes, it has no existential force. As Saul Kripke puts it, "It holds no
terrors in our daily lives." n57

My argument, therefore, relies on the distinction between logical and practical possibility. This distinction
can be illuminated by a brief discussion of an analogous problem with epistemological skepticism. An
epistemological skeptic might claim that we can never really know anything. An anti-skeptic might
respond with an [*479] example of an "easy case" of knowledge: you know that you are currently sitting
in a chair and reading this peculiar article. The skeptic might respond by raising a skeptical possibility: for
all you know you are only a brain in a vat being manipulated by an evil scientist to think you are sitting
and reading this essay, when in fact you are doing neither of these things. n58

Very roughly, it is my view that rule-skepticism can be shown to be toothless for the same reason that this
sort of epistemological skepticism is toothless: worrying about being a brain in a vat will not have any
effect on what you do. Likewise, worrying about rule-skepticism will not have any effect on the way cases
are decided. The skeptical possibilities invoked by both rule-skepticism and epistemological skepticism are
not practical possibilities, and only practical possibilities affect the way one acts.

Page 614 of 1481


Page 615 of 1481
CLS Recreates Oppression (1/2)
TURN- CLS CONFLATES AND CREATES OPPRESSION; WE MUST CRITICIZE FROM THE
OPPRESSEDS PERSPECTIVE

Anthony Cook Professor of Law, Critical Race Theory, Ed. Kimberl Crenshaw et al, 1995, p. 89-90

The third problem with the CLS critique is that it threatens to conflate the unique histories of the various
forms of alienation and oppression engendered by the subconscious acceptance and assimilation of
liberal ideology. the experiences of racism and sexismto name but twoare certainly related to the
way individuals experience liberalism as oppressive but cannot be reduced to that experience. Therefore,
exploration of the various histories of oppression, often ignored by CLSs account can provide an essential
basis for any reconstructed community. Finally, deconstruction should ultimately lead to a reconstructive
vision, which will involve some line-drawing and boundary-setting. CLS should not only explain why
liberalisms boundary-setting is problematic; it must also suggest how to redraw those boundaries to
satisfy other goals. I believe CLS too often falls victim to a myopic preoccupation with the limited role of
theoretical deconstruciton and a too narrowly tailored experiential deconstruction that focuses
exclusively on how individuals experience liberalism. Hegemonic ideologies are never maintained by
logical consistencey alone; knowledge of the full range of conditions under which they remain oppressed,
exposes new problems and possibilities. When one begins to contemplate how alternative visions of
community might look and be implemented, one must consider carefully the view from the bottomnot
simply what oppressors say but how the oppressed respond to what they say. The view from the bottom
may offer insights into why individuals accept their subordinate status in society despite the illogic and
inconsistency of the dominant ideology.

Page 616 of 1481


Page 617 of 1481
CLS Recreates Oppression (2/2)
TRASHING THE LEGAL SYSTEM LEAVES OPPRESSION INTACT AND RESULTS IN REAL
WORLD SUFFERING THE RISK OF OUR IMPACT IS WORSE THAN THEIRS

Goldfarb, Associate Law Professor at Boston College, 92 (Phyllis, A DIVERSITY OF INFLUENCE: From the Worlds
of "Others": Minority and Feminist Responses to Critical Legal Studies, New England Law Review, Spring, 26 New Eng.L.
Rev. 683)

Some CLS theories imply that the use of rights rhetoric by people of [*694] color to try to remove some of the harshest manifestations of racial domination exemplifies a false legal
consciousness or a counterproductive faith in the power of liberalism to produce social change. n34 Certain critical race theorists have responded that this implication stems from a

The decision to pursue a


misapprehension of the options for genuine social struggle open to the socially, economically and politically dispossessed. n35

rights strategy may well represent a conscious and critical assessment of the
constraints imposed by the conditions of racial subjugation . n36 The denial of this possibility may itself
represent a form of false consciousness. n37 As Crenshaw observes: "In the context of white supremacy, engaging in

rights discourse should be seen as an act of self-defense." n38 [*695] Richard Delgado suggests that
rights can protect minorities from those who, in the absence of legal
sanctions, would feel freer to act upon racist impulses . n39 Although certain CLS scholars despair of the vision of
atomized individuals that underlies rights language, n40 Delgado states that minorities , who regularly experience the intrusions of oppression, value the

distance that rights place between themselves and others . n41 Such distance
offers a measure of safety from race-based violence, contempt and abuse. In the
sort of informal community that some CLS writers prize, a community operating by fluid and flexible exercises

of discretion unbounded by rights and rules , n42 Delgado wonders what


structures would protect minorities from racist behavior . n43 For minorities, Delgado indicates,
abandoning formality may mean abandoning security, making the informal
community a setting of disproportionate vulnerability for people of color . n44 These
different attitudes about rights and rule structures are vividly portrayed in a story related by Patricia Williams. In renting an apartment in New York City, Williams insists on a conventional
lease to demonstrate her trustworthiness, while Peter Gabel, her white male colleague, demonstrates his trustworthiness by avoiding a lease and engaging in an informal conversational
transaction. n45 Williams rejects the CLS critique of legalism and formality not because it is inaccurate, but because it voices a single perspective that grows from a particular social experience,
ignoring the experiences of other social groups. n46 Her conclusion is that we should not abandon rights language for all purposes, but that we should "listen intently to each other," to
"bridge the experiential distance" between us, n47 and to "attempt to become multilingual in the semantics of each others' rights-valuation." n48 Robert Williams also ties differential rights-
valuation to the social experiences of different social groups. Williams asserts that CLS theory [*696] has underestimated peoples of color when it worries that they have come to believe in
the "truth" of rights rather than in the simple instrumental character of attaining rights. n49 From the standpoint of the empowered, Williams observes, rights represent abstract,

from the standpoint of the subordinated, rights have a more palpable


metaphysical concepts, but

reality: One cannot experience the pervasive, devastating reality of a "right," . . . except in its absence. One must first be denied that seat on the bus, one must see the desecration of
one's tribe's sacred lands, one must be without sanitary facilities in a farm field, to understand that a "right" can be more than a concept. A right can also be a real, tangible experience. . . .
What else could a right be other than an abstraction to someone who has never had their abstractions taken away or denied. . . . Arising from the historical experience of peoples of color in
United States society "concepts" such as "rights" or "justice" assume a life of their own in an experiential sense. It is in this struggle for the tangible benefits of these "concepts" that peoples of
color mobilize themselves to forge their own discourse. Unavoidably and irredeemably derivative in part of the majority society's discursive practices . . . . this type of discourse which finds its
genesis in the historical struggles of peoples of color strategically employs those concepts, such as "rights," which speak most directly and forcefully to the prejudices of the dominant culture.

n50 Because differences in rights-valuation grow out of their different social experiences, many people of color reject the CLS critique of rights
consciousness in its present form. They view the CLS emphasis on delegitimating legal ideology as a project

that relinquishes too much, since appeals to legal ideology represent one of
the only strategies that has effectively elicited a response to the desperate
needs of subordinated people. Minority scholars seem to read CLS rights critiques simply as cautionary tales about the dangers of engaging liberal
ideology, while they continue to make realistic decisions, given the limited array of options, to risk such engagement. Debunking legal ideology may

indeed meet the needs of those who experience oppression primarily in terms
of feelings of alienation from community. People of color, however, have

Page 618 of 1481


often chosen another strategy, for regardless of the status of legal
consciousness, they have identified racism as an ideology more threatening to
their lives

Page 619 of 1481


Page 620 of 1481
CLS is Nihilistic
CLS COLLAPSES INTO NIHILISM

Johnson 84
[Phillip E., Prof. Law @ Berkeley, Critical Legal Studies Symposium: Do You Sincerely Want To Be
Radical? 36 Stan. L. Rev. 247, January, ln//uwyo-ajl]

The seemingly sophisticated tendency of Critical scholars to see "politics" at the root of every practice is
also unsatisfying. Politics deals with the accommodation and adjustment of claims backed by power, and
to see nothing but politics in law is to adopt the claim of Thrasymachus that justice is the will of the
stronger. n110 That amounts to nihilism, which is a coherent position only if one is prepared to accept the
implication that might makes right. It is clear that the Critical scholars do not want to accept that
implication, which, after all, would make them very wrong indeed. They want to escape the impasse of
nihilism by liberating themselves from an inherited burden of false consciousness that makes hard choices
seem inevitable.

Page 621 of 1481


Page 622 of 1481
No Alternative (1/2)
CLS HAS NO HARD REFERENCE, PREVENTING THE CONSTRUCTION OF A REALISTIC
ALTERNATIVE

Johnson 84
[Phillip E., Prof. Law @ Berkeley, Critical Legal Studies Symposium: Do You Sincerely Want To Be
Radical? 36 Stan. L. Rev. 247, January, ln//uwyo-ajl]

The second major problem with a purely negative use of Marxism is that criticism itself is meaningless
without a standard of reference, whether express or implied. Critical scholars who describe "capitalist"
society as oppressive or hierarchical are like New Yorkers who speak of Cleveland as being in the "West."
Contemporary capitalist society may be oppressive and hierarchical judged by some ideal standard and
yet have less oppression and hierarchy than most or even all other societies that have ever existed.
Critical legal writing systematically evades the question, "Compared to what?"

My point is not that one always has to propose an alternative [*261] when one criticizes, but rather that
failure to specify the standard of reference robs the criticism of meaning. When Critical scholars say that
life in a capitalist society is alienating, I do not know if they mean that this is true because of some
particular characteristic of capitalist society or because life in every known from of society is alienating. If
the latter is the case, then blaming alienation on capitalism is absurd.

In a word, the relationship of Critical legal though to Marxism or any other ideological position is obscure.
Without a firm ideological basis the Critical viewpoint is itself obscure, and indeed it is not easy to explain
how Critical scholarship differes from "liberal" or "traditional" scholarship, except in its greater obscurity.
n42 Liberal scholarship itself is strongly Critical, and may even have prepared the way for nihilism by
undermining so much that had seemed certain.

THE CRITICAL LEGAL ALTERNATIVE IS SO VAGUE THAT IT JUSTIFIES MAINTAINING


THE STATUS QUO

Johnson 84
[Phillip E., Prof. Law @ Berkeley, Critical Legal Studies Symposium: Do You Sincerely Want To Be
Radical? 36 Stan. L. Rev. 247, January, ln//uwyo-ajl]

There is no mystery about what the Critical legal scholars are against: They are against capitalism,
liberalism, and illegitimate hierarchy. It is much harder to say what they are for. In fact, Critical legal
writing has practically nothing to suggest in the way of a positive political program. For a movement that
claims to be political, this is truly an astonishing vacuum. At the 1981 Yale Symposium on Legal
Scholarship, for example, Duncan Kennedy called for "utopian speculation," "dreaming up the ways we
think things might be better than they are," because radicals need to ask, " What would we do with
power, anyway?" n89 On the same occasion, Alan Freeman chided his colleagues for failing to follow
through on the radical implications of their papers. The most he could propose himself, however, was that
radicals should escape from liberal thinking by incorporating "insights from other methods: structuralism,

Page 623 of 1481


phenomenology, advanced Marxist thought, radical empiricism, and comparative methods." n90 Roberto
Mangabeira Unger concluded his book Law in Modern Society by observing that the solution to the
conflict between personal autonomy and community "could be fully worked out only with the help of a
metaphysics we do not yet possess." n91 Whatever may have been their authors' intentions, the political
[*282] implications of these messages seem concervative to me. If we not only don't know how to get
there from here, but also don't know where "there" is, doesn't it follow that we should stay here until
more information comes along?

Page 624 of 1481


Page 625 of 1481
No Alternative (2/2)
CLS HAS NO ALTERNATIVE, REPLACING POLITICAL ACTION WITH USELESS
DREAMING

Johnson 84
[Phillip E., Prof. Law @ Berkeley, Critical Legal Studies Symposium: Do You Sincerely Want To Be
Radical? 36 Stan. L. Rev. 247, January, ln//uwyo-ajl]

The Critical scholars sincerely want to be radicals: Indeed, some of them formed their standards of right
and wrong in a counterculture that associated radical politics with goodness itself and identified liberalism
with "selling out." They are also aware that the existing legal order is not as securely founded upon reason
as some people like to pretend. Unfortunately, they do not have a radical alternative to propose. Their
strategy in this awkward situation is to retreat into a mystical utopianism that is couched in political
language but in fact has little to do with politics. The "incoherence" of liberalism is their incoherence, its
"failure" their failure. Critical legal writing provides a way of sounding like a radical when you don't know
how to be one.

THERE IS NO ALTERNATIVE TO LAW- IT IS THE MOST COHERENT WAY TO SETTLE


SOCIAL ISSUES

ALTMAN 90
(Andrew, Prof of Philosophy @ Georgia State, 1990, Critical Legal Studies: A Liberal Critique, Pg. 119) PHM

There are serious problems with this CLS view of the implications of the patchwork thesis. Even if there
are incompatible principles that underlie different segments of doctrine, it does not follow that the judge
is free to choose which principle to rely on in deciding a case. Recall from the discussion in chapter 2 that
our legal culture incorporates a convention that requires that cases be decided in a way that provides the
greatest degree of logical coherence with the settled rules and decisions. Suppose that in most cases a
decision relying on a particular principle fits better with the settled materials than one relying on a
competing principle. The supposition is not inconsistent with the patchwork thesis, but if it is true, then it
would be wrong to claim, as Dalton does, that equally forceful legal arguments could be given for both
sides in almost any case. The better legal argument would be the one that displays the better fit with the
settled decisions and norms, and the law itself would be highly determinate, even if the patchwork thesis
were true.

CLS HAS NO RECONSTRUCTIVE VISION

Anthony Cook Professor of Law, Critical Race Theory, Ed. Kimberl Crenshaw et al, 1995, p. 89-90

Page 626 of 1481


The third problem with the CLS critique is that it threatens to conflate the unique histories of the various
forms of alienation and oppression engendered by the subconscious acceptance and assimilation of
liberal ideology. the experiences of racism and sexismto name but twoare certainly related to the
way individuals experience liberalism as oppressive but cannot be reduced to that experience. Therefore,
exploration of the various histories of oppression, often ignored by CLSs account can provide an essential
basis for any reconstructed community. Finally, deconstruction should ultimately lead to a reconstructive
vision, which will involve some line-drawing and boundary-setting. CLS should not only explain why
liberalisms boundary-setting is problematic; it must also suggest how to redraw those boundaries to
satisfy other goals. I believe CLS too often falls victim to a myopic preoccupation with the limited role of
theoretical deconstruciton and a too narrowly tailored experiential deconstruction that focuses
exclusively on how individuals experience liberalism. Hegemonic ideologies are never maintained by
logical consistencey alone; knowledge of the full range of conditions under which they remain oppressed,
exposes new problems and possibilities. When one begins to contemplate how alternative visions of
community might look and be implemented, one must consider carefully the view from the bottomnot
simply what oppressors say but how the oppressed respond to what they say. The view from the bottom
may offer insights into why individuals accept their subordinate status in society despite the illogic and
inconsistency of the dominant ideology.

Page 627 of 1481


Page 628 of 1481
Alternative Fails: Elitism
THE ALTERNATIVE IS ELITIST SELF-PRESERVATION, SHORT-CIRCUITING ANY
RADICAL POTENTIAL

White 84
[G. Edward, Critical Legal Studies Symposium: The Inevitability of Critical Legal Studies, 36 Stan. L. Rev.
649, January, ln//uwyo-ajl]

. In calling for the transformation of social institutions, they were calling


The Critical theorists therefore cannot have it both ways

for the transformation of a world in which they have been comfortable and prominent. Few of the
designated beneficiaries of their cells for change share their close identification with a hierarchical
educational system in which the most prestigious members of the hierarchy get the fewest apparent demands made on their time. How many members of the
oppressed classes would applaud a world in which persons designated law professors got paid rather well for teaching five hours a week, or perhaps not at all? How many
would be inclined to think that persons living that kind of life have any idea what it means to be oppressed? And while some Critical theorists might willingly work one month out

. There are
of a year as janitors or secretaries, others might not like to have their salaries equalized even with other law professors, let alone with maintenance workers

powerful forces of self-preservation operating to retard the impact of transformative proposals, and when
one adds to those forces a newly emergent skepticism about the wisdom of elites, one can readily
imagine a scenario in which Critical legal scholars preach their transformative proposals to audiences
wearing headsets.

CRITICAL LEGAL THEORY IS INSULATED WITHIN THE ACADEMY, REINSCRIBING


CAPITAL

Johnson 84
[Phillip E., Prof. Law @ Berkeley, Critical Legal Studies Symposium: Do You Sincerely Want To Be
Radical? 36 Stan. L. Rev. 247, January, ln//uwyo-ajl]

nothing is more vulnerable to a Marxist critique than the CLS movement itself. Most of these
The irony is that

scholars are law professors at prestigious universities, predominantly at Harvard and Stanford; such a career implies
acceptance by the legal intellectual establishment. From this platform they preach a sort of nihilistic
utopianism, a most unconvincing doctrine that in no way threatens the existing order of society. Their
visibility at the elite universities lends credibility to the image of neutrality and tolerance that the Ruling
Hegemony wishes to project. Their rhetoric reassures law students that the only alternatives to the
present system are "utopian." The obvious Marxist explanation of the CLS movement is that it permits a few harmless academic
leftists to adopt a radical pose, while receiving good salaries and excellent fringe benefits for serving the
interests of the capitalists. n54

YOUR ALTERNATIVE HAS BEEN FRACTURED- CLS IS ONLY COMPREHENSIBLE


WITHIN ELITE CIRLCES AND IT IS DEAD

Page 629 of 1481


NEASCU 00
(Dana, Former Asst. Corporate Counsel, 8 J.L. & Pol'y 415, CLS stands for Critical Legal Studies, if Anyone
Remembers, MosE)

Critical Legal Studies ("CLS"), n1 which started as a Left movement within legal academia, n2 has undergone so many [*416] changes,
that one may liken it to products of pop culture, such as the television cartoon show, South Park. n3 South Park
features a character named Kenny, totally unlike any other cartoon hero, tragic or otherwise. Like Kenny, who is an outsider and who speaks a

language unintelligible to all except, astonishingly, his classmates, CLS no longer seems to possess a voice
comprehensible to anyone outside its own small circle. Kenny, unlike all other cartoon figures, dies in every episode. n4 Significantly,
often Kenny's death has been self-inflicted - though not necessarily intentional - when, for instance, he
ignores warnings of imminent danger. Like Kenny, CLS has suffered many often self-inflicted injuries. Like
South Park, generally, CLS is certainly colorful, but often little more than that and, as in the cartoon, except for the certainty of Kenny's death and later resurrection, there

seems more flash than substance in its existence. We are left to guess whether CLS will prove to be as resilient after apparent death, as Kenny .

Page 630 of 1481


Page 631 of 1481
Alternative Fails: Fractures
Movement
YOUR ALTERNATIVE FAILS- FRACTURED LEFT

NEASCU 00
(Dana, Former Asst. Corporate Counsel, 8 J.L. & Pol'y 415, CLS stands for Critical Legal Studies, if Anyone
Remembers, MosE)

As a result of this array of dissenting and conflicting interests, CLS has been left with no cohesive voice,
and it appears now as a mere witness to the powerless atomization of an emasculated radical Left
discourse. This atomization may have promoted certain group solidarities, and possibly offered short term
relief. But, despite CLS's influence on legal discourse, it never seemed able to attain even a partially-
unified leftist discourse. This failure might be the cause of mutual estrangement among all of its
"members" - or at least a failure to offer a common core - that eventually risks oblivion for the movement
as a whole. In response, CLS now must rediscover its voice in the legal community, even though the old
leftist habits and texts have far less luster and glitter than fashionable literary theories.

Page 632 of 1481


Page 633 of 1481
Alternative Fails: Indeterminacy
Kills Criticism
CLSS FOCUS ON INDETERMINACY NEUTRALIZES CRITICISM OF THE LAW,
PREVENTING THE CREATION OF A NEW ORDER

Solum 87
[Lawrence B., Assoc. Prof. Law @ Loyola, On the Indeterminacy Crisis: Critiquing Critical Dogma, 54 U.
Chi. L. Rev. 462, Spring, ln//uwyo-ajl]

Far from enabling a progressive transformation of legal practice, the indeterminacy


But this appeal is superficial.

thesis, at least the strong version, disempowers the critique of legal ideology that critical scholars hope will facilitate
emancipatory social change. Seen in broad terms, their critique has two parts. First, the mystification thesis will unveil the structures of domination masked
by legal doctrine. Second, the indeterminacy thesis will explain how domination circumvents the apparent autonomy of the law and frees legal actors from the apparent
constraints imposed by the existing rules. Thus, mystification and indeterminacy are the intellectual foundations both for a program of external critique that will reveal the law to
the layman for what it is, and for an internal critique through which progressive legal actors will freely use legal practice to achieve emancipatory ends.

the strong indeterminacy thesis undercuts, rather than advances, the projects of both
My contention is that

internal and external critique. Because the strong indeterminacy thesis calls for disengagement from the
form and conventions of discourse that makes legal practice possible, the thesis blunts an internal critique
of the law. Stanley Cavell puts the point as follows:
The internal tyranny of convention is that only a slave of it can know how it may be changed for the better, or know why it should be eradicated. Only masters of a game, perfect
slaves to that project, are in a position to establish conventions which better serve its essence. This is why deep revolutionary changes can result from attempts to conserve a
project, to take it back to its idea, keep it in touch with its history. To demand that the law be fulfilled, every jot and tittle, will destroy the law as it stands, if it has moved too far
from its origins. Only a priest could have confronted his set of practices with its origins so deeply as to set the terms of Reformation. n105

Cavell's idea can be put into a legal context by examining the critical legal theory of Roberto Unger. Unger identifies "deviationalist doctrine" as the positive alternative for legal
scholarship. The project of deviationalist doctrine must maintain "the minimal characteristics of doctrine" that is "the willingness to take the extant [*499] authoritative materials
as starting points." n106 Like the Reformation, Unger's program acknowledges the structure from which it hopes to deviate. The indeterminacy thesis, however, undercuts the

. If there is a measure of determinacy in the law, and legal discourse and reasoning are more
project of deviationalist doctrine at its starting point

Unger's deviationalist doctrine begins with a flawed, but at least functional,


than mere apologies for domination, then

language with which to embark on the creation of a more humane legal order. But if the law is
indeterminate, and legal reasoning a sham, then they cannot serve as the raw material for constructing a
body of doctrine with emancipatory potential -- deviationalist doctrine itself would be incapable of
effecting real change. Instead, the social order would remain governed by the underlying ideology or
political and economic forces -- and if the forces were to change, then the doctrine would not need to do so. Under the strong indeterminacy thesis, legal
doctrine becomes "a wheel that can be turned though nothing else moves with it," and so it "is not part of the mechanism." n107

Page 634 of 1481


Page 635 of 1481
Alternative Fails: Historical
Record of Marxism
MARXISMS LONG HISTORY OF BLOODSHED AND OPPRESSION DELEGITIMIZES THE
FOUNDATION OF THEIR ALTERNATIVE CLAIMS

Johnson 84
[Phillip E., Prof. Law @ Berkeley, Critical Legal Studies Symposium: Do You Sincerely Want To Be
Radical? 36 Stan. L. Rev. 247, January, ln//uwyo-ajl]

A similar uncertainty about the cause of our ills is reflected in the uneasy relationship between Critical
legal scholarship and Marxism. Some of the articles from the CLS movement are explicitly Marxist, and the
movement as a whole employs Marxist jargon and methods of analysis. Marxist remedies, however, are
rarely recommended. Although they do not dwell upon the point, the Critical scholars seem to be aware
of the consistently horrible record of Marxist regimes n31 -- the slave labor camps, the mass deportations,
the suppression [*258] of labor unions, the denial of freedom of conscience, the bureaucratic rigidity,
the personality cults. They appear to recognize that refugee traffic between Marxist and non-Marxist
societies is a one-way affair.

Understandably, even radical scholars in a sophisticated intellectual community hesitate to embrace such
an inviting target for "Critical" scrutiny by others. This ambivalence can lead to amusing equivocations.The
prolific Mark Tushnet, for example, pays Marxism the compliment of saying that it "generates the central
position to which all theories of knowlede respond," n32 and he has tried his hand at sketching a Marxist
analysis of American public law. n33 But we must not assume that Tushnet is therefore a Marxist, for he
has also written that he "uses Marxism" merely as a "rhetorical mode" to show that he realizes that those
in positions of power will not peacefully relinquish those positions when the time comes, and to
demonstrate that he is a real radical and not just another reformer like John Hart Ely or Lawrence Tribe.
n34 How a rhetorical mode can generate a central position to which all theories must respond is not
explained.

What Tushnet and other Critical legal scholars seem to like about Marxism is its doctrine of historical
contingency, its insistence that "all knowledge is a social product and thus that knowledge can have no
transcendent validity." n35 This "Critical" side of Marxism is useful for attacking "capitalism" or
"liberalism" (although it could be equally useful in undermining Marxism itself), n36 and as such it can
[*259] be detached from the Marxist program of party dictatorship. Marxism as a practical revolutionary
program is attrative mainly to those who, like Dostoyevsky's Grand Inquisitor, n37 believe that the
important thing is to feed the hungry and that human liberty is worth sacrificing to that end. The Critical
scholars are well aware that man does not live by bread alone. Their primary concern is for social equality,
for abolition of hierarchies of power. n38 Marxist dictatorship is no solution to that problem.

But discarding the vulnerable positive program of Marxism generates at least two further difficulties,
neither of which has been adequately addressed in any of the Critical legal literature with which I am
familiar. First, how are we to judge the validity of a Marxist critique of capitalist society if Marxism is so
wrong in its positive program? There is an analogy here to the predicament of psychoanalytic theory that
the efficacy of psychoanalysis as a form of treatment has been strongly called into question. n39
Conceivably the Freudian theories of the personality might be true even if treatment based on those
theories has no special power to cure, but the power to cure has always been an important argument for
the truth of the theory. n40 The failure of Marxism as a remedy for exploitation and oppression is so
spectacular as to call into question its central doctrines, [*260] including the premise that economic or

Page 636 of 1481


political institutions are to blame for our psychological and spiritual ills. How are we to verify or falsify a
Marxist or Marxist-style analysis? Critical legal scholarship seems to rule the question out of order. We are
entitled to be suspicious, especially since Critical Theory appeals so powerfully to the egotism of
disaffected intellectuals like the CCLS members by granting them special insight and a pivotal role in
history. n41

Page 637 of 1481


Page 638 of 1481
Alternative Fails: Non-Rights
Strategies Bad
NON-RIGHTS STRATEGIES FAIL BECAUSE DOMINANT SOCIETY CAN MORE EASILY
IGNORE DEMANDS NOT MADE FROM WITHIN THE DOMINANT RIGHT DISCOURSE

Crenshaw, Law Professor at UCLA, 88 (Kimberle Williams, RACE, REFORM, AND RETRENCHMENT:
TRANSFORMATION AND LEGITIMATION IN ANTIDISCRIMINATION LAW, Harvard Law Review, May, 101 Harv. L. Rev.
1331)

Rights have been important. They may have legitimated racial inequality, but they have also been the means by which oppressed
[*1385] groups have secured both entry as formal equals into the dominant order and the survival of their

movement in the face of private and state repression. The dual role of legal change creates a dilemma for Black reformers. As long as race
consciousness thrives, Blacks will often have to rely on rights rhetoric when it is necessary to protect Black interests. The very reforms brought about by appeals to legal ideology, however,
seem to undermine the ability to move forward toward a broader vision of racial equality. In the quest for racial justice, winning and losing have been part of the same experience. The

Critics are correct in observing that engaging in rights discourse has helped to deradicalize and co-opt the
challenge. Yet they fail to acknowledge the limited range of options presented to Blacks in a context
where they were deemed "other," and the unlikelihood that specific demands for inclusion and equality
would be heard if articulated in other terms. This abbreviated list of options is itself contingent upon the ideological power of white race consciousness
and the continuing role of Black Americans as "other." Future efforts to address racial domination, as well as class hierarchy, must consider

the continuing ideology of white race consciousness by uncovering the oppositional dynamic and by
chipping away at its premises. Central to this task is revealing the contingency of race and exploring the connection between white race consciousness and the other
myths that legitimate both class and race hierarchies. Critics and others whose agendas include challenging hierarchy and legitimation must not overlook the importance of revealing the

until whites recognize the


contingency of race. Optimally, the deconstruction of white race consciousness might lead to a liberated future for both Blacks and whites. Yet,

hegemonic function of racism and turn their efforts toward neutralizing it, African-American people
must develop pragmatic political strategies -- self-conscious ideological struggle -- to minimize the costs of liberal reform
while maximizing its utility. A primary step in engaging in self-conscious ideological struggle must be to transcend the oppositional dynamic in which Blacks are cast
simply and solely as whites' subordinate "other." n200 The dual role that rights have played makes strategizing a difficult task. Black people can afford neither to resign themselves to, nor to

The subordinate position of Blacks in this society makes it


attack frontally, the legitimacy and incoherence of the dominant ideology.

unlikely that African-Americans will realize gains through the kind of direct challenge to the legitimacy of
American liberal ideology that is now being waged by Critical scholars. On the other hand, delegitimating [*1386] race consciousness would be directly relevant to Black
needs, and this strategy will sometimes require the pragmatic use of liberal ideology. This vision is consistent with the views forwarded by theoreticians such as Frances Fox Piven and Richard
Cloward, Antonio Gramsci, and Roberto Unger. Piven and Cloward observe that oppressed people sometimes advance by creating ideological and political crisis, but that the form of the crisis-
producing challenge must reflect the institutional logic of the system. n201 The use of rights rhetoric during the civil rights movement created such a crisis by presenting and manipulating the

Challenges and demands made from outside the institutional logic would
dominant ideology in a new and transformative way.

have accomplished little because Blacks, as the subordinate "other," were already perceived as being
outside the mainstream. The struggle of Blacks, like that of all subordinated groups, is a struggle for inclusion, an attempt to manipulate elements of the dominant ideology
to transform the experience of domination. It is a struggle to create a new status quo through the ideological and political tools that are available. Gramsci called this struggle a "War of

direct challenges to the dominant class


Position" and he regarded it as the most appropriate strategy for change in Western societies. According to Gramsci,

accomplish little if ideology plays such a central role in establishing authority that the legitimacy of the
dominant regime is not challenged. Joseph Femia, interpreting Gramsci, states that "the dominant ideology in modern capitalist societies is highly institutionalized
and widely internalized. It follows that a concentration on frontal attack, on direct assault against the bourgeois state ('war of movement' or 'war of

manoeuvre') can result only in disappointment and defeat ." n202 Consequently, the challenge in such societies is to create a counter-hegemony by
maneuvering within and expanding the dominant ideology to embrace the potential for change. Gramsci's vision of ideological struggle is echoed in part by Roberto Unger in his vision of

rather than discarding liberal legal ideology, we


deviationist doctrine. Unger, who represents another strand of the Critical approach, argues that,

should focus and develop its visionary undercurrents : [T]he struggle over the form of social life, through deviationist doctrine, creates opportunities
for experimental revisions of social life in the direction of the ideals we defend. An implication of our ideas is [*1387] that the elements of a formative institutional or imaginative structure

Liberal ideology embraces communal and liberating visions along with


may be replaced piecemeal rather than only all at once. n203

the legitimating hegemonic visions. Unger, like Gramsci and Piven and Cloward, seems to suggest that the strategy toward meaningful
change depends on skillful use of the liberating potential of dominant ideology .

Page 639 of 1481


Page 640 of 1481
Alternative Fails: Praxis (1/3)
CLSS FOCUS ON THEORY ALIENATES ITSELF FROM SOCIAL MOVEMENTS,
MAINTAINING DOMINATION

Sparer 84
[Ed, Prof. Law and Soc Welfare @ Pennsylvania, Fundamental Human Rights, Legal Entitlements, and the
Social Struggle: A Friendly Critique of the Critical Legal Studies Movement, 36 Stan. L. Rev. 509, January,
ln//uwyo-ajl]

"The philosophers have only interpreted the world, in various ways; the point, however, is to change it."
n108 The commitment to theory which contributes to, and in turn is informed and developed by, social
practice concerned with changing the world, is a theme occasionally encountered in Critical legal writing.
n109 In this essay, I use [*553] the word "praxis" to refer to such a unity of theory and action. n110 The
need for praxis should be self-evident to scholars such as those in Critical studies, whose view is that
domination and exploitation of human beings characterizes our social life, since mere tinkering with a
legal system misleads us. Therefore, fundamental transformation of social relations, including those
involved in the production process, is necessary. Richard Flacks, a sociologist, puts it this way:

[I]t seems urgent for academic radicals and Marxists to develop a more reflexive understanding of the
implications for anc relevance of their intellectual work to political practice. It may be a characteristic of
late capitalism that even Marxism can become nothing more than a token in the game of professional
achievement. n111

Despite such a warning, the practical relationship of Critical legal theory to social movement and struggle
in the United States today is, at best, very limited. Neither lawyers nor political activists receive much
enlightenment from Critical legal theory with regard to their actual work. Nor is Critical legal theory itself
much affected by the practical work of such people. While there are exceptions to these generalizations,
n112 the absence of praxis in current Critical legal work seems to be one of its most marked features.
Gordon, a Critical legal theorist, writes:

THE ALTERNATIVE IS REDUCTIONISTIC AND MIRED IN THEORY, PREVENTING THE


ORGANIZATION OF MOVEMENTS AGAINST OPPRESSION

Sparer 84
[Ed, Prof. Law and Soc Welfare @ Pennsylvania, Fundamental Human Rights, Legal Entitlements, and the
Social Struggle: A Friendly Critique of the Critical Legal Studies Movement, 36 Stan. L. Rev. 509, January,
ln//uwyo-ajl]

Gabel is entirely right when he insists on understanding people and social relations in the real, concrete,
specific world in which they exist. But surely that part of the concrete world he summarizes with such
eloquence is not "the social totality within which the psyche is formed." At least a fair number of people
do have experience with a more genuine, personal love. Some people do seek something better in "work"
than "mechanical functioning," at least when they are assured of a job to support their existence. People,

Page 641 of 1481


at least a fair number, are frequently dissatisfied with the "packaged emptiness" on which they spend
their wages. n126

I agree with Karl Klare when he writes: "I regard as inaccurate the view that . . . it is possible to describe
the working class as in any sense satisfied with current standards of living in either the material or cultural
aspects." n127 But if this is so, then it should be possible to struggle now over the conditions which Gabel
describes. Nevertheless, neither Gabel's work nor that of most other Critical legal theorists provides
theory that can aid such struggle. Indeed, it does not even recognize the need for new directions in
scholarship which [*560] would aid such struggle. In the course of constant efforts at delegitimation,
some Critical legal theorists begin to think and talk about "the law" as if it were no more than litigation,
doctrines, and case outcomes -- precisely the narrow view of most conventional legal theorists. Critical
theorists rarely conceive of legal strategies to employ outside the courtroom for the purpose of building
social movement.Somehow, the affirmative relationship of law to social movement becomes lost. n128

Page 642 of 1481


Page 643 of 1481
Alternative Fails: Praxis (2/3)
DELEGITIMIZING THE LAW CREATES HELPLESSNESS, DRIVING ACTIVISTS AWAY
FROM SOCIAL MOVEMENTS

Sparer 84
[Ed, Prof. Law and Soc Welfare @ Pennsylvania, Fundamental Human Rights, Legal Entitlements, and the
Social Struggle: A Friendly Critique of the Critical Legal Studies Movement, 36 Stan. L. Rev. 509, January,
ln//uwyo-ajl]

Therein lies the source of the real sadness (a word more accurate here than "cynicism") of some Critical
legal theorists who are also [*573] law school teachers. In some respects, of course, what we address
here is essentially a continuation of the praxis issue discussed in the last part. But it is more. The Critical
legal professors are not only scholars; they are also teachers.The people they teach are, in the main, not
going to be scholars. They are going to be practitioners. Do the Critical legal professors have anything to
say to these students -- except that they assume the students will discover in their practice those
successful methods of change which the teachers not only have not found but do not care to seek? The
more logical assumption, by far, is that such law teaching will be simply one more law school factor in the
decisions of students once concerned with social change to pursue corporate careers. What, after all, can
the student do as a lawyer in the face of monumental, overpowering, and all-pervasive injustice other
than pursue the same buck that everybody else does?

The radical law teacher's responsibility is not simply to expose doctrinal incoherencies and build historical
accounts. It is to point the way to a different kind of practice, one which utilizes that historical account.
The practice needed is not one which focuses primarily on the law school, however much change in the
law school is needed. It is a practice located "out there," in the world outside the law school, where
injustice, legal procedures and programs, incipient protest, and social movement constantly intermingle.
n174

[*574] The radical teacher's responsibility is to study such practice, analyze its conditions, and
demonstrate it, if need be, by personal example. When I say the "radical law teachers's" responsibility, I
do not mean, of course, the responsibility of each and every law teacher who professes a radical faith. Not
everybody does everything. I do mean that it is central to the tasks of radical law teachers, just as are the
activities and study Freeman espouses. Without at least a collegial relation to those engaged in social
movement practice and theory, the radical teacher will lead more students away from, rather than into,
the social struggle to reconstruct our world by democratizing our civil life.

CLS IS CUT OFF FROM PRACTICE, PREVENTING INSTITUTIONAL CHANGE

Sparer 84
[Ed, Prof. Law and Soc Welfare @ Pennsylvania, Fundamental Human Rights, Legal Entitlements, and the
Social Struggle: A Friendly Critique of the Critical Legal Studies Movement, 36 Stan. L. Rev. 509, January,
ln//uwyo-ajl]

Page 644 of 1481


The first reason that Critical legal scholarship contributes so little to those engaged in social change efforts and learns
delegitimation of liberal legal scholarship (which includes virtually all scholarship
so little from social change practice is its deeply held belief that

outside the Critical legal camp) is the principal contribution that it can make to significant change. The reasoning is that only by
breaking the hold that current liberal thinking has on our minds can we [*556] even begin to create a vision of the sort of society towards which we should be struggling.
Because the principal ideological support of our current social structure is liberalism, exposing that ideology is the obvious task for scholars seeking to end the oppression and
domination that characterize present society.

Not all Critical legal theorists subscribe to this formulation. Kennedy, for example, is insistent that "the critique of liberal legalism is only a small contribution to a valid strategy of
legal leftism." n115 He seeks "a unity of theory and practice" and has some specific suggestions as to what scholars might do in the law schools themselves. n116 But even he

This silence results because Kennedy and


has little to say about theory's use in transformative social struggle in the world outside the law schools.

many other Critical scholars agree with the crux of Freeman's formulation. They do not see what else theory can effectively
do, and thus they concentrate on the inadequacies of liberal doctrines (broadly defined) and on the ways liberal ideology
rationalizes the way things are. n117 But the situation remains unsatisfactory, and I cannot help but believe that some of the same Critical legal scholars who justify the divorce
of theory from the world of social struggle know this. They know this even when they seek to evade it.

Page 645 of 1481


Page 646 of 1481
Alternative Fails: Praxis (3/3)
CLS CANT CHALLENGE THE DOMINANT LOGIC

Kimberl Crenshaw , et al, Professor of Law, Columbia University, Critical Race Theory, Ed. Kimberl Crenshaw
et al, 1995, p. 111

The CLS emphasis on deconstruction as the vehicle for liberation leads to the conclusion that engaging in
legal discourse should be avoided because it reinforces not only the discourse itself but also the society
and the world that it embodies. Yet CLS scholars offer little beyond this observation: their focus on
delegitimating rights rhetoric seems to suggest that once rights rhetoric has been discarded, there exists a
more productive strategy for change, one that does not reinforce existing patterns of domination.
Unfortunately, no such strategy has yet been articulated, and it is difficult to imagine that racial minorities
will ever be able to discover one. As Frances Fox Piven and Richard Cloward point out in their excellent
account of the civil rights movement, popular struggles are a reflection of institutionally determined logic
and a challenge to that logic. People can demand change only in ways that reflect the logic of the
institutions they are challenging. Demands for change that do not reflect the institutional logicthat is
demands that do not engage and subsequently reinforce the dominant ideologywill probably be
ineffective.

ABSENSE OF A PROGRAM MEANS CLS CAN NOT HELP THE OPPRESSED

Harlon Dalton, Professor of Law, Yale University, Critical Race Theory, Ed. Kimberl Crenshaw et al, 1995, p. 80

After acknowledging the practitioner/theorist split, Gordon observes: It is not not at allthat the
practitioners are against theory. (Remember, practitioners equals people of color.) They are
hungry for theory that would help make sense of their practices; that would order them meaningfully into
larger patterns of historical change or structures of social action; that would help to resolve the perpetual
dilemma of whether it is or is not a contradiction in terms to be a radical lawyer, whether one is
inevitably corrupted by the medium in which one works, whether ones victories are in the long run
defeats or ones defeats victories; or that would suggest what tactics, in the boundless ocean of meanness
and constraint that surround us, to try next. I want to affirm that Gordons is a fair and accurate
description both of practitioners and of people of color. We hunger for theory. But, as Gordon goes on to
point out, there is the lingering and widespread suspicion within the CLS movement that the theorists do
not hunger for praxis. And it is this absence of a positive program on the part of many in CLS (with some
quite notable exceptions), and indeed the disdain for program by some, that is one of the central
difficulties that people of color have with the Critical Legal Studies movement. I think that this difficulty is
rooted in biography, in specific history, in what Cornel West refers to as genealogy.

Page 647 of 1481


Page 648 of 1481
A2 Thats Not Our
Indeterminacy Thesis: 1AR
FIRST, THE NEGS GENERALIZED LINKS PROVE THEY USE THE STRONG
INDETERMINACY THESIS. THATS THE ONLY WAY THEY CAN SHOW THAT PLAN IS
BAD WITHOUT SPECIFIC EV.

SECOND, THE STRONG THESIS IS ENDEMIC TO CLS

Solum 87
[Lawrence B., Assoc. Prof. Law @ Loyola, On the Indeterminacy Crisis: Critiquing Critical Dogma, 54 U.
Chi. L. Rev. 462, Spring, ln//uwyo-ajl]

The strong version of the indeterminacy thesis claims that all cases are "hard" cases or, more precisely,
that in every case any result can be derived from the preexisting legal doctrine. Although some critical
scholars have explicitly rejected the strong indeterminacy thesis, contemporary critical legal scholarship
still abounds in [*471] assertions that the law is radically indeterminate. In a recent article, for example,
Clare Dalton writes, "doctrinal inconsistency necessarily undermines the force of any conventional legal
argument, and . . . opposing arguments can be made with equal force. . . . [L]egal argumentation disguises
its own inherent indeterminacy. . . . [L]egal doctrine is unable to provide determinate answers to
particular disputes." n38 Giradeau Spann also affirms the strong version of the thesis: "[T]he
characteristics of [legal] doctrine that made it indeterminate in Chadha will make it indeterminate in all
other cases as well." n39 Likewise, Charles Yablon claims that "[t]he experienced advocate knows that the
doctrinal regime is sufficiently complex that there will always be some set of authoritative materials
which, through skillful manipulation of the level of specificity and characterization of the facts, he can
declare to be 'controlling' of the case at bar" in a way that supports "any position a client wishes to
maintain." n40

Page 649 of 1481


Page 650 of 1481
A2 Reification: 2AC
REIFICIATION ISNT INTRINSICALLY BAD ITS A NECESSARY TOOL TO PREVENT
FUTURE DOMINATION

Sparer 84
[Ed, Prof. Law and Soc Welfare @ Pennsylvania, Fundamental Human Rights, Legal Entitlements, and the
Social Struggle: A Friendly Critique of the Critical Legal Studies Movement, 36 Stan. L. Rev. 509, January,
ln//uwyo-ajl]

But is it bad to "reify"? In Marxist thinking, to reify a concept such as a right is to invest it with qualities
over and above those of the particular human beings who created or use it. It is as if the right had a life of
its own. It exists independently of the particular social setting from which it came and continues
regardless of the conscious choices of the people in a later setting.

Reification, as a general proposition, can have serious and negative consequences but not all "reifying" is
necessarily bad. It is true that when we characterize a certain legal right as "universal" or "inalienable,"
we are reifying it. But this may have a legitimate purpose. For example, we may fear that some group may
in the future dominate our society and attempt to stifle all dissent. We should protect as best we can
against such an event by today acknowledging that dissent is a human value that needs protection. In so
doing, we reify the legal right to dissent in order to protect the human right of self-expression and free
conscience. We should do the same with certain rights of working people. In spite of the difficulties of
drawing a "coherent" line as to what is "inalienable" and what is not, concern for the human values of
free conscience and mutual association, coupled with a deduction from history about what happens in the
absence of such legal rights, justifies such an effort. n42

Page 651 of 1481


Page 652 of 1481
A2 Rights Tradeoff: 2AC
THE POLITICAL RIGHTS OF PLAN ARENT ZERO SUM THEY SPILLOVER AND CREATE
MORE PROTECTIONS FOR EVERYONE

Sparer 84
[Ed, Prof. Law and Soc Welfare @ Pennsylvania, Fundamental Human Rights, Legal Entitlements, and the
Social Struggle: A Friendly Critique of the Critical Legal Studies Movement, 36 Stan. L. Rev. 509, January,
ln//uwyo-ajl]

In this manner, Marx develops the perspective, deeply imbedded in contemporary Critical legal thinking,
that the "inalienable rights" of each person, articulated in our Declaration of Independence and [*530]
the Constitution, are rights which subtract from those of other persons or, at best, separate people from
one another. n53 Even Lynd, in his dipute with Kennedy over liberal rights and the workers' struggles,
seems to accept this point of view. n54

While it is easy to understand how one person's right to separately possess property limits another
person's separate possession of property, I fail to see how one person's exercise of, for example, free
speech and dissent necessarily limits another person's. Quite the contrary; the exercise of these latter
rights can increase the next person's ability to exercise them. It is not the social legitimization which flows
from the formal recognition of rights thast inhibits transformative, humanizing social struggle. Many
factors impede such struggle. But rights such as free speech and dissent protect the ability of groups of
people -- including working people -- to change their society, better their group situation, and expand
their human freedom.

Page 653 of 1481


Page 654 of 1481
A2 Feminist Jurisprudence:
2AC
ESSENTIALIST FEMINISM REINFORCES GENDER STEREOTYPES THROUGH
VALORIZATION OF WOMENS DIFFERENCES, HARMING OURSELVES AND OUR
LISTENERS, AND KILLING THE TRANSFORMATIVE POWER OF THEIR CRITIQUE.

Iris Marion Young, Professor of Public and International Affairs at the University of Pittsburgh, Throwing Like a Girl
and Other Essays in Feminist Philosophy and Social Theory, 1990, p. 89-90

Within the context of antifeminist backlash, the effect of gynocentric feminism may be accommodating to
the existing structure. Gynocentric feminism relies on and reinforces gender stereotypes at just the time
when the dominant culture has put new emphasis on marks of gender difference. It does so, moreover,
by relying on many of those aspects of women's traditional sphere that traditional patriarchal ideology
has most exploited and that humanist feminists such as Beauvoir found most oppressive--reproductive
biology, motherhood, s domestic concerns. Even though its intentions are subversive, such renewed
attention to traditional femininity can have a reactionary effect on both ourselves and our listeners
because it may echo the dominant claim that women belong in a separate sphere.

Humanist feminism calls upon patriarchal society to open places for women within those spheres of
human activity that have been considered the most creative, powerful, and prestigious. Gynocentric
feminism replies that wanting such things for women implies a recognition that such activities are the
most humanly valuable. It argues that in fact, militarism, bureaucratic hierarchy, competition for
recognition, and the instrumentalization of nature and people entailed by these activities are basic
disvalues.24

Yet in contemporary society, men still have most institutionalized power, and gynocentric feminism shows
why they do not use it well. If feminism turns its back on the centers of power, privilege, and individual
achievement that men have monopolized, those men will continue to monopolize them, and nothing
significant will change. Feminists cannot undermine masculinist values without entering some of the
centers of power that foster them, but the attainment of such power itself requires at least appearing to
foster those values. Still, without being willing to risk such co-optation, feminism can be only a moral
position of critique rather than a force for institutional change.

Despite its intention, I fear that gynocentric feminism may have the same consequence as the stance of
moral motherhood that grew out of nineteenth century feminism a resegregation of women to a
specifically women's sphere, outside the sites of power, privilege, and recognition. For me the symptom
here is what the dominant culture finds more threatening. Within the dominant culture a middle-aged
assertive woman's claim to coanchor the news alongside a man appears considerably more threatening
than women's claim to have a different voice that exposes masculinist values as body-denying and selfish.
The claim of women to have a right to the positions and benefits that have hitherto been reserved for
men, and that male dominated institutions should serve women's needs, is a direct threat to male
privilege. While the claim that these positions of power themselves should be eliminated and the
institutions eliminated or restructured is indeed more radical, when asserted from the gynocentric
feminist position it can be an objective retreat.

Gynocentrisms focus on values and language as the primary target of its critique contributes to this
blunting of its political force. Without doubt, social change requires changing the subject, which in turn
means developing new ways of speaking, writing, and imagining. Equally indubitable is the gynocentric

Page 655 of 1481


feminist claim that masculinist values in Western culture deny the body, sensuality, and rootedness in
nature and that such denial nurtures fascism, pollution, and nuclear games. Given these facts, however,
what shall we do? To this gynocentrism has little concrete answer. Because its criticism of existing society
is so global and abstract, gynocentric critique of values, language, and culture of masculinism can remove
feminist theory from analysis of specific institutions and practices, and how they might be concretely
structurally changed in directions more consonant with our visions.

Page 656 of 1481


A2 Fem K of Intl Law: 2AC

FEMINISM HAS NO ALTERNATIVE TO INTERNATIONAL LAW

Hilary Charlesworth, Professor and Director of the Centre for International and Public Law, Faculty of Law,

Australian National University, April, 1999, The American Journal International Law 93 A.J.I.L. 379

[*379] I have mixed feelings about participating in this symposium as the feminist voice. On the one
hand, I want to support the symposium editors' attempt to broaden the standard categories of
international legal methodologies by including feminism in this undertaking. On the other hand, I am
conscious of the limits of my analysis and its unrepresentativeness -- the particularity of my nationality,
race, class, sexuality, education and profession shapes my outlook and ideas on international law. I clearly
cannot speak for all women participants in and observers of the international legal system. I also hope
that one day I will stop being positioned always as a feminist and will qualify as a fully fledged
international lawyer. My reservations are also more general because presenting feminism as one of seven
rival methodological traditions may give a false sense of its nature. The symposium editors' memorandum
to the participants encouraged a certain competitiveness: we were asked, "Why is your method better
than others?" I cannot answer this question. I do not see feminist methods as ready alternatives to any of
the other methods represented in this symposium. Feminist methods emphasize conversations and
dialogue rather than the production of a single, triumphant truth. n1 They will not lead to neat "legal"
answers because they are challenging the very categories of "law" and "nonlaw." Feminist methods seek
to expose and question the limited bases of international law's claim to objectivity and impartiality and
insist on the importance of gender relations as a category of analysis. The term "gender" here refers to
the social construction of differences between women and men and ideas of "femininity" and
"masculinity" -- the excess cultural baggage associated with biological sex.

FEMINISM MUST SPEAK THE LANGUAGE OF THE DOMINANT ORDER TO SUCCEED

Hilary Charlesworth, Professor and Director of the Centre for International and Public Law, Faculty

of Law, Australian National University, April, 1999, The American Journal International Law 93 A.J.I.L. 379

[*380] The philosopher Elizabeth Grosz has pointed out that feminist theorizing typically requires an
unarticulated balance between two goals. Feminist analysis is at once a reaction to the "overwhelming
masculinity of privileged and historically dominant knowledges, acting as a kind of counterweight to the
imbalances resulting from the male monopoly of the production and reception of knowledges" and a
response to the political goals of feminist struggles. n2 The dual commitments of feminist methods are in
complex and uneasy coexistence. The first demands "intellectual rigor," investigating the hidden gender
of the traditional canon. The second requires dedication to political change. The tension between the two
leads to criticism of feminist theorists both from the masculine academy for lack of disinterested
scholarship and objective analysis and from feminist activists for co-option by patriarchal forces through
participation in male-structured debates. n3 Feminist methodologies challenge many accepted scholarly
traditions. For example, they may clearly reflect a political agenda rather than strive to attain an objective
truth on a neutral basis and they may appear personal rather than detached. For this reason, feminist
methodologies are regularly seen as unscholarly, disruptive or mad. They are the techniques of outsiders
and strangers. Just as nineteenth-century women writers used madness to symbolize escape from limited

Page 657 of 1481


and enclosed lives, n4 so twentieth-century feminist scholars have developed dissonant methods to shake
the complacent and bounded disciplines in which they work. At the same time, most feminists are
constrained by their environment. If we want to achieve change, we must learn and use the language and
methods of the dominant order.

Page 658 of 1481


Page 659 of 1481
**CRT**

Page 660 of 1481


CRT Answers: 2AC (1/4)
FIRST, NO LINK WE DONT CLAIM TO USE THE LAW TO END RACISM. WE JUST
CREATE DUE PROCESS RIGHTS

SECOND, THE ALTERNATIVE IS WORSE BECAUSE IT ABDICATES TO THE STATUS


QUO, MAGNIFYING RACISM BY PROVIDING NO DUE PROCESS RIGHTS

THIRD, LIBERAL LEGALISM IS TRANSFORMATIVE TRASHING LEAVES OPPRESSION


INTACT

Crenshaw, Law Professor at UCLA, 88 (Kimberle Williams, RACE, REFORM, AND RETRENCHMENT: TRANSFORMATION AND LEGITIMATION
IN ANTIDISCRIMINATION LAW, Harvard Law Review, May, 101 Harv. L. Rev. 1331)

There are difficulties, however, in


The Critics offer an analysis that is useful in understanding the limited transformative potential of antidiscrimination rhetoric.

attempting to use Critical themes and ideas to understand the civil rights movement and to describe what
alternatives the civil rights constituency could have pursued, or might now pursue. While Critical scholars claim that their project is concerned with
domination, few have made more than a token effort to address racial domination specifically, and their work does not seem grounded in the reality of the racially oppressed. This deficiency
is especially apparent in critiques that relate to racial issues. Critical scholars have criticized mainstream legal ideology for its tendency to portray American society as basically fair, and
thereby to legitimate the oppressive policies that have been directed toward racial minorities. Yet Critical scholars do not sufficiently account for the effects or the causes of the oppression
that they routinely acknowledge. The result is that Critical literature exhibits the same proclivities of mainstream scholarship -- it seldom speaks to or about Black people. The failure of the

this failure leads to an


Critics to incorporate racism into their analysis also renders their critique of rights and their overall analysis of law in America incomplete. Specifically,

inability to appreciate fully the transformative significance of the civil rights movement in mobilizing Black
Americans and generating new demands. Further, the failure to consider the reality of those most oppressed by American institutions means that the Critical
account of the hegemonic nature of legal thought overlooks a crucial dimension of American life -- the ideological role of racism itself. Gordon, Freeman, Tushnet, and Gabel fail to analyze
racism as an ideological pillar upholding American society, or as the principal basis for Black oppression. The Critics' failure to analyze the hegemonic role of racism also renders their
prescriptive analysis unrealistic. In the spirit of Alan Freeman's declaration, Critics often appear to view the trashing of legal ideology "as the only path that might lead to a liberated future."

if trashing is the only path that might lead to a liberated future, Black people are unlikely to
n96 Yet [*1357]

make it to the Critics' promised land. n97 The Critics' commitment to trashing is premised on a notion that people are mystified by liberal legal ideology and
consequently cannot remake their world until they see how contingent such ideology is. The Critics' principal error is that their version of domination by consent does not present a realistic
picture of racial domination. Coercion explains much more about racial domination than does ideologically induced consent. n98 Black people do not create their oppressive worlds moment
to moment but rather are coerced into living in worlds created and maintained by others. Moreover, the ideological source of this coercion is not liberal legal consciousness, but racism. If
racism is just as important as, if not more important than, liberal legal ideology in explaining the persistence of white supremacy, then the Critics' single-minded effort to deconstruct liberal

Critics also disregard the


legal ideology will be futile. Finally, in addition to exaggerating the role of liberal legal consciousness and underestimating that of coercion,

transformative potential that liberalism offers. Although liberal legal ideology may indeed function to mystify, it remains receptive
to some aspirations that are central to Black demands, and may also perform an important function in
combating the experience of being excluded and oppressed . n99 This receptivity to Black aspirations is crucial given the hostile social world
that racism creates. The most troubling aspect of the Critical program, therefore, is that "trashing" rights consciousness may have the

unintended consequence of disempowering the racially oppressed while leaving white supremacy
basically [*1358] untouched. These difficulties are discussed below as they relate to the critiques of Gordon, Freeman, and Tushnet. I. Gordon: The Underemphasis on
Coercion. -- Robert Gordon's explanation of ideological domination illustrates how an exclusive focus on consent leaves gaping holes in his reader's understanding of hegemony. Gordon writes
that beliefs are "the main constraints upon making social life more bearable." n100 Yet how can others understand the fact that Black people, although unable to bring about a world in which
they fully participate, can imagine such a world? Clearly, something other than their own structure of thought prevents Blacks from changing their world. This fact suggests that a more

The coercive power of the state operates to


complete explanation of domination requires that coercion and consent be considered together.

suppress some groups, particularly when there is consensus among others that such coercion is warranted. Racism serves to single out Blacks
as one of these groups "worthy" of suppression. n101 Gordon, however, does not offer any way to understand this. If his exclusive focus on ideological
domination is to be taken literally, one is left believing that Black Americans are unable to change their world because they accept the dominant ideology and thus cannot imagine an
alternative existence. Yet to say that the beliefs of Black Americans have boxed them into a subordinate existence because of what they believe is to ignore the history of coercive racial
subordination. Indeed, it would be difficult for Blacks, given the contradiction between American fiction and Black American reality, to believe as much of the American mythology as whites

do. n102The most significant aspect of Black oppression seems to be what is believed about Black
Americans, not what Black Americans believe. Black people are boxed in largely because there is a consensus among
many whites that the oppression of Blacks is legitimate. This is where consensus and coercion can be understood together: ideology convinces
one group that the coercive domination of another is legitimate. It matters little whether the coerced group rejects the dominant

ideology [*1359] and can offer a competing conception of the world; if they have been labeled "other" by
the dominant ideology, they are not heard. n103 Blacks seem to carry the stigma of "otherness," which effectively
precludes their potentially radicalizing influence from penetrating the dominant consciousness . n104 If this is the

Page 661 of 1481


The challenge for Blacks may be to pursue strategies that
case, then Blacks will gain little through simply transcending their own belief structures.

confront the beliefs held about them by whites. For Blacks, such strategies may take the form of reinforcing
some aspects of the dominant ideology in attempts to become participants in the dominant discourse
rather than outsiders defined, objectified, and reified by that discourse. In this sense, the civil rights movement
might be considered as an attempt to deconstruct the image of "the Negro" in the white mind. By
forcing the political system to respond to Black demands, Blacks rejected images of complacency and
docility that had been invoked by some whites to dismiss Black demands . n105

Page 662 of 1481


Page 663 of 1481
CRT Answers: 2AC (2/4)
FOURTH, PREFER OUR SPECIFIC SOLVENCY TRIBE AND KATYAL INDICATE THAT
PLAN CHALLENGES CURRENT DETAINMENT PRACTICES

FIFTH, PERM DO BOTH


COMBINING MODERN NORMATIVE LEGAL THEORY WITH CRT ENABLES
RESISTANCE AGAINST ENDEMIC RACISM

Harris 94
[Angela P., Prof. Law @ Berkeley, The Jurisprudence of Reconstruction, 82 Calif. L. Rev. 741, July, LN//uwyo-ajl]

CRT's commitment to the liberation of people of color - and the project of critical social science (generally) and normative legal
scholarship (in particular) as a way to further that liberation - suggest a faith in certain concepts and institutions that
postmodernists lack. When race-crits tell modernist stories, they assume that "people of color" describes a coherent category with at least some shared values and interests.

They assume that the idea of "liberation" is meaningful - that racism is something that can one day somehow cease to exist, or cease to exert any
power over us . Modernist narratives assume a "real" reality out there, and that reason can bring us face to face
with it. And modernist narratives have faith that once enough people see the truth, right action will follow: that enlightenment leads to empowerment, and that
empowerment leads to emancipation.

people of color and whites live in the same perceptual and


Modernist narratives, then, are profoundly hopeful. They assume that

moral world, that reason speaks to us all in the same way despite our different experiences, and that reason, rather
than habit or power, is what will motivate people. Modernist narratives also can be profoundly romantic. They imagine heroic action by a formerly oppressed people rising up as
one, "empowered" to be who they "really" are or choose to be, breathing the thin and bracing air of freedom.

faith in reason and


This optimism and romanticism, though easy to caricature, cannot be easily dismissed. As Patricia Williams and Mari Matsuda have pointed out,

truth and belief in the essential freedom of rational subjects have enabled people of color to survive and
resist subordination. n63 Political modernism, more generally, has been a powerful force in the lives of
subjugated peoples; as a practical matter, politically liberal societies are [*754] vastly preferable to the
alternatives. n64 A faith in reason has sustained efforts to educate people into critical thinking and to
engage in debate rather than violence. n65 The passionate and constructive energy of modernist narratives of emancipation is also grounded in a
moral faith: that human beings are created equal and endowed with certain inalienable rights; that oppression is wrong and resistance to oppression right; that opposing

subjugation in the name of liberty, equality, and true community is the obligation of every rational person. In its modernist moments, CRT aims not to topple
the Enlightenment, but to make its promises real. n66

SIXTH, THIS ISNT OFFENSE THE FACT THAT WE CANT SOLVE EVERY PROBLEM
DOESNT MEAN WE SHOULDNT DO SOMETHING

SEVENTH, DE-POLITICIZATION OF LAW FAILS- MEANS A SEPERATION OF LAW AND


POLITICS THAT CREATS A STRUGGLE FOR STATE POWER- THIS MAKES THE ALT
POWER-DRIVEN AS OPPOSED TO DRIVEN FOR SOCIAL PROGRESS

Page 664 of 1481


ALFIERI ET AL 98
(Anthony, Law Prof and Director @ U Miami Law, Spring, BOOK REVIEW: Black And WhiteCritical Race
Theory: The Cutting Edge, La Raza Law Journal, l/n, MosE)

At bottom, the conflicts within CRT and the attacks upon it emanate from CRT's own growing antipathy
toward the traditional civil rights discourse that animates liberal race reform. To Critical Race theorists,
liberal faith in a court-driven, technocratic eradication of racial bias is misplaced. n33 Faith in the
rationality of progressive law reform, they argue, rests on principles of neutrality, objectivity, and value-
free reasoning. Obtaining a set of nonideological, regulative principles, however, requires a
depoliticization of the legal process. Depoliticization, in turn, compels the separation of law and politics.
When pushed outside the domain of liberal theory, CLS teaches, the conceptual separation of law and
politics collapses in the raw, delegitimating competition for state power. n34 Because of this material
inseparability, the depoliticization of law and the liberal state fails. In this way, the CRT politics of race
represents a complex variant of the CLS politics of law: power-driven, instrumental, and value-laden.

Page 665 of 1481


Page 666 of 1481
CRT Answers: 2AC (3/4)
EIGHTH, EXPERIENTIAL DECONSTRUCTION:
ORGANIC INTELLECTUALS MUST CONTEXTUALIZE CRITICISM IN THE CONTEXT OF
SPECIFIC OPPRESION, STRATEGICALLY USING HEGEMONIC NORMS TO CREATE
THEIR ALTERNATIVE ****

Cook 90
[Anthony E., Assoc Prof. Law @ Florida, Beyond Critical Legal Studies, 103 Harv. L. Rev. 985, March, LN//uwyo-ajl]

Because he appreciated the dialectic of theory and the broad-based confrontational strategies of socially
transformative action, King stands as the paradigmatic organic intellectual of twentieth-century American life. King's method and practice offer
direction to progressive scholars concerned about the exclusionary, repressive, and non-communal dimensions of American life.

[*1013] Gramsci's conception of the organic intellectual provides a useful framework for understanding the thought of King and what it has to offer CLS. The organic
intellectual brings philosophy to the masses, not for the merely instrumental purposes of unifying them, "but precisely in order to construct an intellectual-moral bloc which can
Gramsci's organic intellectual struggles
make politically possible the intellectual progress of the mass and not only of small intellectual groups." n78

to transform those who are oppressed as a means of transforming the conditions under which they are
oppressed. n79 Gramsci understands domination in terms of both coercion and consent, the latter constituting what he refers to as hegemony. Under his formulation,
hegemony consists, then, of "[t]he 'spontaneous' consent given by the great masses of the population to the general direction imposed on social life by the dominant
fundamental group." n80 Gramsci argues that "this consent is 'historically' caused by the prestige (and consequent confidence) which the dominant group enjoys because of its
position and function in the world of production." n81 Thus, oppression is not only physical and psychological but also cultural. n82

King, like Gramsci's organic intellectual, empowered his community through a practical effort to bridge the gap
between theory and lived experience. King's work consisted of four interrelated activities. First, he used theoretical
deconstruction to free the mind to envision alternative conceptions of community. Second, he employed
experiential deconstruction to understand the liberating dimensions of legitimating ideologies like liberalism and
Christianity, dimensions easily ignored by the abstract, ahistorical, and potentially misleading critiques that

rely exclusively on theoretical deconstruction. Third, he used the insights gleaned from the first two
activities to postulate an [*1014] alternative social vision intended to transform the conditions of oppression
under which people struggle. Drawing from the best of liberalism and the best of Christianity, King forged
a vision of community that transcended the limitations of each and built upon the accomplishments of
both. Finally, he created and implemented strategies to mobilize people to secure that alternative vision. I
refer to this multidimensional critical activity as "philosophical praxis."

Although many critical theorists engage primarily in theoretical deconstruction, and some appreciate certain forms of experiential deconstruction, n83 few have embraced either

reconstructive theorizing and socially


a full experiential deconstruction or the third and fourth dimensions of philosophical praxis --

transformative struggle. n84 These dimensions of critical activity directly confront the material conditions of oppression
whereas the preoccupation with deconstructing theory does not. King went further than these critical
theorists by examining the subtle and complex ways in which consent was shaped, while fully appreciating
the role of state and private coercion in legitimating authority in the lives of the oppressed.

King filtered his theoretical deconstruction of hegemonic theologies through his


This Part examines how

knowledge of the history and experience of oppression, and thereby made that theoretical deconstruction
richer, more contextual, and ready to engage the existential realities of oppression. The interplay between King's
theoretical and experiential deconstruction is best illustrated by reference to the African-American Church -- the institution providing the organic link between philosophy and the
masses, theory and praxis. n85

My analysis proceeds in four steps. First, I examine how African-American religion served at once to legitimate slave society, delegitimize that society, and inform alternative
visions of community. Second, I examine King's use of theoretical deconstruction and illustrate its dependence on the historic mission of the African-American Church. Like a
true organic intellectual engaged in a philosophical praxis, King used theoretical deconstruction to illustrate the possibilities [*1015] of his reconstructive vision and the
centrality of social struggle in realizing that vision. Third, I discuss King's experiential deconstruction, his unwillingness to be distracted by the reified abstractions of theoretical

the combination of theoretical and experiential deconstruction results in a more


deconstruction. Finally, I show how

contextual framework -- one more appreciative of the conditions of choice within which authority is
legitimated and challenged through reconstructive vision and struggle.

Page 667 of 1481


Page 668 of 1481
CRT Answers: 2AC (4/4)
NINTH, YOUR ALTERNATIVE IS FOCUSED ON A BLACK/WHITE DICHOTOMY- THIS
STOPS COALITIONAL POLITICS AND CREATES A COMPETITIVE DRIVE AMOUNG THE
DISENFRANCHISED FOR ATTENTION

HUTCHINSON 2K4
(Darren, Prof @ Wash College, August, 53 Am. U.L. Rev. 1187, American University Law Review, MosE)

A third area of critical race innovation involves multiracial politics. Internal critics have argued that racial
discourse in the United States fixates upon black/white racial issues, thereby marginalizing Latino, Native
American, and Asian American experiences. n95 Empirically, this observation is indisputable. Race
theorists lack a full understanding of the breadth of racial injustice. The inclusion of the experiences of
Latinos, Native Americans, and Asian Americans in racial discourse can improve CRT in several ways. First,
a multiracial discourse permits a full accounting of the problem of racial inequality and allows for the
construction of adequate remedies for racial subordination. n96 Although all people of color suffer
racism, often in similar ways, racial hierarchies impact communities of color in diverse ways. A narrow
focus on black/white subjugation severely limits the reach of antiracist remedies.
The black/white paradigm also prevents persons of color from engaging in coalition politics. n97 By
treating racism as a problem that affects blacks primarily (or exclusively), racial discourse in the United
States divides persons of color who could align to create formidable political forces in the battle for racial
justice.
Binary racial discourse also causes persons of color to compete for the attention of whites, as
marginalized racial groups treat racial justice as a [*1201] zero-sum game. n98 Instead of recognizing the
pervasiveness and complexity of racial injuries, binary racial discourse leads to the tyranny of oppression
ranking and to competing demands for centrality in a marginalized space of racial victimization.

Page 669 of 1481


Page 670 of 1481
#5 Perm: 1AR
WORKING WITHIN THE SYSTEM ALLOWS US TO TAKE IT DOWN- EDUCATION IS
PROOF THAT WE CAN EFFECTIVELY FIGHT RACISM

LADSON-BILLINGS 99
(Gloria, Prof @ U Wisconsin-Madison, Race isRace isnt, Pg. 23) PHM

Examples of pedagogical countermoves are found in the work of both Chicago elementary teacher Marva
Collins and Los Angeles high school mathematics teacher Jaime Escalante. Although neither Collins nor
Escalante is acclaimed as a "progressive" teacher, both are recognized for their persistence in believing in
the educability of all students. Both remind students that mainstream society expects them to be failures,
and prod them to succeed as a form of counterinsurgency. Their insistence on helping students achieve in
the "traditional" curriculum represents a twist on Audre Lorde's notion that one cannot dismantle the
master's house with the master's tools. Instead, they believe one can only dismantle the master's house
with the master's tools.

Page 671 of 1481


Page 672 of 1481
**Cuomo**

Page 673 of 1481


Preventing Nuke War Is a
Prerequisite to Positive Peace
PREVENTING NUCLEAR WAR IS THE ABSOLUTE PREREQUISITE TO POSITIVE PEACE

Folk, Prof of Religious and Peace Studies at Bethany College, 78 (Jerry, Peace Educations Peace Studies :
Towards an Integrated Approach, Peace & Change, Vol. V, No. 1, Spring, P. 58)

Those proponents of the positive peace approach who reject out of hand the work of researchers and educators coming
to the field from the perspective of negative peace too easily forget that the prevention of a nuclear
confrontation of global dimensions is the prerequisite for all other peace research, education, and action.
Unless such a confrontation can be avoided there will be no world left in which to build positive peace.
Moreover, the blanket condemnation of all such negative peace oriented research, education or action as a
reactionary attempt to support and reinforce the status quo is doctrinaire. Conflict theory and resolution, disarmament studies,
studies of the international system and of international organizations, and integration studies are in themselves
neutral. They do not intrinsically support either the status quo or revolutionary efforts to change or overthrow
it. Rather they offer a body of knowledge which can be used for either purpose or for some purpose in
between. It is much more logical for those who understand peace as positive peace to integrate this
knowledge into their own framework and to utilize it in achieving their own purposes. A balanced peace
studies program should therefore offer the student exposure to the questions and concerns which occupy those who view the field
essentially from the point of view of negative peace.

Page 674 of 1481


Page 675 of 1481
Negative Peace Key to Positive
Peace
NEGATIVE PEACE IS A PRECONDITION FOR POSITIVE PEACE VIOLENCE IS
SOMETIMES NECESSARY TO ACHIEVE THESE GOALS

Sandole, Professor of Conflict Resolution and International Relations at George Mason U, 96


(Dennis J. D., Conflict Resolution, USIA Electronic Journals, Vol. 1, No. 19, December,
usinfo.state.gov/journals/itps/1296/ijpe/pj19sand.htm)

Negative peace, however, does not go far enough; it is one part -- albeit, often an essential part -- of a larger process that is rarely attempted
-- and if attempted, rarely achieved -- by traditional diplomacy. The remaining part consists of " positive peace": the elimination of the underlying
structural causes and conditions that have given rise to the violent conflict which negative peace processes seek to contain. To put it simply, negative
peace deals with symptoms of underlying problems -- "putting out fires" -- while positive peace deals with
the underlying, "combustible" problems themselves. Why doesn't traditional diplomacy deal with positive peace? One reason is that
diplomats are trained in dispute settlement -- reaching agreements about how to establish negative peace -- without, good intentions to the contrary,
necessarily addressing the underlying problems that gave rise to the disputes that are being settled. Hence, negotiations to end wars or to control or reduce
armaments, resulting in treaties or other agreements, are efforts to halt or manage actual or threatened violence resulting from conflicts without necessarily
dealing with their underlying, deep-rooted causes and conditions. [CONTINUES] The stage has been set for this: NATO, under U.S. leadership, established
the North Atlantic Cooperation Council in 1991 and the Partnership for Peace in 1994, to reach out to, and collaborate with, its former Warsaw Pact
adversaries. These developments are a powerful sign that the Cold War is over and therefore, by implication, that nations are undergoing a shift from a narrow
world view based on national security to a comprehensive one based on common security. Hence, the United States and its security partners are conceptually
able to move beyond negative into positive peace. What this will entail in Bosnia is for the United States and its NATO and other partners to remain there long
enough to ensure that negative peace holds. At the same time, they should work with international governmental and nongovernmental (including conflict
With secure negative peace as a
resolution) organizations, and with the conflicting parties, to pursue, achieve, and maintain positive peace.
point of departure, positive peace in Bosnia begins with the reconstruction of the country. But lest the United States and its
partners repeat the failure of the European Union to achieve positive peace in the Bosnian city of Mostar through substantial investments in rebuilding
Mostar's infrastructure, this reconstruction must reflect a comprehensive peacebuilding strategy -- reconciliative as well as physical -- over a period of time.
Some frameworks that could be useful in guiding U.S.-led activities in this regard are: the "contingency model" of Ron Fisher and Loraleigh Keashly, which
matches an intervention with the intensity of a given conflict, and then follows up with other interventions designed to move the parties toward positive
peace; the "multi-track framework" of IMTD's Ambassador John McDonald and Louise Diamond, which combines the resources of nongovernmental conflict
resolution practitioners with those of the business and religious communities, media, funders, and others as well as governmental actors, in the pursuit of
positive peace; and my own design for a "new European peace and security system" which combines elements of these and other frameworks within the
context of the OSCE. There is a working hypothesis implicit in all this: by
expanding their options to include cooperative
processes geared to positive peace as well as competitive processes associated with negative peace, the
United States and its partners will enhance their prospects for success in dealing with the deep-rooted
intrastate ethnic and other conflicts that seem to be the dominant form of warfare in the post-Cold War
world. Intervening in such conflicts may mean "taking casualties," particularly in cases where one party is
attempting to impose a genocidal "final solution" on another, as in Rwanda or Bosnia. In such situations, the use
of an appropriate amount of force to achieve negative peace may be a necessary (but not sufficient)
condition of positive peace. We should not, in such cases, allow the U.S. experience in Somalia to prevent us from
acting. Genocide in Rwanda or Bosnia does, sooner or later, affect the interests of the United States and others. The use of such extreme violence to
"resolve" conflicts anywhere in the world is not only morally reprehensible, but constitutes a model for others to emulate, perhaps increasing the costs of
dealing with it later on. The
implicit emphasis here on early warning and early action is part of the gist of conflict
resolution: being proactive instead of reactive. A proactive approach to problem solving worldwide is in the U.S. national interest. This
means, among other things, pursuing a bipartisan U.S. foreign policy to avoid the necessity of having to issue unrealistic timelines in any future deployment of
forces, plus paying the massive U.S. debt to the United Nations so that the United States can more credibly and effectively lead in the debate over U.N. reform
as well as in efforts to craft effective international responses to problems worldwide.

Page 676 of 1481


Page 677 of 1481
Absolutism Bad
ABSOLUTIST REJECTIONS ARE ULTIMATELY UNPRODUCTIVE WE MUST EMBRACE
THE DIFFERENCES IN PEACE THEORY IN ORDER TO ACHIEVE COMMON GOALS

Folk, Prof of Religious and Peace Studies at Bethany College, 78 (Jerry, Peace Educations Peace
Studies : Towards an Integrated Approach, Peace & Change, Vol. V, No. 1, Spring, P. 59)

The conflicting positions held by various researchers, educators, and activists in the peace studies field can be seen as
complementary rather than contradictory. Tensions, disagreements, and arguments of considerable intensity are
unavoidable and indeed desirable in this as in other fields of endeavor. Such dialectical tensions ensure a depth and
breadth of perception which one position alone could not produce. Truth is often paradoxical, and
therefore a dialectical approach to it is most appropriate. Antagonisms insure that the dialectic is kept alive.
They introduce a third dimension into one's understanding of truth and preserve it from petrification and
sterility. Therefore, premature closures, mutual excommunications, and fixations on a particular but
incomplete position or approach should be avoided. On the other hand, there may indeed be some fringe groups or persons in the
field who, by the ultimate and legalistic commitment to a particular approach or ideology and the absolute rejection of
any other ideas or approaches, call their legitimacy as peace researchers, educators or activists into question. An absolutistic
commitment to the status quo would be one example. Absolutistic and rigid commitments to the capitalist, Marxist
or liberal democratic systems might be another. Rigid and fanatic loyalty to a particular revolutionary or reformist
tradition or to the reformist or revolutionary tradition itself would be a third. None of the approaches or positions with regard to peace studies
which this paper discusses, however, are identical with any of these ideological orientations. Moreover, it is time particularly in the peace studies field,
that the ultimate value commitments of individuals and groups be given more weight than their politics and
philosophical preferences. The preference of one individual or group for Marxist socialism might be based on
precisely the same value commitments which have led another to prefer liberal democracy. In summary, a well-
balanced peace studies program ought to involve researchers, educator and activists. At all three levels, it ought to include some
participants who approach the field primarily from the standpoint of negative peace and others who approach it
using primarily the positive peace paradigm. Among the latter group some should be highly sympathetic to the radical revolutionary
tradition and others more in sympathy with the reformist approach of liberal democracy. Moreover, through the structure and interactions of the
program not only the tension and conflicts but also the positive interrelationships between these various groups ought
to become visible. A program structured according to such principles would admittedly be difficult to construct and even more difficult to administer.
It would, however, be more that merely comprehensive. It would be a microcosm of the world and therefore a laboratory in
which to experiment with the actual building of creative peace among groups and individuals of the most
divergent persuasions.

Page 678 of 1481


**Deep Ecology**
Permutation Solvency: 2AC

HUMAN INTERFERENCE IS INEVITABLE ECO-PRAGMATISM INTEGRATES


DISPARATE ENVIRONMENTAL APPROACHES, BETTER SOLVING ANTHRO BY
UNITING HUMAN AND ENVIRONMENTAL WELL-BEING

Mintz 2004
[Joel A., Prof. law @ Nova Southeastern University, Some Thoughts on the Merits of Pragmatism as a
Guide to Environmental Protection, 31 B.C. Envtl Aff. L. Rev. 1, LN//uwyo-ajl]

Environmental pragmatism is a relatively new direction in modern philosophy. n34 A product of the late 1980s and 1990s, it attempts to
connect the precepts and methods of philosophical pragmatism to the solution of real environmental
issues. n35
The most comprehensive collection of essays by environmental pragmatists may be found in Environmental Pragmatism, edited by Andrew Light and Eric Katz. n36 In their
introduction to this work, Light and Katz accurately observe that environmental pragmatism refers to "a cluster of related and overlapping concepts," as opposed to a single
view. n37 They note that it may take at least four distinct forms:

(1) examinations into the connection between classical American philosophical pragmatism and environmental issues; (2) the articulation of practical strategies for bridging
gaps between environmental theorists, policy analysts, activists, and the public; (3) theoretical investigations into the overlapping normative bases of specific environmental
organizations and movements in order to provide grounds for the convergence of activists on policy choices; and (4) general arguments for theoretical and meta-theoretical
moral pluralism in environmental normative theory. n38

What all of the environmental pragmatist approaches share, however, is a rejection of the view that "adequate and workable environmental ethics must embrace non-
anthropocentrism, holism, moral monism, and, perhaps, a commitment to some form of intrinsic value." n39

[*7] For Kelly Parker, the principal insight of environmental pragmatism is that "the human sphere is embedded at every point in the

broader natural sphere, that each inevitably affects the other in ways that are often impossible to predict,
and that values emerge in the ongoing transactions between humans and environments." n40 Parker defines environment as "the field where experience occurs, where my life
and the lives of others arise and take place." n41 He believes that pragmatism commits us to treating all places where experience unfolds, i.e., all environments, with "equal

, people are encouraged to "restructure our social institutions" so


seriousness." n42 Moreover, under Parker's pragmatic approach

that the public is afforded "a real voice in determining the kinds of environments we inhabit." n43

, human
Like Parker, Sandra B. Rosenthal and Rogene A. Buckholz also emphasize the organic unity of the individual embedded in his or her environment. n44 To them

beings are biological creatures, part of, and continuous with, nature. n45 In light of this, the philosophical
argument over anthropocentrism is meaningless since no real line may be drawn between human and
environmental well-being. n46 Rosenthal and Buckholz see the "systematic focus" of pragmatism as being on "science as method, or as lived through human
activity, on what the scientist does to gain knowledge." n47 Humans exist in the world as active experimenters who create knowledge and formulate ethical values by
integrating "potentially conflicting values and viewpoints." n48

Another leading environmental pragmatist, Bryan G. Norton, also advocates a pluralistic approach. n49 In Norton's opinion:

The goal of seeking a unified, monistic theory of environmental ethics represents a misguided mission, a mission that was formulated under a set of epistemological and

The search for a "Holy Grail" of unified theory in environmental


moral assumptions that harks back to Descartes and Newton. . . .

[*8] values has not progressed towards any consensus regarding what inherent value in nature is, what objects
have it, or what it means to have such a value. n50

Norton's expressed preference is for the integration of multiple values on three "scales" of human concern and valuation: (1) locally developed values that reflect the
preferences of individuals; (2) community values that protect and contribute to human and ecological communities; and (3) global values, which express a hope for the long-
term survival of our species. n51 As Norton views it:

A good environmental policy will be one that has positive implications for values associated with the various scales on which humans are in fact concerned, and also on the
scales on which environmentalists think we should be concerned if we accept responsibility for the impacts of our current activities on the life prospects and options--the
"freedom" of future generations. n52

One particularly provocative aspect of environmental pragmatic thought is its desire for compatibilism, i.e.,
a philosophical framework within which competing environmental theories may be compatible in
practice. n53 Andrew Light is an advocate for this view. n54 Light contrasts the views of social ecologists and materialists, such as Murray Bookchin and Herbert
Marcuse, n55 who view environmental degradation as presupposed by a capitalist economy, and ontologists, including "deep ecologists" like Arne Naess, n56 whose focus is
on reform of the self, and one's relationship with the non-human world, as expressed in individual identity. n57 To harmonize these mutually antagonistic schools of
environmental thought, Light proposes a pragmatic "principle of tolerance." n58 [*9] Under it, theorists and practitioners are required to communicate a "straightforward public
position" that endorses the considerations on which they agree, and the practices best suited to meeting their mutually desired goals, while leaving some questions that divide
them to private dispute. n59

Page 679 of 1481


Permutation Solvency: 1AR

HUMAN AND ENVIRONMENTAL INTEREST ARENT MUTUALLY EXCLUSIVE PLAN


PROTECTS BOTH, CREATING A LESS VIOLENT WORLD

Schroeder 2003
[Christopher, Prof. Law and Pub Policy Studies & Dir. Public Law @ Duke, Environmental Protection as a
Jurisdynamic Experience: Prophets, Priests, and Pragmatists, 97 Minn. L. Rev. 1065, April, LN//uwyo-ajl]

Sorting out the competing source-of-value claims made within the broad literature of environmental
philosophy and ethics would force a greater detour here than I can make. Instead, I will simply provide
three observations to lend some support to the view that a critical stance toward business as usual
regarding the environment and a pragmatic approach to values need not be opposed to one another.
First, Deep Ecologists and others who voice the prophetic message have an established record of
extended and detailed investigations into ways that we might satisfy human needs through methods that
are much less resource consumptive than the current status quo. They order such investigations in
significant part because they recognize that satisfying human needs does indeed have a significant value.
A large part of the prophetic project seeks ways to accommodate both a high degree of human need
satisfaction and environmental protection, not always to denigrate the former. Prophets think that
society's current balance between the two is out of kilter, but they need not think that the two do not
have to be balanced at all. The "doing more with less" movement, soft energy paths, hydrogen-based fuel
cells, recycling - these and other such efforts are not [*1085] merely strategic efforts to reduce amoral or
non-moral opposition to the moral hegemony of environmentalism. Instead, they are efforts to
accommodate competing moral values.

THE PROBLEM OF SHALLOW ECOLOGY ISNT ANTHROPOCENTRISM, BUT A SHORT-


TERM FOCUS SHOULD COMBINE QUALIFIED ANTHROPOCENTRISM WITH
BROADER CONCERN FOR THE HUMAN WORLD OVERCOMES THIS

Grey 93
[William, Lecturer at the University of Queensland, Australia, Anthropocentrism and Deep Ecology,
Australian Journal of Philosophy, Vol. 71, no 4, 1993,
www.uq.edu.au/~pdwgrey/pubs/anthropocentrism.html, acc 9-30-04//uwyo-ajl]

That we habitually assume characteristically anthropocentric perspectives and values is claimed by deep
ecologists to be a defect. And as a corrective to this parochialism, we are invited to assume an
"ecocentric" (Rolston 1986, Callicott 1989) or "biocentric" (Taylor 1986) perspective. I am not persuaded,
however, that it is intelligible to abandon our anthropocentric perspective in favour of one which is more
inclusive or expansive. We should certainly abandon a crude conception of human needs which equates
them (roughly) with the sort of needs which are satisfied by extravagant resource use. But the problem
with so-called "shallow" views lies not in their anthropocentrism, but rather with the fact that they are
characteristically short-term, sectional, and self-regarding. A suitably enriched and enlightened
anthropocentrism provides the wherewithal for a satisfactory ethic of obligation and concern for the
nonhuman world. And a genuinely non-anthropocentric view delivers only confusion.

Page 680 of 1481


Anthro Good/Inevitable (1/3)

DEEP ECOLOGY ISOLATES US FROM NATURE, REINSCRIBES ANTHROPOCENTRISM


ANTHRO COMBINED WITH A HOLISTIC PERSPECTIVE SOLVES BEST BY ALLOWING
BETTER VALUING OF HUMANS AND NATURE

Grey 93
[William, Lecturer at the University of Queensland, Australia, Anthropocentrism and Deep Ecology,
Australian Journal of Philosophy, Vol. 71, no 4, 1993,
www.uq.edu.au/~pdwgrey/pubs/anthropocentrism.html, acc 9-30-04//uwyo-ajl]

There are several very plausible elements in the concerns of deep ecology. First, there is the
worry about the effects of unconstrained human interference in natural systems impoverishing and
degrading them. Human interference and human action is often contrasted with the wisdom of natural cycles and natural development. Contrast the violence of
a strip-mined hillside, or a clear-felled forest with the tranquil majesty of a climax ecosystem such as a tropical rain forest or a coral reef. "Nature knows best", it
.
is said

A second worry focuses on the way that we tend to treat humans and human activity in
isolation from, rather than as a part of nature. This is often characterized as an atomistic conception of humans as discrete and
separate interacting units, in contrast to the holistic organic conception of organisms as nodes in complex biotic webs. The sharp separation between humanity
and nature is said to be one of the characteristic deficiencies of shallow thought, which is often accompanied by the denial that the nonhuman world possesses
intrinsic value.

A third common worry concerns the extremely short-term view which people commonly take about the consequences of their actions. <466>

There is an obvious tension which arises when attempting to rectify the first two worries at the
same time. For extolling the virtues of the natural, while at the same time vilifying the man-made or
artificial, depends on a distinction between the natural and the artificial which the stress on a
continuity between human and nonhuman (the focus of the second worry) undermines. On the one side there is
emphasis on continuity and dependency, and on the other on distinctness and separation. It seems that, while we are a part of nature, our actions are
nevertheless unnatural.

deep ecologists often risk lapsing into an incoherence, from which they are
This is one of the points where

able to save themselves (as I will illustrate) with the help of a little covert anthropocentrism. Or putting the
point another way, a suitably enriched (non-atomistic) conception of humans as an integral part of

larger systemsthat is, correcting the misconception of humanity as distinct and separate from the natural worldmeans that
anthropocentric concern for our own well-being naturally flows on to concern for the
nonhuman world. If we value ourselves and our projects, and part of us is constituted by the
natural world, then these evaluations will be transmitted to the world.

ANTHROPOCENTRISM IS NOT ONLY INEVITABLE, BUT NECESSARY TO STOP THE


COMING GREAT AGE OF EXTINCTION

Grey 93
[William, Lecturer at the University of Queensland, Australia, Anthropocentrism and Deep Ecology,
Australian Journal of Philosophy, Vol. 71, no 4, 1993,
www.uq.edu.au/~pdwgrey/pubs/anthropocentrism.html, acc 9-30-04//uwyo-ajl]

If the concerns for humanity and nonhuman species raised by advocates of deep ecology are expressed as concerns about the fate of the planet, then these

concerns are misplaced. From a planetary perspective, we may be entering a phase of mass extinction of

Page 681 of 1481


the magnitude of the Cretaceous. For planet earth that is just another incident in a four and a
half billion year saga. Life will go onin some guise or other. The arthropods, algae and the ubiquitous bacteria, at least, will almost certainly be
around for a few billion years more. And with luck and good management, some of the more complex and

interesting creatures, such as ourselves, may continue for a while longer as well. Of course our
present disruptive and destructive activities are, or should be, of great concern to us all. But that is a
quite properly human concern, expressing anthropocentric values from an anthropocentric
perspective. Life will continue; but we should take steps to maintain and preserve our sort of
living planet; one that suits us and, with a few exceptions, our biotic co-existents.

allegedly non-anthropocentric points of view incorporate a covert


I will illustrate the way that

anthropocentrism with some representative examples which, I believe, reveal the inevitability of
anthropocentrism and show that it is not necessarily something to be deplored.
Anthropocentrism is natural and inevitable, and when properly qualified turns out to be
perfectly benign. The first illustration concerns a proposal to develop a non-anthropocentric basis for value by grounding it in the naturalness of an
historical process.

Page 682 of 1481


Page 683 of 1481
Anthro Good/Inevitable (2/3)
HUMAN INTERVENTION IS INEVITABLE, ITS A QUESTION OF THE MERITS OF
ACTION, NOT A QUESTION OF WHETHER TO ENGAGE IN IT OR NOT

Bookchin 95
[Murray, Social Ecologist, Philosophy of social ecology, 139//uwyo]

Page 684 of 1481


Page 685 of 1481
Anthro Good/Inevitable (3/3)
HUMAN INTERVENTION IN NATURE IS INEVITABLE

Bookchin 95
[Murray, Social Ecologist, Philosophy of social ecology, 131//uwyo]

Page 686 of 1481


Human Intervention Good

HUMAN ACTIVITY CAN POSITIVELY AFFECT ECOLOGY SOCIETY IS THE ONLY


MEANS OF RESOLVING THE CRISIS

Bookchin 89
[Murray, Social Ecologist, Remaking Society, 17//uwyo]

Page 687 of 1481


Deep Ecology Justifies Ecocide (1/2)

THE ALTERNATIVE SIMPLIFIES NATURE, CAUSING INACTION THAT MAKES


ENVIRONMENTAL DESTRUCTION, NUCLEAR WAR, AND EXTINCTION INEVITABLE

Bookchin 87
[Murray, social ecologist, The modern crisis, 108//uwyo]

Page 688 of 1481


Page 689 of 1481
Deep Ecology Justifies Ecocide
(2/2)
DEEP ECOLOGY REINSCRIBES ANTHROPOCENTRISM AND MAMMAL CHAUVINISM
AND DESTROYS ANY FOUNDATION OF NATURAL VALUE

Grey 93
[William, Lecturer at the University of Queensland, Australia, Anthropocentrism and Deep Ecology,
Australian Journal of Philosophy, Vol. 71, no 4, 1993,
www.uq.edu.au/~pdwgrey/pubs/anthropocentrism.html, acc 9-30-04//uwyo-ajl]

Finally, I consider the "ecocentric" approach advocated, for example, by J. Baird Callicott (1989),
which is another attempt to develop a non-anthropocentric basis for value. This "deep"
approach, inspired by Aldo Leopold (1949), on examination also reveals covert
anthropocentrism. For example, in "On the Intrinsic Value of Nonhuman Species" Callicott
explores various grounds on which we might extend moral consideration to nonhuman
individuals. One particular line which he explores, and revealingly rejects is "holistic
rationalism". Goodness, on this view, is identified above all with the objective harmony of the
biosphere as a whole, which "exemplifies or embodies the Good" (Callicott 1989, p. 142). Since
species serve the good of the biotic whole (which is quite independent of human interest) we
have a non-anthropocentric justification for species preservation. But individual species, from
this perspective, are transitional components of developmental stages of the planet's
evolutionary odyssey:

The Age of Reptiles came to a close (for whatever reason) to be followed by the Age of
Mammals. A holistic rationalist would not regret the massive die-off of the late Cretaceous
because it made possible our yet richer mammal-populated world. The Age of Mammals may
likewise end. But the "laws" of organic evolution and of ecology (if any there be) will remain
operative. In time speciation would occur and species would radiate anew. Future "intelligent"
forms of life may even feel grateful, if not to us then to their God (or the Good), for making their
world possible. The new Age (of Insects, perhaps) would eventually be just as diverse, orderly,
harmonious and stable and thus no less good than our current ecosystem with its present
complement of species.

With friends like the holistic rationalists, species preservation needs no enemies. (Callicott 1989,
p. 142)

This passage is revealing. Note the characterization of the Age of Mammals as "richer" than the
Age of Reptiles. As mammal chauvinists we might agree, but it is not clear on what grounds
Callicott can justify the claim. It is also easy to agree that our demise, and the demise of the
ecosystem which currently supports us, would be a matter of regret. But clearly it would be
regrettable because of a decidedly anthropocentric set of values, interests and perceptionsif
Callicott really eschews such concerns entirely, the grounds on which his regret is based are
deprived of any foundation.

Page 690 of 1481


Deep Ecology Reinscribes Anthropocentrism (1/2)

THE ALTERNATIVE INSERTS HUMAN JUDGMENT IN PLACE OF ECOLOGICAL


INTEREST, ALLOWING FOR TOTALTIARIANISM

Bobertz 97
[Bradley C., Book Review: Of Nature and Nazis, 22 Colum. J. Envtl. L. 353, LN//uwyo-ajl]

The argument that


Apart from the political dangers Ferry associates with deep ecology, he believes the philosophy suffers from a fundamental self-contradiction.

natural objects can possess their own interests strikes Ferry as "one of the most absurd forms of
anthropomorphism." n100 We cannot "think like a mountain," to use Aldo Leopold's famous phrase, n101 because, quite obviously,
we are not mountains. Recalling Sierra Club v. Morton, n102 the famous standing case involving a proposal to construct a ski resort in California's Mineral King
valley, Ferry claims that environmentalists "always suppose that the interests of objects (mountains, lakes and other natural things)

are opposed to development. But how do we know? After all, isn't it possible that Mineral King would be
inclined to welcome a ski slope after having remained idle for millions of years?" n103 Yet few people, including the writers
Ferry labels as deep ecologists, would disagree with the fact that recognizing value in natural objects is an act of human cognition.
Perhaps a person suffering from profound psychosis might claim the ability to understand how a mountain "thinks," but the writers Ferry criticizes do not advance 8540*379
such bizarre claims. n104 For deep ecologists and environmental ethicists, phrases such as "think like a mountain" are metaphorical and heuristic, not literal and agenda-
setting.

a far graver problem with deep ecology lies in its appeal to those who might translate a
According to Ferry,

nature-centered ideology into coercive political action. By promoting the idea that nature has intrinsic
value, deep ecologists necessarily promote an antihuman, antitechnology, and antimodern worldview, Ferry
believes. If we assert that humans are merely "part" of the natural order, our position in that order must be a humble one:

The entire Cosmos may well be assigned a positive coefficient higher than that of humankind itself, since in the hierarchy of beings it constitutes the primary condition: nature
can do without men, but not vice versa, which is why the idea of a "preference for nature" finds itself gradually legitimized as all in all the most logical metaphysical horizon of
deep ecology. n105

DEEP ECOLOGY REINSCRIBES ANTHROPOCENTRIC VALUES

Bookchin 94
[Murray, Social ecologist, Which way for the ecology movement? 3//uwyo]

Page 691 of 1481


Page 692 of 1481
Page 693 of 1481
Deep Ecology Reinscribes
Anthropocentrism (2/2)
THE ALTERNATIVE FALLS BACK INTO RESOURCE CALCULATION, DESTROYING
ENVIRONMENTAL PROTECTION

Cross 89
[Frank B., Assoc. Prof. Bus Law @ Texas, Natural Resource Damage Valuation, 42 Vand. L. rev. 269,
March, LN//uwo-ajl]

Yet those who ascribe to the intrinsic value of nature may themselves oppose the monetary measurement
of that value. Some deep ecologists, for example, are uncomfortable with the capitalist system's [*294]
focus on private property. n122 For these ecologists, relying on economics is "technocratic" and the root
of environmental degradation; monetary natural resource damages contribute to the problem rather than
the solution. n123 These people refuse to place a monetary value on nature, finding that the very effort
demeans the underlying worth of nature. n124 Their refusal leaves the law only two options:
Economically valuing natural resources at zero or at infinity. The former alternative inevitably creates an
incentive to destroy the resources that the naturalist seeks to protect. n125 The latter is transparently
unworkable, as it suggests that the death of a single fly provides grounds for bankrupting the largest
corporations. n126 While it is indisputably difficult to assess the monetary value of natural objects, the
effort should be made. Otherwise, "treating the problem as an inherent incapacity of analysis to
incorporate the intangible can only retard the needed development of these important abilities." n127 No
persuasive methodologies, however, objectively and reliably ascertain the intrinsic worth of natural
resources.

Page 694 of 1481


Deep Ecology Justifies Nazism: 2AC

DEEP ECOLOGY RISKS CO-OPTATION BY GENOCIDAL FASCISTS


A) SCARE-MONGERING GETS HIJACKED
B) SUBORDINATION OF SOCIAL PROBLEMS
C) ANTI-RATIONAL ALTERNATIVE

Smith 2003
[Kev, Greenpepper, Ecofascism: Deep Ecology and Right-Wing Co-optation, Synthesis/Regeneration 32,
http://www.greens.org/s-r/32/32-13.html, acc 10-4-06//uwyo-ajl]

It is striking how many traits the Wandervgel have in common with the Deep Ecology movement. In
particular, their self-conception that they were a non-political response to a deep cultural crisis,
favoring direct emotional experience over social critique and action. In the same paper, Janet Biehl states,
When respect for nature comes to mean reverence, it can mutate ecological politics into a religion that
Green Adolfs can effectively use for authoritarian ends. In Britain, a wing of the National Front issues
the cry, Racial preservation is Green! while in the United States, white supremacist Monique Wolfing
remarks that animals and the environment, are in the same position as we are. Why would we want
something created for ourselves and yet watch nature be destroyed? We work hand in hand with nature
and we should save nature along with trying to save our race.

The key question is whether supporters of Deep Ecology are vulnerable to absorption by far-right groups
in the same way that the Wandervgel were. The main fear for this happening lies in Deep Ecology's
demonization of reason. Deep Ecology sees reason as endemic to human-centered worldviews that have
produced the ecological crisis. Alternatively, Deep Ecology promotes intuition as equal or even superior to
reason. As a result Deep Ecology is subject to the dangers represented by earlier anti-rational and
intuitionist worldviews that, once carried over into the political realm, have produced anti-human and
even genocidal movements. Peter Staudenmaier fears that this is perhaps, the unavoidable trajectory of
any movement which acknowledges and opposes social and ecological problems but does not recognize
their systemic roots or actively resist the political and economic structures which generate them.

Deep Ecology, as a philosophy, seems to be both systematically and morally problematic. Where Deep
Ecology theories have gone wrong is in the extreme reaction to perceived centuries of human exploitation
of nature and the dominance of rationalist thought. The primacy of intuitive thought means that it lacks
the self-analysis that normally acts as a safety check to prevent straying onto moral thin ice. These factors
then serve to prevent an accurate picture of the ecological crisis from emerging. The role of personal
consciousness-raising on both rational and intuitive levels should be complementary rather than
competitive. In the manner of the classic circularity of extreme left and right thought, Deep Ecology has
the potential to find itself back at the totalitarian starting point it intended to usurp.

Page 695 of 1481


Page 696 of 1481
Deep Ecology Justifies Nazism:
1AR (1/2)
EXCESS REVERENCE FOR NATURE DE-SENSITIZES US TO HUMAN AGENCY, A
PROCESS THAT WAS INSTRUMENTAL IN NAZISM

Bookchin 94
[Murray, Social Ecologist, Which way for the ecology movement? 41//uwyo]

Page 697 of 1481


Page 698 of 1481
Deep Ecology Justifies Nazism:
1AR (2/2)
DEEP ECOLOGYS DOGMATIC FOCUS CREATES HUMAN PREJUDICE AND NAZISM

Bookchin 94
[Murray, Social ecologist, Which way for the ecology movement? 7-8//uwyo]

Page 699 of 1481


Deep Ecology Justifies Nazism: Ext (1/2)

TURN: THE NAZIS USED NATURE STORIES TO JUSTIFY GENOCIDE

ONEILL IN 95
Sadhbh, researching an MA thesis on ecophilosophy at UCD, DEEP ECOLOGY AND
FEMINISM: TO THE WORLD AND BACK, 1995, p.
http://www.iol.ie/~mazzoldi/toolsforchange/zine/imb95/ecofemin.htm.

Most recently, a member of Earth First! in Ireland wrote in Common Ground that no qualitative
difference exists between the Jewish holocaust and the "eco-holocaust" currently underway.
That sort of comparison is made, mind, in the spirit of biospherical egalitarianism as formulated
by Naess. Despite the many indications from evolutionary biology and the science of ecology
that nature really does, in rather different ways to human beings, posit values, make choices,
pursue ends and so on, to equate the tragedy we are inflicting on nature with the horror that
we are capable of inflicting on our own fellow species is nothing short of fascism. There are
many examples of how the Nazis, for instance, used naturalistic myths and ecology to justify
oppression and systematic murder in the interests of nature.

NAZI GERMANY JUSTIFIED IMPERIALIST EXPANSION INTO EASTERN EUROPE


ECOLOGICALLYPROTECTING THE ENVIRONMENT FROM THE PEOPLE WHO
WERE POLLUTING IT

STAUDENMAIER IN 98
(Peter, anarchy theorist, Professor for the Institute for Social Ecology, Fascist
Ecology: The "Green Wing" of the Nazi Party and its Historical Antecedents,
February 1998,
http://www.spunk.org/library/places/germany/sp001630/peter.html)

Darr was one of the party's chief "race theorists" and was also instrumental in galvanizing
peasant support for the Nazis during the critical period of the early 1930s. From 1933 until 1942 he
held the posts of Reich Peasant Leader and Minister of Agriculture. This was no minor fiefdom; the
agriculture ministry had the fourth largest budget of all the myriad Nazi ministries even well into the war.38
From this position Darr was able to lend vital support to various ecologically oriented initiatives.
He played an essential part in unifying the nebulous proto-environmentalist tendencies in
National Socialism: It was Darr who gave the ill-defined anti-civilization, anti-liberal, anti-
modern and latent anti-urban sentiments of the Nazi elite a foundation in the agrarian
mystique. And it seems as if Darr had an immense influence on the ideology of National Socialism, as if he
was able to articulate significantly more clearly than before the values system of an agrarian society
contained in Nazi ideology and -- above all -- to legitimate this agrarian model and give Nazi policy a goal
that was clearly oriented toward a far-reaching re-agrarianization.39 This goal was not only quite
consonant with imperialist expansion in the name of Lebensraum, it was in fact one of its

Page 700 of 1481


primary justifications, even motivations. In language replete with the biologistic metaphors of
organicism, Darr declared: "The concept of Blood and Soil gives us the moral right to take back
as much land in the East as is necessary to establish a harmony between the body of our Volk
and the geopolitical space."40 Aside from providing green camouflage for the colonization of
Eastern Europe, Darr worked to install environmentally sensitive principles as the very basis of the Third
Reich's agricultural policy. Even in its most productivist phases, these precepts remained emblematic of Nazi
doctrine. When the "Battle for Production" (a scheme to boost the productivity of the agricultural sector)
was proclaimed at the second Reich Farmers Congress in 1934, the very first point in the program read
"Keep the soil healthy !" But Darr's most important innovation was the introduction on a large scale of
organic farming methods, significantly labeled "lebensgesetzliche Landbauweise," or farming according to
the laws of life. The term points up yet again the natural order ideology which underlies so much
reactionary ecological thought. The impetus for these unprecedented measures came from Rudolf Steiner's
anthroposophy and its techniques of biodynamic cultivation.41

Page 701 of 1481


Page 702 of 1481
Deep Ecology Justifies Nazism:
Ext (2/2)
ABSOLUTE PROTECTION OF THE ECOSYSTEM IS UNSTABLEIGNORES SOCIETY,
COLLAPSES TO FACISM AND BARBARISM

STAUDENMAIER IN 98
(Peter, anarchy theorist, Professor for the Institute for Social Ecology, Fascist
Ecology: The "Green Wing" of the Nazi Party and its Historical Antecedents,
February 1998,
http://www.spunk.org/library/places/germany/sp001630/peter.html)

As noted above, this failure most commonly takes the form of a call to "reform society
according to nature," that is, to formulate some version of 'natural order' or 'natural law' and
submit human needs and actions to it. As a consequence, the underlying social processes and
societal structures which constitute and shape people's relations with their environment are left
unexamined. Such willful ignorance, in turn, obscures the ways in which all conceptions of
nature are themselves socially produced, and leaves power structures unquestioned while
simultaneously providing them with apparently 'naturally ordained' status. Thus the substitution
of ecology for clear-sighted social-ecological inquiry has catastrophic political repercussions, as
the complexity of the society-nature dialectic is collapsed into a purified Oneness. An
ideologically charged 'natural order' does not leave room for compromise; its claims are
absolute. For all of these reasons, the slogan advanced by many contemporary Greens, "We are
neither right nor left but up front," is historically naive and politically fatal. The necessary
project of creating an emancipatory ecological politics demands an acute awareness and
understanding of the legacy of classical ecofascism and its conceptual continuities with present-
day environmental discourse. An 'ecological' orientation alone, outside of a critical social
framework, is dangerously unstable. The record of fascist ecology shows that under the right
conditions such an orientation can quickly lead to barbarism.

DEEP ECOLOGISTS ENDORSE INHUMAN POLICIES AND HUMAN SUFFERING

TOKAR IN 90
Brian, Author of The Green Alternative: Creating an Ecological Future, August
1990,New Internationalist, http://www.newint.org/issue210/eco.htm

A major controversy began when Earth First! co-founder Dave Foreman was quoted in an
interview by deep ecologist Bill Devall making some shockingly misanthropic statements in the
name of deep ecology and Earth First! Deep ecologists claim overpopulation as the underlying
cause of ecological crisis and advocate population reduction. Foreman took this one step

Page 703 of 1481


further, advocating forced sterilizations, ending food aid to starving people (particularly, at the
time, in Ethiopia), and sealing US borders against refugees from the wars in Latin America. To
Foreman, such measures were ways to let nature seek its own balance, and prevent more
destruction of our wilderness, more poisoning of our water and air.

Page 704 of 1481


A2 Were Not Fascists: 1AR

IRRELEVENT THEIR PROGRAM WILL BE CO-OPTED BY OTHERS FOR FASCIST ENDS

Smith 2003
[Kev, Greenpepper, Ecofascism: Deep Ecology and Right-Wing Co-optation, Synthesis/Regeneration 32,
http://www.greens.org/s-r/32/32-13.html, acc 10-4-06//uwyo-ajl]

There are two reasons why I find such a statement from a moderate Deep Ecologist worrying. The first
is that it misses the point that you do not necessarily have to be a fascist in order to propagate right-
wing ideology. Secondly, it still places the issue of population control ahead of the issue of how resources
are unevenly distributed among the global population. It is astonishing how many environmental groups
(and not just Deep Ecologists; the mainstream Dutch environmental group Milieu Defensie is a depressing
recent example) still rate population growth over the systematic over-consumption of the industrialized
world. This misinforms the person on the street, reinforcing fears that their stably populated Western
country may be overrun by the teeming dark-skinned multitudes of the Third World. Such scare-
mongering plays directly into the hands of the new right and lends inadvertent support to calls for stricter
border controls.

We would do well to examine the example of the Wandervgel, a youth movement that arose in Germany
during the first three decades of the 20th Century. Peter Staudenmaier, co-author of the paper
Ecofascism: Lessons From The German Experience, characterizes this movement as a hodge-podge of
counter-cultural elements, blending neo-Romanticism, Eastern philosophies, nature mysticism, hostility to
reason, and a ... search for authentic, non-alienated social relations. Their back-to-the-land emphasis
spurred a passionate sensitivity to the natural world and the damage it suffered. Although some sectors
of the movement gravitated towards various forms of emancipatory politics, most of the Wandervgel
were eventually absorbed by the Nazis.

Page 705 of 1481


Deep Ecology Justifies State/Capitalism

BIOCENTRISM PREVENTS OPPOSITION TO CAPITALISM AND THE STATE BECAUSE IT


SHUNS FOCUS ON SOCIAL PROBLEMS

Bookchin 95
[Murray, Social ecologist, Philosophy of social ecology, 133//uwyo]

Page 706 of 1481


Deep Ecology Creates Suffering

DEEP ECOLOGY SUBORDINATES ALL VALUES TO NATURE, CREATING RACISM AND


HUMAN SUFFERING

Green Fuse 2006


[Deep Ecology Critique, June, http://www.thegreenfuse.org/deepcrit.htm, acc. 10-4-06//uwyo-ajl]

Dave Foreman, founder of Earth First! which claims to draw inspiration from deep ecology, has made
several deeply misanthropic comments.

"It is rather painful to read about some of the positions taken by the Foreman faction in the E.F!
Journal: for example, Foreman arguing that even a nuclear war would not be that damaging to the Earth
and would hasten the end of industrial society... and his remarks elsewhere that we should "allow
Ethiopians to starve"; Christopher Manes suggesting that one solution to overpopulation would be to
dismantle the medical technology designed to save lives, and of AIDS as Nature's solution to
overpopulation; and Reed Noss writing of genetic "deep ecology elite" as a "chosen people" out to save
the Earth (pp. 64, 68, 83-84, 92-3,101-3).

George Sessions, Book Review: Martha Lee, Earth First!. Trumpeter: 13, 4 (1996)

Sessions adds that if such comments claim to draw on deep ecology they show a misunderstanding of its
philosophy.

Murray Bookchin comments:

"They are barely disguised racists, survivalists, macho Daniel Boones and outright social reactionaries
who offer a vague, formless often self contradictory and invertebrate [movement] and a kind of crude
eco-brutalism similar to Hitler's. Deep ecologists feed on human disasters, suffering and misery...[and are
guilty of thinking which]...legitimates extremely regressive, primitivistic and even highly reactionary
notions."

Page 707 of 1481


Case Comes First

MUST ADDRESS EXIGENT ISSUES BEFORE ADDRESSING PERSONAL PERSPECTIVE ON


NATURE

Bookchin 94
[Murray, Social Ecologist, Which way for the social ecology movement? 1//uwyo]

Page 708 of 1481


Alternative Fails: Bad Activism

ANTHROPOCENTRISM IS SO INGRAINED THAT THE ALTERNATIVE IS IMPOSSIBLE

Cross 89
[Frank B., Assoc. Prof. Bus Law @ Texas, Natural Resource Damage Valuation, 42 Vand. L. rev. 269,
March, LN//uwo-ajl]

Perhaps the argument over the intrinsic worth of natural resources is largely pointless. Political realists
contend that concern for inherent animal welfare lacks public credibility. Whatever the metaphysical basis
for nature's intrinsic value, the advocates of this position risk being considered impractical and fuzzy-
headed, if not outright crackpots. Their arguments are treated with more ridicule than respect. n136
Perhaps these critics are partly correct. As long as government is making the legal rules and as long as
only humans vote, the concerns of nature never will be reflected directly in our nation's governmental
policy. Most environmental laws enacted to date focus on protecting people's [*296] interest in the
natural environment. n137 Nature's influence on people may be felt in a myriad of ways, but legislation is
not among them. Inasmuch as the question is phrased in public policy terms, the answer must come from
humans alone. n138

Indeed, the terminology from a discussion of natural resources seems antithetical to intrinsic valuation.
The term "resource" implies usefulness to man. n139 Similarly, "value" may require a human subject to
express a preference regarding the natural object. n140 Remove the human subject, and the concept of
value loses meaning. n141 The legal valuation of natural resources is a human undertaking that is limited
inescapably to human understanding and choice.

Of course, one may be persuaded that nature has intrinsic value for which government should account.
Enlightened human preference thus may capture at least a portion of intrinsic value, but the preference is
predicated necessarily on an informed human understanding of intrinsic value, not on the value itself.
n142 This recognition also helps defeat the antidemocratic and elitist features potentially existing in
concepts of intrinsic value. n143

THE ALT FAILS: RESOURCE-VIEW OF NATURE IS TOO ENTRENCHED

McCullough 95
[Edwin r., solo practice in Chicago, JD Loyola, Through the Eye of a Needle, 10 J. Envtl. L. & Litig. 389,
LN//uwyo-ajl]

Though deep ecology presents a utopian vision of the future, its prospects are about as dim as are the
prospects of Weiss' intergenerational equity theory. Weiss' theory is more mainstream and is backed by a
vast body of law, which is distilled into well-drafted introductory principles. Deep ecology offers strong simple statements for new
legal and/or moral principles and offers a fundamentally different worldview for the future. The principles of deep ecology, however, offer us tools for improving environmental law now.
Indeed, some of the principles of deep ecology, though not expressed in name, have been adopted by other environmentalists.

Two environmental writers, law professor, Earl Finbar Murphy, n111 and conservation biologist, David W. Ehrenfeld, n112 discussed various aspects of ecology with anthropocentric logic
before deep ecology became a familiar notion. They independently arrived at the same point; nonhuman life has value independent of human activity. This is another way of stating the first
principle of the deep ecology platform. [*420]

Dr. Ehrenfeld's article, The Conservation of Non-Resources, begins with the idea of conservation being identified with the preservation of natural resources. The term, resource, can be defined
narrowly as the reserve of commodities that has an appreciable money value to man, either directly or indirectly. Ehrenfeld points out that over the years, conservationists increasingly have

Page 709 of 1481


is the
been preoccupied with preservation of natural features, species, communities, and ecosystems - items which are not conventional resources. His example of a non-resource

endangered Houston toad, an animal with no demonstrated or conjectural resource value to man. n113

He observes that due to the dominant worldview, species and communities that lack economic value are
not easily protected. Thus, a "value" must be discovered by which the non-resource can metamorphose
into a resource. n114 He notes the practical political weakness of concocting a "value" for a non-resource;
this kind of value is not as appealing as those backed by the promise of a short-term economic gain. He
notes that "when everything is called a resource, the word loses all meaning - at least in our value
system." n115 From a conservation viewpoint, it may become quite risky to find economic values for non-
resources.

Page 710 of 1481


Alternative Fails: Premodern Society Bad

PRE-MODERN SOCIETY IS MORE DESTRUCTIVE TO THE EARTH

Green Fuse 2006


[Deep Ecology Critique, June, http://www.thegreenfuse.org/deepcrit.htm, acc. 10-4-06//uwyo-ajl]

Deep ecology sometimes appears to idealize a the society of indigenous hunter-gatherer tribes, but in
reality many primitive tribes are not especially ecocentric.

Riane Eisler, author of The Chalice and the Blade writes:

"...many peoples past and present living close to nature have all too often been blindly destructive of
their environment. While many indigenous societies have a great reverence for nature, there are also
both non-Western and Western peasant and nomadic cultures that have overgrazed and overcultivated
land, decimated forests, and where population pressures have been severe, killed off animals needlessly
and indifferently."

Page 711 of 1481


Asteroid Turn

ANTHROPOCENTRISM IS NECESSARY TO STOP NATURAL PHENOMENA LIKE


ASTEROID COLLISIONS AND ICE AGES, WHICH THREATEN MASS EXTINCTION ON A
SCALE MUCH MORE THREATENING TO THE BIOSPHERE AND BIODIVERSITY THAN
HUMAN ACTIVITY EMPIRICALLY PROVEN BY THE GREAT EXTINCTIONS OF THE
PAST

Grey 93
[William, Lecturer at the University of Queensland, Australia, Anthropocentrism and Deep Ecology,
Australian Journal of Philosophy, Vol. 71, no 4, 1993, www.uq.edu.au/~pdwgrey/pubs/anthropocentrism.html,
acc 9-30-04//uwyo-ajl]

Robert Goodin has proposed a "moderately deep" theory of value, according to which what imparts value to an outcome is the naturalness of the historical
process through which it has come about (Goodin 1991, p. 74). Putting aside the problem, mentioned above, that the distinction between what is natural and

the deliverances of natural historical processes are not


what is cultural (or technological, or artefactual) is problematic,

necessarily benign, nor ones which should command our approval. The traumatic disruptions to the planet
brought about by natural forces far exceed anything which we have been able to effect.
Consider, first, what Lovelock (1979) has called the worst atmospheric pollution incident ever: the
accumulation of that toxic and corrosive gas oxygen some two billion years ago, with devastating
consequences for the then predominant anaerobic life forms. Or the Cretaceous extinction 65 million years ago, which wiped out

the large reptiles, the then dominant life forms. Or the Permian extinction some 225 million years ago,
which eliminated an estimated 96 per cent of marine species. Like the eruption of Mt St Helens, these were natural
events, but it is implausible to suppose that they are to be valued for that reason alone.

There is of course an excellent reason for us to retrospectively evaluate these great planetary disruptions positively from our current position in planetary
history, and that is that we can recognise their occurrence as a necessary condition for our own existence. But what could be more anthropocentric than that?
However, as Gould has pointed out, mass extinctions are awful for those who are caught up in them.

Suppose that astronomers detect a modest asteroid or comet, say five or ten kilometres diameter, on collision course
with planet Earth [8]. The impending collision would be perfectly natural all right, and cataclysmic

enough to do to us what another one rather like it probably did to the dinosaurs. Such periodic
disruptive events are natural all right, though they probably destroy most of the then extant large life forms. These times of renewal provide
opportunities for smaller, flexible organisms to radiate opportunistically into vacated niches, and life goes on. From a biocentric or ecocentric perspective there

Should we, in <470>such


is little doubt that our demise would provide comparable opportunities for development which we currently prevent.

, step aside so that evolution can continue on its majestic course? I think not, and I think further
circumstances

that interference with the natural course of events, if it could be effected, would be no bad thingat least
from our point of view and in terms of our interests, which it is quite legitimate to promote and favour.

Suppose again that we are entering one of the periodic epochs of reduced solar energy flux. An
ice age is imminent, with massive disruptions to the agriculturally productive temperate zones.
However suppose further that by carefully controlled emissions of greenhouse gases it would be possible

to maintain a stable and productive agriculture. No doubt this would be to the detriment of various arctic plant and animal
species, but I do not think that such interference, though "unnatural" would be therefore deplorable. Nature in and of itself is not, I suggest, something to be
modifying our natural environment, we would
valued independently of human interests. It could be argued moreover that in thus

be following the precedent of three billion years of organic evolution, since according to the Gaia hypothesis
of Lovelock (1979), the atmosphere and oceans are not just biological products, but biological

constructions.

Other natural propertiessuch as biodiversity, beauty, harmony, stability, and integrityhave been proposed to
provide a non-anthropocentric basis for value. But unless we smuggle in some anthropocentric
bearings, they fare no better than the property of being the outcome of a natural process in
providing an intuitively plausible ordering of better and worse states of the world. For example, if
biodiversity is taken as a basic value-giving characteristic, then the state of the planet just after the Cambrian explosion (about 570 million years ago) would be
rated much more highly than the world of the present, as it was far richer in terms of the range and diversity of its constituent creatures. Most biology textbooks
recognize between twenty and thirty extant animal phylathe phylum being the fundamental design plan of an organism (and the second broadest

Page 712 of 1481


classification, following 'kingdom', in biological taxonomy). Yet the Burgess Shale, one small quarry in British Columbia dating back some 530 million years,

In
contains the remains of fifteen to twenty organisms so unlike one another, or anything now living, as to each constitute a separate phylum (Gould 1989).

terms of basic diversity, a far greater range of radically different anatomical types existed at that
epoch of evolutionary development.

Page 713 of 1481


HIV Turn

DEEP ECOLOGY PREVENTS US FROM FIGHTING VIRII LIKE HIV AND SMALLPOX OUT
OF RESPECT FOR VIRAL AUTONOMY

Grey 93
[William, Lecturer at the University of Queensland, Australia, Anthropocentrism and Deep Ecology,
Australian Journal of Philosophy, Vol. 71, no 4, 1993,
www.uq.edu.au/~pdwgrey/pubs/anthropocentrism.html, acc 9-30-04//uwyo-ajl]

There are a number of problems with such a permissive criterion of moral considerability. One is
that there are conflicts of interest between goal-directed entities, and something needs to be
said about how these are to be resolved. Smallpox and HIV no doubt have their own viral
autonomy (as well as being the products of natural historical processes), but for all that it is
perfectly legitimate to disregard their interests when they conflict with our own. Yet it is hard to
see how a decision to deny them a place in the scheme of things can be defended except by
appeal to a value system which favours human interests. Plumwood allows that in casting the
moral net widely we will have to "make distinctions for appropriate treatment within each class
of items" (p. 147). It seems reasonable to suspect that human standards of appropriateness will
be brought to bear to settle cases where such conflicts arise.

NEW WAVE OF SUPER VIRII LIKE AIDS WILL USE HUMAN TRAVEL TO CAUSE
EXTINCTION

Leibovich 97
[Lori, staff, X stands for eXtinction: Interview with Frank Ryan, M.D., a prominent physician, Salon,
Newsreal, March 1997, www.salon.com/march97/news/news2970321.html, Acc 9-30-04//uwyo-ajl]

And "Virus X" is one of them? What is "Virus X"?

The title of my book, "Virus X," means a virus that threatens human extinction. The X stands for
"eXtinction." I should add that most of the book is devoted to less terrible, scary but interesting,
scenarios. But it would be foolish not to face the worst-case scenario, which I discuss in the
book.

There were fears that AIDS might fit that description. Is it because of international
transportation and ease of travel that these viruses have become so threatening?

Yes. Human behavior has greatly changed the natural goal posts with regard to the threat of
new plague viruses. Take AIDS, for example. According to my hypothesis, in the past a band of
hunters might have been bitten or scratched by chimpanzees harboring the virus; the result
would have been a lethal attack localized to the hunter band -- or at worst their home village.

Today, thanks to the global village, a new plague virus could perambulate the globe at the speed
of a passenger jet. Then a new step in the plague scenario would take place in the massively
populated cities -- they would become viral "amplification zones."

Page 714 of 1481


Page 715 of 1481
African AIDS Outweighs

HIV INFECTION THREATENS THE EXTINCTION OF THE AFRICAN CONTINENT

KRQE News 2002


[Associated Press, Africans are Faced with Extinction by AIDS, August 28, 2002, 100777.com/doc/205
acc 9-30-04//uwyo-ajl]

The disease will undermine the continent's social and economic stability, with the biggest
increases in early deaths coming among people who are in their 30s, 40s and 50s, when they
should be at their most productive, and will leave a population of AIDS orphans in its wake, the
conference was told.
In five African countries, deaths will outstrip births by 2010, meaning falling populations.
"Unfortunately, many African countries are only beginning to see the impact of high levels of
HIV prevalence," said the Census Bureau's Karen Stanecki.

"By 2010, we project that life expectancies in these countries will be back to levels that have not
been seen since the 19th century."

The Census Bureau's "middle-case scenario," which assumes that the epidemic will begin to
level off in Africa over the next eight years, predicts the average life expectancy in Botswana
and Mozambique will drop to just 27 years.

"We are faced with extinction," said Dr. Banu Khan, head of the National AIDS Co-ordinating
Agency in Botswana.

THE DEVASTATION OF THE AIDS VIRUS IN AFRICA IS A MANIFESTATION OF GLOBAL


APARTHEID OPPOSITION TO ALL DEVALUATION OF BLACK LIFE IS THE MOST
IMPORTANT DISCURSIVE STEP IN FIGHTING RACISM AND COLONIALISM

Deen 2001
[Thalif, Staff, Rights: Caste, Drugs, AIDS have Racism Links, Say US Groups, Inter Press Service, August
23, 2001, www.aegis.com/news/ips/2001/IP010807.html , acc 8-30-04//uwyo-ajl]

Meanwhile, the Washington-based NGO Africa Action said that the global AIDS pandemic must
be seen as a matter of international racism.

"The AIDS crisis - whose epicentre is Africa - is the harvest for an international system of global
apartheid, where the consequences of racism, slavery and colonialism have, five centuries on,
impoverished the African continent and left it on its own to combat the worst plague in human
history." AIDS, it said, is the black plague. So while AIDS is a global threats that knows no
borders and does not discriminate by race, it is mainly killing black people.

Africa Action said the racism conference should recognise that the resolution of the global AIDS
pandemic is directly dependent upon the international fight against racism.

Page 716 of 1481


"It is the devaluation of black life that has enabled the Western world to turn its eyes away from
this global health crisis," it added. "Of all of the struggles against racism that we will discuss in
Durban, none has farther reaching consequences for the immediate future of our common
humanity."

Page 717 of 1481


Singularity Turn

HUMAN TECHNOLOGY IS A COMPONENT OF NATURAL COSMOLOGICAL


EVOLUTION RESISTING ANTHROPOCENTRISM BLOCKS THE SINGULARITY
NECESSARY TO SOLVE ALL WORLDLY PROBLEMS

Glasser 2006
[Micah J., Independent Philosopher, Cosmological Deep Ecology and the Singularity, Event Horizon,
January 25, http://technoeventhorizon.blogspot.com/2006/01/cosmological-deep-ecology-and.html, acc.
10-4-06//uwyo-ajl]

Man is a part of a system. As Man evolves both biologically and technologically so does that system. The system I am talking about is our environment and that environment is the entire
Cosmos. Of course the most important part of that environment is the earth itself.

Some ecologists and environmentalists seem to view man and his technology as something over and against nature. This position
couldn't be further from the truth. Both man and his technology are outgrowths of nature. Nature is not a thing that is static, that, if

it wasn't for man and his technology, would be pristene. Nature is a part of the ever changing Flux and as such it is always in

motion and ever changing. The history of the Cosmos is a history of extraordinary change and complexification. As the Cosmos unfolds new
properties emerge. Two of those properties, at this late stage of cosmological development, are
intelligent life and technology. Are we to believe that the emergence of intelligent life in the Cosmos is merely an accident a contingent epiphenomena and that its
purpose as a component of that vast system is merely to destroy itself no sooner than it emerges? I find such gross pessimism to be both ill founded and, ultimately, misanthropic.

The truth of the matter is that, even though as individuals we may be self-determined, the Cosmos, of which we are an inextricable part, is determined. This determination indicates to me

the exponential technological evolution that leads to singularity is a natural part of the cooling and
that

development of the Cosmos. This does not mean that human civilization can abandon all pretenses of responsibility, but what it does mean is that as our
civilization approaches technological singularity our true nature will become manifest. We will, at that point, be denuded.
The inconceivable technological power unleashed by the event of the singularity will empower man to fulfill that
which he most fundamentally desires, or in other terms, technological singularity will be the point at which man bears the fruit that was latent in the seed which is man.

In any case what ever happens will be a natural occurrence that is no more capable of being controlled than is the gravitational constant or the speed limit of light .

Page 718 of 1481


Page 719 of 1481
**Deleuze and Guattari**

Page 720 of 1481


Perms
THE 1AC IS A SLOW EXPERIMENT; EVEN IF IT FAILS TO LIBERATE US, IT IS BETTER
THAN THE NEGATIVES FAST REJECTION AND OVERDOSE, WHICH LEADS TO
COLLAPSE AND DEATH

Gilles Deleuze, Professor of Philosophy at the University of Paris; and Felix Guattari, psychoanalyst,
1987, A Thousand Plateaus, pp. 160-161h

You have to keep enough of the organism for it to reform each dawn; and you have to keep small supplies
of signifiance and subjectification, if only to turn them against their own systems when the circumstances
demand it, when things, persons, even situations, force you to; and you have to keep small rations of
subjectivity in sufficient quantity to enable you to respond to the dominant reality. Mimic the strata. You
dont reach the BwO, and its plane of consistency, by wildly destratifying. That is why we encountered the
paradox of those emptied and dreary bodies at the very beginning: they had emptied themselves of their
organs instead of looking for the point at which they could patiently and momentarily dismantle the
organization of the organs we call the organism. There are, in fact, several ways of botching the BwO:
either one fails to produce it, or one produces it more or less, but nothing is produced on it, intensities do
not pass or are blocked. This is because the BwO is always swinging between the surfaces that stratify it
and the plane that sets it free. If you free it with too violent an action, if you blow apart the strata without
taking precautions, then instead of drawing the plane you will be killed, plunged into a black hole, or even
dragged toward catastrophe. Staying stratifiedorganized, signified, subjectedis not the worst that can
happen; the worst that can happen is if you throw the strata into demented or suicidal collapse, which
brings them back down on us heavier than ever. This is how it should be done: Lodge yourself on a
stratum, experiment with the opportunities it offers, find an advantageous place on it, find potential
movements of deterritorialization, possible lines of flight, experience them, produce flow conjunctions
here and there, try out continuums of intensities segment by segment, have a small plot of new land at all
times. It is through a meticulous relation with the strata that one succeeds in freeing lines of flight,
causing conjugated flows to pass and escape and bringing forth continuous intensities for a BwO.
Connect, conjugate, continue: a whole diagram, as opposed to still signifying and subjective programs.
We are in a social formation; first see how it is stratified for us and in us and at the place where we are;
then descend from the strata to the deeper assemblage within which we are held; gently tip the
assemblage, making it pass over to the side of the plane of consistency. It is only there that the BwO
reveals itself for what it is: connection of desires, conjunction of flows, continuum of intensities. You have
constructed your own little machine, ready when needed to be plugged into other collective machines.
Castaneda describes a long process of experimentation (it makes little difference whether it is with peyote
or other things): let us recall for the moment how the Indian forces him first to find a place, already a
difficult operation, then to find allies, and then gradually to give up interpretation, to construct flow by
flow and segment by segment lines of experimentation, becoming-animal, becoming-molecular, etc. For
the BwO is all of that: necessarily a Place, necessarily a Plane, necessarily a Collectivity (assembling
elements, things, plants, animals, tools, people, powers, and fragments of all of these; for it is not my
body without organs, instead the me (moi) is on it, or what remains of me, unalterable and changing in
form, crossing thresholds).

Page 721 of 1481


Alternative Increases Oppression

IN PRACTICE THEIR ALTERNATIVE WILL FURTHER TYRANNICAL CONTROL AND


GENOCIDE

Richard Barbrook, coordinator of the Hypermedia Research Centre at the University of Westminster,
8/27/1998, http://amsterdam.nettime.org/Lists-Archives/nettime-l-9808/msg00091.html, accessed 3/3/03

Deleuze and Guattari enthusiastically joined this attack against the concept of historical progress. For them, the 'deterritorialisation' of urban society was the solution to the
contradiction between participatory democracy and revolutionary elitism haunting the New Left. If the centralised city could be broken down into 'molecular rhizomes', direct
democracy and the gift economy would reappear as people formed themselves into small nomadic bands. According to Deleuze and Guattari, anarcho-communism was not the
'end of history': the material result of a long epoch of social development. On the contrary, the liberation of desire from semiotic oppression was a perpetual promise: an ethical
stance which could be equally lived by nomads in ancient times or social movements in the present. With enough intensity of effort, anyone could overcome their hierarchical

rhetoric of unlimited freedom


brainwashing to become a fully-liberated individual: the holy fool.<21> Yet, as the experience of Frequence Libre proved, this

contained a deep desire for ideological control by the New Left vanguard. While the nomadic fantasies of A
Thousand Plateaus were being composed, one revolutionary movement actually did carry out Deleuze and Guattari's dream

of destroying the city. Led by a vanguard of Paris-educated intellectuals, the Khmer Rouge overthrew an oppressive

regime installed by the Americans. Rejecting the 'grand narrative' of economic progress, Pol Pot and his organisation instead
tried to construct a rural utopia. However, when the economy subsequently imploded, the regime embarked on ever
more ferocious purges until the country was rescued by an invasion by neighbouring Vietnam. Deleuze and Guattari had claimed that the

destruction of the city would create direct democracy and libidinal ecstasy. Instead, the application of such
anti-modernism in practice resulted in tyranny and genocide. The 'line of flight' from Stalin had led to Pol Pot.

DELEUZE AND GUATTARI'S BELIEF IN TRANSFORMATION THROUGH FREEDOM


FROM DIALECTICAL OPPOSITION FAILS THE FIGURES AND INSTITUTIONS WHICH
COULD CREATE THIS FREEDOM ARE REAPPROPRIATED BY CONTEMPORARY
OPPOSITIONAL POLITICS, FORECLOSING EXITS FROM THE EXISTING POLITICAL
SYSTEM

Mann, Prof of English at Pomona, 95 (Paul, Stupid Undergrounds, PostModern Culture 5:3, Project MUSE)

Intellectual economics guarantees that even the most powerful and challenging work cannot protect itself
from the order of fashion. Becoming-fashion, becoming-commodity, becoming-ruin. Such instant, indeed retroactive ruins, are the virtual landscape of the stupid
underground. The exits and lines of flight pursued by Deleuze and Guattari are being shut down and rerouted by

the very people who would take them most seriously. By now, any given work from the stupid underground's critical apparatus is liable to be tricked
out with smooth spaces, war-machines, n - 1s, planes of consistency, plateaus and deterritorializations, strewn about like tattoos on the stupid body without organs. The nomad is

already succumbing to the rousseauism and orientalism that were always invested in his figure; whatever
Deleuze and Guattari intended for him, he is reduced to being a romantic outlaw, to a position opposite
the State, in the sort of dialectical operation Deleuze most despised. And the rhizome is becoming just
another stupid subterranean figure. It is perhaps true that Deleuze and Guattari did not adequately
protect their thought from this dialectical reconfiguration (one is reminded of Breton's indictment against Rimbaud for not having prevented, in
advance, Claudel's recuperation of him as a proper Catholic), but no vigilance would have sufficed in any case. The work of Deleuze

and Guattari is evidence that, in real time, virtual models and maps close off the very exits they indicate.
The problem is in part that rhizomes, lines of flight, smooth spaces, BwOs, etc., are at one and the same
time theoretical-political devices of the highest critical order and merely fantasmatic, delirious, narcissistic
models for writing, and thus perhaps an instance of the all-too-proper blurring of the distinction between

Page 722 of 1481


criticism and fantasy. In Deleuze-speak, the stupid underground would be mapped not as a margin surrounding a fixed point, not as a fixed site determined strictly by its
relation or opposition to some more or less hegemonic formation, but as an intensive, n-dimensional intersection of rhizomatic plateaus. Nomadology and rhizomatics conceive such a "space"
(if one only had the proverbial nickel for every time that word is used as a critical metaphor, without the slightest reflection on what might be involved in rendering the conceptual in spatial
terms) as a liquid, colloidal suspension, often retrievable by one or another techno-metaphorical zoning (e.g., "cyberspace"). What is at stake, however, is not only the topological
verisimilitude of the model but the fantastic possibility of nonlinear passage, of multiple simultaneous accesses and exits, of infinite fractal lines occupying finite social space. In the strictest
sense, stupid philosophy. Nomad thought is prosthetic, the experience of virtual exhilaration in modalities already mapped and dominated by nomad, rhizomatic capital (the political
philosophy of the stupid underground: capital is more radical than any of its critiques, but one can always pretend otherwise). It is this very fantasy, this very narcissistic wish to see oneself
projected past the frontier into new spaces, that abandons one to this economy, that seals these spaces within an order of critical fantasy that has long since been overdeveloped, entirely
reterritorialized in advance. To pursue nomadology or rhizomatics as such is already to have lost the game. Nothing is more crucial to philosophy than escaping the dialectic and no project is
more hopeless; the stupid-critical underground is the curved space in which this opposition turns back on itself. It is not yet time to abandon work that so deeply challenges our intellectual
habits as does that of Deleuze and Guattari, and yet, before it has even been comprehended, in the very process of its comprehension, its fate seems secure. One pursues it and knows that
the pursuit will prove futile; that every application of these new topologies will only serve to render them more pointless. The stupid optimism of every work that takes up these figures is, by
itself, the means of that futility and that immanent obsolescence. One must pursue it still.

Page 723 of 1481


Deleuze Bad (General)

DELEUZIAN PERSPECTIVISM COLLAPSES INTO NEOCONSERVATIVE SUPPORT FOR


THE STATUS QUO BECAUSE IT DOESNT PROVIDE A SOLID POINT OF CRITICISM OF
OPPRESSION

Zerzan no date
[John, primitivist, The catastrophe of postmodernism, the Athenaeum Reading Room, www.evans-
experimentalism.freewebspace.com/zerzan01.htm, acc 1-15-05]

The dilemma of postmodernism is this: how can the status and validity of its theoretical
approaches be ascertained if neither truth nor foundations for knowledge are admitted? If we
remove the possibility of rational foundations or standards, on what basis can we operate? How
can we understand what the society is that we oppose, let alone come to share such an
understanding? Foucault's insistence on a Nietzschean perspectivism translates into the
irreducible pluralism of interpretation. He relativized knowledge and truth only insofar as
these notions attach to thought-systems other than his own, however. When pressed on this
point, Foucault admitted to being incapable of rationally justifying his own opinions. Thus the
liberal Habermas claims that postmodern thinkers like Foucault, Deleuze, and Lyotard
are `neoconservative' for offering no consistent argumentation to move in one social
direction rather than another. The pm embrace of relativism (or `pluralism') also means there
is nothing to prevent the perspective of one social tendency from including a claim for
the right to dominate another, in the absence of the possibility of determining standards.

Page 724 of 1481


D & G Exclude Women

D & G EXCLUDE WOMEN

Alice Jardine, Professor of Romance Languages and Literatures at Harvard University, 1984,
http://substance.arts.uwo.ca/44/04jard44.html, accessed 2/21/03

"sexuality itself" which is the


Why then do D + G privilege the word woman? First, as they explain through a series of unanalyzed stereotypes, because it is

ultimate, uncontrollable becoming, when it can manage to escape immediate Oedipalization. ("Sexuality passes
through the becoming-woman of /the/ man and the becoming-animal of the human" [MP, p. 341].) But also because, as "introductory power," "Woman"

is both the closest to the category of "Man" as majority, and yet she remains a distinct minority. D + G explain
that the notions of majority and minority here should not be opposed in any purely quantitative way: "Let us suppose that the constant or standard is Manany white-male-
adult-city-dweller-speaking a standard language-European-heterosexual (the Ulysses of Joyce or of Ezra Pound). It is obvious that "the Man" has the majority, even if he is less
numerous than the mosquitoes, children, Blacks, peasants, homosexuals . . . etc." (MP, p. 133). The problem is not to gain, or accede to, the majority, but to become a minority;
and this is particularly crucial for women if they desire to remain radical, creative, without simply becoming (a) Man: The only becoming is a minority one. Women, regardless of
their number, are a minority, definable as a state or sub-set; but they only create by rendering possible a becoming, of which they do not have the ownership, into which they
themselves must enter, a becoming-woman which concerns all of mankind, men and women included. (MP, p. 134) The woman who does not enter into the "becoming
woman" remains a Man, remains "molar," just like men: Woman as a molar entity must become woman, so that man as well may become one or is then able to become one. It
is certainly indispensable that women engage in molar politics, in terms of a conquest which they conduct from their organization, from their own history, from their own
subjectivity: "We as women . . ." then appears as the subject of the enunciation. But it is dangerous to fall back upon such a subject, which cannot function without drying up a
spring or stopping a flood. The Song of life is often struck up by the driest women, animated by resentment, by the desire for power and by cold mothering.... (MP, p. 339) That
is, woman (with her obligatory connotations: "transparent force, innocence, speed," [MP, p. 354] is what Man (both men and women: "virility, gravity," [MP, p. 354]) must
become. There must be no "becoming man" because he is always already a majority. "In a certain way, it's always 'man' who is the subject of a becoming.... A woman has to
Man is always the subject of any becoming, even if "he"
become woman, but in a becoming-woman of all of mankind" (MP, p. 357). That is,

is a woman. A woman who is not a "woman-become" is a Manand a subject to that extent and to that extent
only. Woman is never a subject but a limita border of and for Manthe "becoming woman" is l'avenir de l'homme tout entier
the future of all Mankind. For D + G, She is what the entire world must become if Man men and womenis truly
to disappear. But to the extent that women must "become woman" first (in order for men, in D + G's words, to
"follow her example"), might that not mean that she must also be the first to disappear? Is it not possible that the process of
"becoming woman" is but a new variation of an old allegory for the process of women becoming obsolete? There
would remain only her simulacrum: a female figure caught in a whirling sea of male configurations. A silent, mutable,

head-less, desire-less, spatial surface necessary only for His metamorphosis? Physicists say: Holes are not the absence of
particles, but particles going faster than light. Flying anuses, rapid vaginas, there is no castration. Gilles Deleuze and Felix Guattari, Mille Plateaux Most important theorists
have a repertory of exemplary fictions, fictions that they call upon frequently to interact with their specific theories in creative if predictable ways. Between the scene of Lacanian
psychoanalysis and that of Lol V. Stein's ravishing, for example, the privileged rapport is one of repetition: for Lacan, Marguerite Duras understood and repeated his teachings
without him.19 Or, between the invagination of Derrida's ecriture and that of the narrator in Maurice Blanchot's L'Arret de mort, what is privileged is the process of mime: for
Derrida, Blanchot understood his writings with him, inseparably. 20 D + G's exemplary fiction writers include Lewis Carroll, Franz Kafka, Pierre Klossowski, and Michel
Tournierto mention only a few. What all of these writers' texts share with those of D + G is the surface quality of their figures: the privileged modality of relationship between
the configurations of Deleuzian becoming and those of fiction is allegory. This is made most clear through Deleuze's essay on Tournier's 1967 novel, Vendredi, ou les limbes du
Pacifique. 21 There it is no longer a question of whether Duras's Lol, as hysterical body, is or is not a subject of narrative; of whether Blanchot's J. and N., as organs of a
hysterical text, are or are not simply new angles for modernity. For here it is a question of Speranza, a true Body-without-Organs: a woman who is not a woman but a female
figure (an island), a space to be unfolded, molded, into new configurations for the metamorphosis of Man. In t, we first stumble across Robinson just after he has been
shipwrecked on his island. Finding himself completely alone, the Only and perhaps Last Man on this island, he first succumbs to depression, evasion, infantile panicleaving
himself exposed, helpless. For Deleuze, this signals Man's first steps outside of intersubjectivity: "What happens when others are lacking in the structure of the world? There
only reigns the brutal opposition of the sun and the earth, of an insupportable light and an obscure abyss . . ." (LS, p. 355). To avoid loss of self, however, this twentieth-century
Robinson first tries the old solutions. He creates for himself a task: he spends months, perhaps years, perhaps even decadesthe length of time does not matterbuilding a
new boat-structure in which he might escape. But once the vessel is completed, it is too large, too heavy, and too cumbersome for him to push to the sea towards freedom.
Robinson succumbs, once again, to the deepest depressionand, indeed, abjection: He kept eating, his nose to the ground, unspeakable things. He went underneath himself
and rarely missed rolling in the soft warmth of his own excrement.... He moved about less and less, and his brief movements always brought him back to the wallow. There he
kept losing his body and delivering himself of its weight in the hot and humid surroundings of the mud, while the noxious emanations of the stagnating waters clouded his mind.
(VLP, p. 38) Haunted by his lost sister (the one who died young), his mother (sometimes cold but always self-sacrificing), his wife (left behind in old England), Robinson-the-
Man has a brush with what the Man calls insanity. And so, as a Man, Robinson decides that he must henceforth master both himself and the island if he is to survive. He sets
about building a kingdom: he creates a calendar; he invents a way to write; he builds a house, cultivates the land. He names the island Speranza and realizes that now, in time
and mastery, she is his slave. Woman is, therefore, no longer absent from Man's adventures, even though he remains outside of inter-subjectivity: Besides, it seemed to him,
when looking a certain way at the map of the island which he had sketched approximately, that it could represent the profile of a headless female body, a woman, yes, seated
with her legs folded under her, in a posture within which it would have been impossible to sort out what there was of submission, of fear, or of simple abandonment. This idea
crossed his mind, then it left him. It would come back. (VLP, p. 46)22 In spite of various humiliations, depressions, and disappointments, Robinson continues his mastery over
Speranza. A decisive step is the introduction of time into this one-Man kingdom with a kind of primitive clock. In the "future," Robinson succumbs to his former states of
abjection within the space of Speranza only when that clock of progress stops. Slowly, however, and in spite of his frenzied, productive activity, Robinson realizes that his
relationship with "himself" is changing. His "self," in fact, can no longer exist in a world without the Other. Robinson is ready to lose his Self, his Manhood: "Who I? The question
is far from being pointless. It isn't even insoluble. Because if it's not him, it must be Speranza. There is from here on a flying I which will sometimes alight on the man,
sometimes on the island, and which makes of me, in turn, one or the other" (VLP, pp. 88-89).

Page 725 of 1481


A2 Life is Carbon

THE AFF IS WRONG THE HUMN BODY ISNT LIMITED TO CARBON, BUT IS
SILICONIC IN THE MACHINIC WAY IT EMERGES FROM INTERSUBJECTIVE FLOWS
LIKE COMMUNICATION AND CAPITAL, INDICATING MEANING TO LIFE BEYOND THE
MATTER THAT COMPOSES US

Beddoes no date
[Diane J., Material gadget, Breeding Demons: A critical enquiry into the relationship
between Kant and Deleuze with specific reference to women, Transmat,
www.cinestatic.com/trans-mat/Beddoes/BD7s4.htm, acc 1-15-05]

Deleuze notes that biologists have often questioned why life is effected through carbon,
rather than through silicon, and goes on to say that la vie des machines modernes passe par
le silicium (the life of modern machines runs through silicon).[377] This is where becoming-
women moves, where money released from capital moves, where life becomes non-
organic, nature becomes a thinking machine, infinities of tiny demons leap, effecting a co-
ordinated and fluid movement, eroding the statues of power, the historical . Becoming-
woman moves towards becoming-imperceptible, but women do not dissolve or disappear in
that movement: it is rather than life itself becomes mobile, because it is not longer in the
womb nor arranged in the organisms which emerge from them, but instead becomes a
movement, a cycle that turns on its hinges. Humans are no longer the privileged class, but
the surrogate reproductive machinery of a machinic phylum which is passing across into a
different base, in a movement which effects the conjunction of teleology and mechanism, and
transforming the nature of intelligence.

HUMAN IDENTITY IS MORE THAN CARBON ITS CODED BY COMMUNICATION


FLOWS, THAT RECOGNITION IS NECESSARY TO RESIST CAPITALIST ALIENATION

Brassier 2001
[Ray, Doctoral candidate at University of Warwick, Alien Theory: The Decline of
Materialism in the Name of Matter, Doctoral Thesis, April,
www.cinestatic.com/trans-mat/Brassier/ALIENTHEORY.pdf, acc 1-14-05//uwyo]

Yet it is a failure which transcendental scepticism may yet help

circumvent through the Alien-subjects unilateralising force-(of)-thought; an

intrinsically sceptical force which constitutes an instance of a priori cognitive

resistance to those epistemic norms and informational codes via which a

triumphant World-Capitalism maintains the structural isomorphy between

Page 726 of 1481


material power and informational force, thereby ensuring its quasitranscendental

dominion over all cognitive experience. A transcendental

scepticism agrees with eliminative naturalism: human beings are simply carbonbased

information processing machines. But it also recognises the necessity of

cross-pollinating that assessment born of evolutionary reductionism with

transcendental insight; an insight which consists in radicalising and generalising

Marxs identification of the material infrastructure as the ultimate determinant

for the ideological superstructure315: World-Capitalism is now the global

megamachine determining a priori the cognitive parameters within which the

phenomenological micromachinery of organically individuated sapience

operates. By acknowledging the fact that political intervention can no longer

afford to ignore this insight; by recognising that empirical agency alone is

incapable of circumventing capitals all-encompassing universality as World-


Capitalism, transcendental scepticism constitutes an instance of a priori

political resistance.

Page 727 of 1481


A2 Death Doesnt Destroy Being: 2AC (1/2)

FIRST, EVEN IF DEATH DOESNT KILL BEING, IT DOES ANNIHILATE


CONSCIOUSNESSES THAT ARE COMPOSED OF PRECISE COMBINATIONS OF ENERGY
AND MATTER, MEANING THAT DEATH EXTINGUISHES THOUGHT PROCESSES THAT
PEOPLE ARE ATTACHED TO, MEANING THAT FORCED DEATH IS VIOLENT AND
UNDESIRABLE

SECOND, THIS IGNORES THE ROLE OF COMMUNICATION IN CREATING HUMAN


IDENTITY. WERE MORE THAN THE MATTER OF OUR PARTS, BUT CREATE MEANING
THROUGH COMMUNICATIVE PROCESSES, SOMETHING DESTROYED BY DEATH

THIRD, CARBON ATOMS ARENT THE KEY COMPONENT OF LIFE, COMPLEX


INFORMATION PROCESSING IS, MEANING THAT DEATH CAUSES ANNIHILATION OF
CONSCIOUSNESS

Tipler 94
[Frank J., Professor of Mathematical Physics at Tulane University, The Physics of
Immortality: Modern Cosmology, God and the Resurrection of the Dead, New York:
Doubleday, 1994, 124-5//uwyo-ajl]

IN ORDER TO INVESTIGATE WHETHER LIFE can continue to exist forever, I shall need to define
"life" in physics language. I claim that a "living being" is any entity which codes information (in
the physics sense of this word) with the information coded being preserved by natural selection.
Thus "life" is a form of information processing, and the human mind-and the human soul-is
a very complex computer program. Specifically, a "person" is defined to be a computer
program which can pass the Turing test, which was discussed in Chapter II.

This definition of "life" is quite different from what the average person-and the average
biologist-would think of as "life." In the traditional definition, life is a complex process
based on the chemistry of the carbon atom. However, even supporters of the traditional
definition admit that the key words are "complex process" and not "carbon atom."
Although the entities everyone agrees are "alive" happen to be based on carbon chemistry,
there is no reason to believe that analogous processes cannot be based on other
systems. In fact, the British biochemist A. G. Cairns-Smith! has suggested that the first living
beings--':our ultim:ate ancestors-were based on metallic crystals, not carbon. If this is true,
then if we insist that living beings must be based on carbon chemistry, we would be forced to
conclude that our ultimate ancestors were not alive. In Cairns-Smith's theory, our ultimate
ancestors were self-replicating patterns of defects in the metallic crystals. Over time, the
pattern persisted, but was transferred to another substrate: carbon molecules. What is
important is not the substrate but the pattern, and the pattern is another name for
information.

Page 728 of 1481


But life of course is not a static pattern. Rather, it is a dynamic pattern that persists overtime.
It is thus a process. But not all processes are alive. The key feature of the "living" patterns is that
their persistence is due to a feedback with their environment: the information coded in
the pattern continually varies, but the variation is constrained to a narrow range by this
feedback. Thus life is, as I stated, information preserved by natural selection.

Page 729 of 1481


Page 730 of 1481
A2 Death Doesnt Destroy
Being: 2AC (2/2)
FOURTH, EVEN IF THERE ARE OTHER POSSIBILTIES AFTER DEATH, THE IDENTITIES
THAT WERE ATTACHED TO WILL BE EXTINGUISHED BECAUSE CONSCIOUSNESS
COMES FROM INFORMATION PROCESSSING THAT REQUIRES PARTICULAR
SEQUENCES OF QUANTUM STATES TO OCCUR

Tipler 94
[Frank J., Professor of Mathematical Physics at Tulane University, The Physics of
Immortality: Modern Cosmology, God and the Resurrection of the Dead, New York:
Doubleday, 1994, 221-3//uwyo-ajl]

The Bekenstein Bound follows from the basic postulates of quantum theory combined with the
further assumptions that (1) the system is bounded in energy, and (2) the system is
bounded, or localized, in space. A rigorous proof of the Bekenstein Bound would require
quantum field theory, but it is easy to describe in outline why quantum mechanics leads to
such a bound on the information coded in a bounded region. In essence, the Bekenstein
Bound is a manifestation of the uncertainty principle. Recall that the uncertainty principle tells
us that there is a limit to the precision with which we can measure the momentum of a particle
and its position. More precisely, the uncertainty principle says that the location of a point in
phase space-a concept I defined in Chapter III-cannot be defined more closely thal1 Planck's
constant h. Since a system's state is defined by where it is located in phase space, this
means that the number of possible states is less than or equal to the size of the phase
space region the system could be in, divided by the size of the minimum phase space
size, Planck's constant. (I've given a mathematical expression of this argument in the Appendix
for Scientists.) This state counting procedure, based on there being an absolute minimum size h
to a phase space interval, is an absolutely essential method of quantum statistical mechanics.
We have already used it in Chapter III to prove the almost periodicity of a bounded quantum
system. It is confirmed by the thousands of experiments which have been based on this
counting method.9 In high energy particle physics, any calculation of the "cross section"
requires counting the possible number of particle initial and final states, and the above state
counting method is used.lO The cross section, which is the measure of how many particles
scatter in a particular direction when they collide in particle accelerators, is the basic quantity
tested in particle physics. The Bekenstein Bound on the number of possible states is thus
confirmed by the correctness of the calculated cross sections. In summary, the Bekenstein
Bound on the total information that can be coded in a region is an absolute solid
conclusion of modern physics, a result as solid as the Rock of Gibraltar.

One can also use the Bekenstein Bound to deduce an upper bound to the rate of
information processing. The time for light to cross a sphere of a given diameter is equal to the
diameter of the sphere divided by the speed of light. Since a state inside the sphere cannot
completely change until a signal has time to travel trom one side to the other, the rate of
information processing is bounded above by the above Bekenstein Bound divided by this
time interval. Putting in the numbers (details in the Appendix for Scientists), we calculate that
the rate of state change is less than or equal to 4 X 1051 bits per second, multiplied by the mass
of the system in kilograms. That is, the rate of information processing possible for a system

Page 731 of 1481


depends only on the mass of the system, not on its spatial size or on any other variable. So a
human being of mass 100 kilograms cannot change state more rapidly than about 4 X 1053
times per second. This number is of course enormous-and in fact a human will probably change
state much, much more slowly than this-but it's finite.

Page 732 of 1481


A2 Life is Meaningless Because the Sun Will Go
Out: 2AC

FIRST, THERES NO WARRANT FOR WHY THE DEATH OF OUR PLANET IN BILLIONS
OF YEARS MAKES LIFE THAT EXIST NOW MEANINGLESS. EACH INDIVIDUALS
CREATES CONTINGENT VALUE FOR THEIR LIFE THROUGH COMMUNICATION AS
DEMONSTRATED BY THE HABERMAS EVIDENCE AND TO FORCE DEATH UPON
THEM BECAUSE OF AN EVENT IN THE UNFATHOMABLE FUTURE IS REPUGNANT

SECOND, HUMANITY WILL ADAPT TO THE DESTRUCTION OF ITS HABITAT BY


INEVITABLY PROGRESSING TO A TYPE III CIVILIZATION

Kaku 95
[Michio, Prof. of theoretical physics at the City College, NY, Hyperspace: A Scientific
Odyssey Through Parallel Universes, Time Warps, and the 10th Dimension. New York:
Ancor Books, March, 281//uwyo-ajl]

Taking the larger view of the development of civilization, Dyson also believes that, at the
current rate of development, we may attain Type I status within a few centuries. He does
not believe that making the transition between various types of civilizations will be very difficult.
He estimates that the difference in size and power separating the various types of
civilizations is roughly a factor of 10 billion. Although this may seem like a large nuimber, a
civilization growing at the sluggish rate of 1 percent per year can expect to make the
transition between the various civilizations within 2,500 years. Thus it is almost
guaranteed that a civilization can steadily progress toward Type III status.

THIRD, THIS OUTWEIGHS ALL OTHER ARGUMENTS BECAUSE 20TH CENTURY


GENOCIDE DEMONSTRATES THE SHEER HORROR OF EXTERMINATING LIFE

Tipler 94
[Frank J., Professor of Mathematical Physics at Tulane University, The Physics of
Immortality: Modern Cosmology, God and the Resurrection of the Dead, New York:
Doubleday, 1994, 11-12//uwyo-ajl]

I shall obtain a hold on this future reality by focusing attention on the physics relevant to the
existence and behavior of life in the far future. I shall provide a physical foundation for
eschatology-the study of the ultimate future--by making the physical assumption that the
universe must be capable of sustaining life indefinitely; that is, for infinite time as experienced

Page 733 of 1481


by life existing in the physical universe. All physical scientists should take this assumption
seriously because we have to have some theory for the future of the physical universe--since it
unquestionably exists-and this is the most beautiful physical postulate: that total death is not
inevitable. All other theories of the future necessarily postulate the ultimate extinction of
everything we could possibly care about. I once visited a Nazi death camp; there I was
reinforced in my conviction that there is nothing uglier than extermination. We physicists
know that a beautiful postulate is more likely to be correct than an ugly one. Why not adopt this
Postulate of Eternal Life, at least as a working hypothesis? I shall show in Chapter n that the
universe is in fact capable of sustaining life at least another million trillion years.
Specifically, I shall demonstrate that it is technically feasible for life to expand out from the
Earth and engulf the entire universe, and that life must do so if it is to survive.

Page 734 of 1481


Page 735 of 1481
**Derrida**

Page 736 of 1481


A2 Deconstruction
DERRIDEAN DECONSTRUCTION PREVENTS POLITICAL STRATEGIZING

Crawford, Prof of Humanities and Comparative Lit @ U of Minnesota, 90 (Claudia, Nietzsche as Postmodernist?,
Ed. Clayton Koelb, P. 197)

Page 737 of 1481


Page 738 of 1481
A2 New International (1/2)
THERE IS ZERO MEANS TO ACTUALIZE THEIR ALTERNATIVE DERRIDA
DECONSTRUCTS HIMSELF TO DEATH

Eagleton, Professor of Cultural Theory at the University of Manchester, 99 (Terry, Marxism without Marxism, in
Ghostly Demarcations: A Symposium on Jacques Derridas Specters of Marx, edited by Michael Sprinker)

There is an exasperating kind of believer who holds what he does until he meets someone else who holds the same. At this point, confronted with the bugbear of an `orthodoxy', he starts

Derrida, who like many a postmodernist appears to feel (it is a matter


nervously to retract, or at least to qualify. There is more than a touch of this adolescent perversity in

that the dominant is ipso facto demonic and the marginal precious per se. One
of sensibility rather than reasoned conviction)

condition of the unthinking postmodern equation of the marginal with the creative, apart from a convenient obliviousness to such marginal groups as Fascists, is
the rolling back of political movements which are at once mass and oppositional . The mark of a genuine radical is a hearty
desire to stop having to be so obdurately oppositional, a sentiment one can hardly imagine as dear to the heart of a deconstructionist. If one takes the point of James Joyce's retort to an
invitation to return to a newly independent Irish republic - `So as to be its first critic?' - one also registers the self-indulgence. Derrida has now taken Marxism on board, or at least dragged it
halfway up the gangplank, because he is properly enraged by liberal-capitalist complacency; but there is also something unavoidably opportunist about his political pact, which wants to

What he wants, in effect, is a Marxism


exploit Marxism as critique, dissent, conveniently belabouring instrument, but is far less willing to engage with its positivity.

without Marxism, which is to say a Marxism on his own coolly appropriative terms. `We would be tempted to distinguish this spirit of the Marxist critique ... at once from
Marxism as ontology, philosophical or metaphysical system, as "dialectical materialism", from Marxism as historical materialism or method, and from Marxism incorporated in the apparatuses
of party, State, or workers' International.' It would not be difficult to translate this into the tones of a (suitably caricatured) liberal Anglicanism: we must distinguish the spirit of Christianity
from such metaphysical baggage as the existence of God, the divinity of Christ, organized religion, the doctrine of the resurrection, the superstition of the Eucharist and the rest. Or: one would
wish to distinguish the spirit of deconstruction from the dreary intellectual paraphernalia of `writing', `difference', `trace', organized journals and conventions, formal reading groups,
movements to install the teaching of philosophy in French schools and so on. It is entirely possible to approve of the spirit of the Huns, with all its admirable robustness, while deploring what

they actually got up to. If Derrida thinks, as he appears to do, that there can be any effective socialism without organization,
then he is merely the victim of some academicist fantasy which he has
apparatuses and reasonably well-formulated doctrines and programmes,
somehow mistaken for an enlightened anti-Stalinism. (He has, in fact, no materialist or historical analysis of Stalinism whatsoever, as opposed to an ethical rejection of it, unlike many more

orthodox currents of Marxism.) The truth is that he is hardly concerned with an effective socialism at all. Deconstruction , with its
is a kind of intellectual equivalent of
preoccupation with slippage, failure, aporia, incoherence, not-quiteness, its suspicion of the achieved, integral or controlling,

a vaguely leftish commitment to the underdog, and like all such commitments is nonplussed when those it speaks up for come to power.
Poststructuralism dislikes success , a stance which allows it some superbly illuminating insights into the pretensions of monolithic literary texts or ideological self-identities and

leaves it a mite wrong-footed in the face of the African National Congress. Derrida's indifference to almost all of the actual historical or theoretical

manifestations of Marxism is a kind of empty transcendence - a typically deconstructie trumping of some alternative position which leaves
one's own case invulnerable only in proportion to its contentlessness. Much the same can be said of his curiously empty, formalistic messianism, which voids this rich theological tradition of
its content and retains its ghostly impulse only, somewhat akin to the Kafka who (as Walter Benjamin remarks) is left with nothing but the transmissible forms of a tradition which has

The critical, negative passion of his politics in this book is one which ought rightly to embarrass
dwindled to nothing.

every academic radical for whom deconstruction is a sexy form of common-or-garden scepticism, or yet another way of keeping
the literary canon alive by plodding through it yet again, this time with a scalpel in hand. Instead of singing the advent of the ideal of liberal democracy and of the capitalist market in the
euphoria of the end of history, instead of celebrating the `end of ideologies' and the end of the great emancipatory discourses, let us never neglect this obvious macroscopic fact, made up of
innumerable singular sites of suffering: no degree of progress allows one to ignore that never before, in absolute figures, have so many men, women, and children been subjugated, starved,
or exterminated on the earth. This is not the kind of thing that is likely to go down well in Ithaca or Irvine, where they learnt long ago that ideology had ended and the great emancipatory

discourses run thankfully aground. And what does Derrida counterpose, in the very next paragraph, to the dire condition he so magnificently
denounces? A `New International', one `without status, without title, and without name ... without party, without country, without national community ...' And, of
course, as one gathers elsewhere in the book, without organization , without ontology, without method, without apparatus. It is the ultimate

poststructuralist fantasy: an opposition without anything as distastefully systemic or drably `orthodox' as an


opposition, a dissent beyond all formulable discourse, a promise which would betray itself in the act of fulfilment, a perpetual excited
openness to the Messiah who had better not let us down by doing anything as determinate as coming. Spectres of Marxism indeed. 85-87

Page 739 of 1481


Page 740 of 1481
A2 New International (2/2)
THEIR CALL FOR DEMOCRACY TO COME IS COUNTER-PRODUCTIVE THEIR
CHANGE IS JUST REFORMISM THEY END UP CRUSHING THE REVOLUTION THAT
SOLVES BETTER

Lewis, member of the International Socialist Organization & Spanish Professor at the University of Iowa, 99 (Thomas, The Politics of
Hauntology in Derridas Specters of Marx, in Ghostly Demarcations: A Symposium on Jacques Derridas Specters of Marx, edited by Michael Sprinker)

New forms of struggle and especially new agents of social change, it is claimed, must either be found or theorized into existence.
Hence, the perceived need arises for something on the order of Derrida's New International `without common
belonging to a class'. I argued above that the contemporary working class includes both `blue collar' and `white collar' workers, and that the internationalization of capitalism has created a
growing international working class. I thereby sought to contest the claim that the working class is increasingly smaller and irrelevant as a social force. I also indicated that divisions among the
working class along lines of gender, race, nationality and sexual orientation have traditionally been the object of intense activity and theoretical discussion within Marxism. While recognizing

it is possible to overcome such divisions through common struggle. Finally, I


the formidable obstacles encountered, I emphasized that

argued thatonly the working class - that is, individuals who may embody a number of specific identities but who act collectively on the basis of their shared interests as
workers - possesses the structural capacity both to bring down capitalism and to create socialism. On this view, it is both theoretically and
politically necessary to affirm the working class as the primary agent of social transformation. Derrida's SM provides a stinging indictment of the contemporary world system, as well as a

SM also presents an elaborate case for reform


serious critique of recently published apologies for capitalism. As I have endeavored to show, however,

socialism over and against revolutionary socialism . This case is based on what, in a friendly spirit, might be termed a `misreading' of the Russian
Revolution. Moreover, the main tenet of the case is the repudiation of the notion that the working class remains

central to the project of winning socialism. Among the more astounding dimensions of SM, therefore, surely must figure the social contexts in which the
book appears. Derrida suggests a reformist road to socialism precisely at the end of a period in which the

political and moral hollowness of traditional social democracy could not be in greater evidence . Socialist parties
all over Western Europe, but particularly in France, Spain, Italy and Germany, have failed to preserve - much less extend - the gains for workers once embodied in the so-called `welfare state'
(Anderson and Camiller 1994; Ross and Jensen 1994; Camiller 1994; Abse 1994; and Padgett and Paterson 1994). These same Socialist parties have not just collaborated with but in numerous
instances have actually initiated the attacks on workers, immigrants and the poor. As if all that were not enough, European social democracy has signally failed to organize an effective
movement from below against the resurgence of Fascism and neo-Fascism. Everything that can be said in criticism of Europe's Socialist parties equally applies to the Democratic Party in the
us. An openly capitalist party, the us Democratic Party advertises itself as the friend of workers and minorities, relying on its image as a `lesser evil' to secure electoral victories. Throughout
the Reagan-Bush years, however, Democraticcontrolled congresses signally failed to challenge the basic premises and policies of Reaganism. Even today, when faced with a cynically selfstyled
`Republican Revolution', disagreements between Republicans and Democrats concern only how fast and how deep to cut social programs. If Republicans demand $270 billion in Medicare
cuts, for example, Democrats respond by demanding $145 billion. The logic and necessity of slashing social programs are never questioned .24 Similarly, the Democrats collude with
Republicans on issues of racism and immigration. Clinton, as much as any Republican, has contributed to the false stereotyping of the recipients of public assistance as African-American
`welfare queens'. And, while many Democrats are on record as deploring Proposition 187 as a legal measure, nearly all Democrats concede to Republicans that an immigration `problem'

Derrida's proposal for


exists. Thus, the Clinton administration has recently beefed up the number of border cops and ordered harsher treatment of undocumented workers. No doubt

a New International represents in part a call to return to the values of `authentic' reform socialism. In the us, Derrida's proposal
represents a call to return to genuinely `progressive' values. The bankruptcy of European social democracy, as well as the vicissitudes of the American Democratic Party, does indeed create

Yet two points remain, each suggesting that attempts to revive reform socialism
political openings in which the socialist Left can and must seek to rebuild.

waste energies. First, the European Socialist parties which eventually found themselves authoring and imposing austerity measures on workers and minorities started out long ago
with sterling anti-capitalist principles. Good intentions are not enough in this regard, however, since politics and the economy are separated in capitalist society, and the latter wields greater
clout. Second, transformed by the discipline demanded by international capitalism, these nominally `socialist' parties occupy several of the very governments against which workers are

presently demonstrating in large numbers. Reform socialism has little to offer workers today. Callinicos has cogently summarized the current crisis in
Europe in this way: `a major recession which has highlighted longer term weaknesses of European capitalism; a withdrawal of popular support from the mainstream political parties; and the
resort to forms of political and social action which, consciously or unconsciously, tend to escape the limits of liberal bourgeois politics' (1994, 9). Soon after the publication of SM in France, for
example, the country was rocked by militant strikes and demonstrations lasting almost nine months between fall 1993 and summer 1994: Air France workers; 1,000,000 French citizens
marching against plans to privatize sectors of education; fishing workers; farmers; hundreds of thousands of French workers marching several times against unemployment and austerity
decrees; tens of thousands of students marching, building barricades and burning fires in protest against tuition hikes and the uncertain, potentially dismal future they face. Even as the
recession seemed to be coming to an end in Europe, the anger of French workers and students exploded again in fall 1995 - this time with sufficient force to sustain a three-week strike in the
public sector. Importantly, in the Air France strike, the anti-privatization campaign in education, the fight against changes in the universities and the recent public sector strike, real
concessions were wrested from the state. None of this renewed workers' activity, nor the fact that victories can be claimed, provides strong support for SMs assertions that barricades and
working-class militancy are out of fashion. In the us, too, polls show today that Americans are more skeptical about their government and its political parties than at any time in memory. A
wave of militant demonstrations followed the 1994 congressional elections that gave Gingrich and the `Contract With America' a majority in the Senate and House of Representatives. Massive
marches on Washington in support of gay rights, women's rights and civil rights have also taken place since the 1994 elections. The number of strikes, moreover, as well as the number of
production hours lost and workers participating in strikes, increased significantly in 1994. And no one who spent any time during the early 90s in Decatur, Illinois or Detroit, Michigan can have
any doubts about the willingness of us workers to fight back. Both areas - which include the struggle of locked-out Staley Workers in Decatur and striking newspaper workers in Detroit - have

In
been accurately referred to as `war zones'. The violence routinely used by state and local cops has been fiercely answered by the militancy and stamina of workers and their families.

every part of the globe political developments during recent years have been characterized by their
speed and volatility. It is important, however, to emphasize the still uneven and ambiguous character of the emerging challenge to the existing order: `It has begun
to liberate forces - in the shape of renewed workers' resistance to capitalist attacks - which could
unleash another upturn in the European [and us, my insertion] class struggle. But it has also given an opening to elements of barbarous reaction that had
been confined to the political margins since 1945' (Callinicos 1994, 37). Nothing guarantees the growth of the Left as a result of the major struggles that look likely to occur over the next few

The same political vacuum which creates opportunities for the Left is also creating, at least at this
years.

juncture, opportunities for the Right: `As yet there is no clear cut direction to events that would mark a decisive shift either to the right or to the left. But the
dynamic evolution of the crisis since 1989 gives no reason for thinking that the situation will remain so open' (Callinicos 1994, 36-7). In time, events will show whether their future
directionality owes more to the subjective agency of the Left in this period - or to the Right. That is why the question of socialist organization stands at the forefront of debate among
the Left today. Derrida's SM, with its call for a New International, should be discussed as a serious contribution to this debate. Nevertheless, SM's 'hauntological politics' must

Page 741 of 1481


be firmly rejected as incapable of answering the demands of our time. `The time is out of joint': Derrida repeatedly works this line from Hamlet in order to suggest that socialist

Greater
revolution is impossible because of the meta physical limitations of Marxism .25 Our present time may indeed be `out of joint', but it is not so because of bad metaphysics.

instabilities in an already crisis-prone system, deepening anger among the world's exploited and
oppressed, and sharper divisions both within and among national and international ruling classes - these
developments make our time one in which classical Marxism and its tradition of revolution from below
have much more to offer than hauntology does in the international struggle for a democratic socialist
society. 157-161

Page 742 of 1481


Page 743 of 1481
**Discourse Kritiks (General)**

Page 744 of 1481


Discourse Kritik Answers: 2AC
(1/3)
FIRST, NO LINK THEY CANT PROVE THAT OUR RHETORIC WAS ARTICULATED
WITH THE INTENT OF MARGINALIZATION

SECOND, LABEL POLITICS MISIDENTIFY THE CAUSE OF PREJUDICE IN


AGGRESSIVITY, SO THE NEW SYMBOL FILLS IN THE SAME ROLL
APPROPRIATIONS OF OLD LABELS RECONCEIVE THEIR MEANING

Zizek 97
[Slavoj, Moving away from the darkness, The Plague of Fantasies, New York: Verso, 1997, 111-2//uwyo-ajl]

In his formidable Fear in the Occident,7 Jean Delumeau draws attention to the unerring succession of atutudes in a medieval city infested by plague: first, people ignore it and
behave as if nothing terrible is really going on; then they withdraw into privacy, avoiding contact with each other; then they start to resort to religious fervour, staging
processions, confessing their sins, and so on; then they say to themselves 'What the hell, let's enjoy it while it lasts!', and indulge passionately in orgies of sex, eating, drinking
and dancing; finally, they return to life as usual, and again behave as if nothing terrible is going on. However, this second 'life as usual' does not occupy the same structural role
as the first: it is, as it were, located on the other side of the Moebius band, since it no longtt signals the desperate attempt to ignore the reality of plague, but, rather its exact

opposite: resigned acceptance of it . . . . Does not the same go for the gradual replacement of (sexually, racially...)
aggressive with more 'correct' expressions, like the chain nigger - Negro - black - African American or crippled - disabled -
bodily challenged? This replacement functions as a metaphorical substitution which potentially proliferate
and enhances the very (racist, etc.) effect it tries to banish, adding insult to injury. In analogy to Delumeau, one should
therefore claim that the only way actually to abolish the hatred-effect is, paradoxically, to create the

circumstances in which one can return to the first link in the chain and use it in a non-aggressive way -like
following the patterns of 'life as usual' the second time in the case of plague. That is to say: as long as the expression 'crippled' contains a

surplus, an indelible mark, of aggressivity this surplus will not only be more or less automatically
transferred on to any of its 'correct' metaphorical substitutes, it will even be enhanced by dint of this
substitution. The strategy of returning to the first link, of course, is risky; however, the moment it is fully
accepted by the group targeted by it, it definitely can work. When radical African-Americans call each other 'niggers', it is wrong to
dismiss this strategy as a mere ironic identification with the aggressor; rather, the point is that it functions as an autonomous act of dismissing the aggressive sting.

THIS HAS TWO IMPLICATIONS


IT MOOTS ALL OF THEIR OFFENSE BECAUSE THE MEANING OF A LABEL IS
RECONCEPTUALIZED AND REINSCRIBED
IT FLIPS THEIR TURN, PROLIFERATING THE OPPRESSION THAT IT TRIES TO SOLVE

THIRD, SPEAKING ERRORS ARE INEVITABLE AND GOOD BECAUSE THEY PROVIDE A
LOCUS FOR CONSTANT CRITICISM, SOMETHING THE NEG BY ITSELF PRECLUDES

Page 745 of 1481


Alcoff 92
[Linda, Prof. of Feminist Studies at the University of Syracuse, The Problem of Speaking for Others,
Cultural Critique, Winter 91-2, 22//uwyo]

But surely it is both morally and politically objectionable to structure ones actions around the desire to avoid
criticism, especially if this outweighs other questions of effectivity. In some cases perhaps the motivation is not so much to avoid criticism as to avoid errors, and the
person believes that the only way to avoid errors is to avoid all speaking for others. However, errors are unavoidable in the theoretical inquiry

as well as political struggle, and moreover they often make contributions. The desire to find an absolute
means to avoid making errors comes perhaps not from a desire to advance collective goals but a desire
for personal mastery, to establish a privileged discursive posotion wherein one cannot be undermined or challenged and thus is
master of the situation. From such a position ones own location and positionality would not require constant

interrogation and critial reflection; one would not hae to constantly engage in this emotionally
troublesome endeavor and would be immune from the interrogaton of others. Such a desire of rmastery
and immunity must be resisted.

Page 746 of 1481


Page 747 of 1481
Discourse Kritik Answers: 2AC
(2/3)
FOURTH, THE CRITICISM ASSUMES STABLE SPEECH ACTS, PREVENTING US FROM
TAKING BACK HURTFUL WORDS AND COLLAPSING INTO A JURIDICAL MODEL OF
STABLE SUBJECTIVITY THAT KILLS ACTIVISM

Judith Butler, Professor of Rhetoric and Comparative Literature, UC Berkeley, Performativity and Performance, Ed.

Parker and Sedgwick, 1995, p. 204

That words wound seems incontestably true, and that hateful, racist, misogynist, homophobic speech
should be vehemently countered seems incontrovertibly right. But does understanding from where
speech derives its power to wound alter our conception of what it might mean to counter that wounding
power? Do we accept the notion that injurious speech is attributable to a singular subject and act? If we
accept such a juridical constraint on thought - the grammatical requirements of accountability - as a point
of departure, what is lost from the political analysis of injury when the discourse of politics becomes fully
reduced to juridical requirements?? Indeed, when political discourse is collapsed into juridical discourse,
the meaning of political opposition runs the risk of being reduced to the act of prosecution. How is the
analysis of the discursive historicity of power unwittingly restricted when the subject is presumed as the
point of departure for such an analysis? A clearly theological construction, the postulation of the subject
as the causal origin of the performative act is understood to generate that which it names; indeed, this
divinely empowered subject is one for whom the name itself is generative.

FIFTH, LANGUAGE DOESNT HAVE A DETERMINATE EFFECT WORDS ARE EMPTY


ABSENT CONTEXT, MEANING OUR RHETORIC CAN BE READ IN A HETERODOX
MANNER TO CHALLENGE VIOLENCE

SIXTH, PERM, DO PLAN AND THE ALTERNATIVE. REPRESENTATIONAL VIOLENCE


DOESNT PRECLUDE THE NEED FOR CONCRETE ACTION

Richard Rorty, Professor of Humanities, University of Virginia, Truth, Politics, and Postmodernism, Spinoza Lectures,

1997, p. 51-2

This distinction between the theoretical and the practical point of view is often drawn by Derrida, another writer who enjoys demonstrating that something very important
meaning, for example, or justice, or friendship is both necessary and impossible. When asked about the implications of these paradoxical fact, Derrida usually replies that

the paradox doesn't matter when it comes to practice. More generally, a lot of the writers who are labeled `post-modernist; and
who talk a lot about impossibility, turn out to be good experimentalist social democrats when it comes to actual

political activity. I suspect, for example, that Gray, Zizek, Derrida and I, if we found ourselves citizens of the same country, would all be voting for the same
candidates, and supporting the same reforms. Post-modernist philosophers have gotten a bad name because of their paradox-mongering habits, and their constant use of
terms like `impossible; `self-contradictory' and `unrepresentable'. They have helped create a cult of inscrutability, one which defines itself by opposition to the Enlightenment
search for transparency - and more generally, to the `metaphysics of presence; the idea that intellectual progress aims at getting things clearly illuminated, sharply delimited,

Page 748 of 1481


I am all for getting rid of the metaphysics of presence, but I think that the rhetoric of impossibility and
wholly visible.

unrepresentability is counterproductive overdramatization. It is one thing to say that we need to get rid of the metaphor of things being
accurately represented, once and for all, as a result of being bathed in the light of reason. This metaphor has created a lot of headaches for philosophers, and we would be
better off without it. But that does not show that we are suddenly surrounded by unrepresentables; it just shows that `more accurate representation' was never a fruitful way to

Even if we agree that we shall never have what Derrida calls "a full presence beyond the reach
describe intellectual progress.

of play"; our sense of the possibilities open to humanity will not have changed. We have learned nothing about the limits of
human hope from metaphysics, or from the philosophy of history, or from psychoanalysis. All that we have learned from `post-modern' philosophy is that we may need a

We have been given no reason to abandon


different gloss on the notion of `progress' than the rationalistic gloss which the Enlightenment offered.

the belief that a lot of progress has been made by carrying out the Enlightenment's political program. Since
Darwin we have come to suspect that whether such progress is made will be largely a matter of luck. But we have been given no reason to stop hoping to get lucky.

Page 749 of 1481


Page 750 of 1481
Discourse Kritik Answers: 2AC
(3/3)
SEVENTH, CRITIQUES OF SPEECH PRODUCES A REACTIONARY POLITICS IN WHICH
CHANGE IS FOCUSED ON LANGUAGE DIRECTLY TRADING OFF WITH EFFORTS TO
REFORM THE SOCIOECONOMIC ROOT CAUSES OF INJUSTICE

Brown, Professor Political Science UC Berkeley, 2K1


(Wendy, Politics Out of History, pg. 35-37)

hate speech
Speech codes kill critique, Henry Louis Gates remarked in a 1993 essay on hate speech.14 Although Gates was referring to what happens when

regulations, and the debates about them, usurp the discursive space in which one might have offered a
substantive political response to bigoted epithets, his point also applies to prohibitions against questioning from within selected political practices or institu-
tions. But turning political questions into moralistic onesas speech codes of any sort donot only prohibits certain

questions and mandates certain genuflections, it also expresses a profound hostility toward political life
insofar as it seeks to preempt argument with a legislated and enforced truth. And the realization of that patently
undemocratic desire can only and always convert emancipatory aspirations into reactionary ones. Indeed, it insulates those aspirations from questioning at the very moment
that Weberian forces of rationalization and bureaucratization are quite likely to be domesticating them from another direction. Here we greet a persistent political paradox: the
moralistic defense of critical practices, or of any besieged identity, weakens what it strives to fortify precisely by sequestering those practices from the kind of critical inquiry out

identity-based
of which they were born. Thus Gates might have said, Speech codes, born of social critique, kill critique. And, we might add, contemporary

institutions, born of social critique, invariably become conservative as they are forced to essentialize the
identity and naturalize the boundaries of what they once grasped as a contingent effect of historically specific social
powers.

But moralistic reproaches to certain kinds of speech or argument kill critique not only by displacing it with arguments about abstract
by configuring political injustice and political righteousness as a problem of remarks, attitude,
rights versus identity-bound injuries, but also

and speech rather than as a matter of historical, political-economic, and cultural formations of power. Rather than offering
analytically substantive accounts of the forces of injustice or injury, they condemn the manifestation of these forces in particular remarks or events. There is, in the inclination to
ban (formally or informally) certain utterances and to mandate others, a politics of rhetoric and gesture that itself symptomizes despair over effecting change at more significant
levels. As vast quantities of left and liberal attention go to determining what socially marked individuals say, how they are represented, and how many of each kind appear in
, the sources that generate racism, poverty, violence against women, and other elements of
certain institutions or are appointed to various commissions

social injustice remain relatively unarticulated and unaddressed. We are lost as how to address those sources; but rather than examine this loss or
disorientation, rather than bear the humiliation of our impotence, we posture as if we were still fighting the big and good fight in our clamor over words and names. Dont mourn,
moralize.

EIGHTH, REJECTING DISCOURSE DOES NOTHING AND LEAVES ATTITUDES


UNCHANGED.

Kelly, 12/98 Peace Review

One might ask, in "listening" to violent language and to the people who use it, whether we are actually condoning such language. This is far from the case. To listen is not to

When I listen to a person who, for example, uses sexist language, I am not lending my approval to
pass judgment.

sexist language. Instead, what I am saying is that the person behind the language, and my desire to make a
connection with that person, are more important than the sexist language. If I refuse to listen to the
person who uses sexist language, then I might prevent one particular case where sexist language is used.
But I do nothing to overcome the person's sexist attitudes. She will continue to use sexist language long after I am out of sight. But if I

Page 751 of 1481


give her a voice, if I show her respect, if I try to take her seriously as a person, then In the future pershapes she will be more apt to take what I say about sexism seriously. If
she knows that sexist language bothers me, then perhaps she will be less likely to use it around me.

Page 752 of 1481


Page 753 of 1481
Newspeak Turn: 1AR
EXTEND THE 2AC ZIZEK 97 EVIDENCE. THEIR ARGUMENT IS PREMISED ON A
MISUNDERSTANDING OF COGNITION

WORDS DONT HAVE INTRSINSIC MEANINGS BUT CONVEY INFORMATION


THROUGH METAPHOR. WHEN YOU REPLACE ONE WORD WITH ANOTHER, THE
NEW WORD CONTAINS AN IMPLICIT REFERENCE TO THE CONNOTATIONS OF THE
OLD WORD. THE DISCRIMINATORY SURPLUS MEANING GETS TRANSFERRED TO
ALTERNATIVE LABELS AND PATRONIZING AGGRESSIVITY IS MAGNIFIED IN THE
GUISE OF AN ILLUSORY CHANGE

THIS HAS THREE IMPLICATIONS


IT DESTROYS ALL THEIR OFFENSE OUR LANGUAGE ISNT ANY WORSE THAN THE
RECONCEPTION

IT FLIPS THEIR IMPACT THEY ENHANCE THE LABELS SUBJECTIVE MEANING BY


REINSCRIBING ITS VIOLENCE

RETURNING TO THE FIRST LINK IN THE SIGNIFYING CHAIN IS THE ONLY WAY TO
DESTROY ITS AGGRESSIVE STING, EXPOSING THE SOURCE OF VIOLENT MEANING

CANT SEEK TO CHANGE CONCEPTS BY SUBSTITUTING WORDS THIS ONLY MASKS


THE EXISTENCE AND USE OF THOSE CONCEPTS AND GUARANTES THAT THEY WILL
REAPPEAR IN THE NEW WORDS. SHOULD USE THE ORIGINAL WORDS TO
HIGHLIGHT THEIR INDETERMINATE INADEQUACY

Bewes 97
[Timothy, doctorate in English Literature at the University of Sussex, Cynicism and Postmodernity, New
York City: Verso, 1997, 48//uwyo-ajl]

Page 754 of 1481


In this light, to begin to use again terms and concepts which had seemed to be theoretically proscribed
(the author, the subject, reality, sexual and cultural identity, the universal) is not necessarily to betray a
reactionary or a nostalgic desire for 'presence'; on the contrary, what the critical insights of post-
structuralism (more specifically, deconstruction) reveal is not only the possibility but the imperative that
such terms continue to be used. There are no others - and if there were, they would by definition not only
be liable to but would comprise exactly the same catachrestic abuses

Page 755 of 1481


Page 756 of 1481
#2 Newspeak Turn: Ext (1/5)
SELF-CENSORSHIP MAINTAINS THE VIOLENT IDEOLOGY OF THE STATUS QUO BY
CREATING A DISTANCE BETWEEN POLITICAL APPEARANCES AND THE DISAVOWED
OBSCENE HATRED THAT SUPPORTS THEM

Zizek '97
[Slavoj, Jazz, The Plague of Fantasies, NYC: Verso, 1997, 25//uwyo-ajl]

The key point not to be missed here is how this fragile coexistence of extreme and violent homophobia
with a thwarted - that is, publicly unacknowledged, `underground' - homosexual libidinal economy bears
witness to the fact that the discourse of the military community can operate only by censoring its own
libidinal foundation. At a slightly different level, the same goes for the practice of hazing (the ceremonial
beating up and humiliating of the US Marines by their elder peers: sticking theii, medals directly on to
their breast skin, etc.): when the public disclosure of these practices (somebody secretly shot them on
video and made the tape public) caused such an outrage, what disturbed the public was not the practice
of hazing itself (everybody was aware that things like this were going on) but the fact of rendering it
public. Outside the confines of military life, do we not encounter a strictly analogous self-censoring
mechanism in contemporary conservative populism, with its sexist and racist bias? Recall the election
campaigns of Jesse Helms, in which the racist and sexist message is not publicly acknowledged (on the
public level, it is sometimes even violently disavowed), but is instead inarticulated `between the lines', in a
series of double-entendres and coded allusions. The point is that this kind of self-censorship (not openly
admitting one's own fundamental message) is necessary if, in the present ideological~conditions, Helms's
discourse is to remain operative: if it were to articulate its racist bias directly, in a public way, this would
make it unacceptable in the eyes of the predominant political discursive regime; if it were effectively to
abandon the self-censored coded racist message, it would endanger the support of its targeted electoral
body. Conservative populist political discourse is therefore an excellent example of a power discourse
whose efficiency depends on the mechanism of self-censorship: it relies on a mechanism which is
operative only in so far as it remains censored. Against the image, ever-present in cultural criticism, of a
radical subversive discourse or practice `censored' by Power, one is even tempted to claim that today,
more than ever, the mechanism of censorship intervenes predominantly to enhance the efficiency of the
power discourse itself.

POLITICALLY CORRECT SIGNIFER REPLACEMENT ALLOWS NEW, AGGRESSIVE


FORMS OF DISCRIMINATORY HUMILIATION IN THE GUISE OF DISTANCE

Zizek '99
[Slavoj, Senior Researcher at Institute for Social Studies, Ljubliana and Badass, The Ticklish Subject: the
absent centre of political ontology, New York: Verso, 1999, 253-4//uwyo-ajl]

Take politically correct probing into hate speech and sexual harassment: the trap into which this effor falls
is not only that it makes us aware of (and thus generates) new forms and layers of humiliation and
harassment (we learn that 'fat', 'stupid', 'short-sighted' . . . are to be replaced by 'weight-challenged',
etc.); the catch is, rather, that this censoring activity itself, by a kind of devilish dialectical reversal, starts
to participate in what it purports to censor and fight is it not immediately evident how, in designating
somebody as 'mentally challenged' instead of 'stupid', an ironic distance can always creep in and give rise

Page 757 of 1481


to an excess of humiliating aggressivity one adds insult to injury, as it were, by the supplementary polite
patronizing dimension (it is well known that aggressivity coated in politeness can be much more painful
than directly abusive words, since violence is heightened by the additional contrast between the
aggressive content and the polite surface form...). In short, what Foucault's account of the discourses of
discipline and regulate sexuality leaves out of consideration is the process by means of which the power
mechanism itself becomes eroticized, that is, contaminated by what it endeavours to 'repress'. It is not
enough to claim that the ascetic Christian subject who, in order to fight temptation, enumerates and
categorizes the various forms of temptation, actually proliferates the object he tries to combat; the point
is, rather, to conceive of how the ascetic who flagellates in order to resist temptation finds sexual
pleasure in this very act of inflicting wounds on himself.

Page 758 of 1481


Page 759 of 1481
#2 Newspeak Turn: Ext (2/5)
PAPERING OVER A TOUCHY SUBJECT IS THE TRIUMPH OF FANTASY

Stavrakakis '99
[Yannis, Teaching Fellow at the University of Essex, Lacan and the Political, New York City: Routledge,
1999, 88-9//uwyo-ajl]

In this regard, Lacan is extremely clear. Through this fantasy modern society returns to a state of myth:

How is one to return, if not on the basis of a peculiar (special) dis-course, to a prediscursive reality? That is
the dream - the dream be-hind every conception (idea) of knowledge. But it is also what must be
considered mythical. There's no such thing as a prediscursive re-ality. Every reality is founded and defined
by a discourse. (XX: 32)

In opposition to such a 'regressive' attitude, Lacanian theory promotes a return to the founding moment
of modernity. Recognising the irreducible character of impossibility, the constitutivity of the real as
expressed primarily in the failure of our discursive world and its continuous rearticula-

tion through acts of identification, far from being a postmodern move, reveals the truly modern character
of the Lacanian project; instead of a postmodern mysticism it leads to a reorientation of science and
knowledge. Recognising the constitutivity of the real does not entail that we stop

symbolising; it means that we start trying to incorporate this recognition within the symbolic itself, in fact
it means that since the symbolic entails lack as such, we abstain from covering it over with fantasmatic
constructs - or, if one accepts that we are always trapped within the field of fantasy, that we never stop
traversing it. The guiding principle in this kind of approach is to move beyond fantasy towards a self-
critical symbolic gesture recognising the contingent and transient character of every symbolic construct.
This is a scientific discourse different from the reified science of standard modernity.

THE REPLACEMENT OF OFFENSIVE LANGUAGE WITH NEW SIGNIFIERS IS EVEN


WORSE, UNDERESTIMATING THE GAP BETWEEN LANGUAGE AND THE CONSCIOUS
REGULATION OF ITS EFFECTS AND ENSURING THAT ANY RESOLUTION IS
ARBITRARY

Zizek 99
[Slavoj, Steelers Linebacker, The Ticklish Subject: The Absent Center of Political Ontology, New York: Verso,
1999, 332-3//uwyo-ajl]

In all these domains, the difJerend seems to be irreducible - that is to say, sooner or later we find ourselves in a grey zone whose mit cannot be dispelled,.by the application of
some single universal rule. Here we encounter a kind of counterpoint' to the 'uncertainty principe' of quan-tum physics; there is, for example, a structural difficulty in determining

Confronted with such a dubious statement, a


whether some comment was actually a case of sexual harassment or one of racist hate speech.

'politically correct' radical a priori tends to believe the complaining victim

(if the victim experienced it as harassment, then harassment it was. . .), while a diehard orthodox liberal tends to believe the
accused (if he sincerely did not mean it as harassment, then he should be acquitted. . .). The point, of course, is that this undecidability is structural and

Page 760 of 1481


unavoid-able, since it is the big Other (the symbolic network in which victim and offender are both
embedded) which ultimately 'decides' on meaning, and the order of the big Other is, by definition, open; nobody can dominate and regulate its effects.

That is the problem with replacing aggressive with 'politically correct' expressions: whan one replaces
'short-sighted' with 'visually challenged', tone can never be sure that this replacement itself will not
generate new effects of patronizing and/or ironic offensiveness, all the more humiliating inasmuch as it is
masked as benevolence. The mistake of this 'politically correct' strategy is that it underestimates the
resistance of the Ianguage we actually speak to the conscious regulation of its effects, epecially effects
that involve Fower relations. So to resolve the deadlock, one convenes a committee to formulate, in an
ultimately arbitrary way, the precise rules of conduct. It is the same with medicine and 'biogenetics (at what point does an acceptable and
even desirable genetic experiment or intervention turn into unacceptable manipulation?), in the application of universal hum all rights (at what point does the protection 0f the
victim's rights turn into an imposition of Western values?), in sexual mores (what is the proper, non-patriarchal procedure of seduc-tion?), not to mention the obvious case of
cyberspace (what is the status of sexual harassment in a virtual community? How does one distinguish between 'mere words' and 'deeds'?). The work of these committees is
caught in a symptomal vicious cycle: on the one hand, they try to legitimate their decisions by reference to the most advanced scientific ,knowledge (which, in the case of
abortion, tells us that a foetus does not yet possess self-awareness and experience pain; which, in the case of a mortally ill person, defines the threshold beyond which
euthanasia is the only meaningful solution); on the other hand, they have to evoke some non-scientific ethical criterion in order to direct and posit a limitation to inherent
scientific drive.

Page 761 of 1481


Page 762 of 1481
#2 Newspeak Turn: Ext (3/5)
CLEANSING IDEOLOGY FROM LANGUAGE IS A CRY FOR HELP A LONGING FOR
FANTASMIC HARMONY EMBODIED IN A THROATY GROAN

Stavrakakis '99
[Yannis, Teaching Fellow at the University of Essex, Lacan and the Political, New York City: Routledge,
1999, 114-5//uwyo-ajl]

We can also approach this constitutive play between possibility and iInpossibility through the example of
communication. What Lacan argues, and here his difference from Habermas is most forcefully
demonstrated, is that "it is exactly because total communication is impossible, because it is exposed as an
impossible fantasy, that communication itself becomes possible. Lacan

starts from the assumption that communication is always a failure: moreover, that it has to be a failure,
and thats the reason we keep on talking. If we understood eachother, we would all remain silent. Luckily
enough, we dont understand each other, so we keep on talking (Verhaeghe, 1995: 81)

The utopian fantasy of a perfect universal language, a language common to all humanity, was designed to
remedy this lack in communication insofar as it is caused by the different idioms and languages in use
(Eco, 1995: 19). The perfect language was conceived as the final solution to this linguistic cbfusion, the
confusio linguarum, which inscribed an irreducible lack at the heart of our symbolic universe, showing its
inability to represent the real. It entailed a fantasmatic return to a pre-confusion state in which a perfect
language existed between Adam and God. This was a language that mirrored reality, an isomorphic
language which had direct and unmediated access to the essence of things: 'In its original form.. .language
was an absolutely certain and transparent sign for things, because it resembled them. The names of
things were lodged in the things they designatedThis transparency was destroyed at Babel as a
punishment for men. (Foucault, 1989: 36). Human imagination never stopped longing for that
lost/impossible state when language, instead of the agency of castration, was the field of a perfect
harmony; hence all the attempts to construct a perfect language, to realise fantasy: Umberto Eco in his
Search for the Perfect Language recounts the history of all these attempts within European culture, from
St. Augustine's fantasy, in which the distance between object and symbol is annulled,17 up to Dante, a
priori philosophical languages and Esperanto. This history is, of course, a genealogy of falures, the history
of the insistence on the realisation of an impossible dream, a dream, however,

that was designed as a perfect solution to the inherent division of the social. As Eco points out, linguistic
confusion is conceived as standing at the root of religious and political division, even of difficulties in
economic exchange (Eco, 1995: 42-3). In that sense-;-the achievement of perfect communication is
articulated as the perfect solution to all these problems. This is clearly a utopian problematic. Alas, as
Antonio Gramsci points out in his text 'UniversaL Language and Esperanto', no advent of a universal
language can be planned in advance:

the present attempts at such a language belong only in the realm of Utopia: they. are the product of the
same mentality that wanted Falangists and happy colonies. In history and social life nothing is fixed, rigid
and final. There never will be... this flow of molten vol-canic matter, burns and annihilates the Utopias
built on arbitrary acts and vain delusions such as those of a universal language and of Esperanto.

Page 763 of 1481


Page 764 of 1481
#2 Newspeak Turn: Ext (4/5)
THE IMPACT PRESUMES STATIC REFERENCE, WHICH IS EITHER A NAVE MISTAKE
OR A VISIOUS LIE

Seshadri-crooks 2000
[Kalpana, Asst prof engl boston college, Desiring Whiteness, NYC: Routledge, 141-2//uwyo-ajl]

Racial identity, too, I would like to suggest - Le., words like black and white, when used as nouns - works
like names. 10 That is, they are rigid designators - they are signifiers that have no signified. They establish
a reference, but deliver no connotations or meaning whatsoever. We can, of course, reasonably argue
that race does not exist insofar as the identity of a person as "black" or "white" is contingent upon a
cluster of concepts that are them-selves too protean to be able to uphold anything like a necessary truth.
We can cite historical evidence to show that groups that were once considered white are no longer
classified as such for this or that reason, etc. But as my discussion in Chapter 1 specified, arguments
leveled at race theory are highly ineffectual and possess insufficient explanatory power. Thus rather than
lapse into the historicist argument, it may be more productive to view racial color designators as
operating not unlike proper names. The proper name is neither wholly one's own (Le., we are all named
by others) nor is it mean-ingful. One inhabits the name as the reference of oneself, and as Kripke asserts,
it bears no relation to a set of properties that establish either its meaning or its reference: Nixon is Nixon,
or as he says, quoting Bishop Butler, "everything is what it is and not another thing" (Kripke 1982: 94). Is
this not true for "black" and "white"? If someone is designated as one or the other, there is a necessary
truth to that designation, but does it mean? What would be the cluster of concepts that could establish
such an identity? Even in identity statements such as "blacks are people of African descent" or "whites are
people of European descent," though the predicates supposedly define and give the meaning of black and
white, establishing the necessity of these concepts in every counter-factual situation will not be possible if
only because national designations, and the notion of descent, are historically volatile and scientifically
invalid respectively. No set of qualitative descrip-tions can establish black or white identity across all
possible worlds, but we cannot therefore say that black and white do not exist, which is the error that a
number of critical race theorists fall into. As Kripke says,

it is not how the speaker thinks he got the reference, but the actual chain of communication, which is
relevant. ... Obviously the name is passed on from link to link. But of course not every sort of causal chain
reaching from me to a certain man will do for me to make a reference. There may be a causal chain from
our use of the term "Santa Claus" to a certain historical saint, but still the children, when they use this, by
this time probably do not refer to that saint. ... It seems to me wrong to think we give ourselves some
properties which somehow qualitatively uniquely pick out an object and determine our reference in that
manner.(Kripke 1982: 93-4)

Page 765 of 1481


Page 766 of 1481
#2 Newspeak Turn: Ext (5/5)
FOCUSING ON LANGUAGE UNDERESTIMATES PREJUDICE SYMBOLIC CONTROL
ONLY BEGS THE QUESTION OF UNCONSCIOUS MOTIVATION

Lane 98
[Chrisopher, English Professor at Emory, Savage Ecstacy: Colonialism and the Death Drive, The
Psychoanalysis of Race, 1998//uwyo-ajl]

The repercussions of Fanons Hegelianism are nonetheless acute. While Fanon complains of being sealed
into thingness (218) by white racism, he also aims toward mastery of language because it affords
remarkable power (18). On one level, we can appreciate why linguistic mastery is threatening to white
racism. However, Fanon also avows, at the beginning of Black Skin, White Masks, that to speak is to exist
absolutely for the other (17). Ironically, Fanon is at his most Lacanian here, for he clarifies that we cannot
limit the tyranny of prejudce to intentional racism; nor can we simply defeat it by mastery of language:
The signifier raises a further, generic dimension of alienation that implicates men and women of all races
while exceeding their capacity for symbolic control. This is surely why Fanon claims The Other will
become the mainstail of [the white mans ] preoccupations and his desires(170), and why he states of
the black man, The goal of his behavior will be The Other (in the guise of the white man) (154).

DIALOGUE OVER LABELS WITH A FIXED GOAL OF EMPOWERMENT AS A KNOWN


AND STABLE END GUARANTEES HEGEMONIC REGIMES OF TRUTH MORE
DISCURSIVELY VIOLENT THAN THE ORIGINAL

Bewes 97
[Timothy, doctorate in English Literature at the University of Sussex, Cynicism and Postmodernity, New
York City: Verso, 1997, 87-9//uwyo-ajl]

The disavowal of political representation, combined with the seduc-tiveness of a 'political' ideology of
absolute sincerity, is one explanation for the appeal of the Communitarian agenda. Another is (ii) the
fetishization of specificity. Etzioni favours localized as opposed to centralized legislation, and informal as opposed to formal structures of law enforcement.
Thus divorce, the principal 'threat' to family life (a tautologous diagnosis that resembles John Major's cosmetic proposals to solve the problem of 'yobbery'), should be not
banned or condemned, but discouraged.98 Similarly 'hate' (in the form of racism or sexism, for example) is best coun-tered not by legislation but by 'dialogue' and 'education'.

of the 'discussions' initiated following incidents of campus racism is voyeuristic, almost


Etzioni's account

pornographic in its attention to episodic detail. One has a sense of salivation, more bloodthirsty than at
any seventeenth-century public evisceration, and of tri-umphalism over the ritual punishment of the
guilty parties: humiliation by workshop.
The next evening the [offended] women organized a meeting with some students in the same dormitory and discussed the matter. They were joined by a supportive professor.
Several white people made it clear that they were deeply embarrassed. The session was followed up by more forums, a press conference, and a seminar at the law school.
These dis-cussions, in turn, triggered a campus-wide debate on the issues at hand. The local newspapers also took note. The article in the campus news-paper included an
apology from the person who had put up the form in the first place. The four women said that toward the end they no longer felt like victims but rather 'empowered'.99

The Communitarian citizen, then, fills the gaps left by the skeletal legal framework, makes 'complete' a legislative structure which must refrain from explicit adjudication on its
own account. By maintaining instead a 'hands-off' policy of implicit governance, Communitarianism proposes an ethos of unwritten rules of behav-iour, under the sign of

. A society of unwritten -that is, not legally binding - laws is


'empowerment', which are no less powerful and effective because they are implicit

assumed to be freer; in fact, as we can see, the tacit legislature of Communitarianism effects a far more

Page 767 of 1481


thorough and indeed repressive policing of the individual. The figure who arouses the most sympathy in Etzioni's story is the
quitevobviously cynical student who, as his college campus succumbed o the viral effect of a positivistic 'anti-racist' consensus, took to emblazoning his notebooks and the
walls of his room with swastikas. 'The work of education is never done,' says Etzioni, a little sinisterly. Clearly not education but rather the problematic of Dostoevsky's man
underground is the issue here: the necessity to prove that one is 'a man and not a prig in a .barrelorgan:. The dawning Influence of Commumtanamsm in British political life IS a
symptom of the 'epidemic of consensus' identified by Baudrillard as a millenarian phenomenon, and of the fear of violence - political, semiotic, historical-identified by Hegel as a
crisis of healthy 'philosophical scepticism.

Page 768 of 1481


Page 769 of 1481
#4 Censorship Bad Turns: 1AR
THE REFUSAL TO REAPPROPRIATE EXCLUSIONARY LANGUAGE IS POLITICALLY
PARALYZING DOGMATISM

Butler, Chair of the Rhetoric Department at U.C.-Berkley, 97 (Judith, Excitable Speech, P. 162)

dogmatism appears as well in the effort to circumscribe speech that injures, excites, threatens, and offends.
Such
Whether it is the censorship of particular kinds of representation or the circumscription of the domain of
public discourse itself, the effort to tighten the reins on speech undercuts those political impulses to exploit
speech itself for its insurrectionary effects. The intellectual opposition to questions that destabilize a sense of reality seems a mundane
academic case in point. To question a term, a term like "the subject" or "universality," is to ask how it plays, what investments it bears, what aims it achieves, what
The changeable life of that term does not preclude the possibility of its use. If a term
alterations it undergoes.
becomes questionable, does that mean it cannot be used any longer, and that we can only use terms that
we already know how to master? Why is it that posing a question about a term is considered the same as e na ct ing a prohibit ion a ga ins t
use? Why is it that we sometimes do feel that if a term is dislodged of its prior and known contexts, that we will not be able to live, to survive, to use
language, to speak for ourselves? What kind of guarantee does this effort to refer the speech act back to its originating context exercise, and what sort of
terror does it forestall? Is it that in the ordinary mode, terms arc assumed, terms like "the subject" and "universality," and the sense in which they "must"
be assumed is a moral one, taking the form of an imperative, and like some moral interdictions, a defense against what terrifies us most? Are we not
paralyzed by a fear of the unknown future of words that keeps us from interrogating the terms that we
need to live, and of taking the risk of living the terms that we keep in question?

DISALLOWING MODIFICATION IGNORES THE POTENTIAL FOR RE-APPROPRIATION


BY RE-ITERATING THE USEFUL CONCEPTS OF THOSE WE CRITICIZE FOR THEIR
EXCLUSIONS

Judith Butler, Professor of Rhetoric and Comparative Literature at UC Berkeley, The Psychic Life Of Power: Theories
In Subjection, 1997, p. 93.

For Foucault, the subject who is produced through subjection is not produced at an instant in its totality.
Instead, it is in the process of being produced, it is repeatedly produced (which is not the same as being
produced anew and again). It is precisely the possibility of a repetition which does not consolidate that
dissociated unity, the subject, but which proliferates effects which undermine the force of normalization.
The term which not only names, but forms and frames the subjectlet us use Foucaults example of
homosexuality--mobilizes a reverse discourse against the very regime of normalization by which it is
spawned.

Page 770 of 1481


Page 771 of 1481
#4 Censorship Bad Turns: Ext
(1/4)
GENDER STUDIES PROVE REDEPLOYMENT GOOD

Jerry Muller, Coming Out Ahead, First Things, August 1993(accessed online)

Butler holds to the Foucaultian axioms that power is ubiquitous and that it is only power which
establishes truth. She concludes that "power can be neither withdrawn nor refused, but only redeployed.
Indeed, in my view, the normative focus for gay and lesbian practice ought to be on the subversive and
parodic redeployment of power rather than on the impossible fantasy of its full-scale transcendence." The
"redeployment" of power, in this case, means that those with the power of interpretation use their power
to subvert the belief that there is anything "natural" about sexual identity, desire, and conduct, and to
break the normative link between biological endowment, behavior, and sexual object. "The loss of gender
norms would have the effect of proliferating gender configurations, destabilizing substantive identity, and
depriving the naturalizing narratives of compulsory heterosexuality of their central protagonists: 'man'
and 'woman.'" Within this agenda, transitive and intermediate forms of sexual identity acquire a special
significance. Drag, cross-dressing, and butch/femme lesbian identities, Butler writes, all serve to "parody"
the notion of a nature-based gender, and reveal that cultural gender does not flow naturally and
inevitably from anatomical sex, but rather is a socially learned role (performance) with no essential link to
anatomy. "Parodic proliferation," she declares, "deprives hegemonic culture and its critics of the claim to
naturalized or essentialist gender identities."

SUPPRESSING LANGUAGE BECAUSE IT IS OFFENSIVE PRESERVES ITS INJURIOUS


MEANING ONLY BY USING THE LANGUAGE CAN SPACE BE OPENED TO
RECONSTRUCT A MORE HUMANE MEANING

Kurtz and Oscarson, Members of National Council of Teachers of English Conference on College Composition
and Communication, 2K3 (Anna and Christopher, BookTalk: Revising the Discourse of Hate, ProQuest)

However, Butler also argues that the daily, repeated use of words opens a space for another, more
empowering kind of performance. This alternative performance, Butler insists, can be "the occasion for something we might
still call agency, the repetition of an original subordination for another purpose, one whose future is partially
open" (p. 38). To think of words as having an "open" future is to recognize that their authority lies less in
their historical than in their present uses; it is to acknowledge that people can revise the meaning of
words even as we repeat them; it is to embrace the notion that the instability of words opens the
possibility that we can use them to (re)construct a more humane future for ourselves and others. Because
words can be revised, Butler contends that it would be counterproductive simply to stop using terms that
we would deem injurious or oppressive. For when we choose not to use offensive words under any
circumstance, we preserve their existing meanings as well as their power to injure. If as teachers, for instance, we
were simply to forbid the use of speech that is hurtful to LGBT students we would be effectively denying the fact that such language still exists. To ignore
words in this way, Butler insists, won't make them go away. Butler thus suggests that we actually use these

Page 772 of 1481


words in thoughtful conversation in which we work through the injuries they cause (p. 1.02). Indeed, Butler insists
that if we are to reclaim the power that oppressive speech robs from us, we must use, confront, and
interrogate terms like "queer." We must ask how such terms affect both the speaker and the subject, what the purpose of their use is, and how their
meaning can be altered to empower those whom they name. Thus, as Butler helps us see, language is violence, but only if we allow it to
be. She encourages us to believe that words can take on new meanings-ones which forbid stasis,
challenge our habits, and open the possibility that teachers and students might be able to create spaces for learning in which
everyone feels safe.

Page 773 of 1481


Page 774 of 1481
#4 Censorship Bad Turns: Ext
(2/4)
REGULATING SPEECH MAKES RE-APPROPRIATING SPEECH IMPOSSIBLE ONLY BY
USING LANGUAGE CAN RESIGNIFICATION OCCUR

Fleche, Assist Prof of English @ Boston College, 99 (Anne, Excitable Speech: A Politics of the Performative,
Theatre Journal, Vol. 51, No. 3, October, Project Muse)

Excitable Speech might seem surprising to readers of Butler's previous work. Having argued , in Gender Trouble and Bodies That Matter, that bodies
Butler now argues that to speak is not quite the same
and subjects are constructed in the cultural forms that articulate them,
as to act. For Butler the conservative conflation of speech and act is neither performative nor, in her sense of the word, constructionist,
because it argues for a notion of free speech that presumes an unconstrained, sovereign subject. Butler considers this probl em and its possible
remedies in her analyses of Supreme Court decisions, anti-pornography arguments, and the policy against homosexuals in the military. In every
instance, she complicates the relation of speech to act, by introducing fantasy, linguistic instability, and temporality, arg uing against censorship
and the legal redress of hate speech and for its critical re-articulation. The key move in the analysis comes in the opening chapter, "On Linguistic
Vulnerability," where Butler deconstructs the relation of the body to speech. Working from texts by Toni Morrison and Shoshana Felman, Butler
argues that language and the body are neither strictly separable nor simply the same, but speak together, as it were, to prod uce the effect known
as the social speaking subject. Thus verbal threats, for example, are also, in som e way, bodily ones: "[T]he body is the blindspot of speech, that
which acts in excess of what is said, but which also acts in and through what is said" (11). Once the body/speech relation is deconstructed,
censorship, with its assumptions of causality between word and act, becomes even more troubling. Butler finds promise in this problem, arguing
that, because speech threatens, delivers and delays, it opens up a future of options. It is the gap between speech
and conduct she wants to emphasize. In theatrical terms, this is the gap in which Brecht sees the actor intervening --in his view performance is not
referential, but a social gest, playful and capable of change. IndeedButler's notion of performativity, sometimes understood
as the ability of language to produce what it names, is nearly the opposite of referentiality: it is an effect of representation that
cannot wholly be controlled. Performativity is what gives a future to the name in name-calling. In the process of
coming out, for example, homosexuality is named but never fully defined: while coming out "renders homosexuality
discursive," Butler emphasizes, "it does not render discourse referential . . . . [I]t is important not to close the gap between the
performative and the referential" (125). To close this gap is to leave no remedy for hate speech short of
state intervention, and the state is certainly not neutral. Butler points out that the Supreme Court has tended to protect
racist behavior as speech, while restricting pornographic literature. In ce nsoring pornography, the court appears to agree with feminist arguments
that pornographic representation is a discriminatory act. Similarly, the policy against gays in the military assumes that to
identify oneself as a homosexual is to act upon another person in a homosexual way, to make such an
identification "contagious," as Butler puts it. And yet, in a case of cross burning, the Supreme Court found that when he
burned a cross in front of a black family's house, a white teenager was expressing a "viewpoint" in the
"free marketplace of ideas" (53). These decisions imply that language should not have power to do what it
says, but that the state, in regulating speech, should. When speech becomes injurious act in some cases
and remains free speech in others, it is clear that a theory of speech, and not a legal remedy, is what is
most urgently needed. Consequently, Butler opposes linguistic determinism and the "anti-intellectualism" of the academy's efforts to
return to "direct" speech. Language is politically and socially useful, she argues , precisely to the extent that it is
"excitable"--by which she means "out of control", in play, "performative:" "Indeed, the act-like character of certain offensive
utterances may be precisely what keeps them from saying what they mean to say or doing what it is they
say" (72). Language is neither fully social nor fully semantic but socially performed and cited, interpellating a body and a social self while
excluding "impossible" bodies, selves and speech. In a brief reference to the argument elaborated in her book The Psychic Life of Power (also
1997), Butler counters the legal arguments for restricting hate speech with Foucault's "less legal" notion of
power as an effect, produced through multiple forces. Foucault's idea of power eliminates the sovereign, accountable subject
(or state) that speech regulation seeks to restore. It is power, Butler argues, that makes speech into censorship, by
legislating what counts. Thus, not all social forms are simply censored, tainted or unusable: the terms of
legibility produce the possibility of breaking silence, of thwarting exclusion, and of acting "with authority
without being authorized" (157), as in the civil disobedience of Rosa Parks. [End Page 348] Rather than offer prescriptions,
Butler uses her own writing to illustrate the power of resignification. In her rhetorical readings of Supreme Court decisions, for
example, the justices' words become surprisingly rich and suggestive. She is herself an expert resignifier. Resignifying word s, Butler acknowledges,
does not take away their hurt. She does think that sometimes people should be prosecuted for injurious speech and that univer sities might need to
regulate speech--but should do so only when they have "a story to tell" about its harm ful effects. She is not opposed to all speech regulation. But

Page 775 of 1481


Excitable Speech asks whether regulation makes it easier or harder to reappropriate speech, and why we fear
to take the exciting risk of language, where a threat might also be a promise.

Page 776 of 1481


Page 777 of 1481
#4 Censorship Bad Turns: Ext
(3/4)
REGULATING LANGUAGE DESTROYS THE HOPE OF TRUE EMANCIPATION WE
MUST BE ABLE TO RESIGNIFY DEROGATORY TERMS TO DEFUSE THEIR INJURIOUS
ABILITIES

Disch, Associate Professor of Political Theory at the University of Minnesota, 99 (Lisa, Judith Butler and the Politics
of the Performative, JSTOR)

Judith Butler's longstanding political concern has been to discern what in the structure of subjectivity makes it so difficul t to shift from
moralized to politicized mobilization and so easy to fall into identity politics and the politics of scapegoating. In The Psychic Life of Power, she
analyzes the psychic and social process of subject formation to disclose the investments that stand in the way of "the development of form s of
differentiation [that could] lead to fundamentally more capacious, generous, and 'unthreatened' bearings of the self in the midst of community"
In Excitable Speech, she rebuts the work of the theorists who introduced hate speech into the
(CR, 140).
legal arsenal. Whereas they share her premise that we are linguistic beings, Butler charges that in
advocating speech codes, censorship, and other regulatory approaches to linguistic injury, hate speech
theorists destroy "something fundamental about language and, more specifically, about the subject's
constitution in language" (ES, 27). Butler proposes to counter injurious speech with "subversive
resignification": the insubordinate use of a derogatory term or authoritative convention to defuse its
power to injure and to expose "prevailing forms of authority and the exclusions by which they proceed"
(ES, 157-58). These two books are especially important for answering the charge that poststructuralist critics of humanism demolish politic al
Butler's theory of "insurrectionary" speech acts opens up the possibility
agency when they take issue with autonomy.
of an agency that does not fantasize "the restoration of a sovereign autonomy in speech" but, rather,
plays our dependency on sanctioned forms of address into an everyday resistance (ES, 145,15). Insur-
rectionary speech does considerable theoretical work to break the impasse between autonomy and
determinism that stalls many discussions of political agency in "postliberatory times" (The Psychic Life of Power [PL],
18). And although this contribution is significant, it may strike some readers as incom plete. Butler is more attentive to examples where
dominant institutions (such as the courts and the military) have subversively resignified potentially insur rectionary initiatives (such as hate
speech) than she is to instances where performative agency has transformed the status quo. Even if Butler's own exam ples do not establish it as
such, I will argue that the "politics of the performative" is a politics of insurrection. First, I offer a brief summary of
Butler's concepts "heterosexual matrix," "heterosexual melancholy," and "gender performativity," as these are indispensable t o appreciating her
recent writings.

CENSORSHIP WILL BE COOPTED BY CONSERVATIVE ELEMENTS TO DESTROY


MINORITY RIGHTS INSTEAD LANGUAGE SHOULD BE USED TO SUBVERT THE
CONVENTIONAL MEANINGS OF THE WORDS

Nye, Prof of Philosophy @ U of Wisconsin Whitewater, 99 (Andrea, Excitable Speech: A Politics of the
Performative; In Pursuit of Privacy: Law, Ethics, and the Rise of Technology, JSTOR)

Excitable Speech and In Pursuit of Privacy will appeal to very different audiences. Judith Buder is a theorist's theorist whose mastery of the complex intellectual gyrations of poststructuralism
and postmodernism will be daunting to all but an initiated few, while Judith Wagner DeCew is a legal scholar who uses traditional reviews of case law and standard techniques of rational
argument to make her point. Nevertheless, they ask the same important questionIn promoting the rights of women, to what extent should feminists call for state action? and they give
the same negative answer: Not very far at all. Butler's concern is with recent controversies surrounding regulation of "hate language," specifically decisions that broadly interpret the "fighting
words55 doctrine, which makes certain uses of speech unprotected under the First Amendment's guarantee of freedom of speech. She argues against Catharine MacKinnon5s claim that
pornography is subject to government intervention because it is action that effectively silences women. DeCew, on the other hand, defends a broad view of the "right to privacy55 that protects
not only private information but also individual decision making from state interference. Their methods in making these points could not be more different. Butler works meticulously through
a dense thicket of the analytic speech act theory of John Austin, the structuralist and poststructuralist theories of Jacques Derrida and Pierre Bourdieu, psychoanalytic constructions in the

Once the state has the


style of Sigmund Freud and Jacques Lacan, and German critical theory to conclude that state regulation of hate language should be resisted.

power to legislate what can be said and not said, she argues, that power will be coopted by conservative
elements to defeat liberal causes and minority rights. State power will also curtail the freedom of speech

Page 778 of 1481


of private individuals that is the very basis for effective antidotes to derogatory name calling. DeCew, however,
painstakingly reviews the legal and philosophical history of privacy rights as well as current debates about its scope and status before she takes on the question of whether feminists have any
interest in preserving a private sphere. For DeCew, too, a major target is MacKinnon, specifically her argument that leaving alone the privacy of home and family means leaving men alone to
abuse and dominate women. DeCew argues that decisions that protect the use of sexually explicit materials in the home, consensual sex practices in private, and personal decisions about
abortion are in the interest of women as well as men, even though in some cases, such as wife beating, there may be overriding considerations that justify state intervention. Both authors

For Butler, the danger is that the state becomes


argue persuasively for a more careful look at the dangers lurking behind calls for state action.

arbiter of what is and is not permissible speech, allowing rulings that the erection of burning crosses by
the Ku Klux Klan is protected speech but that artistic expressions of gay sexuality or statements of gay
identity are actions rather than speech and so are not protected. The danger DeCew sees is that once the right to privacy is denied or
narrowly defined, the state can, on the grounds of immorality, move into women's personal lives to interfere with sexual expression, whether homosexual or heterosexual, or with the right to
choose an abortion established in Roe v. Wade. Both DeCew and Butler, however, provide alternative remedies for the admitted harm that state action is intended to redress. For DeCew, the
right to privacy is not absolute; like freedom, it can be overridden by other rights thus the state can intervene in domestic abuse cases because of the physical harm being done. Butler's

Given the postmodern view that the subject


remedy for harmful hate language is more deeply rooted in postmodern theories of the speaking subject.

can never magisterially use a language with fixed meanings according to clear intentions, it is always pos-
sible to subvert the conventional meanings of words. What is said as a derogatory slur"nigger," "chick," "spic," or "gay,"
for example can be "resignified," that is, returned in such a manner that its conventional meaning in practices of

discrimination and abuse is subverted. Butler gives as examples the revalorization of terms like "black" or "gay," the satirical citation of racial or sexual
slurs, reappropriation in street language or rap music, and expressions of homosexual identity in art depicting graphic sex. These are expressions that any erosion in First
Amendment rights might endanger.

Page 779 of 1481


Page 780 of 1481
#4 Censorship Bad Turns: Ext
(4/4)
LANGUAGE CRITIQUES UNDERMINE THE FREE-PLAY OF DIALOGUE NECESSARY FOR
EDUCATION

Matthew Roskoski and Joe Peabody, A Linguistic and Philosophical Critique of Language
Arguments, 1991, http://debate.uvm.edu/Library/DebateTheoryLibrary/Roskoski&Peabody-LangCritiques,
accessed 10/17/02

As Brennan notes, the mandate "to inculcate moral and political values is not a general warrant to act as
'thought police' stifling discussion of all but state-approved topics and advocacy of all but the official
position." (Brennan 577). Not only does the first amendment create a moral or deontological barrier to
language "arguments", the principles it defends also create a pragmatic barrier. The free and sometimes
irreverent discourse protected by the first amendment is essential to the health and future success of our
society. History has borne out the belief that the freedom to challenge convictions is essential to our
ability to adapt to change. As Hyde and Fishman observe, university scholars must be allowed to "think
the unthinkable, discuss the unmentionable, and challenge the unchallengeable" because "major
discoveries and advances in knowledge are often highly unsettling and distasteful to the existing order."
This leads them to conclude that "we cannot afford" to impose "orthodoxies, censorship, and other
artificial barriers to creative thought" (Hyde & Fishman 1485). Given the rapid pace of political and
technological change that our society faces, and given that debates often focus around the cutting edge of
such changes, the imposition of linguistic straitjackets upon the creative thought and critical thinking of
debaters would seem to uniquely jeopardize these interests. This is not just exaggerated rhetoric, nor is it
merely our old debate disadvantages in new clothes. Hyde & Fishman's claims have been repeatedly
validated by historical events. Had Elie Wiesel debated in Germany, a "Zionist language" argument would
not have been unlikely. As Bennett Katz has argued, The essentiality of freedom in the community of
American Universities is almost self-evident... To impose any strait jacket upon the intellectual leaders in
our colleges and universities would imperil the future of our Nation... Teachers and students must always
remain free to inquire, to study and to evaluate, to gain new maturity and understanding; otherwise our
civilization will stagnate and die. (Katz 156).

Page 781 of 1481


Page 782 of 1481
#7 Discourse Focus Trades off
with Action: 1AR
LANGUAGE CRITIQUES ARE COUNTER-PRODUCTIVE FOR PROGRESSIVE POLITICS

David Foster Wallace, Chair in Creative Writing at Pomona College and MacArthur Fellow, Atlantic Monthly, April
2001, p. 54-55.

There's a grosser irony about Politically Correct English. This is that PCE purports to be the dialect of
progressive reform but is in factin its Orwellian substitution of the euphemisms of social equality for
social equality itselfof vastly more help to conservatives and the U.S. status quo than traditional SNOOT
prescriptions ever were. Were I, for instance, a political conservative who opposed taxation as a means of
redistributing national wealth, I would be delighted to watch PCE progressives spend their time and
energy arguing over whether a poor person should be described as "low-income" or "economically
disadvantaged" or "pre-prosperous" rather than constructing effective public arguments for redistributive
legislation or higher marginal tax rates on corporations. (Not to mention that strict codes of egalitarian
euphemism serve to burke the sorts of painful, unpretty, and sometimes offensive discourse that in a
pluralistic democracy leads to actual political change rather than symbolic political change. In other
words, PCE functions as a form of censorship, and censorship always serves the status quo.)

FOCUSING ON HOW WE TALKED ABOUT THE ISSUE, RATHER THAN


HOW TO DEAL WITH IT, TRADES OF WITH ACTIVISM AND DESTROYS
THE ABILITY TO FORM COALITIONS

Ward Churchill, Keetoowah Cherokee, 25+ year member of the American Indian Movement and
Professor, Indigenous Studies, University of Colorado Boulder. FROM A NATIVE SON, 1996 p. 460.

There can be little doubt that matters of linguistic appropriateness and precision are of serious and
legitimate concern. By the same token, however, it must be conceded that such preoccupations arrive at
a point of diminishing return. After that, they degenerate rapidly into liabilities rather than benefits to
comprehension. By now, it should be evident that much of what is mentioned in this article falls under the
latter category; it is, by and large, inept, esoteric, and semantically silly, bearing no more relevance in the
real world than the question of how many angels can dance on the head of a pin. Ultimately, it is a means
to stultify and divide people rather than stimulate and unite them. Nonetheless, such issues of word
choice have come to dominate dialogue in a significant and apparently growing segment of the Left.
Speakers, writers, and organizers or persuasions are drawn, with increasing vociferousness and
persistence, into heated confrontations, not about what theyve said, but about how theyve said it.
Decisions on whether to enter into alliances, or even to work with other parties, seem more and more
contingent not upon the prospect of a common agenda, but upon mutual adherence to certain elements
of a prescribed vernacular. Mounting quantities of a progressive time, energy, and attention are
squandered in perversions of Maos principle of criticism/self-criticism now variously called process,
line sharpening, or even struggle in which there occurs a virtually endless stream of talk about how

Page 783 of 1481


to talk about the issues. All of this happens at the direct expense of actually understanding the issues
themselves, much less doing something about them. It is impossible to escape the conclusion that the
dynamic at hand adds up to a pronounced avoidance syndrome, a masturbatory ritual through which an
opposition nearly paralyzed by its own deeply felt sense of impotence pretends to be engaged in
something meaningful. In the end, it reduces to a tragic delusion at best, cynical game playing or
intentional disruption at worst. With this said, it is only fair to observe that its high time to get off this
nonsense, and on with the real work of effecting positive social change.

Page 784 of 1481


Page 785 of 1481
#7 Discourse Focus Trades off
with Action: Ext
PUNISHING LANGUAGE TRADES OFF WITH MORE EFFECTIVE SOCIAL CHANGE

Matthew Roskoski and Joe Peabody, A Linguistic and Philosophical Critique of Language
Arguments, 1991, http://debate.uvm.edu/Library/DebateTheoryLibrary/Roskoski&Peabody-LangCritiques,
accessed 10/17/02

Previously, we have argued that the language advocates have erroneously reversed the causal
relationship between language and reality. We have defended the thesis that reality shapes language,
rather than the obverse. Now we will also contend that to attempt to solve a problem by editing the
language which is symptomatic of that problem will generally trade off with solving the reality which is
the source of the problem. There are several reasons why this is true. The first, and most obvious, is that
we may often be fooled into thinking that language "arguments" have generated real change. As Graddol
and Swan observe, "when compared with larger social and ideological struggles, linguistic reform may
seem quite a trivial concern," further noting "there is also the danger that effective change at this level is
mistaken for real social change" (Graddol & Swan 195). The second reason is that the language we find
objectionable can serve as a signal or an indicator of the corresponding objectionable reality. The third
reason is that restricting language only limits the overt expressions of any objectionable reality, while
leaving subtle and hence more dangerous expressions unregulated. Once we drive the objectionable idea
underground it will be more difficult to identify, more difficult to root out, more difficult to counteract,
and more likely to have its undesirable effect. The fourth reason is that objectionable speech can create a
"backlash" effect that raises the consciousness of people exposed to the speech. Strossen observes that
"ugly and abominable as these expressions are, they undoubtably have had the beneficial result of raising
social consciousness about the underlying societal problem..." (560).

Page 786 of 1481


Page 787 of 1481
#8 Alternative Fails: 1AR
CRITICISMS OF LANGUAGE FAIL WE CANNOT OBJECTIVELY DETERMINE WHETHER
CERTAIN WORDS ARE GOOD OR BAD. WE CAN ONLY USE LANGUAGE AS A TOOL
NOT AS AN ACCURATE PICTURE OF THE WORLD

Rorty, Prof of Philosophy at Stanford, 82 (Richard, Consequences of Pragmatism,


http://www.marxists.org/reference/subject/philosophy/index.htm)

This Davidsonian way of looking at language lets us avoid hypostatising Language in the way in which the
Cartesian epistemological tradition, and particularly the idealist tradition which built upon Kant, hypostatised Thought. For it
lets us see language not as a tertium quid between Subject and Object, nor as a medium in which we try
to form pictures of reality, but as part of the behaviour of human beings. On this view, the activity of
uttering sentences is one of the things people do in order to cope with their environment. The Deweyan notion of
language as tool rather than picture is right as far as it goes. But we must be careful not to phrase this analogy so as to
suggest that one can separate the tool, Language, from its users and inquire as to its "adequacy" to
achieve our purposes. The latter suggestion presupposes that there is some way of breaking out of
language in order to compare it with something else. But there is no way to think about either the world
or our purposes except by using our language. One can use language to criticise and enlarge itself , as one can
exercise one's body to develop and strengthen and enlarge it, but one cannot see language-as-a-whole in relation to something else to which it applies, or for which it is a means to an end.
The arts and the sciences, and philosophy as their self-reflection and integration, constitute such a process. of enlargement and strengthening. But Philosophy, the attempt to say "how

It is the impossible
language relates to the world" by saying what makes certain sentences true, or certain actions or attitudes good or rational, is, on this view, impossible.

attempt to step outside our skins-the traditions, linguistic and other, within which we do our thinking and
self-criticism-and compare ourselves with something absolute. This Platonic urge to escape from the
finitude of one's time and place, the "merely conventional" and contingent aspects of one's life, is
responsible for the original Platonic distinction between two kinds of true sentence. By attacking this
latter distinction, the holistic "pragmaticising" strain in analytic philosophy has helped us see how the
metaphysical urge -common to fuzzy Whiteheadians and razor-sharp "scientific realists"-works. It has helped us be sceptical
about the idea that some particular science (say physics) or some particular literary genre (say Romantic poetry,
or transcendental philosophy) gives us that species of true sentence which is not just a true sentence, but rather a
piece of Truth itself. Such sentences may be very useful indeed, but there is not going to be a Philosophical explanation of this utility. That expl anation, like the
original justification of the assertion of the sentence, will be a parochial matter-a comparison of the sentence with alternative sentences formulated in the same or in other
vocabularies. But such comparisons are the business of, for example, the physicist or the poet, or perhaps of the philosopher - not of the Philosopher, the outside expert on
the utility, or function, or metaphysical status of Language or of Thought. The Wittgenstein-Sellars-Quine-Davidson attack on distinctions between classes of sentences is the
special contribution of analytic philosophy to the anti-Platonist insistence on the ubiquity of language. This insistence characterises both pragmatism and recent
"Continental" philosophising. Here are some examples: Man makes the word, and the word means nothing which the man has not made it mean, and that only to some o ther
man. But since man can think only by means of words or other external symbols, these might turn around and say: You mean noth ing which we have not taught you, and then
only so far as you address some word as the interpretant of your thought. . . . . . . the word or sign which man uses is the man himself Thus my language is the sum -total of
myself; for the man is the thought. (Peirce) Peirce goes very far in the direction that I have called the de -construction of the transcendental signified, which, at one time or
another, would place a reassuring end to the reference from sign to sign. (Derrida) . . . psychological nominalism, according to which all awareness of sorts, resemblances,
facts, etc., in short all awareness of abstract entities-indeed, all awareness even of particulars-is a linguistic affair. (Sellars) It is only in language that one can mean something
by something. (Wittgenstein) Human experience is essentially linguistic. (Gadamer) . . . man is in the process of perishing as the being of language continues to shine ever
brighter upon our horizon. (Foucault) Speaking about language turns language almost inevitably into an object . . . and then its reality vanishes. (Heidegger) This chorus should
not, however, lead us to think that something new and exciting has recently been discovered about Language-e.g., that it is more prevalent than had previously been thought.

They are saying that attempts to get back behind language to something
The authors cited are making only negative points.

which "grounds" it, or which it "expresses," or to which it might hope to be "adequate," have not, worked.
The ubiquity of language is a matter of language moving into the vacancies left by the failure of all the
various candidates for the position of "natural starting-points" of thought, starting-points which are prior
to and independent of the way some culture speaks or spoke. (Candidates for such starting-points include clear and distinct ideas,
sense-data, categories of the pure understanding, structures of prelinguistic consciousness, and the like.) Peirce and Sellars and Wittgenstein are saying that the regress - of
interpretation cannot be cut off by the sort of "intuition" which Cartesian epistemology took for granted. Gadamer and Derrid a are saying that our culture has been
dominated by the notion of a "transcendental signified" which, by cutting off this regress, would bring us out from contingen cy and convention and into the Truth. Foucault is
saying that we are gradually losing our grip on the "metaphysical comfort" which that Philosophical tradition provided-its picture of Man as having a "double" (the soul, the
Noumenal Self) who uses Reality's own language rather than merely the vocabulary of a time and a place. Finally, Heidegger is cautioning that if we try to make Language into
a new topic of Philosophical inquiry we shall simply recreate the hopeless old Philosophical puzzles which we used to raise a bout Being or Thought. This last point amounts to
saying that what Gustav Bergmann called "the linguistic turn" should not be seen as the logical positivists saw it -as enabling us to ask Kantian questions without having to
trespass on the psychologists' turf by talking, with Kant, about "experience" or "consciousness." That was, indeed, the initi al motive for the "turn,"" but (thanks to the holism

analytic philosophy of language was able to transcend this Kantian motive and
and pragmatism of the authors I have cited)

adopt a naturalistic, behaviouristic attitude toward language. This attitude has led it to the same outcome
as the "Continental" reaction against the traditional Kantian problematic, the reaction found in Nietzsche and Heidegger. This
convergence shows that the traditional association of analytic philosophy with tough-minded positivism and of "Continental" philosophy with tender-minded Platonism is completely

misleading. The pragmaticisation of analytic philosophy gratified the logical positivists' hopes, but not in the

Page 788 of 1481


fashion which they had envisaged. it did not find a way for Philosophy to become "scientific," but rather
found a way of setting Philosophy to one side. This post-positivistic kind of analytic philosophy thus comes to resemble the Nietzsche-Heidegger-Derrida
tradition in beginning with criticism of Platonism and ending in criticism of Philosophy as such. Both traditions are now in a period of doubt about their own status. Both are living between a
repudiated past and a dimly seen post-Philosophical future.

Page 789 of 1481


Holocaust Trivialization Answers: 2AC (1/3)

1. NO LINK WE DONT ASSERT THAT AN EVENT WILL OCCUR THAT IS IDENTICAL


THE HOLOCAUST, BUT SHOW THE POTENTIAL FOR ATROCITIES TO OCCUR IN THE
FUTURE BASED ON HISTORICAL EXAMPLES, SUCH AS THE ONE PERPETRATED IN
NAZI GERMANY

2. TURN - REJECTION OF HOLOCAUST RHETORIC DEPOLITICIZES IT AND RESULTS IN


FURTHERING OF VICTIMIZATION, IGNORING THIRD WORLD VIOLENCE, AND THE
FAILURE OF RADICAL POLITICAL ENGAGEMENT

Zizek 2001
[Slavoj, Go away, Did Somebody Say Totalitarianism?, 2001, New York: Verso, 2001, 67-8//uwyo]

Are these not the terms that designate the Lacanian encounter of the Real? However , this very depoliticization of the Holocaust, its
elevation into the properly sublime Evil, the un'touchable Exception beyond the reach of 'normal' political
discourse, can also be a political act of utter cynical manipulation, a political intervention aiming at
legitimizing a certain kind of hierarchical political relation. First, it is part of the postmodern strategy of
depoliticization and/or victimization. Second, it disqualifies forms of Third World violence for which Western
states are (co)responsible as minor in comparison with the Absolute Evil of the Holocaust. Third, it serves to cast a shadow over every
rad-ical political project - to reinforce the Denkverbot against a radical political imagination: 'Are you aware that what you propose leads ultimately to the
Holocaust?' In short: notwithstanding the unquestionable sincerity of some of its proponents, the 'objective' ideologico-political content of the

depoliticization of the Holocaust, of its ele-vation into the abyssal absolute Evil, is the political pact of aggressive Zionists
and Western Rightist anti-Semites at the expense oftoday's radical political possibilities. In it, Israeli expansionism
towards Palestinians para-doxically joins hands with the Western anti-Semite's avoidance of the concrete analysis of the political dynamics of anti-Semitism -of how this same

dynamics is today pursued by other means (or, rather, with other goals, displaced on to other targets ).

3. ENDORSING THE UNIQUENESS OF THE HOLOCAUST IMPLICITLY DENIES OTHER


HISTORICAL GENOCIDES, MAKING FUTURE GENOCIDE INEVITABLE

Stannard 96
[David E., Prof. Am Studies @ Hawaii, Uniqueness as Denial, Is the Holocaust Unique? Ed. Rosenbaum,
197]

In addition to the damage that is inherent tin the cultural violence of genocide denial there is the matter of the future dangers that it promotes. As Roger Smith, Eric Markusen,
and Robert Jay Lifton recently have written regarding the continuing denial of the Armenian holocaust :

Where scholars deny genocide, in the face of decisive evidence that it has occurred, they contribute to a false consciousness
that can have the most dire reverberations. Their message, in effect, is: murderers did not really murder;
victims were not really killed, mass murder requires no confrontation, no reflection, but should be ignored, glossed over. In

this way scholars lend their considerable authority to the acceptance of this ultimate human crime. More than that, they encourage indeed invite a

Page 790 of 1481


repetition of that crime from virtually any source in the immediate or distant future. By closing their
minds to truth, that is, such scholars contribute to the deadly psychohistorical dynamic in which unopposed
genocide begets new genocides.
This, of course, is one of the great and justified fears that Jews long have harbored regarding the threat of Holocaust denial that it invites repetition and anti-Jewish mass

violence and killing. Butwhen advocates of the allegedly unique suffering of the Jews during the Holocaust
themselves participate in denial of other historical genocides and such denial is inextricably interwoven
with the very claim of uniqueness they thereby actively participate in making it much easier for those
other genocides to be repeated. And, in the case of genocides against the native peoples of the Americas, not to be repeated but to continue. As, indeed,
they are at this very moment. For never, really, have they stopped.

Page 791 of 1481


Page 792 of 1481
Holocaust Trivialization
Answers: 2AC (2/3)
4. OUR COMPARISONS DONT DENY THE UNIQUENESS OF THE SHOAH, OUR
COMPARISON OF THE DIFFERENCES AND SIMILARITIES PRECLUDES SUCH A
BLANKET ATTITUDE AND DEMONSTRATES HOW EASY IT IS FOR NORMAL PEOPLE
TO LAPSE INTO ATROCITY

Lifton & Markusen 90


[Robert, Psych prof at John Jay College of Criminal Justice, Eric, Researcher at Danish Center for Holocaust
and Genocide Studies, The Genocidal Mentality, 9-11//uwyo]

It is neither easy nor pleasant to invoke the Nazis for comparison with gorups within our own democratic
society. Our image of the Nazis tends to be that of thugs murderers, an image they did much to earn. But
students of Nazi genocide have long stressed what recent work on the Nazi doctors has confimred:
namely, that ordinary Germans became involved in killing, people who had previously shown no particular
inclination toward violence. These findings are especially troubling because they bring the Nazis closer to
the rest of us. We are much more comfortable viewing them as a separate tribe of demons. But the
painful truth is that, they are more part of our century, more involved in historical and psychological
questions that still bedevil us, than we have wished to acknowledge. In our present genocidal
predicament, responsibility lies in seeking to draw form the Nazi project lessions that might head off the
ultimate nuclear Auschwitz.

To use the Nazis comparatively in this manner is in no way to deny the uniqueness of their Holocaust. No
other historical genocide has been so systematically carried out against an entire people, even to attempt
to round up Jews from virtually all over the weorld in order to kill them. We therefore reject the
revisionist position of some German historians to the effect that the Holocaust is just one of the many
examples of cruelty that dominate human history, and should be given no special emphasis. We would, in
fact, insist upon stressing differences or disanalogies between the Nazi and nuclear situations. As
Charles S. Maier explains, while exploring similar quesitons, Comparison is a dual process that scrutinizes
two or more systems to learn what elements they have in common and what elements distinguish them.
It does not assert identity; it does not deny unique components.

The most fundamental difference, of course, is that Nazi mass killing is a matter of historical record, so
that (as one observer put it) even if nuclear-weapons arrangements are viewed as an Auschwitz waiting
to happen, no one is being gassed or cremated. The distinction is between the actual and the potential.
Another fundamental difference has to do with intent. The Nazis killed designated victimes primarily
Jews, but also Gypsies, Poles, Russians, mental patients, and homosexuals. In contrast, the stated nuclear
intent is to prevent war, and the killing would take place only with a failure of that structure of
deterrence. Still another difference is the reality of a dangerous adversary: the Jews posed no threat to
the Nazis, but the Soviets pose a real military threat to us. And many more differences would emerge with
a fuller exploration of the complexities of German history. There is a final, sobering difference having ot
do with victimizers and victims. There was a clear-cut distinction between the Nazis themselves as
perpetrators and those they decided to kill. In the nuclear case, should the weapons be used, there will be
no such distinctions: everybody would become a victim. At Nuremberg, after the Second World War,
there was an attempt to hold individuals and groups accountable for their role in killing. There can be no
nuclear Nuremberg; hope lies only in establishign responsibility for genocide prior to its occurring
responsibility for participating in a genocideal system and a genocidal process.

Page 793 of 1481


Page 794 of 1481
Holocaust Trivialization
Answers: 2AC (3/3)
5. PROHIBITING COMPARISONS BETWEEN THE HOLOCAUST AND OTHER
HISTORICAL OCCURRENCES PUTS US IN AN IMPOSSIBLE POSITION OF NOT BEING
ALLOWED TO REFLECT ON IT, PLAYING INTO THE HANDS OF HOLOCAUST DENIERS

Zizek 2001
[Slavoj, Prof. @Inst. Of Sociology, U. of Ljubliana, Repeating Lenin, 2001, www.lacan.com/replenin.htm
Acc. 8-20-04]

In a closer analysis, one should exhibit how the cultural relativism of the "right-to-narrate" orientation
contains its own apparent opposite, the fixation on the Real of some trauma which resists its
narrativization. This properly dialectical tension sustains today's the academic "holocaust industry." My
own ultimate experience of the holocaust-industry police occurred in 1997 at a round table in the Centre
Pompidou in Paris: I was viciously attacked for an intervention in which (among other things) I claimed,
against the neoconservatives deploring the decline of faith today, that the basic need of a normal human
being is not to believe himself, but to have another subject who will believe for him, at his place - the
reaction of one of the distinguished participants was that, by claiming this, I am ultimately endorsing the
holocaust revisionism, justifying the claim that, since everything is a discursive construct, this includes also
the holocaust, so it is meaningless to search for what really happened there... Apart from displaying a
hypocritical paranoia, my critic was doubly wrong: first, the holocaust revisionists (to my knowledge)
NEVER argue in the terms of the postmodern discursive constructionism, but in the terms of very
empirical factual analysis: their claims range from the "fact" that there is no written document in which
Hitler would have ordered the holocaust, to the weird mathematics of "taking into account the number of
gas ovens in Auschwitz, it was not possible to burn so many corpses." Furthermore, not only is the
postmodern logic of "everything is a discursive construction, there are no direct firm facts" NEVER used to
deflate the holocaust; in a paradox worth noting, it is precisely the postmodern discursive constructionists
(like Lyotard) who tend to elevate the holocaust into the supreme ineffable metaphysical Evil - the
holocaust serves them as the untouchable-sacred Real, as the negative of the contingent language
games.26

The problem with those who perceive every comparison between the holocaust and other concentration
camps and mass political crimes as an inadmissible relativization of the holocaust, is that they miss the
point and display their own doubt: yes, the holocaust WAS unique, but the only way to establish this
uniqueness is to compare it with other similar phenomena and thus demonstrate the limit of this
comparison. If one does not risk this comparison, of one prohibits it, one gets caught in the
Wittgensteinian paradox of prohibiting to speak about that about which we cannot speak: if we stick to
the prohibition of the comparison, the gnawing suspicion emerges that, if we were to be allowed to
compare the holocaust with other similar crimes, it would be deprived of its uniqueness...

Page 795 of 1481


Page 796 of 1481
A2 Representation Links (1/4)
REPRESENTATIONAL VIOLENCE DOES NOT PRECLUDE THE NEED FOR CONCRETE
ACTION

Richard Rorty, Professor of Humanities, University of Virginia, Truth, Politics, and Postmodernism, Spinoza Lectures,

1997, p. 51-2

This distinction between the theoretical and the practical point of view is often drawn by Derrida, another writer who enjoys demonstrating that something very important
meaning, for example, or justice, or friendship is both necessary and impossible. When asked about the implications of these paradoxical fact, Derrida usually replies that

the paradox doesn't matter when it comes to practice. More generally, a lot of the writers who are labeled `post-modernist; and
who talk a lot about impossibility, turn out to be good experimentalist social democrats when it comes to actual

political activity. I suspect, for example, that Gray, Zizek, Derrida and I, if we found ourselves citizens of the same country, would all be voting for the same
candidates, and supporting the same reforms. Post-modernist philosophers have gotten a bad name because of their paradox-mongering habits, and their constant use of
terms like `impossible; `self-contradictory' and `unrepresentable'. They have helped create a cult of inscrutability, one which defines itself by opposition to the Enlightenment
search for transparency - and more generally, to the `metaphysics of presence; the idea that intellectual progress aims at getting things clearly illuminated, sharply delimited,

I am all for getting rid of the metaphysics of presence, but I think that the rhetoric of impossibility and
wholly visible.

unrepresentability is counterproductive overdramatization. It is one thing to say that we need to get rid of the metaphor of things being
accurately represented, once and for all, as a result of being bathed in the light of reason. This metaphor has created a lot of headaches for philosophers, and we would be
better off without it. But that does not show that we are suddenly surrounded by unrepresentables; it just shows that `more accurate representation' was never a fruitful way to

Even if we agree that we shall never have what Derrida calls "a full presence beyond the reach
describe intellectual progress.

of play"; our sense of the possibilities open to humanity will not have changed. We have learned nothing about the limits of
human hope from metaphysics, or from the philosophy of history, or from psychoanalysis. All that we have learned from `post-modern' philosophy is that we may need a
We have been given no reason to abandon
different gloss on the notion of `progress' than the rationalistic gloss which the Enlightenment offered.

the belief that a lot of progress has been made by carrying out the Enlightenment's political program. Since
Darwin we have come to suspect that whether such progress is made will be largely a matter of luck. But we have been given no reason to stop hoping to get lucky.

FOCUS ON REPRESENTATIONAL VIOLENCE WORSENS REAL VIOLENCE

Elana Gomel, Tel-Aviv University, Written in Blood: Serial Killing and Narratives of Identity, Post Identity, Volume
2, Number 1, Winter 1999, p. 24-25, http://ids.udmercy.edu/pi/2.1/PI21_24-70.pdf, accessed 1/28/02

ONE CAN START WITH FOUCAULTS famous and endlessly circulated statement in The Order of Things: It is comforting, however, and a source of profound relief to think
that man is only a recent invention, a figure not yet two centuries old, a new wrinkle in our knowledge, and that he will disappear as soon as this knowledge has discovered a
new form. (xxiii) Man the Universal Subject, a cookie-cutter mold of (post)technological identity, stamping out simulacra of individuality. But why should we be comforted and
experience relief at the thought of his imminent dissolution? Perhaps because, at least from Adorno on, the subject of reason has also been identified as the subject of

the Enlightenment has been reconceptualized as the universal killer, armed with the
violence. The universal Man of

most potent of weaponsrepresentation. In their Introduction to the collection typically entitled Violence of Representation Armstrong and
Tennenhouse offer the basic formula of this approach: The violence of representation is the suppression of difference (8). In this particular reading of Foucault the discursive
constructedness of identity is directly responsible for corporeal violence inflicted by some (post)modern subjects upon others. In his recent book Serial Killerr and in the series
of articles that preceded it Mark Seltzer applies this insight to the fascinating and grisly phenomenon of serial killing, variously identified also as stranger killing and sometimes

The serial killer, I will be


lust murder. For Seltzer the enigma of the serial killers personality consists in an experience of typicality at the level of the subject

arguing, is in part defined by such a radicalized experience of typicality within. Simply put, murder by numbers
(as serial murder has been called) is the form of violence proper to statistical persons. (30-1) Violence of representation,

representation of violence and violence per se smoothly link into an unbroken chain, leading from statistics to mayhem
and from typology of subjects to fingertyping of putrefying bodies. My goal in this essay is to put a hitch into this chain, to question the easy fit

between discursive moulds of identity and the individual self-experience of serial killers, and to suggest that represenration may be not so much the

cause of violence as a post factum defence against it. I do not imply, however, that violence in general or serial murder in particular are totally
free from the constraints of discourse or that the identity of the serial killer is not constructed using the building blocks of cultural narratives (though the narratives in question

the serial form of violence is conditioned not so much by the


are more variegated than Seltzer suggests). Rather, I would claim that

monolithic coherence of representation as by its breakdown. The violent behavior of a serial killer is not a direct outcome of any social
construction but a random, causeless choice which is retrospectively incorporated into a generic narrative of identity. The repeated ritualistic violence, then, becomes a means
of reinforcing this identity but achieves precisely the opposite, its complete disintegration. Rather than being generated by representation, corporeal violence offers a resistance
to it.

Page 797 of 1481


Page 798 of 1481
A2 Representation Links (2/4)
THEY MISUNDERSTAND COGNITION - IDENTIFICATION WITH IMAGES OF
DOMINATION UNDERMINES RELATIONSHIPS OF SUBORDINATION

Krips '99
[Henry, Professor of Communication at the Pitt, Fetish: an erotics of culture, Ithaca, Cornell University
Press, 1999, 5-6//uwyo-ajl]

Arguments against linking the cultural and psychic realms also seem apposite in criticizing MacKinnon's claim
that there exists a direct causal connection between pornography and a psychic characteristic of its male
consumers, namely sexual aggression. At a theoretical level, her argument fails to take into account
Freud's point that identification with a phantasy figure flows readily across gender lines. For example, in the
Dora case, Freud argues that Dora's behavior manifests an unconscious desire for Frau K., her father's lover and suitor's
wife. For Freud her desire does not indicate any sexual instability. Instead, through an identification with her father's
desire, it signals an unconscious paternal identification. In other words, for Freud the significant aspect of Dora's
phantasy is not the sexual content of the desire but rather the paternal position from which she engages
with it. By parity of reasoning, it follows that quite "normal" male readers of porn may identify with the
position of woman victim rather than male aggressor, in which case their aggressive tendencies cannot be
reinforced in the simplistic way that MacKinnon suggests.3 In short, as Laura Kipnis points out, neither
the biology nor gender of readers of Hustler magazine determines the form of their identification with its
pornographic materials, let alone forces them into a common psychic response (Kipnis 1996, 196). In the
same way, one may argue, gender-swapping phantasy games played by Net users do not indicate their gender
instability. On the contrary. one might turn the argument around and conclude that the preponderance of biological
males among Net users suggests that even when playing at being a woman, they are engaging in a "boys' game."

Page 799 of 1481


Page 800 of 1481
A2 Representation Links (3/4)
THEIR CRITIQUE OF REPRESENTATIONS IS UTOPIAN, BECAUSE REPRESENTATION
CAN NEVER BE PERFECT - RECOGNIZING THAT OUR REPRESENTATIONS ARE
IMPERFECT OPENS US THE POSSIBILITY OF AN ETHICAL RELATIONSHIP TO OTHERS

Colebrook,"Questioning Representation", 2000

As I've already suggested, a strain of nostalgia and utopianism runs through both forms of anti-representationalism: both
the desire to return to a world that is lived as present, rather than subjectively re-presented, and the desire to overcome
all commitments to presence in the celebration of a [End Page 59] differential, non-autonomous and post-human writing. If the concept of representation
generates the consistent incoherence of a real that is then represented and a subject who then represents, we have to ask ourselves whether we

can cleanse thought of the risky vocabulary of representation, whether we can return to the lived immediacy of pre-modern pre-
subjective mutual recognition, or whether we can paste over our Cartesian separation and think a world that is not written by us but that writes itself. Is the representational
antinomy or paradox an accident and is it curable?

We might consider post-Kantian anti-representationalism as an increasing anti-subjectivism. Talk of schemes, representations, constructions, and paradigms does generate
notions of what these schemes are schemes of. To talk of representation as a construction, schematization or structuration also implies that there is one who constructs, or that
there is (to use Nietzsche's phrase) a doer behind the deed (Nietzsche 1967, 45). Representation presents us with what Michael Dummett refers to as the danger of falling back
into psychologism (1993, 129).

How possible is it to overcome these illusions and to remain within representation without appealing to what is, or, more important, without demanding autonomy? Perhaps
representation in both its epistemological and ethical/political senses is valuable precisely for the contradictions and tensions it presents for thought. Consider, to begin with,
knowledge as representation and the possibility that we might no longer trouble ourselves with an ultimate foundation for our representations, and this because any attempt to
do so would bring us up against our own representational limit. In Realism with a Human Face, Hilary Putnam distinguishes between two broad readings of Wittgenstein's
notion that the limits of my language are the limits of my world. The first response to such a predicament would be to rule out as nonsensical any attempt to think outside my
world. The second response, favored by Putnam, would be that this recognition brings us up against the very notion that my world is my world (Putnam 1990, 28). While we
have no appeal or foundation that lies outside representation, we sustain a philosophical question in the face of this inability. We might say, then, that rather than be ruled out of
court as a nonsensical illusion, representation functions as a useful antinomy. The idea that our world is always a represented world renders us both responsible for that world,
at the same time as we recognize our separation or non-coincidence with the world.

And this might be how we can retrieve a notion of autonomy through representation in the second, ethical, sense. As I have already suggested, autonomy need not be defined
as the feature of pre-social or pre-linguistic [End Page 60] moral individuals. Rather, to take an act of speech as autonomous is to see it as not grounded in a pre-given, law,
nature or being. Thus the "subject" on this account would not be a substantive entity that authors its own meaning fully, but would be effected through acts of representation.
Why save a notion of subjective autonomy? Think of the converse situation: a world of writing effects, disowned speech acts, performances without performers or moves in a
game without players. Such a world imagines that it is possible to have a form of speech that does not carve out a point of view, that is not located in a way of being, that
presents no resistance to perpetual coming and self-invention. It is a world in which the representational illusion is disavowed, a world in which speech takes place without the
reifying error that I imagine myself as one who speaks. The idea that there is a writing, speaking or language that represents and that can't be owned by subjects does, quite

what such an idea of a


sensibly, challenge the idea that what we say is a straightforward representation of some pre-linguistic meaning or ownness. But

radically anonymous writing in general precludes is the autonomy effects generated through processes of
representation. Just as cultural studies--we are told--dreams of a world in which truth claims, foundations and representational claims are no longer made, and just as
Richard Rorty imagines a world of ironists who accept their language games as nothing more than games and themselves as nothing more than players (Rorty 1989, 80), so
the attempt to think beyond autonomy imagines a world in which what I say is not taken as issuing from the intention of some reified, congealed and illusory notion of man. But
we might think of autonomy alongside the antinomy of representation. To take demands as autonomous is to recognize them as both ungrounded, as well as being demands
for a certain grounding. If what I say makes a claim for autonomy, then it is both owned as what I say (and thereby institutes me as a subject), at the same time as the claim for
autonomy separates this saying from any pre-given subject. To be autonomous, a claim would have to be more than a determined expression of a subject; it would have to
have its own positive, singular and effective force. As Kant argued, true autonomy could not be thought of as issuing from a natural ground; but once we think an autonomous
law this generates the regulative idea (but not knowledge) of a subject from whom this law has issued.

Consider this antinomy in terms of some of the typical approaches to representation in popular culture--in particular, in popular feminism. It is widely asserted that women are
subordinated to alien domains of representation. Eating disorders are explained by referring to the non-representative nature of bodies in the media (Wolf 1990); pornography is
[End Page 61] criticized as a misrepresentation of women as passive and compliant sexual objects (Dworkin 1982); and, in general, the notion of stereo-type functions
throughout feminism and other critical movements to suggest that subjects suffer from alien representations. This critical approach to alien, imposed or stereotypical
representations more often than not issues in the demand for more accurate, authentic or autonomous
representations. In its simplest forms, the diagnosis of certain practices as a form of representational violence is
tied to the demand for an overcoming of the representational divide. This demand would supposedly be
met by more realistic images of women, by non-patriarchal or non-objectifying erotica, and through the freeing of women from the representational closure of
the beauty myth. What is demanded, in short, is that the subject be continuous with representations: that there might be a public domain of representation that is at one with
one's inner being, where subjects would not regard themselves as extrinsic to, or belied by, a general representational norm.

.
However, it is just this demand for non-separation from representation that sustains the problem, and part of this problem lies in not addressing the predicament of autonomy

The idea of a representation that would not be alien to my being would only be possible on two counts: either by
resisting the necessary discontinuity of representation and insisting on the possibility of a proper or essential representation, or by
imagining that we could do away with being altogether, such that representation would not be seen as discontinuous or alien to any pre-
presentational thing. These two possibilities might be cashed out as follows. On the one hand, we could achieve a social domain of complete mutual recognition (perhaps
something like the Greek polis or the bourgeois public sphere) in which the individual is thoroughly at one with the social whole. There would be no need for a demand for
representation precisely because what functioned as a normative representation of the individual would already be thoroughly normal. The domain of representation would be
entirely proper, not an alienation of my being, but its adequate expression. On the other hand, the representational scar might be healed by a radical resistance to
representation in general: the refusal of all norms, stereotypes or reified concepts of the individual. This would issue in the pulverization of the representational domain, a
multiplication of images, writing effects, simulacra or texts without author, identity or subject.

Page 801 of 1481


On both these accounts, what is resisted, refused or targeted as a symptom is autonomy: the idea of a self or subject outside the domain of representation. In the first model of
recognition, autonomy is lamented as a [End Page 62] symptom of a public/private divide that has alienated the subject from socially recognizable being. For, it is argued, I
need only demand autonomy in a world that already seems set over against me, in a world that is not fully my world. In the second anti-representational model of proliferating
simulacra or the virtual, what is resisted is the idea that there is an autonomous subject who represents (or is represented). There is, rather, nothing other than representation;
and this means that, strictly speaking, we are no longer talking about representation. In both these cases, one imagines a continuity with the world, a non-separation of
representation such that the horrors of anthropologism are resisted: life is not subordinated to some alien, imposed, or externally given notion of man.

continued

Page 802 of 1481


Page 803 of 1481
A2 Representation Links (4/4)
continued

Butwhat gets lost in this post-representational utopia (or retrieval of the pre-representational polity) is the predicament of
autonomy and its attendant responsibility. If we acknowledge an essential separation of representation, then we also have to allow the question of who represents or
what is demanding representation. The error of not doing so would be anthropomorphism: a failure to recognize that

experiencing a world is always experiencing it in a certain way, from a certain point of view, and through a certain form of life. This
does not entail substantive subjectivism - the idea that the world is given to some pre-given subject. But the world is given in a certain way, and this establishes a position with
regard to the world. Acknowledging this minimal form of ownness or location of knowledge therefore entails that we cannot think of the world as writing itself, giving itself or
offering itself in a dispersed, anonymous or continuous representation. The idea of autonomy enables us to think the point or determination of the world's representation:

the dream of a pre-


autonomy, not in the sense of giving oneself a law, but in recognizing an effected lawfulness of the self. By contrast,

representational world in which all human beings recognized themselves in a common good, disavows the extent to which this
common good, as common, must already have separated itself from the immediacy of any single
experience. If we think of autonomy as a responsibility for the essential separation of representation, then we bring back a fruitful tension.

To represent oneself is to submit to a trans-individual system of


The idea of autonomous representation is, perhaps, an oxymoron.

language, signification or representation. But any such representational scheme can never be fully disowned, rendered
anonymous, collective, inhuman or fully dispersed beyond all subjectivity. Rather, the act of representation institutes autonomy, or places a self in a point of view. Autonomy
ought not to be [End Page 63] defined in terms of a being that is then expressed. Rather, the procedure of autonomy is a recognition that there is no foundational being other
than its continual institution through a representation that dislocates itself from a prior presence. If we do not recognize that representation effects an autonomy that it can then
be seen to belie, if we try to overcome this scar of representation, then we do so at the expense of forgetting what it is to think. In short, we attack the error of anthropologism--
the idea of a general human subject who represents us all--with the error of anthropomorphism: the idea of a world that is fully and adequately given, without representation,

separation or the contribution of thought .

REPRESENTATIONAL VIOLENCE IS NOT THE SAME AS ACTUAL VIOLENCE

Elana Gomel, Post Identity, Winter 1999

http://ids.udmercy.edu/pi/2.1/PI21_24-70.pdf

The universal Man of the Enlightenment has been reconceptualized as the universal killer, armed with the
most potent of weaponsrepresentation. In their Introduction to the collection typically entitled Violence
of Representation Armstrong and Tennenhouse offer the basic formula of this approach: The violence of
representation is the suppression of difference (8). In this particular reading of Foucault the discursive
constructedness of identity is directly responsible for corporeal violence inflicted by some (post)modern
subjects upon others. In his recent book Serial Killers and in the series of articles that preceded it Mark
Seltzer applies this insight to the fascinating and grisly phenomenon of serial killing, variously identified
also as stranger killing and sometimes lust murder. For Seltzer the enigma of the serial killers
personality consists in an experience of typicality at the level of the subject:

The serial killer, I will be arguing, is in part defined by such a radicalized experience of typicality within.
Simply put, murder by numbers (as serial murder has been called) is the form of violence proper to
statistical persons. (301)

Violence of representation, representation of violence and violence per se smoothly link into an unbroken
chain, leading from statistics to mayhem and from typology of subjects to fingertyping of putrefying
bodies. My goal in this essay is to put a hitch into this chain, to question the easy fit between discursive
moulds of identity and the individual self-experience of serial killers, and to suggest that representation
may be not so much the cause of violence as a post factum defence against it.

I do not imply, however, that violence in general or serial murder in particular are totally free from the
constraints of discourse or that the identity of the serial killer is not constructed using the building blocks

Page 804 of 1481


of cultural narratives (though the narratives in question are more variegated than Seltzer suggests).
Rather, I would claim that the serial form of violence is conditioned not so much by the monolithic
coherence of representation as by its breakdown

Page 805 of 1481


Page 806 of 1481
A2 Indigenous Peoples Labels
Bad: 2AC
CAN APPROPRIATE FLAWED RHETORIC OF OTHERS IN ORDER TO CHALLENGE IT AS
LONG AS OUR OWN RHETORIC IS LIBERATORY

Pewewardy 99
[Cornel, Professor of Education at University of Kansas, Multicultural Educaiton, Vol 6 No 3, Spring 1999,
6//uwyo-ajl]

Previous research focusing on aboriginal peoples in the United States have used American Indian, Indian,
and Native American as the nomenclature for this population. This article subverts this traditiona by
instead using the terms Indigenous Peoples and First Nations People. These terms are capitalized
because they are proper nouns (particular persons) and not adjectives (words describing nouns). It is also
capitalized to signifiy and recognize the cultural heterogeneity and political sovereignty of Indigenous
Peoples in the western hemisphere (Yellow Bird, personal communication, 1997). In this respect, the
consciousness of the oppressor transforms Inidgenous identity into a commodity of its domination and
disposal (Freire, 1997). Ceasing to call Indigenosu Peoples American Indians is more than an attempt at
political correctness. It is an act of intellectual liberation and it is a correction to a distorting narrative of
imperialist discovery and progress that has been maintained far too long by Europeans and Euro-
Americans. Thus, American Indian and Indian are sometiems used interchangeably in the vernacular of
this article only when trying to make a point in an attempt to liberate and combat linguistic hegemony,
which is both a direct and indrect power block to the identity of Indigenous Peoples (Yellow Bird, personal
communication, 1997).

FOCUSING ON LABELS FOR INDIGENOUS PEOPLES REIFIES NON-INDIGENOUS


RIGHTS TO LAND AND DEFLECTS LEGITIMATE CRITICISM, CREATING MORE
DISEMPOWERMENT

dErrico 98
[Peter, Prof of Legal Studies at University of Massachusettes/Amherst, Native American Studies A Note
on the Name, www.umass.edu/nativestudies/name.html, April 1998, acc 9-20-04//uwyo-ajl]

Concern for political correctness focused more on appearances than reality. As John Trudell observed at
the time, "They change our name and treat us the same." Basic to the treatment is an insistence that the
original inhabitants of the land are not permitted to name themselves. As an added twist, it seems that
the only full, un-hyphenated Americans are those who make no claim of origin beyond the shores of this
land. Many of these folk assert that they are in fact the real "native" Americans.

Page 807 of 1481


EPrime Answers: 2AC (1/3)

EPRIME EXISTS ONLY AS A LINGUISTIC TOOL- IT IS NOT AN INSTRUMENT OF


POWER

WILSON NO DATE
[Robert Anton, nqa, http://www.nobeliefs.com/eprime.htm ]

Another concern I hear from people involves a false belief that those who advocate the use of E-prime
wish to change the English language through some form of coercion, or lawful action. Folks, E-prime
serves as a linguistic tool, not as an instrument of power. I know of no advocate of E-prime, including its
inventors, who desire to change the history of literature or to force people to use E-prime. Almost all of
the works of literature, poetry, and religious scripture contain abundant uses of non-E-prime and I've yet
to meet an E-prime advocate who wishes to change that.

EPRIME EXCLUDES ALTERNATIVE FORMS OF LANGUAGE- USING EPRIME IS


CENSORSHIP

FRENCH 1992
[James D, Computer Programmer that writes a lot about linguistics and semantics including articles
published in The Journal of Symbolic Logic, Top Ten Arguments Against Eprime, Et Cetera, http://learn-
gs.org/library/etc/49-2-french.pdf //wyo-pinto]

THE CLAIM THAT E-Prime has an inherent, beneficial effect on

a person's writing ability seems highly questionable, considering

that E-Prime deliberately eliminates a whole class of

statements from the language, resulting in fewer alternatives .

The English writer can use all of the statements available to

the E-Prime writer, plus a whole class of statements containing

the verb "to be." The greater variety of available wordings

should make the English writer's efforts more interesting

to read, not less. (Any bad writing that occurs because of the

over-use of the verb "to be" - a common failing - can be

more easily overcome by simply cutting back on one's use of

"to be," rather than resorting to E-Prime .)

Page 808 of 1481


YOUR CONFLICT CLAIMS ARE BULLSHIT- CONTEXT SOLVES THE LINKS TO YOUR
ARGUMENT

FRENCH 1992
[James D, Computer Programmer that writes a lot about linguistics and semantics including articles
published in The Journal of Symbolic Logic, Top Ten Arguments Against Eprime, Et Cetera, http://learn-
gs.org/library/etc/49-2-french.pdf //wyo-pinto]

The harmful effects that may result from the use of the isof-

identity and the is-of-predication are often ameliorated by

the context, and so the need to eliminate all such statements

from our language is not as great as the advocates of E-Prime

apparently assume. It is one thing to say, "The rose is red" in

a flat statement of "fact"; it is quite another to say, "The rose is

red to me." If in response to the question, "What does John

Jones do for a living?" I answer, "He's a professor," there

seems to be little that a general semanticist should quarrel

with, given that the response is occurring within the context

of asking what the man does for a living, a context that greatly

affects the meaning of the answer .

Page 809 of 1481


EPrime Answers: 2AC (2/3)

NO BENEFIT TO EPRIME- IT SHUTS OFF LANGUAGE

FRENCH 1992
[James D, Computer Programmer that writes a lot about linguistics and semantics including articles
published in The Journal of Symbolic Logic, Top Ten Arguments Against Eprime, Et Cetera, http://learn-
gs.org/library/etc/49-2-french.pdf //wyo-pinto]

The range of perfectly acceptable "to be" statements covers

a vast expanse, and includes asymmetrical relations, e.g., "Mt.

McKinley is higher in elevation than Mt . Shasta"; negation,

"The map is not the territory" ; location, "Oakland is on the

west coast" ; auxiliary, "It is raining," "I am going to the store,"

etc.; and possibly many other unidentified forms, e.g., "I am

aware of that ." These forms must be sacrificed when adopting

E-Prime, at considerable cost for no proven benefit .

TO BE KEY TO PROGRESS

FRENCH 1992
[James D, Computer Programmer that writes a lot about linguistics and semantics including articles
published in The Journal of Symbolic Logic, Top Ten Arguments Against Eprime, Et Cetera, http://learn-
gs.org/library/etc/49-2-french.pdf //wyo-pinto]

There may be considerable benefits to humankind in the

use of the verb "to be" that the formulations of general semantics

do not take into consideration . We know that one of

the best languages for time-binding is mathematics, a language

that relies heavily on the notion of equivalence and

equality. "Y = Z" seems quite similar in form to "John Jones

is that professor." Mathematicians do not ascribe content to

Page 810 of 1481


their languages, however, whereas English speakers frequently

confuse language and "reality ." For the purposes of

time-binding and progress, it may be better to keep "to be" in

the language - but cut the link between identity-in-thelanguage

and identification-in-our-reactions (by training ourselves

in general semantics) - rather than to take a meat-axe

to the verb "to be ."

Page 811 of 1481


Page 812 of 1481
EPrime Answers: 2AC (3/3)

EPRIME STOPS THE HUMAN RACE FROM ADVANCING AND SHUTS OFF INDIVIDUAL
AGENCY

FRENCH 1992
[James D, Computer Programmer that writes a lot about linguistics and semantics including articles
published in The Journal of Symbolic Logic, Top Ten Arguments Against Eprime, Et Cetera, http://learn-
gs.org/library/etc/49-2-french.pdf //wyo-pinto]

The phrase "the natural order of evaluation," as a general

semantics formulation, refers to the process of moving from

lower orders of abstraction to higher; from, for example, the

notions of test-taking, attending classes, and reading textbooks,

to the generalized notion of "student ." A civilization

advances when it can move from the idea of individual trees

to that of "forest." Korzybski claimed that the capacity to produce

higher and higher abstractions leads to a general consciousness

of abstracting, which he described as "the very key

to further human evolution ." (Science and Sanity, 3rd ed.,

p.xxi) E-Prime tends to make the expression of higher orders

of abstraction more difficult; instead of describing someone

as a student, for example, the E-Prime speaker is more likely

to say, "She attends classes at the university," or some such

thing. That sort of forced return to lower orders of abstraction

may have drawbacks that the advocates of E-Prime have

not examined . It would seem more in line with the timebinding

of the human race, to leave the individual free to

choose the appropriate order of abstraction in the given case,

rather than to erect a structure that forces him or her to lower

orders. Of course, many individuals do neglect the lower

orders of abstraction in their talking and reacting, but training

in general semantics may be a better prescription for that

Page 813 of 1481


malady than E-Prime .

Page 814 of 1481


Page 815 of 1481
EPrime Bad (Jack Attack!)
EPRIME IS AN INSUFFICIENT ACTIVIST MOVE THE STATIC NATURE OF THE TO BE
VERB IS RECREATED BETWEEN THE LINES

Burroughs '69
[William S., amazing writer and junkie, The Job, 199//uwyo]

Consider now the human voice as a weapon. To what extent can the unaided human voice duplicate effects that can be done with a tape recorder? Learning to speak with the
mouth shut, thus displacing your speech, is fairly easy. You can also learn to speak backwards which is fairly difficult. I have seen people who can repeat what you are saying

after you and finish at the same time. This is a most disconcerting trick, particularly when practiced on a mass scale at a political rally. IS IT POSSIBLE TO
ACTUALLY SCRAMBLE SPEECH? A FAR-REACHING BIOLOGIC WEAPON CAN BE FORGED FROM A NEW LANGUAGE. In fact such a language already
exists. It exists as Chinese, a total language closer to the multi-level structure of experience, with a script derived from hieroglyphs, more closely related to the objects and
areas described. The equanimity of the Chinese is undoubtedly derived from their language being structured for greater sanity. I notice the Chinese, wherever they are, retain

. THE AIM OF THE PROJECT IS TO BUILD A


the written and spoken language, while other immigrant peoples will lose their language in two generations

LANGUAGE IN WHICH CERTAIN FALSIFICATIONS INHERENT IN ALL EXISTING WESTERN LANGUAGES WILL
BE INCAPABLE OF FORMULATION. The follow-falsifications to be deleted from the proposed language.

The IS of identity. You are an animal. You are a body. Now whatever you may be you are not an "animal," you are not a "body," because these are verbal
labels.The IS of identity always carries the implication of that and nothing else, and it also carries the
assignment of permanent condition. To stay that way. All naming callling presupposes the IS of identity. The concept
is unnecessary in a hieroglyphic language like ancient Egyptian and in fact frequently omitted. No need to say the sun IS in the sky, sun in sky suffices. The verb to

be can easily be omitted from any language and the followers of Count Korzybski have done this,
eliminating the verb to be in English. HOWEVER, IT IS DIFFICULT TO TIDY UP THE ENGLISH LANGUAGE BY
ARBITRARY EXCLUSION OF CONCEPTS WHICH REMAIN IN FORCE SO LONG AS THE UNCHANGED
LANGUAGE IS SPOKEN"

Page 816 of 1481


Page 817 of 1481
**Fear Bad**

Page 818 of 1481


A2 Fear of Death Bad: 2AC
(1/5)
1. NO LINK THEIR EVIDENCE IS DESCRIPTIVE OF PEOPLE WHO ARE OBSESSED
WITH DEATH IN THE PAST AND NOT ADVOCACY OF POLICIES THAT ACTUALLY
PREVENT FUTURE VIOLENCE, LIKE THE 1AC

2. NOT COMPETITIVE THE 1AC DOESNT ARGUE, ON FACE, THAT DEATH IS A BAD
THING. THEY ASSUMED THAT FOR THEMSELVES, WHICH PROVES THAT FEAR OF
DEATH IS INEVITABLE

3. GOOD FEAR OF DEATH IS DISTINCT FROM IRRATIONAL FEAR IT ALLOWS US TO


REDUCE DANGER, LIVE ETHICALLY, AND PREPARE FOR A PEACEFUL DEATH ON OUR
OWN TERMS

Gyatso 2003
[Geshe Kelsang, fully accomplished meditation master, internationally renowned spiritual teacher who
can show us from his experience how to begin our spiritual paths, author, Fear of Death, Tharpa
Publications, www.tharpa.com/background/fear-of-death.htm, acc 10-26-04//uwyo-ajl]

Generally, our fear of death is an unhealthy and unrealistic fear-we don't want to die, so we ignore
the subject, deny it, or get morbidly obsessed by it and think that life is meaningless. However, right now
we cannot do anything about dying, so there is no point fearing death itself. What kind of fear is useful?

A healthy fear of death would be the fear of dying unprepared, as this is a fear we can do
something about, a danger we can avert. If we have this realistic fear, this sense of danger, we are
encouraged to prepare for a peaceful and successful death and are also inspired to make the
most of our very precious human life instead of wasting it.

This "sense of danger" inspires us to make preparations so that we are no longer in the danger
we are in now, for example by practicing moral discipline, purifying our negative karma, and
accumulating as much merit, or good karma, as possible.

We put on a seat belt out of a sense of danger of the unseen dangers of traffic on the road, and that seat
belt protects us from going through the windshield. We can do nothing about other traffic, but we can do
something about whether or not we go through the windscreen if someone crashes into us.

Similarly, we can do nothing about the fact of death, but we can seize control over how we prepare
for death and how we die. Eventually, through Tantric spiritual practice, we can even attain a deathless
body.

Page 819 of 1481


Page 820 of 1481
A2 Fear of Death Bad: 2AC
(2/5)
4. EVERY AFFIRMATIVE ETHICAL STANCE REQUIRES A REPRESSED ELEMENT OF
NEGATION, MEANING THAT EVERY AFFIRMATION OF LIFE OCCURS AGAINS THE
BACKGROUND OF FEAR OF DEATH

Zizek '99
[Slavoj, Senior Researcher at Institute for Social Studies, Ljubliana and Badass, The
Ticklish Subject: the absent centre of political ontology, New York: Verso, 1999, 153-
4//uwyo-ajl]

It would therefore be tempting to risk a Badiouian-Pauline reading of the end of psychoanalysis,


determining it as a New Beginning, a symbolic 'rebirth' - the radical restructuring of the
analysand's subjectivity in such a way that the vicious cycle of the superego is suspended, left
behind. Does not Lacan himself provide a number of hints that the end of analysis opens up the
domain of Love beyond Law, using the very Pauline terms to which Badiou refers? Nevertheless,
Lacan's way is not that of St Paul or Badiou: psychoanalysis is not 'psychosynthesis'; it does not
already posit a 'new harmony', a new Truth-Event; it - as it were - merely wipes the slate clean
for one. However, this 'merely' should be put in quotation marks, because it is Lacan's
contention that, in this negative gesture of 'wiping the slate clean', something (a void) is
confronted which is already 'sutured' with the arrival of a new Truth-Event. For Lacan,
negativity, a negative gesture of withdrawal, precedes any positive gesture of enthusiastic
identifiction with a Cause: negativity functions as the condition of (im)possibility of the
enthusiastic identification - that is to say, it lays the ground, opens up space for it, but is
simultaneously obfuscated by it and undermines it. For this reason, Lacan implicitly changes the balance between Death and
Resurrection in favour of Death: what 'Death' stands for at its most radical is not merely the passing of
earthly life, but the 'night of the world', the self-withdrawal, the absolute contraction of
subjectivity, the severing of its links with 'reality' - this is the 'wiping the slate clean' that opens
up the domain of the symbolic New Beginning, of the emergence of the 'New Harmony' sustained by a newly emerged
Master-Signifier. Here, Lacan parts company with St Paul and Badiou: God not only is but always-already was dead - that is to say, after Freud,
one cannot directly have faith in a Truth-Event;
every such Event ultimately remains a semblance obfuscating a
preceding Void whose Freudian name is death drive. So Lacan differs from Badiou in the determination of
the exact status of this domain beyond the rule of the Law. That is to say: like Lacan, Badiou delineates the contours of a
domain beyond the Order of Being, beyond the politics of service des biens, beyond the 'morbid' super ego connection
between Law and its transgressive desire. For Lacan, however, the Freudian topic of the death drive cannot be accounted
for in the terms of this connection:
the 'death drive' is not the outcome of the morbid confusion of Life
and Death caused by the intervention of the symbolic Law. For Lacan, the uncanny domain
beyond the Order of Being is what he calls the domain 'between the two deaths', the pre-
ontologicalf domain of monstrous spectral apparitions, the domain that is 'immortal', yet not in
the Badiouian sense of the immortality of participating in Truth, but in the sense of what Lacan
calls lamella, of the monstrous 'undead' object-libido.18

Page 821 of 1481


Page 822 of 1481
A2 Fear of Death Bad: 2AC
(3/5)
5. FEAR OF DEATH MOTIVATES HEROIC PROTECTION OF OTHERS, ALLOWING US
TO TRANSCEND OUR ISOLATION TOWARDS A LIBERATORY ETHICS OF LOVE AND
JUSTICE FOR THE OPPRESSED

Greenspan 2004
[Miriam, Psychotherapist who understands the Chakras, Excerpt, Healing through the Dark Emotions:
The Wisdom of Grief, Fear, and Despair, May 11, 2004,
www.spiritualityhealth.com/newsh/excerpts/bookreview/excp_5513.html, Acc 10-26-04//uwyo]

"Fear is a very powerful emotion. When you feel fear in your body, it's helpful to relate to it as

an energy that can be mobilized for life. It may feel like a constriction in your chest, throat, or
abdomen. Breathe through it without judgment and allow yourself to feel it as a very strong force. If you
pray for help, you can begin to expand this energy we call 'fear' and use it for healing and
transformation.

"In this regard, we can take our model from the heroes of Flight 93 who. realizing that they were
bound for death, stormed the plane and brought it down without hitting a civilian target. One
cannot even imagine being able to do this without fear. Fear for the lives of others was the energy
that mobilized them to do something meaningful with their last moments of life. Some of these
people said good-bye to their husbands and wives and wished them happiness before they left this earth.
They had found some peace in their last moments, peace in the midst of turbulence. And they found it
through their last wish, which they heroically put into action: to help others live.

"Perhaps there is nothing that can redeem the dead but our own actions for the good. This is a time to
find out what we want to do for the world and do it. And, as every trauma survivor knows, this is the way
to make meaning out of pain, perhaps the most effective way: to draw something good out of evil.
The heroes of September 11 point us to the choice we each have: to help create a state of global
peace and justice that we, like they, will not see before we die. It is in giving ourselves to this
vision, out of love for this world that we inhabit together, that we stand a chance of transcending
the human proclivity to damage life. And that we honor those we have brought into this world and
who must inherit it. . . .

"Our only protection is in our interconnectedness. This has always been the message of the dark emotions
when they are experienced most deeply and widely. Grief is not just "my" grief; it is the grief of every
motherless child, every witness to horror in the world. Despair is not just "my" despair; it is everyone's
despair about life in the twentyfirst century. Fear is not just 'my' fear; it is everyone's fear of
anthrax, of nuclear war, of truck bombs, of airplane hijackings, of things falling apart, blowing up,
sickening and dying.

"If fear is only telling you to save your own skin, there's not much hope for us. But the fact is that in
conscious fear, there is a potentially revolutionary power of compassion and connection that can
be mobilized en masse. This is the power of fear. Our collective fear, which is intelligent, is telling us
now: Find new ways to keep this global village safe. Find new forms of international cooperation
that will root out evil in ways that don't create more victims and more evil. Leap out of the
confines of national egos. Learn the ways of peace. Find a ceremony of safety so that not just you
and I but all of us can live together without fear."

Page 823 of 1481


Page 824 of 1481
A2 Fear of Death Bad: 2AC
(4/5)
6. INACTION LEADS TO POWERLESSNESS AND AN ENDLESS CYLE OF DESPAIR
WORSE THAN FEAR ONLY ACTION AGAINST NUCLEAR VIOLENCE CAN OVERCOME
IT

Sandman & Valenti 86


[Peter M. & Joann M., Scared stiff or Scared into Action, Bulletin of the Atomic Scientists, January
1986, 12-6//uwyo]

The main obstacle to action, writes Frank, is neither apathy nor terror but simply a feeling of
helplessness. To combat it, I have perhaps overemphasized the small signs that antinuclear activities are
at last beginning to influence the political process.(19) Helplessness, hopelessness, futility, and despair
are words one hears even more often than fear from the barely active and the formerly active. And like
fear, these emotions can easily lead to psychic numbing. Those who feel powerless to prevent nuclear war
try not to think about it; and it serves the needs of those who do not wish to think about nuclear
war to feel powerless to prevent it. Messages of hope and empowerment, however, break this
vicious circle.

The label hope, as we use it, subsumes a wide range of overlapping concepts: for example,
optimism, a sense of personal control and efficacy, confidence in methods and solutions, a
sense of moral responsibility, and a vision of the world one is aiming for.

It is well established (and hardly surprising) that hope is closely associated with willingness to
act. Activism appeals most to people who feel positive about both the proposed solution and
their personal contribution to its achievement. Over the long term, this means that antinuclear
organizers must communicate a credible vision of a nuclear-free world. Meanwhile, they must
offer people things to do that seem achievable and worthwhile. The nuclear-weapons-freeze
campaign attracted millions of new activists in 1982 because it offered credible hope. By 1985
many of those millions could no longer ground their hope in the freeze; some found other
approaches and some returned to inactivity.

Most social psychologists today see the relationship between hope and action as independent
of fear or other feelings. For example, Kenneth H. Beck and Arthur Frankel conclude that three
cognitions (not emotions) determine whether people will do something about a health risk:
recognizing the danger as real, believing the recommended plan of action will reduce the
danger, and having confidence in their ability to carry out the plan. (20)

Page 825 of 1481


Page 826 of 1481
A2 Fear of Death Bad: 2AC
(5/5)
7. DENIAL OF FEAR CAUSES POWERLESSNESS THAT MAINTAINS THE PARANOIAC
POSTURES OF NATIONALISM, NUCLEARISM, AND STATE VIOLENCE FEAR OF
DESTRUCTION MOTIVATES US TO OVERCOME THOSE DIVISIONS

Gleisner 83
[John, consultant psychiatrist at the North Western Regional Health Authority in Greater Manchester,
active in the Medical Campaign Against Nuclear Weapons, The enemy within, new internationalist 121,
March, www.newint.org/issue121/enemy.htm, acc 10-26-04//uwyo]

The noted sociologist and psychologist Gregory Bateson drew an analogy between nuclear deterrence and
drug addiction: the fix (new weapon) gives a sense of wellbeing that gradually fades only to require a
bigger fix. What the two also have in common is a powerful dose of denial. Denial of the danger of
nuclear war underlies government thinking on defense. The publics denial may be less strong, but
they are hampered in their understanding by a pervading sense of powerlessness, which in turn
leads to more denial: nuclear war may well happen. But not to me.

Our thinking cannot change without combatting denial and projection the mechanisms of the
psychological war machine. Logical argument in the face of paranoia is as ineffective as with a person in
the grips of a psychotic episode. Emotion is whats needed emotion directed appropriately. Fear
of nuclear war and its effects are legitimate and appropriate and can lead to reappraisal of the old
fear the Russian threat. Another method of penetrating denial is to look for the absurdity in the whole
upside-down logic of the old them-us thought structure.

Confronting denial and projection can be painful, disorienting and can leave one feeling powerless.
But another new belief network is gaining ground. The peace movement is at last building another
way of thinking that can sway governments as countless people are daring to reject the old them-
us psychology. But the question is: can we develop this new way of thinking in time to avert
catastrophe?

8. FEAR OF DEATH IS NECESSARY TO PREVENT GENOCIDE AND EXTINCTION

Beres, PhD at Princeton, 96 (Louis Rene, No Fear, No Trembling Israel, Death and the Meaning of
Anxiety, www.freeman.org/m_online/feb96/beresn.htm)

Fear of death, the ultimate source of anxiety, is essential to human survival. This is true not only for individuals, but
also for states. Without such fear, states will exhibit an incapacity to confront nonbeing that can hasten
their disappearance. So it is today with the State of Israel. Israel suffers acutely from insufficient existential dread. Refusing to tremble before the
growing prospect of collective disintegration - a forseeable prospect connected with both genocide and war - this state is now unable to take the necessary
steps toward collective survival. What is more, because death is the one fact of life which is not relative but absolute, Israel's blithe unawareness of its national
mortality deprives its still living days of essential absoluteness and growth. For states, just as for individuals, confronting death can give the

Page 827 of 1481


most positive reality to life itself. In this respect, a cultivated awareness of nonbeing is central to each state's pattern of potentialities as well as
to its very existence. When a state chooses to block off such an awareness, a choice currently made by the State of Israel, it
loses, possibly forever, the altogether critical benefits of "anxiety." There is, of course, a distinctly ironic resonance to this argument. Anxiety,
after all, is generally taken as a negative, as a liability that cripples rather than enhances life. But anxiety is not something we "have." It is something we (states
and individuals) "are." It is true, to be sure, that anxiety, at the onset of psychosis, can lead individuals to experience literally the threat of self-dissolution, but
this is, by definition, not a problem for states. Anxiety
stems from the awareness that existence can actually be
destroyed, that one can actually become nothing. An ontological characteristic, it has been commonly called Angst, a word related to anguish (which
comes from the Latin angustus, "narrow," which in turn comes from angere, "to choke.") Herein lies the relevant idea of birth trauma as the prototype of all
anxiety, as "pain in narrows" through the "choking" straits of birth. Kierkegaard identified anxiety as "the dizziness of freedom," adding: "Anxiety
is the
reality of freedom as a potentiality before this freedom has materialized." This brings us back to Israel. Both individuals and
states may surrender freedom in the hope of ridding themselves of an unbearable anxiety. Regarding states, such
surrender can lead to a rampant and delirious collectivism which stamps out all political opposition. It can also lead to a national self-
delusion which augments enemy power and hastens catastrophic war. For the Jewish State, a lack of pertinent anxiety, of
the positive aspect of Angst, has already led its people to what is likely an irreversible rendezvous with extinction.

Page 828 of 1481


#3 Good Fear of Death: 1AR (1/2)

PLAN SOLVES BY EMBRACING AN ETHIC OF UNIVERSAL SYMPATHY WE CAN


COME TO TERMS WITH OUR OWN DEATH BY ALLEVIATING THE SUFFERING OF
OTHERS

Schulz, Professor of Philosophy @ Tubigen U, 2K (Walter, Continental Philosophy Review 33, P. 483-485)

It is significant that the death of others is thematized neither by Scheler nor by Heidegger and Sartre. These thinkers begin their analyses of death always from
the self that is in each case mine as an isolated individual. The meaning that the death of others has for me is not regarded by them. The centering of the death
problem in the question of ones own death may be conditioned by the hegemony of the principle of interiority in the epoch of Christian metaphysics, whose
one may not, when one wants to
secularized form is existential philosophy. This is factically a constriction of the problematic. Thus,
comprehend the whole problem of death, look only ahead towards ones own death. However, it is this
shall once more be expressly exhibited just as necessary, to go against the other extreme which confronts us in
modern sociological observations of images of death. Its characteristic is that it thematizes only death,
more exactly: the dying of others. This modern approach blocks so we think from its point of view the complex of questions in an almost
stronger way than the existentialist perspective, insofar as here the fact is excluded along with disregarding his own death that man is a self-understanding
and as such fears death. In opposition to the one-sidedness of both either thematizing only my death or observing only the dying of others one
ought
to treat the phenomenon of death dialectically; that is, to refer to the facts, that man following Kierkegaard is himself and the same
time his species. In The Concept of Anxiety Kierkegaard brought to our attention the meaning of this complex in relation to history. Every individual for himself
takes as his point of departure his history and advances the history of the species which, however, represents its own dimension. This means that the
individual can just as little be released from universal history as the latter can be released from the individual, whereby the individuals history and the history
of the species can exhibit not only different tendencies, but also both make it possible to experience in relation to one another a different evaluation: one can
lose oneself in universal history or over-emphasize ones own singularity.This dialectical approach, which has still in no way been philosophically
estimated in its universal meaning, now says in our context we are here pulling together our argumentation my death as an individual and
death in general, which occurs to the human species, must not be thought without the other . My death appears
to me as the essential, and at the same time I do in fact know that my death is only a special case of death in general. This dialectic, from which a
mediation appears possible between existential introspection and sociological extrospection, becomes first concrete through the
insertion of a mediating determination between my death and death in general. This mediating
determination is the death of other men or women, which, existentially and sociologically regarded, can in fact become
relevant for me in thoroughly different degrees and under the most differentiating respects. None of these three
determinations dyingness in general, the death of others and my death are, however, posited for themselves, rather all of them are to be mediated with
the other. The structure of this mediation shall be made more clear by way of example in the brevity required here. The general determination of dyingness
and transitoriness becomes for me first and foremost tangible and concrete in the death of others. It becomes in no way superfluous through this
concretization. It remains essential as a background determination, and that means it indicates the possibility of my death. The observation of death, more
exactly, the dying of others, is certainly the only real experience of death. But in this extrospection the possible relation to my death comes into play and plays
along always already more or less concealed, because the other and myself are subjugated to the same destiny of dying. Vice versa: the passing into death or
more simply said: the thought, I myself must die, which comes over the aging human being becomes a little more tolerable in dialectically looking away from
myself, that means in view of the universal lot of dying, that itself only appears in stark reality, when we actually see humans dying and observe the uncanny
change from life to death in order to cite an interpretation of Max Schur on Freuds sentence from the work Reflections upon War and Death: Human
beings actually die, not only a few, rather all of them, each and every one of us, when it is his turn. This dialectic - in
which I look away from myself to others or from others to myself, uniting us under the universal lot of transitoriness is no solution to the problem of death,
not even a recipe against the fear of death. But the possibility of a resigned acquiesce that stands opposite both tendencies at work today the
struggle
against violent death over against the help for the dying indicates certainly here that they can be taken
up in their positivity without falling into the illusion that death can be abolished and that the fear of death
is an archaic remnant and in itself irrational. Both these tendencies find their foundation in the thought of
a universal sympathy that binds me to all things living. This sympathy actualizes itself as sympathy, which
means as a return behind selfishness in all its forms. This return is identical with the immediate
recognition that the other is equal to me insofar as he is also a living thing, which must expire and become
nothing. This connectedness between human beings that reveals itself in the light of the common
determinateness of death retains in its ground that is, in the thought of universal transitoriness the
form of negativity. But it also refers to the fact that the individual does not have to stare spellbound at his
own imminent end. Rather if surely also to a small degree only the individual is able to think beyond his
death in view of the task common to everyone, reducing suffering within the world in the face of death.

Page 829 of 1481


Page 830 of 1481
#3 Good Fear of Death: 1AR
(2/2)
HEALTHY FEAR OF DEATH PREVENTS SHORT-TERM WORSE FORMS OF DEATH,
MOTIVATING ACTION FOR A BETTER LIFE

Gyatso 2003
[Geshe Kelsang, fully accomplished meditation master, internationally renowned spiritual teacher who
can show us from his experience how to begin our spiritual paths, author, Fear of Death, Tharpa
Publications, www.tharpa.com/background/fear-of-death.htm, acc 10-26-04//uwyo-ajl]

According to Buddhism, there is unhealthy fear and healthy fear. For example, when we are afraid of
something that cannot actually harm us - such as spiders - or something we can do nothing to avoid -
such as old age or being struck down with smallpox or being run over by a truck - then our fear is
unhealthy, for it serves only to make us unhappy and paralyze our will.

On the other hand, when someone gives up smoking because they are afraid of
developing lung cancer, this is a healthy fear because the danger is real and there are
constructive steps they can take to avoid it. We have many fears-fear of terrorism, fear of
death, fear of being separated from people we love, fear of losing control, fear of commitment,
fear of failure, fear of rejection, fear of losing our job, the list is never-ending! Many of our
present fears are rooted in what Buddha identified as "delusions" - distorted ways of looking at
ourself and the world around us. If we learn to control our mind, and reduce and eventually
eliminate these delusions, the source of all our fear-healthy and unhealthy-is eradicated.
However, right now we need the healthy fear that arises from taking stock of our present
situation so that we can resolve to do something about it. For example, there is no point in
a smoker being scared of dying of lung cancer unless there is something that he or she can or
will do about it, i.e. stop smoking. If a smoker has a sufficient fear of dying of lung cancer,
he or she will take steps to kick the habit. If he prefers to ignore the danger of lung cancer,
he will continue to create the causes of future suffering, living in denial and effectively giving up
control.

Just a smoker is vulnerable to lung cancer due to cigarettes, it is true that at the moment we are
vulnerable to danger and harm, we are vulnerable to ageing, sickness, and eventually death, all
due to our being trapped in samsara the state of uncontrolled existence that is a reflection of
our own uncontrolled minds. We are vulnerable to all the mental and physical pain that arises
from an uncontrolled mind-such as the pains that come from the delusions of attachment,
anger, and ignorance.

We can choose to live in denial of this and thereby give up what control we have, or we
can choose to recognize this vulnerability, recognize that we are in danger, and then find
a way to avert the danger by removing the actual causes of all fear (the equivalent of the
cigarettes) - the delusions and negative, unskillful actions motivated by those delusions. In this
way we gain control, and if we are in control we have no cause for fear. A balanced fear of our
delusions and the suffering to which they inevitably give rise is therefore healthy because it
serves to motivate constructive action to avoid a real danger. We only need fear as an
impetus until we have removed the causes of our vulnerability through finding spiritual,
inner refuge and gradually training the mind.

Page 831 of 1481


#4 Repression Turn: 1AR (1/3)

EXTEND THE ZIZEK 99 CARD. THERES NO SUCH THING AS A PURE AFFIRMATION


OF LIFE. EVERY TIME YOU SAY THAT SOMETHINGS GOOD, BETWEEN THE LINES
YOURE SAYING THAT SOMETHING ELSE, LIKE DEATH, IS BAD BECAUSE THE
UNCONSCIOUS STAIN OF YOUR OWN MORTALITY IS IN THE BACKGROUND OF
EVERY ACTION THAT YOU TAKE, WHETHER ITS EXPLICT OR NOT. THEIR YES TO
LIFE IS AN IMPLICIT NO TO THE SAME DEATH THAT WERE SAYING IS BAD. FEAR OF
FINITUDE IS STILL CONTAINED IN ALL OF THEIR ARGUMENTS, REPRESSED BENEATH
THE SURFACE OF THEIR WORDS, MEANING THAT IT EMERGES IN EVEN WORSE
FORMS THAT YOU CANT INTERROGATE BECAUSE YOUVE MADE THEM INVISIBLE,
TURNING THE CRITICISM.

THIS MEANS WELL WIN THE UNIQUENESS FOR OUR TURNS BECAUSE SOME FORM
OF ANXIETY FROM DEATH IS INEVITABLE IN ALL DISCOURSE, THE ONLY QUESTION
IS OF WHETHER THOSE REPRESENTATIONS INTERROGATE THE FUNDAMENTAL
FANTASY OF POLITICAL REALITY BY ACKNOWLEDING OUR INEVITABLE
RELATIONSHIP TO THE TRAUMA OF DEATH THATS INHERENTLY REPRESSED BY THE
SYMBOLIC. THERES ONLY A RISK OF THE ALTERNATIVE MASKING FEARS
INFLUENCE.

Page 832 of 1481


Page 833 of 1481
#4 Repression Turn: 1AR (2/3)
THEORETICAL DISTANCE FROM OUR OWN LIKELIHOOD OF DEATH IS AN ATTEMPT
TO PHANTASMICALLY ERASE THE ANXIETY THAT IT CAUSES, MASKING HOW IT
AFFECTS OUR DAILY ACTIVITY IN COVERT WAYS

Park 2001
[James, Philosopher, Loneliness, Depression, Anxiety & Death, 4th ed.,
www.tc.umn.edu/~parkx032/XP181.html, acc 10-26-04//uwyo-ajl]

The 'fear of death' is a composite experience encompassing:


(1) the abstract, objective, external, empirical fact of biological death;
(2) our personal, subjective, emotional fear of ceasing-to-be,
which arises from our awareness of our own finitude, and
(3) our ownmost ontological anxiety,
our Existential Predicament disguised as the fear of ceasing-to-be.
This least understood and most repressed existential dimension of death,
which has also been called "being-towards-death"
and "the anxiety-of-nonbeing",
will be the central focus of this phenomenological investigation.

Whenever "death" is mentioned, we think first of biological death,


but this tendency to focus exclusively on the objective, terminal fact of dying
may well be a trick of thought designed to protect us
from noticing our fear of ceasing-to-be or our even deeper ontological anxiety.

We have other protective techniques as well:


religious illusions, philosophical desensitization, and diversionary small-talk.
Most of these distracting ploys amount to seeing death exclusively
as an objective event, which befalls all plants, animals, and people eventually.
All such attempts to picture and talk about death as a fact
are (at least in part) attempts to evade the two deeper dimensions of death
by interpreting death only from the point of view of a spectator.
Even our scholarly symposia about death
are often designed to provide an objective understanding of death.
Such approaches keep death outside of ourselves
a phenomenon we know about only as observers, never as participants.

Page 834 of 1481


Page 835 of 1481
#4 Repression Turn: 1AR (3/3)
ONLY DIRECTLY ADDRESSING THE ANXIETY PROVOKED BY FINITUDE BY OPENLY
ACKNOWLEDGING ITS INFLUENCE CAN TRAVERSE THAT MASKING PROCESS,
ALLOWING US TO COME TO TERMS WITH DEATHS HOLD OVER US

Park 2001
[James, Philosopher, Loneliness, Depression, Anxiety & Death, 4th ed.,
www.tc.umn.edu/~parkx032/XP181.html, acc 10-26-04//uwyo-ajl]

But it will be more difficult to separate the deeper dimensions of death: our terrifying fear
of ceasing-to-be and our underlying ontological anxiety. If we probe even below our
personal fear of ceasing-to-be-in-the-world, we may discover the cause of much of our evasive
talk and deceptive posturing; we may pull the covers off our trembling, naked ontological
anxiety. If we find ways to look deeply into ourselves, exposing even our most clever
tricks of thought, then not only will we begin truly to fear our own deaths, but we may
even confront our underlying ontological anxiety.

This ontological anxiety is obscurely felt by all of us as a subjective awareness drifting up


from our inner depths, a pervasive haunting of our whole being, which we are reluctant to
confront because we have no easy way to handle it. This continuous inner state-of-being is
not the result of the fact of dying; it is not worry arising from the inevitability of actual death.
Rather, our ontological anxiety is the deepest truth of our existence, obviously deeper than the
external, objective, empirical fact of biological death, but even deeper than our inward,
subjective, personal fear of ceasing-to-be. Our ontological anxiety does not arise from the fact
of death, but much of our concern about death arises from our ontological anxiety! (This
paradoxical statement should become clear in the next 70 pages.)

If our ontological anxiety truly grips us, we can go either of two possible ways: (1) We can
organize our lives around this all-pervasive 'threat', courageously embracing our ontological
anxiety, moving ourselves toward "Authentic Existence". Or (2) we can be freed from our
ontological anxiety after having fully acknowledged it (and attained some Authenticity),
thereby coming into the new inner state-of-being "Existential Freedom".

Page 836 of 1481


#5 Fear is Key to Love: 1AR

LOVE AND FEAR ARE COMPATIBLE FEAR IS NECESSARY TO PROTECT LOVED ONES

Sandman and Valenti, Prof of Human Ecology @ Rutgers and Preeminent Risk Communications Expert,
86 (Peter and JoAnn, Scared stiff or scared into action , Bulletin of the Atomic Scientists, January 1986, P. 1216,
http://www.psandman.com/articles/scarstif.htm)

Love Anger without love soon becomes sterile or uncontrolled, while love without anger can still inspire a movement. But there is no need to choose; love and
anger are compatible. Nothing could better symbolize anger than the powerful bolt-cutters with which the Greenham Common women routinely destroy the
fence surrounding the cruise missile site. Nothing could better symbolize love than the webs of twine and ribbon and memorabilia with which they decorate
the same fence. We suspect it is this combination the anger not rancid, the love not languid that has captured the imaginations of peace activists around
the world.Love is compatible with fear as well. As we suggested earlier, some evidence indicates that people are more affected
by fear appeals targeted at their loved ones than by those aimed at themselves. Ironically, one of the
classic studies from the early 1960s tried to persuade citizens to support community fallout shelters;
strong fear appeals threatening family safety worked better than threats to the individual.(17) But love is not
compatible with psychic numbing. Just as numbness interferes with the ability to love freely, so active love drives
away the numbness. Antinuclear activists almost universally report that they remain active less for themselves than for those they love, and that
without love they could not stay with the fight. This is not to suggest that these activists are more loving than their neighbors, only that their love helps them
stay active and that their activism is a powerful expression of love. It is relevant that the children of activists are far more confident of their futures than most
Just as activists rely on love to keep them going, one can mobilize the uninvolved by talking
children.(18)
about the people, places, and values one holds dear and encouraging listeners to do the same. Something
or someone to fight for is as indispensable to activism as something or someone to fight against.

Page 837 of 1481


#6 Inaction Turn: 1AR

POLICY ACTION AGAINST NUCLEAR WAR SPURS RADICAL CHANGE TOWARDS A


POLITICS OF PEACE AND LOVE

Sandman & Valenti 86


[Peter M. & Joann M., Scared stiff or Scared into Action, Bulletin of the Atomic Scientists, January
1986, 12-6//uwyo]

While people are most likely to take action against nuclear war when they feel angry, loving, and hopeful
rather than terrorized into numbness, it is also true that action against nuclear war tends to liberate
peoples anger, love, and hope. The growth of commitment is circular, in other words, with feelings,
understandings, and behaviors alternating in complex patterns; action, however, is likely to begin the
process.

The notion that behavior is as much a cause as a result of feeling, attitude, and knowledge is
commonplace among clinicians, who often urge clients to try new behaviors as a way of breaking patterns
and opening a path to new understanding. It is familiar ground also for social psychologists and provides
the foundation for one of psychologys most robust persuasion models, Leon Festingers theory of
cognitive dissonance, whereby behavior triggers an effort to regain consistency by finding information
and building attitudes to support the behavior itself. (23)

This theory makes sense of what petition-circulators have universally observed: that people are more
likely to read the literature they are offered after signing than before. If before signing they experience
the literature as an unwelcome prod, after signing (out of politeness, perhaps) they need the literature to
justify their new behavior.

The lesson for the antinuclear movement is clear: Any experience such as signing petitions,
wearing buttons, or going to rallies however partial or even irrelevant its motivation can
provide a reason to consider the issues more deeply, and this consideration can launch a cycle
of incrementally increasing commitments to peace.

Page 838 of 1481


#7 Fear Solves War: 1AR

FEAR OF NUCLEAR WAR IS CRITICAL TO PREVENTING ALL OUT STATE VIOLENCE


AND WAR

Futterman 94
[J.A.H., fortysomething male, enjoys classical, folk, bluegrass, jazz, some rock, &
some rap, self-identifies as a Lutheran Jewpiscopalian, and watches Iron Chef,
Obscenity and Peace: Meditations on the Bomb, Virtual Church of the Blind
Chihuahua, www.dogchurch.org/scriptorium/nuke.html, acc. 10-26-04//uwyo]

I could say that if I didn't do it, someone else would, but that answer was rejected at
Nuremberg. (It's also a better reason to leave the weapons program than to stay.) I continue to
support the nuclear weapons business with my effort for many reasons, which I discuss
throughout this piece. But mostly, I do it because the fear of nuclear holocaust is the only
authority my own country or any other has respected so far when it comes to
nationalistic urges to make unlimited war. As William L. Shirer states in his preface to The
Rise and Fall of the Third Reich (Touchstone Books, New York, 1990),

"Adolf Hitler is probably the last of the great adventurer-conquerors in the tradition of
Alexander, Caesar, and Napoleon, and the Third Reich the last of the empires which set out on
the path taken earlier by France, Rome and Macedonia. The curtain was rung down on that
phase of history, at least, by the sudden invention of the hydrogen bomb, of the ballistic missile,
and of rockets which can be aimed to hit the moon."

Now this contrasts with the argument of those who would "reinvent government" by putting up
bureaucratic roadblocks to maintaining the reliability of the US nuclear arsenal through research
and testing. They reason that if the reliability of everyone's nuclear arsenals declines, everyone
will be less likely to try using them. The problem is that some "adventurer-conqueror" may arise
and use everyone's doubt about their arsenals to risk massive conventional war instead. An
expansionist dictatorship might even risk nuclear war with weapons that are simpler, cruder,
less powerful, much riskier (in terms of the possibility of accidental detonation) but much more
reliable than our own may eventually become without adequate "stockpile stewardship."[14]

But the inhibitory effect of reliable nuclear weapons goes deeper than Shirer's deterrence of
adventurer-conquerors. It changes the way we think individually and culturally, preparing
us for a future we cannot now imagine. Jungian psychiatrist Anthony J. Stevens states, [15]

"History would indicate that people cannot rise above their narrow sectarian concerns
without some overwhelming paroxysm. It took the War of Independence and the Civil War to
forge the United States, World War I to create the League of Nations, World War II to create the
United Nations Organization and the European Economic Community. Only catastrophe, it
seems, forces people to take the wider view.

Or what about fear? Can the horror which we all experience when we contemplate the
possibility of nuclear extinction mobilize in us sufficient libidinal energy to resist the archetypes
of war? Certainly, the moment we become blas about the possibility of holocaust we are
lost. As long as horror of nuclear exchange remains uppermost we can recognize that
nothing is worth it. War becomes the impossible option. Perhaps horror, the experience of
horror, the consciousness of horror, is our only hope. Perhaps horror alone will enable
us to overcome the otherwise invincible attraction of war."

Page 839 of 1481


Thus I also continue engaging in nuclear weapons work to help fire that world-historical warning
shot I mentioned above, namely, that as our beneficial technologies become more powerful, so
will our weapons technologies, unless genuine peace precludes it. We must build a future more
peaceful than our past, if we are to have a future at all, with or without nuclear weapons a
fact we had better learn before worse things than nuclear weapons are invented. If you're a
philosopher, this means that I regard the nature of humankind as mutable rather than fixed, but
that I think most people welcome change in their personalities and cultures with all the
enthusiasm that they welcome death thus, the fear of nuclear annihilation of ourselves
and all our values may be what we require in order to become peaceful enough to
survive our future technological breakthroughs.[16]

Page 840 of 1481


Page 841 of 1481
Spectacle of Death Good (1/4)
DEATH IMAGERY AFFIRMS LIFE AND OPENS UP THE POSSIBILITY OF CHANGE

Fox, Philosophy Professor at Queens, 85 (Michael Allen, Nuclear War: Philosophical Perspectives, ed. Fox and
Groarke, p. 127)

Security is not the absence of


There remains but one choice: we must seek a reduction of world tensions, mutual trust, disarmament, and peace.35

fear and anxiety, but a degree of stress and uncertainty with which we can cope and remain

mentally healthy. For security, understood in this way, to become a feature of our lives, we must

admit our nuclear fear and anxiety and identify the mechanisms that dull or mask our
emotional and other responses. It is necessary to realize that we cannot entrust security to ourselves, but, strange as it seems
and however difficult to accept, must entrust it to our adversary Just as the safety and security of each of us, as individuals, depends upon the good will of
every other, any one of whom could harm us at any moment, so the security of nations finally depends upon the good will of other nations, whether or not we

The disease for which we must find the cure also requires that we
willingly accept this fact.

continually come face to face with the unthinkable in image and thought and recoil from it. 36 In this
manner we can break its hold over us and free ourselves to begin new initiatives. As Robert J. Lifton points out, confronting massive

death helps us bring ourselves more in touch with what we care most about in
life. We [will then] find ourselves in no way on a death trip, but rather
responding to a call for personal and professional actions and commitments on
behalf of that wondrous and fragile entity we know as human life.

Page 842 of 1481


Page 843 of 1481
Spectacle of Death Good (2/4)
DEATH IS OMNIPRESENT. THE SPECTACLE IS THE ONLY WAY ONE CAN EVEN BEGIN
TO COME TO TERMS WITH DEATH

Frank Gonzalez-Crussi, M.D. Pathologist, Children's Memorial Hospital, Confronting the Margin, June 2,
2000, http://seeingthedifference.berkeley.edu/gonz-crussi.html, accessed 1/23/03

What one may actually perceive in the spectacle of death depends on the individual. Seeing is invariably in
the eye of the beholder. I shall briefly refer to two styles of seeing that I may call "culturally dependent,"
for lack of a better term. In the traditional Mexican culture, which is permeated by a strong current of
Indian naturalism, death is something very concrete. I am not an anthropologist, but this much I can say
from my subjective impressions during my youth: death in Mexico is always embodied. Death is this
cadaver, right here. It is something that may be palpated, touched, weighed, turned around. I was always
impressed by the directness with which the survivors addressed the cadaver during a funeral ceremony in
the lower socio-economic strata. There is much display of emotion, and the bereaved talk to the
deceased. It is a new form of relationship. The survivors speak to the dead person: they reproach him for
having left this world; they remind him of the joys and sorrows that they shared together; they make
confessions, grant absolutions or admit having wronged him; and they promise him that they will
remember him forever. They talk to him, not at him. I am sure that, if these addresses were only
monologues, they were the kind of monologue that absolutely required the presence of the cadaver as
mediator of the monologizing. The present-absent is much more present than absent, if I may thus
express this unique status. In other words, the corporeal reality of the departed is strongly felt. It is a
powerful sign that propitiates the illusion that the dead are still with us. Death is primarily a presence.
When the dead are deprived of their corporal wrappings--the flesh, the nerves, the arteries (by now
utterly superfluous)--there remains the skeleton. The skeleton is the almost universal emblem of death.
But because it is eminently tangible and concrete--solid, stone-like--it has had a great career in Mexico. In
the Mexican culture, the symbolic skeleton, the calavera, is not only felt, palpated, and even played with,
but is also tasted, in the form of the sugar skulls that are consumed on All Souls Day, the day of the dead.
For it is not only recent death that has a presence. Death is recurrently present, eminently present in the
mind, at least on All Souls Day, the Dia de los Muertos, the "Day of the Dead," year after year. It is
otherwise for cultures in which death is primarily an absence or a disappearance. In one philosophical
tradition of Anglo-American culture, the living person is easily destroyed. Recall that John Locke says that
personal identity is "inseparable from thinking," a mere consciousness displaying unity across time. And
David Hume saw the person as "a train of perceptions" glued together by certain relations. Consciousness
must attach itself to an animal body, or, as we say today more specifically, to a functioning brain. But
body and brain were secondary, and in a sense irrelevant. Consciousness alone conferred identity.
Consciousness alone embodied the essence of personhood. But if the person is merely a precarious
bundle of mental activities, the dead person must be flimsier yet. The "Great Iceberg of Cotton Wool" of
which Henri Michaux speaks in one of his poems, can erase all traces of the person. Death thus becomes
an erasure, that is to say a disappearance, an absence, or a mere attribute of the insubstantial mind, of
the fleeting consciousness, like the person itself. Not a concrete osseous framework--as is the Mexican
skeleton, the calavera--but a wholly immaterial entity. Defined as an absence, it absented itself. Because
it could not be seen, it ended up suffering the fate announced in the popular saying "Out of sight, out of
mind." It was proscribed, and it became the Unmentionable. To finish these comments, I wish to say that
I believe there is a parallel between the death-related Mexican naturalism, and--strange to recount--
certain ideas that I have found in the pages of Russian novelists. It has been remarked that Tolstoy never
approached death as a philosophical problem. He never seems to be looking for comprehensive concepts,
conclusions, or intellectual approaches to death. He is not striving to create a philosophy of death; he is
merely describing the experience of living beings. Since death cannot be understood, conceptualized,
reduced to system, or dealt with syllogistically, the only thing left is to look at it. Such is the gist of the

Page 844 of 1481


Mexican attitude. The gaze will not penetrate to the essence of the problem. It will barely skim its surface,
but that is all we can do. And this is what Tolstoy does: to describe tirelessly, to evoke every detail of the
external corporeality of death, to all the minutiae. Read the last pages of Ivan Ilyich, to find there a
recreation of every sensory impression, the sounds, the sights, the odors that impressed a child. Vladimir
Jankelevitch points out that this is one constant throughout the whole Tolstoyan work. He revels in the
details, in the concrete particularities.

Page 845 of 1481


Page 846 of 1481
Spectacle of Death Good (3/4)
THE SPECTACLE OF DEATH IS EMBEDDED IN AMERICAN CONSCIOUSNESS. THE
ONLY QUESTION IS WHETHER DEATH CAN BE LEVERAGED TO FORCE RECOGNITION
OF OUR COMPLICITY AS IN VIETNAM, OR IF IT IS VIRTUALIZED AND USED TO
JUSTIFY DOMINANCE. SPEAKING DEATH AGAINST FURTHER NUCLEAR VIOLENCE
ROBS IT OF ITS ABILITY TO PACIFY

Walter Davis, Death's Dream Kingdom: The American Psyche After 9-11, July 29, 2002,
http://goinside.com/02/7/death.html, accessed 1/23/03

For if we found ourselves abject objects of the others wrath at Pearl Harbor we now had a way to bring
about a complete and lasting transformation of that situation. Projective identification finds in the Bomb
a way to take everything weak and vulnerable in oneself and invest it in an other who is reduced to an
object of contempt and obliteration. The resulting mania banishes any threat of a return of depressive
anxieties. In the Bomb the manic triadtriumph, contempt, and dismissal (Klein, 1957) celebrates its
Sabbath. Metapsychologically, the transformation is complete and can be schematized thus: abjection
reversed; blockage overcome; aggression unbound. Narcissistic grandiosity thereby finds the fullest
possible expansion; the perfect phallic mirror in the mushroom cloud rising above the spectacle as proof
of the Bombs power to compel submission to its will. Evacuation attains an exorcism of an
unprecedented ordera psychotic attack on linking (Bion 1959) that is totalizing in its scope and that
scoffs at all humanistic considerations. Thanatos in the bomb achieves the condition Freud feared: a
condition in which death has been fully eroticized. Pleasureor jouissanceunder the Bomb equals
releasing a destructiveness that voids all inner tensions in an aggression that has the blessing of the
super-ego, an aggression that feels righteous. As confirmation consider this, but one example among
many: Navy Day, October, 1945, a crowd of 120,000 gather in the Los Angeles Coliseum to celebrate a
simulated reenactment of the Bombing of Hiroshima, complete with a mushroom cloud that rises from
the fifty yard line to the joyful cheers of that rapt throng (Boyer, 1985). The first Super-Bowl. The society
of the spectacle (Debord,1994) here announces its truth as a mass audience cums to the ritual that
confers on it a lasting, ghostly identity: the howl of joy that rises as a hymn of praise to the burgeoning
cloud is the new American collectivity in Hosanna before the image of its inhumanity as it blossoms
before them, big with the future. A History Lesson From which follows a quick tour of the underside of
American history from 1945 to the present. The debacle of Vietnam. The error: the image came home to
roost. With the evening news America each night supped full with horror. The lesson: no more images.
The solution: Iraq, the Nintendo war, a war represented on TV as a video game. No images of the
100,000 Iraqui dead entered the American conscience to trouble our sleep. Instead, with victory the
proclamation of George H. Bush : Weve finally put an end to Vietnam syndrome. The lesson of
history learned the son now deploys it globally in a war where, he informs us, much will happen that we
will never get to hear about or see. Extremes meet: the image is banished but the promise of global
action is affirmed. George W. Bush is an apt pupil. He knows that in order to resolve the trauma of 9-11
he must satisfy an outraged public by finding a way to repeat the psychological operations perfected in
Hiroshima. He knows that nothing less than a global war against terrorism will suffice. But he also
knows that the pleasure of the image must be replaced by another kind of satisfaction, one appropriate
to the information age, an age in which pleasure has itself become virtual. Subjects formed by what is
today perhaps the primary relationship, the relationship to the computer, dance to the subtext, heeding
the command to enjoy our symptom (Zizek,1989). For it is now possible to imagine and experience
scorched-earths as so many blips on a computer screen with disavowal already in place and pleasure
assured in a jouissance that is one with Thanatos: the reduction of the human to the statistical and the
boundless power one feels in manipulating, at the speed of light, a world so rendered into ones hands.
The society of the spectaclea society that needed Hiroshima and Navy Day in the L.A. Coliseumis

Page 847 of 1481


replaced by the society of the virtual. The post-modern subject has entered a condition of bliss, the
hegemony of Thanatos assured by the sacrifice of the image. Mass carnage grows apace: over a million
Iraqui civilians have now died as a result of our sanctions; more civilians (collateral damage) have now
died in Afghanistan as a result of our bombings than perished at the WTC. But the knowledge of these
things has become virtual, disembodied, imageless and thus is already fading, leaving no residue in the
national consciousness.

Page 848 of 1481


Page 849 of 1481
Spectacle of Death Good (4/4)
THEIR EVIDENCE DESCRIBES THE WAY DEATH IS PERCEIVED IN THE STATUS QUO.
WE CALL THAT INTO QUESTION BECAUSE IT WEDS US TO THE PSYCHE THAT
DROPPED THE BOMB. REFUSING TO ENGAGE THE IMAGE OF VIOLENCE
WROUGHT BY THE BUSH DOCTRINE LETS AMERICA GET AWAY WITH
EXPLOITATION.

Walter Davis, Death's Dream Kingdom: The American Psyche After 9-11, July 29, 2002,
http://goinside.com/02/7/death.html, accessed 1/23/03

What, then, are the possibilities of healing and renewal that we can derive from an awareness of the
tragic complexities of 9-11 and its aftermath? A responsible reply must begin with the recognition that it
was through us that terror on a global scale first came into the world; and that we remain its primary
global practitioner. For an internalization of that fact delivers a death-blow to the belief that catharsis
and renewal require the reassertion of adolescent myths about ourselves and our place in history.
Historical memory must become instead the process of creating a tragic culture: one for whom memory
is conscience and not hagiography; one for whom the past weighs like a nightmare precisely because it
has not been constituted. That is the true meaning of Hiroshima. Ground-zero haunts us not because
we feel guilt about it but because we dont. Which is why, whenever we are traumatized, we repeat the
psychological operations we perfected in Hiroshima in a progressive self-reification that we remain
powerless to reverse as long as we refuse to internalize what actually happened on 8-6-45. But to do that
we must begin the long, hard task of rooting out everything in our culture that weds us to the psyche
that dropped the bomb. Such an effort requires, moreover, that we free ourselves from our own liberal,
mental health myth: the belief, articulated by Lifton (Lifton and Mitchell, 1995) that admitting error
assures renewal through the power of the American protean self to reclaim the ideals that make
American history the story of inevitable progress. What Hiroshima teaches us, on the contrary, is that
history remains irreversible in its tragic consequences until we find our own equivalent of Gandhis ethic:
that the way out of hell is one that sustains trauma and depressive mourning as the destiny of historical
subjects who know that reversal begins only when we are willing to plumb the depths of our collective
disorder. A tragic understanding of history assures us no catharsis, no renewal, no guarantees. What it
offers instead is the realization that to sustain and deepen the trauma is our only hope. (7) For the
alternative is truly horrifying: the Bush doctrine a blank check for whatever carnage will be needed to
satisfy our blood-lust and to preserve our right to ravage the planets resources. Because one fact
above all others is, as Marx would say, determinative in the last instance of what is going on in the
world today. 5% of the worlds population consume 25% of its resourcesand they do so by exerting
control over the destiny of other countries. Bin Laden is a symptom, a nostalgic religious fanatic, but his
fanaticism derives from a condition that is actual. In Rio de Janeiro, at the one ecological conference he
attended, George H. Bush delivered a proclamation even more chilling than his crowing about Vietnam
syndrome: The American way of life is not negotiable. As long as that dogma remains in place there
will be many more ground-zeroes.

Page 850 of 1481


Page 851 of 1481
**Empire**

Page 852 of 1481


Movements Fail
HARDT AND NEGRI PROVIDE NO MECHANISM FOR THE CREATION OF SUCCESSFUL
SOCIAL MOVEMENTS THE FACT THAT CLASS OPPRESSION ALREADY EXISTS AND
HAS BEEN GETTING WORSE FOR HUNDREDS OF YEARS MEANS THAT IT EITHER
SHOULD HAVE HAPPENED BY NOW OR IT WONT HAPPEN

Cox, Prof of Sociology @ National U of Ireland, 2K1 (Lawrence, Social Movements and Empire, Rethinking
Marxism Vol. 13, No. 3-4)

Page 853 of 1481


Page 854 of 1481
Alternative Causes Violence
ACTUAL ENDORSEMENT OF THEIR ALTERNATIVE WOULD CAUSE A VIOLENT
BACKLASH PROVES THAT THEIR CALL IS HOLLOW

Zizek, Fellow @ Institute for Sociology @ Ljubljana, 2K1 (Slavoj, Have Michael Hardt and Antonio Negri Rewritten
the Communist Manifesto for the 21st Century, Rethinking Marxism, Vol. 13 No. 3-4)

Page 855 of 1481


Page 856 of 1481
Alternative is False Radicalism
THEIR IMPOSSIBLE DEMAND IS MADE IN ORDER TO RETAIN THE SEMBLANCE OF
RADICALISM WITHOUT ACTUALLY HAVING TO RISK TAKING RADICAL ACTIONS

Zizek, Fellow @ Institute for Sociology @ Ljubljana, 2K1 (Slavoj, Repeating Lenin,
http://www.lacan.com/replenin.htm)

My personal experience is that practically all of the "radical" academics silently count on the long-term
stability of the American capitalist model, with the secure tenured position as their ultimate professional
goal (a surprising number of them even play on the stock market). If there is a thing they are genuinely horrified of, it is a
radical shattering of the (relatively) safe life environment of the "symbolic classes" in the developed
Western societies. Their excessive Politically Correct zeal when dealing with sexism, racism, Third World
sweatshops, etc., is thus ultimately a defense against their own innermost identification, a kind of compulsive
ritual whose hidden logic is: "Let's talk as much as possible about the necessity of a radical change to
make it sure that nothing will really change!" Symptomatic is here the journal October: when you ask one of the editors to what the title
refers, they will half-confidentially signal that it is, of course, THAT October - in this way, one can indulge in the jargonistic analyses
of the modern art, with the hidden assurance that one is somehow retaining the link with the radical
revolutionary past... With regard to this radical chic, the first gesture towards the Third Way ideologists and practitioners should be that of praise:
they at least play their game in a straight way, and are honest in their acceptance of the global capitalist coordinates, in contrast to the pseudo-radical
academic Leftists who adopt towards the Third Way the attitude of utter disdain, while their own radicality
ultimately amounts to an empty gesture which obliges no one to anything determinate.

Page 857 of 1481


Page 858 of 1481
Capitalism is Sustainable
HARDT AND NEGRIS ASSUMPTIONS ABOUT THE INSTABILITY OF CAPITALISM ARE
WRONG THE KRITIK WILL FAIL

Kimball, Managing Editor of New Critierion, 2K1 (Roger, The new anti-Americanism, The New
Critierion, Vol. 20, No. 2, October, http://www.newcriterion.com/archive/20/oct01/empire.htm)

I suspect that part of the reason Empire is such a hit in the academy is its superior insulation. Hardt and Negri have sealed every point of ingress: no hint of
The single greatest embarrassment to Marxist theory has always been the longevity of
reality is allowed to seep in.
capitalism. It was supposed to implode from internal contradictions long ago. But here it is 2001 and
capitalism is still going strong and making the world richer and richer. Attempting to explain this is the greatest test of a Marxists ingenuity.
Here is how Hardt and Negri handle the problem: As we write this book and the twentieth century draws to a close, capitalism is miraculously healthy, its
accumulation more robust than ever. How can we reconcile this fact with the careful analyses of numerous Marxist authors at the beginning of the century
who point to the imperialist conflicts as symptoms of an impending ecological disaster running up against the limits of nature? They
offer three
hypotheses for this imponderable situation. One, that capitalism has reformed itself and so is no longer in
danger of collapse (an option they dismiss out of hand). Two, that the Marxist theory is right except for
the timetable: Sooner or later the once abundant resources of nature will run out. Threewell, it is a little difficult to say what the third hypothesis
is. It has to do, they say, with the idea that capitalisms expansion is internal rather than external, that it
subsumes not the noncapitalist environment but its own capitalist terrain that is, that the subsumption is no longer
formal but real. I wont attempt to explain this for the simple reason that I havent a clue about what it means. Is there any important option
they have neglected? Could it, just possibly, be that the careful analyses of numerous Marxist authors was
just plain wrong? This is a possibility apparently too awful to contemplate, for Hardt and Negri never raise it.

THE ALT FAILS EMPIRE WILL NOT OVERSHOOT AND CAPITALISM WILL NOT
COLLAPSE ON ITSELF

Kimball, Managing Editor of New Critierion, 2K1 (Roger, The new anti-Americanism, The New Critierion, Vol. 20,
No. 2, October, http://www.newcriterion.com/archive/20/oct01/empire.htm)

Empire is based on a laughably tiny


Eakins is also wrong to suggest that Empire may represent the Next Big Idea. This is mainly because
idea, and one that is also old and wrong. The idea, again, is Marxs idea about the inevitable collapse of
capitalism. It seemed big once upon a time. It is now as thoroughly discredited as an historical or political idea can
be. Hardt and Negri gussy up Marx with a formidable panoply of New Age rhetoric about globalization.
But the creaking you hear as you make your way through the book is the rusty grinding of the dialectic: it
goes nowhere, it means nothing, but it keeps creaking along.

Page 859 of 1481


Page 860 of 1481
Resistance Fails
RESISTANCE FROM THE MULTITUDES WILL FAIL 9/11 PROVES THAT ACTS OF
RUPTURE WILL BE RECUPERATED

Passavant and Dean, Assoc Profs of Political Science @ Hobart and William College, 2K2 (Paul and Jodi,
Representation and the Event, Theory and Event, Vol. 5, No. 4)

Page 861 of 1481


Page 862 of 1481
Alternative = Oppression
CALLS FOR UNITY EXCLUDE MARGINALIZED POSITIONS HARDT AND NEGRIS
VISION OF THE MULTITUDE WILL OPPRESS AND IGNORE DISADVANTAGED VOICES
WE SHOULD FIGHT CAPITALISM FROM THE INSIDE

Rofel, Prof of Anthropology @ UC Santa Cruz, 2K1 (Lisa, Discrepant Modernities and Their Discontents, Positions:
East Asia Cultures Critique, Vol. 9, No. 3, Project Muse)

Why must we be forced into a dream of unity? Why can we not dream of flexible alliances and articulations? On one level
Hardt and Negri would certainly agree. Their vision of rhizomatic politics inspired by Deleuze and Guattari leaves room for a wide variety of
alliances. Yet I find their dream of a common language frightening. Who will establish the proper grammar of
this language? Who will set the communicative import of terms? What of those who wish to speak in
multiple tongues? They traipse over the issue of translation as if it were merely a pragmatic dilemma
rather than, as many scholars have shown, a question of power. For those who live on the sexual margins, for example,
the dream of the multitude brings not hope but fear. What reassurances do Hardt and Negri offer that the
recent history of degraded existence for those forced out of the multitude in the name of sexual
respectability will not be repeated in their version of unity? Can we not dream of fighting capitalism
through articulations and alliances of variously identified subjects? Can we not dream of fighting
capitalism in the manner, for example, of those who have fought AIDS? AIDS activism has addressed the
mutual imbrication of power in the endless relays between expert discourse and institutional authority,
between medical truth and social regulation, and between popular knowledge practices and struggles for
survival. AIDS activism has thus multiplied the sites of political contestation to include immigration policy,
public health policy, the practice of epidemiology and clinical medicine, the conduct of scientific research, the operation of the insurance and
pharmaceutical industries, the role of the media, the decisions of rent-control boards, the legal definition of family, and ultimately the public
and private administration of the body.17 It is unsettling that Hardt and Negri do not discuss these politics. Why must they
dismiss them as merely about co-optation? Hardt and Negri have missed the enormous body of work that
has shown that we do not have to pit class against other identities but, rather, can conceive of class in a
manner that does not implicitly make the class subject a white, masculine, Euro-American subject. If
bodies do matter, then Hardt and Negri still have a long way to go.

Page 863 of 1481


Page 864 of 1481
Alternative Fractures Other
Movements
HARDT AND NEGRIS ALTERNATIVE IS EXPLICITLY GROUNDED IN A DENIAL OF THE
ABILITY OF MOVEMENTS TO COMMUNICATE AND MAKE ALLIANCES THEY DENY
THE LOCALIZED EFFORTS OF REAL RESISTANCE MOVEMENTS AND TRY TO
SUBSTITUTE THE IMPOSSIBLE MODEL OF THE MULTITUDE

Cox, Prof of Sociology @ National U of Ireland, 2K1 (Lawrence, Social Movements and Empire, Rethinking
Marxism Vol. 13, No. 3-4)

Page 865 of 1481


Page 866 of 1481
Page 867 of 1481
Alternative Causes Terrorism
HARDT AND NEGRIS STRATEGIES OF RESISTANCE FORCLOSE ANY POLITICAL
NEGOTIATION INTELLECTUALLY ENDORSING THE KRITIK CAUSES TERRORISM

Kimball, Managing Editor of New Critierion, 2K1 (Roger, The new anti-Americanism, The New Critierion, Vol. 20,
No. 2, October, http://www.newcriterion.com/archive/20/oct01/empire.htm)

Empire is a contemporary redaction of the radicalism and anti-Americanism of the 1960s. It is the intellectual rationalization of
attitudes whose practical effects were demonstrated so vividly on September 11. Books like Empire are not
innocent academic inquiries. They are incitements to violence and terrorism. This is something that Antonio Negri, at any rate,
understands perfectly well. Emily Eakin described Negri as a flamboyant Italian philosopher and suspected terrorist mastermind who is
serving a 13-year prison sentence in Rome for inciting violence during the turbulent 1970s.

Page 868 of 1481


Page 869 of 1481
**Exceptionalism (USC)**

Page 870 of 1481


Exceptionalism Answers: 2AC
FIRST NO LINK PLAN NEVER POSITS GUANTANAMO AS A SITE OF EXCEPTION OR
CLAIMS TO LIBERATE DETAINEES FROM SOVEREIGNTY, MEANING THERES NO RISK
OF MASKING POWER

SECOND, WE SOLVE THE IMPACT THEIR NOLL EV ASSUMES THAT


EXTRAORDINARY RENDITION HAPPENS AND THAT WE DEFINE IT AS NOT BEING AN
HR VIOLATION, WHICH IS IMPOSSIBLE BECAUSE PLAN CREATES GENEVA
ADHERENCE

THIRD, THEIR AUTHOR CONCLUDES AFF AGAMBENS ALTERNATIVE IS


PARALYZING AND DELINKS THE LAW AND JUSTICE, ENABLING TOTALITARIANISM

Kohn 2006
[Margaret, Asst. Prof. Poli Sci @ Florida, Bare Life and the Limits of the Law,.Theory and Event, 9:2,
http://muse.jhu.edu/journals/theory_and_event/v009/9.2kohn.html, Retrieved 9-26-06//uwyo-ajl]

Is there an alternative to this nexus of anomie and nomos produced by the state of exception? Agamben
invokes genealogy and politics as two interrelated avenues of struggle. According to Agamben, "To show
law in its nonrelation to life and life in its nonrelation to law means to open a space between them for
human action, which once claimed for itself the name of 'politics'." (88) In a move reminiscent of
Foucault, Agamben suggests that breaking the discursive lock on dominant ways of seeing, or more
precisely not seeing, sovereign power is the only way to disrupt its hegemonic effects. Agamben clearly
hopes that his theoretical analysis could contribute to the political struggle against authoritarianism, yet
he only offers tantalizingly abstract hints about how this might work. Beyond the typical academic conceit
that theoretical work is a decisive element of political struggle, Agamben seems to embrace a utopianism
that provides little guidance for political action. He imagines, "One day humanity will play with law just as
children play with disused objects, not in order to restore them to their canonical use but to free them
from it for good." (64) More troubling is his messianic suggestion that "this studious play" will usher in a
form of justice that cannot be made juridical. Agamben might do well to consider Hannah Arendt's
warning that the belief in justice unmediated by law was one of the characteristics of totalitarianism.

It might seem unfair to focus too much attention on Agamben's fairly brief discussion of alternatives to
the sovereignty-exception-law nexus, but it is precisely those sections that reveal the flaws in his analysis.
It also brings us back to our original question about how to resist the authoritarian implications of the
state of exception without falling into the liberal trap of calling for more law. For Agamben, the problem
with the "rule of law" response to the war on terrorism is that it ignores the way that the law is
fundamentally implicated in the project of sovereignty with its corollary logic of exception. Yet the
solution that he endorses reflects a similar blindness. Writing in his utopian-mystical mode, he insists,
"the only truly political action, however, is that which severs the nexus between violence and law."(88)
Thus Agamben, in spite of all of his theoretical sophistication, ultimately falls into the trap of hoping that
politics can be liberated from law, at least the law tied to violence and the demarcating project of
sovereignty.

Page 871 of 1481


FOURTH, THIS ISNT OFFENSE, ITS A BAD PMN PLAN CREATES COMPARATIVELY
MORE DUE PROCESS, SOLVING OUR INTERNATIONAL PERCEPTION ADVANTAGES

[READ YOUR AGAMBEN ANSWERS]

Page 872 of 1481


Page 873 of 1481
**Feminism**

Page 874 of 1481


Feminism Answers: 2AC (1/2)
DECLARING SEXUAL INTELLIGIBILITY IS AN ACT OF BODILY ADMINISTRATION AND
ESTABLISHES A CATEGORY OF EXPENDABLE LIFE.

Judith Butler, Johns Hopkins University, Sexual Inversions, Foucault and the Critique of Institutions.
1993; mac//sam

How does this inversion from early to late modern power affect Foucault's discussion of yet another
inversion, that between sex and sexuality) Within ordinary language we sometimes speak, for instance, of
being a given sex, and having a certain sexuality, and we even presume for the most part that our
sexuality in some way issues from that sex, is perhaps an expression of that sex, or is even partially or fully
caused by that sex. Sexuality is understood to come from sex, which is to say that the biological locus of
11 sex' in and on the body is somehow conjured as the originating source of a sexuality that, as it were,
flows out from that locus, remains inhibited within that locus, or somehow takes its bearings with respect
to that locus. In any case, "sex" is understood logically and temporally to precede sexuality and to
function, if not as its primary cause, then at least as its necessary precondition.

However, Foucault performs an inversion of this relation and claims that this inversion is correlated with
the shift from early to late modern power. For Foucault, "it is apparent that the deployment of sexuality,
with its different strategies, was what established this notion of 'sex'."' Sexuality is here viewed as a
discursively constructed and highly regulated network of pleasures and bodily exchanges, produced
through prohibitions and sanctions that quite literally give form and directionality to pleasure and
sensation. As such a network or regime, sexuality does not emerge from bodies as their prior cause;
sexuality takes bodies as its instrument and its object, the site at which it consolidates, networks, and
extends its power. As a regulatory regime, sexuality operates primarily by investing bodies with the
category of sex, that is, making bodies into the bearers of a principle of identity. To claim that bodies are
one sex or the other appears at first to be a purely descriptive claim. For Foucault, however, this claim is
itself a legislation and a production of bodies, a discursive demand, as it were, that bodies become
produced according to principles of heterosexualizing coherence and integrity, unproblematically as
either female or male. Where sex is taken as a principle of identity, it is always positioned within 11 held
of two mutually exclusive and fully exhaustive identities; one is either male or female, never both at once,
and never neither one of them.

Foucault writes

the notion of sex brought about a fundamental reversal; it made it possible to invert the representation of
the relationships of power to sexuality, causing the latter to appear, not in its essential and positive
relation to power, but as being rooted in a specific and irreducible urgency which power tries as best it
can to dominate; thus the idea of "sex" makes it possible to evade what gives "power" its power; it
enables one to conceive power solely as law and taboo.'

For Foucault, sex, whether male or female, operates as a principle of identity that imposes a fiction of
coherence and unity on an otherwise or unrelated set of biological functions, sensations, pleasures. Under
the regime of sex, every pleasure becomes symptomatic of "sex,"

"sex" itself functions not merely as the biological ground or cause of

but as that which determines its directionality, a principle of

teleology or destiny, and as that repressed, psychical core that furnishes c clues to the interpretation of its
ultimate meaning. As a fictional imposition of uniformity, sex is "an imaginary point" and an "artificial

Page 875 of 1481


unity, but as fictional and as artificial, the category wields enormous power ' Although Foucault does not
quite claim it, the science of reproduction produces intelligible "sex" by imposing a compulsory
heterosexuality on the description of bodies. One might claim that sex is here according to a heterosexual
morphology.

The category of "sex" thus establishes a principle of intelligibility for human beings, which is to say that no
human being can be taken to be is human, unless that being is fully and coherently marked by sex And yet
it would not capture Foucault's meaning merely to claim that there are humans who are marked by sex
and thereby become intelligible. The point is stronger: to qualify as legitimately human, one must be
coherently sexed. The incoherence of sex is precisely what marks off the abject and the dehumanized
from the recognizably human.

Page 876 of 1481


Page 877 of 1481
Feminism Answers: 2AC (2/2)

Page 878 of 1481


Page 879 of 1481
Feminism Answers: 2AC (3/3)

Page 880 of 1481


Page 881 of 1481
A2 Feminism: 1AR
FEMINIST IDENTITY CATEGORIES ARE CONSTITUTED BY NORMALIZATION ONLY
QUESTIONING THEM CAN PROVIDE FREEDOM FROM GENDER SUBORDINATION

Butler 95
[Judith, Prof of Rhetoric at UC Berkeley, Contingent Foundations, Feminist
Contentions: A Philosophical Exchange, New York: Routledge, 50//wfi-ajl]

Paradoxically, it may be that only through releasing the category of women from a fixed
referent that something like "agency" becomes possible. For if the term permits of a
resignification, if its referent is not fixed, then possibilities for new configurations of the term
become possible. In a sense, what women signify has been taken for granted for too long, and
what has been fixed as the "referent" of the term has been "fixed," normalized, immobilized,
paralyzed in positions of subordination. In effect, the signified has been conflated with the
referent, whereby a set of meanings have been taken to inhere in the real nature of women
themselves. To recast the referent as the signified, and to authorize or safeguard the category
of women as a site of possible resignifications is to expand the possibilities of what it means to
be a woman and in this sense to condition and enable an enhanced sense of agency.

FEMINISMS STABLE FEMININE SUBJECT NORMALIZES IDENTITY, VIOLENTLY


MARGINALIZING THE PEOPLE IT CLAIMS TO DEFEND, REINSCRIBING OPPRESSION
***

Butler 99
[Judith, prof. of rhetoric at UC Berkeley, Gender Trouble: Feminism and the
Subversion of Identity, New York: Routledge, 1999, 7-8//wfi-ajl]

My suggestion is that the presumed universality and unity of the subject of feminism is
effecitvely undermined by the constraints of the representational discourse in which it
functions. Indeed,

the premature insistence on a stable subject of feminism, understood as a seamless category of


women, inevitably generates multiple refusals to accept the category. These domains of
exclusion reveal the coercive and regulatory consequences of that construction, even when the
construction has been elaborated for emancipatory purposes. Indeed, the fragmentation within
feminism and the paradoxical opposition to feminism from women whom feminism claims to
represent suggest the necessary limits of identity politics. The suggestion that feminism can
seek wider representation for a subject that it itself consturcts has the ironic consequence that
feminist goals risk failure by refusing to take account of the constitutive powers of their own
representational claims. This problem is not ameliorated through an appeal to the category of
women for merely strategic purposes for which they are intended. In this case, exclusion itself

Page 882 of 1481


might qualify as such an unintended yet consequential meaning. By conforming to a
requirement of representational politics that feminism articulate a stable subject, feminism thus
opens itself to charges of gross misrepresentation.

Page 883 of 1481


Page 884 of 1481
White Feminism Bad: 1AR

Page 885 of 1481


Page 886 of 1481
Page 887 of 1481
**Gift**

Page 888 of 1481


A2 The Gift: 2AC (1/4)
YOUR AUTHORS CONCEDE THAT WE SHOULDNT ABANDON ATTEMPTS TO MAKE
THE WORLD A BETTER PLACE OR THAT WE CANT EVER TAKE POLITICAL ACTION

Arrigo & Williams, Their Authors from the Califonria School of Professional Psychology, 2K
(Bruce & Christopher, The (Im)Possibility of Democratic Justice and the Gift of the Majority On Derrida,
Deconstruction, and the Search for Equality, Journal of Contemporary Criminal Justice, Volume 16,
Number 3, August)

With regard to the (im)possibility of a legally imposed equality in search of a


transformative justice, we (as social and political beings) must go beyond what is consciously imaginable,
calculable, and knowable. We must go beyond the realm of recognized possibility. This article does
not assume the position, as some critics of Derrida may suggest, that, given the ruse of
the gift, affording minority populations opportunity to attain equality should
therefore be discarded entirely (see Rosenfeld, 1993, on the dilemmas of a Derridean and
deconstructive framework for affirmative action). This article is far from a right-wing cry for
cessation of those undertakings that would further the cause of equality in
American society. This article is also not a statement of despair , a skeptical and
nihilistic pronouncement on the (im)possibility of justice (Fish, 1982) in which we are all rendered
incapable of establishing a provisional, deconstructive political agenda for meaningful
social change and action.

THEIR ARGUMENT IS THAT THE STATE SHOULD NEVER TAKE ANY ACTION AND
THEY ABANDON ALL LAWS IN WHICH CASE THEY LINK TO ALL OF OUR STATE GOOD
ARGUMENTS AND ALL OF OUR ANARCHY BAD ARGUMENTS THEIR ALTERNATIVE
IN THIS CASE INCREASES HUMAN SUFFERING AND ABANDONS STRATEGIES THAT
CAN CHIP AWAY AT STATE POWER

Chomsky, Renowned Political Activist & Linguistics Professor at MIT, 4-24-2K (Noam, Talking
'Anarchy' With Chomsky, By David Barsamian, http://www.thenation.com/doc/20000424/chomsky)

Comment on an African proverb that perhaps intersects with what we're talking about: "The master's tools will
never be used to dismantle the master's house." If this is intended to mean,
don't try to improve conditions for suffering people, I don't agree. It's true that
centralized power, whether in a corporation or a government, is not going to willingly commit
suicide. But that doesn't mean you shouldn't chip away at it , for many reasons. For one
thing, it benefits suffering people. That's something that always should be

Page 889 of 1481


done, no matter what broader considerations are . But even from the point of view of dismantling the master's house, if
people can learn what power they have when they work together, and if they can see dramatically at just what point they're going to be stopped, by force, perhaps, that teaches very valuable

The alternative to that is to sit in academic seminars and talk about


lessons in how to go on.

how awful the system is.

Page 890 of 1481


Page 891 of 1481
A2 The Gift: 2AC (2/4)
TURN: GLOBALIZATION MOVEMENTS

LITTLE A: THEIR ALTERNATIVE RELIES ON AND DEMANDS THE CREATION OF A


WORLD BASED ON DERRIDAS CONCEPTION OF DEMOCRACY TO COME IF THEY
SAY IT DOESNT THEY DONT SOLVE ANY OF THE KRITIK.

Arrigo & Williams, Their Authors from the Califonria School of Professional Psychology, 2K (Bruce &
Christopher, The (Im)Possibility of Democratic Justice and the Gift of the Majority On Derrida, Deconstruction, and
the Search for Equality, Journal of Contemporary Criminal Justice, Volume 16, Number 3, August)

What we do suggest, however, is simply the following: That political and/or legislative attempts at
empowerment (as they currently stand) are insufficient to attain the deconstructive and discursive condition of equality for minority citizen groups (Collins, 1993). More
significant, we contend that construction of these initiatives as Derridean gifts advance , at best, fleeting vertiginous moments of inequality

and injustice. Still further, we recommend the (im)possible ; that which, at first blush, admittedly delivers no pragmatic value for social analysts.

Our invitation is for a fuller, more complete displacement of equality and initiatives pertaining to it such that there
would be no giving for its own sake; that giving would not be construed as giving, but as the way of democratic justice (i.e., its foreseeabilitywould be [un]conscious, its recognizability would
be with[out] calculation). If we are able to give without realizing that we have done so, the possibility of reciprocation, reappropriation, and the economy of narcissism and representation are
abruptly interrupted and perhaps indefinitely stalled. This form of giving more closely embodies the truth of human existence; that which betters life for all without regard for differential

this justice both of and


treatment, neither promoting nor limiting those who are other in some respect or fashion. This re-presentation of equality,

beyond the calculable economy of the law (Derrida, 1997), requires a different set of
principles by which equality is conceived and justice is rendered. What would this difference entail? Howwould it be embodied in
civic life? In the paragraphs that remain, our intent is to suggest some protean guidelines as ways of identifying thework that lies ahead for the (im)possibility of justice and the search for

A cultural politics of difference grounded in an affirmative postmodern


aporetic equality.

frameworkwould necessarily prevail (Arrigo, 1998a; Henry&Milovanovic, 1996). In this more emancipatory, more liberatory vision, justice
would be rooted in contingent universalities (Butler, 1992; McLaren, 1994). Provisional truths, positional knowledge, and relational meanings would abound (Arrigo, 1995). New egalitarian
social relations, practices, and institutions would materialize, producing a different, more inclusive context within which majority and minority sensibilities would interact (Mouffe, 1992). In
otherwords, the multiplicity of economic, cultural, racial, gender, and sexual identities that constitute our collective society would interactively and mutually contribute to discourse on
equality and our understanding of justice. These polyvalent contributions would signify a cut in the fabric of justice, a text that pretends to be a whole (i.e., the whole of democratic justice)

(Derrida, 1997, p. 194).Equality on these terms would become an ethical, fluid narrative : an
anxiety-ridden moment of suspense (Derrida, 1997, pp. 137-138) cycling toward the possibility of justice. For Derrida

(1997), this is the moment of undecidability . The cacophony of voices on which this aporetic equality would be based would displace any

fixed (majoritarian) norms that would otherwise ensure an anterior, fortified, anchored justice. Instead, the undecidable, as an essential

ghost (Derrida, 1994), would be lodged in every decision about justice and equality (Desilva
Wijeyeratne, 1998). For Derrida (1997), this spectral haunting is the trace, the differance .19 It is

the avenir or that which is to come. The avenir is the event that exceeds
calculation, rules, and programs: It is the justice of an infinite giving (Desilva Wijeyeratne,
1998, p. 109). It is the gift of absolute dissymmetry beyond an economy of calculation (Derrida, 1997). This is what makes justice, and the search for equality, an aporia: It is possible only as an

it is the very (im)possibility of justice itself that renders the


experience of the impossible. However,

experience, and the quest for equality, amovement toward a destination that
is forever deferred, displaced, fractured, and always to come .

Page 892 of 1481


Page 893 of 1481
A2 The Gift: 2AC (3/4)
LITTLE B: THEIR DISCURSIVE CALL FOR THE DEMOCRACY TO COME DIVERTS REAL
WORLD MOVEMENTS FROM MORE EFFECTIVE SOLUTIONS AND UNDERMINES
THEIR POWER

Bedggood, Senior Lecturer in Sociology at the University of Auckland, 99 (David, Saint Jacques: Derrida and the
Ghost of Marxism, Cultural Logic, located at: http://eserver.org/clogic/2-2/bedggood.html)

in his misappropriation of Marx, Derrida offers the young idealists of today a brand of
71. So

anarchism they can consume in the belief that their actions constitute a rebellion for
"democracy" and "emancipation" against the dehumanising norms and conventions that
alienate them. Just as Stirner's "association of egoists" was a figment of his "Thought", Derrida's new International has the potential to
divert a new generation of alienated youth into discursive acts against the symptomatic phrases
rather than the materialist substance of capitalist crisis . 72. In his response to his critics who deride the idea of an "international" without class he
replies: Whenever I speak of the New International in Specters of Marx, emphasising that, in it, solidarity or alliance should not depend, fundamentally and in the final analysis, on class affiliation, this in no wise signifies,
for me, the disappearance of "classes" or the attenuation of conflicts connected with "class" differences or oppositions (or, at least, differences or oppositions based on the new configurations of social forces for which I
do in fact believe that we need new concepts and therefore, perhaps new names as well) . . . the disappearance of power relations, or relations of social domination . . . . At issue is, simply, another dimension of
analysis and political commitment, one that cuts across social differences and oppositions of social forces (what one used to call, simplifying, "classes"). I would not say that such a dimension (for instance, the
dimension of social, national, or international classes, or political struggles within nation states, problems of citizenship or nationality, or party strategies, etc.) is superior or inferior, a primary or a secondary concern,
fundamental or not. All that depends, at every instant, on new assessments of what is urgent in, first and foremost, singular situations and of their structural implications. For such an assessment, there is, by definition,
no pre-existing criterion or absolute calculability; analysis must begin anew every day everywhere, without ever being guaranteed by prior knowledge. It is on this condition, on the condition constituted by this injunction,

the term "international" is a mystique. It


that there is, if there is, action, decision and political responsibility -- repoliticization.108 73. In other words,

covers for a nihilistic cult. Its Marxist meaning is inverted ; just as messianicity is messianism without a given messiah -- because everyone
is one's own messiah. There is no prior knowledge that can guide any collective action because that

pre-anything (society, religion, etc.) is spectral, is the unfilled "void". There are only irreducible acts which individuals
perform at any given moment by personally attempting to calculate, on the spot as it were, which of
many "dimensions" or "forces" immediately concern them, "responsibly" and in the name
of "justice" (whose gift?). If there is one name to apply to this contingent conjunction of "forces" which tries to "name" the "new" it is as I have argued above, performativity.109 Moreover, as I set out to
prove, Derrida's performativity is the idealist philosophical license for the political/social concept of reflexivity as developed by Soros and Giddens to express their abstract understanding of the 'structure-agency' problem

we could not get a better prescription for "demobilising" and


in the new global economy.110 Teamed-up, as performo-reflexivity,

"depoliticising" the masses in the face of the current world crisis of capitalism.

Page 894 of 1481


Page 895 of 1481
A2 The Gift: 2AC (4/4)
AND, THE FAILURE OF ANTI-GLOBALIZATION MOVEMENTS CAUSES EXTINCTION

Shiva, Physicist & Ecologist and Director of the Research Foundation for Science Technology and
Natural Resource Policy, 12-12-99 (Vandana, The Historic Significance of Seattle, located at:
http://flag.blackened.net/global/1299arshiva.htm)

The failure of the W.T.O Ministerial meeting in Seattle was a historic watershed, in more than one way. Firstly, it has
demonstrated that globalisation is not an inevitable phenomena which must be accepted at all costs but a political project which
can be responded to politically. 50,000 citizens from all walks of life and all parts of the world were responding politically when

they protested peacefully on the streets of Seattle for four days to ensure that there would be no new round of trade negotiations for accelerating and expanding the
process of globalisation. Trade Ministers from Asia, Africa, Latin America and the Caribbean were responding politically when they refused to join hands to provide support to a
"contrived" consensus since they had been excluded from the negotiations being undertaken in the "green room" process behind closed doors. As long as the conditions of
transparency, openness and participation were not ensured, developing countries would not be party to a consensus. This is a new context and will make bulldozing of

. The rebellion on the streets and the rebellion within the W.T.O. negotiations
decisions difficult in future trade negotiations

has started a new democracy movement - with citizens from across the world and the governments of
the South refusing to be bullied and excluded from decisions in which they have a rightful share . Seattle had
been chosen by the U.S to host the Third Ministerial conference because it is the home of Boeing and Microsoft, and symbolises the corporate power which W.T.O rules are
designed to protect and expand. Yet the corporations were staying in the background, and proponents of free-trade and W.T.O were going out of their way to say that W.T.O
was a "member driven" institution controlled by governments who made democratic decisions. The refusal of Third World Governments to rubber-stamp decisions from which
they had been excluded has brought into the open and confirmed the non-transparent and anti-democratic processes by which W.T.O rules have been imposed on the Third
World and has confirmed the claims of the critics. W.T.O has earned itself names such as World Tyranny Organisation because it enforces tyrannical anti-people, anti-nature
decisions to enable corporations to steal the world's harvests through secretive, undemocratic structures and processes. The W.T.O institutionalises forced trade not free trade,
and beyond a point, coercion and the rule of force cannot continue. The W.T.O tyranny was apparent in Seattle both on the streets and inside the Washington State
Convention centre where the negotiations were taking place. Non violent protestors including young people and old women, labour activists and environmental activists and
even local residents were brutally beaten up, sprayed with tear gas, and arrested in hundreds. The intolerance of democratic dissent, which is a hallmark of dictatorship, was
unleashed in full force in Seattle. While the trees and stores were lit up for Christmas festivity, the streets were barricaded and blocked by the police, turning the city into a war
zone. The media has referred to the protestors as "power mongers" and "special interest" groups. Globalisers, such as Scott Miller of the U.S. Alliance for Trade Expansion
said that the protestors were acting out of fear and ignorance. The thousands of youth, farmers, workers and environmentalists who marched the streets of Seattle in peace and
solidarity were not acting out of ignorance and fear, they were outraged because they know how undemocratic the W.T.O is, how destructive its social and ecological impacts
are, and how the rules of the W.T.O are driven by the objectives of establishing corporate control over every dimension of our lives - our food, our health, our environment, our
work and our future. When labour joins hands with environmentalists, when farmers from the North and farmers from the South make a common commitment to say "no" to
genetically engineered crops, they are not acting in their special interests. They are defending the common interests and common rights of all people, everywhere. The divide
and rule policy, which has attempted to put consumers against farmers, the North against the South, labour against environmentalists had failed. In their diversity, citizens were

united across sectors and regions. While the broad based citizens campaigns stopped a new Millennium Round of W.T.O from being launched in
Seattle, they did launch their own millennium round of democratisation of the global economy. The real Millennium
Round for the W.T.O is the beginning of a new democratic debate about the future of the earth and the future of it's people. The centralized, undemocratic

rules and structures of the W.T.O that are establishing global corporate rule based on monopolies and
monocultures need to give way to an earth democracy supported by decentralisation and diversity. The
rights of all species and the rights of all people must come before the rights of corporations to make
limitless profits through limitless destruction. Free trade is not leading to freedom. It is leading to slavery. Diverse
life forms are being enslaved through patents on life, farmers are being enslaved into high-tech slavery,
and countries are being enslaved into debt and dependence and destruction of their domestic
economies. We want a new millennium based on economic democracy not economic totalitarianism. The future is possible for humans and
other species only if the principles of competition, organised greed, commodification of all life,
monocultures, monopolies and centralised global corporate control of our daily lives enshrined in the W.T.O are
replaced by the principles of protection of people and nature, the obligation of giving and sharing
diversity, and the decentralisation and self-organisation enshrined in our diverse cultures and national
constitutions. A new threshold was crossed in Seattle - a watershed towards the creation of a global
citizen-based and citizen-driven democratic order. The future of the World Trade Organisation will be shaped far more by what happened on
the streets of Seattle and in the non-governmental (NGO) organisation events than by what happened in the Washington State Convention Centre. The rules set by

the secretive World Trade Organisation violate principles of human rights and ecological survival. They violate
rules of justice and sustainability. They are rules of warfare against the people and the planet. Changing these rules is

the most important democratic and human rights struggle of our times. It is a matter of survival.

Page 896 of 1481


Page 897 of 1481
Anti-Globalization Turn: 1AR
(1/2)
EXTEND OUR ANTI-GLOBALIZATION TURN: THEIR ALTERNATIVE IS PREMISED ON
DERRIDAS POSTMODERN CONCEPTION OF THE NEW INTERNATIONAL AND
DEMOCRACY TO COME. THEIR DISCURSIVE CALL FOR THESE STRUCTURES
DEMOBILIZIE AND DEPOLITICIZE REAL WORLD MOVEMENTS THAT ARE NECESSARY
BRING DOWN THE WALLS OF SOVEREIGNTY AND HEGEMONY. THEIR
ALTERNATIVE IS TOO IDEALISTIC IT FORSAKES ORGANIZATION AND ALLIANCES
THAT ARE NEEDED TO SOLVE. THE IMPACT IS RAMPANT EXPANSION OF
GLOBALIZATION WHICH CAUSES EXTINCTION.

OUR ARGUMENTS FLIPS THE ALTERNATIVE EVILS WILL CONTINUE TO EXIST IN


THEIR WORLD BUT THE DEMOCRACY TO COME WILL UNDERMINE ANY WAY TO
SOVLE THEM THIS ANSWERS ALL OF THEIR OFFENSE ABOUT CHANGING THE WAY
THE WORLD WORKS

Froment-Meurice, Professor of French at the University of Vanderbilt, 2K1 (Marc, Specters of M, Parallax,
Volume 7, Number 3)

Without (sans) is the sense of the new International. This untimely link new alliance without alliance, messianism without messianism,
without content is a relationless relation, without belonging or pertaining, or, better yet, what I call departing/disowning (de partenance). It is the party of the partyless and the a-political
although it is not a third way opening between the two traditional parties (conservative or progressive, right or left); above all it is not a gathering, an assembly of whoever happens to be
unhappy with traditional politics, with the democratic system based on alternation (even if this points to a blatant lie, since most of the time it all boils down to the same: new heads simply
alternate with old heads). Nor is it a matter of assembling all those who are uninterested in politics or who seek an alternative way free radicals in the margins. In general, the point

is to assemble and gather individuals but to recognize that the option of


precisely

community vs. individualism plays the same game , based on the same conception of the modern
subject as autonomous and autarkic. U.S. society offers good evidence for this: it is everything except a society, while displaying a most
depressing show of people obsessed by community whatever it may be: it starts with the neighborhood or the churches, not to mention family and its values, of course. Yet there has never

Essentially there has never been a more destructive myth


been a more dissociated and divided society.

than that of the individual who associates with others to form a we that is
nothing but a facade or may even be completely factitious . First and this should be the starting point
there is no individual who cannot be infinitely divided. Should there be only one, the individual is never one, except materially, if I may say so, although nearly every part of the body is
replaceable nowadays (not to mention the sex). And now here is we, of this new International (which has never existed and thus is not new in the sense that we speak of a new car model):
it (we) should not designate a community to which we belong except in terms of that to which we do not belong: not a family, not a nation, not a party, not a sex, not a language, and so

Yet surely if we belong to nothing at all, it will not take


on everything and anything, stretching it to the limit.

anyone long to notice that we are nothing at all an abstraction, a ghost, even more so than the clouds of ideology and
also that we cannot help but belong de facto to a language, for instance: just as the International(e) was written in French. But that didnt stop it from becoming the Soviet anthem until
Stalin replaced it with his anthem, with its clear nationalist resonance. An untimely link is a link nonetheless, or rather an alliance an engagement, complete with a commitment and a
(diamond) ring. But this alliance does not rely on any positive contents for its definition, or on the items of a program to be carried out. That might imply that this alliance does not commit to
anything only to witnessing itself (herself, alliance being feminine): like language said by Holderlin (quoted by Derrida in Specters) to have been given to human beings so that they can bear
witness to what they are: speaking beings, first and foremost. Having quoted precisely the same fragment by Holderlin, I called this circularity deposition: What man [the human being] is he
receives it from the word, and this being is being the witness of the word or its warden answering for it. Deposition is what one might call such circularity: to be the depositary of Being and
making a deposition for its manifestness in speaking the received language.8 But it is important always to underscore, as Derrida does, that Inheritance is never a given, it is always a task9
; that the human being has to be, like Dasein. (Have / to [a / a` ]: have as in have to. . . and not possess such is the sense of the ownmost in Heidegger. At least in one of several
Heideggers, the one I address in that he speaks to me.) Further, a language has to be learned, starting with ours, the one we owe it to (so) to speak; the one we owe ourselves to, inasmuch
as it has given us its word, given not as a fact but as a promise. Such circularity without origin constitutes a ring: infinite circulation of meaning, stopping nowhere (this would translate into the
concept of God if God could be a concept and therefore nowhere a God or nowhere as God). What I also call langagement gives the formal structure of language (in quotation marks: the
concept of formality is just meant to prevent any positive content from keeping its countenance): its transmissibility (or translatability) precisely prohibits any closure and thus any
appropriation without remainder in one unique and universal language. To the very extent that the promise (the gift of language, of the word as given word) is not incarnated in any

Page 898 of 1481


determined language just as there cannot really be any country corresponding to the Promised Land10 to that (de-ceptive) extent the idiom bears witness to this infinite engagement: it
(the idiom) is the witness that, at the heart of that which allows the circulation of meaning, there is some resistance. The idiom will not yield to translatability unabridged and integral, and
likewise the new International attests to the existence or occurrence in the bosom of universal westernization the merchandising of the planet now called globalization of some thing
that resists any appropriation insofar as this thing is not actually a thing and, deep down, is nothing at all or is this nothing without which, as it happens, no whole or totalization is possible

(thus impossible: no totalization is able to totalize nothing or a bunch of spectres). This sketchy alliance is spectral, first of all. It haunts
the home like nihilism, described by Nietzsche as the uncanniest, most unheimlich of guests. I need not really mention again how it all starts and what ushers
in the Specters, namely that it is Marx himself who speaks of the specter of communism.1 1 But I will mention it because this starting point also

conditions the nature of the new alliance and in this sense communism is not
only not dead, it also cannot die: to the extent that a spectre cannot ever be anything except dead surviving inasmuch as it is chased, hounded,
warded of, professed dead at last, for good, once and for all.

Continues

Page 899 of 1481


Page 900 of 1481
Anti-Globalization Turn: 1AR
(2/2)
No one, perhaps, has paid enough attention to the oppositional dialectics that result from this apparition of the specter. Not only are the powers of old Europe forming a Holy Alliance in order to ward off this spectre, the
Communists also have to come out and manifest themselves, perhaps first and foremost, since they are henceforth a recognized player or power, in order to crush the fiction or hallucination of the communist specter
(even if they owe their recognition to him). Communists must get up an erection well rendered by the first two lines of the International(e) in French, repeating Debout, Arise and they have to get up, lift, pick up,
sublate (Hegels Auf hebung is the lever) the wicked specter, a specter who is, uncannily enough, communist. Communists must manifest themselves as such, being communist beings: as the spectres negation. But
does this not make them run the risk of disabling what made communism such a formidable, uplifting power, namely that it appeared only as a spectre, a hallucination? Its a thousand times easier to fight against real
(manifest) powers than against thin air. In the same spirit, Heidegger describes Angst exactly as a fear of nothing: related to nothing special, only to being as a whole. All this points to why I have never had a need to
be anti-communist also known as a swine, said Sartre: yet communists managed to be so even more so than the most bigoted anti-communists. Besides, the latter the reactionaries of the Holy Alliance were the
ones most directly touched by the fall of communism. Suddenly they lost their entire raison detre, became unable to oppose anything, and therefore in these well-lubricated dialectics haunting Marxism they could no
longer pose or recognize themselves in their opponents negation. Am I saying that now, now that communist beings have virtually vanished (except for a handful of survivors), communist Being is going to make a
comeback, come back to spook us like Marxs spectre in old Europe, except that this time everyone in the whole world would be affected the globalized world said to be opposed by the new International? I am
actually asking whether the question has any meaning: without any beings to embody it and have it materialize, how to speak of a pure communist Being? Does it not make it all fall back into the ontotheological clouds
Marx denounced for being ideological superstitions (pure spirit, bodyless Holy Ghost)? But also how not to see that, by defining the new International in terms of an opposition (an opposition, simply, to reality), one
repeats and sublates Hegels specter haunting (the various) Marxism(s), driving it (them) to its (their) demise and/or end(s), each time and whatever the attempts to cut the theory from what makes it work (for instance in

The need urgent and


the invention of a so-called epistemological shift thanks to which true scientific Marxism might be cut off from its stubborn sprite without which it could not even pretend to be true)?

challenging to conceive the struggle in terms that do not call for opposition , of classes for instance, is plain. Is class

struggle nothing but the same old song echoing? Has it outlived its given time, its usefulness? But that still doesnt entitle anyone to be so naive

(full of counterfeit or rather interested naivete) and proclaim , urbi et orbi, that there is no struggle left , nothing but the euphoria of a classless society. The

struggle is lasting, the fight goes on, even if the last fight (It is the last fight / The International(e) / Shall be the
human race (all of humankind) [Cest la lutte finale / LInternationale / Sera le genre humain]) is also a struggle for the end of fighting. (Unlike the Marseillaise, the International(e) has no warmongering
and nationalist strain but delivers a vibrant call to abolish all discipline and encourage deserters.) Whoever proclaims that there should be nothing

but the International(e) as the whole of humankind in the future seems to give in to a dangerous or at least idealistic utopia:
the ends of the new Alliance, deep down, are the dissociation of any (interest) group, including the association of the International,
by the same token (Let us band together: that is its motif, if I may repeat it; with an us possible only if it is opposed to them).

Page 901 of 1481


Page 902 of 1481
Anti-Globalization Movements
Up Now (1/2)
ANTI-GLOBALIZATION MOVEMENTS ARE STRONGER THAN EVER THEY SOLVE
BECAUSE THEY ARE ORGANIZED AND BROAD BASED

LA Times 11-24-2K2

Authorities on development issues , including some of globalization's stalwart defenders, say the movement in this
country has broadened, matured and become more influential in the 33 months since Seattle. "The

movement is getting much more sophisticated, even the activists in the


streets," said Nancy Birdsall, a former World Bank official who heads the Center for Global Development in Washington. "It's gone from anti-
globalization to alternative globalization to managing globalization." Development
experts credit activist pressure at least in part for a range of developments , including
a decision by the World Bank to give poor countries a bigger voice in developing poverty-reduction plans and agreement by the World Trade Organization to give top priority to the needs of
poor countries in the round of worldwide trade talks launched last year. Globalization critics denounce some of those initiatives as inadequate. But if nothing else, they represent an
acknowledgment that wealthy nations and their financial institutions cannot afford to appear indifferent to global injustice. "They won the verbal and policy battle," said Gary Hufbauer, a pro-
globalization economist at the Institute for International Economics in Washington. "They did shift policy. Are they happy that they shifted it enough? No, they're not ever going to be totally

Experts see evidence of the movement's growing influence in


happy, because they're always pushing."

other arenas. Several high-profile economists, including Nobel Prize winner Joseph Stiglitz, have
endorsed some of the specific criticisms and objectives of the movement . Their critique
was reinforced by growing evidence of the failure of "Washington consensus" formulas to foster growth in Africa, Asia and Latin America. The issue of Third World debt relief resonated with a
much wider audience when Treasury Secretary Paul H. O'Neill and Irish rock star Bono jointly toured some of sub-Saharan Africa's poorest countries. Many development experts point to
Jubilee 2000, the Third World debt-relief group whose work has been championed by Bono, as the non-government organization with perhaps the most influence over public policymaking.

Jubilee 2000 had a tremendous impact in mobilizing focus and political


"

support for the decisions that were eventually made ," said Mats Karlsson, the World Bank's vice president for
external affairs. The result, he said, "is a very radical debt relief program that is now being implemented country by country." Other groups have had an effect too. Oxfam, the London-based
relief organization, made waves with a report stating that more trade liberalization, if managed properly, is the best prescription for reducing world poverty. The International Labor

All of the major organizations have


Organization has convened a high-profile working group to assess the social implications of globalization. "

grown enormously more powerful and effective. The only thing that's shrunk is the street protests," said Mark Weisbrot,
co-director of the liberal Center for Economic and Policy Research in Washington. " The movement hasn't lost momentum at all.

It just shifted to a different set of tactics." For every organization involved in


what some call the "movement of movements," there have also been smaller
but symbolically important victories. Jubilee USA's crusade has been joined by a remarkably wide range of organizations, from conservative
evangelical churches to the San Francisco 49ers football team. For the World Bank Bond Boycott, which hopes to generate the kind of financial pressure that helped end apartheid, a big
turning point was the Milwaukee City Council's 13-1 vote this spring to join the campaign. "We've seen a huge shift," said boycott coordinator Neil Watkins. "When we started in 2000, there's

Leaders say the movement's evolving profile reflects


no way we could have even talked to the city of Milwaukee."

a deliberate decision to tone down the increasingly provocative street


mobilizations staged outside meetings of the IMF, World Bank, World Trade Organization and other global institutions. Although authorities said the vast majority of
participants were peaceful, small groups of Black Bloc anarchists and other extremists were giving the protests a violent edge. In Seattle, their antics contributed to $2 million in property
damage and 500 arrests. Then came Sept. 11. Public revulsion for terrorism and heightened concern about security created even more ambivalence within the movement about the merits of
street mobilizations. Anti-globalization groups had been planning a Seattle-size protest at the fall 2001 meetings of the IMF and World Bank in Washington, but the sessions were canceled
shortly after Sept. 11. When the institutions held their spring meetings here in April, only 1,000 or so protesters rallied outside their headquarters. "After 9/11, the U.S. movement obviously
reevaluated its tactics and its tone," said Lori Wallach, who has directed Public Citizen's Global Trade Watch operation since 1990. "But even before 9/11, there was a strategy judgment that

the movement's current level of energy and


we needed to diversify the ways in which we organized and mobilized." Wallach said

engagement far exceeds what prevailed during the struggle over ratification of
the North American Free Trade Agreement.

Page 903 of 1481


Page 904 of 1481
Page 905 of 1481
Anti-Globalization Movements
Up Now (2/2)
STATUS QUO MOVEMENTS SOLVE GLOBALIZATION ORGANIZATION IS CRITICAL

Workers Power Global 2K1 ([anti] capitalisim: from resistance to revolution, Workers Powers Action
Guide To the Anti-Capitalist Movement, June, located at: http://www.workerspower.com/wpglobal/anticap0.html)

We are present at a turning point in history. From Seattle to Genoa. In less than two years the anti-globalisation movement
has travelled a long and spectacular journey. Washington, Melbourne, Prague, Seoul, Nice, Quebec, Barcelona to name only a few cities have seen
major demonstrations against corporate exploitation and environmental destruction as well as against the hollowing out of democracy by the governments of G8 and their pliant international

agencies. Along the way, the movement has got bigger and ever more clearly targeted on the real enemy: the
capitalist system. Since Seattle, tens of thousands of police, innumerable rounds of tear gas, batons, steel perimeter fences, vicious police dogs, exclusion orders, sealed borders,
closed airports, blockaded roads, midnight raids all have been deployed by the capitalist governments to stop our voices being heard. But the movement is growing
despite all that. Seattle, 30 November, was a defining moment when the movement became conscious of its power. But it did not come from nowhere. Years of grassroots collective action in
the USA culminated in Seattle. Students had been at the heart of it, campaigning against the unleashing of corporate depravity that marked politics in the Clinton years. A new generation of
activists on campuses across the USA and Canada became politicised by the invasion of the mind-snatchers as the big corporations made their move to take over of education. Faced with the
hubris of money, student politics moved on from the politics of identity and introspection to anti-corporatism - to stem and turn back the agents of Nike, Coca-Cola and McDonalds dressed up
as educationalists. Heavy-handed attempts at censorship or blackmail in the face of criticism of the big brand names only radicalised them more. They investigated the operations of the big
corporations away from their campuses and found that the money used to bribe their administrators was sucked out of sweatshop labour in the Philippines, Indonesia, Vietnam and China
the one-dollar-a-day impoverished billions of the Third World. Seattle put it all together. As Manning Marable said: "The demonstrations in Seattle showed that growing numbers of Americans
are recognising that all of these issues Third World sweatshops, the destruction of unions, deteriorating living standards, the dismantling of social programs inside the US are actually
interconnected." But the campus campaigns in the USA were only one strand of the emerging anti-globalisation movement. The Zapatista uprising on New Years Day 1995 in the Chiapas
region of Mexico was a rebellion against land hunger and violent autocracy and for indigenous rights and the end of the countrys enslavement to US companies, exploitation and foreign debt.

Tens of thousands of new and old activists rallied to their call to support them and to open up many
fronts of struggle against imperialism. A Zapatista internationalism was born in the Laconda rainforests and quickly formed cross-currents with the North
American and then European anti-capitalists. Another strand that emerged in the 1990s was the radicalisation of some NGOs. In Britain, 1997 and 98 saw Jubilee 2000 mobilise 70,000 and
50,000 respectively to demand the G7 cancel the debts of the Third World. In the South, many of the smaller, more independent,1 NGOs who were closer to the suffering caused by
government and business alike signed up to the anti-globalisation movement. Paradoxically, the "privatisation" of healthcare and famine relief removed the shackles of apolitical
humanitarianism and allowed a generation of NGO workers to become overtly radical. But by far the biggest component of the emerging world anti-globalisation movement has been the
millions of workers who have taken to the streets and gone on strike to resist the many attacks on them which originated in IMF "structural adjustment programmes" during the 1980s and

tens of
1990s. The IMF has engineered cuts in health and education programmes, let rip state controlled prices for foodstuffs and fuel and downsized the public sector workforce. But

millions have fought back time and again in South Asia, West Africa and Latin America. Sometimes they
have won concessions. But often they have been betrayed by reformist and nationalist leaders. All too often they have not received
active solidarity from trade unionists and leftists in the North. Yet, until the mid-1990s, we were in an era of rearguard actions against

the sweeping tide of globalisation and neo-liberalism. US imperialism swept all before it in the wake of its victory in the Cold War. As Walden Bello noted, this era
peaked with the founding of the World Trade Organization in 1994-95, the apogee of capitalism in the era of globalization. But it spawned a movement

against itself and this connected with other movements . Success in stalling the Multilateral Agreement on Investment (MAI) gave it confidence.
Then came the Asian financial crisis of 1997-1998, which Bello has called the Stalingrad of the IMF when it became clear that the IMF itself, with its prescription for capital account
liberalization, helped create the crisis, and with its cure of tight money and tight budgets, converted a financial crisis into economic collapse in Thailand, Indonesia, and Korea. Across the
WTO, IMF and finally the World Bank a complete crisis of legitimacy set in during the closing years of the 20th century. Their defensiveness and confusion only emboldened the movement

The broadening of the anti-globalisation movement has been


against them, leading to the turning point that was Seattle.

accompanied since then by its ideological deepening, in particular a growing sense of practical
internationalism and conscious anti-capitalism. The phenomenon of summit-hopping is one expression of this, as is the proliferation of counter-
conferences and teach-ins with representatives from all over the world. The massive anti-Davos summit in Porto Alegre, Brazil, in January 2001, gathered toghether all wings of the anti-

The movement of one no and many yeses intensified the debate around alternative
globalisation movement.

visions and programmes for a world free from exploitation and oppression and what alliances and tactics
are necessary to get there. That is welcome. But the course of the movement itself has posed the question of which way forward? far more directly than any forum
could. The buzz of success is giving way to a sharp debate over goals, strategy and tactics. After the Gothenburg violence we are hearing loud pleas for moderation and compromise from a
self-appointed layer of go-betweens in the movement. All they ever wanted was a place at the negotiating table - and their support for protests that put them there has to be understood in
the light of that. Susan George, an early icon of the movement who praised it last year for "doing more in one year than all her books have down in the last 25 years" was quick to condemn
plainly and clearly the protestors action on the streets of Gothenburg because violence is invariably the game of our adversary.even in the case of provocation, even when the police is
responsible for having opened hostilities Even those that proclaim to be revolutionary buckle under the pressure of bourgeois denuciation of street violence. The Socialist Party in Sweden a
so-called Trotskyist group - denounced those responsible for attacking police and property for scar[ing] the life out of the population in Gothenburg. They criticise several so called left
organisations that still refuse to resolutely distance themselves from a direction which is totally stillborn . . Instead of total repudiation and contempt these organisations try to fish in the
swamps of political street violence, said the Swedish section of the Fourth international. The Swedish SP counterposes work in mass movements to street violence. The fact is, effective
mass protest has always been met with police violence. The fact is that those who denounce violence do not share our goal or that of hundreds of thousands of youth today: to smash the
apparatus of capitalist repression that keeps our movement down and guarantees the continued rule of the big corporations. Christophe Aguiton, leader of ATTAC, anxious also to distance
himself from the violence at Gothenburg, claims that the coalition of peaceful forces inside the anti-globalisation movement has meant that the question is no more, as in the 1970s, in the
great majority of cases, to conquer the Power via revolutionary organisations, but to find other ways for radical protest. We draw the opposite conclusion. The ferocity of the state shown in

this movement needs to


Gothenburg and Barcelona in June 2001, the removal or restriction of our democratic rights under way as we prepare for Genoa, show that

raise its game. If we dont, we risk falling back to the isolated and fragmented protests over debt, pollution etc that
characterised the 1980s and early 1990s. Indeed, that is where some of the NVDA activists are headed as if frightened by the power of the mass movement they helped
create that is shaking capitalism to its foundations. Today, this minute, we have the best chance since the 1970s to build revolutionary organisations that have a mass base
among young people and organised workers. Today the spectre" of anti-capitalism stalks the worlds rulers literally it is just yards away from their pampered international
gatherings. So it is time the movement outlined its goals clearly. Anti-capitalism means expropriation of all the MNCs, banks, and the other large companies and landowners
too, so that economic power is put in the hands of the workers and peasants without which rational economic planning will prove impossible. It means fighting for the overthrow
of the bosses and bureaucrats in G7 and G77 countries alike. It means workers and peasants taking power into their own hands by means of general strikes and armed militias.

Page 906 of 1481


It means working class people running their own lives - through the forums of elected and recallable delegates in councils. Lets grasp the opportunity to build a revolutionary
international movement. Globalisation has sounded an alarm call to the youth and activists at the base of the worlds workers' movement. The dramatic surge in the concentration and
centralisation of capital, the size and velocity of capital movements, the power of the G8 dominated "world economic institutions", the downsizing or privatisation of social welfare all
threaten workers and small farmers and a substantial proportion of the lower middle classes. But enormous new opportunities also lie ahead. The greater unification of the world economy
the higher levels of education and literacy called for by the introduction of new information and communications technology means that workers can spread the struggles and the lessons of

A revolutionary fight that links the anti-


struggles at the speed of thought, to use Bill Gates phrase. One no and many yes-es will not destroy capitalism.

capitalist movement with the multi-millioned organised working class will destroy capitalism . This pamphlet is an
action guide for building that movement.

Page 907 of 1481


Page 908 of 1481
Provisional Truth Turn: 2AC
(1/2)
TURN: PROVISIONAL TRUTHS

LITTLE A: THEIR ALTERNATIVE RELIES ON THE CREATION AND ACCEPTANCE OF


PROVISIONAL TRUTHS

Arrigo & Williams, Their Authors from the Califonria School of Professional Psychology, 2K (Bruce &
Christopher, The (Im)Possibility of Democratic Justice and the Gift of the Majority On Derrida, Deconstruction, and
the Search for Equality, Journal of Contemporary Criminal Justice, Volume 16, Number 3, August)

This justice that is to come, this equality as an aporetic destination, resides in discourse. The
production of provisional truths and knowledge requires that the voice(s) of
alterity emerge to construct new visions of relational and positional equality
and justice. Thus, the undecidability of interaction the inclusion of minority discourse with majoritarian discourse as
differance represents a radically democratic in-road producing multilingual,

multicultural, and multiracial effects for equality . This is what Caputo (1997) refers to as a highly miscegenated
polymorphism (p. 107). For Derrida (1991, 1997), a radical democracy is constituted by preparedness for

the incoming of the other. Derrida (1997) advocates highly heterogenous, porous, selfdifferentiating quasi-identities, [and] unstable identities . . . that . . .
do not close over and form a seamless web of the selfsame (p. 107). In short, a receptacle for difference that receives the

provisional truths, positional knowledge, and supplemental processes of


meaning making is necessary in the struggle for (im)possible equality .

Page 909 of 1481


Page 910 of 1481
Provisional Truth Turn: 2AC
(2/2)
LITTLE B: OPENING SPACE FOR THESE TYPES OF PROVISIONAL TRUTHS OPENS THE
DOOR FOR HOLOCAUST DENIAL

Sherry, Earl R. Larson Civil Liberties & Civil Rights Law Professor at the University of Minnesota, 96 (Suzanna, THE
SLEEP OF REASON, Georgetown Law Journal, February, 84 Geo. L.J. 453)

The consequences of accepting epistemological pluralism go much deeper than making some
If we
epistemological pluralists look inconsistent or undermining attacks on the status quo, and are much more troubling than simply failing to fulfill the expectations of its proponents.

cannot confidently assert that the earth is round or that evolution occurred, because those
with a different epistemology present a counterargument that is valid in their
world even if not in ours, then the same must be true of other scientific or historical statements. It is
only the tools of the Enlightenment tradition that allow us to refute such
unsupported claims as that virtually all of what we now consider the accomplishments of Western civilization was stolen from black Africans, n160 or that the
tragic bombing of the Oklahoma City federal building was the work of agents of the United States government. It is only the acceptance of

reason and empiricism as the epistemological standard that allows us to reject such
pseudoscientific theories, currently fashionable in some quarters, as that melanin is "one of the strongest electromagnetic field forces in the universe"
with the power to make its possessors intellectually superior, n161 or that Jewish doctors are injecting black babies

with the AIDS virus. n162 Nor is it a defense that the modern alternative epistemologies advocated by radical and religious scholars do not always lead to such absurdity. n163
The point is that antirational epistemologies, unlike the principles of the Enlightenment, offer no weapons against

a variety of intellectual and political atrocities . As Marvin Frankel points out, "for most of Judaism's 5700-plus years, . . . the
great Western religions neither caused democracy to happen nor exhibited discomfort about its absence." n164 [*483] Even today, the religious epistemologies that mandate discrimination
against gays and lesbians are indistinguishable from those in the not too distant past that mandated discrimination against blacks. n165 And if the melanin or AIDS myths are not sufficiently

there is a more horrific example of the beliefs that become acceptable


silly or frightening,

when reason and empiricism are demoted as socially constructed


epistemologies. Deborah Lipstadt notes that postmodern doctrines have allowed Holocaust
denial theories to flourish and to be treated as "the other side ," another "point of view," or a
"different perspective": n166 [The postmodern doctrines of Fish and Rorty] fostered an atmosphere in which it became harder
to say that an idea was beyond the pale of rational thought. At its most radical it contended that there was no bedrock thing

such as experience. . . . Because deconstructionism argued that experience


was relative and nothing was fixed, it created an atmosphere of
permissiveness toward questioning the meaning of historical events and made
it hard for its proponents to assert that there was anything "off limits" for this
skeptical approach. n167 Thus, those who deny that the Holocaust occurred are, in
an epistemologically plural world, as entitled to demand public recognition of
their beliefs as are the creationists, the Afrocentrists, and all the others who reject the epistemology of the Enlightenment. They can demand -- and many defenders of
epistemological pluralism, if not current case law, would support such demands from other groups -- that textbooks should reflect the existence and potential soundness of denial theories;
that if the public schools teach the Holocaust as a historical event, they must also teach that it may not have happened; that if parents object to their children being taught what they consider
a historical fabrication, the [*484] children should be excused from history class; that if a state university funds student speech on historical topics generally it must also fund a group

Page 911 of 1481


Lipstadt sees Holocaust denial as "a threat to all those who
dedicated to denying the Holocaust.

believe in the ultimate power of reason ," n168 but the converse is also true: the
denial of the ultimate power of reason is a threat to those who would keep
the memory of the Holocaust alive.

Page 912 of 1481


Page 913 of 1481
Provisional Truth: 1AR
THE ALTERNATIVE CREATES UNDECIDABILITY, CONTINGENT UNIVERSALITIES, AND
PROVISIONAL TRUTHS

Arrigo & Williams, Their Authors from the Califonria School of Professional Psychology, 2K (Bruce &
Christopher, The (Im)Possibility of Democratic Justice and the Gift of the Majority On Derrida, Deconstruction, and
the Search for Equality, Journal of Contemporary Criminal Justice, Volume 16, Number 3, August)

This article conceptually explores the problem of democratic justice in the form of
legislated equal rights for minority citizen groups. Following Derridas critique of Western logic and thought, at issue is the (im)possibility of justice for under-
and nonrepresented constituencies. Derridas socioethical treatment of justice , law, hospitality, and community

suggests that the majority bestows a gift (ostensible sociopolitical empowerment); however, the ruse of this gift is that the giver
affirms an economy of narcissism and reifies the hegemony and power of the majority. This article concludes by speculating on

the possibility of justice and equality informed by an affirmative postmodern


framework. A cultural politics of difference, contingent universalities,
undecidability, dialogical pedagogy, border crossings, and constitutive thought
would underscore this transformative and deconstructive agenda .

Page 914 of 1481


Page 915 of 1481
**Global/Local**

Page 916 of 1481


Micropolitics Only Benefit
Privileged
ATTENTION GIVEN TO MICROPOLITICS OBSCURES ACTUAL SUCCESSES OF
DISFAVORED GROUPS AND ONLY BENEFITS THE PRIVILEGED

Patricia Hill Collins, Prof. of Sociology - Dept. of African-American Studies at Univ. of Cincinnati, Fighting Words,
1998, 135-7

In this academic context, postmodern treatment of power relations suggested by the rubric of decentering may provide some relief to intellectuals who wish to resist oppression
in the abstract without decentering their own material privileges. Current preoccupations with hegemony and microlevel, local politicstwo emphases within postmodern
treatments of powerare revealing in this regard. As the resurgence of interest in Italian Marxist Antonio Gramscis work illustrates (Forgacs 1988), postmodern social theorists
seem fascinated with the thesis of an all-powerful hegemony that swallows up all resistance except that which manages to survive within local interstices of power. The ways in
which many postmodernist theorists use the heterogeneous work of French philosopher Michel Foucault illustrate these dual emphases. Foucaults sympathy for disempowered
people can be seen in his sustained attention to themes of institutional power via historical treatment of social structural change in his earlier works (see., e.g., Foucaults
analysis of domination in his work on prisons [979] and his efforts to write a genealogy linking sexuality to institutional power [ii98oa]). Despite these emphases, some
interpretations of his work present power as being everywhere, ultimately nowhere, and, strangely enough, growing. Historical context is minimizedthe prison, the Church,
France, and Rome all disappearleaving in place a decontextualized Foucauldian theory of power. All of social life comes to be portrayed as a network of power relations
that become increasingly analyzed not at the level of large-scale social structures, but rather at the local level of the individual (Hartsock 1990). The increasing attention given
to micropolitics as a response to this growing hegemony, namely, politics on the local level that are allegedly plural, multiple, and fragmented, stems in part from this reading of
history that eschews grand narratives, including those of collective social movements. In part, this tendency to decontextualize social theory plagues academic social theories
of all sorts, much as the richly textured nuances of Marxs historical work on class conflict (see, e.g., The Eighteenth Brumaire of Louis Bonaparte [1963]) become routinely
recast into a mechanistic Marxist theory of social class. This decontextualization also illustrates how academic theories empty out the more political and worldly substance of
postmodern views of power that overemphasize
radical critiques (West 1993, 41) and thus participate in relations of ruling. In this sense,

local politics provide a seductive mix of appearing to challenge oppression while secretly believing that
hegemony and

such efforts are doomed. Hegemonic power appears as ever expanding and invading. It may even attempt to annex the counterdiscourses that have
developed, oppositional discourses such as Afrocentrism, postmodernism, feminism, and Black feminist thought. This is a very important insight. However, there is a difference
between being aware of the power of ones enemy and arguing that such power is so pervasive that resistance will, at best, provide a brief respite and, at worst, prove
This emphasis on power as being hegemonic and seemingly absolute, coupled with a belief in local
ultimately futile.

resistance as the best that people can do, flies in the face of actual, historical successes. African-
Americans, women, poor people, and others have achieved results through social movements, revolts,
revolutions, and other collective social action against government, corporate, and academic structures. As
James Scott queries, What remains to be explained is why theories of hegemonyhaveretained an enormous intellectual appeal to social scientists and historians (1990,

emphasizing hegemony and stressing


86). Perhaps for colonizers who refuse, individualized, local resistance is the best that they can envision. Over

nihilism not only does not resist injustice but participates in its manufacture. Views of power grounded exclusively in notions
of hegemony and nihilism are not only pessimistic, they can be dangerous for members of historically marginalized groups. Moreover, the emphasis on local

versus structural institutions makes it difficult to examine major structures such as racism, sexism, and
other structural forms of oppression.7 Social theories that reduce hierarchical power relations to the level of
representation, performance, or constructed phenomena not only emphasize the likelihood that resistance
will fail in the face of a pervasive hegemonic presence, they also reinforce perceptions that local, individualized micropolitics constitutes the most effective terrain of
struggle. This emphasis on the local

If politics becomes reduced to


dovetails nicely with increasing emphasis on the personal as a source of power and with parallel attention to subjectivity.

the personal, decentering relations of ruling in academia and other bureaucratic structures seems
increasingly unlikely. As Rey Chow opines, What these intellectuals are doing is robbing the terms of oppression of their critical and
oppositional import, and thus depriving the oppressed of even the vocabulary of protest and rightful demand (1993, 13). Viewing
decentering as a strategy situated within a larger process of resistance to oppression is dramatically different from perceiving decentering as an academic theory of how
scholars should view all truth. When weapons of resistance are theorized away in this fashion, one might ask, who really benefits? Versions of decentering as presented by
postmodernism in the American academy may have limited utility for African-American women and other similarly situated groups. Decentering provides little legitimation for

the way to be legitimate within


centers of power for Black women other than those of preexisting marginality in actual power relations. Thus,

postmodernism is to claim marginality, yet this same marginality renders Black women as a group
powerless in the real world of academic politics. Because the logic of decentering opposes constructing new centers of any kind, in effect the
stance of critique of decentering provides yet another piece of the new politics of containment. A depoliticized decentering disempowers Black

women as a group while providing the illusion of empowerment. Although individual African-American women intellectuals may
benefit from being able to broker the language and experiences of marginality in a commodified American academic marketplace, this in no way substitutes for sustained
groups already privileged under hierarchical power
improvement of Black women as a group in these same settings. In contrast,

relations suffer little from embracing the language of decentering denuded of any actions to decenter
actual hierarchical power relations in academia or elsewhere. Ironically, their privilege may actually
increase.

Page 917 of 1481


Page 918 of 1481
Localism Causes Oppression (1/2)

THE LOCAL IS UTILIZED TO FULFILL THE INTERESTS OF ELITES

David Engel, SUNY Buffalo School of Law, Injury and Identity: The Damaged Self in Three Cultures, April 24,
1999, http://homepage1.nifty.com/tkitamura/engel.html, accessed 11/16/02

Law plays a distinctive role in relation to the social settings in which injuries occur. The judge may act as
arbiter, choosing among the contending realities in the locales where conflict arises, but the judges re-
presentation of reality has its own distinctive qualities. This was partially evident in Vosburg v. Putney,
with its explication of the norms of the playground as well as the classroom. It was fully evident in the
runaway ox-cart case, where the judge sustained familiar village concepts of remedy while subverting
familiar village concepts of marriage and family. The judge is not just an arbiter; the judge tells his or her
own story. The judge is a mythmaker. The court projects a version of reality back upon the social setting
from which the case emerges, and this refashioned version of local truths inevitably redefines them.
Judges, like politicians, understand the power of the myth of the local. They may attempt to legitimate
their pronouncements about order and responsibility by relying on romanticized images of schools,
families, and communities, selectively rendered and stripped of contestation and ambiguity. Although
citizens differ about particular local norms or practices, most would respond to the importance of local-
ness itself as a value in opposition to intrusions by big government, big business, or alien persons into
local settings. By using, and inevitably distorting, the norms and practices they discover in local settings,
the spokespersons for state law shield themselves from the accusation that they are intruding on the
common sense of ordinary people. They legitimate their decisions as mere reflections of a mythic local
community. Of course, the laws selectivity and distortion shapes identities in particular ways. Law
transforms symbols and images drawn from local settings and redeploys them as authoritative
pronouncements that can potentially change the very settings from which they are drawn.

Page 919 of 1481


Page 920 of 1481
Localism Causes Oppression
(2/2)
PRIVILEGING THE LOCAL CEMENTS CULTURAL OPPRESSION

Aihwa Ong, Professor of Anthropology at UC Berkeley, Flexible Citizenship: The Cultural Logic of Transnationality,
1999, p. 33-34

More broadly, postcolonial theorists focus on recovering the voices of subjects silenced by patriarchy and
colonial rule (The Empire Writes Back is the title of one popular collection); they assume that all
contemporary racial, ethnic, and cultural oppressions can all be attributed to Western colonialisms.
American appropriations of postcolonial theory have created a unitary discourse of the postcolonial that
refers to highly variable situations and conditions throughout the world; thus, Gayatri Spivak is able to talk
about the paradigmatic subaltern woman, as well as New World Asians (the old migrants) and New
Immigrant Asians (often model minorities) being disciplinarized together? Other postcolonial feminists
also have been eager to seek structural similarities, continuities, conjunctures, and alliances between the
postcolonial oppressions experienced by peoples on the bases of race, ethnicity, and gender both in
formerly colonized populations in the third world and among immigrant populations in the United States,
Australia, and England. Seldom is there any attempt to link these assertions of unitary postcolonial
situations among diasporan subjects in the West to the historical structures of colonization,
decolonization, and contemporary developments in particular non-Western countries. Indeed, the term
postcolonial has been used to indiscriminately describe different regimes of economic, political, and
cultural domination in the Americas, India, Africa, and other third-world countries where the actual
historical experiences of colonialism have been very varied in terms of local culture, conquest, settlement,
racial exploitation, administrative regime, political resistance, and articulation with global capitalism. In
careless hands, postcolonial theory can represent a kind of theoretical imperialism whereby scholars
based in the West, without seriously engaging the scholarship of faraway places, can project or speak
for postcolonial situations elsewhere. Stuart Hall has warned against approaches that universalize racial,
ethnic, and gender oppressions without locating the actual integument of powerin concrete
institutions. A more fruitful strand of postcolonial studies is represented by subaltern scholars such as
Partha Chatterjee, who has criticized the Indian national projects, which are based on Western models of
modernity and bypass many possibilities of authentic, creative, and plural development of social
identities, including the marginalized communities in Indian society. He suggests that an alternative
imagination that draws on narratives of community would be a formidable challenge to narratives of
capital. This brilliant work, however, is based on the assumption that both modernity and capitalism are
universal forms, against which non-Western societies such as India can only mobilize pre-existing cultural
solidarities such as locality, caste, tribe, religious community, or ethnic identity. This analytical opposition
between a universal modernity and non-Western culture is rather old-fashioned it is as if Chatterjee
believes the West is not present in Indian elites who champion narratives of the indigenous community.
Furthermore, the concept of a universal modernity must be rethought when, as Arif Dirlik observes, the
narrative of capitalism is no longer the narrative of the history of Europe; non-European capitalist
societies now make their own claims on the history of capitalism. The loose use of the term the
postcolonial, then, has had the bizarre effect of contributing to a Western tradition of othering the Rest;
it suggests a postwar scheme whereby the third world was followed by the developing countries,
which are now being succeeded by the postcolonial. This continuum seems to suggest that the further
we move in time, the more beholden non-Western countries are to the forms and practices of their
colonial past. By and large, anthropologists have been careful to discuss how formerly colonized societies
have developed differently in relation to global economic and political dominations and have repositioned
themselves differently vis-a-vis capitalism and late modernity. By specifying differences in history, politics,

Page 921 of 1481


and culture, anthropologists are able to say how the postcolonial formation of Indonesia is quite different
from that of India, Nicaragua, or Zaire.

Page 922 of 1481


Page 923 of 1481
Globalism Key to Resistance
GLOBAL ORGANIZATION CAN HIGHLIGHT AND UNITE THE STRUGGLES OF MANY
LOCALITIES

Maximilian Forte, Department of Anthropology, University of Adelaide, Renewed Indigeneity in the Local-Global
Continuum and the Political Economy of Tradition: The Case of Trinidads Caribs and The Caribbean Organization of
Indigenous People, March 18-21, 1998, http://www.centrelink.org/renewed.html, accessed 11/16/02

One may also begin to see how local and global levels or arenas each acts as restraints, parameters and
motivations that both condition and inspire, constrain and enable the revitalization of Carib identification.
Globalized terms, images, motifs and practices of indigeneity act as fund of materials which are engaged
and sifted through by Caribbean Amerindians to define who they were, are, and who they are to be (see
Robertson 1992). Mere association with and invocation of the names of large international indigenous
bodies can serve as symbolic capital for these groups. To a certain extent, the material for a reformulated
and revived indigenous identity comes from and is negotiated out of a globalized aboriginality and a
globally organized political economy of tradition where even the most seemingly inane or trivial
traditional practices take on large, global significance as an alternative to the evils of modernization. I
see these trends as adding, paradoxically, to the Caribs own authenticity claims paradoxical because they
emphasize the icons and language of local continuity (despite the various global indigenous elements
drafted into the process of recreating the meaning of those elements said to be locally continuous), and
yet, in the "public eye" in Trinidad their indigenous qualities may be enforced insofar as they are endorsed
by "foreign" trends and institutions, thus rendering them more serious, more "real." I see the Caribs self-
understanding of indigeneity as very much a "work-in-progress," an indigenous "site under
(re)construction," within a growing global network. We might even tentatively begin to posit that this
global network is the indigene, that is, adaptable to and derivative of a variety of local platforms. A
central process in the reconstruction of Carib indigeneity is the organization, reproduction and display of
"Carib culture" for a national audience and for the tourist market (for a similar case, see Friedman 1990).
Presentation itself thus becomes an instrument in the constitution of selfhood. Processes of globalization
help to "lift" the discourse of local aboriginal issues and struggles to a global plane. Aboriginal global
organization helps in itself to further define aboriginality. International organizations, whether inter-statal
or indigenous, assist in the creation of an "international personality" of indigenous peoples. The Caribs
increasingly come to define themselves, in part, within and through the international network of
indigenous organizations.

Page 924 of 1481


Page 925 of 1481
Alternative Kills Movements
THERE IS NO DIFFERENCE BETWEEN THE GLOBAL AND THE LOCAL CREATING
SUCH A FALSE DICHOTOMY DESTROYS THE ABILITY TO UNDERSTAND THE POWER
OF SOCIAL MOVEMENTS

Stehr, Sociologist and Senior Research Fellow of the Sustainable Development Research Institute @ U of
British Colombia, 2K1 (Nico, The Fragility of Modern Socities: Knowledge and Risk in the Information
Age, P.99100)

I also do not accept any rigid


My own perspective on the effects of globalization, however, is not a post-modern one.

dichotomy between local (or regional) and global phenomena, according to


which every characterization of the social sphere must either take on strictly local
attributes or eliminate them altogether in the wake of the effects of
globalization. As long as the local/global axis is dichotomized in such a rigid and
asymmetric manner, the theoretical and empirical dilemma will be that the
political, social and economic processes have to be categorized under the heading
either of global attributes or of authentic local circumstances that bear at best
only a superficial resemblance to phenomena alleged to be global in scope. As a result, a
prohibition against a synchronism the incommensurable' (Max Frisch) or the 'non-contemporaneity of the contemporaneous' (Wilhelm Finder) must be strictly observed

To follow such prohibitions is to miss out on significant societal


(see Mannheim, (1928; 1993: 358).

processes that result from struggles between local and global phenomena, or that
constitute emerging social structures, cultural processes of political developments
that succeed in joining these forces in novel ways. Cultural manifestations are
never created ex nihilo and strictly in accordance with either local or global reference
(Tomlinson, 1999). But it is also unrealistic to expect that forms of knowledge disappear

altogether in the wake of the effects of globalization without leaving any trace (see Stehr, 1991),
or that well-established patterns of social conduct vanish without signs of lasting
influence. With globalization and fragmentation proceeding concurrently, global
192

as well as particularistic political developments generate and heighten conflicts (see


Schmidt, 1995), for example between claims of universal human rights and particularistic identities based on language, religion, nationality, race and ethnicity (see

political opportunities that result from extending the boundaries of


Benhatib, 1999). Indeed, images of

governance beyond the nation-state and ominous threats that issue from the same
developments for the nation-state appear to co-exist side-by-side in many of the
accounts of the political implications of globalization.

Page 926 of 1481


Page 927 of 1481
Rejection Bad
THE CRITIQUE PARALYZES. CRITICISM MUST COME FROM WITHIN STRUCTURES OF
GLOBAL POWER

Arun Agrawal, assistant professor of political science at Yale University, Peace & Change, Oct 96, Vol. 21 Issue 4,
p464, 14p.

The stance of this reviewer may be summarized as "I will engage, I must critique"--in contrast to the
poststructuralist position of "I will critique, I will reject." Throughout this essay, I have tried to highlight
the two dilemmas inherent in adopting a poststructuralist stance. One is led either to a position that
repeats one's initial assumptions, or one is forced into contradictions that result from questioning
metanarratives. In response, I suggest two small strategic shifts for poststructuralist scholars, the first of
which can already be witnessed in the work of Stacy Leigh Pigg.[9] Instead of avowing an explicit
commitment to poststructuralism and calling for a repudiation of "development," it might be far more
fruitful to examine the ways in which attempts by the state to foster development are often used as
instruments of legitimation and extension of political control, yet also often engender resistance and
protest. It was Foucault, after all, who pointed to the positive as well as the negative aspects of
power.[10] A second productive move might be to accept the impossibility of questioning all
metanarratives and instead to rethink how development can be profitably contested from within as well
as from outside. Persistent criticisms of "development" are indispensable; calls to go beyond it make
sense primarily as signifiers of romantic utopian thinking. In posing the dualisms of local and global,
indigenous and Western, traditional and scientific, society and state--and locating the possibility of
change only in one of these opposed pairs--one is forced to draw lines that are potentially ridiculous, and
ultimately indefensible.[11] Development, like progress, rationality, or modernity, may be impossible to
give up. Harboring the seeds of its own transformation, it may be far more suited to co-optation than
disavowal. Rather than fearing the co-optation by "development" of each new strategy of change, it may
be time to think about how to co-opt "development." "[R]eversing, displacing, and seizing the apparatus
of value-coding"[12] is not just the task of the postcolonial position; it is the impossible task of all critical
positions.

Page 928 of 1481


A2 Localism

THERES GOOD GLOBALISM AND BAD GLOBALISM. WE MUST SUPPORT THE GOOD
TO OVERCOME THE BAD.

Norman Solomon, executive director of the Institute for Public Accuracy, 2/20/2003,
http://www.fair.org/media-beat/030220.html, accessed 2/23/03

One of the big media buzzwords to emerge in recent years is "globalization." By now, we're likely to know
what it means. That's unfortunate -- because at this point the word is so ambiguous that it doesn't really
mean much of anything. News outlets have reported that key international pacts like NAFTA and the
World Trade Organization gained U.S. approval during the 1990s because most politicians in Washington
favor "globalization." According to conventional media wisdom, those globalizers want to promote
unfettered communication and joint endeavors across national boundaries. Well, not quite. These days,
at the White House and on Capitol Hill, the same boosters of "globalization" are upset about certain types
of global action -- such as the current grassroots movement against a war on Iraq. For the most part, the
same elected officials and media commentators who have applauded money-driven globalization are now
appalled by the sight of anti-war globalization. The recent spectacle of millions of people demonstrating
against war on the same day around the world was enough to cause apoplexy at the White House. That's
consistent with a recurring pattern: "Pro-globalization" forces are unhappy to see the globalizing of
solidarity for labor rights, economic justice, the environment and alternatives to war. A similar
contradiction belies the media image of "anti-globalization" activists as foes of internationalism who want
to rigidify national boundaries, reinforce isolation and prevent worldwide interactions. On the contrary,
advocates for human rights, environmental protection and peace -- while largely opposing global
superstructures like NAFTA and the WTO -- have been busily creating ways to work with like-minded
people all over the planet. The form of "globalization" deemed worthy of the name by media is corporate
globalization, which gives massive capital even more momentum to flatten borders and run roughshod
over national laws. Deluging every country with Nikes, Burger Kings and ATMs is presumptively indicative
of progress, no matter how bad the working conditions, how unhealthy the products or how unjust the
economic consequences. Meanwhile, fans of "globalization" routinely contend that protection of labor
rights or the environment amounts to unfair restraint of trade, retrograde protectionism and antiquated
resistance to "reforms." By itself, "globalization" is much too simplistic a word to tell us anything. The
term is so murky that we may need to discard it, or at least develop some new phrases to bring realities
into focus. Today, the war-crazed Bush administration and the bipartisan majority of enablers in Congress
are fervent proponents of what might be called "isolationist intervention." Sure, the present-day
American leaders proclaim their global vision and declare that they want to engage with the world, but on
their own terms -- with the U.S. government reserving the right to determine its policies in isolation from
any nation that fails to offer subservient support. With hefty corporate backing, they insist that the United
States has the right to intervene militarily overseas. Why? Because they say so. The gist of this approach
to "globalization" was well expressed by the glib pundit Thomas Friedman, whose 1999 book "The Lexus
and the Olive Tree" lauded the tandem roles of corporate capitalism and American militarism. "The
hidden hand of the market will never work without a hidden fist," he wrote. "McDonald's cannot flourish
without McDonnell Douglas, the designer of the U.S. Air Force F-15. And the hidden fist that keeps the
world safe for Silicon Valley's technologies to flourish is called the U.S. Army, Air Force, Navy and Marine
Corps." This veiled hand-and-fist stance is being actively rejected by millions of people marching through
cities in many parts of the world. And the leaders of numerous countries are giving voice to that rejection.
Speaking to the U.N. Security Council on Feb. 18, Malaysia's prime minister Mahathir Mohamed -- the
incoming chair of the Non-Aligned Movement -- combined realism with idealism. "We have no military or
financial strength," he said, "but we can join the world movement to oppose war on moral grounds." The
globalization of that movement is something to behold. And nurture.

Page 929 of 1481


Permutation

GLOBAL PERSPECTIVES ARE SUCCESSFULLY REDEPLOYED FOR LOCAL ENDS

Michel Foucault, Society Must Be Defended: Lectures at the College De France 1975-1976,2003, p. 6

So I would say: for the last ten or fifteen years, the immense and proliferating criticizability of things,
institutions, practices, and discourses; a sort of general feeling that the ground was crumbling beneath
our feet, especially in places where it seemed most familiar, most solid, and closest [nearest] to us, to our
bodies, to our everyday gestures. But alongside this crumbling and the astonishing efficacy of
discontinuous, particular, and local critiques, the facts were also revealing something that could not,
perhaps, have been foreseen from the outset: what might be called the inhibiting effect specific to
totalitarian theories, or at leastwhat I mean isall-encompassing and global theories. Not that all-
encompassing and global theories havent, in fairly constant fashion, providedand dont continue to
provide tools that can be used at the local level; Marxism and psychoanalysis are living proof that they
can. But they have, I think, provided tools that can be used at the local level only when, and this is the real
point, the theoretical unity of their discourse is, so to speak, suspended, or at least cut up, ripped up, torn
to shreds, turned inside out, displaced, caricatured, dramatized, theatricalized, and so on. Or at least that
the totalizing approach always has the effect of putting the brakes on. So that, if you like, is my first point,
the first characteristic of what has been happening over the last fifteen years or so: the local character of
the critique; this does not, I think, mean soft eclecticism, opportunism, or openness to any old theoretical
undertaking, nor does it mean a sort of deliberate asceticism that boils down to losing as much theoretical
weight as possible. I think that the essentially local character of the critique in fact indicates something
resembling a sort of autonomous and noncentralized theoretical production, or in other words a
theoretical production that does not need a visa from some common regime to establish its validity.

Page 930 of 1481


Page 931 of 1481
**Habeas Corpus**

Page 932 of 1481


Habeas Corpus Answers: 2AC
(1/3)
FIRST, PERM DO BOTH
YOU CAN ACKNOWLEDGE THAT HABEAS VIOLATIONS ELSEWHERE ARE BAD AND
STILL GRANT IT TO ENEMY COMBATANTS

SECOND, CRITICIZING REPRESENTATIONS DOESNT PRECLUDE THE NEED FOR


CONCRETE ACTION

Richard Rorty, Professor of Humanities, University of Virginia, Truth, Politics, and Postmodernism, Spinoza Lectures,

1997, p. 51-2

This distinction between the theoretical and the practical point of view is often drawn by Derrida, another writer who enjoys demonstrating that something very important
meaning, for example, or justice, or friendship is both necessary and impossible. When asked about the implications of these paradoxical fact, Derrida usually replies that

the paradox doesn't matter when it comes to practice. More generally, a lot of the writers who are labeled
`post-modernist; and who talk a lot about impossibility, turn out to be good experimentalist social democrats when it
comes to actual political activity. I suspect, for example, that Gray, Zizek, Derrida and I, if we found ourselves citizens of the same country, would all be
voting for the same candidates, and supporting the same reforms. Post-modernist philosophers have gotten a bad name because of their paradox-mongering habits, and their
constant use of terms like `impossible; `self-contradictory' and `unrepresentable'. They have helped create a cult of inscrutability, one which defines itself by opposition to the
Enlightenment search for transparency - and more generally, to the `metaphysics of presence; the idea that intellectual progress aims at getting things clearly illuminated,

. I am all for getting rid of the metaphysics of presence, but I think that the rhetoric of
sharply delimited, wholly visible

impossibility and unrepresentability is counterproductive overdramatization. It is one thing to say that we need to get rid of the
metaphor of things being accurately represented, once and for all, as a result of being bathed in the light of reason. This metaphor has created a lot of headaches for
philosophers, and we would be better off without it. But that does not show that we are suddenly surrounded by unrepresentables; it just shows that `more accurate
Even if we agree that we shall never have what Derrida calls "a full
representation' was never a fruitful way to describe intellectual progress.

presence beyond the reach of play"; our sense of the possibilities open to humanity will not have
changed. We have learned nothing about the limits of human hope from metaphysics, or from the philosophy of history, or from psychoanalysis. All that we have learned
from `post-modern' philosophy is that we may need a different gloss on the notion of `progress' than the rationalistic gloss which the Enlightenment offered. We have

been given no reason to abandon the belief that a lot of progress has been made by carrying out the
Enlightenment's political program. Since Darwin we have come to suspect that whether such progress is made will be largely a matter of luck. But we
have been given no reason to stop hoping to get lucky.

THIRD, SPECIFIC SOLVENCY TRUMPS - PLAN SOLVES WORSE INJUSTICE BY


GUARANTEEING DUE PROCESS TO ENEMY COMBATANTS AND ENDING INDEFINITE
DETAINMENT AND TORTURE UNDER MILLIGAN. CROSS-APPLY TRIBE AND KATYAL

FOURTH, THEIR LINK EV IS TERRIBLE IT JUST SAYS THERE ARE TWO KINDS OF
HABEAS WITHOUT SHOWING WHY WE USE ONE OR THE OTHER OR GIVING A
REASON THAT THATS BAD

Page 933 of 1481


Page 934 of 1481
Habeas Corpus Answers: 2AC
(2/3)
FIFTH, APPROPRIATING THE OTHER VIOLENTLY SEIZES THE RIGHT TO SPEAK FOR
SELFISH ENDS

Routledge 96
[Antipode]

The issue of representation is a vexed one which has received much attention within the social
sciences. For example, in discussing the academic strategy of polyphony, Crang (1992) raises
issues of how the voices of others are (re)presented; the extent to which these voices are
interwoven with persona of narrator the degree of authorial power regarding who initiates
research, who decides on textual arrangements, and who decides which voices are heard; and
the power relations involved in the cultural capital conferred by specialist knowledge.
Moreover, Harrison (quoted in McLaren 1995 240) argues that polyphony can end up being
aform of romantic ventroloquism creating the magical notion of the Others coming to voice.
These questions have important political implications for research which must be negotiated
according to the specific circumstances of a particular project. It is all too easy for academics to
claim solidarity with the oppressed and act as relays for their voices within social scientific
discourse. This raises the danger of an uncritical alignment with resisters on the assumption that
they know all there is to know without the intervention of intellectuals; and hence an
academics role becomes that of helping them seize the right to speak.

SIXTH, THE STATUS QUO IS WORSE COMBATANTS HAVE NO RIGHTS. PLAN AT


LEAST GIVES THEM TO SOME PEOPLE

SEVENTH, THEIR BRIGHT EV INDICTS THE ALTERNATIVE ACTION IS THE ONLY


REMEDY TO INDIFFERENCE. PLAN IS KEY TO TAKING A SIDE AND DOING
SOMETHING

EIGHTH, TURN- UPHOLDING LEGAL PRINCIPLES PROVES THE LAWS FRAUDULENCE


AND HOLDS IT ACCOUNTABLE

Vclav Havel, playwright, political prisoner, and president elect of Czechoslovakia, 1986 (Living in Truth, p. 137-
38)

Page 935 of 1481


appeal to the laws not just to the laws concerning human rights, but to all laws does not mean at all that
A persistent and never-ending

those who do so have succumbed to the illusion that in our system the law is anything other than what it is. They
are well aware of the role it plays. But precisely because they know how desperately the system depends on it on the noble version of the law, that is they also know how
enormously significant such appeals are. Because the system cannot do without the law, because it is hopelessly tied down by the necessity of pretending the laws are

Demanding that the laws be upheld is thus an act of living within the
observed, it is compelled to react in some way to such appeals.

truth that threatens the whole mendacious structure at its point of maximum mendacity. Over and over again,
such appeals make the purely ritualistic nature of the law clear to society and to those who inhabit its power
structures. They draw attention to its real material substance and thus, indirectly, compel all those who take refuge behind the law
to affirm and make credible this agency of excuses, this means of communication, this reinforcement of the social arteries outside of which their will
could not be made to circulate through society. They are compelled to do so for the sake of their own consciences, for the impression they make on outsiders, to maintain
themselves in power (as part of the systems own mechanism of self-preservation and its principles of cohesion), or simply out of fear that they will be reproached for being
clumsy in handling the ritual. They have no other choice: because they cannot discard the rules of their own game, they can only attend more carefully to those rules. Not to
react to challenges means to undermine their own excuse and lose control of their mutual communications system.

To assume that the laws are a mere facade, that they have no validity and that therefore it is pointless to appeal to them would mean to
go on reinforcing those aspects of the law that create the facade and the ritual. It would mean confirming the law
as an aspect of the world of appearances and enabling those who exploit it to rest easy with the cheapest (and therefore the most

mendacious) form of their excuse. I have frequently witnessed policemen, prosecutors or judges if they were dealing with an experienced Chartist or a
courageous lawyer, and if they were exposed to public attention (as individuals with a name, no longer protected by the anonymity of the apparatus) suddenly and anxiously

the very existence


begin to take particular care that no cracks appear in the ritual. This does not alter the fact that a despotic power is hiding behind that ritual, but

of the officials anxiety necessarily regulates, limits and slows down the operation of that despotism.

Page 936 of 1481


Page 937 of 1481
Habeas Corpus Answers: 2AC
(3/3)
NINTH, THIS ISNT OFFENSE, ITS A BAD PMN THERE WILL ALWAYS BE INJUSTICE
THAT PLAN DOESNT SOLVE. USING THAT TO VOTE NEG DEJUSTIFIES DOING
ANYTHING, CREATING NIHILISM

TENTH, THE ALTERNATIVE OPTS FOR INACTION IN THE FACE OF DOMINATION


ONLY POLICY DISCUSSIONS CAN REORIENT INTELLECTUALS TOWARDS FIGHTING
INJUSTICE

SAID (University Professor, Columbia University) 94


[Edward W., The Intellectuals and the War, The Politics of Dispossession: The Struggle

for Palestinian Self-Determination, 1969-1994, New York: Vintage, p. 316-19]

HARLOW: What are the political, intellectual, and cultural imperatives for combating this agenda? In 1967 Chomsky wrote the essay Responsibility of Intellectuals. What
would be the main component of such an essay today?

jargonistic postmodernisms that now dot the landscape. They are worse than useless.
SAID: One would have to pretty much scuttle all the jaw-shattering

are neither capable of understanding and analyzing the power structure of this country nor are they
They

capable of understanding the particular aesthetic merit of an individual work of art. Whether you call it deconstruction or
postmodernism or poststructuralism or post-anything, they all represent a sort of spectacle of giving back tickets that the entrance and saying, were really out of it. We want to
check into our private resort and be left alone. [317]

Reengagement with intellectual processes has very little to do with being politically correct, or citing
fashionable names, or striking acceptable poses, but rather having to do with a return in a way to a kind of old-
fashioned historical, literary, and above all, intellectual scholarship based upon the premise that human beings, men and
women, make their own history. And just as things are made, they can be unmade and re-re-remade. That sense of intellectual and political and citizenry
empowerment is what I think the intellectual class needs.

Theres only one way to anchor oneself, and that is by affiliation with a cause, with a political movement. There has to be some identification, not with the powers that

; there has to be an affiliation with matters involving


be, with the Secretary of State or the great leading philosopher of the time or sage

justice, principle, truth, conviction. Those dont occur in a laboratory or a library. For the American intellectual, that simply means, at bottom, in a
globalized environment, that there is today one superpower, and the relationship between the United States and

the rest of the world, based upon profit and power, has to be altered from an imperial one to one of
coexistence among human communities that can make and remake their own histories together. This
seems to me to be the number-one priority---theres nothing else.
An American has a particular role. If youre an anthropologist in America, its not the same thing as being an anthropologist in India or France; its a qualitatively
different thing.

HARLOW: Were both professors in English departments, despite the fact that the humanities have been quite irresponsible, unanswerable

SAID: Not the humanities. The professors of humanities.

HARLOW: Well, OK, the professors, but there is this question

SAID: I take the general view that, for all its inequity, for all its glaring faults and follies, the university in this society remains a relatively utopian place, a place of great privilege .
There needs to be some sense of the university as a place in which these issues are not, because it is that
kind of place, trivialized. Universities cannot afford to become just a platform for a certain kind of
narcissistic specialization and jargon. What you need is a regard for the product of the human mind. And

Page 938 of 1481


thats why Ive been very dispirited, I must tell you, but aspects of the great Western canon debate, which really suggest that the oppressed of the world, in wishing to be
heard, in wishing their work to be recognized, really wish to do dirt on everything else. Thats not the spirit of resistance. We come [318] back to Aime Cesaires line, There is
room for all that at the rendezvous of victory. Its not that some have to be pushed off and demeaned and denigrated. The question is not whether we should read more black
literature or less literature by white men. The issue is excellence---we need everything, as much as possible, for understanding the human adventure in its fullest, without
resorting to enormous abstractions and generalizations, without replacing Euro-centrism with other varieties of ethnocentrism, or say, Islamo-centrism or Afro-centrism or gyno-
centrism. Is it a game of substitutions? Thats where intellectuals have to clarify themselves.

HARLOW: I agree, but at least within certain university contexts there have been lately two major issues: the Gulf War and multiculturalism. I have not seen any linkage
between the two.

SAID: The epistemology and the ethic of specialization have been accepted by all. If youre a literature professor, thats what you talk about. And if youre an education
specialist, thats what you talk about. The whole idea of being in the university means not only respect for what others do, but respect for what you do. And the sense that they
all are part of a community. The main point is that we ascribe a utopian function to the intellectual. Even inside the university, the prevalence of norms based upon domination
and coercion is so strong because the idea of authority is so strong---whether its authority derived from the nation-state, from religion, from the ethnos, from tradition---is so
powerful that its gone relatively unchallenged, even in the very disciplines and studies that we are engaged in. Part of intellectual work is understanding how authority is

And if you can understand that, they your work is conducted in


formed. Like everything else, authority is not God-given. Its secular.

such a way as to be able to provide alternatives to authoritative and coercive norms that dominate so
much of our intellectual life, our national and political life, and our international life above all.
HARLOW: What can alternative publications do to interrupt that particular way of presenting authority?

SAID: One is to remind readers that there are always other ways of looking at the issue---whatever it happens to be---than those that are officially credentialed. Second, one of
the things that one needs to do in intellectual enterprises is to---Whitehead says somewhere---always try to write about an author keeping in mind what he or she might say of
what youre writing. To adapt from that: some sense in which your constituency might be getting signals about what youre doing. The agenda isnt set only by you; its set by
others. You cant represent the others, but you can take them into account by soliciting their attention. Let such a publication be a place in which its pages that which is
occluded or suppressed or has disappeared from the consciousness of the West, of the intellectual, can be allowed to appear. Third, some awareness of the methodological
issues involved, and the gathering of information, the production of scholarship, the relationship between scholarship and knowledge. The great virtue of these journals is that
they are not guided by professional norms. Nobody is going to get tenure out of writing for these journals. And nobody is trying to advance in a career by what he or she does
there. So that means therefore that one can stand back and look at these things and take questions having to do with how people know things. In other words, a certain
emphasis on novelty is important and somewhat lacking. You dont want to feel too virtuous in what you are doing: that Im the only person doing this, therefore, I must continue
doing it. Wit is not such a bad thing.

Page 939 of 1481


Page 940 of 1481
**Habermas**

Page 941 of 1481


Habermas Answers: 2AC
HABERMAS HAS NOTHING NEW TO OFFER

David E. McClean, New School University, The Cultural Left and the Limits of Social Hope, Presented at the 2001 Annual Conference of the Society for the Advancement of American
Philosophy, www.american-philosophy.org/archives/2001%20Conference/Discussion%20papers/david_mcclean.htm.

Take Habermas, whose writings are admittedly the most relevant of the group. I cannot find in Habermas's lengthy narratives regarding communicative
very much more than I have found in the Federalist Papers, or in Paine's
action, discourse ethics, democracy and ideal speech situations

Common Sense, or in Emerson's Self Reliance or Circles. I simply don't find the concept of uncoerced and fully informed

communication between peers in a democratic polity all that difficult to understand, and I don't much see
the need to theorize to death such a simple concept, particularly where the only persons that are apt to
take such narratives seriously are already sold, at least in a general sense. Of course, when you are trying to justify yourself in the face of the other
members of your chosen club (in Habermas's case, the Frankfurt School) the intricacy of your explication may have less to do with simple

concepts than it has to do with parrying for respectability in the eyes of your intellectual brethren. But I
don't see why the rest of us need to partake in an insular debate that has little to do with anyone that is
not very much interested in the work of early critical theorists such as Horkheimer or Adorno, and who
might see their insights as only modestly relevant at best. Not many self-respecting engaged political
scientists in this country actually still take these thinkers seriously, if they ever did at all.

Page 942 of 1481


Page 943 of 1481
**Heidegger**

Page 944 of 1481


Ethics Turn
HEIDEGGERS FOCUS ON THE ONTOLOGY IGNORES THE TRANSCENDENT FACE OF
ETHICAL RESPONSIBILITY

Michael Shapiro, professor of political science at the University of Hawaii, Moral Spaces: Rethinking Ethics and

World Politics, ed. by Campbell and Shapiro, 1999, p. 64-65

The primary Levinasian struggle with philosophical discourse is conducted within Heideggerian language
because the philosophical depth of Levinas's ethics of infinite responsibility to alterity is revealed in both
his debts to and departures from Heidegger. Prepositions are crucial here for while Levinas accepts
Heidegger's notion of the individual's intimate connection with alterity, he rejects the Heideggerian
grammar of the self-other relationship. The rejection takes the form of two grammatical shifts enacted in
Levinas's writing, beginning with the change from "with" to "in front of." Whereas for Heidegger the
relationship to the other "appears in the essential situation of Miteinandersein, reciprocally being with
another, Levinas expresses resistance to the "association of side-by-side" that Heidegger's Mit suggests:
"[I]t is not the preposition mitthut should describe the original relationship with the other." It is instead
the in front of, the face-to-face that locates the ethical relation to the other. This grammatical shift to the
face-to-face acknowledges the fundamental separation of the self from the Other. To maintain an ethical
bond with the Other, to maintain the infinity of the Other, is to see the self in its relation to something "it
cannot absorb.

REFLECTION NEGATES THE OTHER: ONLY OUR FRAMEWORK PRECLUDES MURDER

Emmanuel Levinas, Professor of Philosophy at the Sorbonne, 1996, Levinas Basic Philosophical Writings p. 9-10

In relation to beings in the opening of being, comprehension finds a signification for them on the basis of being. In this sense, it does not invoke these beings but only names
them, thus accomplishing a violence and a negation. A partial negation which is violence. This partiality is indicated by the fact that, without disappearing, those beings are in
my power. Partial negation, which is violence, denies the independence of a being: it belongs to me. Possession is the mode whereby a being, while existing, is partially denied.
It is not only a question of the fact that the being is an instrument, a tool, that is to say, a means. It is an end also. As consumable, it is nourishment, and in enjoyment, it offers
itself, gives itself, belongs to me. To be sure, vision measures my power over the object, but it is already enjoyment. The encounter with the other (autrui) consists in the fact
that despite the extent of my domination and his slavery, I do not possess him. He does not enter entirely into the opening of being where I already stand, as in the field of my
freedom. It is not starting from being in general that he comes to meet me. Everything which comes to me from the other (autrui) starting from being in general certainly offers
itself to my comprehension and possession. I understand him in the framework of his history, his surroundings and habits. That which escapes comprehension in the other
(autrui) is him, a being. I cannot negate him partially, in violence, in grasping him within the horizon of being in general and possessing him. The Other (Autrui) is the sole being

whose negation can only announce itself as total: as murder. The Other (Autrui) is the sole being I can wish to kill. I can wish. " And yet this
power is quite the contrary of power. The triumph of this power is its defeat as power. At the very moment when my power
to kill realizes itself, the other (autrui) has escaped me. I can, for sure, in killing attain a goal; I can kill as I hunt or slaughter animals, or as I fell
trees. But when I have grasped the other (autrui) in the opening of being in general, as an element of the world where I stand,

where I have seen him on the horizon, I have not looked at him in the face, I have not encountered his face. The temptation of

total negation, measuring the infinity of this attempt and its impossibility - this is the presence of the face. To be in relation with the other (autrui) face

to face is to be unable to kill. It is also the situation of discourse. If things are only things, this is because the relation with them is established as
comprehension. As beings, they let themselves be overtaken from the perspective of being and of a totality that lends them a signification. The immediate is not an object of
comprehension. An immediate given of con- sciousness is a contradiction in terms. To be given is to be exposed to the ruse of the understanding, to be seized by the mediation
of a concept, by the light of being in general, by way of a detour, "in a roundabout way." To be given is to signify on the basis of what one is not. The relation with the face,
speech, an event of collectivity, is a relation with beings as such, as pure beings. That the relation with a being is the invocation of a face and already speech, a relation with a
certain depth rather than with a horizon - a breach in the horizon - that my neighbor is the being par excellence, can indeed appear somewhat surprising when one is
accustomed to the conception of a being that is by itself insignificant, a profile against a luminous horizon and only acquiring signification in virtue of its presence within this

horizon . The face signifies otherwise. In it the infinite resistance of a being to our power affirms itself
precisely against the murderous will that it defies; because, completely naked (and the nakedness of the face is not a figure
of style), the face signifies itself. We cannot even say that the face is an opening, for this would be to make it relative to an environing plenitude. Can things take
on a face? Is not art an activity that lends faces to things? Does not the facade of a house regard us? The analysis thus far does not suffice for an answer. We ask ourselves all
the same if the impersonal but fascinating and magical march of rhythm does not, in art, substitute itself for sociality, for the face, for speech. To comprehension and
signification grasped within a horizon, we oppose the signifyingness of the face. Will the brief indications by which we have introduced this notion allow us to catch sight of its
role in comprehension itself and of all the conditions which delineate a sphere of relations barely suspected? In any case, that which we catch sight of seems suggested by the

Page 945 of 1481


the encounter
practical philosophy of Kant, to which we feel particularly close. In what way the vision of the face is no longer vision but audition and speech; how

with the face - that is, moral consciousness - can be described as the condition of consciousness tout
court and of disclosure; how consciousness is affirmed as the impossibility of killing; what are the conditions of the
appearance of the face as the temptation and the impossibility of murder; how I can appear to myself as a face; in what manner, finally, the relation with the

other (autrui) or the collectivity is our relation, irreducible to comprehension, with the infinite - these are the themes

that proceed from this first contestation of the primacy of ontology. Philosophical research, in any case,
cannot be content with a mere reflection on the self or on existence. Reflection offers only the tale of a
personal adventure, of a private soul, which returns incessantly to itself, even when it seems to flee itself.
The human only lends itself to a relation that is not a power.

Page 946 of 1481


Ontological Fascism Turn: 2AC
**** ONTOLOGICAL CRITICISM TRADES OFF WITH ONTIC POLITICAL ENGAGEMENT,
ALLOWING ONE TO BECOME COMPLICIT IN GRAND IDEOLOGIES OF FASCISM AND
TECHNOLOGY THAT ONE CRITICIZES ONE DISMISSES THE OFFICIAL VERSION OF
IDEOLOGY ON BEHALF OF ITS INNER GREATNESS, BECOMING A MORE EFFECTIVE
COG IN ITS OPERATION

Zizek 99
[Slavoj, Unapologetic Self-Plagierist, The Ticklish Subject: The absent centre of political ontology, NYC:
Verso, 1999,13-4//uwyo-ajl]

Apropos of this precise point, I myself run into my first trouble with Heidegger (since I began as a
Heideggereian my first published book was on Heidegger and language). When, in my youth, I was
bombarded by the official Communist philosophers stories of Heideggers Nazie engagement, they left
me rather cold; I was definitely more on the side of the Yugoslav Heideggerians. All of a sudden, however,
I became aware of how these Yugoslav Heideggerians were doing the same thing with respect to the
Yugoslav ideology of self-management as Heidegger himself did with respect to Nazism: in ex-Yugoslavia,
Heideggerians entertained the same ambiguously assertive relationship toward Socialist self-
management, the official ideology of the Communist regime in their eyes, the essense of self-
management was the very essence of modern man, which is why the philosophical notion of self-
management suits the ontological essence of our epoch, while the standard political ideology of the
regime misses this inner greatness of self-managementHeideggerians are thus eternally in search of a
positive, ontic political system that would come closest to the epochal ontological truth, a strategy which
inevitably leads to error which, of course, is always acknowledged only retroactively, post factum, after
the disastrous outcome of ones engagement.

As Heidegger himself put it, those who come closest to the ontological Truth are condemned to err at the
ontic level err about what? Precisely about the line between ontic and ontological. The paradox not to
be underestimated is that the very philosopher who focused his interest on the enigma of ontological
difference who warned again and again against the metaphysical mistake of conferring ontological
dignity on some ontic content (god as the highest Entity, for example) fell into the trap of conferring on
Nazism the ontoligcal dignity of suiting the essence of modern man. The standard defense of Heidegger
against the reproach of his Nazi past consists of two points: not only was his Nazi engagement a simple
personal error (a stupidity [Dummheit], as Heidegger himself put it) in no way inherently related to his
philosophical project; the main counter-argument is that it is Heideggers own philosophy that enables us
to discern the true epochal roots of modern totalitarianism. However, what remains unthought here is
the hidden complicity between the ontological indifference towards ocncrete social systems (capitalism,
Frascism, Communism), in so far as they all belong to the same horizon of modern technology, and the
secret priveleging of a concrete sociopolitical model (Nazism with Heidegger, Communism with some
Heideggerian Marxists), as closer to the ontological truth of our epoch.

Here, one should avoid the trap that caught Heideggers defenders, who dismissed Heideggers Nazi
engagement as a simple anomaly, as a fall into the ontic level, in blatant contradiction to his thought,
which teaches us not to confuse ontological horizon with ontic choices (as we have already seen,
Heidegger is at his strongest when he demonstrates how, on a deeper structural level, ecological,
conservative, and so on, oppositions to the modern universe of technology are already embedded in the
horizon of what they purport to reject: the ecological critique of the technological exploitation of nature
ultimately leads to a more environmentally sound technology, etc. Heidegger di not engage in the Nazi

Page 947 of 1481


political project in spite of his ontological philosophical approach, but because of it; this engagement
was not beneath his philosophical level on the contrary, if one is to understand Heidegger, the key
point is to grasp the complicity (in Hegelese: speculative identity) between the elevation above ontic
concerns and the passionate ontic Nazi political engagement.

Page 948 of 1481


Page 949 of 1481
Ontology = Nazism: 1AR
HEIDEGGERS FOCUS ON ONTOLOGY MAKES FASCIST IDEOLOGY POSSIBLE
THROUGH A FALSE INNER DISTANCE, BLINDING ITSELF TO ONTIC ATROCITY

Zizek 99
[Slavoj, Unapologetic Self-Plagierist, The Ticklish Subject: The absent centre of political ontology, NYC:
Verso, 1999,14-5//uyo-ajl]

One can see the ideological trap that caught Heidegger: when he criticizes Nazi racism on behalf of the
true inner greatness of the Nazi movement, he repeats the elementary ideological gesture of
maintaining an inner distance towards the ideological text of claiming that there is something more
beneath it, a non-ideological kernel: ideology exerts its hold over us by means of this very insistence that
the Cause we adhere to is not merely ideological. So where is the trap? When the disappointed
Heidegger turns away from active engagement in the Nazi movement, he does so because the Nazi
movement did not maintain the level of its inner greatness, but legitimized itself with inadequate (racial)
ideology. In other ords, hat he expected from it was that it should legitimize itself through direct
awareness of its inner greatness. And the problem lies in this very expectation that a political movement
that will directly refer to its historico-ontological foundaiton is possible. This expectation, however, is in
itself profoundly metaphysical, in so far as it fails to recognize the gap separating the direct ideological
legitimization of a movement from its inner greatness (its historico-ontological essence) its constitutive,
a positive condiiton of its functioning. To use the terms of the late Heidegger, ontological insight
necessarily entails ontic blindness and error, and vice versa that is to say, in order to be effecitve at the
ontic level, one must disregard the ontological horizon of ones activity. (In this sense, Heidegger
emphasizes that science doesnt think and that, far from being is limitation, this inability is the very
motor of scientific progress.) In other words, what Heidegger seem suntable to endorse is a concrete
political engagement that would accept its necessary, constitutive blindness as if the moment we
acknowledge the cap separating the awareness of the ontological horizon from ontic engagement, any
ontic engagement is depreciated, loses its authentic dignity.

Page 950 of 1481


Ontology = Nazism: Ext (1/3)

EXCLUSIVE FOCUS ON THE ONTOLOGY OF TECHNOLOGY COLLAPSES INTO A


FASCIST FANTASY OF BLOOD AND SOIL

Daniels 2002
[Michael D., Trinity University, Heidegger and Nazism, Prometheus: The John
Hopkins Student Journal of Philosophy, Vol. I, Fall 2002,
http://www.jhu.edu/prometheus/Heidegger%20and%20Nazism.pdf, acc 10-11-
04//uwyo-ajl]

The Nazis, Heidegger believed, were opposed to the view of technology held by

Americanism and communism, rather they trusted the feelings and sensibilities of the

Volk and the need to create a German state out of German Volk. The Volk stresses historical
realization and exaltation of the Germans as German.38

Alan Paskow notes that Volk is merely a metaphysical justification for racism, however,
Heidegger saw much more in the Volk. Central to Heideggers support of the Nazis was that
their radicalism made possible a courageous confrontation with the question of Being. This, in
his view would make possible a truly human, or what Heidegger called a truly spiritual, world. 39
If spiritual leaders pose this question radically enough, a common questioning will pervade the
community. Thereby, the Volk can play an active role in shaping its fate by placing history into
the openness of the overpowering might of all the world-shaping forces of human existence and
by struggling ever anew to secure its spiritual world.40

Heidegger believed that Hitler was committed to facing the deepest and most

troubling questions. Furthermore, Heidegger hoped Hitler would evoke a communal

reflection on the question of Being. In effect, Hitler would engineer a communal escape

from the Platonic cave into the light of reality.41

In Heideggers view, the Nazis understood that knowledge was fundamentally rooted in praxis
and thus reconstituting the unity of life in a way unknown since the pre-Socratics. For the
Greeks before Plato, there was no theory apart from, or above, practice. The Greeks understood
that theory was the highest mode of human activity, but they understood it as the supreme
realization of practice.

THE SEARCH FOR AN AUTHENTIC MOVEMENT, FREE OF TECHNOLOGY REQUIRES


NAZISM BECAUSE EVERY CONCEPTION OF RIGHTS IS INFECTED WITH ONTIC
REASONING

Zizek 99

Page 951 of 1481


[Slavoj, Unapologetic Self-Plagierist, The Ticklish Subject: The absent centre of
political ontology, NYC: Verso, 1999, 12-3//uwyo-ajl]
Here, however, complications arise: on closer inspection, it soon becomes clear that Heideggers
argumentative strategy is twofold. On the one hand, he rejects every concern for democracy
and human rights as a purely ontic affair unworthy of proper philosophical ontological
questioning democracy, Fascism, Communism, they all amount ot the same with regard to the
epochal destiny of the West; on the other hand, his insistence that he is not convinced that
democracy is the political form which best suits the essence of technology none the less
suggests that there is another political form which suits this ontological essence better - for
some time Heidegger thought he had found it in the Fascist total mobilization (but,
significantly, never in Communism, which always remains for him epochally the same as
Americanism). Heidegger, of course, emphasizes again and again how the ontological
dimension of Nazism is not to be quated with Nazism as an ontic ideologico-political order; in
the well-known passage from An Introduction to Metaphysics, for example, he repudiates the
Nazi biologist race ideology as something that totally misses the inner greatness of the Nazi
movement, which lies in the encounter between modern man and technology. None the less,
the fact remains that Heidegger never speaks of the inner greatness of, say, liberal democracy
as if liberal democracy is just that, a superficial world-view with no underlying dimension of
assuming ones own epochal destiny.

Page 952 of 1481


Page 953 of 1481
Ontology = Nazism: Ext (2/3)
THE HEIDEGGEREAN FOCUS NECESSITATES VALORIZATION OF BLOOD AND WORK,
ALLOWING FASCISM TO ENTER THROUGH THE WINDOW

Wolin 2000
[Richard, Department of History at City University of New York, Arbeit Macht Frei: Heidegger as the Philosopher of
the German Way, Nietzshe, Heidegger, and the Future of Democracy, Winter Quarter, January 24, 2000,
olincenter.uchicago.edu/pdf/wolin.pdf, Acc. 10-12-04//uwyo-ajl]

Since the Logic is a lecture course rather than a public political statemen (which, therefore, might necessitate certain compromises with the regime), it

presumably reflects Heideggers genuine views. That is why it is so disturbing to find him, at least in

one instance, flirting with the regimes racial-biological doctrines. Equally disturbing is

the fact that Heidegger suggests one might reconcile Nazisms racial precepts (the
concepts of blood and racial descent) with his own pet existential themes and ideals
(mood, work, historicity):

Blood, racial descent (das Geblt) can only be [reconciled] with the foundationa mood of man when it is determined by temperament and mood [das Gemt].
The contribution of blood comes from the foundational mood of man and belongs to the determination of our Dasein through work. Work = the historical
present. The present (die Gegenwart) is not merely the now; instead it is the present insofar as it transposes our Being in the emancipation of existence that is
accomplished through work. As someone who works man is transported into the publicness o

existence. Such being-transported belongs to the essence of our Being: that is, to

our being-transported amid things in the world. . . . As something original,


existence never reveals itself to us via the scientific cognition of objects, but32 instead in the essential moods of that flourish in work and in the historical
vocation of a Volk that predetermines all else.

One of the Nazis major domestic political concerns in the regimes initial years

was whether they would be successful in integrating the German working classes
traditionally, staunch supporters of the political left -- within the National Socialist

Volksgemeinschaft. To that end they established the German Labor Front to assure

German workers that their role in the new state was an indispensable one. Both the

strength through joy and beautification of labor programs discussed earlier were an

offshoot of the same effort.47 In his vigorous celebration of the joy of work

(Arbeitsfreudigkeit), Heidegger once again demonstrates the elective affinities between

Existenzphilosophie and the National Socialist worldview:

The question of the joy of work is important. As a foundational mood, joy is the
basis of the possibility of authentic work. In work as something present, the making present of Being occurs. Work is
presencing in the original sense to the extent that we insert ourselves in the preponderance of Being; through work we attain the whole of Being in all its
greatness, on the basis of the great moods of wonder and reverence, and thereby enhance it in its greatness (102).

Page 954 of 1481


HEIDEGGERS CRITICISM OF WESTERN RATIONALISM OPENS UP THE SPACE FOR
NATIONAL SOCIALISM BECAUSE OF THEIR PERCEIVED INAUTHENTICITY

Wolin 2000
[Richard, Department of History at City University of New York, Arbeit Macht Frei: Heidegger as the Philosopher of
the German Way, Nietzshe, Heidegger, and the Future of Democracy, Winter Quarter, January 24, 2000,
olincenter.uchicago.edu/pdf/wolin.pdf, Acc. 10-12-04//uwyo-ajl]

Needless to say, a rejection of universal concepts by no means entails a

commitment to Nazism. Yet, with this radical philosophical maneuver, Heidegger left

himself vulnerable to political movements whose major selling point in opposition to

the presumed decrepitude of Western liberalism -- was an unabashed celebration of

volkish particularism. The same normative criticisms Heidegger had brought to bear

against Western rationalism were also used by him as arguments against their

corresponding political forms: cosmopolitanism, rights of man, constitutionalism. Search

as one may through Heideggers voluminous philosophical corpus, one is extremely hard
pressed to find a positive word concerning the virtues of political liberalism. His

philosophical and political predilections were related to one another necessarily rather

than contingently.

Page 955 of 1481


Page 956 of 1481
Ontology = Nazism: Ext (3/3)
HEIDEGGERS OBSESSION WITH FACTICITY AND AUTHENTIC PARTICULARISM
ALLOWS ONE TO LAPSE INTO CONSERVATIVE GLORIFICATION OF A RURAL
FANTASY AND ENDORSEMENT OF FASCISM

Wolin 2000
[Richard, Department of History at City University of New York, Arbeit Macht Frei:
Heidegger as the Philosopher of the German Way, Nietzshe, Heidegger, and the
Future of Democracy, Winter Quarter, January 24, 2000,
olincenter.uchicago.edu/pdf/wolin.pdf, Acc. 10-12-04//uwyo-ajl]

The epistemological emphasis on faciticity, which celebrates the particularism

of ones own immediate heritage/life/milieu, is a logical corollary of a perspective that

esteems the concrete over the abstract. In this regard, the central position that

Heidegger in Being and Time accords to mineness (Jemeinigkeit) is also indicative and

revealing. When in 1933 Heidegger turned down an offer for a position at Humboldt

University in Berlin, he justified his decision by glorifying the provincial values of

locality and region, which he contrasted to the corrupting influences of modern city life:

The world of the city runs the risk of falling into a destructive error. A very loud

and very active and very fashionable obstrusiveness often passes itself off as

concern for the world and existence of the peasant. But this goes exactly contrary

to the one and only thing that now needs to be done, namely, to keep ones

distance from the life of the peasant, to leave their existence more than ever to its

own law, to keep hands off lest it be dragged into the literatis dishonest chatter

about folk character and rootedness in the soil.4

According to intimate Heinrich Petztet, the Freiburg philosopher felt ill at ease

with big-city life, "and this was especially true of that mundane spirit of Jewish circles,

which is at home in the metropolitan centers of the West."5 In the late 1920s Heidegger

would vigorously protest the growing Jewification (Verjudung) of German spiritual

life.6 Thus, in Heideggers corpus the boundaries between philosophy and

weltanschauung are fluid and not impenetrable.7 To date, the predominant formalphilosophical

interpretations of his work have systematically neglected its ideological

Page 957 of 1481


dimensions, to their own detriment. By proceeding from a philosophical standpoint that

consistently valued the particular over the universal, Heideggers thought was exposed

from the outset to grave ethical and political deficits. This conclusion suggests that in

seeking to account for Heideggers 1933 political lapsus, the existential standpoint he

cultivated in the early 1920s is as important as the historical-biographical contingencies

stressed by his defenders.

Page 958 of 1481


A2 We Dont Advocate Nazism: 1AR

THEYRE MISSING THE BOAT ON OUR ARGUMENT. WE ARENT SAYING THAT YOU
SHOULD REJECT THEM BECAUSE THEY CITED SOMEONE WHO WAS A NAZI, BUT
RATHER, THAT EXCLUSIVE FOCUS ON HOW TECHNOLOGY AFFECTS BEING LEADS
TO POLITICAL PARALYSIS AND ACCEPTANCE OF ATROCITY IN THE ONTIC REALM.

IF YOU SPEND ALL OF ENERGY SEARCHING FOR A WORLD THAT WILL DISCLOSE
ITSELF IN ITS AUTHENTICITY, THAT ALLOWS YOU TO PARTICIPATE IN HORRIFIC
VIOLENCE BECAUSE IT DOESNT MEET THE CRITERIA OF AN ONTOLOGICAL IMPACT.
HEIDEGGER EVEN ADMITS THAT THE TWO KINDS OF QUESTIONING TRADE OFF.
CROSS-APPLY ZIZEK 99

Page 959 of 1481


A2 Nazism is Inauthentic: 1AR

EVEN IF NATIONAL SOCIALISM IS REALLY INAUTHENTIC, THATS NON-RESPONSIVE


TO THE ARGUMENT THAT THE ATTEMPT TO ASSERT AUTHENTICITY DELUDES ONE
INTO ACCEPTING ONTICALLY ATROCIOUS POLITICAL CHOICES. CROSS-APPLY ZIZEK
99

IF THIS IS TRUE, OUR ARG IS A TURN. THE SEARCH FOR AUTHENTICITY RESULTS IN
COMPLICITY IN POLITICS THAT ARE EVEN MORE INAUTHENTIC, REINSCRIBING THE
WILL TO WILL THAT THEYRE CRITICIZING. WE THINK THAT WE HAVE A PURE
RELATIONSHIP WITH THE WORLD BUT ARE EVEN MORE TIED TO THE MEDIATION
OF REALITY

THIS ARGUMENT BETRAYS HISTORICAL REVISIONISM THE CRITICISM OF


INAUTHENTICITY IS DIRECTLY TIED TO REJECTION OF ONTIC ETHICS BECAUSE OF
THEIR MODERNITY

Wolin 2000
[Richard, Department of History at City University of New York, Arbeit Macht Frei: Heidegger as the Philosopher of
the German Way, Nietzshe, Heidegger, and the Future of Democracy, Winter Quarter, January 24, 2000,
olincenter.uchicago.edu/pdf/wolin.pdf, Acc. 10-12-04//uwyo-ajl]

In the massive secondary literature on Being and Time, the concept of historicity

has suffered from relative neglected. Perhaps this is because it represents the aspect of

Heideggers treatise where the philosopher stands in the greatest proximity to

contemporary politics and, hence, the moment where the ideological aspects of his

thought are most exposed. The reasons for this neglect are in part comprehensible. To

date Being and Time has primarily been interpreted in a Kierkegaardian/existential vein.

It portrays a highly individualized Dasein wrestling with a series of basic ontological

questions: the struggle for authenticity, the meaning of death, the nature of care. Yet,

the discussion of historicity, which in many respects represents a culmination of the

books narrative, emphasizes a set of concerns -- destiny, fate, the nature of authentic

historical community (Gemeinschaft) -- that are difficult to reconcile with the

Kierkegaardian interpretation of the work as basically concerned with Dasein as an

Page 960 of 1481


isolated individual Self. To be sure, were this Heideggers standpoint, it would be very

difficult to reconcile the idea of historical political commitment with his intentions, and

one would have to view Heideggers later political commitment as standing in

contradiction with Being and Times basic ideals. It has often been argued in the

philosophers defense that since Heideggers actions on behalf of Nazism demanded a

surrender of individuality to the ends of the historical community, his political choice

stood at cross-purposes with his philosophy. According to this reading, therefore,

Heideggers political involvement represented an instance of inauthenticity. However,

this interpretation forfeits its cogency once the concept of historicity -- in which

Heidegger unambiguously declares the centrality of collective historical commitment -- is

taken seriously.

As Lwith understood, it is but a short step from the facticity and particularism of

individual Existenz to a celebration of volkish parochialism in collective-historical terms.

For Heidegger the mediating link between these two aspects of Dasein -- the individual

and the collective -- was the conservative revolutionary critique of modernity. This

strident lament concerning the world-historical decadence of bourgeois existence was

first articulated in the work of Nietzsche, Spengler as well as countless lesser

Zivilisationskritiker. In Thomas Manns Confessions of an Unpolitical Man, for example,

the antinomy between Kultur and Zivilisation occurs over one hundred times.

Page 961 of 1481


Page 962 of 1481
Heidegger Kills Change
HEIDEGGERS ALTERNATIVE IS BASED ON DOGMATIC AUTHORITARIANISM THAT
CAN NEVER LEAD TO POSITIVE CHANGE

Thiele, Prof of Political Science @ U of Florida, 2K3 (Leslie Paul, The Ethics and Politics of Narrative,
Foucalt and Heidegger: Critical Encounters, Ed. Rosenberg and Milchman)

The pursuit of knowledge continues unabated for the skeptic. Yet it proceeds with a suspicious eye. There are inherent limitations to and a price to pay for
the pursuit of knowledge. Charles Scott describes Foucault's efforts in this regard: Far from the skepticism that argues that nothing is really
knowablegenealogies embody a sense of the historical limits that define our capacities for knowing and believing. Things are known. But they are known in
ways that have considerable social and cultural costs. 8 Both Heidegger and Foucault maintain that there is no legitimate
basis for the radical skeptic's conviction that knowledge is impossible or unworthy of pursuit. This sort of
skepticism, Heidegger states, consists merely in an addiction to doubt. 9 The skeptical nature of political
philosophical thought, in contrast, is grounded in the imperative of endless inquiry. The point for Heidegger and Foucault is
to inquire not in order to sustain doubt, but to doubt that one might better sustain inquiry. At the same time, inquiry is tempered with a
sensibility of the ethico-political costs of any knowledge that is gained. Doing political philosophy of this
sort might be likened to walking on a tightrope. If vertigo is experienced, a precarious balance may be
lost. Falling to one side leaves one mired in apathy, cynicism, and apoliticism. This results when skeptical
inquiry degenerates into a radical skepticism, an addictive doubt that denies the value of (the search for)
knowledge and undermines the engagements of collective life, which invariably demand commitment
(based on tentatively embraced knowledge). Falling to the other side of the tightrope leaves one mired in
dogmatic belief or blind activism. Authoritarian ideologies come to serve as stable foundations, or a
reactive iconoclasm leads to irresponsible defiance. Apathy, cynicism, and apoliticism, on the one side,
and dogmatic authoritarianism or reactive iconoclasm, on the other, are the dangerous consequences of
losing one's balance. These states of mind and their corresponding patterns of behavior relieve the vertigo of political philosophical inquiry, but at a
prohibitive cost. It has been argued that Foucault did not so much walk the tightrope of political philosophy as straddle it, at times leaving his readers hopeless
and cynical, at times egging them on to an irresponsible monkeywrenching. For some, the Foucauldian flight from the ubiquitous powers of normalization
undermines any defensible normative position. Hopelessness accompanies lost innocence. Cynicism or nihilism become the only alternatives for those who
spurn all ethical and political foundations. By refusing to paint a picture of a better future, Foucault is said to undercut the impetus to struggle. Others focus on
Foucault's development of a tool kit whose contents are to be employed to deconstruct the apparatuses of modern power. Yet the danger remains that
Foucault's hyperactive tool-kit users will be unprincipled activists, Luddites at best, terrorists at worst. In either case, Foucault provides no overarching
theoretical vision. Indeed, Foucault is upfront about his rejection of ethical and political theories and ideals. I think that to imagine another system is to
extend our participation in the present system, Foucault stipulates. Reject theory and all forms of general discourse. This need for theory is still part of the
system we reject. 10 One might worry whether action is meant to take the place of thought. If Foucault occasionally straddles the tightrope of political
philosophy, Heidegger obviously stumbled off it. In the 19305, Heidegger enclosed himself within an authoritarian
system of thought grounded in ontological reifications of a folk and its history. Heidegger's
historicization of metaphysics led him to believe that a new philosophic epoch was about to be inaugurated. It implicitly called for a
philosophical Fuehrer who could put an end to two millennia of ontological forgetting. 11 The temptation
for Heidegger to identify himself as this intellectual messiah and to attach himself to an authoritarian
social and political movement capable of sustaining cultural renewal proved irresistible. Whether Heidegger ever
fully recovered his balance has been the topic of much discussion. Some argue that Heidegger's prerogative for political
philosophizing was wholly undermined by his infatuation with folk destiny, salvational gods, and political
authority. 12

Page 963 of 1481


Page 964 of 1481
Heidegger Irrelevent
HEIDEGGERS PHILOSOPHY DOES NOT APPLY AND IS DANGEROUS IN THE
POLITICAL REALM BECAUSE METAPHYSICAL RESULTS WILL NOT RESULT FROM
POLITICS

Wolin, Prof of Modern European Intellectual History @ Rice, 90 (Richard, The Politics of Being, P. 117-118)

Moreover, as Harries indicates, Heidegger's theory of the state as a "work" is modeled upon his theory of the work of art. Thus, as we have seen, in Heidegger's
view, both works of art and the state are examples of the "setting-to-work of truth." In essence, the state becomes a giant work of art: like the work of art, it
participates in the revelation of truth, yet on a much more grandiose and fundamental scale, since it is the Gesamtkunstwerk within which all the other sub-
works enact their preassigned roles. However, the idea of basing political judgments on analogy with aesthetic judgments is an extremely tenuous proposition.
Though we may readily accept and even welcome Heidegger's claim that works of art reveal the truth or
essence of beings ("The work [of art] ... is not the reproduction of some particular entity that happens to be present at any given time," observes
Heidegger; "it is, on the contrary, the reproduction of the thing's general essence"),66 we must question the attempt to transpose
aesthetico-metaphysical criteria to the realm of political life proper. Is it in point of fact meaningful to
speak of the "unveiling of truth" as the raison d'etre of politics in the same way one can say this of a work
of art or a philosophical work? Is not politics rather a nonmetaphysical sphere of human interaction, in
which the content of collective human projects, institutions, and laws is articulated, discussed, and agreed
upon? Is it not, moreover, in some sense dangerous to expect "metaphysical results" from politics? For is not
politics instead a sphere of human plurality, difference, and multiplicity; hence, a realm in which the more
exacting criteria of philosophical truth must play a subordinate role? And thus, would it not in fact be to
place a type of totalitarian constraint on politics to expect it to deliver over truth in such pristine and
unambiguous fashion? And even if Heidegger's own conception of truth (which we shall turn to shortly) is sufficiently tolerant and pluralistic to
allay such fears, shouldn't the main category of political life be justice instead of truth? Undoubtedly, Heidegger's
long-standing prejudices against "value-philosophy prevented him from seriously entertaining this
proposition; and thus, as a category of political judgment, justice would not stand in sufficiently close
proximity to Being. In all of the aforementioned instances, we see that Heideggers political philosophy is
overburdened with ontological considerations that end up stifling the inner logic of politics as an
independent sphere of human action.

Page 965 of 1481


Page 966 of 1481
Rejecting Tech Leads to
Extinction
HEIDEGGERS PHILOSOPHY OF REJECTING ALL TECHNOLOGY MAKES LIFE
MEANINGLESS, CULMINATING IN EXTINCTION

Hicks, Prof and Chair of philosophy @ Queens College of the CUNY, 2K3 (Steven V., Nietzsche, Heidegger, and
Foucault: Nihilism and Beyond, Foucault and Heidegger: Critical Encounters, Ed. Alan Milchman and Alan Rosenberg, p.
109, Questia)

Heidegger's insightful reading of Nietzsche


Why a philosophical shock? The answer, in part, may be that from Foucault's perspective,
and the problem of nihilism is itself too ascetic. Heidegger's emphasis on silence as proper to Dasein's being, his
frequent use of quasireligious (even Schopenhauerean) terms of grace and call of conscience, his many
references to the destiny of the German Volk, his avoidance of politics and the serious quietistic tone of
Heideggerian Gelassenheit are all reminiscent of the life-denying ascetic ideal Nietzsche sought to avoid.
65 Moreover, Foucault seems to join with Derrida and other neo-Nietzscheans in regarding Heidegger's idea
of letting Being behis vision of those who have left traditional metaphysics behind and with it the
obsession with mastery and technology that drives contemporary civilizationas too passive or apathetic
a response to the legitimate problems of post-Nietzschean nihilism that Heidegger's own analysis
uncovers. 66 Here we have arrived at a key difference between Heidegger and Foucault: for Foucault, Heidegger takes insufficient account of the playful
and even irreverent elements in Nietzsche and of Nietzsche's critique of the dangers of the ascetic ideal. Foucault joins with other new Nietzscheans in
promoting, as an alternative to Heideggerian Gelassenheit, the more Nietzschean vision of playing with the textwhich in Foucault's case means
promulgating active and willful images of resistance and struggle against particular practices of domination, rebellion against micro-powers, and blatant
disregard for tradition (cf. DP, 27). 67 This context-specific, unambiguously confrontational nature of Foucault's critique of the forms of domination and
technologies of power lodged in modern institutions offers a more Nietzsche-like response than the one Heidegger offers to the nihilistic problems of Western
civilization. As Foucault sees it,the lessons Heidegger would have us draw from Nietzsche throw us back to the
passive nihilism of emptiness that Nietzsche feared. While not predicting the emergence of better times, Foucault tries to offer a
better (less passive, less ascetic) model for reforming our background practices and for cultivating an affirmative attitude toward life
that he and other neo-Nietzscheans think may be our only chance to keep from extinguishing life on
earth altogether. 68

Page 967 of 1481


Page 968 of 1481
Alternative Fails: Lapses Into
Ontic Thought
EVERY ATTEMPT TO RECONCEPTUALIZE BEING LAPSES INTO ONTIC THOUGHT,
MEANING THAT THE AFF IS STILL TIED TO THE HISTORICAL POLITICAL HORIZON
THAT THEYRE CRITICIZING

Wolin 2000
[Richard, Department of History at City University of New York, Arbeit Macht Frei:
Heidegger as the Philosopher of the German Way, Nietzshe, Heidegger, and the
Future of Democracy, Winter Quarter, January 24, 2000,
olincenter.uchicago.edu/pdf/wolin.pdf, Acc. 10-12-04//uwyo-ajl]

That the standpoint of Being and Time is informed by the conservative revolutionary worldview
suggests that Heideggers existential analytic, far from a purely

formal undertaking, is in fact laden with ontic content -- a content derived from the

Zeitgeist of the interwar years. The critique of everydayness in Division I of

publicness, falling, curiosity, and the they emerges precisely therefrom.

Inattention to this dimension of Heideggers work suggests the pitfalls a purely textimmanent
reading, in which the filiations between politics and philosophy are a priori

extruded.

The intimate relationship between fundamental ontology and the German

ideology should come as no surprise. Heidegger always insisted that ontological

questioning can never be atemporal and never comes to pass in an historical void. Instead,

it is unavoidably saturated with historicity. As he remarks at one point in Being and

Time: every ontology presupposes a determinate ontic point of view. . .

Outfitted with a measure of historical perspective, we are now aware of the extent

to which the early Heidegger made this critique his own.10 As Lwith comments:

Whoever. . .reflects on Heideggers later partisanship for Hitler, will find in this

first formulation of the idea of historical existence the constituents of his

political decision of several years hence. One need only abandon the still quasireligious

isolation and apply [the concept of] authentic existence -- always particular to each individual
-- and the duty that follows therefrom to specifically German existence and its historical
destiny in order thereby to introduce into the general course of German existence the energetic
but empty movement of existential categories (to decide for oneself; to take stock of 9
oneself in the face of nothingness; wanting ones ownmost destiny; to take responsibility

Page 969 of 1481


for oneself) and to proceed from there to destruction now on the terrain of politics. It is not
by chance if one finds in Carl Schmitt a political

decisionism -- in which the potentiality-for-Being-a-whole of individual

existence is transposed to the totality of the authentic state, which is itself

always particular -- that corresponds to Heideggers existentialist philosophy.

Page 970 of 1481


Page 971 of 1481
Alternative Fails: Tech Returns
REJECTING A CERTAIN FORM OF TECHNOLOGICAL DOMINATION MERELY
REPLACES IT WITH A NEW ONE, FURTHERING THE TECHNOLOGIZATION OF LIFE
AND REINSCRIBING THE TARGET OF CRITICISM

Zizek 99
[Slavoj, Unapologetic Self-Plagierist, The Ticklish Subject: The absent centre of
political ontology, NYC: Verso, 1999, 11-12//uwyo-ajl]

In his project of overcoming metaphysics, Heidegger fully ensorses this Nietzschean dismissal of quick
and easy exists from metaphysics; the only real way to break the metaphysical closure is to pass through
it in its most dangerous form, to endure the pain of metaphysical nihilism at its most extreme, which
means that one should reject as futile all false sedatives, all direct attempts to suspend the mad vicious
cycle of modern technology by means of a return to premodern traditional Wisdom (from Chrsitianity
through Oriental thought), all attempts to reduce the threat of modern technology to the effect of some
ontic social wrong (capitalist exploitation, patriarchal domination, mechanicist paradigm). These
attempts are not only ineffectual: the true problem with them is that, on a deeper level, they incite the
evil they are fighting even further. An excellent example here is the ecological crisis: the moment we
reduce it to disturbances provoked by our excessive technological exploitation of nature, we silently
already surmise that the solution is to rely again on technological innovations: new green technology,
more efficient and global in its control of natural processes and human resources Every concrete
ecologogical concern and project to change technology in order to improve the state of our natural
surroundings is thus devalued as relying on the very source of the trouble.

Page 972 of 1481


Page 973 of 1481
Alternative Causes Suffering
HEIDEGGER UNDERMINES MORALS AND POLITICS THE ALTERNATIVE WILL ONLY
CAUSE SUFFERING AND THE DESTROY ALL ETHICS

Thiele, Prof of Political Science @ U of Florida, 2K3 (Leslie Paul, The Ethics and Politics of
Narrative, Foucalt and Heidegger: Critical Encounters, Ed. Rosenberg and Milchman)

The complementarity of Heidegger's and Foucault's accounts of modern demons and saving graces should not be too surprising. Foucault's indebtedness to
and fascination with Heidegger is well documented. 1 My intent in this chapter is neither to focus on the complementarity of these visions, nor to outline the
striking philosophical and political differences that remain in Heidegger's and Foucault's work. Rather, I attempt to make a claim for what at first blush might
Heidegger and Foucault are often castigated as ethico-
appear a lost cause. Despite their originality and intellectual brilliance,
political dead-ends. They are criticized for their unwillingness or inability to supply the grounds for sound
moral and political judgment. Heidegger's embrace of Nazism, in particular, is frequently identified as
proof positive that he has little, if anything, to contribute to the ethico-political domain. The standard
charge is that his highly abstract form of philosophizing, empyrean ontological vantage point, and
depreciation of das Man undermines moral principle and political responsibility. From his
philosophical heights, it is suggested, Heidegger remained blind to human sufferings, ethical imperatives,
and political practicalities. He immunized himself against the moral sensitivity, compassion, and prudence
that might have dissuaded him from endorsing and identifying with a brutal regime. Those who embrace
his philosophy, critics warn, court similar dangers. In like fashion, it is held that Foucault dug himself into an equally deep, though
ideologically relocated, moral and political hole. Genealogical studies left Foucault convinced of the ubiquity of the disciplinary matrix. There would be no final
liberation. The sticky, normalizing webs of power were inescapable and a hermeneutics of suspicion quashed any hope of gaining the ethical and political
high ground. 2 As such, critics charge, Foucault stripped from us all reason for resistance to unjust power and all hope of legitimating alternative ethico-political
institutions. In a Foucauldian world of panoptic power that shapes wants, needs, and selves, critics worry, one would have no justification for fighting and
nothing worth fighting for. 3 In sum, Heidegger's and Foucault's critics suggest that both thinkers undermine the foundations of the practical wisdom needed
to ethically and politically navigate late modernity. Despite the brilliance and originality of their thought, arguably the greatest philosopher and the greatest
social and political theorist of the twentieth century remain ungrounded ethically and divorced from political responsibility. Critics
argue that
Heidegger's statements and actions endorsing and defending Nazi authoritarianism and Foucault's radical
anarchism, as displayed in his discussions of popular justice with Maoists, demonstrate that neither
thinker is capable of supplying us with the resources for sound moral and political judgment.

Page 974 of 1481


Page 975 of 1481
Alternative Causes Paralysis
(1/2)
HEIDEGGERS PHILOSOPHY HAS MORAL CONSEQUENCES AND LEADS TO
PARALYSIS IT JUSTIFIES SITTING BACK AND ALLOWING FOR THE HOLOCAUST
WHILE CRITICIZING THE TECHNOLOGY USED TO KILL THE JEWS

Bookchin, Founder of the Institute for Social Ecology and Former Professor @ Ramapo College, 95
(Murray, Re Re-Enchanting Humanity: A Defense of the Human Spirit Against Antihumanism,
Misanthropy, Mysticism and Primitivism, p. 168-170)

"Insofar as Heidegger can be said to have had a project to shape human lifeways, it was as an endeavor to resist, or should I say, demur from, what he
conceived to he an all-encroaching technocratic mentality and civilization that rendered human beings 'inauthentic' in their relationship to a presumably self-
generative reality, 'isness', or more esoterically, 'Being' (Sein). Not unlike many German reactionaries, Heidegger
viewed modernity' with its
democratic spirit, rationalism, respect for the individual, and technological advances as a 'falling'
(Gefallen) from a primal and naive innocence in which humanity once 'dwelled, remnants of which he
believed existed in the rustic world into which he was born a century ago. 'Authenticity', it can be said
without any philosophical frills, lay in the pristine Teutonic world of the tribal Germans who retained their
ties with the Gods, and with later peoples who still tried to nourish their past amidst the blighted traits
of the modern world. Since some authors try to muddy Heidegger's prelapsarian message by focusing on his assumed belief in individual freedom
and ignoring his hatred of the French Revolution and its egalitarian, 'herd'-like democracy of the 'They', it is worth emphasizing that such a view withers m the
light of his denial of individuality. The individual by himself counts for nothing', he declared after becoming a member of the National Socialist party in 1933.
'The fate of our Volk m its state counts for everything.'22 As
a member of the Nazi party, which he remained up to the defeat
of Germany twelve years later, his antihumanism reached strident, often blatantly reactionary
proportions. Newly appointed as the rector of the University of Freiburg upon Hitler's ascent to power, he
readily adopted the Fuehrer-principle of German fascism and preferred the title Rektor-Fuhrer, hailing the
spirit of National Socialism as an antidote to 'the darkening of the world, the flight of the gods, the
destruction of the earth [by technology], the transformation of men into a mass, the hatred and suspicion
of everything free and creative.28 His most unsavory remarks were directed in the lectures, from which these lines are taken, 'from a
metaphysical point of view', against 'the pincers' created by America and Russia that threaten to squeeze 'the farthermost corner of the globe ... by technology
and ... economic exploitation.'29 Technology, as Heidegger construes it, is 'no mere means. Technology is a way of
revealing. If we give heed to this, then another whole realm for the essence of technology will open itself
up to us. It is the realm of revealing, i.e., of truth.30 After which Heidegger rolls out technology's
transformations, indeed mutations, which give rise to a mood of anxiety and finally hubris,
anthropocentricity, and the mechanical coercion of things into mere objects for human use and
exploitation. Heidegger's views on technology are part of a larger weltanschauung which is too multicolored to discuss here, and demands a degree of
interpretive effort we must forgo for the present in the context of a criticism of technophobia. Suffice it to say that there is a good deal of
primitivistic animism in Heidegger's treatment of the 'revealing' that occurs when techne is a 'clearing' for
the 'expression' of a crafted material - not unlike the Eskimo sculptor who believes (quite wrongly, I may add) that
he is 'bringing out' a hidden form that lies in the walrus ivory he is carving. But this issue must be seen
more as a matter of metaphysics than of a spiritually charged technique. Thus, when Heidegger praises a
windmill, in contrast to the 'challenge' to a tract of land from which the hauling out of coal and ore' is
subjected, he is not being 'ecological'. Heidegger is concerned with a windmill, not as an ecological
technology, but more metaphysically with the notion that 'its sails do indeed turn in the wind; they are
left entirely to the wind's blowing'. The windmill 'does not unlock energy from the air currents, in order to
store it'.31 Like man in relation to Being, it is a medium for the 'realization' of wind, not an artifact for
acquiring power. Basically, this interpretation of a technological interrelationship reflects a regression -
socially and psychologically as well as metaphysically into quietism. Heidegger advances a message of
passivity or passivity conceived as a human activity, an endeavor to let things be and 'disclose'

Page 976 of 1481


themselves. 'Letting things be' would be little more than a trite Maoist and Buddhist precept were it not
that Heidegger as a National Socialist became all too ideologically engaged, rather than 'letting things
be', when he was busily undoing 'intellectualism,' democracy, and technological

[continues]

Page 977 of 1481


Page 978 of 1481
Alternative Causes Paralysis
(2/2)
[continues]

intervention into the 'world'. Considering the time, the place, and the abstract way in which
Heidegger treated humanity's 'Fall' into technological inauthenticity a Fall that he, like Ellul, regarded
as inevitable, albeit a metaphysical, nightmare - it is not hard to see why he could trivialize the Holocaust,
when he deigned to notice it at all, as part of a techno-industrial condition. 'Agriculture is now a
motorized (motorsierte) food industry, in essence the same as the manufacturing of corpses in the gas
chambers and extermination camps,' he coldly observed, 'the same as the blockade and starvation of the
countryside, the same as the production of the hydrogen bombs.32 In placing the industrial means by
which many Jews were killed before the ideological ends that guided their Nazi exterminators, Heidegger
essentially displaces the barbarism of a specific state apparatus, of which he was a part, by the technical
proficiency he can attribute to the world at large! These immensely revealing offhanded remarks, drawn
from a speech he gave in Bremen m 1949, are beneath contempt. But they point to a way of thinking that
gave an autonomy to technique that has fearful moral consequences which we are living with these days
in the name of the sacred, a phraseology that Heidegger would find very congenial were he alive today.
Indeed, technophobia, followed to its logical and crudely primitivistic conclusions, finally devolves into a dark reactionism and
a paralyzing quietism. For if our confrontation with civilization turns on passivity before a disclosing of
Being, a mere dwelling on the earth, and a letting things be, to use Heideggers verbiage much of
which has slipped into deep ecologys vocabulary as well the choice between supporting barbarism and
enlightened humanism has no ethical foundations to sustain it. Freed of values grounded in objectivity,
we are lost in a quasi-religious antihumanism, a spirituality that can with the same equanimity hear the
cry of a bird and ignore the anguish of six million once-living people who were put to death by the
National Socialist state.

Page 979 of 1481


Page 980 of 1481
Heidegger Was a Nazi
HEIDEGGER IS A NAZI AND THAT UNIQUELY INFLUENCED HIS PHILOSOPHY

Thiele, Prof of Political Science @ U of Florida, 2K3 (Leslie Paul, The Ethics and
Politics of Narrative, Foucalt and Heidegger: Critical Encounters, Ed. Rosenberg and Milchman)

Heidegger was a Nazi, and a rather unrepentent one at that. Some suggest Heidegger's Nazism cannot be
separated from his philosophy, that indeed the former follows from the latter. The argument , in short, is that
Heidegger's political biography pretty well tells the whole story. This position has been rearticulated periodically since the end
of the Second World War, each time creating something of an academic row. 16 To be sure, the story of Heidegger's life does not well
illustrate an education in sound moral and political judgment, except perhaps as an example of a lesson left unlearned. Yet the
story that Heidegger himself tells about human life, about human being in history, can do much to
cultivate moral and political judgment. I assert this despite insightful critiques of Heidegger that accuse him of ignoring and eliding
phronesis as a human potentiality. 17 My argument, then, is not that Heidegger's work explicitly celebrates prudence, but that his philosophical
narrative facilitates its cultivation.

Page 981 of 1481


Page 982 of 1481
Page 983 of 1481
Anti-Humanism Justifies
Genocide
ANTI-HUMANISMS ALL-OR-NOTHING MENTALITY JUSTIFIED THE HOLOCAUST
WE MUST CHALLENGE THE REDUCTION IN HUMANIST VALUES IN ORDER TO
PREVENT ANOTHER HOLOCAUST

Ketels, Assoc Prof of English @ Temple U, 96 (Violet B., Havel to the Castle! The Power of the Word, 548
Annals 45, November, Lexis)

our terror of being found


History has survived them and provides a regenerative, other view against nihilism and detachment. It testifies that
guilty of phrases too smooth or judgment too simple is not in itself a value. Some longing for
transcendence persists in the human spirit, some tenacious faith that truth and goodness exist and can
prevail. What happened in the death camps, the invasion of Prague by Russian tanks, the rape of Muslim women, the dismembering of Bosnian men, the
degrading of a sophisticated society to subsistence and barbarous banditry: these things do not become fictions simply because we cannot speak of them
adequately or because composing abstractions is safer than responding to the heinous reality of criminal acts. No response to the Holocaust and its murderous
wake or to the carnage in the former Yugoslavia could possibly be adequate to the atrocities alphabetized in file folders of perpetrators or to the unspeakable
experiences burned into brains and bodies of survivors. But no response at all breeds new catastrophe. Saul Bellowwarned about the
"humanistic civilized moral imagination" that, seized with despair, "declines into lethargy and sleep." n15
Imagine the plight of human creatures if it were to be silenced altogether, extinguished or forgotten. " Humanism did not produce the
Holocaust, and the Holocaust, knowing its enemies, was bent on the extermination of humanism. It is an
odd consequence of an all-or-nothing mentality to repudiate humanist values because they are
inadequate as an antidote to evil." n16 Basic human rights asserted in words cannot be restored in reality unless they are matched to practices
in all the spheres of influence we occupy. We feel revulsion at the repudiation of humanist values so visible in the
savagery of the battlefield and the councils of war. Yet we seem inoculated against seeing the brutalities
of daily human interactions, the devaluing of values in our own intellectual spheres, the moral and ethical
debunking formally incorporated into scholarly exegesis in literature, philosophy, the social sciences, and
linguistics, the very disciplines that cradled humanist values. Remembering for the future by rehearsing the record, then, is not
enough, as the most eloquent witnesses to Holocaust history have sorrowfully attested. We must also respond to the record with
strategies that challenge humanist reductionism in places where we tend to overlook it or think it
harmless. Our moral outrage should be intensified, not subdued, [*50] by what we know. We must search
out alternatives to the anomie that seizes us when the linguistic distance between words and reality
seems unbridgeably vast, and reflections upon historical events ill matched to the dark complexities of the
human experience we would illumine.

ANTI-HUMANISM DESTROYS HUMAN DIGNITY

Campbell, Prof of International Politics @ U of Newcastle, 99 (David, The Deterritorialization of


Responsibility, Moral Spaces, Eds. Michael J. Shapiro & David Campbell)

Liberalism is thus insufficient for human dignity because the election that justifies man "comes from a god or Godwho beholds him
in the face of the other man, his neighbor, the original 'site' of the Revelation."34 Similarly, humanism is insufficient, and "modern
antihuman-ism ... is true over and beyond the reasons it gives itself." What Levinas finds laudable in
antihumanism is that it "abandoned the idea of person, goal and origin of itself, in which the ego is still a
thing because it is still a being." As such, antihumanism does not eradicate the human, but "clears the place for

Page 984 of 1481


subjectivity positing itself in abnegation, in sacrifice, in a substitution which precedes the will." It would
therefore be a grave error to conclude in haste that Levinas's antihumanism is either inhuman or inhumane. To the contrary. Levinas declares that
"humanism has to be denounced only because it is not sufficiently human,"3'' because it is insufficiently
attuned to alterity. If one understood "humanism" to mean a "humanism of the Other," then there would
be no greater humanist than Levinas.36

Page 985 of 1481


Page 986 of 1481
Liberal Humanism Solves
Oppression
LIBERAL HUMANISM LIBERATES MORE THAN IT DESTROYS AND STOPS THE WORST
OPPRESSION IN HISTORY THE WESTS FIGHT AGAINST COMMUNISM PROVES

Kors, Prof of History @ U of Pennsylvania and Senior Fellow at the Foreign Policy Research Institute, 2K1 (Alan
Charles, Triumph without Self-Belief, Orbis, Summer, ebsco)

For generations, and to this day, the great defenders of the humane consequences of the allocation of capital by free markets--Ludwig von Mises, Friedrich Hayek, and Milton Friedman, for
example--have remained unexplored, marginalized, or dismissed as absurd by most American intellectuals. The lionized intellectuals were and are, in sentimental memory, those who
dreamed about and debated how one would make the transition from unproductive and unjust capitalism to the cornucopia of central planning. For a full generation, academic intellectual
culture above all generally viewed the West's anticommunist military strength, let alone its willingness to project that strength, as the great obstacle to international justice and peace, and
derided the doctrine of peace through strength as the slogan of the demented. For at least a generation, Western intellectual contempt for the West as a civilization, a set of ideals, and the
object of hope for the potentials of humanity has been the curriculum of the humanities and "soft" social sciences. Given these ineffably sad phenomena, the seeming triumph of the West
(both the collapse of neo-Marxist theory at universities outside the West, and especially the downfall of the Soviet empire) will be understood by Western intellectuals as showing, in the latter
case, how absurd Western fears were from the start, and, in both cases, not so much a victory for the West as merely the economic collapse of communists who in various ways betrayed their
ideals or failed to temper them with adequate pragmatism or relativism. One must recall, however, the years 1975-76 in the world of the intellectual Left: the joy at American defeat in
Indochina; the excitement over Eurocommunism; the anticipation of one, ten, a hundred Vietnams; the contempt for Jean-Francois Revel's The Totalitarian Temptation; the ubiquitous
theories of moral equivalence; the thrill Of hammers and sickles in Portugal; the justifications of the movement of Cuban troops into those great hopes for mankind, Angola and Mozambique;
the loathing of all efforts to preserve Western strategic superiority or even parity. One must recall, indeed, the early 1980s: the romanticization of the kleptomaniacal and antidemocratic
Castroite Sandinistas and the homicidal megalomaniac Mengistu of Ethiopia; the demonization of Reagan's foreign policy; the outrage when Susan Sontag declared the audience of Reader's
Digest better informed than readers of The Nation about the history of the USSR; the mockery of the president's description of the Soviet Union as the "evil empire" and of communism as a
vision that would end on "the dustbin of history"; and the academic associations that approved politically correct resolutions for a nuclear freeze. The latter included the American Historical
Association, which voted in overwhelming numbers to inform the American government and public that, as professional historians, they knew that Reagan's rearmament program and
deployment of missiles in Europe would lead to a severe worsening of U.S.-Soviet relations, end the possibilities of peace, and culminate in an exchange of weapons in an ineluctable conflict.

The initial
All of that will be rewritten, forgotten, indirectly justified, and incorporated into a world view that still portrays the West as empire and the rest of the world as victim.

appeal of communism and romanticized Third World leaders--Kwame Nkrumah, Julius Nyerere, Mao Zedong, Ho Chi Minh, Fidel
Castro, Sekou Toure, and Daniel Ortega--who would redefine human well-being and productivity (well, they certainly redefined something) reflected the
Western pathology whereby intellectuals delude themselves systematically about the non-West, about
that "Other" standing against and apart from the society that does not appreciate those intellectuals' moral and practical authority and status.
However, when an enemy arose that truly hated Western intellectuals--namely, fascism--and whose defeat depended upon the West's self-belief, Western intellectuals quickly became
masters of judgments of absolute superiority and had no difficulty in defining a contest between good and evil. Cognitive dissonance is an astonishing phenomenon, and in academic circles, it

the most murderous regime in all of human history, the Bolsheviks in


prevents three essential historical truths from being told. First,

power, has fallen: its agents were guilty of irredeemable crimes against humanity, and its apologists
should do penance for the remainder of their lives. Anticommunists within the law were warriors for human freedom; communists
and anti-anticommunists, whatever their intentions, were warriors for human misery and slavery. The most that can be said in
communism's favor is that it was capable of building, by means of, slave labor and terror, a simulacrum of Gary, Indiana,
once only, without ongoing maintenance, and minus the good stuff. Secondly, voluntary exchange among individuals held morally
responsible under the rule of law has demonstrably created the means of both prosperity and diverse
social options. Such a model has been a precondition of individuation and freedom, whereas regimes of
central planning have created poverty, and (as Hayek foresaw) ineluctable developments toward totalitarianism
and the worst abuses of power. Dynamic free-market societies, grounded in rights-based individualism, have altered the entire human
conception of freedom and dignity for formerly marginalized groups. The entire "socialist experiment," by contrast, ended in stasis, ethnic hatreds, the absence
Thirdly,
of even the minimal preconditions of economic, social, and political renewal, and categorical contempt for both individuation and minority rights.
the willingness to contain communism, to fight its expansion overtly and covertly, to sacrifice wealth and
often lives against its heinous efforts at extension--in Europe, Vietnam, Central Asia, Central America, Korea, Laos, Cambodia,
and, indeed, Grenada--was, with the struggle against Nazism over a much briefer period, the great gift of American taxpayers and
the American people to planet earth. As Britain under Churchill was "the West" in 1940, so was the United States from 1945 to 1989, drawing from its values to
stand against what was simultaneously its mutant offspring and its antithesis. In the twentieth century, the West met and survived its greatest trial. On the whole, however, Western
intellectuals do not revel in these triumphs, to say the least. Where is the celebration? Just as important, where is the accounting? On the Left, to have either would be to implicate one's own
thought and will in the largest crime and folly in the history of mankind. We have seen myriad documentaries on the collective and individual suffering of the victims of Nazism, but where is

the countless
the Shoah, or the Night and Fog, let alone the Nuremberg trails of the postcommunist present? As Solzhenitsyn predicted repeatedly in The Gulag Archipelago,

victims who froze to death or were maimed in the Arctic death camps would go unremembered; the
officers and guards who broke their bodies and often their souls would live out their lives on pensions,
unmolested; and those who gave the orders would die peacefully and unpunished. Our documentary makers and moral intellectuals do not let us forget any
victim of the Holocaust. We hunt down ninety-year-old guards so that the bones of the dead might have justice, and properly so.
The bones of
Lenin's and Stalin's and Brezhnev's camps cry out for justice, as do the bones of North Vietnam's
exterminations, and those of Poi Pot's millions, and Mao's tens of millions. In those cases, however, the same intellectuals cry out
against--what is their phrase?--"witch-hunts," and ask us to let the past be the past. We celebrated the millennium with jubilation; we have not yet celebrated the triumph of the West. Ask

Page 987 of 1481


American high school or even college students to number Hitler's victims and Columbus's victims, and they will answer, for both, in the tens of millions. Ask them to number Stalin's victims
and, if my experience is typical, they will answer in the thousands. Such is their education, even now. The absence of celebration, of teaching the lessons learned, and of demands for

Convinced that the West above all has been the source of artificial relationships of
accountability is perhaps easily understood on the Left.

dominance and subservience, the commodification of human life, and ecocide, leftist intellectuals have little
interest in objectively analyzing the manifest data about societies of voluntary exchange, or in coming to terms with the
slowly and newly released data about the conditions of life and death under the Bolsheviks and their heirs, or in confirming or refuting various theories on the outcome of the Cold War (let
alone, given their contemporary concerns, in analyzing ecological or gender politics under communist or Third World regimes). Less obvious, but equally striking in some ways, has been the
absence of celebration on so much of the intellectual Right, because it is not at all certain something worth calling Western civilization did in fact survive the twentieth century.

Page 988 of 1481


Page 989 of 1481
Humanism Solves Genocide
STRENGTHENING HUMAN VIRTUES IS THE ONLY WAY TO MAINTAIN PEACE
SOLVES FURTHER HOLOCAUSTS

Ketels, Assoc Prof of English @ Temple U, 96 (Violet B., Havel to the Castle! The Power of the Word, 548 Annals
45, November, Lexis)

[*46] THEpolitical bestiality of our age is abetted by our willingness to tolerate the deconstructing of
humanist values. The process begins with the cynical manipulation of language. It often ends in stupefying
murderousness before which the world stands silent, frozen in impotent "attentism"--a wait-and-see
stance as unsuited to the human plight as a pacifier is to stopping up the hunger of a starving child. We
have let lapse our pledge to the 6 million Jewish victims of the Holocaust that their deaths might
somehow be transfiguring for humankind. We allow "slaughterhouse men" tactical status at U.N. tables
and "cast down our eyes when the depraved roar past." n1 Peacemakers, delegated by us and circumscribed by our fears,
temporize with thugs who have revived lebensraum claims more boldly than Hitler did. In the Germany of the 1930s, a demonic idea was born in a demented
brain; the word went forth; orders were given, repeated, widely broadcast; and men, women, and children were herded into death camps. Their offshore
signals, cries for help, did not summon us to rescue. We had become inured to the reality of human suffering. We could no longer hear what the words meant
or did not credit them or not enough of us joined the chorus. Shrieking victims perished in the cold blankness of inhumane silence. We were deaf to the
apocalyptic urgency in Solzhenitsyn's declaration from the Gulag that we must check the disastrous course of history. We were heedless of the lesson of his
only the unbending strength of the human spirit, fully taking its stand on the shifting frontier of
experience that
encroaching violence and declaring "not one step further," though death may be the end of it--only this
unwavering firmness offers any genuine defense of peace for the individual, of genuine peace for
mankind at large. n2 In past human crises, writers and thinkers strained language to the breaking point to keep alive the memory of the unimaginable,
to keep the human conscience from forgetting. In the current context, however, intellectuals seem more devoted to abstract assaults on values than to
thoughtful probing of the moral dimensions of human experience. "Heirs of the ancient possessions of higher knowledge and literacy skills," n3 we
seem
to have lost our nerve, and not only because of Holocaust history and its tragic aftermath. We feel
insecure before the empirical absolutes of hard science. We are intimidated by the "high modernist rage
against mimesis and content," n4 monstrous progeny of the union between Nietzsche and philosophical
formalism, the grim proposal we have bought into that there is no truth, no objectivity, and no
disinterested knowledge. n5 Less certain about the power of language, that "oldest flame of the [*47]
humanist soul," n6 to frame a credo to live by or criteria to judge by, we are vulnerable even to the discredited
Paul de Man's indecent hint that "wars and revolutions are not empirical events . . . but 'texts' masquerading
as facts." n7 Truth and reality seem more elusive than they ever were in the past; values are pronounced to
be mere fictions of ruling elites to retain power. We are embarrassed by virtue. Words collide and crack under these new
skeptical strains, dissolving into banalities the colossal enormity of what must be expressed lest we forget. Remembering for the future has become doubly dispiriting by our having to
remember for the present, too, our having to register and confront what is wrong here and now. The reality to be fixed in memory shifts as we seek words for it; the memory we set down is
flawed by our subjectivities. It is selective, deceptive, partial, unreliable, and amoral. It plays tricks and can be invented. It stops up its ears to shut out what it does not dare to face. n8 Lodged
in our brains, such axioms, certified by science and statistics, tempt us to concede the final irrelevance of words and memory. We have to get on with our lives. Besides, memories
reconstructed in words, even when they are documented by evidence, have not often changed the world or fended off the powerful seductions to silence, forgetting, or denying. Especially
denying, which, in the case of the Holocaust, has become an obscene industry competing in the open market of ideas for control of our sense of the past. It is said that the Holocaust never
happened. Revisionist history with a vengeance is purveyed in words; something in words must be set against it. Yet what? How do we nerve to the task when we are increasingly disposed to
cast both words and memory in a condition of cryogenic dubiety? Not only before but also since 1945, the criminality of governments, paraded as politics and fattening on linguistic
manipulation and deliberately reimplanted memory of past real or imagined grievance, has spread calamity across the planet. "The cancer that has eaten at the entrails of Yugoslavia since
Tito's death [has] Kosovo for its locus," but not merely as a piece of land. The country's rogue adventurers use the word "Kosovo" to reinvoke as sacred the land where Serbs were defeated by
Turks in 1389! n9 Memory of bloody massacres in 1389, sloganized and distorted in 1989, demands the bloody revenge of new massacres and returns civilization not to its past glory but to its
gory tribal wars. As Matija Beckovic, the bard of Serb nationalism, writes, "It is as if the Serbian people waged only one battle--by widening the Kosovo charnel-house, by adding wailing upon
wailing, by counting new martyrs to the martyrs of Kosovo. . . . Kosovo is the Serbianized [*48] history of the Flood--the Serbian New Testament." n10 A cover of Suddeutsche Zeitung in 1994

We stand benumbed before


was printed with blood donated by refugee women from Bosnia in an eerily perverse afterbirth of violence revisited. n11

multiplying horrors. As Vaclav Havel warned more than a decade ago, regimes that generate them "are the avant garde of
a global crisis in civilization." The depersonalization of power in "system, ideology and apparat,"
pathological suspicions about human motives and meanings, the loosening of individual responsibility, the
swiftness by which disastrous events follow one upon another "have deprived us of our conscience, of our
common sense and natural speech and thereby, of our actual humanity." n12 Nothing less than the
transformation of human consciousness is likely to rescue us.

Page 990 of 1481


Page 991 of 1481
A2 Reject Technology: 2AC
THE PROBLEM ISNT TECHNOLOGY, BUT OUR RELATIONSHIP TO IT. REJECTING IT
MASKS THE PROBLEM AND ONLY ACKNOWLEDGING ITS INCOMPLETION CAN
TRAVERSE ITS HOLD OVER US

Daniels 2002
[Michael D., Trinity University, Heidegger and Nazism, Prometheus: The John Hopkins Student Journal of
Philosophy, Vol. I, Fall 2002, http://www.jhu.edu/prometheus/Heidegger%20and%20Nazism.pdf, acc 10-11-
04//uwyo-ajl]

Despite his grave concerns about technology, he was never simply an opponent

of it nor did he seek its abolition or destruction. The problem, Heidegger believed, was

not technology per se, but the hegemony that technology had come to exercise over

human action. Techne as a form of uncovering reveals the world as a process of

production. Everything within the world is thus merely the equipment with which this

productive enterprise is carried out. Modern man imagines that technology produces

goods to satisfy his wants and desires, providing a nice lifestyle. Technology, however,

can only serve human beings if they live according to something other than technical

and economic imperatives. Only if distinctively human action is placed at the center of

our concern will technology serve our ends. We can only become active, as opposed to

productive, beings if we are guided by phronesis. Phronetic insight, however, is only

possible if we resolutely face the possibility of our own death and accept the destiny

that is revealed in the moment of vision. Thus, we must resolve ourselves to face the

question of Being. Without resolve to do this, we will lose the capacity for action and

become mere cogs in the equipment that constitutes the world uncovered by techne.

Page 992 of 1481


Page 993 of 1481
A2 Spanos: 2AC (1/3)
SPANOS DEMAND FOR PURITY ENSURES HIS MARGINALIZATION AND
FORECLOSES ON COALITION BUILDING ONLY THE PERM OFFERS A WAY OUT

Perkin 93
[J. Russell, Prof. English @ St. Marys, Theorizing the Culture Wars, Postmodern Culture 3: 3, 1993,
Muse//uwyo]

My final criticism is that Spanos, by his attempt to put all humanists into the same category and to break
totally with the tradition of humanism, isolates himself in a posture of ultraleftist purity that cuts him off
from many potential political allies, especially when, as I will note in conclusion, his practical
recommendations for the practical role of an adversarial intellectual seem similar to those of the liberal
pluralists he attacks. He seems ill-informed about what goes on in the everyday work of the academy, for
instance, in the field of composition studies. Spanos laments the "unwarranted neglect" (202) of the work
of Paulo Freire, yet in reading composition and pedagogy journals over the last few years, I have noticed
few thinkers who have been so consistently cited. Spanos refers several times to the fact that the
discourse of the documents comprising The Pentagon Papers was linked to the kind of discourse that first-
year composition courses produce (this was Richard Ohmann's argument); here again, however, Spanos is
not up to date. For the last decade the field of composition studies has been the most vigorous site of the
kind of oppositional practices The End of Education recommends. The academy, in short, is more diverse,
more complex, more genuinely full of difference than Spanos allows, and it is precisely that difference
that neoconservatives want to erase.

By seeking to separate out only the pure (posthumanist) believers, Spanos seems to me to ensure his self-
marginalization. For example, several times he includes pluralists like Wayne Booth and even Gerald Graff
in lists of "humanists" that include William Bennett, Roger Kimball and Dinesh D'Souza. Of course, there is
a polemical purpose to this, but it is one that is counterproductive. In fact, I would even question the
validity of calling shoddy and often inaccurate journalists like Kimball and D'Souza with the title "humanist
intellectuals." Henry Louis Gates's final chapter contains some cogent criticism of the kind of position
which Spanos has taken. Gates argues that the "hard" left's opposition to liberalism is as mistaken as its
opposition to conservatism, and refers to Cornel West's remarks about the field of critical legal studies,

"If you don't build on liberalism, you build on air" (187). Building on air seems to me precisely what
Spanos is recommending. Gates, on the other hand, criticizes "those massively totalizing theories that
marginalize practical political action as a jejune indulgence" (192), and endorses a coalition of liberalism
and the left.

PERM DO BOTH
SPANOS ALONE ISNT EMANCIPATORY COMBINING THE CRITICISM WITH
PROBLEM SOLVING IS OPTIMAL

Lewandowski 94
[John, Prof @ SUNY Binghamton, Philosophy and Social Criticism 20, 119]

Page 994 of 1481


Spanos rightly rejects the textuality route in Heidegger and Criticism precisely because of its totalizing
and hypostatizing tendencies. Nevertheless, he holds on to a destructive hermeneutics as disclosure. But
as I have already intimated, disclosure alone cannot support a critical theory oriented towards
emancipation. I think a critical theory needs a less totalizing account of language, one that articulates
both the emphatic linguistic capacity to communicate, solve problems in and criticize the world. The
essential task of the social critic and any literary theory that wants to be critical is to couple world
disclosure with problem-solving, to mediate between the extra-ordinary world of textuality and the
everyday world of texts. In this alternative route, literary theory may become the kind of emancipatory
oriented critical theory it can and should be.

Page 995 of 1481


Page 996 of 1481
A2 Spanos: 2AC (2/3)

Page 997 of 1481


Page 998 of 1481
A2 Spanos: 2AC (3/3)

Page 999 of 1481


**Human Rights Bad (Imperialism)**

Page 1000 of 1481


HR Bad Answers: 2AC (1/4)
FIRST, NO LINK WE DONT DICTATE THE INTERNATIONALIZATION OF RIGHTS.
OUR EV JUST SHOWS THAT OTHERS WILL VOLUNTARILY MODEL PLAN

SECOND, WE CONTROL UNIQUENESS ZAKARIA SHOWS THAT DEMOCRATIC


PROMOTION IS INEVITABLE NOW. WE JUST SEND A BETTER SIGNAL THAT
STABILIZES STATUS QUO TRANSITIONS

THIRD, ESSENTIALISM TURN


A. THEIR ARGUMENT ESSENTIALIZES NON-WESTERN CULTURES, DEPRIVING THEM
OF AGENCY

Mered 96
[Sohail, JD Candidate @ Western Reserve University School of Law, Its Not a Cultural Thing: Disparate Domestic
Enforcement of Internaitonal Criminal Procedure Standards A Comparison of the United States and Egypt, Case
Western Reserve Journal of International Law, Winter, LN//uwyo-ajl]

. Most significant and dangerous is the assumption by relativists that a culture is


The cultural relativist position fails in several ways

monolithic. Their reliance on stereotypes of entire races, ethnicities, and religions stems from that
assumption. n65 The result is an argument which must fail because of its oversimplification. No culture can be viewed as a homogeneous
grouping of people; nor can religion alone characterize a culture. Relativists like to refer to the "Islamic culture," thereby obliterating
significant cultural differences which exist among peoples from Morocco to Indonesia (passing through some sub-Saharan African nations, such as Nigeria). n66 These
cultural differences, due to the diversity of race, and ethnicity, as well as historical experience, all give insight into the way these different states may behave. A simplistic

scholarly argument which conveniently overlooks intricacies and complexities necessarily raises suspicions and
destroys itself.

B. THIS MIMICS COLONIZATION, ENTRENCHING OPPRESSION

Butler 99
[Judith, prof. of rhetoric at UC Berkeley, Gender Trouble: Feminism and the
Subversion of Identity, New York: Routledge, 1999, 18-19//uwyo-ajl]

Feminist critique ought to explore the totalizing claims of a masculinist signifying economy, but also remain self-critical with respect to the totalizing gestures of feminism. The effort
to identify the enemy as singular in form is a reverse-discourse that uncritically mimics the strategy of the
oppressor instead of offering a different set of terms. That the tactic can operate in feminist and antifeminist contexts alike suggests that the colonizing gesture is not primarily or
irreducibly masculinist. It can operate to effect other relations of racial, class, and heterosexist subordination , to name but a few. And clearly, listing the varieties of oppression, as I began to
do, assumes their discrete, sequential coexistence along a horizontal axis that does not describe their convergences within the social field. A vertical model is similarly insufficient; oppressions
cannot be summarily ranked, causally related, distributed among planes of originality and derivativeness. Indeed, the field of power structured in part by the imperializing gesture of

Page 1001 of 1481


dialectical appropriation exceeds and encompasses the axis of sexual difference, offering a mapping of intersecting differentials which cannot be summarily hierarchized either within the

Rather than an exclusive tactic of masculinist


terms of phallogocentrism or any other canddidate for the position of primary condition of oppression.

signifying economies, dialectical appropriation and suppression of the Other is one tactic among many
deployed centrally but not exclusively in the service of expanding and rationalizing the masculinist domain.

FOURTH, NO ALTERNATIVE CROSS-CULTURAL EXCHANGE IS INEVITABLE IN A


GLOBALIZING WORLD, PREVENTING CULTURAL AUTHENTICITY

Page 1002 of 1481


Page 1003 of 1481
HR Bad Answers: 2AC (2/4)
FIFTH, THE ALTS RELATIVISM RISKS CRUSHING TOTALITARIANISM

Farber & Sherry 95


[Daniel A., Henry J. Fletcher Professor of Law and Assoc Dean of Faculty @ U. of Minnesota, & Suzanna, Earl R. Larson
Prof. of Civ Rights and Civ Liberties Law @ U. of Minnesota, Is the Radical Critique of Merit Anti-Semitic? California
Law Review, May, LN//uwyo-ajl]

This unsettling possible alignment of radical constructivism with the worst totalitarian regime of this century should also -
upon reflection - seem less than shocking. n147 The core of the radical constructivist paradigm is a rejection of the Enlightenment and its emphasis on rationality and
scientific explanation. n148 Instead, radical constructivists seek to explain the world solely as the result - deliberate or unconscious - of ideology and the pursuit of dominance. But that
standard leaves little room for shared concepts of merit, morality, or anything else. n149 As other scholars have noted, radical constructivism "leaves no ground whatsoever for distinguishing

it can readily slide into


reliable knowledge from superstition." n150 As a feminist philosopher who sympathizes with what we have called radical constructivism has warned,

moral relativism n151 - only one step away from relying on raw power to determine truth. For if ideas are mere
reflections of the exercise of power, it becomes difficult to find a basis for criticizing social arrangements. And if raw power is the test

of truth, totalitarians are merely the most unabashed constructors of reality. Much as radical constructivists may dislike this conclusion,
its potential is present in their conceptual apparatus.

SIXTH, TURN PLAN SOLVES THE WORSE IMPERIALISM OF FORCIBLY DETAINING


ENEMY COMBATANTS FOR LIFE WITHOUT DUE PROCESS

SEVENTH, TURN: THEIR PATRONIZING RESPECT FOR OTHERS IS RACISM UNDER


ANOTHER GUISE AND SMOOTHES THE PATH FOR CAPITAL-DRIVEN DESTRUCTION

Zizek '99
[Slavoj, Senior Researcher at Institute for Social Studies, Ljubliana and Badass, The
Ticklish Subject: the absent centre of political ontology, New York: Verso, 1999, 215-6

How, then, does the universe of Capital relate to the form of nation-state in our era of global capitalism? Perhaps this relationship is best designated as 'autocolonization': with the direct
multinational functioning of Capital, we are no longer dealing with the standard opposition between metropolis and colonized countries; a global company, as it were, cuts its umbilical cord
with its mother-nation and treats its country of origin as simply another territory to be colonized. This is what is so disturbing to patriotically orientated right-wing populists, from Le Pen to
Buchanan: the fact that the new multinationals have exactly the same attitude towards the French or American local population as towards the population of Mexico, Brazil or Taiwan. Is there
not a kind of poetic justice in this self-referential turn of today's global capitalism, which functions as a kind of 'negation of negation', after national capitalism and its internationalist!
colonialist phase? At the beginning (ideally, of course), there is capitalism within the confines of a nation-state, and with the accompanying international trade (exchange between sovereign

the
nation-states); what follows is the relationship of colonization, in which the colonizing country subordinates and exploits (economically, politically, culturally) the colonized country;

final moment of this process is the paradox of colonization, in which there are only colonies, no colonizing
countries - the colonizing power is no longer a nation-state but the global company itself. In the long term, we shall all not only wear Banana Republic shirts
but also live in banana republics.

the ideal form of ideology of this global capitalism is multiculturalism, the attitude which, from a kind of
And, of course,

empty global position, treats each local culture as the colonizer treats colonized people - as 'natives' whose mores are
to be carefully studied and 'respected'. That is to say: the relationship between traditional imperialist colonialism and global capitalist self-colonization is exactly the same as the relationship
between Western cultural imperialism and multiculturalism - just as global capitalism involves the paradox of colonization without the colonizing nation-state metropolis,

multiculturalism involves a patronizing Eurocentrist distance and/or respect for local cultures without
roots in one's own particular culture. In other words, multiculturalism is a disavowed, inverted, self-referential
form of racism, a 'racism with a distance' - it 'respects' the Other's identity, conceiving the Other as a self-enclosed 'authentic'

Page 1004 of 1481


community towards which the multiculturalist maintains a distance made possible by his/her privileged
universal position. Multiculturalism is a racism which empties its own position of all positive content' (the multiculturalist is not a direct racist; he or she does not oppose the
the Other the particular values of his or her own culture); none the less he or she retains this position as the privileged empty point of universality from which one is able to appreciate (and

multiculturalist respect for the Other's specificity is the very form of asserting
depreciate) other particular cultures properly -

one's own superiority. Pursuing multiple perspectives legitimizes racism and disables us from solving
ecological and social disasters

Page 1005 of 1481


Page 1006 of 1481
HR Bad Answers: 2AC (3/4)
EIGHTH, PERM DO BOTH
COMBINING UNIVERSALITY AND RELATIVISM CHECKS ETHNOCENTRISMAND
LOCAL REPRESSION

Donoho 2001
[Douglas Lee, Prof. Law @ Southeastern University, Autonomy, Self-Governance,
and the Margin of Appreciation: Developing a Jurisprudence of Diversity within
Universal Human Rights, 15 Emory International Law Review 391, Fall, LN//uwyo-ajl]

On the other side of the debate, reliance on relativism by non-Western states and scholars reflects a mixture of contrasting motives .
For
some repressive regimes, the lure of relativism undoubtedly lies in its potential for
deflecting international scrutiny. Universalists' deep suspicion regarding the motives of those who champion relativism
seems well founded. Indeed, prominent among states promoting relativism at the World Conference in Vienna were those on the short list of
the World's most egregious violators - measured on virtually any scale - of basic human dignity. n63 Only a true Pollyanna would fail to suspect
thatmany of these government's espoused claims of cultural or [*414] religious
imperative are nothing more than cynical manipulations meant to undermine the
effectiveness of rights. Yet the appeal of relativism is hardly limited to repressive governments. Especially among non-
Westerners, arguments about relativism are a reflection of something far more profound than
the misleading, "either/or" dichotomy of universal versus relative rights. For many, the appeal of a
seemingly relativist perspective is simply a means to advocate genuine concern over the cultural, social, and political domination of Western values. n64 It similarly reflects
an understandable desire to preserve local traditions and values - a desire that on some level clearly conflicts with progressive human rights development and may serve as
the unwitting ally of oppression. n65 Finally, the relativist perspective may be used to promote self-governance and autonomy - the prerogative to develop the specific
meaning of human rights, in accordance with local terms of reference. n66 To a significant extent, genuine concerns for diversity, pluralism and local autonomy have been
obscured by the West's legitimate fear that "relativism" could serve as the "last refuge for oppression." n67 The "relativist" label has thus become, in the [*415] words of

, the fears and corresponding rhetoric of the West


Makau Wa Mutua, a bit like "human rights name-calling." n68 In this sense

have created a misleading oppositional narrative that obscures the real and difficult
issues that genuine diversity poses for the international human rights system. n69 As is often
true in political debates, the competing motivations of universalist and relativist governments have been manifested in arguments imprecisely
cast in "either/or" terms; that is, all rights are, in all of their manifestations, either universal or relative. Yet one plausible reading of the
compromise language of the Vienna Declaration suggests that this is a false dichotomy. Rather, it may be that the Vienna
Declaration reflects the notion that international human rights can be simultaneously universal and
variant.

NINTH, TURN: THEIR RELATIVISM IS ITSELF ETHNOCENTRIC

Morgan-Foster 03
[Jason, JD Cand at U. of Michigan School of Law, A New Perspective on the Universality DebA2 Reverse Moderate
Relativism in the Islamic Context, ILSA Journal of Intl and Comparative Law, Fall, LN//uwyo-ajl]

: the notion that all values are culturally relative, the belief in "the
Strict cultural relativism has been criticized as self-contradictory

equal dignity and worth of all cultures," or "the equal right of all peoples to participate in the formation of international law" are themselves
culturally shaped value judgments, which would be void under the cultural relativist's own theory. There is no
reason for cultural relativists to accept these starting points as universal in order to support a doctrine which denies the legitimacy of [*43] universals. n37 From a normative human rights
perspective, strict cultural relativism is also questionable because it has little to no support in human rights conventions. The only treatment of strict cultural relativism in a human rights
convention is article 63(3) of the European Convention on Human Rights, which says that "[t]he provisions of this Convention shall be applied in [colonial territories] with due regard, however,
to local requirements." n38 A strict cultural relativist reading of this provision has been rejected by the European Court of Human Rights in Tyrer v. United Kingdom, where the local custom of
corporal punishment was at issue. n39

Page 1007 of 1481


Thus, because of the logical self-contradiction inherent in strict cultural relativism, and because of the virtual complete lack of support for strict cultural relativism in the human rights
discourse, strict cultural relativism fails as a paradigm to conceptualize the universality discourse.

Page 1008 of 1481


Page 1009 of 1481
HR Bad Answers: 2AC (4/4)
TENTH, TURN GROUNDING RESISTANCE TO A FALLEN IDENTITY OF PRE-WESTERN
INTERVENTION RENDERS THE COLONIZED PASSIVE VICTIMS WITHOUT AGENCY
ACTIVISM WITHIN COLONIALISM USES ITS OWN EXCESSES TO DISMANTLE IT

Zizek '99
[Slavoj, Senior Researcher at Institute for Social Studies, Ljubliana and Badass, The Ticklish Subject: the absent centre
of political ontology, New York: Verso, 1999, 256-7//uwyo-ajl]

Against Butler, one is thus tempted to emphasize that Hegel was well aware of the retroactive
process by means of which oppressive power itself generates the form of resistance is not this
very paradox contained in Hegel's notion of positing the presuppositions, that is, of how the
activity of positing-mediating does not merely elaborate the presupposed immediate-natural
Ground, but thoroughly transforms the very core of its identity? The very In-itself to which
Chechens endeavour to return is already mediated-posited by the process of modernization,
which deprived them of their ethnic roots.

This argumentation may appear Eurocentrist, condemning the colonized to repeat the European
imperialist pattern by means of the very gesture of resisting it however, it is also possible to
give it precisely the opposite reading. That is to say: if we ground our resistance to imperialist
Eurocentrism in the reference to some kernel of previous ethnic identity, we automatically
adopt the position of a victim resisting modernization, of a passive object on which imperialist
procedures work. If, however, we conceive our resistance as an excess that results from the way
brutal imperialist intervention disturbed our previous self-enclosed identity, our position
becomes much stronger, since we can claim that our resistance is grounded in the inherent
dynamics of the imperialist system that the imperialist system itself, through its inherent
antagonism, activates the forces that will bring about its demise. (The situation here is strictly
homologous to that of how to ground feminine resistance: if woman is 'a symptom of man', the
locus at which the inherent antagonisms of the patriarchal symbolic order emerge, this in no
way constrains the scope of feminine resistance but provides it with an even stronger
detonating force.) Or to put it in yet another way the premise according to which

resistance to power is inherent and immanent to the power edifice (in the sense that it is
generated by the inherent dynamic of the power edifice) in no way obliges us to draw the
conclusion that every resistance is co-opted in advance, including in the eternal game Power
plays with itself the key point is that through the effect of proliferation, of producing an excess
of resistance, the very inherent antagonism of a system may well set in motion a process which
leads to its own ultimate downfall.

It seems that such a notion of antagonism is what Foucault lacks: from the fact that every
resistance is generated ('posited') by the Power edifice itself, from this absolute inherence of
resistance to Power, he seems to draw the conclusion that resistance is co-opted in advance,
that it cannot seriously undermine the system that is, he precludes the possibility that the
system itself, on account of its inherent inconsistency, may give birth to a force whose excess it
is no longer able to master and which thus detonates its unity, its capacity to reproduce itself. In
short, Foucault does not consider the possibility of an effect escaping, outgrowing its cause, so
that although it emerges as a form of resistance to power and is as such absolutely inherent to
it, it can outgrow and explode it. (the philosophical point to be made here is that this is the
fundamental feature of the dialectical-materialist notion of 'effect': the effect can 'outdo' its

Page 1010 of 1481


cause; it can be ontologically 'higher' than its cause.) One is thus tempted to reverse the
Foucauldian notion of an all-encompassing power edifice which always-already contains its
transgression, that which allegedly eludes it: what if the price to be paid is that the power
mechanism cannot even control itself, but has to rely on an obscene protuberance at its very
heart? In other words: what effectively eludes the controlling grasp of Power is not so much the
external In-itself it tries to dominate but, rather, the obscene supplement which sustains its own
operation.

Page 1011 of 1481


#3 Essentialism Turn: 1AR

MUTUA ESSENTIALIZES NON-WESTERN CULTURES, RE-CREATING THE HIERARCHY


THAT THEY CRITICIZE. REIFYING AN IMAGINED MACRO CULTURE ESTABLISHES THE
FRAMEWORK WHERE SUBORDINATION OF BOTH INDIVIDUALS AND CULTURES.

THIS TURNS THE CRITIQUE, AND ACTS AS A SOLVENCY TAKEOUT THE


IMPLICATIONS OF THE K ARE INEVITABLE.

THEIR ARGUMENT ESSENTIALIZES NON-WESTERN CULTURES, SUPPRESSING THOSE


WHO DEMAND HUMAN RIGHTS

Mered 96
[Sohail, JD Candidate @ Western Reserve University School of Law, Its Not a
Cultural Thing: Disparate Domestic Enforcement of Internaitonal Criminal Procedure
Standards A Comparison of the United States and Egypt, Case Western Reserve
Journal of International Law, Winter, LN//uwyo-ajl]

In stereotyping entire peoples, and assuming a monolithic cultural thought, relativists fail to account for
the dissenters in a society (or as relativists would characterize it, a culture), who are the intellectuals and
the individuals who claim their human rights have been abused. Relativists discount the intellectual
dissent as Westernized. n71 The simplicity of this argument is intellectually dishonest. The entire
intellectual voice of a country cannot be so flippantly ignored and discounted. It is the perfect argument
from the relativists' point of view since the crux of [*153] their position is that any Western idea is simply
inapplicable to the nonWest, so any non-Westerner who espouses ideas determined to be Western can
be discounted as Westernized and thus irrelevant to the debate about his or her own culture. The
intellectual elite in the West is regarded as the most eloquent representative of their societies. Relativists
have offered no convincing argument why non-Western elites do not represent their societies. The only
adequate retort to this dismissal of non-Western elites is to prove that the human rights are universal,
and thus the intellectual dissenters are necessarily only espousing values of their own culture. At the 1993
U.N. World Conference on Human Rights, the Dalai Lama, leader of Tibetan Buddhism, stated that it was
in "the inherent nature of all human beings to yearn for freedom, equality, and dignity." n72 A more
peculiar aspect of the relativist position is the minimal importance it lends to the individual complainants
from the non-Western world who claim their government has violated their human rights. The relativists
fail to address the question of why there are individuals who complain if the rights allegedly violated are
foreign to their cultural concepts. Individuals claim the rights they know are inalienable because they are
human beings, regardless of their culture. These individual complainants cannot be dispensed as
Westernized intellectuals. They are perhaps the most effective argument in support of the position that
international human rights standards speak universally to all peoples. n73

Page 1012 of 1481


Page 1013 of 1481
#5 Relativist Apologism Turn:
1AR
THE APOLOGETIC NATURE OF THE KRITICISM IS AN ATTEMPT TO REGARNISH TIES
BETWEEN THE WEST AND EAST UNDER A FEAR OF ORIENTAL RETALIATION. THE
IMPLICATIONS OF THE CRITICISM INDICATE THAT MATUA AND HUMAN RIGHT
LIBERALS ARE AFRAID OF A STRONGER ORIENT THAT WILL BACKLASH AGAINST
THEIR OPPRESSERS, THIS APPOLIGETIC NATURE REASTABLISHES A NORM OF
WESTERN DOMINANCE, WITHOUT TAKING DOWN THE SOCIETAL NORMS THAT
ALLOWS FOR THESE HIERARCHIES

THEIR RELATIVISTIC POSITION MAKES IT IMPOSSIBLE TO CRITICIZE AND


DISMANTLE OPPRESSION, ALLOWING TOTALITARIANISM TO FILL THE VOID

Farber & Sherry 95


[Daniel A., Henry J. Fletcher Professor of Law and Assoc Dean of Faculty @ U. of
Minnesota, & Suzanna, Earl R. Larson Prof. of Civ Rights and Civ Liberties Law @ U. of
Minnesota, Is the Radical Critique of Merit Anti-Semitic? California Law Review,
May, LN//uwyo-ajl]

This unsettling possible alignment of radical constructivism with the worst totalitarian regime of this
century should also - upon reflection - seem less than shocking. n147 The core of the radical constructivist
paradigm is a rejection of the Enlightenment and its emphasis on rationality and scientific explanation.
n148 Instead, radical constructivists seek to explain the world solely as the result - deliberate or
unconscious - of ideology and the pursuit of dominance. But that standard leaves little room for shared
concepts of merit, morality, or anything else. n149 As other scholars have noted, radical constructivism
"leaves no ground whatsoever for distinguishing reliable knowledge from superstition." n150 As a feminist
philosopher who sympathizes with what we have called radical constructivism has warned, it can readily
slide into moral relativism n151 - only one step away from relying on raw power to determine truth. For if
ideas are mere reflections of the exercise of power, it becomes difficult to find a basis for criticizing social
arrangements. And if raw power is the test of truth, totalitarians are merely the most unabashed
constructors of reality. Much as radical constructivists may dislike this conclusion, its potential is present
in their conceptual apparatus

Page 1014 of 1481


Page 1015 of 1481
#8 Permutation: 1AR (1/3)
BALANCING RIGHTS AND PLURALITY SOLVES BEST

Donoho 2001
[Douglas Lee, Prof. Law @ Southeastern University, Autonomy, Self-Governance,
and the Margin of Appreciation: Developing a Jurisprudence of Diversity within
Universal Human Rights, 15 Emory International Law Review 391, Fall, LN//uwyo-ajl]

It is probable, therefore, that the elements of relativist language in the Vienna Declaration
reflect no more than a widespread desire among non-Western states to manifest local
preferences, preserve a degree of autonomy in the implementation of rights, and
promote diversity values. Thus, in its best light, such language is motivated by the idea that
the manifestation of human rights must somehow accommodate communal preferences and
recognize diversity and self-governance wherever possible without violating underlying
universal values. n81

This position suggests an undefined balance between universal values and local
preferences. At a minimum, the international community's continued emphasis on universality
demands that culturally based variations in [*421] rights must be compatible with, and
preserve, core universal values. It similarly requires that diversity and autonomy
concerns not undermine the progressive development of human rights or serve as an
excuse for oppression or uncritical preservation of the status quo.

The Vienna Declaration may be seen, therefore, as a practical compromise among


competing motivations. It essentially directs international institutions to accomplish a
difficult and delicate task - interpret the specific meaning of rights in ways that allow
diversity, self-governance, and autonomy, while maintaining core, universal human
rights values. In the case of select rights, such as those relating to the physical integrity of the individual, there may be little or no room
for variation. n82 For other rights, there may be little actual consensus over their specific meaning and significant potential for variations that
nevertheless preserve core universal values. n83 For still others, it may turn out that consensus is lacking even over the supposed core value
represented in the abstract normative standard. In such cases, the level of shared understanding over specific meaning may be so shallow as to
cast doubt on the existence of the right itself as a meaningful international standard.

ETHICAL UNIVERSALISM IS COMPATIBLE WITH CULTURAL DIVERSITY AND IS


NECESSARY TO FIGHT ETHNOCENTRISM

Tilley 2000
[John J., Assoc. Prof of Philosophy at Indiana University-Purdue University
Indianapolis, Cultural Relativism, Human Rights Quarterly, 22.2, The John Hopkins
University Press, 539//uwyo-ajl]

The trouble with the ethnocentrism argument is quite simple: to grant universalism is not to be
ethnocentric. In fact, it's consistent with universalism to advance the following as universally valid:

Page 1016 of 1481


"Ethnocentrism is immoral." So the ethnocentrism argument fails. The same goes for arguments that
substitute "imperialistic," "authoritarian," or "antipluralistic" for "ethnocentric." For example, although
universalism implies that some moral requirements are the same for everyone, it does not imply that we
all have a moral requirement to be the same, nor that we have any moral requirement that discourages
cultural diversity. Most likely, one of our main requirements is to respect such diversity (and hence to
respect cultural integrity). 67 Therefore, universalism is compatible with cultural pluralism.

Page 1017 of 1481


Page 1018 of 1481
#8 Permutation: 1AR (2/3)
A BLANKET REJECTION OF HEGEMONIC IDEALS ALSO REJECTS THE ONLY HOPE FOR
ENDING OPPRESSION. WE SHOULD COMBINE WESTERN ENLIGHTENMENT WITH
THE ALTERNATIVE.

Michael Thompson, 2003 Iraq, Hegemony, and the Question of the American Empire accessed online
http://www.logosjournal.com/thompson_iraq.htm

Hegemony in international terms without some kind of competing force, such as the Soviets, can clearly
lead to the abuse of power and a unilateralist flaunting of international institutions that do not serve at
the imperium's whim. But this should not mean that hegemony itself is a negative concept. Although
empire is something rightfully reviled, hegemony may not be as bad as everyone thinks. We need to
consider what is progressive and transformative in the ideas and values of the western republican and
liberal traditions. We need to advocate not an anti-hegemonic stance in form, but an anti-hegemonic and
anti-imperialist stance in content, one that advocates the particular interests of capital of the market in
more broad terms rather than the universal political interests of others. Rather than choose between
western hegemony on the one hand and political and cultural relativism on the other, we need to
approach this problem with an eye toward cosmopolitanism and what the political theorist Stephen Eric
Bronner has called "planetary life."

Simple resistance to American "imperial" tendencies is no longer enough for a responsible, critical and
rational left. Not only does it smack of tiers-mondisme but at the same time it rejects the realities of
globalization which are inexorable and require a more sophisticated political response. The real question I
am putting forth is simply this: is it the case that hegemony is in itself inherently bad? Or, is it possible to
consider that, because it can, at least in theory, consist of the diffusion of western political ideas, values
and institutions, it could be used as a progressive force in transforming those nations and regions that
have been unable to deal politically with the problems of economic development, political disintegration
and ethnic strife?

It is time that we begin to consider the reality that western political thought provides us with unique
answers to the political, economic and social problems of the world and this includes reversing the
perverse legacies of western imperialism itself. And it is time that the left begins to embrace the ideas of
the Enlightenment and its ethical impulse for freedom, democracy, social progress and human dignity on
an international scale. This is rhetorically embraced by neoconservatives, but it turns out to be more of a
mask for narrower economic motives and international realpolitik, and hence their policies and values run
counter to the radical impulses of Enlightenment thought. Western ideas and institutions can find
affinities in the rational strains of thought in almost every culture in the world, from 12th century
rationalist Islamic philosophers like Alfarabi, Avicenna (Ibn Sinna) and Averroes (Ibn Rushd) to India's King
Akbar and China's Mencius. The key is to find these intellectual affinities and push them to their concrete,
political conclusions.

Clearly, the left's problem with the idea of the spread of western political ideas and institutions is not
entirely wrong. There was a racist and violent precedent set by the French and English imperial projects
lasting well into the 20th century. The problem is in separating the form from the content of western
hegemonic motives and intentions. And it is even more incorrect to see the occupation of Iraq as a
symptom of western ideas and Enlightenment rationalism. Nothing could be further from the case and
the sooner this is realized, the more the left will be able to carve out new paths of critique and resistance
to a hegemony that is turning into empire.

Page 1019 of 1481


And it is precisely for this reason why, in institutional terms, the UN needs to be brought back in. Although
there are clearly larger political and symbolic reasons for this, such as the erosion of a unilateralist
framework for the transition from Hussein's regime, there is also the so-called "effect of empire" where
Iraq is being transformed into an instrument of ideological economics. The current U.S. plan for Iraq, one
strongly supported by Bremer as well as the Bush administration, will remake its economy into one of the
most open to trade, capital flows and foreign investment in the world as well as being the lowest taxed.
Iraq is being transformed into an neo-liberal utopia where American industries hooked up to the infamous
"military-industrial complex" will be able to gorge themselves on contracts for the development of
everything from infrastructure to urban police forces.

continued

Page 1020 of 1481


Page 1021 of 1481
#8 Permutation: 1AR (3/3)
continued

As time moves on, we are seeing that Iraq provides us with a stunning example of how hegemony
becomes empire. It is an example of how the nave intention of "nation building" is unmasked and laid
bare, seen for what it truly is: the forceful transformation of a sovereign state into a new form suited to
narrow western (specifically American) interests. Attempts to build a constitution have failed not from the
lack of will, but from the lack of any political discourse about what form the state should take and about
what values should be enshrined in law. Ruling bodies have become illegitimate almost immediately upon
their appointment because there exists almost complete social fragmentation, and the costs of knitting it
together are too great for America to assume.

In the end, America has become, with its occupation of Iraq and its unilateralist and militaristic posture,
an empire in the most modern sense of the term. But we should be careful about distinguishing empire
from a hegemon and the implications of each. And since, as Hegel put it, we are defined by what we
oppose, the knee-jerk and ineffectual response from the modern left has been to produce almost no
alternative at all to the imperatives that drive American empire as seen in places such as Iraq. To neglect
the military, economic and cultural aspects of American power is to ignore the extent to which it provokes
violent reaction and counter-reaction. But at the same time, to ignore the important contributions of
western political ideas and institutions and their power and efficacy in achieving peace and mutual
cooperation, whether it be between ethnic communities or whole nations themselves, is to ignore the
very source of political solutions for places where poverty, oppression and dictatorships are the norm and
remain stubbornly intact.

Page 1022 of 1481


Page 1023 of 1481
#10 Zizek Presymbolism: 1AR
(1/2)
THE KRITIK IS GROUNDED IN A GOAL TO RETURN CULTURES BACK TO THEIR
UNTOUCHED PRE-COLONIAL STATEONE CANNOT UNDUE THE EFFECTS OF
COLONIALISM. THE ALT OBLITERATES ANY COALITION BUILDING POSSIBILITIES AS
YOUR MOVEMENT IS PREMISED ON VICTIMHOOD. THE APARTHEID AND TAIWAN
ARE EXAMPLES OF SOCIETIES THAT WERE COLONIZED WHOS IDENTITIES AND
SOCIETIES ARE ALTERED INDEFINATELY.

AND, THE 1AC FUNCTIONS AS A CRITICISM OF IMPERIALISM, WE ARE THE EXCESS


OF OPPRESSION THAT LEADS TO IMPERIALISMS ULTIMATE COLLAPSE.

THE ZIZEK EVIDENCE IS A UNIQUE LINK TURN TO THEIR ALTERNATIVECOUNTER-


HEGEMONIC MOVEMENTS WILL COLLAPSE WESTERN IDEALISM NOW THE
ALTERNATIVE COLLAPSES THESE MOVEMENTS PERPETUATING THE SYSTEM
THROUGH REFORMIST MEASURES

THIS IS A NET BENEFIT TO THE PERMONLY A RISK THAT OUR ETHIC WILL BUILD
COALITIONS AROUND THE RIGHT TO LIFE TO TAKE DOWN VIOLENT SYSTEMS OF
HEGEMONY

Page 1024 of 1481


Page 1025 of 1481
#10 Zizek Presymbolism: 1AR
(2/2)
MULTICULTURALISM REINCRIBES EUROCENTRISM BY APPROPRIATING OTHER
CULTURES INTO THE WESTERN MACHINE SINCE WESTERN THOUGHT IS
INEVITABLE, ONLY AN ASSERTION OF ENLIGHTENMENT-BASED CONTINGENCY CAN
OVERCOME IMPERIALISM ***

Zizek '92
[Slavoj, Doesn't like sharing Chinese food, Enjoy Your Symptom! Jacques Lacan in
Hollywood and Out, New York City: Routledge, 1992, 184-5//uwyo-ajl]

Consequently, our position here is radically "Eurocentric": the break of the Enlightenment is
irreversible, the epoch of the Enlightenment is "an epoch to end all epochs," i.e., by means of
the Versagung which constitutes the subject of the Enlightenment, an abyss becomes visible
against the background of which all other epochs can be experienced in their epochal closure,
as something ultimately contingent. 59 The point is simply that the Enlightenment, like a
cancerous tissue, contaminates all preceding organic unity and changes it retroactively into an
affected pose. In Hegelese: as soon as we enter the Enlightenment, every presupposition (of an
organic ground) is suspected of being "posited." Suffice it to recall the returns to oriental
wisdom, the rejections of the so-called "Western Protestant-Cartesian imperialist paradigm,"
which abound today. Apropos of them, one usually emphasizes the need to distinguish
authentic cases of such "returns" from their commercialized distortions (newspaper ads for
"transcendental meditation," e.g.). Yet perhaps such an opposition is all too naive; perhaps what
appears as a commercialized distortion of the authentic oriental wisdom is today its truth;
perhaps the very "return to the lost oriental wisdom" is already in service of the late capitalist
social machine, facilitating the untroubled run of its nuts and bolts-perhaps we betrayed
"oriental wisdom" the moment we uprooted it from its pretechnologicallife world and
transfunctionalized it into an individual therapeutic means. In other words, here, also, the
dialectical maxim "the cleaner you are, ther dirtier you are" is in force: the more "truly" you
return to oriental wisdom, the more your effort contributes to the transformation of oriental
wisdom into a cog in the Western social machine. . . The reverse of it is that those who preach
"multicultural decenterment," "openness toward non-European cultures," etc., thereby
unknowingly affirm their "Eurocentrism," since what they demand is imaginable only within the
"European" horizon: the very idea of cultural pluralism relies on the Cartesian experience of the
empty, substanceless subjectivity-it is only against the background of this experience that every
determinate form of substantial unity can appear as something ultimately contingent.

Page 1026 of 1481


Page 1027 of 1481
No Link
OTHERS ADOPT US CULTURE BECAUSE IT REFLECTS THE DIVERSITY OF AMERICAN
SOCIETY AND, NATIONS REALIZE THAT THERE IS NO ALTERNATIVE TO US
IDEOLOGY

Victor Davis Hanson, Ph. D. in Classics, Senior Fellow at the Hoover Institution, Stanford University, a Professor
Emeritus at California University, Fresno, A Funny Sort of Empire: Are Americans really so imperial? National Review
Online, November 27, 2002, http://www.victorhanson.com/articles/hanson112702.html, UK:Fisher

In that regard, America is also a revolutionary, rather than a stuffy imperial society. Its crass culture
abroad rap music, Big Macs, Star Wars, Pepsi, and Beverly Hillbillies reruns does not reflect the
tastes and values of either an Oxbridge elite or a landed Roman aristocracy. That explains why Le Monde
or a Spanish deputy minister may libel us, even as millions of semi-literate Mexicans, unfree Arabs, and
oppressed southeast Asians are dying to get here. It is one thing to mobilize against grasping, wealthy
white people who want your copper, bananas, or rubber quite another when your own youth want
what black, brown, yellow, and white middle-class Americans alike have to offer. We so-called imperialists
don't wear pith helmets, but rather baggy jeans and backwards baseball caps. Thus far the rest of the
globe whether Islamic fundamentalists, European socialists, or Chinese Communists has not yet
formulated an ideology antithetical to the kinetic American strain of Western culture.

Page 1028 of 1481


Relativism Is Self-Refuting

RELATIVISM REFUTES ITSELF

Schick and Vaughn 2002


[Theodore, Jr., Muhlenberg College & Lewis, How to Think about Weird Things:
Critical Thinking for a New Age, Third ed., Boston: McGraw Hill, 87//uwyo-ajl]

According to the relativist whether a subjectivist, a social constructivist, or a conceptual


relativist everything is relative. To say that everything is relative is to say that no
unrestricted universal generalizations are true (an unrestricted generalization is a statement
to the effect that something holds for all individuals, societies, or conceptual schems). But the
statement No unrestricted universal generalizations are true is itself an unrestricted
universal generalization. So if relativism in any of its forms is true, its false. As a result, it
cannot possibly be true.

To avoid such self-contradiction, the relativist may try to claim that the statement
Everything is relative is only relatively true. But this claim wont help, because it just
says that relatavists (or their society or their conceptual scheme) take relativism to be true.
Such a claim should not give the nonrelativist pause, for the fact that relativists take relativism
to be true is not in question. The question is whether a non-relativist should take relativism to
be true. Only if relativists can provide objective evidence that relatvisim is true should a
nonrelativist believe that its true. But this evidence is precisely the kind that relatvists
cant provide, for, in their view, there is no objective evidence.

Relativists, then, face a dilemma: If they interpret their theory objectively, they defeat
themselves by providing evidence against it. If they interpret their theory relativistically,
they defeat themselves by failing to provide any evidence for it. Either way, relativists
defeat themselves.

RELATIVISM PRESUPPOSES THAT CULTURAL HEGEMONY IS UNIVERSALLY


NEGATIVE, DISPROVING ITSELF

Tilley 2000
[John J., Assoc. Prof of Philosophy at Indiana University-Purdue University
Indianapolis, Cultural Relativism, Human Rights Quarterly, 22.2, The John Hopkins
University Press, 528-9//uwyo-ajl]

Perhaps relativists will complain that the effectiveness of the examples stems from act-
descriptions that refer to motives. This calls for two replies. First, there is nothing underhanded
about such descriptions. They are a common way of producing highly definite moral judgments.

Page 1029 of 1481


Second, relativists should be wary about granting "effectiveness" to the examples. If [End Page
529]

they mean that the examples are indeed universally valid, they have abandoned their thesis, for
they have admitted that some moral judgments are valid for everyone. This admission
contradicts relativism no matter what act-descriptions appear in the judgments. Also, it implies
that there is nothing about moral predicates that prevents the judgments in which they occur
from being valid for all cultures. So it's likely that many such judgments are universally valid,
including many that say nothing about motives.

Some relativists (though not the diehard ones) are likely to make a second complaint. They will exclaim:
"But we don't deny that such judgments are universally valid! The whole point of our thesis is that cruelty
and oppression are universally wrong, that respect and tolerance are universally right!" But if this is
indeed their "whole point," they have nothing to contribute to moral theory.

If relativism is not an alternative to universalism, if it is merely a set of commonplace remarks that most
any brand of universalism can accommodate, it lacks the philosophical importance its defenders claim for
it. 52 To the extent that it has that importance, it conflicts with universalism, which means that it does
deny, implicitly at least, that the example judgments are universally valid.

Page 1030 of 1481


Defense: Non-Westerners Want Dignity

EVEN IF HUMAN RIGHTS ARE WESTERN IN ORIGIN, NON-WESTERNERS STILL


DESIRE THEM BECAUSE THEY PROVIDE BASIC DIGNITY

Mered 96
[Sohail, JD Candidate @ Western Reserve University School of Law, Its Not a
Cultural Thing: Disparate Domestic Enforcement of Internaitonal Criminal Procedure
Standards A Comparison of the United States and Egypt, Case Western Reserve
Journal of International Law, Winter, LN//uwyo-ajl]

Relativists, notably non-Western scholars, argue that some violations of human rights are due
to the fact that non-Western cultures value collective societal rights over individual rights. The
concept of individual rights originated in Europe, and thus cannot be applied to the non-
Western world. Conceding the point that the concept of individual rights as referred to in
international instruments has Western origins, relativists have nevertheless failed to prove that
non-Western cultures do not value individual rights as well. This argument assumes that the
natural law ideals of Locke can only be referred to in those Western terms used in international
conventions. The fact that some cultures value collective rights more than the West does not
preclude the same cultures valuing the concepts embodied in the human rights which protect
basic human dignity. When non-Westerners allude to the greater emphasis on group rights,
they are referring to a greater consideration for units such as the family or the community. n70
They have failed to prove, however, that the weaker emphasis on individualism in their society
would permit a state to strip an individual of his or her civil rights. Relativists have yet to prove
that the concepts are mutually exclusive.

EVEN IF RIGHTS ARE CULTURAL, THEYRE PRAGMATICALLY DESIRABLE BECAUSE


THEY ENSURE FAIRNESS

Binder 99
[Guyora, Prof. of Law @ SUNY Buffalo, Cultural Relativism and Cultural Imperialism
in Human Rights Law, Buffalo Human Rights Law Review, 1999, LN//uwyo-ajl]

At the same time, and for the same reasons, the admission that support for international
protection of civil and political rights rests on culturally specific value judgments does not refute
those value judgments. Advocates sought a foundation for international human rights law in the
natural liberty of individuals only in order to overcome the foundationalist arguments of
defenders of the absolute autonomy of sovereign states. But arguments for and against
international human rights law or state autonomy need no foundations. We can always assess
international legal institutions and doctrines in pragmatic terms, as contributing to human
betterment, or as embodying broadly participatory decisions emerging from acceptably fair

Page 1031 of 1481


processes, or as tolerably useful and superior to available alternatives, or to the costs of
pursuing change. n11 This is how we commonly assess domestic political institutions. Why
should we treat international legal institutions any differently?

Page 1032 of 1481


A2 Foundationalism Bad

EVEN IF WE HAVE NO CERTAIN FOUNDATIONS, WE CAN USE CHAINS OF


INFERENCE TO CREATE PRAGMATIC ETHICAL CODES THE ONLY ALTERNATIVE IS
THE REJECTION OF ALL KNOWLEDGE INCLUDING THE K

Tilley 2000
[John J., Assoc. Prof of Philosophy at Indiana University-Purdue University
Indianapolis, Cultural Relativism, Human Rights Quarterly, 22.2, The John Hopkins
University Press, 537//uwyo-ajl]

Second (and at the price of some repetition), fallibilism, as it pertains to moral beliefs, implies
merely that such beliefs are "tentative" or "provisional" in the special sense fallibilists give those
terms. It implies that moral beliefs are corrigible, or in principle revisable, and as such are in the
same boat with the following beliefs (all of which, according to fallibilism, are in principle
revisable): "1=1;" "I exist;" "others besides myself exist;" "my birth preceded my reading of
Folkways;" "there is more than one culture in the world;" "relativists and universalists use
language when defending their views."

Does anyone, including any relativist, lack confidence in these beliefs? Of course not. Nor is
there any need to, even if we reject foundationalism. Foundationalism is neither the only
plausible account of justification, nor the only one at home with the commonsense view that
some beliefs warrant considerable confidence. 64 So the rejection of foundationalism does not
put "substantial limits" on the confidence we can place in our beliefs. If it be said that special
difficulties attend confidence in moral beliefs, my reply is that this needs to be shown; it does
not follow from fallibilism. If it is shown, it will apply to all moral beliefs, including the ones
relativists are eager to vindicate--namely, those that aspire to merely "local," or culturally
specific, validity. Hence it will advance the relativist's cause not a whit.

Page 1033 of 1481


A2 Morality Is Culturally Created

NOT TRUE [TWO REASONS]


A) NO EVIDENCE OF CULTURAL DETERMINISM
B) IT UNDERMINES RELATIVISM

Tilley 2000
[John J., Assoc. Prof of Philosophy at Indiana University-Purdue University
Indianapolis, Cultural Relativism, Human Rights Quarterly, 22.2, The John Hopkins
University Press, 539-40//uwyo-ajl]

The weakness of this argument resides in the word "biased." The fact that a thesis is culturally
biased discredits the thesis only if "biased" means roughly the same as "distorted" or
"mistaken." 70 But if it has that meaning, two problems arise. First, cultural determinism is not
confirmed by any evidence marshaled for it, because according to cultural determinism, that
evidence is not evidence at all, but a batch of mistakes or distortions. Second, the relativist's
new argument fails to make relativism more plausible than universalism. Its main premise,
cultural determinism, implies that every [End Page 540] product of the human mind is culturally
biased. So every such product is discredited, including cultural relativism and cultural
determinism.

In short, the relativist has shot himself in the foot. His argument rests on a premise which, if
interpreted so that it can do the work assigned to it, discredits both itself and relativism. (Of
course, if it discredits itself we can dismiss it as false, in which case it discredits nothing. Such
are the puzzles spawned by self-discrediting premises.) His problem is similar to one he faced
earlier, when he claimed that every truth is merely a local truth. His present argument rests on a
similar claim, one that thwarts his aims just as surely as the earlier one did.

Perhaps the relativist will respond by revising cultural determinism so that it concerns only
normative moral theories. He then can use it against such theories without threatening either
relativism or cultural determinism. 71 This tactic fails. For one thing, metaethical theories are no
less biased than normative ones, in any sense of "biased" that supports the view that normative
theories are inescapably biased. Ironically, this is especially true of the metaethical thesis of
relativism, which owes much of its popularity to historically specific "biases," among them the
anti-Victorian attitude of early twentieth century intellectuals. 72

Page 1034 of 1481


K = Imperialist

THE CRITICISM IS ITSELF THE RESULT OF WESTERN CULTURAL NORMS, IMPOSING


THEM UPON THE WORLD

Morgan-Foster 2003
[Jason, JD Cand at U. of Michigan School of Law, A New Perspective on the
Universality DebA2 Reverse Moderate Relativism in the Islamic Context, ILSA
Journal of Intl and Comparative Law, Fall, LN//uwyo-ajl]

Strict cultural relativism has been criticized as self-contradictory: the notion that all values are
culturally relative, the belief in "the equal dignity and worth of all cultures," or "the equal right
of all peoples to participate in the formation of international law" are themselves culturally
shaped value judgments, which would be void under the cultural relativist's own theory. There
is no reason for cultural relativists to accept these starting points as universal in order to
support a doctrine which denies the legitimacy of [*43] universals. n37 From a normative
human rights perspective, strict cultural relativism is also questionable because it has little to no
support in human rights conventions. The only treatment of strict cultural relativism in a human
rights convention is article 63(3) of the European Convention on Human Rights, which says that
"[t]he provisions of this Convention shall be applied in [colonial territories] with due regard,
however, to local requirements." n38 A strict cultural relativist reading of this provision has
been rejected by the European Court of Human Rights in Tyrer v. United Kingdom, where the
local custom of corporal punishment was at issue. n39
Thus, because of the logical self-contradiction inherent in strict cultural relativism, and because
of the virtual complete lack of support for strict cultural relativism in the human rights
discourse, strict cultural relativism fails as a paradigm to conceptualize the universality
discourse.

Page 1035 of 1481


Page 1036 of 1481
**Kappeler**

Page 1037 of 1481


Kappeler Answers: 2AC (1/5)
FIRST, NO LINK WE DONT SAY THAT VIOLENCE IS AN ABERRATION. WE
ACKNOWLEDGE THAT ITS INEVITABLE AND THAT REALISM IS THE LEAST BAD
APPROACH

SECOND, WE OUTWEIGH THE VIOLENCE THEY DESCRIBE IS LOWSCALE. MINIMAL


COERCION IS NECESSARY IN THE FACE OF MUCH LARGER SCALE THREATS OF
ANNIHILATION

THIRD, PERM DO BOTH


YOU SHOULD ACKNOWLEDE THAT VIOLENCE IS AN AGENCY ISSUE AND PASS PLAN

Kappeler, Assoc Prof at Al-Akhawayn U, 95 (Susanne, The Will to Violence: The politics of personal behavior, P.8)

Moreover, personal behavior is no alternative to political action; there is no question of either/or. My


concern, on the contrary, is the connection between these recognized forms of violence and the forms of
everyday behavior which we consider normal but which betray our own will to violence- the connection,
in other words, between our own actions and those acts of violence which are normally the focus of our
political critiques. Precisely because there is no choice between dedicating oneself either to political
issues or to personal behavior, the question of the politics of personal behavior has (also) to be moved
into the centre of our politics and our critique.

FOURTH, WE SOLVE PLAN TAKES RESPONSIBILITY FOR VIOLENCE DONE IN OUR


NAME BY CHALLENGING UNILATERAL DETAINMENT WITHOUT DUE PROCESS.
CROSS-APPLY TRIBE AND SANYAL

FIFTH, KAPPELERS CRITICISM COLLAPSES REAL AND VIRTUAL VIOLENCE,


PREVENTING MOBILIZATION AGAINST ATROCITY

Bronfen 86
[Elisabeth, U. of Munich, Disavowal and Insight, Art History 11:1, March, ASP//uwyo-ajl]

Page 1038 of 1481


There is undoubtedly a heuristic value in focusing on structural similarities and in denying that a fictional
representation is fundamentally different from a documentary one when seen from the point of view of
the function of this image. This allows Kappeler to reveal how violation can take place on more than just
the literal level. Yet it seems necessary to me to see that there is also a fundamental difference between a
depiction based on or involving the real violence done to a physical body (Thomas Kasire, snuff movies)
and the imagined one, representing this violence on paper, canvas or celluloid, without any concretely
violated body as its ultimate signified. Not because the latter can then be absolved from any responsibility
toward the material of its depiction, but because to collapse the two levels on which signification works
might also mean not doing justice to the uniquely horrible violence that occurs when a body is used quite
literally as the site for an inscription by the other.

Page 1039 of 1481


Page 1040 of 1481
Kappeler Answers: 2AC (2/5)
SIXTH, MULTILATERALISM SOLVES WE REPLACE THE VIOLENCE OF HARD POWER
WITH INTERNATIONAL COOPERATION, SHORT-CIRCUITING THEIR IMPACT. CROSS-
APPLY NYE

SEVENTH, EVERY AFFIRMATIVE ETHICAL STANCE REQUIRES A REPRESSED ELEMENT


OF NEGATION, MEANING THAT THE ALTERNATIVE OCCURS AGAINS THE
BACKGROUND OF COVERT VIOLENCE

Zizek '99
[Slavoj, Senior Researcher at Institute for Social Studies, Ljubliana and Badass, The Ticklish Subject: the
absent centre of political ontology, New York: Verso, 1999, 153-4//uwyo-ajl]

It would therefore be tempting to risk a Badiouian-Pauline reading of the end of psychoanalysis, determining it as a
New Beginning, a symbolic 'rebirth' - the radical restructuring of the analysand's subjectivity in such a way that the
vicious cycle of the superego is suspended, left behind. Does not Lacan himself provide a number of hints that the end
of analysis opens up the domain of Love beyond Law, using the very Pauline terms to which Badiou refers?
Nevertheless, Lacan's way is not that of St Paul or Badiou: psychoanalysis is not 'psychosynthesis'; it does not
already posit a 'new harmony', a new Truth-Event; it - as it were - merely wipes the slate clean for one.
However, this 'merely' should be put in quotation marks, because it is Lacan's contention that, in this
negative gesture of 'wiping the slate clean', something (a void) is confronted which is already 'sutured'
with the arrival of a new Truth-Event. For Lacan, negativity, a negative gesture of withdrawal, precedes
any positive gesture of enthusiastic identifiction with a Cause: negativity functions as the condition of
(im)possibility of the enthusiastic identification - that is to say, it lays the ground, opens up space for it,
but is simultaneously obfuscated by it and undermines it. For this reason, Lacan implicitly changes the balance between Death and
Resurrection in favour of Death: what 'Death' stands for at its most radical is not merely the passing of earthly life, but
the 'night of the world', the self-withdrawal, the absolute contraction of subjectivity, the severing of its
links with 'reality' - this is the 'wiping the slate clean' that opens up the domain of the symbolic New
Beginning, of the emergence of the 'New Harmony' sustained by a newly emerged Master-Signifier. Here, Lacan parts company with St Paul and Badiou:
God not only is but always-already was dead - that is to say, after Freud, one cannot directly have faith in a Truth-Event; every such Event
ultimately remains a semblance obfuscating a preceding Void whose Freudian name is death drive. So Lacan
differs from Badiou in the determination of the exact status of this domain beyond the rule of the Law. That is to say: like Lacan, Badiou
delineates the contours of a domain beyond the Order of Being, beyond the politics of service des biens, beyond the 'morbid' super ego
connection between Law and its transgressive desire. For Lacan, however, the Freudian topic of the death drive cannot be accounted for
in the terms of this connection: the 'death drive' is not the outcome of the morbid confusion of Life and Death
caused by the intervention of the symbolic Law. For Lacan, the uncanny domain beyond the Order of
Being is what he calls the domain 'between the two deaths', the pre-ontologicalf domain of monstrous
spectral apparitions, the domain that is 'immortal', yet not in the Badiouian sense of the immortality of
participating in Truth, but in the sense of what Lacan calls lamella, of the monstrous 'undead' object-
libido.18

Page 1041 of 1481


Page 1042 of 1481
Kappeler Answers: 2AC (3/5)
EIGHTH, KAPPELER DOESNT GET COGNITION THE VIOLENT IMAGE ALLOWS THE
VIEWER TO EMPATHIZE WITH THE VICTIM, ENABLING QUESTIONING OF OUR
REPRESENTATIONS AT THE SAME TIME AS IT DEPICTS THEM

Bronfen 86
[Elisabeth, U. of Munich, Disavowal and Insight, Art History 11:1, March, ASP//uwyo-ajl]

to use another as object for self-expression always involves a shift or non-


What she ignores in her argument, however, is that

identity between self and self-reflecting image. As Lacan points out, to see oneself in an image is recognition as
misrecognition. The interesting thing is that a third term/body is needed for this to occur, even if it involves the reduction of a subject to an object, from the gazing
subjects point of view. That is to say, narcissistic self-recognition occurs only through the introduction of difference, even if an attempt is made to efface this difference again in

, the subjectivity of the objectified other is always latently possible, present, and potentially
the process. Due to this

signifiable, even if not signified. That is to say, the attempt to efface the others voice is a strategy that can
unwittingly turn upon itself and expose its own limitations.
Thus the process by which the gazing man recognizes himself in the look of the gazed-at woman always also implies an element of duplicity. For since she is sexually different
from him he both can and cannot see himself in her. His objectifying gaze depends on the transformation of the otherness of the other into an image of similarity yet it is
precisely this otherness that seems to make the reduction so satisfying. Even if the ultimate goal is homophobic bonding, it occurs over a body which will always give back the
sought-for look of self-recognition only imperfectly. As such, the woman/object is always double, both confirming and not confirming the male gaze, similar but not the same.
Thus I would argue that her text (voice) is always also inscribed in the male text, even if we are asked to be blind to it, even if it is that which marks where the dominant
structure of representation is staged in this scenario falters. The dynamics involved in violating the body of a woman by transforming it into a Woman/victim as figure for
something alterior to herself seems to me to be more complicated. What, for example, remains unexplained by Kappelers formula is why the representation of another is
needed to bring about self-expression, why a straightforward self-portrait will not suffice or, to put it another way, why patriarchy needs to designate certain members of society
as other, in order to stabilize its own power. Clearly what this suggests is that the violent creation of similarity out of difference is more satisfying than a static homogeneous
space. Clearly also, the charm of reducing another to a silent object which will not respond in word or gaze allows an unlimited plethora of inscriptions and semantizations by
the gazing subject that remain unchallenged. But if the object of the representation is always only a silent victim, the question remains, why is it possible that the victim can
mirror the master?

In part as a response to Barthess discussion of de Sades writings, Kappeler distinguishes further between two forms of victimization. The first form is a straightforward act of
objectification, annihilating the womans subjectivity, with the victim objecting to the vexation and crying out in pain. The second form involves a complicit victim: faking
subjectivity, she chooses (in Barthess terminology) to ejaculate or discharge, to transform herself into a libertine, and enjoy herself in her vexation. Yet Kappelers point is that
while the subject of this situation desires the womans complicity and pleasure, wants her to want to be a victim masquerading as subject, it is ultimately the subjects feeling of
pleasure that is at stake. She sees this analogous to Barthess notion of the authors search for his readers pleasure as a way to guarantee his own pleasure as supreme
writing object. The point of her comparative reading is to show that where the question of complicity and collaboration is involved, the object (the willing woman libertine) and

the reader (the willing co-player of the authors game) are in similar positions, serving similar functions, namely to confirm the speaking male subject. What this
collaboration is blind to, is the possibility of identifying with some position other than that of the
speaking subject, for example that of the victim/object. For her this second form of violence, the collaboration with the master-plot, is
doubly perfidious because it not only denies the subjectivity of another but pretends to deny its own elision of the other. Astute as her analysis is, it does raise the

question of what Kappeler is willing to ignore in her will to expose the literal content of figural language.
For one could also say that by pretending to deny the others victimization, by faking an objectified others

subjectivity, a space is opened that ironically (and critically) questions these strategies at exactly the same
moment that it stages them.

NINTH, POWER IS ZERO SUM THE ALTERNATIVE ONLY SHIFTS POWER ELSEWHERE

John Mearsheimer, Professor at University of Chicago, 2001 (The Tragedy of Great Power Politics p. 34)

Consequently, states pay close attention to how power is distributed among them, and they make a special effort to maximize
their share of world power. Specifically, they look for opportunities to alter the balance of power by acquiring
additional increments of power at the expense of potential rivals. States employ a variety of meanseconomic,
diplomatic, and militaryto shift the balance of power in their favor, even if doing so makes other states suspicious or even
hostile. Because one states gain in power is another states loss, great powers tend to have a zero-sum

Page 1043 of 1481


mentality when dealing with each other. The trick, of course, is to be the winner in this competition and to dominate
the other states in the system. Thus, the claim that states maximize relative power is tantamount to arguing that states are
disposed to think offensively toward other states, even though their ultimate motive is simply to survive. In short, great
powers have aggressive intentions.

Page 1044 of 1481


Page 1045 of 1481
Kappeler Answers: 2AC (4/5)
TENTH, KAPPELERS ARGUMENT IS PREMISED ON AN ESSENTIALIST APPLICATION
OF SEXUALITY TO VIOLENCE

Pringle 96
[Rosemary, Book Reviews, Gender, Place & Culture: A Journal of Feminist Geography 3:2, July,
ASP//uwyo-ajl]

Kappeler insists that we have 'ample opportunity in situations of no such threat to challenge the legitimacy of violence and to practise alternatives' (p. 258). We must scrutinise
our own will to power and find alternative political ways of resisting oppression and domination. 'We' is used deliberately as a shifting signifier in relation to which the reader is
free to recognise herself, to identify as included or excluded, perhaps either at different times. Most of the time I felt excluded. I admired the honesty of her account of white
women's racism, and I laughed wryly at her attack on the more self-indulgent moments of therapy. Beyond that, I was unable to connect at all with her critique of sexuality and
. Kappeler goes over much old ground in insisting that sexuality
desire, with a politics that seems to take so little account of subjectivity

is fundamentally gender-specific and cannot be democratised. If she believes this, it is hard to see why
she would bother about politics in the first place. Any attempt to claim sexual subjectivity or an active
desire is interpreted as part of a will to power and hence an act of violence. Dworkin, Mackinnon, Pateman and Jeffries are
quoted approvingly. If anything, Kappeler outdoes them in her insistence that sexuality may have to be given up in

order to eradicate violence:

if experience shows that sex indeed means violence and sexual excitement the pleasure of power--that sex minus the violence does not leave
us with non-violent sex but simply 'no sex' at all--it does not follow that we therefore must accept violence; it follows that 'sex' as such is

unacceptable. (p. 181)

ELEVENTH, THIS MIMICS COLONIZATION, ENTRENCHING OPPRESSION

Butler 99
[Judith, prof. of rhetoric at UC Berkeley, Gender Trouble: Feminism and the
Subversion of Identity, New York: Routledge, 1999, 18-19//uwyo-ajl]

Feminist critique ought to explore the totalizing claims of a masculinist signifying economy, but also remain self-critical with respect to the totalizing gestures of feminism. The effort
to identify the enemy as singular in form is a reverse-discourse that uncritically mimics the strategy of the
oppressor instead of offering a different set of terms. That the tactic can operate in feminist and antifeminist contexts alike suggests that the colonizing gesture is not primarily or
irreducibly masculinist. It can operate to effect other relations of racial, class, and heterosexist subordination , to name but a few. And clearly, listing the varieties of oppression, as I began to
do, assumes their discrete, sequential coexistence along a horizontal axis that does not describe their convergences within the social field. A vertical model is similarly insufficient; oppressions
cannot be summarily ranked, causally related, distributed among planes of originality and derivativeness. Indeed, the field of power structured in part by the imperializing gesture of
dialectical appropriation exceeds and encompasses the axis of sexual difference, offering a mapping of intersecting differentials which cannot be summarily hierarchized either within the

Rather than an exclusive tactic of masculinist


terms of phallogocentrism or any other canddidate for the position of primary condition of oppression.

signifying economies, dialectical appropriation and suppression of the Other is one tactic among many
deployed centrally but not exclusively in the service of expanding and rationalizing the masculinist domain.

Page 1046 of 1481


Page 1047 of 1481
Kappeler Answers: 2AC (5/5)
TWELFTH, TURN - ATTEMPTING TO CLEANSE LANGUAGE OF VIOLENCE FETISHIZES
AUTHENTICITY, RESULTING IN POLITICAL DISENGAGEMENT BECAUSE OF THE
VIOLENCE AT THE HEART OF ALL LANGUAGE AND INTERACTION

Bewes 97
[Timothy, doctorate in English Literature at the University of Sussex, Cynicism and Postmodernity, New
York City: Verso, 1997, 137-8//uwyo-ajl]

Thus, what secondly distinguishes the 'metaphysical innocence' of Rameau is his pursuit of violence - not only the violence of determinate negation, of alienation from culture
and the serial progression of knowledge, but the violence of imperfection, of disrupted subjectivity, of unforeseen catastrophes and superfluous resources, of human
inconsistency and what Gillian Rose calls the 'agon' of existence. Violence, like suffering and fickleness for Dostoevsky, represents subjective (as against objective) culture, a

. Violence increases as the result not


last manifestation of individual volition, and a point of resistance to what BaudriUard calls the 'triumph' of simulation

of a deterioration in social behaviour but of a lowering in the cultural threshold beyond which action
appears as violence. In such a context Rameau's disintegration, his 'epigrammatic' existence and his cultivation of violence represent the final recourse of a
disfranchised and alienated subjectivity faced with an apparently sewn up, indifferent world.

In postmodernity this threshold between action and violence is lower, perhaps, than ever before. Political correctism, 'Queer' theory, Communitarianism, the liberation discourse
of the Internet, calls for homogenization of the private and public lives of politicians, the new discipline of 'postmodern ethics', all are varying instances of a collective endeavour
to put a freeze on reason as risk, the consequence of a fetishization of objective culture. To find intolerable the violence of
linguistic oppression, of 'inauthentic' sexual identity (the product of Freud's 'family romance', etc.), of political antagonism, of the formalization of truth in its
dissemination, of the compart mentalization of public and private life, of the indeterminacy of moral options, is in every case to subscribe to a peculiar

literalism, to evince a profound discomfort with the signifying relation, to take the signifier persistently for
the thing itself, in such a way that political activity is replaced with a series of cosmetic adjustments to
objective culture.
Rameau's cynicism therefore represents a commitment to subjective culture, to reality, to the referent and to the signified, to the truth of the world and of the individual.
Cynicism constitutes a certain necessary indifference to objective culture, a certain subjective wager, a projection of the self beyond objective culture and beyond its own limits.

In a climate in which 'authenticity' is at a premium, where all action has been proscribed as intolerably
violent, and where self consciousness is therefore only a disabling mechanism to be discad, cynicism appears as a spirit in disintegration,
the monopoly broker of disinvestment in the present, the sole locus of reason and of faith in anything other than the phenomenal here and
now, the disposition which alone embodies both energy and depth.

Page 1048 of 1481


Page 1049 of 1481
#5 Alternative Causes Violence:
1AR (1/2)
THE ALTERNATIVE DISEMPOWERS RESISTANCE KAPPELERS FOCUS ON THE
INDIVIDUAL RATHER THAN PROVIDING A MEANS TO COMBAT VIOLENCE
PREVENTS EFFECTIVE RESISTANCE TO VIOLENCE

Gelber 95 (Kath, Review of The Will to Violence: The Politics of Personal Behaviour, Green Left Weekly,
http://www.greenleft.org.au/back/1995/198/198p26b.htm)

The Will to Violence presents a powerful and one-sided critique of the forces which enable violence between individuals to occur. Violence between individuals is taken in this

Kappeler's thesis is that violence in all these cases is


context to mean all forms of violence, from personal experiences of assault to war.

caused in the final instance by one overriding factor -- the individual choice to commit a violent act. Of
course, in one sense that is true. Acknowledging alternative models of human behaviour and analyses of the social causes of violence, Kappeler dismisses
these as outside her subject matter and exhorts her readers not to ignore the agent's decision to act as he [sic] did, but to explore the personal decision in favour of violence.
Having established this framework, she goes on to explore various aspects of personal decisions to commit violence. Ensuing chapters cover topics such as love of the other,

Kappeler is
psychotherapy, ego-philosophy and the legitimation of dominance. However, it is the introduction which is most interesting. Already on the third page,

dismissive of social or structural analyses of the multiple causes of alienation, violence and war. She
dismisses such analyses for their inability to deal with the personal decision to commit violence. For example,
some left groups have tried to explain men's sexual violence as the result of class oppression, while some

Black theoreticians have explained the violence of Black men as a result of racist oppression. She
continues, The ostensible aim of these arguments may be to draw attention to the pervasive and
structural violence of classism and racism, yet they not only fail to combat such inequality, they actively
contribute to it. Kappeler goes on to argue that, although such oppression is a very real part of an agent's life context, these `explanations' ignore the fact that not
everyone experiencing the same oppression uses violence, i.e. the perpetrator has decided to violate. Kappeler's aim of course was to establish a framework for her particular

project: a focus on the individual and the psychological to find a cause for violence. However, her rejection of alternative analyses not only as
of little use, but as actively contributing to the problem, frames her own thesis extremely narrowly . Her
argument suffers from both her inability, or unwillingness, to discuss the bigger picture and a wilful distortion of what she sees as her opponents' views. The result is

less than satisfactory. Kappeler's book reads more as a passionate plea than a coherent argument. Her overwhelming focus on the
individual, rather than providing a means with which to combat violence, in the end leaves the reader
feeling disempowered. After all, there must be huge numbers of screwed up and vengeful people in the
world to have chosen to litter history with war, environmental destruction and rape. Where do we go from here?
Those lucky enough to have read Kappeler's book are supposed to decide not to use violence ourselves.
A worthy endeavour, but hardly sufficient to change the world.

Page 1050 of 1481


Page 1051 of 1481
#5 Alternative Causes Violence:
1AR (2/2)
FOCUS ON REPRESENTATIONAL VIOLENCE WORSENS REAL VIOLENCE

Elana Gomel, Tel-Aviv University, Written in Blood: Serial Killing and Narratives of Identity, Post Identity, Volume
2, Number 1, Winter 1999, p. 24-25, http://ids.udmercy.edu/pi/2.1/PI21_24-70.pdf, accessed 1/28/02

ONE CAN START WITH FOUCAULTS famous and endlessly circulated statement in The Order of Things: It
is comforting, however, and a source of profound relief to think that man is only a recent invention, a
figure not yet two centuries old, a new wrinkle in our knowledge, and that he will disappear as soon as
this knowledge has discovered a new form. (xxiii) Man the Universal Subject, a cookie-cutter mold of
(post)technological identity, stamping out simulacra of individuality. But why should we be comforted
and experience relief at the thought of his imminent dissolution? Perhaps because, at least from Adorno
on, the subject of reason has also been identified as the subject of violence. The universal Man of the
Enlightenment has been reconceptualized as the universal killer, armed with the most potent of
weaponsrepresentation. In their Introduction to the collection typically entitled Violence of
Representation Armstrong and Tennenhouse offer the basic formula of this approach: The violence of
representation is the suppression of difference (8). In this particular reading of Foucault the discursive
constructedness of identity is directly responsible for corporeal violence inflicted by some (post)modern
subjects upon others. In his recent book Serial Killerr and in the series of articles that preceded it Mark
Seltzer applies this insight to the fascinating and grisly phenomenon of serial killing, variously identified
also as stranger killing and sometimes lust murder. For Seltzer the enigma of the serial killers
personality consists in an experience of typicality at the level of the subject The serial killer, I will be
arguing, is in part defined by such a radicalized experience of typicality within. Simply put, murder by
numbers (as serial murder has been called) is the form of violence proper to statistical persons. (30-1)
Violence of representation, representation of violence and violence per se smoothly link into an unbroken
chain, leading from statistics to mayhem and from typology of subjects to fingertyping of putrefying
bodies. My goal in this essay is to put a hitch into this chain, to question the easy fit between discursive
moulds of identity and the individual self-experience of serial killers, and to suggest that represenration
may be not so much the cause of violence as a post factum defence against it. I do not imply, however,
that violence in general or serial murder in particular are totally free from the constraints of discourse or
that the identity of the serial killer is not constructed using the building blocks of cultural narratives
(though the narratives in question are more variegated than Seltzer suggests). Rather, I would claim that
the serial form of violence is conditioned not so much by the monolithic coherence of representation as
by its breakdown. The violent behavior of a serial killer is not a direct outcome of any social construction
but a random, causeless choice which is retrospectively incorporated into a generic narrative of identity.
The repeated ritualistic violence, then, becomes a means of reinforcing this identity but achieves precisely
the opposite, its complete disintegration. Rather than being generated by representation, corporeal
violence offers a resistance to it.

Page 1052 of 1481


Page 1053 of 1481
#7 Negation: 1AR
EXTEND THE 2AC#1 ZIZEK 99 CARD. THERES NO SUCH THING AS A PURE
AFFIRMATION OF LIFE. EVERY TIME YOU SAY THAT SOMETHINGS GOOD,
BETWEEN THE LINES YOURE SAYING THAT SOMETHING ELSE, LIKE DEATH AND
VIOLENCE, ARE BAD. THEIR YES TO LIFE IS AN IMPLICIT NO TO THE SAME DEATH
AND VIOLENCE THAT WERE SAYING IS BAD. FEAR OF APOCALYPTIC VIOLENCE IS
STILL CONTAINED IN ALL OF THEIR ARGUMENTS, REPRESSED BENEATH THE
SURFACE OF THEIR WORDS.

THIS MEANS WELL WIN THE UNIQUENESS FOR OUR TURNS BECAUSE SOME FORM
OF VIOLENT REPRESENTATION IS INEVITABLE IN ALL POLITICAL DISCOURSE, THE
ONLY QUESTION IS OF WHETHER THOSE REPRESENTATIONS INTERROGATE THE
FUNDAMENTAL FANTASY OF POLITICAL REALITY BY ACKNOWLEDING OUR
INEVITABLE RELATIONSHIP TO THE TRAUMA OF DEATH AND VIOLENCE THATS
INHERENTLY REPRESSED BY THE SYMOBLIC

CONCEIVING OF VIOLENCE AS AN UNDESCRIBABLE HORROR IS A FANTASY THAT


ALLOWS US TO AVOID THE TRAUMATIC ANTAGONISM THAT CONSTITUTES
REALITY ONLY IDENTIFICATION OF ITS OBSCENE UNDERSIDE ALLOWS US TO
INTERROGATE ITS IDEOLOGICAL GROUNDING

Zizek 2001
[Slavoj, Megalomaniacal mercy killer, Welcome to the Desert of the Real: Five Essays on September 11 and
Related Dates, New York: Verso, 30-2//uwyo]

the passion for the Real is this identification with this heroic gesture of fully assuming the
The very core of

dirty obscene underside of Power: the heroic attitude of Somebody has to do the dirty work, so lets do it!, a kind of mirror-reversal of the Beautiful Soul
which refuses to recognize itself in its result. We find this stance also in the properly Rightist admiration for the celebration of heroes who are ready to do the necessary dirty
work: it is easy to do a noble thing for ones country, up to sacrificing ones life for it it is much more difficult to commit a crime for ones countryHitler knew very well how to
play this double game apropos of the Holocaust, using Himmler ot spell out the dirty secret. In his speech to the SS leaders in Posenon October 4 1943, Himmler spoke quite
openly about the mass killing of the Jews as a glorious page in our history, and one that has never been written and never can be written; he explicitly included the killing of
women and chilrden:

We faced the question: what should we do with the women and children? I decided here too to find a completely clear solution. I did not regard myself as justified in
exterminating the men that is ot say, to kill them or have them killed and to allow the avengers in the shape of chilrden to grow up for our sons and grandchildren. The
difficult decision had to be taken to have this people disappear from the earth.

The very next day, the SS leaders were ordered to attend a meeting where Hitler himself gave an account of the state of the war; here, Hitler did not have to mention the Final
Solution directly oblique references to the SS leaders knowledge and to their shared complicity, were enough: The entire German people know that it is a matter of whether

it is along these lines that we can


they exist or do not exist. The bridges have been destroyed behind them. Only the way forward remains. And, ideally,

oppose the reactionary and the progressive passion for the Real: while the reactionary one is the
endorsement of the obscene underside of the Law, the progressive one is confrontation with the Real of the

Page 1054 of 1481


antagonism denied by the passion for purification, which in both its versions, the Rightist and the Leftist assumes that the Real is touched in and through the
destruction of the excessive elemtn which introduces antagonism. Here, we should abandon the standard metaphorics of the Real as the terrifying Thing that is impossible to
confront face to face, as the ultimate Real concealed beteath the layers of imaginary and/or symbolic Veils: the very idea that, beneath the deceptive appearances, ther elies
hidden some ultimate Real Thing too horrible for us to look at directly is the ultimate appearance this Real Thing is a fantasmic spectre whose presence guarantees the
consistency of our symbolic edifice, thus enabling us to avoid confronting its constitutive inconsistency (antagonism). Take Nazi ideology: the Jew as its Real is a spectre
evoked in order to conceal social antagonism that is, the figure of the Jew enables us to perceive social totality as an organic Whole. And does not the same go for the figure
of Woman Thing inaccessible to the male grasp? Is she also not the ultimate Spectre enabling men to avoid the constitutive deadlock of the sexual relationship?

Page 1055 of 1481


Page 1056 of 1481
#8 Subversion: 1AR
THEY MISUNDERSTAND COGNITION - IDENTIFICATION WITH IMAGES OF
DOMINATION UNDERMINES RELATIONSHIPS OF SUBORDINATION

Krips '99
[Henry, Professor of Communication at the Pitt, Fetish: an erotics of culture, Ithaca, Cornell University
Press, 1999, 5-6//uwyo-ajl]

Arguments against linking the cultural and psychic realms also seem apposite in criticizing MacKinnon's claim
that there exists a direct causal connection between pornography and a psychic characteristic of its male
consumers, namely sexual aggression. At a theoretical level, her argument fails to take into account
Freud's point that identification with a phantasy figure flows readily across gender lines. For example, in the
Dora case, Freud argues that Dora's behavior manifests an unconscious desire for Frau K., her father's lover and suitor's
wife. For Freud her desire does not indicate any sexual instability. Instead, through an identification with her father's
desire, it signals an unconscious paternal identification. In other words, for Freud the significant aspect of Dora's
phantasy is not the sexual content of the desire but rather the paternal position from which she engages
with it. By parity of reasoning, it follows that quite "normal" male readers of porn may identify with the
position of woman victim rather than male aggressor, in which case their aggressive tendencies cannot be
reinforced in the simplistic way that MacKinnon suggests.3 In short, as Laura Kipnis points out, neither
the biology nor gender of readers of Hustler magazine determines the form of their identification with its
pornographic materials, let alone forces them into a common psychic response (Kipnis 1996, 196). In the
same way, one may argue, gender-swapping phantasy games played by Net users do not indicate their gender
instability. On the contrary. one might turn the argument around and conclude that the preponderance of biological
males among Net users suggests that even when playing at being a woman, they are engaging in a "boys' game."

Page 1057 of 1481


#12 Authenticity: 1AR

EXTEND THE 2AC #4 BEWES 97 EVIDENCE. THE CALL TO ERASE VIOLENCE FROM
LANGUAGE IS A FANTASY THAT IGNORES THE WAY THAT VIOLENCE IS ENDEMIC TO
ALL POLITICS. ATTEMPTS TO COVER IT UP AND PRETEND THAT WE CAN HAVE A
NON-VIOLENT COMMUNICATION PROVIDE FALSE DISTANCE BETWEEN US AND
THAT VIOLENCE, ALLOWING YOU TO ACT IN EVEN MORE DESTRUCTIVE WAYS
WHEN YOU HYSTERICALLY LASH OUT AT PERCEIVED THREATS TO YOUR LIFE WHILE
BELIEVING YOUVE ESCAPED VIOLENCE IN THE GUISE OF A SUPERFICIAL LINGUISTIC
ALTERATION.

THE EXTERNAL IMPACT IS DEPOLITICIZATION THE ONLY WAY TO ERASE


VIOLENCE IS TO CEASE ALL POLITICAL ENGAGEMENT BECAUSE EVERY ACTION
CONTAINS THE STAIN OF VIOLENCE. THAT POSITION MAKES US COMPLICIT IN
OPPRESSION AND EVEN MORE HORRIFIC FORMS OF PHYSICAL ANNIHILATION,
WHICH OUTWEIGHS ANY NEGLIBILE IMPACT OF IN ROUND DISCURSIVE VIOLENCE
BECAUSE IT IMPLICATES YOUR VERY ABILITY TO CONDUCT POLITICS, SUCH AS THE
1AC AND THE CRITICISM, IN THE FIRST PLACE

AND, RETREAT FROM VIOLENT RHETORIC INTO FANTASIES OF METAPHYSICAL


INNOCENCE ALLOWS US TO SPEND HOURS DEBATING THE FINER POINTS OF THE
AUTHENTICITY OF OUR WORDS WHILE GAS CHAMBERS ARE BUILT

Bewes 97
[Timothy, doctorate in English Literature at the University of Sussex, Cynicism and Postmodernity, New
York City: Verso, 1997,146-7//uwyo-ajl]

If it is unreasonable to suppose that the Final Solution was potentiated or even necessarily facilitated by
Schmitt's theories, it is certainly the case that this metaphysical structure of domination in the Third
Reich, whereby the status of public citizens is reduced to a level determined entirely in the 'natural' or
biological realm of necessity, is foreshadowed in his 1927 essay. In an abstract and insidious way Schmitt
introduces the idea that the 'transcendent' realm of the political, as a matter of course, will not
accommodate a people with insufficient strength to ensure its own participation, and that such a fact is
ipso facto justification for its exclusion. 'If a people no longer possesses the energy or the will to maintain
itself in the sphere of politics, the latter will not thereby vanish from the world. Only a weak people will
disappear.'130 Schmitt's concept of the 'political', quite simply, is nothing of the sort - is instead weighed
down by necessity, in the form of what Marshall Berman calls German-Christian interiority - by its
preoccupation with

Page 1058 of 1481


authenticity, that is to say, and true political 'identity'. Auschwitz is a corollary not of reason, understood
as risk, but of the fear of reason, which paradoxically is a fear of violence. The stench of burning bodies is
haunted always by the sickly aroma of cheap metaphysics.

Page 1059 of 1481


Page 1060 of 1481
**Kato**

Page 1061 of 1481


Kato Answers: 2AC (1/4)
FIRST, WE OUTWEIGH: EVEN IF WE IGNORE PAST NUCLEAR WARS, THE ONES THAT
PLAN SOLVES ARE BAD IN AND OF THEMSELVES AND WOULD KILL THE VERY
PEOPLE THEY DESCRIBE

SECOND, PERM - DO THE PLAN AND ACCEPT THAT NUCLEAR WARS VIA NUCLEAR
TESTING AND URANIUM MINING ARE BEING CARRIED OUT AGAINST INDIGENOUS
PEOPLES AND THE FOURTH WORLD. THIS SOLVES BECAUSE THE ALT IS LITERALLY
PLAN PLUS.

THIRD, NO LINK: KATO CRITICIZES NOT RECOGNIZING TESTING AS AN ACTUAL


NUCLEAR WAR WE ONLY SAY THAT THE PLAN PREVENTS A NUCLEAR WAR,
RECOGNIZING ONGOING NUCLEAR WARS

Kato, Political Science Professor at the University of Hawaii at Honolulu, 93 (Masahide, Nuclear Globalism:
Traversing Rockets, Satellites, and Nuclear War, Alternatives, V. 18, N. 3)

Nuclear criticism finds the likelihood of "extinction" as the most fundamental aspect of nuclear
catastrophe. The complex problematics involved in nuclear catastrophe are thus reduced to the single possible instant of extinction. The task of nuclear critics is clearly designated by
Schell as coming to grips with the one and only final instant "human extinction-whose likelihood we are chiefly interested in finding out about:" Deconstructionists, on the other hand, take a
detour in their efforts to theologize extinction. Jacques Derrida, for example, solidified the prevailing mode of representation by constituting extinction as a fatal absence: Unlike the other
wars, which have all been preceded by wars of more or less the same type in human memory (and gunpowder did not mark a radical break in this respect), nuclear war has no precedent. It
has never occurred, itself; it is a non-event. The explosion of American bombs in 1945 ended a "classical," conventional war, it did not set off a nuclear war The terrifying reality of the nuclear

By representing the possible


conflict can only be the signified referent, never the real referent (present or past) of a discourse or text At least today apparently."

extinction as the single most important problematic' of nuclear catastrophe (posing it as either a threat or a symbolic void), nuclear'
criticism disqualifies the entire history of nuclear violence, the "real" of nuclear catastrophe as a
continuous and repetitive process. The "real" of nuclear war is designated by nuclear critics as a "rehearsal' (Derrik De Kerkhove) or "preparation" (Firth) for what
they reserve as the authentic catastrophes' The history of nuclear violence offers, at best, a reality effect to the imagery of

"extinction." Schell summarized the discursive position of nuclear critics very succinctly, by stating that nuclear catastrophe should not be conceptualized "in the context of direct
slaughter of hundreds of millions people by the local effects: "8 Thus the elimination of the history of nuclear violence by nuclear critics stems from the process of discursive "delocalization" of
nuclear violence. Their primary focus is not local catastrophe, but delocalized, unlocatable, "global" catastrophe

FOURTH, EXTINCTION OF THE SPECIES IS THE MOST HORRIBLE IMPACT


IMAGINEABLE, PUTTING RIGHTS FIRST IS PUTTING A PART OF SOCIETY BEFORE THE
WHOLE

Schell 1982
(Jonathan, Professor at Wesleyan University, The Fate of the Earth, pages 136-137 uw//wej)

Implicit in everything that I have said so far about the nuclear predicament there has been a perplexity that I would now like to take up explicitly, for it leads, I
believe, into the very heart of our response-or, rather, our lack of response-to the predicament. I have pointed out that our species is the most important of all
the things that, as inhabitants of a common world, we inherit from the past generations, but it does not go far enough to point out this superior importance, as
though in making our decision about ex- tinction we were being asked to choose between, say, liberty, on the one hand, and the survival of the species, on the

Page 1062 of 1481


world, and to speak of sacrificing the
other. For the species not only overarches but contains all the benefits of life in the common

species for the sake of one of these benefits involves one in the absurdity of wanting to de- stroy
something in order to preserve one of its parts, as if one were to burn down a house in an
attempt to redecorate the living room, or to kill someone to improve his character. ,but even to point out this absurdity fails to take the
full measure of the peril of extinction, for mankind is not some invaluable object that lies outside us and that we must protect so that we can go on benefiting
from it; rather, it is we ourselves, without whom everything there is loses its value. To say this is another way of saying that extinction is unique not because it
destroys mankind as an object but because it destroys mankind as the source of all possible human subjects, and this, in turn, is another way of saying that
extinction is a second death, for one's own individual death is the end not of any object in life but of the subject that experiences all objects. Death, how- ever,
places the mind in a quandary. One of-the confounding char- acteristics of death-"tomorrow's zero," in Dostoevski's phrase-is that, precisely because it
removes the person himself rather than something in his life, it seems to offer the mind nothing to take hold of. One even feels it inappropriate, in a way, to try
to speak "about" death at all, as. though death were a thing situated some- where outside us and available for objective inspection, when the fact is that it is
within us-is, indeed, an essential part of what we are. It would be more appropriate, perhaps, to say that death, as a fundamental element of our being,

"thinks" in us and through us about whatever we think about, coloring our thoughts and moods with its presence throughout our lives .

Page 1063 of 1481


Page 1064 of 1481
Kato Answers: 2AC (2/4)
FIFTH, NUCLEARISM IS INEVITABLE MAINTENANCE AND DETERRENCE ARE
NECESSARY FOR WORLD PEACE

Robinson 2001
[C. Paul, Sandi National Laboraties, A White Paper:Pursuing a New Nuclear Weapons Policy for the 21st Century,
March 22, www.mindfully.org/Nucs/Nuclear-Weapons-Policy-21stC.htm, 9-23-06//uwyo-ajl]

I served as an arms negotiator on the last two agreements before the dissolution of the Soviet Union and have spent most of my career enmeshed in the complexity of nuclear
weapons issues on the government side of the table. It is abundantly clear (to me) that formulating a new nuclear weapons policy for the start of the 21st Century will be a most
difficult undertaking. While the often over-simplified picture of deterrence during the Cold War-two behemoths armed to the teeth, staring each other down-has thankfully

, there are nevertheless huge arsenals of nuclear weapons and delivery systems, all in quite usable
retreated into history

states, that could be brought back quickly to their Cold War postures. Additionally, throughout the Cold War and ever since, there
has been a steady proliferation of nuclear weapons and other weapons of mass destruction by other nations around the globe. The vast majority of these newly armed states
are not U.S. allies, and some already are exhibiting hostile behaviors, while others have the potential to become aggressors toward the U.S., our allies, and our international
interests.

Russia has already begun to emphasize the importance of its arsenal of nuclear weapons to compensate for its limited conventional capabilities to deal with hostilities that
appear to be increasing along its borders. It seems inescapable that the U.S. must carefully think through how we should be preparing to deal with new threats from other
corners of the world, including the role that nuclear weapons might serve in deterring these threats from ever reaching actual aggressions.

I personally see the abolition of nuclear weapons as an impractical dream in any foreseeable future. I came to this view

from several directions. The first is the impossibility of ever "uninventing" or erasing from the human
mind the knowledge of how to build such weapons. While the sudden appearance of a few tens of nuclear weapons causes only a small stir in a world where
several thousands of such weapons already exist, their appearance in a world without nuclear weapons would produce huge

effects. (The impact of the first two weapons in ending World War II should be a sufficient example.) I believe that the words of Winston Churchill, as quoted by Margaret
Thatcher to a special joint session of the U.S. Congress on February 20, 1985, remain convincing on this point: "Be careful above all things not to let go of the atomic weapon
until you are sure, and more sure than sure, that other means of preserving the peace are in your hands."

the majority of the nations who have now acquired arsenals of nuclear weapons
Similarly, it is my sincere view that

believe them to be such potent tools for deterring conflicts that they would never surrender them. Against this
backdrop, I recently began to worry that because there were few public statements by U.S. officials in reaffirming the unique role which nuclear weapons play in ensuring U.S.
and world security, far too many people (including many in our own armed forces) were beginning to believe that perhaps nuclear weapons no longer had value. It seemed to
me that it was time for someone to step forward and articulate the other side of these issues for the public: first, that nuclear weapons remain of vital importance to the security
nuclear weapons will likely have an enduring role in
of the U.S. and to our allies and friends (today and for the near future); and second, that

preserving the peace and preventing world wars for the foreseeable future. These are my purposes in writing this paper.

SIXTH, IMAGINING NUCLEAR ANNIHILATION IS A PROJECT OF SURVIVAL THEIR


ALTERNATIVE CREATES REPRESSION AND DENIAL WHICH MAKES NUCLEAR WAR
MORE LIKELY

Lenz, Science and Policy Professor at SUNY, 90 (Nuclear Age Literature For Youth, p. 9-10)

A summary of Franks thought in Psychological Determinants of the Nuclear Arms Race notes how all
people have difficulty grasping the
magnitude and immediacy of the threat of nuclear arms and this psychological unreality is a basic obstacle to eliminating
that threat. Only events that people have actually experienced can have true emotional impact. Since Americans have escaped the devastation of nuclear
weapons on their own soil and nuclear weapons poised for annihilation in distant countries cannot be seen, heard, smelled, tasted, or touched, we find
it easy to imagine ourselves immune to the threat. Albert Camus had the same phenomenon in mind when he wrote in his essay
Neither Victims nor Executioners of the inability of most people really to imagine other peoples death (he might have added or their own). Commenting on

Page 1065 of 1481


Camus, David P. Barash and Judith Eve Lipton observed that this distancing from deaths reality is yet another aspect of our
insulation from lifes most basic realities. We make love by telephone, we work not on matter but on machines, and we kill and are killed
by proxy. We gain in cleanliness, but lose in understanding. If we are to heed Camuss call to refuse to be either the victims of
violence like the Jews of the Holocaust, or the perpetrators of it like the Nazi executioners of the death camps, we must revivify the
imagination of what violence really entails. It is here, of course, that the literature of nuclear holocaust can
play a significant role. Without either firsthand experience or vivid imagining, it is natural, as Frank points out, to deny the
existence of death machines and their consequences. In psychiatric usage denial means to exclude from
awareness, because letting [the instruments of destruction] enter consciousness would create too strong a level of anxiety or other painful emotions. In
most life-threatening situations, an organisms adaptation increases chances of survival, but ironically, adapting ourselves to nuclear fear is
counterproductive. We only seal our doom more certainly. The repressed fear, moreover, takes a psychic toll.

Page 1066 of 1481


Page 1067 of 1481
Kato Answers: 2AC (3/4)
SEVENTH, CRITICIZING REPRESENTATIONS OF NUCLEAR PRESENCE DOESNT
PRECLUDE THE NEED FOR CONCRETE ACTION

Richard Rorty, Professor of Humanities, University of Virginia, Truth, Politics, and Postmodernism, Spinoza Lectures,

1997, p. 51-2

This distinction between the theoretical and the practical point of view is often drawn by Derrida, another writer who enjoys demonstrating that something very important
meaning, for example, or justice, or friendship is both necessary and impossible. When asked about the implications of these paradoxical fact, Derrida usually replies that

the paradox doesn't matter when it comes to practice. More generally, a lot of the writers who are labeled
`post-modernist; and who talk a lot about impossibility, turn out to be good experimentalist social democrats when it
comes to actual political activity. I suspect, for example, that Gray, Zizek, Derrida and I, if we found ourselves citizens of the same country, would all be
voting for the same candidates, and supporting the same reforms. Post-modernist philosophers have gotten a bad name because of their paradox-mongering habits, and their
constant use of terms like `impossible; `self-contradictory' and `unrepresentable'. They have helped create a cult of inscrutability, one which defines itself by opposition to the
Enlightenment search for transparency - and more generally, to the `metaphysics of presence; the idea that intellectual progress aims at getting things clearly illuminated,

. I am all for getting rid of the metaphysics of presence, but I think that the rhetoric of
sharply delimited, wholly visible

impossibility and unrepresentability is counterproductive overdramatization. It is one thing to say that we need to get rid of the
metaphor of things being accurately represented, once and for all, as a result of being bathed in the light of reason. This metaphor has created a lot of headaches for
philosophers, and we would be better off without it. But that does not show that we are suddenly surrounded by unrepresentables; it just shows that `more accurate

Even if we agree that we shall never have what Derrida calls "a full
representation' was never a fruitful way to describe intellectual progress.

presence beyond the reach of play"; our sense of the possibilities open to humanity will not have
changed. We have learned nothing about the limits of human hope from metaphysics, or from the philosophy of history, or from psychoanalysis. All that we have learned
from `post-modern' philosophy is that we may need a different gloss on the notion of `progress' than the rationalistic gloss which the Enlightenment offered. We have

been given no reason to abandon the belief that a lot of progress has been made by carrying out the
Enlightenment's political program. Since Darwin we have come to suspect that whether such progress is made will be largely a matter of luck. But we
have been given no reason to stop hoping to get lucky.

EIGHTH, PLAN SOLVES WORSE IMPERIALISM BY ENDING THE UNILATERAL AND


INDEFINITE DETAINMENT AND TORTURE OF ENEMY COMBATANTS BY THE
EXECUTIVE

NINTH, APPROPRIATING THE OTHER VIOLENTLY SEIZES THE RIGHT TO SPEAK FOR
SELFISH ENDS

Routledge 96
[Antipode]

The issue of representation is a vexed one which has received much attention within the social
sciences. For example, in discussing the academic strategy of polyphony, Crang (1992) raises
issues of how the voices of others are (re)presented; the extent to which these voices are
interwoven with persona of narrator the degree of authorial power regarding who initiates
research, who decides on textual arrangements, and who decides which voices are heard; and
the power relations involved in the cultural capital conferred by specialist knowledge.

Page 1068 of 1481


Moreover, Harrison (quoted in McLaren 1995 240) argues that polyphony can end up being
aform of romantic ventroloquism creating the magical notion of the Others coming to voice.
These questions have important political implications for research which must be negotiated
according to the specific circumstances of a particular project. It is all too easy for academics to
claim solidarity with the oppressed and act as relays for their voices within social scientific
discourse. This raises the danger of an uncritical alignment with resisters on the assumption that
they know all there is to know without the intervention of intellectuals; and hence an
academics role becomes that of helping them seize the right to speak.

Page 1069 of 1481


Page 1070 of 1481
Kato Answers: 2AC (4/4)
TENTH, THEY PORTRAY THE FOURTH WORLD AS POWERLESS VICTIMS. THIS IS THE
NEW MEANS OF COLONIAL PACIFICATION IT PRESUPPOSES THE INEVITABLE
DEFEAT OF THE FOURTH WORLD AND UNDERMINES ANY MOVES TOWARDS REAL
SOLIDARITY THE IMPACT GUTS THEIR ALTERNATIVE SOLVENCY AND FLIPS THE K

Root, Professor of Critical Theory at the University of Toronto 97 (Deborah, Borrowed Power: Essays on Cultural
Appropriation, Edited by Bruce Ziff)

Why would Karma and his countercultural predecessors identify with people who, time and time again, are presented as victims? First Nations writer Deborah Doxtator makes the point that

it is precisely the image of Indians as doomed victims that some white people identify with: she calls this the "I'm a victim too
complex. Indeed, Friedrich Nietzsche conceived, something like this complex as the very core of Christian culture, underlining the link between pity and contempt. Thinking of

someone else as a victim is a way of displacing one's own pain: in reactive Christian thinking, I am less of a victim than you because you are
more of a victim than me. White hippies do tend to recognize some of the oppressive aspects of industrial, consumerist society

but manifest this by focusing on and identifying with people who seem to be even more oppressed, thus
reproducing the 1970s movie version of Natives as defeated victims who exist only in the past. Western
culture is permeated with the duplicitous, Christian notion of victimization, which on the one hand implies a
moral or spiritual superiority and on the other a kind of weakness that is to be overcome. Martyred saints are
represented as suffering physical torment with a heroic steadfastness of faith. Yet the body, whether sinful or suffering, is thought to be inherently abject. Thus, to be a victim is to be both

heroic and abject. White representations (both "sympathetic" and explicitly racist) of colonial wars tend to maintain this
definition and underline the view that Native heroism derives from and is the consequence of defeat. The white
fascination with the romantic, abstract heroism of Native people is thus able to function as another
means of colonial pracification because it presupposes the inevitable defeat and disappearance of the
nations. Colonialism adds a new twist to the Christian view that people are victims by their very nature or essence, and here the relation between aggressor and victim becomes wholly
static and cannot shift. Every-one is frozen into his or her position and role. And, of course, conceiving of an enemy nation as heroic also

makes the oppressors look good because they have defeated a truly worthy and valiant enemy . This, too, is nothing
new in Western culture. Recall the famous Roman sculpture of the dying Gaul, an image of a heroic, yet defeated enemy. Here we approach what it was we all forgot in our eagerness to

embrace the representation of Inidans as heroic victims:if Native nations are portrayed as inherently abject and doomed to defeat,
white viewers will not feel any connection to colonialism, either in the past or in the present. This is why the phony Native culture of movies,
Edward Curtis photographs, and television is so appealing to white people: if, as Hollywood and capitalism would have it, the nations are foreordained to

assimilate and vanish, then white viewers need not question racism or face the discomfort of
interrogating our continuing position as members of a colonizing nation. We will not feel connected to
ongoing struggles in James Bay, Chiapas, Kanesatake, and elsewhere and to the different relation to the land that these struggles express. Any sense of
connection to events occurring on the ground is lost, and "Native" becomes another empty category that can
be mined for its trappings and images. And the "love" of Indians professed by counterculture old and new continues to have

nothing to do with Native people and certainly nothing to do with supporting contemporary Native
struggles. Westerns and other colonial narratives are in the business of producing binarisms which have had effects on all of us. As white people, we need to
rethink and recover the histories erased by popular culture and school textbooks. There were always alternatives to John Wayne. We also need
to think through the nature of power and its relation to culture. John Trudell said somewhere that there is a difference between being oppressed and being powerless: Native people may be

It is up to us to
oppressed, but the traditions have power; white people may be "in charge" within a colonial context, but our culture has lost its heart, soul, and life-its power.

look into how our traditions were taken over and distorted by a destructive, soulless ethos and find ways
to heal our cultural diseases. This is where Karma's approach breaks down: he thinks he has to turn himself into a "white-skinned Indian" because he cannot find a way
to transform and locate power in his own tradition. Because of the elided histories, he is unable to identify with the white people who have resisted oppression over the centuries. He, too, is
rendered passive by the romantic discourse of inevitable defeat and disappearance. And because Karma thinks white culture is one thing-the dead, shopping-mall culture of our time-
appropriation becomes his only escape, and it becomes impossible for him to imagine standing side by side with Native people as equals.

Page 1071 of 1481


Page 1072 of 1481
**Levinas/Derrida**

Page 1073 of 1481


A2 Infinite Responsibility (1/3)
[you might want to read Calculability Good]

DERRIDAS ETHIC TOWARDS THE OTHER REQUIRES THINKING THROUGH THE


OPPRESSORS EYES, DESTROYING ETHICS

Jack M. Balkin, Knight Professor of Constitutional Law and the First Amendment at Yale Law, Transcendental
Deconstruction, Transcendent Justice-- Part II, 92 Mich. L. Rev. 1131, 1994,
http://www.yale.edu/lawweb/jbalkin/articles/trans02.htm

Derrida's ethics of Otherness has a second component: It employs a different sense of individuality and uniqueness. Under this view, justice
requires one to speak in the language of the Other by trying to see things from the Other's point of view.
(78) This conception of justice seems most attractive when we are the injurer or the stronger party in a relationship, or when we are in the position of a judge who is attempting
to arbitrate between competing claims. For example, suppose that we are the State, the stronger party, the oppressor, or the injurer, or suppose that we are contemplating an
action that might put us in such a position. It seems only just that we should try to understand how we have injured or oppressed the Other (or might be in a position to injure or
oppress). We can only do this if we try to see the problem from the Other's perspective and understand her pain and her predicament in all of its uniqueness. The duty we owe
to the Other is the duty to see how our actions may affect or have affected the Other; to fulfill this duty we must put away our own preconceptions and vocabulary and try to see
things from her point of view. Similarly, if we are a judge in a case attempting to arbitrate between the parties, the ethics of Otherness demands that we try to understand how

Suppose, however, that we are not the injurer,


our decision will affect the two parties, and this will require us to see the matter from their perspective.

but the victim; not the State, but the individual; not the strong, but the weak; not the oppressor, but the
oppressed. Does justice require that we speak in the language of the person we believe is injuring or
oppressing us? Must a rape victim attempt to understand her violation from the rapist's point of view?
Does justice demand that she attempt to speak to the rapist in his own language - one which has treated
her as less than human? Must a concentration camp survivor address her former captor in the language of
his worldview of Aryan supremacy? We might wonder whether this is what justice really requires,
especially if the injustice we complain of is precisely that the Other failed to recognize us as a person, refused
to speak in our language, and declined to consider our uniqueness and authenticity.

VOTE TO SAVE LIVESTHE EXISTENCE OF ENDANGERED 3RD PARTIES MAKES


RESPONSIBILITY IMPOSSIBLE TO DETERMINE

David Campbell, professor of international politics at the University of Newcastle, Moral Spaces: Rethinking Ethics

and World Politics, ed. by Campbell and Shapiro, 1999, p. 35-36

there is no
Levinas's thought is appealing for rethinking the question of responsibility, especially with respect to situations like the Balkan crisis, because it maintains that

circumstance under which we could declare that it was not our concern. As Levinas notes, people can (and obviously do)
conduct their relationship to the Other in terms of exploitation, oppression, and violence. But no matter how allergic to the other is the self,

"the relation to the other, as a relation of responsibility, cannot be totally suppressed, even when it takes the form of
politics or warfare." In consequence, no self can ever opt out of a relationship with the other: "[I]t is impossible to free myself by saying, 'It's

not my concern.' There is no choice, for it is always and inescapably my concern. This is a unique 'no choice,' one that is not slavery." This unique lack of choice
comes about because in Levinas's thought ethics has been transformed from something independent of subjectivitythat is, from a set of rules and regulations adopted by
pregiven, autonomous agentsto something insinuated within and integral to that subjectivity. Accordingly, ethics can be understood as something not ancillary to the
existence of a subject; instead, ethics can be appreciated for its indispensability to the very being of the subject. This argument leads us to the recognition that "we" are always
already ethically situated, so making judgments about conduct depends less on what sort of rules are invoked as regulations and more on how the interdependencies of our
relations with others are appreciated. To repeat one of Levinas's key points: "Ethics redefines subjectivity as this heteronomous responsibility, in contrast to autonomous
freedom." Suggestive though it is for the domain of international relations where the bulk of the work on ethics can be located within a conventional perspective on responsibility

Levinas's formulation of responsibility, subjectivity, and ethics nonetheless possesses some problems when it comes to

Page 1074 of 1481


politics. What requires particular attention is the means by which the elemental and omnipresent status
the implications of this thought for

face-to-face relationship, can function in circumstances marked by a


of responsibility, which is founded in the one-to-one or

multiplicity of others. Although the reading of Levinas here agrees that "the ethical exigency to be responsible to the other undermines the ontological primacy of
the meaning of being," and embraces the idea that this demand "unsettles the natural and political positions we have taken up in the world and predisposes us to a meaning
that is other than being, that is otherwise than being:" how those disturbances are negotiated so as to foster the maximum responsibility in a world populated by others in

I want to consider Levinas's


struggle remains to be argued. To examine what is a problem of considerable import given the context of this essay,

discussion of "the third person," the distinction he makes between the ethical and the moral, andof particular importance in a consideration of the
politics of international actionthe role of the state in Levinas's thought.

Page 1075 of 1481


Page 1076 of 1481
A2 Infinite Responsibility (2/3)
EMPATHIZING WITH THE OTHER IGNORES LARGER STRUCTURES OF DOMINATION,
REINSCRIBING THE GAP BETWEEN THE SELF AND THE OTHER

Rey Chow, Associate Professor of English and Comparative Literature at the University of California at Irvine,
Writing Diaspora: Tactics of Intervention in Contemporary Cultural Studies, 1993, p. 12 15, UK: Fisher

In the "cultural studies" of the American academy in the 1990s, the Maoist is reproducing with prowess.
We see this in the way terms such as "oppression," "victimization," and "subalternity" are now being
used. Contrary to Orientalist disdain for contemporary native cultures of the non-West, the Maoist turns
precisely the "disdained'' other into the object of his/her study and, in some cases, identification. In a
mixture of admiration and moralism, the Maoist sometimes turns all people from non-Western cultures
into a generalized "subaltern" that is then used to flog an equally generalized "West." 21

Because the representation of "the other" as such ignores (1) the class and intellectual hierarchies within
these other cultures, which are usually as elaborate as those in the West, and (2) the discursive power
relations structuring the Maoist's mode of inquiry and valorization, it produces a way of talking in which
notions of lack, subalternity, victimization, and so forth are drawn upon indiscriminately, often with the
intention of spotlighting the speaker's own sense of alterity and political righteousness. A comfortably
wealthy white American intellectual I know claimed that he was a "third world intellectual," citing as one
of his credentials his marriage to a Western European woman of part-Jewish heritage; a professor of
English complained about being "victimized" by the structured time at an Ivy League institution, meaning
that she needed to be on time for classes; a graduate student of upper-class background from one of the
world's poorest countries told his American friends that he was of poor peasant stock in order to
authenticate his identity as a radical "third world" representative; male and female academics across the
U.S. frequently say they were "raped" when they report experiences of professional frustration and
conflict. Whether sincere or delusional, such cases of self-dramatization all take the route of self-
subalternization, which has increasingly become the assured means to authority and power. What these
intellectuals are doing is robbing the terms of oppression of their critical and oppositional import, and
thus depriving the oppressed of even the vocabulary of protest and rightful demand. The oppressed,
whose voices we seldom hear, are robbed twicethe first time of their economic chances, the second
time of their language, which is now no longer distinguishable from those of us who have had our
consciousnesses "raised."

In their analysis of the relation between violence and representation, Armstrong and Tennenhouse write:
"[The] idea of violence as representation is not an easy one for most academics to accept. It implies that
whenever we speak for someone else we are inscribing her with our own (implicitly masculine) idea of
order." 22 At present, this process of "inscribing" often means not only that we "represent" certain
historic others because they are/were ''oppressed"; it often means that there is interest in representation
only when what is represented can in some way be seen as lacking. Even though the Maoist is usually
contemptuous of Freudian psychoanalysis because it is "bourgeois," her investment in oppression and
victimization fully partakes of the Freudian and Lacanian notions of "lack." By attributing "lack," the
Maoist justifies the "speaking for someone else" that Armstrong and Tennenhouse call "violence as
representation." As in the case of Orientalism, which does not necessarily belong only to those who are
white, the Maoist does not have to be racially "white" either. The phrase "white guilt" refers to a type of
discourse which continues to position power and lack against each other, while the narrator of that
discourse, like Jane Eyre, speaks with power but identifies with powerlessness. This is how even those
who come from privilege more often than not speak from/of/as its "lack." What the Maoist demonstrates
is a circuit of productivity that draws its capital from others' deprivation while refusing to acknowledge its

Page 1077 of 1481


own presence as endowed. With the material origins of her own discourse always concealed, the Maoist
thus speaks as if her charges were a form of immaculate conception.

[Continues.No Text Removed]

Page 1078 of 1481


Page 1079 of 1481
A2 Infinite Responsibility (3/3)
[Continued.No Text Removed]

The difficulty facing us, it seems to me, is no longer simply the "first world" Orientalist who mourns the
rusting away of his treasures, but also students from privileged backgrounds Western and non-Western,
who conform behaviorally in every respect with the elitism of their social origins (e.g., through powerful
matrimonial alliances, through pursuit of fame, or through a contemptuous arrogance toward fellow
students) but who nonetheless proclaim dedication to "vindicating the subalterns." My point is not that
they should be blamed for the accident of their birth, nor that they cannot marry rich, pursue fame, or
even be arrogant. Rather, it is that they choose to see in others' powerlessness an idealized image of
themselves and refuse to hear in the dissonance between the content and manner of their speech their
own complicity with violence. Even though these descendents of the Maoist may be quick to point out
the exploitativeness of Benjamin Disraeli's "The East is a career," 23 they remain blind to their own
exploitativeness as they make "the East" their career. How do we intervene in the productivity of this
overdetermined circuit?

Page 1080 of 1481


Page 1081 of 1481
Levinas Destroys Ethics (1/2)
LEVINASIAN ETHICS IS ORIENTED TOWARDS PRE-ONTOLOGICAL ALTERITY, BASED
ON ORIGINARY NEGATIVITY, BLOCKING AFFIRMATION OF ETHICS IN SPECIFIC
CONTEXTS

Hallward 2001
[Peter, Nip/Tuck junky, Ethics: An Essay on the Understanding of Evil, Trans. Peter Hallward, New York:
Verso, 2001, xxii-xxiii//uwyo-ajl]

For Badiou, true ethical questions can arise only in a specific situation and under circumstances which,
however divisive , are essentially indifferent to differences, concerning subjects 'disinterested' in the other as such, the other
qua other (i.e. in the circumstances created by a truthprocedure). The 'ethical ideology', by contrast, precisely presumes

to transcend all situated restrictions and to prevail in a consensual realm beyond


division, all the while orientated around the imperious demands of difference and
otherness qua otherness, the difference of the altogether other as much as the irreducibly incommensurable demands of every particular
other. As Badiou is the first to recognize, nowhere is the essential logic more clearly articulated than in

Levinas's philosophy, where 'the Other comes to us not only out of context but also
without mediation... .'28 According to Levinas, there can be no ethical situation as such,
since ethics bears witness to a properly meta- or preontological responsibility (roughly, the
responsibility of a creature to its transcendent creator, a creator altogether beyond the ontological field of creation). For Levinas, as for Derrida after him, the
other is other only if he immediately evokes or expresses the absolutely (divinely) other.

our
Since the alterity of the other is simultaneously 'the alterity of the human other [Autruzl and of the Most High [Tres Haut]' ,29 so then

responsibility to this other is a matter of 'unconditional obedience' , 'trauma', 'obsession', 'persecution', and
so on.30 Of course, the limited creatures that we are can apprehend the Altogether-Other only if this otherness appears in some sense 'on our own level', that
is, in the appearing of our 'neighbour' (of our neighbour's face): there is only 'responsibility and a Self because the trace of the [divinely] Infinite . . . is inscribed
in my 'non-relation' with the Other, 'the
in proximity'.31 But this inscribing in nearness in no sense dilutes the essential fact that

Other remains absolute and absolves itself from the relation which it enters into'.32 The relation
with the other is first and foremost a 'relation' with the transcendent.beyond as such. Levinasian ethics, in short, is a form of what Badiou criticizes as anti-

philosophy, that is, the reservation of pure or absolute value to a realm beyond all conceptual distinction .

LEVINAS ARGUMENT DEPENDS ON THE THEOLOGICAL INFINITY OF GOD. SECULAR


APPROPRIATION LAPSES INTO FINITUDE, BLOCKING RESPONSIBILITY

Badiou 2001
[Alain, Number muncher, Ethics: An Essay on the Understanding of Evil, Trans. Peter Hallward, New York:
Verso, 2001, 21-3//uwyo-ajl]

: the ethical primacy of the


The difficulty, which also defines the point of application for these axioms, can be explained as follows

Other over the Same requires that the experience of alterity be ontologically 'guaranteed'
as the experience of a distance, or of an essential non-identity, the traversal of which is the ethical experience itself. But
nothing in the simple phenomenon of the other contains such a guarantee. And this simply because
the finitude of the other's appearing certainly can be conceived as resemblance, or as imitation, and thus lead back to the logic of the Same. The other always
resembles me too much for the hypothesis of an originary exposure to his alterity to be necessarily true.

The phenomenon of the other (his face) must then attest to a radical alterity which he nevertheless does not contain by himself. The Other, as he appears to
me in the order of the finite, must be the epiphany of a properly infinite distance to the other, the traversal of which is the originary ethical experience.

Page 1082 of 1481


n order to be intelligible, ethics requires that the Other be in some sense
This means that i

carried by a principle of alterity which transcends mere finite experience. Levinas calls
this principle the 'Altogether-Other', and it is quite obviously the ethical name for God.
There can be no Other if he is not the immediate phenomenon of the AltogetherOther. There can be no finite devotion to the non-identical if it is not sustained

by the infinite devotion of the principle to that which subsists outside it . There can be no ethics without God the
ineffable.

to
In Levinas's enterprise, the ethical dominance of the Other over the theoretical ontology of the same is entirely bound up with a religious axiom;

believe that we can separate what Levinas's thought unites is to betray the intimate
movement of this thought, its subjective rigour. In truth, Levinas has no philosophy - not even philosophy as the
'servant' of theology. Rather, this is philosophy (in the Greek sense of the word) annulled by theology, itself no longer a theology (the terminology is still too
Greek, and presumes proximity to the divine via the identity and predicates of God) but, precisely, an ethics.

To make of ethics the ultimate name of the religious as such (i.e. of that which relates [re-lie] to the Other under the ineffable authority of the Altogether-Other)
is to distance it still more completely from all that can be gathered under the name of 'philosophy'.

every effort to turn ethics into the


To put it crudely: Levinas's enterprise serves to remind us, with extraordinary insistence, that

principle of thought and action is essentially religious. We might say that Levinas is the coherent and inventive
thinker of an assumption that no academic exercise of veiling or abstraction can obscure: distanced from its Greek usage (according to which it is clearly
subordinated to the theoretical), and taken in general, ethics is a category of pious discourse.

Page 1083 of 1481


Page 1084 of 1481
Levinas Destroys Ethics (2/2)
OBSESSION WITH ABSTRACT RESPONSIBILITY ALLOWS US TO IGNORE
SINGULARITY, MEANING THAT NONE OF THEIR SWEEPING DEMANDS MANIFEST IN
THE REAL WORLD WE FEEL BETTER ABOUT OURSELVES WHILE PEOPLE ARE
ENSLAVED

Badiou 2001
[Alain, Number muncher, Ethics: An Essay on the Understanding of Evil, Trans. Peter Hallward, New York:
Verso, 2001, 14-6//uwyo-ajl]

3. Finally, thanks to its negative and a priori determination of evil, ethics prevents itself
from thinking the singularity of situations as such, which is the obligatory starting point of all
properly human action. Thus, for instance, the doctor won over to 'ethical' ideology will
ponder, in meetings and commissions, all sorts of considerations regarding 'the sick',
conceived of in exactly the same way as the partisan of human rights conceives of the
indistinct crowd of victims - the 'human' totality of subhuman entities [reels].

But the same doctor will have no difficulty in accepting the fact that this particular person
is not treated at the hospital, and accorded all necessary measures, because he or she is
without legal residency papers, or not a contributor to Social Security. Once again, 'collective'
responsibility demands it! What is erased in the process is the fact that there is only one
medical situation, the clinical situation,7 and there is no need for an 'ethics' (but only for a
clear vision of this situation) to understand that in these circumstances a doctor is a doctor only
if he deals with the situation according to the rule of maximum possibility - to treat this person
who demands treatment of him (no intervention here!) as thoroughly as he can, using
everything he knows and with all the means at his disposal, without taking anything else into
consideration. And if he is to be prevented from giving treatment because of the State budget,
because of death rates or laws governing immigration, then let them send for the police! Even
so, his strict Hippocratic duty would oblige him to resist them, with force if necessary.

'Ethical commissions' and other ruminations on 'healthcare expenses' or 'managerial


responsibility', since they are radically exterior to the one situation that is genuinely
medical, can in reality only prevent us from being faithful to it. For to be faithful to this
situation means: to treat it right to the limit of the possible. Or, if you prefer: to draw from
this situation, to the greatest possible extent, the affirmative humanity that it contains. Or
again: to try to be the immortal of this situation.
As a matter of fact, bureaucratic medicine that complies with ethical ideology depends on
'the sick' conceived as vague victims or statistics, but is quickly overwhelmed by any
urgent, singular situation of need. Hence the reduction of 'managed', 'responsible' and
'ethical' health-care to the

abject task of deciding which sick people the 'French medical system' can treat and which others
- because the Budget and public opinion demand it - it must send away to die in the
shantytowns of Kinshasa.

Page 1085 of 1481


Page 1086 of 1481
Levinas/Derrida Destroy Ethics
THE ALTERNATIVE IS PREMISED ON THE INCALCULABLE CALL OF THE OTHER,
PREVENTING AN UNCONDITIONAL COMMITMENT TO A TRUTH EVENT THAT
PRODUCES SUBJECTIVITY, PROVIDING THE ONLY ACCESS TO THE SINGULARITY
BECAUSE OF ITS UNIVERSAL ACCESSIBILITY

Hallward 2001
[Peter, Nip/Tuck junky, Ethics: An Essay on the Understanding of Evil, Trans. Peter Hallward, New York:
Verso, 2001, xxv-xxvii//uwyo-ajl]

Like Badiou, Derrida is careful to distinguish the realm of decision from the realm of knowledge. To reduce my decision to
respond to the calculus of reasons and the assessment of possibilities is to eliminate its radical character as a decision.
The decision must always concern what I cannot know. Ethics is a matter of
'responsibility in the experience of absolute decisions made outside of knowledge or given norms' .39 But Derrida does
not stop there.

The responsible decision must concern not only the notknown, it must evade
conceptualization altogether. 'In order for [absolute responsibility] to be what it must be it must remain
inconceivable, indeed unthinkable.'40 The decision becomes precisely what is impossible for the subject as such. If, then, a
Like Abraham
response or a decision does take place, it can only have been 'the decision of the other in me'. 41
responding to God's instruction to sacrifice his son, I must respond without trying to
interpret (and thus appropriate) the other's meaning. I must respond simply because radical otherness demands it; only
then do I become the unknowing vehicle for this other's decision.

Hence the mysterium tremendum whose 'trembling' quivers throughout Donner la mort : 'we fear and tremble
before the inaccessible secret of a God who decides for us although we remain
responsible' .42 Hence, too, the irreducibly 'tragic' and 'guilty' quality of Derrida's ethical
responsibility (54-5/51), the impasse of a responsibility to impossibly overwhelming (and impossibly
incommensurable) obligations. This impasse, moreover, is only exacerbated by any attempt to justify an ethical decision.
Since every such decision must be made by a fully solitary or 'irreplaceable' subject, so then its justification according to the
necessarily general or universal criteria of collective ethics threatens 'to dissolve my singularity in the medium of the
concept', to betray my secret within the publicity of language - in short, to threaten me with replacement.43 If it is to be a
genuine decision, it seems, the decision must take place as a pure leap of faith, one that resists any location in the situation,
any justification by its subject, and any 'conceptualization' by philosophy.

Badiou's emphasis on the material topology of a truthprocedure, by contrast, is designed


precisely to situate every such leap and to justify every apparently 'unjustifiable'
commitment in terms of its eternal and universal address.

The decision is no less 'incalculable', no less extra-ordinary or extra-legal. But for Badiou, an
ordinary (replaceable) individual becomes irreplaceable, becomes a (singular) subject,
only through this very commitment itself; it is only the commitment to a truth-process that 'induces a
subject'.44 Whereas Derrida maintains that responsibility to 'the absolute singularity of the other. . . calls for a betrayal of
we can access the
everything that manifests itself within the order of universal generality' ,45 Badiou declares that
realm of singularity only through adherence to strictly universal criteria - that is, to the
universality produced by a truth-procedure. Derrida's responsibility keeps itself 'apart and secret', it 'holds to what is apart
the literal basis
and secret' (33/26tm); whereas Badiou's commitment, inspired by Lacan's logic of the matheme
for an 'integral transmission' of truth46 - pursues clarity for all. Derrida's tension between (singular)
subject and (collective) justification disappears here without trace, as does every hint of pathos roused by a responsibllity
deemed impossible a priori. A true statement, as Badiou conceives it, is precisely one that can be made
by anyone, anyone at all.47 Again, with Badiou, impossibility is invariably thought in terms of a particular
situation, that is, as the Real of that situation, the void around which it is structured in its systematic
entirety - and thus the point from which, through a process of eminently' logical revolt' ,48 it becomes possible

Page 1087 of 1481


to transform the situation as a whole. And whereas both Badiou and Derrida orientate their ethics around the
advent of something 'to come' that escapes incorporation within any logic of anticipation or figuration, Badiou's event
remains situated vis-a-vis the state of the' situation (the elements of the 'symptomal' or 'evental' site
[site evenementiel] are perfectly accessible 'in their own right'; they are inaccessible only from within the perspective
whereas Derrida's messianic event is simply 'monstrous' in
adopted by the state of the situation),
the strong sense, consigned to a general 'formlessness'.

Page 1088 of 1481


Page 1089 of 1481
**Nietzsche**

Page 1090 of 1481


Nietzsche Answers: 2AC (1/6)
FIRST, TURN THE 1AC IS AN AFFIRMATION OF LIFE AGAINST THE NEGATIVITY OF
ENEMY COMBATANT DOCTRINE

SECOND, PERM DO BOTH


DEFENSE OF MULTIPLE PERSPECTIVES IS NECESSARY FOR CONSTANT CRITICISM,
CHALLENGING BEING BY THINKING IN FRAGMENTS

Bleiker 97
[Roland, PhD Cand @ Australian National U. of Political Sci, Alternatives 22, 57-85//uwyo]

No concept will ever be sufficient, will ever do justice to the object it is trying to capture. The objective then becomes to conceptualize thoughts so that they do not silence other
voices, but coexist and interact with them. Various authors have suggested methods for this purpose, methods that will always remain attempts without ever reaching the ideal

state that they aspire to. We know of Mikhail Bakhtins dialogism, a theory of knowledge and language that tries to avoid the excluding tendencies of monological thought
forms. Instead, he accepts the existence of multiple meanings, draws connections between differences, and searches
for possibilities to establish conceptual and linguistic dialogues among competing ideas, values, speech forms, texts, and validity claims, and the like. Jurgen
Habermas attempts to theorize the preconditions for ideal speech situations. Communication, in this case, should be as unrestrained as possible, such that claims to truth and
rightness can be discursively redeemed, albeit, one should add, though a rationalism and universalism that it violently anti-Bakhtinian and anti-Adornian. Closer to the familiar

empathetic cooperation, which aims at opening up questions of gender


terrain of IR we find Christine Sylvesters feminist method of

by a process of positional slippage that occurs when one listens seriously to the concerns, fears, and agendas of
those one is unaccustomed to heeding when building social theory. But how does one conceptualize such attempts if concepts can
ever do justice to the objects they are trying to capture?

The daring task is, as we know from Adorno, to open with concepts what does not fit into concepts, to resist the distorting power of reification and return the conceptual to the
nonconceptual. This disenchantment of the concept is the antidote of critical philosophy. It impedes the concept from developing its own dynamics and from becoming an
absolute in itself. The first step toward disenchanting the concept is simply refusing to define it monologically. Concepts should achieve meaning only gradually in relation to
each other. Adorno even intentionally uses the same concept in different way in order to liberate it from the harrow definition that language itself had already imposed on it. That

contradictions could arise out of this practice does not bother Adorno. Indeed, he considers them essential .

One cannot eliminate the contradictory, the fragmentary, and the discontinuous. Contradictions are only
contradictions if one assumes the existence of a prior universal standard of reference. What is different
appears as divergent, dissonant, and negative only as long as our consciousness strives for a totalizing standpoint,
which we must avoid if we are to escape the reifying and excluding dangers of identity thinking. Just as reality is
fragmented, we need to think in fragments. Unity then is not to be found be evening out discontinuities. Contradictions are to be

referred over artificially constructed meanings and the silencing of underlying conflicts. Thus, Adorno
advocates writing in fragments, such that the resulting text appears as if it always could be
interrupted, cut off abruptly, any time, and place. He adheres to Nietzsches advice that one should
approach deep problems like taking a cold bath, quickly into them and quickly out again. The belief that one does
not reach deep enough this way, he claims, is simply the superstition of those who fear cold water. But Nietzsches bath has already catapulted us into the vortex of the next
linguistic terrain of resistance the question of style.

THIRD, NO LINK WE DONT ASSERT DEVOTION TO A TRANSCENDENT LAW. PLAN


IS ONLY A CONTINGENT CONTESTATION OF NEGATIVITY

Page 1091 of 1481


Page 1092 of 1481
Nietzsche Answers: 2AC (2/6)
FOURTH, EVERY AFFIRMATIVE ETHICAL STANCE REQUIRES A REPRESSED ELEMENT
OF NEGATION, MEANING THAT EVERY AFFIRMATION OF LIFE OCCURS AGAINS THE
BACKGROUND OF HUMN DEATH AND FINITUDE

Zizek '99
[Slavoj, Senior Researcher at Institute for Social Studies, Ljubliana and Badass, The Ticklish Subject: the
absent centre of political ontology, New York: Verso, 1999, 153-4//uwyo-ajl]

It would therefore be tempting to risk a Badiouian-Pauline reading of the end of psychoanalysis, determining it as a New Beginning, a symbolic 'rebirth' - the radical restructuring
of the analysand's subjectivity in such a way that the vicious cycle of the superego is suspended, left behind. Does not Lacan himself provide a number of hints that the end of
analysis opens up the domain of Love beyond Law, using the very Pauline terms to which Badiou refers? Nevertheless, Lacan's way is not that of St Paul or Badiou:

does not already posit a 'new harmony', a new Truth-Event; it - as it were -


psychoanalysis is not 'psychosynthesis'; it

merely wipes the slate clean for one. However, this 'merely' should be put in quotation marks, because it is Lacan's contention that, in this
negative gesture of 'wiping the slate clean', something (a void) is confronted which is already 'sutured' with
the arrival of a new Truth-Event. For Lacan, negativity, a negative gesture of withdrawal, precedes any positive
gesture of enthusiastic identifiction with a Cause: negativity functions as the condition of (im)possibility of
the enthusiastic identification - that is to say, it lays the ground, opens up space for it, but is simultaneously
obfuscated by it and undermines it. For this reason, Lacan implicitly changes the balance between Death and Resurrection in favour of Death: what 'Death'
stands for at its most radical is not merely the passing of earthly life, but the 'night of the world', the self-
withdrawal, the absolute contraction of subjectivity, the severing of its links with 'reality' - this is the
'wiping the slate clean' that opens up the domain of the symbolic New Beginning, of the emergence of the 'New Harmony'
sustained by a newly emerged Master-Signifier. Here, Lacan parts company with St Paul and Badiou: God not only is but always-already was dead - that is to say, after Freud,

every such Event ultimately remains a semblance obfuscating a preceding Void


one cannot directly have faith in a Truth-Event;

whose Freudian name is death drive. So Lacan differs from Badiou in the determination of the exact status of this domain beyond the rule of the Law.
That is to say: like Lacan, Badiou delineates the contours of a domain beyond the Order of Being, beyond the politics of service des biens, beyond the 'morbid' super ego

: the
connection between Law and its transgressive desire. For Lacan, however, the Freudian topic of the death drive cannot be accounted for in the terms of this connection

'death drive' is not the outcome of the morbid confusion of Life and Death caused by the intervention of
the symbolic Law. For Lacan, the uncanny domain beyond the Order of Being is what he calls the domain 'between
the two deaths', the pre-ontologicalf domain of monstrous spectral apparitions, the domain that is 'immortal', yet not in the
Badiouian sense of the immortality of participating in Truth, but in the sense of what Lacan calls lamella, of the monstrous 'undead' object-libido.18

FIFTH, THE ALT DOESNT SOLVE WITHOUT PLAN, UNILATERAL DETAINMENT WILL
CONTINUE, LOCKING IN THE SLAVE MORALITY OF THE STATUS QUO

Page 1093 of 1481


Page 1094 of 1481
Nietzsche Answers: 2AC (3/6)
SIXTH, CALL TO REJECT RE-INVENTS HIERARCHIES POLITICAL ACTION IS KEY TO
TRANSCEND THE NIHILISTIC BINARY OF THE ALTERNATIVE

Newman 2001
[Saul, Sociology @ Macquarie University, Philosophy & Social Criticism 27: 3, pp. 4-6//uwyo]

It must be made clear, however, that Derrida does not simply want to invert the terms of these binaries so that the
subordinated term becomes the privileged term. He does not want to put writing in the place of speech, for instance. Inversion in this
way leaves intact the hierarchical, authoritarian structure of the binary division. Such a strategy only re-
affirms the place of power in the very attempt to overthrow it. One could argue that Marxism fell victim to this logic by replacing the
bour- geois state with the equally authoritarian workers state. This is a logic that haunts our radical political imaginary. Revolutionary political theories

have often succeeded only in reinventing power and authority in their own image. However, Derrida also
recognizes the dangers of subversion that is, the radical strategy of overthrowing the hierarchy altogether,
rather than inverting its terms. For instance, the classical anarchists critique of Marxism went along the lines that Marxism neglected political power in particular the power of
the state for economic power, and this would mean a restoration of political power in a Marxist revolution. Rather, for anarchists, the state and all forms of political power must

Derrida believes that subversion and inversion both culminate in the same
be abolished as the first revolutionary act. However,

thing the reinvention of authority, in different guises. Thus, the anarchist critique is based on the Enlightenment idea of a rational and
moral human essence that power denies, and yet we know from Derrida that any essential identity involves a radical exclusion or sup- pression of other identities. Thus,

anarchism substituted political and economic authority for a rational authority founded on an Enlighten-
ment-humanist subjectivity. Both radical politico-theoretical strategies then the strategy of inversion, as
exemplified by Marxism, and the strategy of subversion, as exemplified by anarchism are two sides of
the same logic of logic of place. So for Derrida:
What must occur then is not merely a suppression of all hierarchy, for an- archy only consolidates just as surely the established order of a metaphys- ical hierarchy; nor is it a
simple change or reversal in the terms of any given hierarchy. Rather the Umdrehung must be a transformation of the hierar- chical structure itself.

to avoid the lure of authority one must go beyond both the anarchic desire to destroy hierarchy,
In other words,

and the mere reversal of terms. Rather, as Derrida suggests, if one wants to avoid this trap the hierar- chical structure itself must be transformed.
Political action must invoke a rethinking of revolution and authority in a way that traces a path between
these two terms, so that it does not merely reinvent the place of power. It could be argued that Derrida propounds an anarchism
of his own, if by anarchism one means a questioning of all authority, including textual and philosophical authority, as well as a desire to avoid the trap of reproducing authority
and hierarchy in ones attempt to destroy it.

This deconstructive attempt to transform the very structure of hier- archy and authority, to go beyond the binary opposition, is also found in Nietzsche. Nietzsche believes that

One must, he argues,


one cannot merely oppose auth- ority by affirming its opposite: this is only to react to and, thus, affirm the domination one is supposedly resisting.

tran- scend oppositional thinking altogether go beyond truth and error, beyond being and becoming, beyond good and evil.
For Nietzsche it is simply a moral prejudice to privilege truth over error. However, he does not try to counter
this by privileging error over truth, because this leaves the opposition intact. Rather, he refuses to confine his view of the
world to this opposition: Indeed what compels us to assume that there exists any essential antithesis between true and false? Is it not enough to suppose
grades of apparentness and as it were lighter and darker shades and tones of appearance? Nietzsche displaces, rather than replaces, these oppositional and authoritarian
structures of thought he displaces place. This strategy of displacement, similarly adopted by Derrida, provides certain clues to developing a non-essentialist theory of resist-

. Rather than reversing the terms of the binary opposition, one should perhaps question,
ance to power and authority

and try to make prob- lematic, its very structure.

Page 1095 of 1481


Page 1096 of 1481
Nietzsche Ansers: 2AC (4/6)
SEVENTH, THE ALTERNATIVE ALONE DESTROYS ANY CHECK ON CRUELTY,
LEGITIMIZING ATROCITY

May, College Research Fellow in Philosophy @ Birkbeck College, 99 (Simon, Nietzsches ethic versus morality:
The new ideal, Nietzsches Ethics and his War on Morality, P. 132-133)

An apologist for Nietzsche might suggest that his ethic is not alone in effectively legitimizing inhumanity. He might argue, for example, that some forms of
utilitarianism could not prevent millions being sacrificed if greater numbers could thereby be saved; or that heinous maxims could be consistently universalized
by Kant's Categorical Imperativemaxims against which Kant's injunction to treat all human beings as ends in themselves would afford no reliable protection,
both because its conception of 'humanity' is vague and because it would be overridden by our duty, as rational agents, to respect just such universalized
maxims. To this apologist one would reply that with Nietzsche there is not even an attempt to produce a systematic
safety net against cruelty, especially if one judges oneself to be a 'higher' type of person with life-
enhancing pursuitsand, to this extent, his philosophy licenses the atrocities of a Hitler even though, by his
personal table of values, he excoriates anti-Semitism and virulent nationalism. Indeed, to that extent it is irrelevant whether or not
Nietzsche himself advocates violence and bloodshed or whether he is the gentle person described by his
contemporaries. The reality is that the supreme value he places on individual life-enhancement and self-
legislation leaves room for, and in some cases explicitly justifies, unfettered brutality. In sum: the point here is not to
rebut Nietzsche's claim that 'everything evil, terrible, tyrannical in man' serves his enhancement 'as much as its opposite does' (BGE, 44my emphasis)for
It is rather to suggest that the necessary balance between
such a rebuttal would be a major ethical undertaking in its own right.
danger and safety which Nietzsche himself regards as a condition for flourishing (for example, in this quote from BGE,
44) is not vouchsafed by his extreme individualism. Indeed, such individualism seems not only self-
defeating, but also quite unnecessary: for safeguards against those who have pretensions to sovereignty
but lack nobility could be accepted on Nietzsche's theory of value as just another 'condition for the
preservation' of 'higher' types. Since the overriding aim of his attack on morality is to liberate people from
the repressiveness of the 'herd' instinct, this unrelieved potential danger to the 'higher' individual must
count decisively against the successand the possibility of successof his project.

EIGHTH, THE SURFACE/DEPTH MODELS SEARCH FOR SUBJECTIVE INTEGRITY


RELIES ON ASSUMPTIONS OF METAPHYSICAL INNOCENCE, FETISHIZING AN
AUTHENTICITY THAT NEVER EXISTED

Bewes 97
[Timothy, doctorate in English Literature at the University of Sussex, Cynicism and Postmodernity, New
York City: Verso, 1997, 195-6//uwyo-ajl]

postmodernism has actually become something. Its


Despite the diligence and the sterling efforts of its best theoreti-cians, then, it seems that

principal characteristic is the retreat from and disavowal of the violence of representation - both political and semiotic. There are three further aspects to this
essentially ignominious cultural operation: (i) a cultivation of stupidity (what I have called Kelvinism, or 'metaphysical innocence') as a means of circumventing the ideational

a recourse to the idea of an internal or subjective 'truth of the soul' which transcends
'brutality' of the political life; (ii)

political reality, along with the contingencies of representation. Both of these signal an attachment to a
surface/ depth model of subjectivity which in each case amounts to a fetishization of authenticity, whether by
opting to 'remain' on the surface, or by retreating 'inwards'; (iii) a collapse of faith by individuals and even politicians themselves, not
only in the political infrastructure but in the very' concept of political engagement - here it becomes apparent that Tony Blair, for example, is more 'postodern' than any
theoretician. .

Page 1097 of 1481


these three responses stand in an approximately analogous relationship to the archetypal forms in which consciousness, in a
It should be clear that

state of anxiety, shrinks from the violence of determinate negation and 'strives to hold on to what it is in danger of losing'. 59 At
various points throughout the present work I have used the terms 'decadence', 'irony' and 'relativism' to refer to these instances of an epistemological loss of nerve, this

capitulation to 'things as they are'; it may be as well here to remind ourselves of the terms in which Hegel describes these manifestations of a retreat from
truth. Consciousness, he says, at the decisive moment in which it is required to go beyond its own limits, (i) 'wishes to remain in a state' of unthinking inertia'; (ii) gloats over its
own understanding, 'which knows how to dissolve every thought and always find the same barren Ego instead of any content'; (iii) 'entrenches itself in sentimentality, which
assures us that it finds everything to be good in its kind'. 60 Postmodernism, an empirical social condition - by which I mean that a series of critical-theoretical strategies has
attained a certain concrete form - legitimizes these symptoms of cultural anxiety; postmodernism becomes synonymous, therefore, with deceleration, with a sense of cultural
and political conclusivity; postmodernism is the principal vehicle of what Baudrillard calls 'the illusion of the end'.

Page 1098 of 1481


Page 1099 of 1481
Nietzsche Answers: 2AC (5/6)
NINTH, AUTHENTICITY FETISHIZATION AND ITS FEAR OF REASON FORCES A
RETREAT FROM THE POLITICAL WHILE GAS CHAMBERS ARE BUILT

Bewes 97
[Timothy, doctorate in English Literature at the University of Sussex, Cynicism and Postmodernity, New
York City: Verso, 1997,146-7//uwyo-ajl]

If it is unreasonable to suppose that the Final Solution was potentiated or even necessarily facilitated by
Schmitt's theories, it is certainly the case that this metaphysical structure of domination in the Third
Reich, whereby the status of public citizens is reduced to a level determined entirely in the 'natural' or
biological realm of necessity, is foreshadowed in his 1927 essay. In an abstract and insidious way Schmitt
introduces the idea that the 'transcendent' realm of the political, as a matter of course, will not
accommodate a people with insufficient strength to ensure its own participation, and that such a fact is
ipso facto justification for its exclusion. 'If a people no longer possesses the energy or the will to maintain
itself in the sphere of politics, the latter will not thereby vanish from the world. Only a weak people will
disappear.'130 Schmitt's concept of the 'political', quite simply, is nothing of the sort - is instead weighed
down by necessity, in the form of what Marshall Berman calls German-Christian interiority - by its
preoccupation with authenticity, that is to say, and true political 'identity'. Auschwitz is a corollary not of
reason, understood as risk, but of the fear of reason, which paradoxically is a fear of violence. The stench
of burning bodies is haunted always by the sickly aroma of cheap metaphysics.

TENTH, TURN THE SEARCH FOR HIDDEN MOTIVES ENGAGES IN A HERMENEUTICS


OF SUSPICION, RISKING SPIRAL INTO PROFOUND SKEPTICISM

Berman 2001
[Paul Schiff, Assoc. Prof. Law @ U. of Connecticut, Yale Journal of Law and the Humanities, LN]

Ricoeur contrasts two different "poles" among hermeneutic styles. At one pole, "hermeneutics is understood as the manifestation and restoration of ...
meaning." 23 At the other pole, hermeneutics is "understood as a demystification, as a reduction of illusion." 24 It is not entirely clear to me precisely what
a hermeneutics of faith to be one that treats the object of
Ricoeur means by these two categories. Nevertheless, I understand

study as possessing inherent meaning on its own terms. In contrast, the hermeneutics of
suspicion seeks to expose societal practices as illusory edifices that mask underlying
contradictions or failures of meaning. I will return to the first pole in Part Four of this Essay, but for now I wish to focus on the
hermeneutics of demystification and suspicion.

t each of these
Ricoeur locates in the work of Nietzsche, Marx, and Freud the central hallmarks of this suspicious approach. He argues tha

thinkers makes "the decision to look upon the whole of consciousness primarily as
"false' consciousness." 25 Ricoeur sees this perspective as an extension of Descartes' fundamental position of doubt at the dawn of the
Enlightenment. According to Ricoeur, "The philosopher trained in the school of Descartes knows that things are doubtful, that they are not such as they appear;
but he does not doubt that consciousness is such as it appears to itself; in consciousness, meaning and consciousness of meaning coincide." 26

The hermeneutics of suspicion takes doubt one step farther, by distrusting even our
perceptions.
This suspicious position questions the so-called "correspondence [*104] theory" of truth. As we go through our lives, most of us generally assume that our
mental perceptions accord with reality because we believe we have direct access to reality through our senses or through reason. This is the legacy of the
Enlightenment, the "answer" to the fundamental Cartesian doubt. But the hermeneutics of suspicion maintains that human beings create false truths for
themselves.

Page 1100 of 1481


Such false truths cannot be "objective" because they always serve some interest or
purpose.
By discovering and revealing those interests or purposes, suspicious analysis seeks to expose so-called "false consciousness" generated through social
ideology or self-deception. False consciousness may arise in many different ways. Nietzsche looked to people's self-deceit in the service of the "will to power."
Marx focused on the social being and the false consciousness that arises from ideology and economic alienation. Freud approached the problem of false
consciousness by examining dreams and neurotic symptoms in order to reveal hidden motivations and desires. Thus, "the Genealogy of Morals in Nietzsche's
sense, the theory of ideologies in the Marxist sense, and the theory of ideas and illusions in Freud's sense represent three convergent procedures of
demystification." 27

Page 1101 of 1481


Page 1102 of 1481
Nietzsche Answers: 2AC (6/6)
ELEVENTH, AND, SKEPTICISM STOPS SOCIAL CHANGE THEIR PARANOIA
FORECLSOES UPON REVOLUTION

Berman 2001
[Paul Schiff, Assoc. Prof. Law @ U. of Connecticut, Yale Journal of Law and the Humanities, LN]

, one might view this as a positive development. One might think people should
Of course

stop being lulled into a false sense of believing that the rhetoric of public life really
matters. If people began to view such rhetoric as a construction of entrenched power, so
the argument might go, they would form the nucleus of a truly revolutionary political
movement.

I doubt that such an eventuality is likely to occur. Moreover, I am not sure that a culture of
suspiciousness is the most effective way to seek political (or personal) change anyway. Suspicious
analysis seeks to expose the dangers of our enchantment with reason or truth or collectivity, but there are dangers that arise from relentless disenchantment as
well. As [*123] Richard K. Sherwin has observed,

Without the means of experiencing more profound enchantments , without communal rituals and social
dramas through which the culture's deepest beliefs and values may be brought to life and collectively reenacted, those beliefs ultimately
lose their meaning and die... . Forms of enchantment in the service of deceit, illicit desire, and
self-gratification alone must be separated out from forms of enchantment in the service of

feelings, beliefs, and values that we aspire to affirm in light of the self, social, and legal realities they help to construct and maintain. 112

Page 1103 of 1481


Page 1104 of 1481
Nietzsche = Nihilism
NIETSZCHES DENIAL OF BEING LEADS TO NIHILISM REMOVING ALL MEANING IN
LIFE THIS LEADS TO AN ENDLESS SEARCH FOR POWER WHICH NEVER IS
SUCCESSFUL

Hicks, Prof and Chair of Philosophy @ Queens College of the CUNY, 2K3 (Steven V., Nietzsche, Heidegger, and
Foucault: Nihilism and Beyond, Foucault and Heidegger: Critical Encounters, Ed. Alan Milchman and Alan Rosenberg, P.
109, Questia)

Here again, one might raise objections to Heidegger's equating of Nietzsche's doctrine of will to power with the metaphysics of subjectivity. After all, Nietzsche often attacked Descartes's ego

Nietzsche still follows Descartes's lead in making human


cogito as a logical or linguistic fiction (cf. BGE, 16, 54). Yet according to Heidegger,

beings the subject or foundation of things. Unlike Descartes, however, Nietzsche's subject is not a fixed mental substance, but the body interpreted as a
center of instincts, drives, affects, and sublimations, i.e., as will to power. Heidegger claims that this body as given idea still involves

Nietzsche in a fixity that brings him into the philosophy of presence: Nietzsche argues that being is as fixated, as
permanent (N, 2:200). And this forced sense of presence, Heidegger thinks, leads to the dangers of radical
objectifiability and to the disposability of beings, i.e., treating beings as nothing but objects of use,
control, and management. 32 Moreover, like its Cartesian counterpart, the Nietzschean subject reins supreme over the whole of beings and posits the measure for the
beingness of every being (N, 4:121). 33 In claiming that truths are illusions and that Being is an empty fiction, Nietzsche fashions for the subject an absolute power to enjoin what is
true and what is false and hence to define what it means to be or not to be a being (N, 4:145). According to Nietzsche, what is truewhat has beingis that which serves the interest of

Being is thus reduced to the status of a value or


the subject whose essence is will to power (in the mode of existence of eternal recurrence; cf. N, 2:203).

a condition of the preservation and enhancement of the will to power (N, 4:176). This is why Heidegger
considers Nietzsche the consummation, and not the overcoming, of Western metaphysics: by reducing
Being to a value, the doctrine of will to power makes the nihilism of the metaphysical tradition (the assumption
that Being itself is nothing and the human will everything) a matter of philosophical principle. 34 Thus Nietzsche's counter-
ideals of will to power and eternal recurrence, far from overcoming nihilism, actually express or
exemplify the loss of any sense of Being, or the withdrawal of Being itself, in favor of beings (i.e., products of
human will). As Heidegger reads him, Nietzsche understands Being in terms of value (or what is useful for enhancing the human will) because
Being itself has totally withdrawn in default. And this brings to completion traditional metaphysics, which, according to Heidegger, is the history of Being in its withdrawal. As Heidegger sees

Nietzsche's metaphysics of will to power is the most extreme withdrawal of Being and thus the
it,

fulfillment of nihilism proper (N, 4:204, 232). So Nietzsche brings to completion, in his denial of Being, the
very nihilism he wanted to overcome. Far from twisting free of the ascetic ideal, Heidegger claims, Nietzsche 's doctrine of will
to power actually provides the basis for its most complete expression in the modern secularized ascetic
will-tocontrol everything. In other words, instead of seeking salvation in a transcendent world by means of
ascetic self-denialthe aspect of metaphysics that Nietzsche most obviously rejectssalvation is now,
Heidegger claims, sought exclusively in the free self-development of all the creative powers of man (N, 4:89). This
unlimited expanding of power for power's sake parallels in many ways what Nietzsche characterized as
the most terrifying aspect of the ascetic ideal: the pursuit of truth for truth's sake. It is, according to Heidegger, the hidden
thorn in the side of modern humanity (cf. N, 4:99). This hidden thorn expresses itself variously in the Protestant work ethic and in the iron cage of bureaucratic-technological rationality
(discussed in the works of Max Weber); it also expresses itself in the various power aims of modern scientific/technological culture as well as in the frenzied impulse to produce and consume

Nietzsche's own figure of the Overman (Ubermensch) foreshadows the


things at ever faster rates. Heidegger even suggests that

calculating, technological attitude of modern secularized asceticism: His Overman [stands] for the
technological worker-soldier who would disclose all entities as standingreserve necessary for enhancing
the ultimately aimless quest for power for its own sake.35 This emerging technological human, grounded
in a control-oriented anthropocentrism, compels entities to reveal only those one-dimensional aspects of
themselves that are consistent with the power aims of a technological/productionist culture. Instead of dwelling and
thinking in a world unified by what Heidegger metaphorically terms the fourfold of earth and sky, gods and mortals, impoverished modern technocrats occupy a world bereft of gods in
which thinking becomes calculating, and dwelling becomes tantamount to the technological domination of nature and what Nietzsche calls the common economic management of the
earth in which mankind will be able to find its best meaning as a machine in the service of this economy (WP, 866). Thus citizens come to be viewed primarily as consumers, wilderness is
looked upon in terms of wildlife management areas, and genuine human freedom is replaced by the organized global conquest of the earth, and the thrust into outer space (N, 4:248). As
Heidegger sees it, our era entertains the illusion that man, having become free for his humanity, has freely taken the universe into his power and disposition (N, 4:248). In summary,
Nietzsche tried to combat the nihilism of the ascetic ideal (e.g., the collapse of the Christian table of values) by bringing forth new nonascetic values that would enhance rather than devalue

humanity's will to power. According to Heidegger, however, instead of overcoming nihilism, Nietzsche simply reinforced it. By
characterizing Being as an empty fiction and the last smoke of a vaporized reality (TI, 2:2, 481), and by degrading it to
the status of a value for enhancing the subject's will to power, Nietzsche loses any sense of Being as such. For him it is a mere nothing, a nihil. And this

Page 1105 of 1481


brings to completion the fundamental movement of history in the West, which is nihilism: the
withdrawal of Being itself and the consequent focus on beings as objects for consolidating the power of
Will and for expanding it out beyond itself in an ever-increasing spiral. 36 As Heidegger sees it, this eternally
recurring will to power, or will to will, is a will-to-control that only reinforces the nihilism Nietzsche
feared: the loss of meaning or direction, the devaluation of the highest values, the constructs of
domination, and the devotion to frenzied consumption and production.

Page 1106 of 1481


Page 1107 of 1481
Nietzsche Legitimizes Genocide
(1/2)
NIETZSCHES PHILOSOPHIES LEGITIMIZED NAZISM AND THE HOLOCAUST

Ortega-Cowan, B.A. @ Boston College and J.D. with Honors @ Florida State U College of Law,
2K3 (Roman, Dubious Means to Final Solutions: Extracting Light from the Darkness of Ein F Hrer and
Brother Number One, 31 Fla. St. U.L. Rev. 163, Fall)

Nietzsche's emphasis on the triumph of the will over emotion gave the Nazis the mental strength to
Finally,
accomplish the horrors of the Holocaust. n35 The choice of self-definition through hardness was seen as
central to the establishment and assertion of a new national identity, and such emphasis led to a devaluation
of human compassion and other emotions. n36 With a set ideology of hatred founded upon angry anti-
Semitism, a belief in "scientific" racial superiority, and a will immune from emotional influence, the Nazis
embarked on a catastrophic mission targeting a clearly defined enemy. After taking control of the government, they
quickly built a wall of legal repression around the Jews, which culminated in the Nuremberg Laws and
Kristallnacht decrees and left the Jews vulnerable to the violence that lay ahead. n37

NIETZSCHES PHILOSOPHIES LEGITIMIZED THE HOLOCAUST HIS NOTIONS OF


MASTER MORALITY FUELED THE FIRE BEHIND GENOCIDES OF THE WEAK AND
IMPERFECT FRAMING THEM AS MANS GREATEST DANGER

Aschheim, Prof of German Cultural and Intellectual History @ Hebrew U, Jersulem, 97 (Steven E.,
Nietzsche, Anti-Semitism and the Holocaust, Nietzsche and Jewish Culture, Ed. Jacob Golomb, P. 13-16)

At any rate, what I am proposing here is that both in its overall bio-eugenic political and medical vision, its programmatic obsession with degeneration and
regeneration, whether in parodistic form or not, there are clear informing parallels with key Nietzschean categories and goals. From one perspective, as Robert
Jay Lifton has recently persuasively argued, Nazism
is about the "medicalisation of killing". Its genocidal impulses were
implicit within a bio-medical vision and its vast, self-proclaimed programmatic task of racial and eugenic-
hygiene. On an unprecedented scale it would assume control of the human biological future, assuring
health to positive racial stock and purging humanity of its sick, degenerative elements. Its vision of
"violent cure", of murder and genocide as a "therapeutic imperative", Lifton argues, resonates with such
Nietzschean themes.40 While every generation may emphasize their particular Nietzsche, there can be little
doubt that in the first half of this century various European political circles came to regard him as the
deepest diagnostician of sickness and degeneration and its most thoroughgoing regenerative therapist.
"The sick", he wrote, "are man's greatest danger; not the evil, not the 'beasts of prey' ."41 To be sure, as was his
wont, he employed these notions in multiple, shifting ways, as metaphor and irony (he even has a section on "ennoblement through degeneration"42) but
most often, most crucially, it was represented (and understood) as a substantial literal danger whose overcoming through drastic measures was the
Although he was not alone in the wider
precondition for the urgent re-creation of a "naturalized", non-decadent humankind.
nineteenth-century quasi-bio-medical, moral, discourse of "degeneration"43 - that highly flexible,
politically adjustable tool that cut across the ideological spectrum, able simultaneously to locate, diagnose
and resolve a prevalent, though inchoate, sense of social and cultural crisis through an exercise of eugenic
labeling and a language of bio-social pathology and potential renewal44 - he formed an integral part in

Page 1108 of 1481


defining and radicalizing it. He certainly constituted its most important conduit into the emerging radical right. What else was Nietzsche's
Lebensphilosophie, his reassertion of instinct and his proposed transvaluation whereby the healthy naturalistic
ethic replaced the sickly moral one (a central theme conveniently ignored or elided by the current post-structuralist champions of Nietzsche).
"Tell me, my brothers", Zarathustra asks, "what do we consider bad and worst of all? Is it not degeneration}'"15 In this world, the reassertion of all that is
natural and healthy is dependent upon the ruthless extirpation of those anti-natural ressentiment sources of degeneration who have thoroughly weakened
and falsified the natural and aristocratic bases of life. Over and over again, and in different ways, Nietzsche declared that "The species requires that the ill-
constituted, weak, degenerate, perish".46 The
Nazi bio-political understanding of, and solution to "degeneration", as I
have tried to show here and elsewhere, was in multilayered ways explicitly Nietzsche-inspired. From the
World War I through its Nazi implementation, Nietzschean exhortations to prevent procreation of "anti-
life" elements and his advocacy of euthanasia, of what he called "holy cruelty" - "The Biblical

contiued

Page 1109 of 1481


Page 1110 of 1481
Nietzsche Legitimizes Genocide
(2/2)
continued

prohibition 'thou shalt not kill'", he noted in The Will to Power, "is a piece of naivete compared with the seriousness of the prohibition of life to decadents:
'thou shalt not procreate!'. . . Sympathy for decadents, equal rights for the ill-constituted - that would be the profoundest immorality, that would be antinature
itself as morality!"47 - both inspired and provided a "higher" rationale for theorists and practitioners off such measures.48 The
translation of
traditional anti-Jewish impulses into genocide and the murderous policies adopted in different degrees to
other labeled outsiders (Gypsies, physically and mentally handicapped, homosexuals, criminals, inferior Eastern peoples and Communist political
enemies) occurred within the distinct context of this medico-bio-eugenic vision. There were, to be sure,
many building-blocks that went into conceiving and implementing genocide and mass murder but I would
argue that this Nietzschean framework of thinking provided a crucial conceptual precondition and his
radical sensibility a partial trigger for its implementation. Related to but also going beyond these programmatic parallels and links
we must raise another highly speculative, though necessary, issue: the vexed question of enabling preconditions and psychological motivations. Clearly, for
events as thick and complex as these no single theoretical or methodological approach or methodology will suffice. Yet, given the extraordinary nature of the
events, more conventional modes of historical analysis soon reach their limits and demand novel answers (the study of Nazism has provided them in
abundance, some more, some less convincing49). I am not thus claiming exclusiveness for the Nietzschean element at this level of explanation, but rather
arguing for his continued and important relevance. To be sure, of late, many accounts of the ideas behind, and the psychological wellsprings enabling, mass
murder have been, if anything, anti-Nietzschean in content. For Christopher Browning it was hardly Nietzschean intoxication, the nihilistic belief that "all is
permitted", that motivated the "ordinary killers" - but rather prosaic inuring psychological mechanisms such as group conformity, deference to authority, the
dulling powers of alcohol and simple (but powerful) processes of routinization.50 For George L. Mosse, far from indicating a dynamic anti-bourgeois
Nietzschean revolt, the mass murders represented a defense of bourgeois morality, the attempt to preserve a clean, orderly middle-class world against all
those outsider and deviant groups that threatened it.51 These contain important insights but, in my view, leave out crucial experiential ingredients, closely
related to the Nietzschean dimension, which must form at least part of the picture. At some point or another, the realization must have dawned on the
conceivers and perpetrators of this event that something quite extraordinary, unprecedented, was occurring and that ordinary and middle-class men were
committing radically transgressive, taboo-breaking, quite "un-bourgeois" acts.52 Even if we grant the problematic proposition that such acts were done in
order to defend bourgeois interests and values, we would want to know about the galvanizing, radicalizing trigger that allowed decision-makers and
perpetrators alike to set out in this direction and do the deed. To argue that it was "racism" merely pushes the argument a step backward, for "racism" on its
own -while always pernicious - has to be made genocidal. We are left with the issue of the radicalizing, triggering forces.
These may be many in number but it seems to me that Nietzsche's determined anti-humanism (an atheism
that, as George Lichtheim has noted, differs from the Feuerbachian attempt to replace theism with humanism33), apocalyptic imaginings and
exhortatory visions, rendered such a possibility, such an act, conceivable in the first place (or, at the very
least, once thought of and given the correct selective readings easily able to provide the appropriate
ideological cover). This Nietzschean kind of thought, vocabulary and sensibility constitutes an important
(if not the only) long-term enabling precondition of such radical elements in Nazism. With all its affinities
to an older conservatism, it was the radically experimental, morality-challenging, tradition-shattering
Nietzschean sensibility that made the vast transformative scale of the Nazi project thinkable. Nietzsche,
as one contemporary commentator has pointed out, "prepared a consciousness that excluded nothing
that anyone might think, feel, or do, including unimaginable atrocities carried out on a gigantic order".54 Of
course, Nazism was a manifold historical phenomenon and its revolutionary thrust sat side by side with petit-bourgeois, provincial, traditional and conservative
beyond its doctrinal emphases on destruction and violent regeneration, health and
impulses.55 But surely,
disease, the moral and historical significance of Nazism lies precisely in its unprecedented transvaluations
and boundary-breaking extremities, its transgressive acts and shattering of previously intact taboos. It is
here - however parodistic, selectively mediated or debased - that the sense of Nazism, its informing project and
experiential dynamic, as a kind of Nietzschean Great Politics continues to haunt us.

Page 1111 of 1481


Page 1112 of 1481
Nietzsche Legitimizes Patriarchy
NIETZSCHES CALLS FOR DISTINCTIONS BETWEEN THE WEAK AND THE STRONG
ARE NOTIONS OF CLASSISM AND SERVE TO REINFORCE PATRIARCHAL DOMINANCE

Schutte, Assist Prof of Philosophy @ U of Florida, 84 (Ofelia, Nietzsches Politics, Beyond Nihilism: Nietzsche
without Masks, P. 186-188)

As long as one gives philosophical credibility to the rhetoric of the "superior type" or "higher" person and sets aside the political and practical implications of
how this rhetoric is instantiated, both by Nietzsche and by the historical patriarchal tradition in which we still live, we are defending what I shall call "the
While the logic of unspecified prejudice calls for the higher/lower distinction
politics of unspecified prejudice."
without committing itself to any particulars to fill those categories, Nietzsche has made it quite clear what
groups by "nature" or "destiny" are higher and what lower. Here are two statements regarding women and
workers, two groups Nietzsche has condemned to the "low." Reversing Goethe's statement that "the eternal feminine draws us
higher," the author of Beyond Good and Evil wrote: "I do not doubt that every nobler woman will resist this faith, for she believes the same about the Eternal-
Masculine."62 The criterion of a woman's "nobility," then, is her "faith" that the male, as male, is more noble than herself.
This insidious rhetoric is
also applied to the slave, who is urged to believe that his exploitation is justified because the
master/aristocrat is more noble than he. When one unmasks the realities of this rhetoric, one sees that
the practical advantages do not go to "superior" personseven assuming there were so pure a typebut
simply to the privileged classes of the established society. Nietzsche himself points this out in Twilight of the Idols: The labor
question. The stupidityat bottom, the degeneration of instinct, which is today the cause of all stupiditiesis that there is a labor question at all. Certain
things one does not question: that is the first imperative of instinct. . . . But what was done? . . . The instincts by virtue of which the worker becomes possible
as a class, possible in his own eyes, have been destroyed through and through with the most irresponsible thoughtlessness. The worker was qualified for
military service, granted the right to organize and to vote: is it any wonder that the worker today experiences his own existence as distressingmorally
speaking, as an injustice? But what is wanted? ... If one wants an end, one must also want the means: if one wants slaves, one is a fool if one educates them to
be masters.63 The theme of the "strength" of not questioning the structure of power that serves the interests
of a privileged class is not simply anti-liberal to the point of malice (as Nietzsche suggests in the aphorism that precedes this
one). It is anti-critical to the point of malice. These statements on women, the working class, and the need of the
privileged class for thoughtless and obedient "slaves" are not simply isolated opinions on Nietzsche's part,
as sometimes they tend to be read. They are logically tied to other notions that Nietzsche is commended
for holdingsuch as the distinction between the "superior" person and the "herd," the belief in a "strong"
culture, and even the love of one's fate. The fact that we ignore the concrete side of the issue while holding on to the more abstract side
shows that in this case we are much less logical than Nietzsche, for we are the ones caught in a logical dilemma, while Nietzsche is not. Nietzsche, however, is
caught in a much larger type of contradiction even though his logic is tight with respect to the connection between elitism and oppression. This is the
contradiction between his intended affirmation of life and his reactionary and nihilistic politics. Still, the political implications of Nietzsche's thought can be
turned around to some extent if we ask: was not Nietzsche correct in insisting upon a logical connection between a "strong" masculine ideal, a "strong" culture,
and a blind system of political exploitation and psychological repression? Is it not true that if the goal of one's values is to implement a "strong" patriarchal
system where a few will command and the rest will obey, it is then foolish to allow moral codes which favor the notions of the universal brotherhood and
sisterhood of human beings? Does not the morality of universal human dignity entail in theory, if not also in practice, the elimination of all forms of elitism,
thanks to his
domination, and oppression? In Nietzsche's idea of "greatness" one finds the logic of the extremeof this he was well aware. But
uninhibited articulation of the extreme he has exposed the logic of patriarchal domination in its essence.
While Nietzsche has outlined various incentives for overturning the democratic influences of modern times and for instituting a "purer" system of patriarchal
it is up to us, not him, to make the choice as
domination under the banner of overcoming the "evils" of "effeminacy" and "decadence,"
to what we want our political future and our moral values to be. His appeals to destiny, intolerance, and
the suspension of critical questioning of authoritarian political institutions are not convincing.

Page 1113 of 1481


Page 1114 of 1481
Alternative Causes Annihilation
THE ALTERNATIVES DESTRUCTION OF ONTOLOGY CAUSES VIOLENCE AND
ANNIHILATION

Hicks, Prof and Chair of Philosophy @ Queens College of the CUNY, 2K3 (Steven V., Nietzsche, Heidegger, and
Foucault: Nihilism and Beyond, Foucault and Heidegger: Critical Encounters, Ed. Alan Milchman and Alan Rosenberg, P.
109, Questia)

This response, however, only succeeds in postponing Heidegger's real objection. For according to Heidegger, psychology (and indeed, all of the human
sciences) are caught up in the web of traditional metaphysical thinking. As such, Nietzsche's 'psychology' is simply coterminous with metaphysics . [it] lies
grounded in the very essence of modern metaphysics (N, 4:2, 8). Heidegger argues that modern metaphysics is defined
precisely by the fact that man becomes the measure and center of beings, and this , in turn, results in the
modern technological understanding of beings as objects for use and control, or as Heidegger says, entities
wholly present as standing-reserve (Bestand) (QT, 17). 26 This extends even to human beings themselves, who
are increasingly transformed by the human sciences (and their technological systems) into resources for
objectification and control (cf. N, 4:23445). Here, Heidegger anticipates Foucault's claim that modern technological systems
attempt to make human beings wholly present as bio-power, or subjects completely present for
surveillance and control via the disciplinary practices of institutions (psychological, juridical, carceral) whose aim is to
normalize human life. 27 Thus from Heidegger's perspective, the actual nihilism Nietzsche feared annihilation,
spreading violence, and so forthis evoked by the preponderance, in the modern world, of this
productionist, technological objectification of being, and by the complete ordering of all beings in the
sense of a systematic securing of stockpiles for further technological usage, control, and domination (N,
4:22934). The relentlessness of [this] usage extends so far that the abode of Beingthat is, the essence of manis omitted; man is threatened
with the annihilation of his essence, and Being itself is endangered (N, 4:245). Ironically, Heidegger argues, it was
precisely Nietzsche's proposing of Being as a value posited by the will to power that led to this final
[nihilistic] step of modern metaphysics, in which Being comes to appear as will to power (N, 4:234). Simply put,
Nietzsche's doctrine of the will to power succeeds in reducing the whole question of Being to the status of
a value; and this completes the metaphysics of subjectivity initiated by Descartes, which in turn results in a blindness to the
whole question as to what Being itself is. This blindness to Being, Heidegger argues, is at the root of all
nihilism and is connected to the modern technological/productionist attitude toward the world (cf. N, 4:231
32). Why does Heidegger make this claim? Heidegger believes that metaphysics is essentially the history of Being, a history in which Being discloses itself as
withdrawn in default or concealed (cf. N, 4:23032). He basically reads the whole history of Western philosophy as the history of Being and its gradual self-
concealment. In this context, Heidegger praises Nietzsche for his insight into the basic development of that history: In his [Nietzsche's] view it is nihilism .
The phrase 'God Is Dead' is not an atheistic proclamation; it is a formula for the fundamental experience of an event in Occidental history (N, 1:156).
Heidegger even suggests that Nietzsche came close to recognizing (albeit opaquely) that the fundamental
question of Being had been omitted, forgotten, or suppressed within the metaphysical tradition of
previous philosophy, and that this omission of the default of Being in its unconcealment is the very
essence of nihilism (cf. N, 4:23032). For example, when Nietzsche denies truth or refers to Being as an empty
fiction (see TI, 481), Heidegger claims that he is actually experiencing and expressing the nothing or
omission of Being itself in the history of Western philosophy, which is tantamount to nihilism:

Page 1115 of 1481


Page 1116 of 1481
Nihilism Fails
NIHILISM IS AN INEFFECTIVE MEANS OF RESISTANCE THAT REPLICATES
EVERYTHING BAD ABOUT THE STATUS QUO

Mann, Prof of English @ Pomona, 95 (Paul, Stupid Undergrounds, PostModern Culture 5:3, Project Muse)

One might find it amusing to assume the pose of someone who states problems with brutal simplicity. As in this little nugget: Every historical form of cultural
and political revolt, transgression, opposition, and escape has turned out to be nothing more than a systemic function. The notion of recuperation has
encountered a thousand alibis and counter-tropes but still constitutes the closest thing cultural study has to a natural law. Collage, antimelodic high-decibel
music, antimasterpieces, romantic primitivism, drunkenness and drugs, renegade sexuality, criticism itself: it is amazing that a single radical claim can still be
Every conceivable form of negation has been dialectically
made for any of this, and entirely characteristic that it is.
coordinated into the mechanism of progress. The future of the anti has not yet been reconceived. That is why
it is ridiculous to accuse some poor kid with a bad attitude or some putative grownup with a critique but no "positive program for change" of being nihilistic:
strictly speaking, nihilism doesn't exist. What was once called nihilism has long since revealed itself as a
general, integral function of a culture that, in all its glorious positivism, is far more destructive than the
most vehement no. Nothing could be more destructive, more cancerous, than the positive proliferation of
civilization (now there's a critical clich), and all the forms of opposition have long since revealed themselves as
means of advancing it. As for the ethos of "resistance": just because something feels like resistance and
still manages to offend a few people (usually not even the right people) hardly makes it effective. It is merely
ressentiment in one or another ideological drag. And how can anyone still be deluded by youth, by its tedious shrugs of revolt? Even
the young no longer believe their myth, although they are quite willing to promote it when convenient. Punk nihilism was never more than the nihilism of the
commodity itself. You should not credit Malcolm McLaren with having realized this just because he was once pro-situ. All he wanted was to sell more trousers
without boring himself to death; indeed he is proof that the guy with the flashiest ressentiment sells the most rags. And if he wasn't bored, can he be said to
have advanced the same favor to us?

NIHILISM ENTRENCHES IDEAS THAT PROMOTE ARROGANCE AND VICTIMIZATION

Dyson, Assist Prof of Law @ SMU, 05 (Maurice R., Awakening an Empire of Liberty: Exploring the Root of Socratic
Inquiry and Political Nihilism in American Democracy, Democracy Matters: Winning the Fight Against Imperialism, Vol.
83 No. 2, Wash Univ Law Quarterly)

for West, these three entrenched dogmas are in turn driven by three forms of "political
Furthermore,
nihilism." These are evangelical nihilism, paternalistic nihilism, and sentimental nihilism. "Evangelical
nihilism" is a notion of arrogant superiority that justifies might as right, or in other words, the belief that the U.S.
would not be so powerful if we were not right. West terms it "evangelical" because of its perceived militant intolerance for dissension as
well as blind faith to the belief that the exercise of power is a predicate to ensuring security and prosperity. For West, the quintessential evangelical nihilist is
Paternal nihilism, on the
derived from Plato's Republic in the form of Thrasymachus who debates with Socrates the moral superiority of might.(FN1)
treats American citizens as victims of deception by government actors who in turn attempt to
other hand,
superficially appease the masses. These governmental leaders fundamentally accept corrupt regimes and
policies rather than question them. He finds in Fyodor Dostoyevsky's The Brothers Karamazov the literary metaphor for paternal nihilism in
the form of the Grand Inquisitor. As West points out, this character knows full well the atrocities of the Inquisition represent a gross distortion of the Christian
gospel, but nonetheless, personally takes part in condemning infidels to death sentences because he believes the corrupted church is the best that mankind
The political nihilist is faulted here not just for his failure of imagination to envison a truer
can hope for.(FN2)
democracy, but for his lack of conviction to battle corrupt elites even when history has shown these
battles can be vigorously waged.(FN3) Sentimental nihilism refers to West's belief that the news media's
oversimplification and sensationalized reporting of global events sacrifices truth for distraction.
Sentimental nihilism pacifies the American people by blunting the critical aspects of news events that
implicate corruption in government.

Page 1117 of 1481


Page 1118 of 1481
Page 1119 of 1481
Nihilism Causes Terrorism (1/2)
NIHILISM IS THE ROOT OF ALL TERRORISM IT PERVERTS THE NOTION OF
NATIONALISM AND PRIDE AND LEGITIMIZES UNENDING CYCLES OF VIOLENCE

Ignatieff, Carr Prof of Human Rights @ Harvards Kenndy School of Government, 2K4 (Michael, The
Temptations of Nihilism, an extract from The Lesser Evil: Political Ethics in an Age of Terror, New England
Review Vol. 25 No. 1/2, P. 54-74)

Neither side in a war on terror is immune from this temptation of coming to see violence as an end in
itself. Agents of a democratic state may find themselves driven by the horror of terror to torture, to
assassinate, to kill innocent civilians, all in the name of rights and democracy. Succumbing to this inversion is the principal
way that both groups slip from the lesser evil to the greater. If, however, this temptation is strong, a strategy of combating it with lesser evils may not be plausible at all. A lesser
evil morality may be too rational. It makes the assumption that violence by a liberal democratic state faced with terror can be controlled in the name of ethically appropriate ends
like rights and dignity. A lesser evil approach to a war on terror would assume, for example, that agents of a liberal democratic state should be able to hold the line that divides
intensive interrogation from torture, or the line that separates targeted assassination of enemy combatants from assassinations that entail the death of innocent civilians.
Current U.S. policy does not allow assassination of civilians in peacetime but does permit killing of enemy combatants in wartime, with the proviso that such assassinations
must be discriminate and avoid collateral damage. This policy--a lesser evil approach if there ever was one--implies that the agents charged with defending a state have the
strength of character, together with a clear enough sense of the values of the society they are defending, to be trusted with morally ambiguous means. But a perfectionist case
against such an approach would argue that morally equivocal means are hard to control and thus liable to end in betrayal of the values that a liberal democracy should stand
for. Hence liberal states should not allow those who defend them to have any of the moral discretion implied in lesser evil approaches. States should absolutely ban extreme
interrogations, targeted assassinations, and other uses of violence, because once you start with means like these, it becomes next to impossible to prevent the lesser from
shading into the greater evil. Another problem with the lesser evil would be that liberal democratic regimes encourage a kind of moral narcissism, a blinding belief that because
this kind of society authorizes such means, they must be acceptable. Thus democratic values, instead of preventing the lesser from shading into the greater evil, may actually
blind democratic agents to the moral reality of their actions. The nobility of ends is no guarantee against resort to evil means; indeed, the more noble they are, the more
ruthlessness they can endorse. This is why democracy depends on distrust, why freedom's defense requires submitting even noble intentions to the test of adversarial review. I

can see three distinct ways--the tragic, the cynical, and the fanatical--in which nihilism can come to dominate
both a terrorist campaign and a war on terror. The first might be called tragic because it occurs despite
the political intentions of all concerned, when terrorists and counterterrorists become trapped in a
downward spiral of reprisal and counterreprisal. One side kills to avenge its last victim; the other side
replies to avenge its last victim. Both sides start with an ethic of restraint and end up in a struggle without
end. Here shedding of blood creates two communities--the terrorists and the counterterrorists--in which loyalty
to the group prevails over institutional accountability or individual principle. Both sides are bonded to their own because both have blood

on their hands or blood to avenge. Their bonds to the group are stronger than any they have to the institutions that could possibly restrain their behavior.
Violence creates belonging and belonging produces closure. Terrorists listen only to themselves and no
longer to restraining messages from the communities their violence is supposed to serve. Counterterrorist
agencies, having suffered losses, bond with each other, view their civilian superiors as spineless libertarians, chafe
under operational restrictions on their use of force, seek to evade these wherever possible, covering up as
they do so, and seek to fight the terrorists on their own terms. At the bottom of this downward spiral, constitutional
police forces and counterterror units can end up behaving no better than the terrorist cells they are trying
to extirpate. Their moral conduct becomes dependent on the increasingly repellent conduct of the other
side. This is the unintentional path to nihilism, taken by constitutional forces to defend the fallen and to
revenge their losses. In the process, torture and extrajudicial killing may become routine. Gillo Pontecorvo's masterful film The
Battle of Algiers (I965) portrays the Algerian war for independence, between i955 and 196Z, as a tragic duel in which two sides, conscientiously believing in the rightness of
their course, become trapped in just such a downward spiral as we have been considering. The film may be fictional, but it is drawn from extensive documentary research into
the actual history of the Algerian struggle. While clearly siding with the Algerian revolution, Pontecorvo takes care to avoid any moral caricature of the French, and shows why
torture could be seen as a rational and effective way to break up the terrorist cells working in the Algiers Casbah. Nor does the filmmaker conceal the bloody reality of the
liberation struggle, showing the full horror of an attack on a caf that leaves the street strewn with mangled bodies and traumatized survivors. The film maintains an
extraordinarily subtle moral balance, supporting the Algerian struggle for freedom without mitigating the crimes committed in its name, condemning the French use of torture
without failing to do justice to the reality that it was committed not by brutes but by people with dedicated convictions. The Battle of Algiers thus becomes a testament to the
In the tragic path,
tragedy of terrorist war. Calling this path tragic is not to excuse it, merely to distinguish it from a second path, which is altogether more cynical.

violence, once used as a means, becomes an end in itself, to the horror of those who are trapped by the
conduct of the other side. In the second path, violence doesn't begin as a means to noble ends. It is used, from the
beginning, in the service of cynical or self-serving ones. On both the terrorist and counterterrorist sides,
there are bound to be individuals who actually enjoy violence for its own sake. Violence and weapons
exert a fascination all their own, and their possession and use satisfy deep psychological needs. It isn't necessary
to delve into the question of why human beings love violence and seek to use weapons as instruments of power and even of sexual gratification. The fact that violence attracts
as well as repels is a recurring challenge to the ethics of a lesser evil, since it explains why the appetite for violence can become insatiable, seeking ever more spectacular
effects even though these fail to produce any discernible political result. Many terrorist groups use political language to mask the absence of any genuine commitment to the
cause they defend. In their cynicism, they can become uncontrollable, because once violence is severed from the pursuit of determinate political ends, violence will not cease
even if these goals are achieved. What is true of terrorists can also characterize counterterrorists. The type of personnel attracted to police and antiterrorist squads may be
recruited because they are drawn to violent means. These means confer power, boost sexual confidence, and enable them to swagger and intimidate others. The type of
personality attracted into a counterterror campaign may not have any intrinsic or reflective commitment to democratic values of restraint. Rules of engagement for the use of
deadly force need be obeyed only when superiors are watching and can be disregarded at any other time. There may always be a gap, therefore, between the values of a
liberal democracy when it is under attack and

Page 1120 of 1481


continued

Page 1121 of 1481


Page 1122 of 1481
Nihilism Causes Terrorism (2/2)
continued

the conduct of the counterterrorist forces who have to take the war to the enemy. There is no necessary reason to suppose that those who defend a
democracy do so out of any convinced belief in its values. Their chief motivation may be only the thrill of the chase and the glamour of licensed violence.
Liberal states cannot be protected by herbivores. But if we need carnivores to defend us, keeping them in check, keeping them aware of what it is they are
defending, is a recurrent challenge. On the terrorist side, there will always be a gap between those who take the political goals of a terrorist campaign seriously
and those who are drawn to the cause because it offers glamour, violence, money, and power. It is anyone's guess how many actual believers in the dream of a
united Ireland there are in the ranks of the IRA. But it is a fair bet to suppose that many recruits join up because they want to benefit from the IRA's profitable
protection rackets. The IRA bears as much relation to the Mafia as it does to an insurrectionary cell or a radical political party, and the motivations that draw
young people into the movement are often as criminal as they are political. When criminal goals predominate over political ones, it becomes difficult for
leaders to prevent their followers from turning violence into an end in itself. The criminal allure of terrorist groups and the cynicism of those who join them are
additional reasons why it is a mistake to conciliate or appease a group like the IRA with political concessions. Their political goals may be subsidiary to their
criminal interests, and like any criminal enterprise they can be driven out of business only by the force of the law. Equally, to express surprise that they tarnish
political ideals with squalid tactics, or that they seem to be indifferent to the costs that their violence imposes on the communities they purport to represent,
would be to misunderstand their real nature and purpose. Not all terrorists, however, are moral cynics. Not all terrorist groups use politics as an excuse for
There are other groups whose political purposes are genuine, but who
other straightforwardly violent ends.
nonetheless end up turning violence into a way of life. These are the groups that have the characteristics,
not of criminal gangs, but of fanatic sects. Here nihilism takes the form, not of believing in nothing, but of
believing in too much. What I mean is a form of conviction so intense, a devotion so blind, that it becomes
impossible to see that violence necessarily betrays the ends that conviction seeks to achieve. Here the delusion
is not tragic, as in the first case, because believers are not trapped into violence by the conduct of the other side. Nor is it cynical: for these are true believers.
They initiate violence as a sacred and redemptive duty. This is the third path to nihilism, the fanatical use
of high principle to justify atrocity. What is nihilistic is the belief that such goals license all possible means,
indeed obviate any consideration of the human costs. Nihilism here is willed indifference to the human
agents sacrificed on the altar of principle. Here nihilism is not a belief in nothing at all; it is, rather, the
belief that nothing about particular groups of human beings matters enough to require minimizing harm
to them. The high principles commonly used to justify terrorism were once predominantly secular--varieties of conspiratorial Marxism--but today most of
the justifying ideologies are religious. To call religious justifications of violence nihilistic is, of course, to make a certain kind of value judgment, to assert that
there cannot be, in principle, any metaphysical or God-commanded justification for the slaughter of civilians. From a human rights standpoint, the claim that
such inhumanity can be divinely inspired is a piece of nihilism, an inhuman devaluation of the respect owed to all persons, and moreover a piece of hubris,
since, by definition, human beings have no access to divine intentions, whatever they may be. The hubris is not confined to vocalizing divine intention. It also
consists in hijacking scriptural tradition. The devil can always quote scripture to his use, and there is never a shortage in any faith of texts justifying the use of
force. Equally, all religions contain sacred texts urging believers to treat human beings decently. Some may be more universalistic in these claims than others.
Some may confine the duties of benevolence to fellow believers, while others may extend these duties to the whole of humankind. But whatever the ambit of
their moral concern, all religious teaching offers some resistance to the idea that it is justifiable to kill or abuse other human beings. This resistance may range
nihilist use of religious doctrine is one that perverts the
from outright condemnation to qualified justification as a last resort. A
doctrine into a justification for inhuman deeds and ignores any part of the doctrine which is resistant to
its violent purposes. The nihilism here engages in a characteristic inversion: adjusting religious doctrine to
rationalize the terrorist goal, rather than subjecting it to the genuine interrogation of true faith. It is
unnecessary here to document the extent to which Al Qaeda has exploited and distorted the true faith of Islam. To take but one example, the tradition of jihad,
which refers to the obligation of the believer to struggle against inner weakness and corruption, has been distorted into an obligation to wage war against Jews
and Americans. In the hands of Osama bin Laden, the specifically religious and inner-directed content of jihad has been emptied out and replaced by a doctrine
justifying acts of terror. This type of religious justification dramatically amplifies the political impact of terrorist actions. When Al Qaeda strikes, it can claim that
it acts on behalf of a billion Muslims. This may be a lie, but it is an influential one nonetheless. Appropriating religious doctrine in this way also enables the
group to offer potential recruits the promise of martyrdom. Immortality complicates the relationship between violent means and political ends, for the
promise of eternal life has the effect of making it a secondary matter to the suicide bomber whether or not the act achieves anything political at all. What
Once violent
matters most is securing entry into Paradise. Here political violence becomes subservient not to a political end but to a personal one.
means cease to serve determinate political ends, they take on a life of their own. When personal
immortality becomes the goal, the terrorists cease to think like political actors, susceptible to rational
calculation of effect, and begin to act like fanatics. It is not easy to turn human beings into fanatics. In order to do so, terrorist groups
that use suicide bombers have to create a cult of death and sacrifice, anchored in powerful languages of belief. Osama bin Laden used an interview with an
American journalist in May I998 in Afghanistan to justify terrorism in the language of faith: The terrorism we practice is of the commendable kind for it is
directed at the tyrants and the aggressors and the enemies of Allah, the tyrants, the traitors who commit acts of treason against their own countries and their
own faith and their own prophet and their own nation.What
is noticeable here is the use of religion not just to justify killing
the infidel but to override the much more serious taboo against killing fellow believers. The function of
nihilism here is to recast real, living members of the Islamic faith as traitors deserving death. Nihilism
takes the form of nullifying the human reality of people and turning them into targets.

Page 1123 of 1481


Page 1124 of 1481
Nihilism is the Root Cause of
Violence
NIHILISM IS THE ROOT OF ALL VIOLENCE AND MANIFESTS PERMANENT DEATH
CULTS IN SOCIETIES

Ignatieff, Carr Prof of Human Rights @ Harvards Kenndy School of Government, 2K4 (Michael, The
Temptations of Nihilism, an extract from The Lesser Evil: Political Ethics in an Age of Terror, New England Review Vol.
25 No. 1/2, P. 54-74)

Nihilism--which is the blunt name for taking the gloves off--holds real dangers for both sides. When a
democratic state licenses all means to repress a terrorist group, it may only play into the hands of its
enemy. Some terrorist groups deliberately seek to draw reprisals upon themselves in order to radicalize
their own population. As the state's repression increases, the terrorists respond by tightening their screws on their base of support, replacing a
political relation to their own side with one of unvarnished tyranny, killing or intimidating anyone who questions whether the costs of the campaign are
outweighing the gains. Populations that once supported armed struggle for reasons of conviction become trapped either in fanaticism or in complicit silence. In
the process, political regulation of terrorist groups by their community at large becomes impossible. Moderate voices who might persuade a community to
withdraw their support from terror are silenced. In place of a properly political culture, in which groups and interests compete for leadership, a people
represented by suicide bombers ceases to be a political community at all and becomes a cult, with all the attendant hysteria, intimidation, and fear. This is the
process by which nihilism leads to a war without end. In such a terrorist cult, many praiseworthy moral virtues are inverted, so that they serve not life but
death. Terrorist groups typically expropriate the virtues of the young--their courage, their headstrong disregard for consequences, their burning desire to
establish their own significance--and use these to create an army of the doomed. In this way, violence becomes a career, a way of life that leads only to death.
Once violence becomes part of a community death cult, the only rational response by a state under attack
must be to eliminate the enemy one by one, either by capture and lifelong imprisonment or by execution.
Those for whom violence has become the driving rationale of conduct cannot be convinced to desist. They
are in a deathly embrace with what they do, and argument cannot reach them. Nor can failure. It counts for
nothing that violence fails to achieve their political objective because such achievement has long since
ceased to be the test of their effectiveness. It is redemption they are after, and they seek death sure that they have attained it. They
have nothing to negotiate for, and we have nothing to gain by negotiating with them. They will take gestures of
conciliation as weakness and our desire to replace violence with dialogue as contemptible na"ivet. To say we are at war with Al Qaeda and suicide bombers in
general is to say that political dialogue is at an end. We have nothing to say to them nor they to us. Either we prevail or they do, and force
must be the arbiter.

Page 1125 of 1481


Page 1126 of 1481
Nihilism Causes
Authoritarianism
NIHILISM CULIMINATES IN AUTHORITARIANISM

Christenson, Nippert Prof of Law and Dean @ U of Cincinnati College of Law, 85 (Gordon A., Uncertainty in
Law and its Negation: Reflections, 54 U. Cin. L. Rev. 347)

Some dramatically characterize the trends just described as legal nihilism or the negation of the exercise of legitimate power without the assertion of

nihilism, whether real or imagined, leads


substantive theory in its place. As Michael Polanyi so cogently has noted,

[*357] inexorably to authoritarian responses and to the rise of ideology. The second
phenomenon which gave rise to our particular predicament thus emerged from the conversion of subjective moral judgment into ideology. Whether
derived from the twentieth century revolutions based on socialism or Marxism, on the human rights movement, or on a resurgence of neo-conservatism,

resulting
the intellectual roots of such movements are well described in European and Latin American literatures. Symbolic of that literature, and

in the negation of law and value, are Neitzche's moral and ethical superiority ,
Dostoyevski's novels and short stories and the works of the phenomenologists, existentialists and structuralists. All ask similar questions. Post-Marxist
thinkers -- Habermas, Foucault and Berger and other non-legal critical scholars -- have gained influence in legal scholarship which finds them to be useful

If there is no common basis for law or morality other than through a


analytic tools.

subjective or ideological construct, then the question is not what values


underpin a particular legal system, but how one's subjective preferences may
be infused with power, strategy and tactics throughout the general
community or imposed by coercion. The lawyer-advocate has long used various techniques based on pragmatic ideas of
progress, the frontier and change. These have been associated with the romanticism of the defender of the poor and downtrodden, the fighter for civil
rights, the human-rights warrior and the social reformer, who use courts and law as instruments of social change. In this construct, law as a secular system
has no normative content that is not ultimately subjective. If God is dead, all things are morally possible. The main claim to legitimacy or validity rests in
process; namely that the advocates who represent a particular morality or a particular social philosophy fight and prevail as warriors and advocates in an
existing decisionmaking process, akin to chivalry, aimed at changing official behavior or custom by fighting injustice, admittedly a subjective construct.
Once, however, the subjective advocacy model of changing the social structure is an accepted way of life, the natural reaction is that sauce for the goose is
sauce for the gander. If the objective validity of the normative system tacitly is rejected by those who seek to change it, then radicals holding an opposite
belief might just as well produce a similar claim by an activism with subjective preferences even more firmly rooted within the vices of common life. The
dialectic of thesis, antithesis and synthesis that seemed to move outward from the subjective to an objective world-view could work for the radical right
just as well as for the Marxist left!

Page 1127 of 1481


Page 1128 of 1481
**Nonviolence**

Page 1129 of 1481


Nonviolence Answers: 2AC (1/6)
FIRST, NO LINK PLAN DOESNT TAKE A STANCE ON VIOLENT RESPONSE. IT ONLY
ENDS CURRENT DETAINMENT PRACTICES

SECOND, PERM DO BOTH


WE MUST BE PRAGMATIC PACIFISTS TO END STATE VIOLENCEABSOLUTE
PACIFISM FAILS TO CHALLENGE THE POLICIES OF THE STATE BY OPTING OUT OF
THE GAME ENTIRELY

Robert L. Phillips, professor of philosophy, War and Justice, 1984, p. 114-6

It conceivable that governments might grant selective objection the same legal status as it gives to
pacifism? The answer, I fear, is no. And that tells us something important about pacifism. Governments
are prepared to tolerate pacifism, because it poses no threat either to their political policies or to the
manner in which wars are conducted. The pacifist objects equally to all wars waged by all governments. In
this sense he opts out of the game altogether. By contrast, the selective objector will be forced to analyze
both the policy decisions of the government as well as the conduct of the armed forces. He will be publicly
carrying out an officially sanctioned comparison between mutually agreed just-war criteria and the actual
performance of the government. That is a lot to expect of governments as we know them, but there is still
more. What would be the implication of a state granting an exemption on selective grounds?
Fundamentally, the state would be agreeing with the claim that its war policies may be reasonably
interpreted as unjust. The belief that all war is wrong is a proposition which states might agree is
debatable among rational men, and, therefore, claims to exemption on this basis may be allowed. It is a
very different matter, however, to grant exemption for a particular war, for here we are faced not with
two philosophical theories about violence but with a factual dispute. Selective objection presupposes that
both the government and the claimant agree upon the criteria for undertaking a justified war and the
rules for conducting it. The claimant would have to show, in order to qualify for an exemption, that his
government is engaged in acts of war which a person might reasonably characterize as immoral. As such
an admission is inseparable from policy questions, it is inconceivable that any government would be
willing (or politically able) to wage war while publicly agreeing that there is sufficient reason to doubt the
morality of that war to grant exemptions from it. This is not to say that individuals should not refuse to
fight in wars which they believe are immoral but to acknowledge that governments cannot be expected to
institutionalize such a practice. The evenhandedness of the pacifist who objects to all wars does not
threaten the particular policies of any state. In condemning them all equally, pacifism exempts itself from
political reality: What is needed, then, is not a general pacifism but a discriminating conscientious refusal
to engage in war in certain circumstances. States have not been loath to recognize pacifism and to grant it
a special status. The refusal to take part in all war under any conditions is an unworldly view bound to
remain a sectarian doctrine. It no more challenges the states authority than the celibacy of priests
challenges the sanctity of marriage. By exempting pacifists from its prescriptions the state may even seem
to display a certain magnanimity. But conscientious refusal based upon the principles of justice as they
apply to particular conflicts is another matter. For such refusal is an affront to the governments
pretensions, and when it becomes widespread, the continuation of an unjust war may prove impossible.

Page 1130 of 1481


THIRD, PLAN IS NECESSARY FOR ALTERNATIVE SOLVENCY IN THE SQUO PEOPLE
WILL STILL BE VIOLENTLY DETAINED. THIS MAKES A DOUBLE BLIND EITHER THE
ALT CAUSES PLAN AND THERES NO LINK DIFFERENTIAL OR IT DOESNT SOLVE

Page 1131 of 1481


Page 1132 of 1481
Nonviolence Answers: 2AC (2/6)
FOURTH, MULTILAT SOLVES BY ALLOWING US TO ADDRESS PROBLEMS WITH
INTERNATIONAL COOPERATION, SOLVING BAD FORMS OF VIOLENCE. CROSS-
APPLY NYE

FIFTH, WAR AND VIOLENCE ARE ENDEMIC TO IR POLITICS, MOVING AWAY WILL
INEVITABLY RESULT IN GREAT POWER WARS

MEARSHEIMER 2001
[John, Co-Director of IR Policy at University of Chicago and Former research fellow at the Brookings
institute, The Tragedy of Great Power Politics, pg xi-xii. )

The twentieth century was a period of great international violence.In World War I (1914-18), roughly nine million
people died on European battlefields. About fifty million people were killed duringWorld War 11(1939-45), well over half of them civilians. Soon after the end of
World War II, the Cold War engulfed the globe. During this con-frontation, the Soviet Union and its Warsaw Pact allies never directly fought the United States
and its North Atlantic Treaty Organization allies,but many millions died in proxy wars in Korea, Vietnam, Afghanistan, Nicaragua, Angola, El Salvador, and
elsewhere. Millions also died in the century's lesser, yet still fierce, wars, including the Russo-Japanese con-flicts of 1904-5 and 1939, the Allied intervention in
the Russian Civil War from 1918 to 1920, the Russo-Polish War of 1920-21, the various Arab-Israeli wars, and the han-Iraq War of 1980-88. This cycle of

Hopes for peace will probably not be realized, because the great
violence will continue far into the new millennium.

powers that shape the international system fear each other and compete for power as a result.
Indeed, their ultimate aim is to gain a position of dominant power over others, because having
dominant power is the best means to ensure one's own survival. Strength ensures safety, and
the greatest strength is the greatest insurance of safety. States facing this incentive are fated to clash as each competes
for advantage over the others. This is a tragic situation, but there is no escaping it unless the states that make up the system

agree to form a world government. Such a vast transformation is hardly a realistic prospect, however, so conflict and war are bound to

continue as large and enduring features of world politics.

SIXTH, THEIR AUTHORS MISUNDERSTAND IR, WHICH IS A SELF HELP SYSTEM.


RATIONAL ACTORS ARE DETERRED BY VIOLENCE, CREATING WORLD PEACE

Page 1133 of 1481


Page 1134 of 1481
Nonviolence Answers: 2AC (3/6)
SEVENTH, NON-VIOLENCE FAILS AND MAKES US COMPLICIT WITH GENOCIDE
THEIR ALTERNATIVE DEVALUES LIFE AND LEADS TO MORE CONFLICT AND
MILLIONS OF DEATHS

Ketels, Assoc Prof of English @ Temple U, 96 (Violet B., Havel to the Castle! The Power of the Word, 548
Annals 45, November, Lexis)

Havel stresses the potential of truth and humane values to transform human consciousness incrementally over time. We must constantly work for every good thing and struggle against

Violence may be unavoidable in the face of totalitarian


violence. But Havel is tough-minded, his vision comprehensive and realistic.

savagery. Still, it must remain a means of last resort. Repeatedly, he warns that violence breeds violence. Havel is not, however, a
pacifist, as that term applies to Quakers or others who organize peace movements. n40 Although the regime Havel and his fellow dissidents resisted for more than thirty years accused
them of terrorist tactics and plots, they conscientiously sought legal justification for their resistance, using the letter even of unjust laws to manifest support for the principle of legality. Their
attitude was "fundamentally hostile to the notion of violent change--simply because it places its faith in violence," Havel writes in one place. He immediately restates the point, however, in a

"the 'dissident' attitude can only accept violence as a necessary evil in extreme
powerfully significant parenthesis:

situations, when direct violence can only be met by violence and where remaining passive would in effect
mean supporting violence." n41 He recalls us to the tragic blindness of European pacifism that helped to
prepare the ground for World War II. He points to the fact that the Czechs sent troops to the Persian Gulf and stood
willing to contribute to a U.N. force in the former Yugoslavia. But he is at pains to condemn violence used as a quick fix to change political systems--the
sacrifice of human beings here and now for "abstract political visions of the future." The problems in human society "lie far too deep to be settled through [*55] mere systemic changes, either
governmental or technological." n42 Havel writes and thinks out of a unique humanist tradition that has been continuous in Czech history. He has specifically identified with the humanism
of the founder of the Czech state, Tomas Masaryk, who regarded "ethical, aesthetic and scientific categories" as "no less real than bread and butter." Masaryk felt the need for a social
revolution "more moral and less materialistic than that envisaged by the Marxists." Like Havel, he hoped to avoid violence, but he does not rule it out altogether. His language is as

We may, should, must


circumspect as Havel's: We must consistently reject every act of violence; otherwise we shall never be able to disentangle ourselves from violence.

protect, defend ourselves. In extreme cases with the sword. But even in self-defense we must restrain ourselves from new, active acts of
violence. n43 In an address prepared for delivery at a 1985 peace conference, Havel explains the reticence of Europeans to join Western peace

movements as rooted in the skepticism of those who have already been burned by succumbing to other
forms of utopianism, specifically the Stalin-Leninist variety, which grotesquely deformed its utopian
principles as soon as it got power. The very word "peace" has been drained of all content by the European
experience of "peace in our time." n44 The Western version of peace sounds far too much like appeasement. Havel speculates whether World War II, with its
millions of corpses, could have been avoided if the Western democracies had stood up to Hitler forcefully and in time. He ascribes to the Czech people as a whole the firmly rooted idea that

the inability to risk, in extremis, even life itself to save what gives it meaning and a human dimension
leads not only to the loss of meaning but finally and inevitably to the loss of life as well--and not one life
only but thousands and millions of lives. n45

Page 1135 of 1481


Page 1136 of 1481
Nonviolence Answers: 2AC (4/6)
EIGHTH, OUR PLAN IS THE EXEMPTION NUCLEAR ANNIHILATION MUST BE
PREVENTED BEFORE NON-VIOLENCE

Marty, Professor of Political Science. Mepbis State University, 71 (William R., The Journal Politics, Vol. 33, No. I,
Feb, p. 19-20, JSTOR)

Defenders of nonviolence sometimes level a final crushing charge against violencethat it is, in an age
of nuclear weapons, a sure path to annihilation. Dr. King, for example, argued that our choice is no
longer nonviolence or violence, rather it is nonviolence or nonexistence.17 The only new element in
this argument for nonviolence is the threat of nuclear annihilation. That threat, presumably, makes
total commitment to nonviolence both necessary and possible. In fact, however, certain types of
violence pose no threat of nuclear warfare, hence the horrors of nuclear warfare provide no reason or
incentive to give up these types of violence; other types of violence pose a real threat of nuclear
warfare, but the dangers involved in abandoning conventional weapons will seem greater and more
immediate, hence conventional violence is unlikely to be abandoned; and, finally, realistic plans for
community order and nuclear disarmament, the most likely path to survival, depend, at least
potentially, upon violent enforcement. For these reasons, the call to total commitment to nonviolence
in order to avoid total nuclear annihilation is neither rationally necessary nor psychologically likely to
be adopted. Each of the listed objectives deserves elaboration. First, certain types of violence pose no
threat of nuclear annihilation. The man or woman who keeps a weapon in the home to deal with
intruders (burglars, sex criminals, rioters) may be unwise for several reasons, but not because his or her
weapon poses a threat of nuclear warfare. Whether this person uses a weapon against an intruder, or
resists nonviolently, or submits, will have no effect on whether nuclear war is waged between nations,
though it will have considerable effect on his or her personal safety. To ask this person to disarm in
order to avoid nuclear warfare is as ridiculous as it would be to ask city officials having no say whatever
in the decision to wage nuclear warfare to disarm their police in order to avoid nuclear annihilation.
Even on a national and international level there are types of violence that pose little threat of nuclear
warfare. In Chad and Sudan, for example, there has been guerrilla and civil warfare for years, but the
threat of nuclear warfare resulting from these conflicts is small or nonexistent because nuclear
weapons don't exist in those nations and because nations with nuclear weapons have no incentive to
intervene that is worth a nuclear confrontation. In these cases the threat of nuclear warfare is
inadequate as an incentive to adopt nonviolence because no apparent threat of nuclear warfare exists.
In sum, from the individual to the international level, there are types of violence that pose no real
threat of nuclear warfare, and certainly are not perceived by those employing them as threatening
nuclear warfare; hence they have no incentive to adopt nonviolence as an alternative to nuclear
warfare. In other cases, such as the continuing crisis in the Middle East, the possibility of nuclear
warfare is real, but the threat is unlikely to cause renunciation of violence because other dangers seem
greater and more immediate. To Israel the dangers of adopting nonviolence in the face of Arab hatred
and calls for national extinction seem greater than the dangers of nuclear warfare resulting from armed
defense. The Israelis are unlikely to make a total commitment to nonviolence in all circumstances
despite a real threat of nuclear confrontation. The same situation occurs in Vietnam. There was at least
a remote chance of nuclear confrontation in Vietnam at one time, but that did not provide adequate
incentive to any of the involved parties, from the Viet Cong to the United States and Russia, to
renounce all types of violence, though it did produce some restraints on United States and Russian
intervention. Again, when the danger of death is already great by conventional means, and when
abandonment of conventional weapons appears as suicidal, then the threat of nuclear warfare will he
inadequate as an incentive to renounce all types of violence. An appeal to the North Vietnamese and

Page 1137 of 1481


Viet Cong, or to the Saigon government, or to both, to abandon violence in order to avoid the
possibility of nuclear war would be fruitless.

Page 1138 of 1481


Page 1139 of 1481
Nonviolence Answers: 2AC (5/6)
EVEN NON-VIOLENT MOVEMENTS INEVITABLY BECOME TOTALITARIAN AND
VIOLENTTHE LEADERS CANNOT CONTROL THE MASSES

Steger, Prof in the Dept of Politics and Government @ Illinois State U, 00 (Manfred B., Gandhis Dilemma:
Nonviolent Principles and Nationalist Power)

Urging all Indians to crown the swadeshi campaign by publicly burning their foreign-made clothes, Gandhi spoke in glowing terms of the "inspiring sight" of
large piles of garments going up in smoke: "And as the flames leapt up and enveloped the whole pyramid [of clothes], there was a shout of joy resounding
through the air. It was as if our shackles had been broken asunder. A glow of freedom passed through the vast concourse. It was a noble act nobly
performed."62 Yet, the flames of swadeshi kindled by thousands of ordinary Indian also symbolized, like no other satyagraha action, the fundamental tension
at the core of Gandhi's nonviolent nationalism. For the Ma-hatma, the burning clothes manufactured in England conveyed India's economic, political,
and spiritual emancipation from the threads of oppression. He viewed these spectacles as symbols of the nonviolent purification of a corrupted civilization
and its materialist culture, and, therefore, the purgation of a tainted Indian identity. For othersincluding some of Gandhi's closest associates and
friends, like Charlie Andrewsthe
flames of swadeshi signified a rather violent act of self-definition that seemed
to be an ominous sign of things to come: the obliteration of the Other by nationalist passions set ablaze.
Indeed, the first indication that Gandhi was incapable of controlling the nationalist passions of the masses
set free during the noncooperation campaign came as early as April 1921, when a sub-inspector of police
and four constables were killed in an act of mob violence provoked by the trial of Khilafat workers in the city of Malegaon. Gandhi
chided the perpetrators for having "put back the hands of the clock of progress," and reminded them that, "Non-violence is the rock on which the whole
structure of non-co-operation is built."63 Yet
another incident took place in Bombay on November 17, 1921, the day the Prince
of Wales arrived there for an official visit. Violent attacks were launched by Hindu and Muslim noncoop-
erators upon Parsi and Christian Indians who had voluntarily taken part in the Prince's welcome. The
violence escalated as many non-cooperators looted shops and burned clothes. Soon these actions
expanded to the torching of entire buildings and the beating of government officials, ultimately leading to
the deaths of several policemen and demonstrators. When, after three days of violence, the passions had finally cooled down,
fifty-eight Bombay citizens had been killed and nearly four hundred had been injured.64

EVERY AFFIRMATIVE ETHICAL STANCE REQUIRES A REPRESSED ELEMENT OF


NEGATION, MEANING THAT THE ALTERNATIVE OCCURS AGAINS THE
BACKGROUND OF COVERT VIOLENCE

Zizek '99
[Slavoj, Senior Researcher at Institute for Social Studies, Ljubliana and Badass, The Ticklish Subject: the
absent centre of political ontology, New York: Verso, 1999, 153-4//uwyo-ajl]

It would therefore be tempting to risk a Badiouian-Pauline reading of the end of psychoanalysis, determining it as a New Beginning, a symbolic 'rebirth' - the radical restructuring
of the analysand's subjectivity in such a way that the vicious cycle of the superego is suspended, left behind. Does not Lacan himself provide a number of hints that the end of
analysis opens up the domain of Love beyond Law, using the very Pauline terms to which Badiou refers? Nevertheless, Lacan's way is not that of St Paul or Badiou:

psychoanalysis is not 'psychosynthesis'; it does not already posit a 'new harmony', a new Truth-Event; it - as it were -
merely wipes the slate clean for one. However, this 'merely' should be put in quotation marks, because it is Lacan's contention that, in this
negative gesture of 'wiping the slate clean', something (a void) is confronted which is already 'sutured' with
the arrival of a new Truth-Event. For Lacan, negativity, a negative gesture of withdrawal, precedes any positive
gesture of enthusiastic identifiction with a Cause: negativity functions as the condition of (im)possibility of
the enthusiastic identification - that is to say, it lays the ground, opens up space for it, but is simultaneously
obfuscated by it and undermines it. For this reason, Lacan implicitly changes the balance between Death and Resurrection in favour of Death: what 'Death'
stands for at its most radical is not merely the passing of earthly life, but the 'night of the world', the self-withdrawal, the absolute contraction of

Page 1140 of 1481


subjectivity, the severing of its links with 'reality' - this is the 'wiping the slate clean' that opens up the
domain of the symbolic New Beginning, of the emergence of the 'New Harmony' sustained by a newly emerged Master-Signifier. Here, Lacan parts
company with St Paul and Badiou: God not only is but always-already was dead - that is to say, after Freud, one cannot directly have faith in a Truth-Event; every such

Event ultimately remains a semblance obfuscating a preceding Void whose Freudian name is death drive. So
Lacan differs from Badiou in the determination of the exact status of this domain beyond the rule of the Law. That is to say: like Lacan, Badiou delineates the contours of a
domain beyond the Order of Being, beyond the politics of service des biens, beyond the 'morbid' super ego connection between Law and its transgressive desire. For Lacan,

the 'death drive' is not the outcome of the


however, the Freudian topic of the death drive cannot be accounted for in the terms of this connection:

morbid confusion of Life and Death caused by the intervention of the symbolic Law. For Lacan, the uncanny
domain beyond the Order of Being is what he calls the domain 'between the two deaths', the pre-ontologicalf
domain of monstrous spectral apparitions, the domain that is 'immortal', yet not in the Badiouian sense of the immortality of participating in Truth,
but in the sense of what Lacan calls lamella, of the monstrous 'undead' object-libido.18

Page 1141 of 1481


Page 1142 of 1481
Nonviolence Answers: 2AC (6/6)
TURN - ATTEMPTING TO CLEANSE LANGUAGE OF VIOLENCE FETISHIZES
AUTHENTICITY, RESULTING IN POLITICAL DISENGAGEMENT BECAUSE OF THE
VIOLENCE AT THE HEART OF ALL LANGUAGE AND INTERACTION

Bewes 97
[Timothy, doctorate in English Literature at the University of Sussex, Cynicism and Postmodernity, New
York City: Verso, 1997, 137-8//uwyo-ajl]

Thus, what secondly distinguishes the 'metaphysical innocence' of Rameau is his pursuit of violence - not only the violence of determinate negation, of alienation from culture
and the serial progression of knowledge, but the violence of imperfection, of disrupted subjectivity, of unforeseen catastrophes and superfluous resources, of human
inconsistency and what Gillian Rose calls the 'agon' of existence. Violence, like suffering and fickleness for Dostoevsky, represents subjective (as against objective) culture, a

. Violence increases as the result not


last manifestation of individual volition, and a point of resistance to what BaudriUard calls the 'triumph' of simulation

of a deterioration in social behaviour but of a lowering in the cultural threshold beyond which action
appears as violence. In such a context Rameau's disintegration, his 'epigrammatic' existence and his cultivation of violence represent the final recourse of a
disfranchised and alienated subjectivity faced with an apparently sewn up, indifferent world.

In postmodernity this threshold between action and violence is lower, perhaps, than ever before. Political correctism, 'Queer' theory, Communitarianism, the liberation discourse
of the Internet, calls for homogenization of the private and public lives of politicians, the new discipline of 'postmodern ethics', all are varying instances of a collective endeavour

to put a freeze on reason as risk, the consequence of a fetishization of objective culture. To find intolerable the violence of
linguistic oppression, of 'inauthentic' sexual identity (the product of Freud's 'family romance', etc.), of political antagonism, of the formalization of truth in its
dissemination, of the compart mentalization of public and private life, of the indeterminacy of moral options, is in every case to subscribe to a peculiar

literalism, to evince a profound discomfort with the signifying relation, to take the signifier persistently for
the thing itself, in such a way that political activity is replaced with a series of cosmetic adjustments to
objective culture.
Rameau's cynicism therefore represents a commitment to subjective culture, to reality, to the referent and to the signified, to the truth of the world and of the individual.
Cynicism constitutes a certain necessary indifference to objective culture, a certain subjective wager, a projection of the self beyond objective culture and beyond its own limits.

In a climate in which 'authenticity' is at a premium, where all action has been proscribed as intolerably
violent, and where self consciousness is therefore only a disabling mechanism to be discad, cynicism appears as a spirit in disintegration,
the monopoly broker of disinvestment in the present, the sole locus of reason and of faith in anything other than the phenomenal here and
now, the disposition which alone embodies both energy and depth.

DETERRENCE IS NECESSARY FOR WORLD PEACE

Robinson 2001
[C. Paul, Sandi National Laboraties, A White Paper:Pursuing a New Nuclear Weapons Policy for the 21st Century,
March 22, www.mindfully.org/Nucs/Nuclear-Weapons-Policy-21stC.htm, 9-23-06//uwyo-ajl]

I served as an arms negotiator on the last two agreements before the dissolution of the Soviet Union and have spent most of my career enmeshed in the complexity of nuclear
weapons issues on the government side of the table. It is abundantly clear (to me) that formulating a new nuclear weapons policy for the start of the 21st Century will be a most
difficult undertaking. While the often over-simplified picture of deterrence during the Cold War-two behemoths armed to the teeth, staring each other down-has thankfully

, there are nevertheless huge arsenals of nuclear weapons and delivery systems, all in quite usable
retreated into history

states, that could be brought back quickly to their Cold War postures. Additionally, throughout the Cold War and ever since, there
has been a steady proliferation of nuclear weapons and other weapons of mass destruction by other nations around the globe. The vast majority of these newly armed states
are not U.S. allies, and some already are exhibiting hostile behaviors, while others have the potential to become aggressors toward the U.S., our allies, and our international
interests.

Russia has already begun to emphasize the importance of its arsenal of nuclear weapons to compensate for its limited conventional capabilities to deal with hostilities that
appear to be increasing along its borders. It seems inescapable that the U.S. must carefully think through how we should be preparing to deal with new threats from other
corners of the world, including the role that nuclear weapons might serve in deterring these threats from ever reaching actual aggressions.

I personally see the abolition of nuclear weapons as an impractical dream in any foreseeable future. I came to this view

Page 1143 of 1481


from several directions. The first is the impossibility of ever "uninventing" or erasing from the human
mind the knowledge of how to build such weapons. While the sudden appearance of a few tens of nuclear weapons causes only a small stir in a world where
several thousands of such weapons already exist, their appearance in a world without nuclear weapons would produce huge

effects. (The impact of the first two weapons in ending World War II should be a sufficient example.) I believe that the words of Winston Churchill, as quoted by Margaret
Thatcher to a special joint session of the U.S. Congress on February 20, 1985, remain convincing on this point: "Be careful above all things not to let go of the atomic weapon
until you are sure, and more sure than sure, that other means of preserving the peace are in your hands."

the majority of the nations who have now acquired arsenals of nuclear weapons
Similarly, it is my sincere view that

believe them to be such potent tools for deterring conflicts that they would never surrender them. Against this
backdrop, I recently began to worry that because there were few public statements by U.S. officials in reaffirming the unique role which nuclear weapons play in ensuring U.S.
and world security, far too many people (including many in our own armed forces) were beginning to believe that perhaps nuclear weapons no longer had value. It seemed to
me that it was time for someone to step forward and articulate the other side of these issues for the public: first, that nuclear weapons remain of vital importance to the security

nuclear weapons will likely have an enduring role in


of the U.S. and to our allies and friends (today and for the near future); and second, that

preserving the peace and preventing world wars for the foreseeable future. These are my purposes in writing this paper.

Page 1144 of 1481


#2 Pragmatic Pacifism Perm: 1AR (1/2)

WE MUST ADOPT AN INTRINSIC FORM OF PACIFISMABSOLUTE PACIFISM


JUSTIFIES PASSIVITY IN THE FACE OF ATROCIOUS ACTS LIKE RAPE OR GENOCIDE

Robert L. Phillips, professor of philosophy, War and Justice, 1984, p. 101-2

Let us label this position intrinsicalism and contrast it with what I shall call tactical pacifism. Someone
who believes that it is morally permissible to use force to resist or prevent violence might adopt the
pacifist stance as a purely tactical matter. He might judge that pacifism is likely to be the best means of
bringing about peace. This could happen in at least two ways. It might be thought that pacifism is the
appropriate response because of peculiar historical circumstances. Thus, India in 1946 and the United
States in the 1960s could be seen as places where nonviolent resistance would be an appropriate tactic. In
both of those places the rule of law obtained to the degree that the penalties for such disobedience were
relatively mild, and there was a chance that such tactics might succeed. However, the same person could
well decide that pacifism was not obligatory in Nazi Germany or Stalinist Russia. Someone might also
adopt tactical pacifism based upon a judgment about the actual possibility of using force justly in the
modern era. While admitting the theoretical possibility of justified force, it may be thought that as long as
certain sorts of weapons are retained, or as long as terror is officially sanctioned, then a justified war
simply cannot be fought. Both of these versions of tactical pacifism are compatible with bellum justum;
indeed, they are entailed by that doctrine. Neither makes an a priori commitment to the position that the
use of force will always, under all conceivable circumstances, be wrong. The behavior of the tactical
pacifist may be indistinguishable from that of the intrinsicalist on many occasions, but the former leaves
open the question of whether force is justified in a given circumstance, and this marks an important moral
difference. Thus, intrinsicalism is the only version of pacifism which can be described as a moral position
opposed to bellurn justum. In Narvesons words, To hold the pacifist position as a genuine, full-blooded
moral principle is to hold that nobody has a right to fight back when attacked, that fighting back is
inherently evil, as such. It means we are mistaken in supposing we have a right of self-protection.

MUST BACK UP NON-VIOLENCE WITH THE THREAT OF VIOLENCE ACTION OF MLK


AND MALCOLM X PROVE THIS SOLVES BEST

J. A. H. Futterman, Ph.D. from UT-Austin and Physicist at the University of California's Lawrence Livermore
National Laboratory, Obscenity and Peace: Meditations on the Bomb, 1990-94,
http://www.dogchurch.com/scriptorium/nuke.html, UK: Fisher

Even when non-violence does succeed, it does so by rallying the majority of the population toward whom
it is directed to stop the direct perpetrators of injustice by force -- the force of law in the form of the
police, the prisons, and the polls -- force that necessarily includes the threat of violence. In other words,
non-violent resistance harnesses (or co-opts), rather than eliminates violence.

In fact, non-violence is sometimes even helped by the threat of violence to achieve its objectives. The
non-violence of Dr. Martin Luther King, Jr., was complemented by the willingness to use "any means
necessary" of Malcolm X. These two men were sending white America the same message concerning
justice and racial equality. If whites failed to respond to the message stated gently, whites would be given

Page 1145 of 1481


the opportunity to respond to it stated violently. It took both statements to achieve the progress made
thus far.

Page 1146 of 1481


Page 1147 of 1481
#2 Pragmatic Pacifism Perm:
1AR (2/2)
WE DONT ADVOCATE VIOLENCE, JUST DEFENSE AGAINST OPPRESSION

Hans Massaquoi, Ebony, February 1993, p. 36

Now the reporter wants to know whether Malcolm X suggests using violence. The benign expression
vanishes and his eyes become fierce. We dont advocate violence, but non-violent tactics based solely
on morality can only succeed when you are dealing with a basically moral people, he explains. A man
who oppresses another man because of his color is not moral. It is the duty of every Afro-American to
protect himself [herself] against mass murderers, bombers, lynchers, floggers, brutalizers and exploiters.
If the government is unable or unwilling to protect us, we reserve our right as citizens to defend ourselves
by whatever means necessary. A man with a rifle or club can only be stopped by a person armed with a
rifle or club.

[herself is my feminist editing, ceo]

Page 1148 of 1481


A2 Violence Snowballs: 1AR

THE STRATEGIC EXERCISE OF VIOLENCE DOESNT SNOWBALL INTO A CULT OF


TERRORIT IS A NECESSARY PART OF OUR STRATEGIC REPERTOIRE OF
RESISTANCE

Ward Churchill, (Keetoowah Band Cherokee) Professor of Ethnic Studies and Coordinator of American Indian

Studies, 2001, Pacifism as Pathology, p. 90-92

This perception of pacifism as a self-justifying ideological preemption of proper praxical consideration,


subliminally intended to perpetuate the privileged status of a given progressive elite, is helpful in
determining what is necessary to arrive at a true liberatory praxis within advanced capitalist contexts. The
all but unquestioned legitimacy accruing to the principles of pacifist practice must be continuously and
comprehensively subjected to the test of whether they, in themselves, are capable of delivering the
bottom-line transformation of state-dominated social relations which alone constitutes the
revolutionary/liberatory process. Where they are found to be incapable of such delivery, the principles
must be broadened or transcended altogether as a means of achieving an adequate praxis. By this, it is
not being suggested that nonviolent forms of struggle are or should be abandoned, nor that armed
struggle should be the normative standard of revolutionary performance, either practically or
conceptually. Rather, it is to follow the line of thinking recently articulated by Kwame Ture (Stokely
Carmichael) when he noted: If we are to consider ourselves as revolutionaries, we must acknowledge that
we have an obligation to succeed in pursuing revolution. Here, we must acknowledge not only the power
of our enemies, but our own power as well. Realizing the nature of our power, we must not deny
ourselves the exercise of the options available to us; we must utilize surprise, cunning and flexibility; we
must use the strength of our enemy to undo him, keeping him confused and off-balance. We must
organize with perfect clarity to be utterly unpredictable. When our enemies expect us to respond to
provocation with violence, we must react calmly and peacefully; just as they anticipate our passivity, we
must throw a grenade. What is at issue is not therefore the replacement of hegemonic pacifism with
some cult of terror. Instead, it is the realization that, in order to be effective and ultimately successful,
any revolutionary movement within advanced capitalist nations must develop the broadest possible range
of thinking/action by which to confront the state. This should be conceived not as an array of component
forms of struggle but as a continuum of activity stretching from petitions/letter writing and so forth
through mass mobilization/demonstrations, onward into the arena of armed self-defense, and still
onward through the realm of offensive military operations (e.g., elimination of critical state facilities,
targeting of key individuals within the governmental/corporate apparatus, etc.). All of this must be
apprehended as a holism, as an internally consistent liberatory process applicable at this generally-
formulated level to the late capitalist context no less than to the Third World. From the basis of this fun-
damental understanding and, it may be asserted, only from this basis can a viable liberatory praxis
for North America emerge. It should by now be self-evident that, while a substantial even
preponderant measure of nonviolent activity is encompassed within any revolutionary praxis, there is
no place for the profession of principled pacifism to preclude much less condemn the utilization of
violence as a legitimate and necessary method of achieving liberation. 167 The dismantling of the false
consciousness inherent in the ideology of nonviolent revolution is therefore of primary importance in
attaining an adequate liberatory praxis.

TACTICAL USE OF VIOLENCE IS NECESSARY

Page 1149 of 1481


Ed Mead, a guy who went to prison for violent protest, 1998, Pacifism as Pathology, p. 14

Those who denounce the use of political violence as a matter of principle, who advocate nonviolence as a
strategy for progress, are wrong. Nonviolence is a tactical question, not a strategic one. The most vicious
and violent ruling class in the history of humankind will not give up without a physical fight. Nonviolence
as a strategy thus amounts to a form of liberal accommodation and is bound to fail. The question is not
whether to use violence in the global class struggle to end the rule of international imperialism, but only
when to use it.

Page 1150 of 1481


Page 1151 of 1481
#5 Violence Inevitable: 1AR
POWER IS ZERO SUM THE ALTERNATIVE ONLY SHIFTS POWER ELSEWHERE

John Mearsheimer, Professor at University of Chicago, 2001 (The Tragedy of Great Power Politics p. 34)

Consequently, states pay close attention to how power is distributed among them, and they make a special effort to maximize
their share of world power. Specifically, they look for opportunities to alter the balance of power by acquiring
additional increments of power at the expense of potential rivals. States employ a variety of meanseconomic,
diplomatic, and militaryto shift the balance of power in their favor, even if doing so makes other states suspicious or even
hostile. Because one states gain in power is another states loss, great powers tend to have a zero-sum
mentality when dealing with each other. The trick, of course, is to be the winner in this competition and to dominate
the other states in the system. Thus, the claim that states maximize relative power is tantamount to arguing that states are
disposed to think offensively toward other states, even though their ultimate motive is simply to survive. In short, great
powers have aggressive intentions.

Page 1152 of 1481


Page 1153 of 1481
#7 Pacifism Allows Atrocity: 1AR
VIOLENCE IS KEY TO RESIST OMNICIDEPACIFISM IS MORALLY BANKRUPT
COMPLICITY IN THE VIOLENCE OF THE STATE

Mike Ryan, Canadian anti-imperialist, 2001, Pacifism as Pathology, p. 161

We recognize the right of oppressed peoples to respond to their oppression with violence, but we abstain
from engaging in violence ourselves. Thus we recognize our own participation in the oppression of other
peoples while we also attempt to deny the critical situation in which we ourselves are found today, a
circumstance described by Rosalie Bertell in an earlier quote. If, as Bertell suggests, we are sitting upon a
dying earth, and consequently dying as a species solely as a result of the nature of our society, if the
technology we have developed is indeed depleting the earth, destroying the air and water, wiping out
entire species daily, and steadily weakening us to the point of extinction, if phenomena such as Chernobyl
are not aberrations, but are (as I insist they are) mere reflections of our daily reality projected at a level
where we can at last recognize its true meaning, then is it not time--long past time --when we should do
any thing, indeed everything, necessary to put an end to such madness? Is it not in fact an act of
unadulterated self-defense to do so? Our adamant refusal to look reality in its face, to step outside our
white skin privilege long enough to see that it is killing us, not only tangibly reinforces the oppression of
people of colour the world over, it may well be the single most important contributor to an incipient
omnicide, the death of all life as we know it. In this sense, it may well be that our self-imposed inability to
act decisively, far from having anything at all to do with the reduction of violence, is instead perpetuating
the greatest process of violence in history. It might well be that our moral position is the most mammoth
case of moral bankruptcy of all time.

Page 1154 of 1481


Page 1155 of 1481
Pacifism = State Collusion (1/2)
PACIFIST PROTEST IS A COLLUSION WITH THE STATEIT HAS ZERO
REVOLUTIONARY POTENTIAL

Ward Churchill, (Keetoowah Band Cherokee) Professor of Ethnic Studies and Coordinator of American Indian

Studies at University of Colorado, 2001, Pacifism as Pathology, p. 61-62

Precisely. The preoccupation with avoiding actions which might provoke violence is thus not based on a
sincere belief that violence will, or even can, truly be avoided. Pacifists, no less than their unpacifist
counterparts, are quite aware that violence already exists as an integral component in the execution of
state policies and requires no provocation; this is a formative basis of their doctrine. What is at issue then
cannot be a valid attempt to stave off or even minimize violence per se. Instead, it can only be a conscious
effort not to refocus state violence in such a way that it would directly impact American pacifists
themselves. This is true even when it can be shown that the tactics which could trigger such a refocusing
might in themselves alleviate a real measure of the much more massive state-inflicted violence occurring
elsewhere; better that another 100,000 Indochinese peasants perish under a hail of cluster bombs and
napalm than Americas principled progressives suffer real physical pain while rendering their
governments actions impracticable. Such conscientious avoidance of personal sacrifice (i.e., dodging the
experience of being on the receiving end of violence, not the inflicting of it) has nothing to do with the
lofty ideals and integrity by which American pacifists claim to inform their practice. But it does explain the
real nature of such curious phenomena as movement marshals, steadfast refusals to attempt to bring the
seat of government to a standstill even when a million people are on hand to accomplish the task, and the
consistently convoluted victim-blaming engaged in with regard to domestic groups such as the Black
Panther Party. Massive and unremitting violence in the colonies is appalling to right-thinking people but
ultimately acceptable when compared with the unthinkable alternative that any degreee of real violence
might be redirected against mother country radicals. Viewed in this light, a great many things make
sense. For instance, the persistent use of the term responsible leadership in describing the normative
nonviolent sector of North American dissent always somewhat mysterious when applied to supposed
radicals (or German Jews) is clarified as signifying nothing substantially different from the
accommodation of the status quo it implies in more conventional settings. The rules of the game have
long been established and tacitly agreed to by both sides of the ostensible oppositional equation:
demonstrations of resistance to state policies will be allowed so long as they do nothing to materially
interfere with the implementation of those policies. The responsibility of the oppositional leadership in
such a trade-off is to ensure that state processes are not threatened by substantial physical disruption;
the reciprocal responsibility of the government is to guarantee the general safety of those who play
according to the rules This comfortable scenario is enhanced by the mutual understanding that certain
levels of appropriate (symbolic) protest of given policies will result in the oppositional victory of their
modification (i.e., really a tuning of policy by which it may be rendered more functional and efficient,
never an abandonment of fundamental policy thrusts), while efforts to move beyond his metaphorical
medium of dissent will be squelched by any means necessary and by all parties concerned. Meanwhile,
the entire unspoken arrangement is larded with a layer of stridently abusive rhetoric directed by each side
against the other. We are left with a husk of opposition, a ritual form capable of affording a
sentimentalistic Im OK, youre OK satisfaction to its subscribers at a psychic level but utterly useless in
terms of transforming the power relations perpetuating systemic global violence. Such a defect can,
however, be readily sublimated within the aggregate comfort zone produced by the continuation of North
American business as usual; those who remain within the parameters of nondisruptive dissent allowed by
the state, their symbolic duty to the victims of U.S. policy done (and with the bases of state power wholly
unchallenged), can devote themselves to the prefiguration of the revolutionary future society with which
they proclaim they will replace the present social order (having, no doubt, persuaded the state to

Page 1156 of 1481


overthrow itself through the moral force of their arguments).92 Here, concrete activities such as sexual
experimentation, refinement of musical/artistic tastes, development of various meat-free diets, getting in
touch with ones id through meditation and ingestion of hallucinogens, alteration of sex-based
distribution of household chores, and waging campaigns against such bourgeois vices as smoking
tobacco become the signifiers of correct politics or even revolutionary practice. This is as opposed to
the active and effective confrontation of state power.

Page 1157 of 1481


Page 1158 of 1481
Pacifism = State Collusion (2/2)
PACIFISM IS PLAYING BOTH SIDES OF THE ROADIT FACILITATES THE RACIST CO-
OPTATION OF THIRD WORLD VIOLENT STRUGGLE

Ward Churchill, (Keetoowah Band Cherokee) Professor of Ethnic Studies and Coordinator of American Indian

Studies at University of Colorado, 2001, Pacifism as Pathology, p. 72-74

It is possible then to visualize a world revolutionary process in which the necessity of armed participation
and attendant physical suffering) by white radicals is marginalized or dispensed with altogether. Their role
in this scenario becomes that of utilizing their already attained economic and social advantages to
prefigure, both intellectually and more literally, the shape of the good to be shared by all in the
postrevolutionary context; is presumed that they will become a (perhaps the) crucial social element,
having used the space (comfort zone) achieved through state concessions generated by the armed
pressure exerted by others to the constructive rather than destructive purpose of developing a
superior model of societal relations. The function of responsible oppositional leadership in the
mother country as opposed to the irresponsible variety that might precipitate some measure of
armed resistance from within before the Third World has bled itself in diminishing state power from
without (and who might even go so far as to suggest whites could directly participate) is first and
foremost to link the mother country movements inaction symbolically and rhetorically to Third World
liberation struggles. The blatant accommodation to state power involved in this is rationalized (both to
the Third Worlders and to the movement rank-and-file) by professions of personal and principled
pacifism, as well as in the need for working models of nonviolent behavior in postrevolutionary society.
From there, the nonviolent American movement (by now overwhelmingly composed of white
progressives) can be steered into exactly the same symbolic and rhetorical solidarity with an emerging
nonwhite armed revolution within the United States and voila! positive social transformation has
not only been painlessly achieved (for whites), but they (being the prefigurative nonviolent experts on
building postrevolutionary society) have maneuvered themselves into leading roles in the aftermath. All
of this, of course, is predicated on the assumption that the colonized, both within and without, will
ultimately prove equal to their part, and that revolutionary transformation will actually occur. In the event
that the colonizing state ultimately proves the stronger of parties in such a contest, the nonviolent
movement having restricted its concrete activities to limits sanctioned by that same state will have a
natural fall-back position, being as it were only a variant of the loyal opposition.22 The result of the
carefully-constructed balance (between professed solidarity with armed Third World insurgents on the
one hand, and tacit accommodation to the very state power against which they fight on the other) is that
North American adherents to nonviolence are intended to win regardless of the outcome; the comfort
zone of white skin privilege is to be continued in either event. Or this is the outcome that fence-sitting is
expected to accomplish. The range of tremendous ethical, moral, and political problems inherent in this
attitude are mostly so self-evident as to require no further explanation or targeting, and elimination of
some internal entity as the subversive element undercutting the national will and purpose. At such
times the state needs no, indeed can tolerate no hint of, domestic opposition; those who are tainted by
a history of even the milder forms of antisocial behavior can be assured of being selected as the
scapegoats required for this fascist sort of consensus building. While the precise form which might be
assumed by the scapegoating involved in a consolidation of North American fascism remains unknown, it
is clear that the posture of the mass nonviolent movement closely approximates that of the Jews in
Germany during the 1930s. The notion that it cant happen here is merely a parallel to the Jewish
perception that it wouldnt happen there, insistence on inhabiting a comfort zone even while thousands
upon thousands of Third World peasants are cremated beneath canisters of American napalm is only a
manifestation of the attitude of going on with business as usual, even in a holocaust.27 Ultimately, as
Bettelheim observed, it is the dynamic of attempting to restrict opposition to state terror to symbolic and

Page 1159 of 1481


nonviolent responses which gives the state the idea that [its victims can] be gotten to the point where
they [will] walk into the gas chambers on their own.128 And, as the Jewish experience has shown for
anyone who cares to look the matter in the face, the very inertia of pacifist principles prevents any
effective conversion to armed self-defense once adherents are targeted for systematic elimination by the
state.

Page 1160 of 1481


Page 1161 of 1481
Embracing Violence =
Nonviolence
IN ORDER TO HAVE A SUCCESSFUL TRANSITION TO NONVIOLENCE, WE MUST FIRST
SUBMIT TO THE VIOLENCE

Walter Wink, professor of theology, The Powers that be: Theology for a new Millennium, 1999, p. 119-120

What Gandhi learned from this experiment is that it is impossible to move oppressed people directly from
submission to active nonviolence. They need first to own their feelings of rage and even hatred and be
willing to fight against their oppressors. They need to be energized by their anger. Then they can freely
renounce violence for a nonviolent alternative that transforms the energy of their anger into a dynamic
and resolute love. We can apply Gandhis insight practically. If our children are being bullied at school, of
course we would prefer a nonviolent solution, and one can usually be found. But it may be important for
our children at least to be willing to fight on their own behalf before turning to a nonviolent solution.
Otherwise, requiring them literally to turn the other cheek can simply encourage cowardice. It will be
submission to evil rather than a creative alternative to violence. Heres how one boy dealt with a bully on
a school bus. The child was too slight of build to fight the far sturdier bully. But he had a weakness that he
made into a strength: chronic sinusitis. One day, exasperated at the bullys behavior, he noisily blew a
load of snot into his right hand and approached his nemesis, hand outstretched, saying, I want to shake
the hand of a real bully. The bully retreated, wide-eyed, to his seat. That ended the career of that bully.
Those sinuses were the ultimate weapon, and they were always at the ready!

Page 1162 of 1481


Pacifism = Violence (1/3)

NONVIOLENCE RISKS APPEASEMENT WHICH RESULTS IN MORE CONFLICT

Rummel, Prof of Political Science @ U of Hawaii, 81 (R.J., The Just Peace,


http://www.hawaii.edu/powerkil1s/TJP.CHAP 10.HTM)

Violent conquest is usually wrong (the Just Package). Forcibly imposing one's values and goals on another, aside from its general immorality, can create
smoldering resentment, grievance, and hostility that later may burst into greater conflict and violence. Nonetheless, insome exceptional conflict
situations, the only resolution possible or desirable may be through conquest: a test of strength and the
unambiguous violent defeat of the other side--as of Hitler's Germany. To believe that conflict should always be resolved
through negotiation, mediation, and compromise invites an aggressor to assume that what is his is his, but what is yours is negotiable. Resisting
aggression forces a test of interests, capabilities, and will--if the aggressor so wants it. And this may be a
faster, ultimately less conflictful, less violent way of resolving conflict than conciliation or appeasement. In
resisting aggression, gauge different power responses. Do not automatically respond to aggression in kind. The most effective response is one
which shifts power to bases which can be employed more effectively, while lessening the risk of violent
escalation. And respond proportionally. To meet aggression in equal measure is legitimate, while overreaction risks escalation to a more extended and
intense conflict, and underreaction appears weak and risks defeat and repeated aggression.

YOU CANT IMAGINE THE WORLD AS PEACEFUL THIS SELF DECEPTION BEGETS
MORE VIOLENCE

Laren 2K1 (Carter, Pacificism Empowers Terrorism, Capitalism Magazine, October 4,


http://www.CapMag.com/article.asp?ID= 1128)

Pacifists would argue that they are idealists, as if


being an idealist meant being excused from having to defend those
ideals. Consider an individual engaged in the following line of reasoning: "It would be ideal if all people knew how to perform open- heart surgery, so I am
going to behave as if everyone is a heart surgeon. I am an idealist." Although this may be idealism, it is also idiocy (and self-
destructive). Pacifists think that by pretending that violence doesn't exist, eventually it won't. This is not
just silly; it is a vicious, deadly lie. Aggression cannot be defeated by rewarding it. Organizers of "Don't turn tradgedy
[sic] into a war" rallies across the country would have Americans believe that the proper response to the murder of thousands of innocent lives is a candlelight
vigil and impromptu poetry readings. This
is mass suicide. It is an invitation to the Hitlers, the Stalins, the Attilas, and
the Bin Ladens of the world to slaughter the American people and to gut their corpses. Implicit in the pacifist's drivel
is the implication: "may the worst man win." Only two types of people can accept a philosophy like this: a fiend or a fool. A fiend hates everyone, including
himself, and so doesn't care if the "worst man" wins. A
fool believes that if he smiles sheepishly at Adolf Hitler, Hitler will
suddenly change his mind and decide to take-up knitting. They are both wrong, and they are both evil,
[because in both cases such a policy can only lead to the destruction of the good. To promote this evil in
the wake of the recent terrorist attacks, pacifists have added a few extra deceptions to their arsenal. One of
these is the equation of war and racism. "War and Racism are Not the Answer," reads an anti-war poster at a San Francisco university. This statement blatantly
implies that those who support war against terrorist-harboring nations are racist. It relies on the insecurity of the reader by convincing him to oppose war for
A war against the Afghan, Iranian, and other terrorist-supporting governments
fear of being (unjustly) labeled "racist."
does not constitute racism. It constitutes self-defense. Racism is clearly wrong, but pacifism doesn't hold a
monopoly on that idea.

Page 1163 of 1481


Page 1164 of 1481
Pacifism = Violence (2/3)
PACIFISM CAUSES IMMEDIATE ESCALATION OF VIOLENCE*

Robert L. Phillips, professor of philosophy, War and Justice, 1984, p. 102-3

Narveson argues that what I have called intrinsicalism is a genuine moral position in virtue of its claim that
the use of force in self-defense is in itself evil and that pacifism is incumbent on everyone, not just those
who happen to believe that the use of force is evil. So intrinsicalism satisfies the Kantian test for calling a
particular principle a moral imperative. Narvesons objection to pacifism is directed toward what he
claims are logical inconsistencies in the principle itself. For the belief that the use of force is inherently evil
must minimally entail that people have a right not to be the object of violent attacks. At least part of what
must be meant by saying that a particular action is wrong is that people have a right not to have that sort
of thing done to them; they have a right to take steps to prevent the abridgement of that right. This is an
interesting line of argument because its effectiveness depends not upon an opposed set of moral
principles but upon an analysis of what it means to possess a right in general. If the notion of having a
right is to make any sort of sense, if it is not be be merely an expression, then to say that someone has a
right must also be to say that he is justified in taking steps to prevent that right from being abridged. The
must is a logical must: To say that you have a right to X but no one has any justification whatever for
preventing people from depriving you of it, is self-contradictory. If you claim a right to X, then to describe
some action as an act of depriving you of X, is logically to imply that its absence is one of the things you
have a right to.8 A pacifist might well reply that this logical point, while sound, is not a description of his
position. What Narveson shows is that if a person has a right, [they] cannot also have a duty to be purely
passive in the face of an attack upon that right. Of course, the pacifist will insist that his is not a doctrine
of passivity or fatalism. As a moral agent, he is concerned with ethical imperatives, with guiding action. So
the question becomes not whether a person does or does not have a duty to acquiesce in the removal of
one of his rights but, rather, what level of force is appropriate in face of a violent attack. Narveson might
well reply that the commitment always to refrain from violence is in effect a permission to abrogate his
rights. For if an opponent knows in advance that the pacifist will only resist up to the level of actual
fighting, then [they] will simply escalate his attacks beyond that point. This is precisely what happened
when nonviolent resistance was tried briefly in Nazi-occupied Norway.

**This evidence has been gender modified

Page 1165 of 1481


Page 1166 of 1481
Pacifism = Violence (3/3)
TRUE PEACE NECESSITATES SOME UNAVOIDABLE VIOLENCE VIOLENCE IS KEY TO
DIGNITY AND OTHER VIRTUES

Rummel, Prof of Political Science @ U of Hawaii, 81 (R.J., The Just Peace,


http://www.hawaii.edu/powerkil1s/TJP.CHAP 10.HTM)

Such are major subprinciples of peacemaking. Conflict engages what the parties want and can and will do in a situation in which relevant status quo
expectations are disrupted. Situational perceptions, expectations, interests, capabilities, and will are the elements of the conflict--and of peacemaking.
Material things--land, people, wealth, ports, borders--are merely the tools or objects of conflict. And material conditions, such as the topography of a
country or a mountainous border between states, only frame and physically limit conflict. The essence of conflict is an opposition of minds. The arena of
conflict is the mental field. The principles and rules for its resolution are psychological. Now, peacemaking
is not necessarily the best
and most immediate response to conflict. Doubtlessly, some conflicts are unnecessary, some needlessly intense and long-lasting. But
some also are a real and unavoidable clash, the only means through which one, as a partisan, can protect or further vital interests and achieve a more
war against Hitlers Germany from 1939 to 1945 cost millions lives,
satisfactory and harmonious just peace. For example,
but it prevented the greater misery, the terror, the executions, the cold-blooded murders which
probably would have occurred had Hitler consolidated his control of Europe and subjugated the Soviet
Union. We always can end a conflict when we want by surrender. But some ideas are more important
than peace: Dignity. Freedom. Security. That is, peace with justice--a just peace. There is another relevant
qualification. The term "peacemaking" is well established, and I used it accordingly. Unfortunately, the verb "make" can imply that peace is designed and
constructed, as a house is planned and erected brick by brick or a road engineered and built. This implication is especially seductive in this age when society
is seen as manmade (rather than having evolved),9 and many believe that communities should be centrally planned and managed. But peace is not
constructed like a bridge. Peace emerges from the balancing of individual mental fields. What the leaders of a group or nation honestly believe, actually
want, truly are willing to get, are really capable of achieving are unknown to others--and perhaps only partially to themselves. Nonetheless only they can
best utilize the information available to them to justly satisfy their interests. For a third party to try to construct and enforce an abstract peace imposed on
others is foolhardy. Such a peace would be uncertain, forestall the necessary trial-and-error balancing of the parties themselves, and perhaps even create
greater conflict later. The best peace is an outcome of reciprocal adjustments among those involved. At most, peacemaking should ease the process. A final
qualification. Pacifists
believe that violence and war cannot occur if people laid down their arms and refused
to fight. But this ignores unilateral violence. Under threat, a state or government may try to avoid
violence by submission. The result may be enslavement, systematic execution, and elimination of
leaders and "undesirables." The resulting genocide and mass murder may ultimately end in more
deaths than would have occurred had people fought to defend themselves. I agree that in some situations
nonviolence may be an effective strategy for waging conflict,10 as in the successful Black civil rights demonstrations of the
1960s in America; or the successful nonviolent, civil disobedience movement for Indian independence from Britain begun by Mahatma Gandhi in 1922. In
there are also conflicts,
some situations refusal to use violence may avoid unnecessary escalation and ease peacekeeping. However,
especially involving actual or potential tyrants, despots, and other such oppressors, in which
nonviolence cannot buy freedom from violence by others or a just resolution of a dispute. Then a down
payment on such a peace requires public display of one's capability and a resolve to meet violent
aggression in kind.

Page 1167 of 1481


Pacifism Doesnt Solve Violence

PACIFISM IS NOTHING MORE THAN A FORM OF MORAL EGOISM THAT ALLOWS


INDIVIDUALS TO BE PASSIVE IN THE FACE OF BRUTALITY

Robert L. Phillips, professor of philosophy, War and Justice, 1984, p. 103-4

There is one way of attempting to get around this internal contradiction which intrinsicalism appears to
carry with it. Instead of seeing pacifism as a moral position in the ordinary sense, perhaps we should
understand it as a commitment to an ideal type. The pacifist will concentrate on developing into the kind
of person for whom nonviolence is a permanent part of the soul, and by example he will encourage others
to do the same. The pacifist would admit that the world does contain men who commit violent attacks
upon others, but his concern will be to demonstrate by his own example that an alternative way of life is
possible: men do not have to take life; they do not have to adopt the posture of the utilitarian bargainer.
This kind of saintliness does, however, seem irresponsible. The unwillingness of the pacifist to dirty his
hands is no doubt the source of the charge that he is more concerned about the state of his soul than
with the preservation of life. The unwillingness to kill or injure may be part of the pacifists very being, but
what happens to his respect for life defense when his refusal to fight causes loss of lives which could
have been saved? Critics of the argument that pacifism is part of a program to attain an ideal of self-hood
respond with the charge of moral egoism. It [moral egoism] differs from ordinary egoism only in its
allegedly spiritual quality. It is a thoroughgoing refusal to dirty ones own hands.... I suggest that those
whose concerns are thus limited are warped, self-righteous and ultimately self-serving. The pacifist
saint who stands by while others are being murdered or brutalized.., how does he differ from a moral
idiot, except in point of pretentiousness?

Page 1168 of 1481


Page 1169 of 1481
Pacifist Activism Fails: General
NONVIOLENT CHANGE IS TEMPORARY

Brian Martin, STS U of Wollongong 2001Nonviolence Versus Capitalism


http://www.uow.edu.au/arts/sts/bmartin/pubs/01nvc/nvcall.html

It is important to note that not all uses of nonviolent action lead to long-lasting, worthwhile change.
Nonviolent action is not guaranteed to succeed either in the short term or long term. The 1989
prodemocracy movement in China, after a short flowering, was crushed in the Beijing massacre. Perhaps
more worrying are the dispiriting aftermaths following some short-term successes of nonviolent action. In
El Salvador in 1944, the successful nonviolent insurrection against the Martnez dictatorship did not lead
to long term improvement for the El Salvadorean people. There was a military coup later in 1944, and
continued repression in following decades. The aftermath of the Iranian revolution was equally
disastrous. The new Islamic regime led by Ayatollah Khomeini was just as ruthless as its predecessor in
stamping out dissent.

ABSOLUTE PACFICISM IS IMPOSSIBLE

Mohandes Gandhi, as quoted in The Pacifist Conscience, ed. by Peter Mayer, 1966, p 214

I do believe that, where there is only a choice between cowardice and violence, I would advise violence.
Thus when my eldest son asked me what he should have done, had he been present when I was almost
fatally assaulted in 1908, whether he should have nm away and seen me killed or whether he should have
used his physical force which he could and wanted to use, and defended me, I told him that it was his
duty to defend me even by using violence. Hence it was that I took part in the Boer War, the so called Zulu
rebellion and the late War. Hence also do I advocate training in arms for those who believe in the method
of violence. I would rather have India resort to aims in order to defend her honour than that she should in
a cowardly manner become or remain a helpless witness to her own dishonour.

NONVIOLENCE ALONE FAILS.

Brian Martin STS @ U of Wollongong 2001Nonviolence Versus Capitalism


http://www.uow.edu.au/arts/sts/bmartin/pubs/01nvc/nvcall.html

The consent theory of power Gandhi approached nonviolent action as a moral issue and, in practical
terms, as a means for persuading opponents to change their minds as a result of their witnessing the
commitment and willing sacrifice of nonviolent activists. While this approach explains some aspects of the
power of nonviolent action, it is inadequate on its own. Moral persuasion sometimes works in face-to-
face encounters, but has little chance when cause and effect are separated. Bomber pilots show little

Page 1170 of 1481


remorse for the agony caused by their weapons detonating far below,[24] while managers of large
international banks have little inkling of the suffering caused by their lending policies in foreign countries.

Page 1171 of 1481


Page 1172 of 1481
Pacifist Activism Fails: Law is
Violent
ALL LAW IS VIOLENCE. THE CRITIQUE OF MILITARISM FAILS.

Jacques Derrida, in Deconstruction and the Possibility of Justice, Drucilla Cornell, ed, 92, p. 40-2.

To discuss the conservative violence of law, Benjamin sticks to relatively modern problems, as modern as
the problem of the general strike was a moment ago. Now it is a question of compulsory military service,
the modern police or the abolition of the death penalty. If, during and after World War I, an impassioned
critique of violence was developed, it took aim this time at the law-conserving form of violence.
Militarism, a modern concept that supposes the exploitation of compulsory military service, is the forced
use of force, the compelling (twang) to use force or violence (Gewalt) in the service of the state and its
legal ends. Here military violence is legal and conserves the law, and thus it is more difficult to criticize
than the pacifists and activists believe; Benjamin does not hide his low esteem for these declaimers. The
ineffectiveness and inconsistency of anti-military pacifists results from their failure to recognize the legal
and unassailable character of this violence that conserves the law. Here we are dealing with a double bind
or a contradiction that can be schematized as follows. On the one hand, it appears easier to criticize the
violence that founds since it cannot be justified by any preexisting legality and so appears savage. But on
the other hand, and this reversal is the whole point of this reflection, it is more difficult, more illegitimate
to criticize this same violence since one cannot summon it to appear before the institution of any
preexisting law: it does not recognize existing law in the moment that it founds another. Between the two
limits of this contradiction, there is the question of this ungraspable revolutionary instant that belongs to
no historical, temporal continuum but in which the foundation of a new law nevertheless plays, if we may
say so, on something from an anterior law that it extends, radicalizes, deforms, metaphorizes or
metonymizes, this figure here taking the name of war or general strike. But this figure is also a
contamination. It effaces or blurs the distinction, pure and simple, between foundation and conservation.
It inscribes iterability in originarity, in unicity and singularity, and it is what I will call deconstruction at
work, in full negotiation: in the "things themselves"and in Benjamin's text. As long as they do not give
themselves the theoretical or philosophical means to think this co-implication of violence and law, the
usual critiques remain naive and ineffectual. Benjamin does not hide his disdain for the declamations of
pacifist activism and for the proclamations of "quite childish anarchism" that would like to exempt the
individual from all constraints. The reference to the categorical imperative ("Act in such a way that at all
times you use humanity both in your person and in the person of all others as an end, and never merely as
a means," p. 285), however uncontestable it may be, allows no critique of violence. Law (droit) in its very
violence claims to recognize and defend said humanity as end, in the person of each individual. And so a
purely moral critique of violence is as unjustified as it is impotent. For the same reason, we cannot
provide a critique of violence in the name of liberty, of what Benjamin here calls "gestaltlose Freiheit,"
"formless freedom," that is, in short, purely formal, as empty form, following a Marxist-Hegelian vein that
is far from absent throughout this meditation. These attacks against violence lack pertinence and
effectiveness because they remain alien to the juridical essence of violence, to the Rechtsordnung, the
order of law (droit). An effective critique must lay the blame on the body of droit itself, in its head and in
its members, in the laws and the particular usages that law adopts under protection of its power (Macht).
This order is such that there exists one unique fate or history (nur ein einziges Schicksal, "only one fate," p.
285). That is one of the key concepts of the text, but also one of the most obscure, whether it's a question
of fate itself or of its absolute uniqueness. That which exists, which has consistency (das Bestehende) and
that which at the same time threatens what exists (das Drohende) belong inviolably (unverbriichlich) to
the same order and this order is inviolable because it is unique. It can only be violated in itself. The notion
of threat is important here but also difficult, for the threat doesn't come from outside. Law is both

Page 1173 of 1481


threatening and threatened by itself: This threat is neither intimidation nor dissuasion, as pacifists,
anarchists or activists believe. The law turns out to be threatening in the way fate is threatening. To reach
the "deepest meaning" of the indeterminacy (Unbestimmtheit, "uncertainty," p. 28S) of the legal threat
(der Rechtsdrohung), it will later be necessary to meditate upon the essence of fate at the origin of this
threat.

Page 1174 of 1481


Page 1175 of 1481
Pacifist Activism Fails: Final
Solution (1/3)
JEWISH PACIFISM IN THE FACE OF THE FINAL SOLUTION IS THE ULTIMATE PROOF
OF THE INADEQUACY OF NON-VIOLENT RESISTANCE

Ward Churchill, (Keetoowah Band Cherokee) Professor of Ethnic Studies and Coordinator of American Indian

Studies at University of Colorado, 2001, Pacifism as Pathology, p. 32-37

Pacifism possesses a sublime arrogance in its implicit assumption that its adherents can somehow dictate
the terms of struggle in any contest with the state. Such a supposition seems unaccountable in view of
the actual record of passive/nonviolent resistance to state power. Although a number of examples can be
mustered with which to illustrate this point including Buddhist resistance to U.S. policies in Indochina,
and the sustained efforts made to terminate white supremacist rule in southern Africa none seems
more appropriate than the Jewish experience in Hitlerian Germany (and later in the whole of occupied
Europe). The record is quite clear that, while a range of pacifist forms of countering the implications of
nazism occurred within the German Jewish community during the l930s, they offered virtually no physical
opposition to the consolidation of the nazi state. To the contrary, there is strong evidence that orthodox
Jewish leaders counseled social responsibility as the best antidote to nazism, while crucial political
formulations such as the Zionist Hagana and Mossad el Aliyah Betactually seem to have attempted to co-
opt the nazi agenda for their own purposes, entering into cooperative relations with the SS Jewish Affairs
Bureau, and trying to use forced immigration of Jews as a pretext for establishing a Jewish homeland in
Palestine.8 All of this was apparently done in an effort to manipulate the political climate in Germany
by not exacerbating conditions and not alienating the German people any further in a manner
more favorable to Jews than the nazis were calling for.~ In the end, of course, the nazis imposed the final
solution to the Jewish question, but by then the dynamics of passive resistance were so entrenched in
the Jewish zeitgeist (the nazis having been in power a full decade) that a sort of passive accommodation
prevailed. Jewish leaders took their people, quietly and nonviolently, first into the ghettos, and then onto
trains evacuating them to the east. Armed resistance was still widely held to be irresponsible.
Eventually, the SS could count upon the brunt of the nazi liquidation policy being carried out by the
Sonderkommandos, which were composed of the Jews themselves. It was largely Jews who dragged the
gassed bodies of their exterminated people to the crematoria in death camps such as
Auschwitz/Birkenau, each motivated by the desire to prolong his own life. Even this became rationalized
as resistance; the very act of surviving was viewed as defeating the nazi program. By 1945, Jewish
passivity and nonviolence in the face of the weltanschauung der untermenschen had done nothing to
prevent the loss of millions of lives. The phenomenon sketched above must lead to the obvious question:
[How could] millions of men [sic] like us walk to their death without resistance? In turn, the mere asking
of the obvious has spawned a veritable cottage industry among Jewish intellectuals, each explaining how
it was that the process had left the Jewish people no choice but to go along, to remain passive, to
proceed in accordance with their aversion to violence right up to the doors of the crematoria and
beyond. From this perspective, there was nothing truly lacking in the Jewish performance; the Jews
were simply and solely blameless victims of a genocidal system over which it was quite impossible for
them to extend any measure of control. The Jews having suffered horribly under nazi rule,6 it has come
to be considered in exceedingly poor taste antisemitic, according to the logic of the Anti-Defamation
League of Bnai Brith to suggest that there was indeed something very wrong with the nature of the
Jewish response to nazism, that the mainly pacifist forms of resistance exhibited by the Jewish community
played directly into the hands of their executioners. Objectively, there were alternatives, and one need
not look to the utterances of some lunatic fringe to find them articulated. Even such a staid and
conservative political commentator as Bruno Bettelheim, a former concentration camp inmate, has

Page 1176 of 1481


offered astute analysis of the role of passivity and nonviolence in amplifying the magnitude of the
Holocaust. Regarding the single known instance

Page 1177 of 1481


Page 1178 of 1481
Pacifist Activism Fails: Final
Solution (2/3)
CONTINUED

in which inmates physically revolted at Auschwitz, he observes that: In the single revolt of the twelfth
Sonderkommando, seventy SS were killed, including one commissioned officer and seventeen non-
commissioned officers; one of the crematoria was totally destroyed and another severely damaged. True,
all eight hundred and fifty-three of the kommando died. But. . . the one Sonderkommando which revolted
and took such a heavy toll of the enemy did not die much differently than all the other
Sonderkommandos. Aside from pointing out that the Jews had literally nothing to lose (and quite a lot to
gain in terms of human dignity) by engaging in open revolt against the 55, Bettelheim goes much further,
noting that such actions both in and outside the death camps stood a reasonable prospect of greatly
impeding the extermination process. He states flatly that even individualized armed resistance could have
made the Final Solution a cost-prohibitive proposition for the nazis: There is little doubt that the [Jews],
who were able to

provide themselves with so much, could have provided themselves with a gun or two had they wished.
They could have shot down one or two of the SS men who came for them. The loss of an SS with every
Jew arrested would have noticeably hindered the functioning of the police state.2 Returning to the revolt
of the twelfth Sonderkommando, Bettelheim observes that: They did only what we should expect all
human beings to do; to use their death, if they could not save their lives, to weaken or hinder the enemy
as much as possible; to use even their doomed selves for making extermination harder, or maybe impos-
sible, not a smooth-running process If they could do it, so could others. Why didnt they? Why did they
throw their lives away instead of making things hard for the enemy? Why did they make a present of their
very being to the SS instead of to their families, their friends, even to their fellow prisoners[?] Rebellion
could only have saved either the life they were going to lose anyway, or the lives of others. . Inertia it was
that led millions of Jews into the ghettos the SS had created for them. It was inertia that made hundreds
of thousands of Jews sit home, waiting for their executioners. Bettelheim describes this inertia, which he
considers the basis for Jewish passivity in the face of The persecution of the Jews was aggravated, slow
step by slow step, when no violent fighting back occurred. It may have been Jewish acceptance, without
retaliatory fight, of ever harsher discrimination and degradation that first gave the SS the idea that they
could be gotten to the point where they would walk into the gas chambers on their own . . . [I] n the
deepest sense, the walk to the gas chamber was only the last consequence of the philosophy of business
as usual. Given this, Bettelheim can do little else but conclude (correctly) that the post-war rationalization
and apologia for the Jewish response to nazism serves to stress how much we all wish to subscribe to
this business as usual philosophy, and forget that it hastens our own destruction, to glorify the attitude of
going on with business as usual, even in a holocaust.

THE FINAL SOLUTION PROVES THE TOTAL FAILURE OF PACIFISM

Ward Churchill, (Keetoowah Band Cherokee) Professor of Ethnic Studies and Coordinator of American Indian

Studies at University of Colorado, 2001, Pacifism as Pathology, p. 40-

One may assume for the moment that such a gross distortion of reality is hardly the intent of even the hardiest pacifist polemicists, although it may well be an intrinsic aspect of

their position. Worse than this is the Inconsistency of nonviolent premises. For instance, it has been abundantly documented that nazi policy toward the Jews,

Page 1179 of 1481


from 1941 onward, was bound up in the notion that extermination would proceed until such time as the entire
Jewish population within German occupied territory was liquidated?~ There is no indication whatsoever that nonviolent
intervention/mediation from any quarter held the least prospect of halting, or even delaying, the genocidal process.
To the contrary there is evidence that efforts by neutral parties such as the Red Cross had the effect of speeding

up the slaughter. That the Final Solution was halted at a point short of its full realization was due solely to the massive
application of armed force against Germany (albeit for reasons other than the salvation of the Jews). Left to a pacifist prescription for the altering of
offensive state policies, and the effecting of positive social change, World Jewry at least in its Eurasian variants would have offered total extermination by mid-1946 at

. Even the highly symbolic trial of SS Colonel Adolph Eichmann could not be accomplished by nonviolent
the latest

means, but required armed action by an Israeli paramilitary unit fifteen years after the last death camp was closed by Russian tanks.
There is every indication that adherence to pacifist principles would have resulted in Eichmanns permanent avoidance of justice, living out his life in reasonable comfort until

With reference to the Jewish


to paraphrase his own assessment he leapt into the grave laughing at the thought of having killed six million Jews.

experience, nonviolence was a catastrophic failure, and only the most extremely violent intervention by
others saved Europes Jews at the last moment from slipping over the brink of utter extinction. Small wonder that the survivors insist, Never
again!

Page 1180 of 1481


Pacifist Activism Fails: Final Solution (3/3)

NON VIOLENCE WOULD HAVE HAD NO CHANCE TO STOP THE NAZIS DENMARKS
STRATEGY WOULDNT HAVE WORKED ON A GLOBAL SCALE A THOUSAND YEAR
REICH WOULD OF RESULTED

J. A. H. Futterman, Ph.D. from UT-Austin and Physicist at the University of California's Lawrence Livermore
National Laboratory, Obscenity and Peace: Meditations on the Bomb, 1990-94,
http://www.dogchurch.com/scriptorium/nuke.html, UK: Fisher

That said, I admit that I admire non-violent resistance. [4] Remember, however, that non-violent
resistance is a sophisticated technique that works only when used by the "right" people at the "right"
time against the "right" opponents. For example, the Indians successfully used non-violent resistance to
persuade the British to end the Raj, because the British eventually acknowledged that the Indians, led by
the British-educated Gandhi, were human beings like themselves.

The Nazis, who with their "Master Race" ideology admitted only so-called "Aryans" to the category of
human, provide an example counter to that of the British. There were some successful acts of non-violent
confrontation against the Nazis, like King Christian of Denmark's public declaration that he would wear
the yellow star if it were introduced in his country. He did so in response to the Nazi practice of ordering
Jews to wear yellow-starred armbands so that the Nazis could more easily isolate them from their
surrounding society. That many Danes followed their king's example helped camouflage many Jews until
they could escape to Sweden in fishing boats. [5] Now this resistance worked partly because the Nazis
considered the Danes to be "Aryans" like themselves. Had the Poles tried the same thing, the Nazis would
have been perfectly happy to use the event as an excuse for liquidating more Poles. Rather than awaken
the Nazis' moral sense, non-violent confrontation on the part of the Poles would probably have enabled
the Nazis to carry out their agenda in Poland more easily. The other reason these acts succeeded was that
overwhelming violence of the Allies had stretched the Nazi forces too thin to suppress massive action by a
whole populace, and eventually deprived the Nazis of the time they needed to find other ways to carry
out their "final solution."

In other words, non-violence resistance alone would have been very slow to work against the Nazis, once
they had consolidated their power. And while it slowly ground away at the evil in the Nazi soul, how many
millions more would have died, and how much extra time would have been given to Nazi scientists trying
to invent atomic bombs to go on those V-2 rockets? The evil of Nazism may well have expended itself, but
perhaps after a real "thousand-year Reich," leaving a world populated only by blue-eyed blondes. In other
words, if the world had used non-violence alone against the Nazis, the results may have been much worse
those of the war.[6]

Page 1181 of 1481


Page 1182 of 1481
Civil Disobedience Fails (1/2)
CIVIL DISOBEDIENCE CAUSES MARGINALIZATION, DISPROPORTIONAL
PUNISHMENT, AND FAILS TO HAVE TRANSFORMATIVE EFFECTS

Rachael E. Schwartz, J.D. 1981 Georgetown University Law Center, BOSTON UNIVERSITY
INTERNATIONAL LAW JOURNAL, Fall 1994, p. 256-257.

Recent decades have seen an impressive expansion of the extent to which international law recognizes
fundamental individual rights. Public international law no longer concerns itself solely with the subject of
relations between states. However, the establishment of a law of individual remedies and the
accompanying enforcement institutions to assure vindication of these rights has not kept as swift a pace
as the recognition of the existence of the rights themselves. As a result, many people have bare rights
with no legal substantive basis or process for obtaining redress for violations of these rights. Faced with
such a state of affairs, several options are available to the aggrieved individual. One may simply resign
one's self to one's fate. While it is understandable that some may take this route as a means of short-term
self-preservation, it is unacceptable to many. Alternatively, one may attempt to avail one's self of those
limited legal avenues which are open. This is admirable, but ultimately may well prove a futile risk; the
individual may gain nothing only to become known to the government as a "troublemaker." A third
possibility is civil disobedience; that is, open and non-violent breaking of the law of a state with voluntary
acceptance of such punishment as may be imposed pursuant to that law. Prominent practitioners of civil
disobedience include Henry David Thoreau, Mohandas Gandhi, and Martin Luther King, Jr. This too has
obvious risks: there may be little chance that the punishment will be fair or that those in authority will be
persuaded by a display of moral integrity to correct their behavior rather than retaliate.

Nonviolent change is temporary. The 1989 prodemocracy movement in China was crushed in
the Beijing massacre. In El Salvador 1944, the nonviolent insurrection against the Martnez
dictatorship didnt lead to long term improvement. Iranian non-violent revolutions have been
ultimately unsuccessful, as was Jewish pacifism. Ghandis non-violence was only successful in
the context of global armed resistance to British colonialism. For every Martin Luther King,
there is a counter-example

Page 1183 of 1481


Page 1184 of 1481
Civil Disobedience Fails (2/2)
WE MUST BE PRAGMATIC PACIFISTS TO END STATE VIOLENCEABSOLUTE
PACIFISM FAILS TO CHALLENGE THE POLICIES OF THE STATE BY OPTING OUT OF
THE GAME ENTIRELY

Robert L. Phillips, professor of philosophy, War and Justice, 1984, p. 114-6

It conceivable that governments might grant selective objection the same legal status as it gives
to pacifism? The answer, I fear, is no. And that tells us something important about pacifism.
Governments are prepared to tolerate pacifism, because it poses no threat either to their
political policies or to the manner in which wars are conducted. The pacifist objects equally to
all wars waged by all governments. In this sense he opts out of the game altogether. By
contrast, the selective objector will be forced to analyze both the policy decisions of the
government as well as the conduct of the armed forces. He will be publicly carrying out an
officially sanctioned comparison between mutually agreed just-war criteria and the actual
performance of the government. That is a lot to expect of governments as we know them, but there is still more. What would be the implication of a
state granting an exemption on selective grounds? Fundamentally, the state would be agreeing with the claim that its war policies may be reasonably interpreted as unjust. The
belief that all war is wrong is a proposition which states might agree is debatable among rational men, and, therefore, claims to exemption on this basis may be allowed. It is a
very different matter, however, to grant exemption for a particular war, for here we are faced not with two philosophical theories about violence but with a factual dispute. Selec-
tive objection presupposes that both the government and the claimant agree upon the criteria for undertaking a justified war and the rules for conducting it. The claimant would
have to show, in order to qualify for an exemption, that his government is engaged in acts of war which a person might reasonably characterize as immoral. As such an
admission is inseparable from policy questions, it is inconceivable that any government would be willing (or politically able) to wage war while publicly agreeing that there is
sufficient reason to doubt the morality of that war to grant exemptions from it. This is not to say that individuals should not refuse to fight in wars which they believe are immoral

of the pacifist who objects to all


but to acknowledge that governments cannot be expected to institutionalize such a practice. The evenhandedness

wars does not threaten the particular policies of any state. In condemning them all equally,
pacifism exempts itself from political reality: What is needed, then, is not a general pacifism
but a discriminating conscientious refusal to engage in war in certain circumstances. States
have not been loath to recognize pacifism and to grant it a special status. The refusal to take
part in all war under any conditions is an unworldly view bound to remain a sectarian doctrine.
It no more challenges the states authority than the celibacy of priests challenges the sanctity
of marriage. By exempting pacifists from its prescriptions the state may even seem to display a
certain magnanimity. But conscientious refusal based upon the principles of justice as they
apply to particular conflicts is another matter. For such refusal is an affront to the
governments pretensions, and when it becomes widespread, the continuation of an unjust war
may prove impossible.

THEIR NOTION OF DISOBEDIENCE CEMENTS THE STATES MONOPOLY OF VIOLENCE

Jordan J. Paust, Professor of Law, University of Houston, EMORY LAW JOURNAL, Spring 1983, p. 549-550.

With such a focus, one should discover that private individuals and groups can and do engage
in numerous forms of permissible violence. It is too simplistic to say, therefore, that
authoritative violence can only be engaged in by "the government" or by governmental elites
and functionaries. As Professor Reisman stated, the notion that only state institutions can

Page 1185 of 1481


permissibly use high levels of violent coercion "is a crucial self-perception and deception of
state elites." Thus, the useful question is not whether private violence is permissible, but what forms of private violence are permissible, when, in what social context,
and why. As Professor Reisman further suggests: [I]nsistence on non-violence and deference to all established

institutions in a global system with many injustices can be tantamount to confirmation and
reinforcement of those injustices. In certain circumstances, violence may be the last appeal or the first expression of demand of a group or
unorganized stratum for some measure of human dignity. Of course, such an injunction can also have particular relevance concerning the question of revolutionary social

violence. Here, as elsewhere , no facile "rule" or simplistic prohibition will do.

Page 1186 of 1481


A2 Violence Alienates the People: 2AC

PASSIVE PROTEST DOESNT WIN THE MASSES-VIOLENCE CHANGES THE LEVEL OF


CONSCIOUSNESS

Mike Ryan, Canadian anti-imperialist, 2001, Pacifism as Pathology, p. 134-135

Turning now to the argument that violence alienates the people, I find myself face to face with several
unanswerable flaws of logic. If violence alienates the people, are we to refrain from engaging in any but
passive acts of protest (and here I use the term protest rather than resistance quite consciously) because
this will win popular support? If this is the case, I am forced to ask why, after years of consistent
nonviolent protest, no qualitative growth, and only the slightest quantitative, has occurred within our
movement? From these questions, I would go on to suggest that catering our activity to our perception
(which might not even be accurate) of the level of resistance acceptable to people, far from being
revolutionary, is in fact counter to the development of revolutionary Consciousness: A party (or, in our
case, an organization or movement) which bases itself on an existing average level of consciousness and
activity, will end up reducing the present level of both. It is the partys responsibility to lead, to change the
existing level of consciousness and activity, raise them to higher levels) It is clear that the peace
movement, rather than offering vital connections and a direction for popular discontent (which plainly
exists), has failed to offer anything more than a repetitive and increasingly boring spectacle. The
government in Ottawa, and the general populus, has increasingly taken to yawning at our activities.

Page 1187 of 1481


A2 Non-Violence Key to Prevent Eradication of
Movement: 2AC

NON-VIOLENCE ONLY AVOIDS PERSECUTION BECAUSE IT IS INEFFECTIVEANY


SUCCESSFUL REVOLUTION HAS TO DEAL WITH THE VIOLENCE OF THE STATE

Mike Ryan, Canadian anti-imperialist, 2001, Pacifism as Pathology, p. 135-136

The argument that violence brings repression down on the left indicates a naivete bordering on sheer
madness. Do we really believe that if we could devise a nonviolent means of eliminating the state we
would be allowed to proceed unhindered in carrying it out? The state is violent in its very nature. The
police, the army, and prisons stand as immediate, tangible evidence of this. The genocide of Third and
Fourth World peoples stands as evidence of this. Canadas role as an arms producer and supplier for the
Indonesian colonization of East Timor is a daily, ongoing act of violence. Violence, overt and covert,
aggressive and preventive, is fundamental to the function of the Canadian state. No violence issuing from
the movement could hope to be more than a pale reflection of the constant violence of the repressive
apparati. That this violence generally remains invisible is more a statement of our failure than of our
success, a reflection of the degree to which we have remained within the limits acceptable to the state. As
Mao said in 1939: It is good and not bad if the enemy fights against us: I think it is bad for us be it for
individual, a party, an army, or a school of thought if the enemy does not take a stand against us,
because in that case it could only mean that we are hand in glove with the enemy. If we are being fought
by the enemy, then that is good: it is proof that we have drawn a clear line between us and the enemy. If
the enemy goes vigorously into action against us, and accepts nothing at all, then this is even better: it
shows that we have not only drawn a clear line between us and the enemy, but that our work has
achieved tremendous success.

Page 1188 of 1481


Page 1189 of 1481
Pacifism Bad: War Good (1/2)
ATTEMPTS AT PACIFISM FAIL EVIL EXISTS IN THE WORLD ATTEMPTS AT
UTOPIANISM ARE SUICIDAL

Adam G. Mersereau, Served in the enlisted and officer ranks of the United States Marine Corps from 1990 to
1995; now an attorney, Down with the Peace Movement: The trouble with the antiwar warriors, National Review
Online, January 15, 2003, http://www.nationalreview.com/comment/comment-mersereau011503.asp

Many members of the peace movement also hold tightly to a loosely defined utopianism. They believe
that the human race (save conservative Republicans) is evolving toward a higher and more noble plane of
social existence. The activists themselves are, of course, at the forefront of the evolutionary curve; while
the Cro-Magnon in the White House and his Cabinet of Neanderthals stubbornly resist progress. Although
the Left has largely declared the concepts of "good" and "evil" to be pass, the peace activist believes that
the heart of man is intrinsically "good," and that it would be "evil" if we do not give Saddam Hussein every
chance to let his goodness shine through.

Utopianism is dead in the minds of most people, because as veterans of the 20th century, which was the
bloodiest century ever, we cannot deny that "good" and "evil" are entangled within the hearts of men and
many of his ideologies, and that peace is little more than a welcome respite between wars. We also
know that unless the Saddam Hussein's and Kim Jong-il's of the world are Utopians too, then to champion
utopianism in America or Europe is useless. Utopianism is folly; unilateral utopianism is suicidal. But
rather than adjust their policy to reflect reality, the peace activists will march in circles, carry their signs,
and wait for reality to reflect their policy.

PACIFISM THE WORST OF ALL WORLDS CAUSES MILLIONS OF DEATHS,


APPEASEMENT OF ENEMIES, AND THERE IS NO ALTERNATIVE

Alex Epstein, Graduate of Duke University, BA Philosophy, Junior fellow at the Ayn Rand Institute, Peacenik
Warmongers, Ayn Rand Institute, December 9, 2002,
http://www.aynrand.org/site/News2?page=NewsArticle&id=7458, UK: Fisher

Pacifism necessarily invites escalating acts of war against anyone who practices it.

There is an increasingly vocal movement that seeks to engage America in ever longer, wider, and more
costly wars--leading to thousands and perhaps millions of unnecessary deaths. This movement calls itself
the "anti-war" movement. Across America and throughout the world, "anti-war" groups are staging
"peace rallies" that attract tens and sometimes hundreds of thousands of participants, who gather to
voice their opposition to an invasion of Iraq and to any other U.S. military action in the War on Terrorism.
The goal of these rallies, the protesters proclaim, is to promote peace. "You can bomb the world to
pieces," they chant, "but you can't bomb it into peace." If dropping bombs won't work, what should the
United States do to obtain a peaceful relationship with the numerous hostile regimes, including Iraq, that
seek to harm us with terrorism and weapons of mass destruction? The "peace advocates" offer no
answer. The most one can coax out of them are vague platitudes (we should "make common cause with
the people of the world," says the prominent "anti-war" group Not in Our Name) and agonized soul-

Page 1190 of 1481


searching ("Why do they hate us?"). The absence of a peacenik peace plan is no accident. Pacifism is
inherently a negative doctrine--it merely says that military action is always bad. As one San Francisco
protestor put the point: "I don't think it's right for our government to kill people." In practice, this leaves
the government only two means of dealing with our enemies: to ignore their acts of aggression, or to
appease them by capitulating to the aggressor's demands.

Page 1191 of 1481


Page 1192 of 1481
Pacifism Bad: War Good (2/2)
PACIFISM MAKES WARS BIGGER AND LONGER HARDLINE MILITARISM
WOULDVE PREVENTED THE RISE OF HITLER

J. A. H. Futterman, Ph.D. from UT-Austin and Physicist at the University of California's Lawrence Livermore
National Laboratory, Obscenity and Peace: Meditations on the Bomb, 1990-94,
http://www.dogchurch.com/scriptorium/nuke.html, UK: Fisher

Of course, if deterrence is not enough, if your opponent is that crazy, what do you do? Running away may
work for individuals, but not for nations, so I will neglect that option. Negotiation is also unworkable,
because you can't reason with bullies. They exhibit a kind of willful mindlessness, a demonic will to
unconsciousness. They don't negotiate back, they merely use your forbearance to buy time and
opportunity to get at you, or to get around you like Hitler did, while Chamberlain declared, "peace in
our time." You assert your position, and set some limits. And if they exceed your limits, you use force.

But is it moral to use force? Those of us who might contemplate calling the police in order to stop a
murder must believe that occasionally it is. Further, I maintain that sometimes it may be immoral to do
anything else. Remember that Hitler could have been stopped easily by a show of force when he
threatened to annex the Sudetenland. That force was not brought to bear in a timely manner is due
largely to the pacifist sentiment in Europe and America at the time. Instead of engaging in a minor
military expedition which would have forced Hitler to back down, to lose face, and ultimately to lose
political power, the world passively sold out Czechoslovakia to him, paving the way for a much more
prolonged and bloody conflict later a conflict that resulted in the development of the first atomic
bombs. In other words, I think a reflexive pacifism is no more entitled to a presumption of moral
innocence than nuclear weapons work, and that pacifism applied in the wrong way at the wrong time
contributed to the development of the nuclear weapons that pacifists now find so abhorrent. In short,
pacifism can sometimes help to make wars bigger and worse than they have to be.

Page 1193 of 1481


Page 1194 of 1481
Pacifism Bad: Unethical
PACIFISM EQUATES DEFENDANT TO AGGRESSOR, RAPIST TO RAPE VICTIM
DESTROYS ABILITY TO MAKE NORMATIVE JUDGMENTS ABOUT VIOLENCE THAT
ARE CRITICAL TO SAFETY

Kevin Delaney, Freelance Writer in Los Angeles, Debunking the Clichs of Pacifism, Capitalism Magazine,
October 13, 2001, http://www.capmag.com/article.asp?ID=1157, UK: Fisher

The philosophy of pacifism can be expressed in a single principle: "The use of force is morally wrong." This
means that ALL force - any kind of force - is out of the question and must be opposed. If you spend any
amount of time thinking about the issue (which most pacifists do not), you'll very quickly be able to think
of a number of situations in which the use of force is clearly not only not morally wrong, but clearly
necessary - a woman fighting off a rapist, for example. Take a few moments to come up with several such
"exceptions," then abstract their common element, and you'll arrive at the ominous error at the root of
the pacifist philosophy: pacifism makes no distinction between force which is initiated, and force which is
used in self-defense. Were a pacifist totally consistent in his philosophy, he'd have to say that the woman
who fights off the rapist is wrong to do so - after all, she's certainly committing an act of force. If the
pacifist were also consistent in his use of clichs, he'd say that in fighting the rapist off, the woman has
"sunk to the rapist's level." She has "resorted to violence," and is now "just like him." This same thought
process (or lack of it) is behind the pacifists' opposition to war - specifically, in the case of our current
situation, the opposition to a country fighting back when war has been initiated against it. To the pacifist,
attacker and victim are moral equals. Which side initiated the war is of no interest to him; his mind knows
only the abstraction "war," and that he's against it. Pacifism used to be known as "nonresistance," which
names the heart of the matter: total passivity and surrender when faced with any kind of threat. Of
course, you never hear the position stated this way: today's pacifists almost always make their case
exclusively in terms of what they're against, rarely what they're for (except in the most general sense,
such as "world peace," etc.). Full-fledged pacifists are relatively rare, yet their clichs are nevertheless
having an effect on many minds, throwing monkey-wrenches into people's convictions at a time when this
country needs every ounce of moral certainty it can muster. Over the past few weeks, I'm sure you've
heard at least once, something to the effect of: "If we bomb our enemies, we'll just be doing to them
what they did to us. We'll be sinking to their level!" If you understand the pacifists' basic error, you can
see very clearly what's wrong with this picture: the failure to differentiate between the force of an
aggressor, and force used in retaliation against the aggressor in self-defense. No, it's not morally wrong to
fight back against someone who's attacking you; if you value your life, it's absolutely essential that you do.

Page 1195 of 1481


Pacifism Causes Oppression

THEIR ARGUMENT EQUATES TO SUPPORT FOR OPPRESSIVE DICTATORSHIPS

Steven Brockerman, Assistant Editor for Capitalism Magazine, Pacifists and Professors of Oppression,
Capitalism Magazine, October 12, 2001, http://www.capmag.com/article.asp?ID=1154, UK: Fisher

Righteous in their indignation against the use of American military force, the pacifists and professors,
nonetheless, willingly accept the preemptive annihilation of an entire city in Syria by that nation's
despotic ruler. They who speak intolerantly of American racism are, nevertheless, willing to tolerate the
slave trade thriving in Sudan. Avowed defenders of the Palestinians, they look the other way when those
who even mildly publicly criticize Arafat have their tongues cut out or worse. About all that oppression
they are silent. What those pacifists and professors are not silent about, though, is their opposition to
America's right to self-defense. They who tolerate Mid-Eastern Arab tyrannies are not silent in their
intolerance of America. The pacifists and the professors cannot accept that America -- a nation they hate
with the religious fervor of an Islamic terrorist -- is morally right and, thus, morally superior; therefore,
they are willing to grimly evade not only the reason America was attacked on September 11, but also the
reason they -- the pacifists and the professors -- are attacking America now. America was attacked, not
because the U.S. has oppressed the Arab people, but because the U.S. represents the greatest threat to
those Mid-East dictatorships that do and, thus, represents the greatest hope to the Arab peoples. America
is now being attacked by many in the universities, not because the U.S. is racist or imperialist, but because
the U.S. stands for individual rights, capitalism and the pursuit of happiness, which are the greatest
rebukes to the beliefs of the pacifists and the professors and anyone else whose ideas make possible and
then excuse dictatorship, poverty and oppression.

Page 1196 of 1481


Page 1197 of 1481
Pacifism Causes Aggression (1/2)

PEACE MOVEMENTS ULTIMATELY FAIL EVEN IF THE US TURNS MORE PEACEFUL,


OTHER ENEMIES WONT

Adam G. Mersereau, Served in the enlisted and officer ranks of the United States Marine Corps
from 1990 to 1995; now an attorney, Down with the Peace Movement: The trouble with the antiwar
warriors, National Review Online, January 15, 2003,
http://www.nationalreview.com/comment/comment-mersereau011503.asp

Peace activists may be well intentioned; but at their worst, they are more helpful to America's enemies
than to America. The best we can say is that they are clinically nave. They are as insufferable as a college
freshman who believes he and his political-science professor can end poverty if only people would listen.
It is as if the peace activists believe they have discovered for the first time those self-evident and thus
ancient truths that human life is sacred, and war is tragic. Little do they know that a majority of the Iraqis
who stroll past their peace marches in Baghdad support an American invasion. Many would eagerly fight
and risk death in an armed revolution if they could obtain the resources and momentum to launch one for
themselves.

Navet allows the peace movement to thrive, but it is animated by arrogance.

THE ARROGANCE

While campaigning for the presidency, candidate Bush said that his administration would conduct its
foreign policy with less arrogance than past administrations had displayed. He is now widely accused of
forsaking the less-arrogant approach and of choosing, instead, to rattle his saber at any dictator he thinks
he can rattle. But is it really arrogant for the president to insist that a violent and unpredictable dictator
with ambitions to control the world's oil supply who is also a friend of al Qaeda should be denied a
secret nuclear, chemical- and biological-weapons program? Is it arrogant to suggest that Saddam Hussein
should be removed from power if he continues to defy and deceive the international community?
Likewise, is it arrogant to expect the North Koreans to abide by the Agreed Framework, under which the
U.S. promised to inject millions of U.S. tax dollars into the faltering North Korean economy? Perhaps it is
slightly arrogant, but the peace movement is fantastically more arrogant.

The peace movement is founded upon a subtle ethnocentrism that escapes detection even by the
multicultural Left where most peace activists are bred. The group that most openly celebrates the
diversity of mankind does not understand that many people in the world hold diverse beliefs and
subscribe to ideologies that are entirely independent of American influence. In the mind of the peace
activist, America is not just the sole superpower, it is the center of gravity for all world events; and so
every world event is simply an equal (and sometimes opposite) reaction to a prior American action. Peace
activists believe that America's economy and culture are such dominant forces in the lives of people
throughout the world that the actions and policies of other nations can be interpreted only as mere
reactions to the actions and policies of the United States government. Therefore, they believe America
has the unbounded ability to manipulate foreign governments through economic and cultural means.

Peacenik foreign policy is really very simple: Without an action by the United States, there will be no
reaction by others. If America does not start a war, there will be no war. This is the arrogant
ethnocentrism of the peace movement. Under this view, it is unthinkable that quaint little dictators
such as Saddam Hussein or Kim Jong-il might deign to manipulate America as much or more than
America tries to manipulate them. It is unthinkable that a nation would resort to building nuclear
weapons if they did not first feel threatened by the world's only super-bully. It is inconceivable that
Saddam Hussein or Kim Jong-il might have diabolical plans and evil aspirations that were not created by,
and are not controlled by, the U.S. State Department. The peace activist then reaches the conclusion that

Page 1198 of 1481


the United States can make a unilateral decision for peace, simply by choosing to lay down its arms. If the
United States would ignore open and notorious breaches of U.N. directives and treaties, and simply refuse
to disturb the current state of peace, then peace would prevail by default.

Of course, the choice between war and peace is not ours alone. There could be war and likely will be
war regardless of our course of action. The only questions are: on whose terms, and on whose turf?

Page 1199 of 1481


Page 1200 of 1481
Pacifism Causes Aggression
(2/2)
NONVIOLENCE IS CODE FOR APPEASEMENT HISTORY PROVES THAT EVEN
RHETORICAL STANCES AGAINST THE USE OF FORCE CAN EMBOLDEN AGGRESSORS

Sowell 2K1 (Thomas, September 23, Pacifism on Principle is Suicide, Capitalism Magazine,
http://capmag.com/articlePrint.asp?ID= 1108)

Although most Americans seem to understand the gravity of the situation that terrorism has put us in -- and the need for some serious military response, even
if that means dangers to the lives of us all -- there are still those who insist on posturing, while on the edge of a volcano. In the forefront are college students
the pacifist
who demand a peaceful response to an act of war. But there are others who are old enough to know better, who are still repeating
platitudes of the 1930s that contributed so much to bringing on World War II. A former ambassador from the
weak-kneed Carter administration says that we should look at the root causes behind the attacks on the World
Trade Center and the Pentagon. We should understand the alienation and sense of grievance against us by various people in the Middle
East. It is astonishing to see the 1960s phrase root causes resurrected at this late date and in this context. It was precisely this kind of thinking. which sought
the root causes of crime during that decade, creating soft policies toward criminals, which led to skyrocketing crime rates. Moreover, these soaring crime
rates came right after a period when crime rates were lower than they had been in decades. On
the international scene, trying to
assuage aggressors feelings and look at the world from their point of view has had an even more
catastrophic track record. A typical sample of this kind of thinking can be found in a speech to the British Parliament by Prime Minister Neville
Chamberlain in 1938: It has always seemed to me that in dealing with foreign countries we do not give ourselves a chance of success unless we try to
understand their mentality, which is not always the same as our own, and it really is astonishing to contemplate how the identically same facts are regarded
Chamberlain sought to remove the causes of strife or
from two different angles. Like our former ambassador from the Carter era,
war. He wanted a general settlement of the grievances of the world without war. In other words, the British
prime minister approached Hitler with the attitude of someone negotiating a labor contract, where each side gives a little and everything gets worked out in
What Chamberlain did not understand was that all his concessions simply led to new demands from
the end.
Hitler -- and contempt for him by Hitler. What Winston Churchill understood at the time, and Chamberlain did not, was that Hitler was
driven by what Churchill called currents of hatred so intense as to sear the souls of those who swim upon them. That was also what drove the men who
Pacifists of the 20th century had a lot of blood on their hands for
drove the planes into the World Trade Center.
weakening the Western democracies in the face of rising belligerence and military might in aggressor
nations like Nazi Germany and imperial Japan. In Britain during the 1930s, Labor Party members of Parliament voted repeatedly against
military spending, while Hitler built up the most powerful military machine in Europe. Students at leading British universities signed pledges to refuse to fight in
the event of war. All
of this encouraged the Nazis and the Japanese toward war against countries that they
knew had greater military potential than their own. Military potential only counts when there is the will to
develop it and use it, and the fortitude to continue with a bloody war when it comes. This is what they did
not believe the West had. And it was Western pacifists who led them to that belief. Then as now, pacifism
was a statement about ones ideals that paid little attention to actual consequences. At a Labor Party
rally where Britain was being urged to disarm [!!!]as an example to others, economist Roy Harrod asked
one of the pacifists: You think our example will cause Hitler and Mussolini to disarm? The reply was:
Oh, Roy, have you lost all your idealism? In other words, the issue was about making a statement --that is, posturing on the edge of a
volcano, with World War II threatening to erupt at any time. When disarmament advocate George Bemard Shaw was asked what Britons should do if the Nazis
What a shame our schools and college neglect
crossed the channel into Britain, the playwright replied, Welcome them as tourists.
history, which could save us from continuing to repeat the idiocies of the past, which are even more
dangerous now in a nuclear age.

Page 1201 of 1481


Page 1202 of 1481
**Normativity**

Page 1203 of 1481


Normativity Answers: 2AC (1/7)
FIRST, EVEN IF THERE IS NO STABLE, OBJECTIVE LEGAL SUBJECT, PEOPLE STILL ACT
IN RESPONSE TO THE LAW, MAKING IT THE BEST PRAGMATIC MEANS OF SOCIAL
CHANGE. CROSS-APPLY OUR SPECIFIC TRIBE AND KATYAL SOLVENCY

SECOND, PERFORMATIVE CONTRADICTION REJECTING NORMATIVE LEGAL


THROUGHT PRESCRIBES A NON-NLT LEGAL NORM, WHICH IS BAD BECAUSE IT
PREVENTS US FROM LINKING OFFENSE, DESTROYS ARGUMENTATIVE
ACCOUNTABILITY, AND IS A VOTER FOR FAIRNESS AND EDUCATION

THIRD, PERM DO BOTH


ABANDONING NORMATIVITY IS IMPOSSIBLE. ACKNOWLEDGING THE LIMITS OF
LIBERALISM WHILE VICARIOUSLY PARTICIPATING IN LITIGATION CREATES
SUBJECTIVE FREEDOM THROUGH THE LAWS REPEATED FAILURE, COMING TO
TERMS WITH LEGAL APORIA

Carlson 99
[David Gray, Prof Law @ Cardozo, Duellism in Modern American Jurisprudence, 99 Colum. L. Rev. 1908,
November, LN//uwyo-ajl]

of Professor Schlag's points about legal scholarship


Should normative legal scholarship be abolished, as Professor Schlag suggests? Some

are undoubtedly well taken. But it doesn't follow that it should or even could be abolished. In truth, whether he
admits it or not, Professor Schlag himself does legal scholarship. He does not follow his own advice about not doing it. Nor could he. If legal

scholarship stands for participation in the realm of the symbolic, then legal scholarship - i.e., culture - is the
very medium that perpetuates self-consciousness.
Schlag is very hard on law professors who give advice to judges. He mocks their work as mere "pretend-law," n313 mere journalism. n314 "One need only pick up a judicial
opinion, a state statute, a federal regulation, or a law review article to experience an overwhelming sense of dread and ennui." n315 Meanwhile, judges are not even paying
attention to legal scholarship n316 - which, experience teaches, is disappointingly true.

Vicarious participation in litigation or legislation can nevertheless be defended as a participation in culture


itself. Law professors can contribute to that culture by making law more coherent, and in this sense their project is at
least as worthy as any that philosophy, history or astrophysics [*1951] could devise. Law has an objective structure that exceeds mere subjectivity. This objective structure can

be altered by hard work. An altered legal world, however, is not the point. Evidence of consequential impact is gratifying, but this is simply what mere egotism requires. It is in
the work itself that the value of legal scholarship can be found. Work is what reconciles the failure of the
unhappy consciousness to achieve justice. Work is, in Hegel's view,
desire held in check, fleetingness staved off... work forms and shapes the thing. The negative relation to the object becomes its form and something permanent... This negative
middle term or the formative activity is at the same time the individuality or pure being-for-self of consciousness which now... acquires an element of permanence. n317

By working the law, lawyers, judges, private citizens, and even academics
Hegel, then, gives a spiritual turn to that worthy slogan "publish or perish."

can make it more permanent, more resilient, more "existential," n318 but, more to the point, they make themselves more resilient,
more "existential." n319 Work on law can increase freedom - the positive freedom that relieves the worker
of "anxiety" - fear of disappearance into the Real. n320 When work is done, the legal universe swells and fills itself out - like an appetite that
"grows by what it feeds on." n321 But far more important, the self gains a place in the world by the very work done. Work is the

Page 1204 of 1481


means of "subjective destitution" or "narcissistic loss" n322 - the complete externalization of the subject and the surrender of the
fantasy support upon which the subject otherwise depends. In Lacanian terms, "subjective destitution" is the wages of cure at the end of
analysis. n323 Or, in Hegelian terms, cure is "the ascesis that is necessary if consciousness is to reach genuine philosophic knowledge." n324 In this state, we

precisely lose the suspicion that law (i.e., the big Other) does not exist. n325 In Hegel's inspirational words:
Each individual consciousness raises itself out of its allotted sphere, no longer finds its essence and its work in this particular sphere, but grasps itself as the Notion of will,
grasps all spheres as [*1952] the essence of this will, and therefore can only realize itself in a work which is a work of the whole. n326

I make no special claim that legal academic work is worthy of extra-special respect. It is a craft, like any other. As such, it is at least worthy of its share of respect. If spirit
unfolds and manifests itself in the phenomenal world of culture, n327 why should it not also manifest itself in the law reviews?

Page 1205 of 1481


Page 1206 of 1481
Normativity Answers: 2AC (2/7)
FOURTH, NO IMPACT SCHLAG JUST SAYS NORMATIVITY HAPPENS WITHIN A
FIELD OF VIOLENCE, NOT THAT IT CAUSES VIOLENCE

FIFTH, THE IMPOSSIBILITY OF JUSTICE MAKES JUSTICE POSSIBLE BY SUBLIMATING


ETHICS THE ALTERNATIVE IS A FANTASY SCREEN THAT INSTALLS A SENSE OF
SUBJECTIVE COMPLETION THAT NEVER EXISTED

Carlson 99
[David Gray, Prof Law @ Cardozo, Duellism in Modern American Jurisprudence, 99 Colum. L. Rev. 1908,
November, LN//uwyo-ajl]

For this very reason, justice is quite opaque to general definition. Being a phallic trope, justice never has
been and never will be defined. Any definition of justice could only occur by use of signifiers, yet justice is
precisely what is beyond signification. n51 So conceived, it is clear that justice must always fail. n52

Doing justice is therefore always an act of "sublimation" - in sublimation, I "elevate an object to the
dignity of the Thing." n53 Justice, as this void between legal concepts, participates in what Slavoj <hac
Z>i<hac z>ek calls the "ethics of the Real," which is

the moral Law in its impenetrable aspect, as an agency that arouses anxiety by addressing me with the
empty, tautological and, for that very reason, enigmatic injunction 'Do your duty!', leaving it to me to
translate this injunction into a determinate moral obligation - I, the moral subject, remain forever plagued
by uncertainty, since the moral Law provides no guarantee that I "got it right"... n54

Justice, I contend, is Professor Schlag's "robust referent." Yet what Schlag does not consider is that justice
always necessarily fails. Justice is a negative located in the interstices of law. Any attempt to legislate
justice [*1918] is mere sublimation. To deliver on this promise of justice, law would have to fill the legal
universe and crowd out the negative moment of justice. n55 To the extent law fails to deliver on its
promise - when it fails to fill the legal universe - it precisely leaves open the possibility of justice itself. n56
Justice is designed to fail!

According to the false Lacanian autobiography, law has promised justice, but it cannot deliver. Law has
castrated the subject but has not lived up to its side of the bargain. It has defaulted on its promise of
restitution. Law only fills the field of justice with more signifiers, on a logic by which law is remade with
every instance of legal practice. Revealingly, Schlag writes: "To be really good at 'doing law,' one has to
have serious blind spots and a stunningly selective sense of curiosity." n57 Professor Schlag captures the
practice of law acutely in this remark. "Doing law" is filling the gap with signifiers, a practice that does
indeed require serious blind spots in the performance of it. To speak or to act is literally to forget - that
the castrated subject is not whole. n58

There is no sense, however, in being angry about judicial failure. Law cannot be blamed for what it cannot
deliver. The healthy subject comes to learn that this failed bargain is a falsehood. The subject never had
the phallus. n59 Nor does the symbolic realm withhold it. Nothing has been lost and no restitution is due.
n60 Schlag's insinuation, that the symbolic realm has breached its obligation to deliver justice is thus false.
n61

Page 1207 of 1481


SIXTH, NO LINK WE DONT CLAIM THAT LAW ACHIEVES FINAL JUSTICE, JUST
THAT PLAN IS PERCEIVED IN A CONSEQUENTIALLY BENEFICIAL WAY

Page 1208 of 1481


Page 1209 of 1481
Normativity Answers: 2AC (3/7)
SEVENTH, THE CRITICISM MISIDENTIFIES THE LACKING NATURE OF LANGUAGE,
REINSCRIBING STABLE SUBJECTIVITY

Carlson 99
[David Gray, Prof Law @ Cardozo, Duellism in Modern American Jurisprudence, 99 Colum. L. Rev. 1908,
November, LN//uwyo-ajl]

I began by suggesting that Pierre Schlag assumes the position of a duellist. He thinks legal academics are
either fools or knaves. But he mistakes his opponent. The villain is language itself. Language is what
causes the split in the subject, and Professor Schlag has made the classic error of assuming that legal
academics are deliberately withholding l'objet petit a. They hold surplus enjoyment and are to blame for
the pain and the lack that always accompanies the presence of the subject in the symbolic order.

If this psychoanalytic suggestion explains the angry tone of Schlag's work, it also explains the basic errors
into which he falls. When one considers this work as a whole, most of these errors are obvious and
patent. Indeed, most of these errors have been laid by Schlag himself at the doorstep of others. But, in
surrendering to feeling or, as perhaps Schlag would put it, to context (i.e., the pre-theoretical state),
Schlag cannot help but make these very same errors. Some examples:

(1) Schlag's program, induced from his critiques, is that we should rely on feeling to tell us what to do. Yet
Schlag denounces in others any reliance on a pre-theoretical self. n328

(2) Schlag warns that, by definition, theory abstracts from context. n329 He warns that assuming the right
answer will arise from context unmediated by theory is "feeble." n330 Yet, he rigorously and repetitively
denounces any departure from context, as if any such attempt is a castration - a wrenching of the subject
from the natural realm. He usually implies that context alone can provide the right answer - that moral
geniuses like Sophocles or Earl Warren can find the answer by consulting context.

[*1953] (3) Schlag complains that common law judges are "vacuous fellows" when they erase themselves
so that law can speak. n331 Yet, Schlag, a natural lawyer, likewise erases himself so that context can
speak without distortion.

(4) Schlag warns that merely reversing the valences of polarities only reinstates what was criticized. n332
Yet he does the same in his own work. In attacking the sovereignty of the liberal self, he merely asserts
the sovereignty of the romantic self. Neither, psychoanalytically, is a valid vision. One polarity is
substituted for another. n333

(5) Schlag scorns the postulation of ontological entities such as free will, but makes moral arguments to
his readers that depend entirely on such postulation.

(6) Schlag denounces normativity in others, but fails to see that he himself is normative when he advises
his readers to stop being normative. The pretense is that Schlag is an invisible mediator between his
reader and context. As such, Schlag, the anti-Kantian, is more Kantian than Kant himself. Thus, context
supposedly announces, "Stop doing normative work." Yet context says nothing of the sort. It is Schlag's
own normative theory that calls for the work slowdown.

(7) Schlag urges an end to legal scholarship when he himself continues to do legal scholarship. He may
wish to deny that his work is scholarship, but his denial must be overruled. We have before us a legal
scholar, like any other.

Page 1210 of 1481


Page 1211 of 1481
Normativity Answers: 2AC (4/7)
EIGHTH, DISAVOWAL OF THE VIOLENCE OF REPRESENTATION AND CALLS FOR
INTERNAL RETHINKING RELY ON ASSUMPTIONS OF METAPHYSICAL INNOCENCE,
FETISHIZING AN AUTHENTICITY THAT NEVER EXISTED

Bewes 97
[Timothy, doctorate in English Literature at the University of Sussex, Cynicism and Postmodernity, New
York City: Verso, 1997, 195-6//uwyo-ajl]

postmodernism has actually become something. Its


Despite the diligence and the sterling efforts of its best theoreti-cians, then, it seems that

principal characteristic is the retreat from and disavowal of the violence of representation - both political
and semiotic. There are three further aspects to this essentially ignominious cultural operation: (i) a cultivation of stupidity (what I
have called Kelvinism, or 'metaphysical innocence') as a means of circumventing the ideational 'brutality' of the

political life; (ii) a recourse to the idea of an internal or subjective 'truth of the soul' which transcends
political reality, along with the contingencies of representation. Both of these signal an attachment to a
surface/ depth model of subjectivity which in each case amounts to a fetishization of authenticity,
whether by opting to 'remain' on the surface, or by retreating 'inwards'; (iii) a collapse of faith by individuals and even politicians
themselves, not only in the political infrastructure but in the very' concept of political engagement - here it becomes apparent that Tony Blair, for example, is more 'postodern'
than any theoretician. .

these three responses stand in an approximately analogous relationship to the archetypal forms in which consciousness, in a
It should be clear that

state of anxiety, shrinks from the violence of determinate negation and 'strives to hold on to what it is in danger of losing'. 59 At various
points throughout the present work I have used the terms 'decadence', 'irony' and 'relativism' to refer to these instances of an epistemological loss of nerve, this

capitulation to 'things as they are'; it may be as well here to remind ourselves of the terms in which Hegel describes these manifestations of a retreat from
truth. Consciousness, he says, at the decisive moment in which it is required to go beyond its own limits, (i) 'wishes to remain in a state' of unthinking inertia'; (ii) gloats over its
own understanding, 'which knows how to dissolve every thought and always find the same barren Ego instead of any content'; (iii) 'entrenches itself in sentimentality, which

assures us that it finds everything to be good in its kind'. 60Postmodernism, an empirical social condition - by which I mean that a series of critical-theoretical
strategies has attained a certain concrete form - legitimizes these symptoms of cultural anxiety; postmodernism becomes synonymous,
therefore, with deceleration, with a sense of cultural and political conclusivity; postmodernism is the principal vehicle of what Baudrillard calls

'the illusion of the end'.

AND, AUTHENTICITY FETISHIZATION AND ITS FEAR OF REASON AND VIOLENCE


ALLOW US TO SPEND HOURS DEBATING THE FINE POINTS OF BAUDRILLARIAN
ETHICS WHILE GAS CHAMBERS ARE BUILT

Bewes 97
[Timothy, doctorate in English Literature at the University of Sussex, Cynicism and Postmodernity, New
York City: Verso, 1997,146-7//uwyo-ajl]

If it is unreasonable to suppose that the Final Solution was potentiated or even necessarily facilitated by
Schmitt's theories, it is certainly the case that this metaphysical structure of domination in the Third
Reich, whereby the status of public citizens is reduced to a level determined entirely in the 'natural' or
biological realm of necessity, is foreshadowed in his 1927 essay. In an abstract and insidious way Schmitt
introduces the idea that the 'transcendent' realm of the political, as a matter of course, will not
accommodate a people with insufficient strength to ensure its own participation, and that such a fact is
ipso facto justification for its exclusion. 'If a people no longer possesses the energy or the will to maintain

Page 1212 of 1481


itself in the sphere of politics, the latter will not thereby vanish from the world. Only a weak people will
disappear.'130 Schmitt's concept of the 'political', quite simply, is nothing of the sort - is instead weighed
down by necessity, in the form of what Marshall Berman calls German-Christian interiority - by its
preoccupation with authenticity, that is to say, and true political 'identity'. Auschwitz is a corollary not of
reason, understood as risk, but of the fear of reason, which paradoxically is a fear of violence. The stench
of burning bodies is haunted always by the sickly aroma of cheap metaphysics.

Page 1213 of 1481


Page 1214 of 1481
Normativity Answers: 2AC (5/7)
NINTH, PUBLIC CRITICISM OF EXECUTIVE POWER CREATES CRITICAL MOMENTUM
THAT FORCES THE COURTS TO HOLD PRESIDENTIAL ABUSES ACCOUNTABLE WEN
HO LEE PROVES

Yamamoto 2005
[Eric K., Prof. Law @ Hawaii, White (House) Lies: Why the Public Must Compel the Courts to Hold the
President Accountable for National Security Abuses, 68 Law & Contemp. Prob. 285, Spring, LN//uwyo-ajl]

These events and counter-messages garnered national press. Belatedly, the mainstream media responded
with new images and opinions. Writers who had contributed significantly to the public vilification of Dr.
Lee now became critical of his treatment and even advocated for his release. n116 On August 22 and 23,
2000 - during the crucial days when Judge Parker was considering the defense's key motions for bail,
discovery, and evidence - editorials and [*310] headlines in major newspapers declared: "Wen Ho Lee
Deserves Bail and Fair Treatment" (San Francisco Chronicle); "Is Lee Guilty Until Proven Innocent?"
(Chicago Tribune); "Free Wen Ho Lee" (St. Louis Post-Dispatch); "Wrong One Is on Trial in Lee Case" (Los
Angeles Times); and "Bail for Wen Ho Lee" (New York Times). n117

Organizers also employed an effective strategy of public education through print ads in major
newspapers. For example, on August 7, 2000, Chinese for Affirmative Action organized a full-page ad in
the New York Times demanding "Drop all charges. Free Dr. Wen Ho Lee now." The ad was titled, "Wen Ho
Lee & The Nuclear Witch Hunt" and focused on being "charged with being ethnic Chinese." The ad
presented Dr. Lee in a different light than New York Times readers had become accustomed to seeing.
Instead of the stereotypical foreign spy, Dr. Lee was presented as an "American scientist" separated from
his wife and two children for eight months, countering the usual media image of Dr. Lee in shackles.

Op-ed pieces also countered the executive lies and stereotypes. Attorneys Theodore Wang and Victor
Hwang published an opinion piece titled, "Charged With Being Ethnic Chinese." n118 In it they exposed
the racial profiling and challenged the premise on which the government based its racist actions. They
correctly framed the issue as "not only for Lee but for all Americans concerned about whether the
government should be able to launch criminal investigations based on the race of a suspect." They also
argued that "by focusing only on Asian Americans, a real spy may have escaped the scrutiny of the federal
government altogether." n119 This and other op-ed pieces strategically framed the issue of racial profiling
as one for "all Americans" and publicly questioned the effects of allowing the government to continue
such practices without accountability. n120

Critical legal advocacy and organized pressure helped reframe for the public, and for Judge Parker, the
real issues - selective racial prosecution Executive lies and the need for accountability. This new sense in
the public culture of what was really going on and what was really at stake provided the backdrop for
courtroom decisions. Amid intensifying demands to free Dr. Lee and put the Justice Department on trial
instead, Judge Parker ordered the government to disclose documents on racial profiling and negotiate a
release agreement with Dr. Lee. n121

Page 1215 of 1481


Page 1216 of 1481
Normativity Answers: 2AC (6/7)
TENTH, DEBATE AND DELIBERATIVE POLITICS ARE CONSTANT RENEGOTIATION OF
VALIDITY CLAIMS, CARRYING THE CAPACITY FOR CONSTANT INNOVATION OF
MEANING

Kulynych 97
[Jessica, Asst. Prof. of Poli Sci @ Winthrop, Polity 30: 2, Winter//uwyo]

If we interpret the "to show" here not as pointing out what is wrong with disciplinary society (which would leave Foucault subject to Fraser's normative criticism), but rather as
"showing," or "showing up," then we no longer need the introduction of normative notions, we are merely doing disciplinary society one better. Making a point is a function of
discourse, the ability to align and arrange arguments that support a position. Yet, the performative protestor does not argue against the state, he mocks it. The protestor works
at the margins of discourse, utilizing puns and jokes and caricature to "expose" the limits of what is being said. Thus, performative resistance, when considered as critique,
does not need to tell us what is wrong, rather it reveals the existence of subjection where we had not previously seen it. I am not suggesting that we can get a normative anchor

performativity is
out of the notion of performativity. To the contrary, I am suggesting performative resistance makes no such normative distinctions, or rather, that

not about normative distinctions. We bring normativity to our performances as ethical principles that are themselves subject to resistance. By
unearthing the contingency of the "self-evident," performative resistance enables politics. Thus, the
question is not should we resist (since resistance is always, already present), but rather what and how we
should resist.

Habermasian deliberative participation. Just as a


This notion of performativity is also important for understanding the possibilities for innovation in

protestor exposes the contingency of concepts like justice, a dialogue exposes the limits and contingency of rational argumentation.
Once we are sensitive to the performative nature of speech, language and discourse, then we can see that deliberative politics cannot be confined to the rational statement of

that that which cannot be argued for finds expression.


validity claims. Deliberation must be theatrical: it is in the performance of deliberation

Indeed it is precisely the non-rational aspects of deliberation that carry the potential for innovation. In his
description of the poignant reminders of demonstration Chaloupka recognizes that it is at the margins that the actual force of the demonstration resides, no matter what
happens at the microphone. The oral histories of demonstrations (the next day over coffee) linger over the jokes and funny signs and slogans, the outrages and improprieties,

Any convincing account of the politics of deliberation must take


more than the speeches and carefully coherent position papers.(68)

account of the creative potential that resides in the performance of debate.

ELEVENTH, THE ALTERNATIVE COLLAPSES BACK INTO NORMATIVITY,


REINSCRIBING LEGAL VIOLENCE

Annelise Riles, Ford Fellow in Public International Law, Harvard Law School, 1993 94, 1994 U. Ill. L. Rev. 597, *650

If Geertz's argument signals a loss of faith in the interdisciplinary scholar's ability to combine the theories of each discipline, is there hope for the current effort to break law and
society into myriad component parts and relate these anew, as Geertz sought to do with his turn to fact and law? In this respect, I think, the two disciplines share a moment of

in manipulating one dichotomy after another, the scholar has the sinking sense that all the
theoretical impasse, for

possible positions are prefigured. As noted at the outset, practitioners of legal anthropology now pessimistically perceive the possibilities of their discipline.
Likewise, although it is now increasingly fashionable for lawyers to turn outside their discipline for grand insights, they do so with increasing wariness. The image of what

anthropology might have to offer, the totally new insight, the epistemology-bursting perspective, never seems fulfilled. Every combination and
recombination, every construction and deconstruction seems already prefigured. Just as the "whole" of culture now has ceased to do
the work of organizing our arguments, the "whole" of the discipline certainly no longer seems worth supporting or opposing. But neither do its parts. The effect of this

change is that there no longer is much rhetorical force in claiming dangerous or creative spaces in-between. How can
Leach's disciplinary terrorism be maverick if the opposition he bridges is no longer real for us? How can Geertz's shuttling between fragmented points of relation feel innovative
if the parameters within which these points lie are entirely familiar from the start? To claim that there is nothing new to combine, or that relation no longer works, is to relinquish
the identity of the productive scholar -- who is productive because he or she makes new forms. B. Normative and Reflexive Knowledge As noted at the opening of this part, one
must stand for something in an article such as this one; reflecting on the arguments of others in itself is not enough. If the task of relationship building in interdisciplinary
scholarship has lost its force, therefore, I now must argue for an alternative. This understanding pervades the works we have considered from Henry Maine to the present day.
The imperative to harness observations, as here about the state of interdisciplinary scholarship, into a claim, as here for a future direction of interdisciplinarity, and the difficulty
experienced in doing so, characterizes much contemporary interdisciplinary work. Indeed, one of the enduring characteristics of the tradition we have considered is precisely
this transformation from what we might call a reflexive mode of knowledge into a normative mode and back again. Every work we have considered in the preceding pages has

insight always is produced by observing a


made its contribution to legal knowledge by approaching its subject reflexively. By this, I mean that

topic in European or American law from another, wider vantage point. Maine, for example, reflects upon legal positivism from the *644 point of

Page 1217 of 1481


view of the history of European civilization. Leach takes the problem of an international response to terrorism and recasts it in terms of violence in primitive societies. This
reflexivity involves a broadening of perspective, and it often is achieved by a kind of movement beyond one's starting position to another position and back again, as when
Geertz takes us on a tour of the world's legal systems or when Maine moves through successive stages of historical development. When contemporary interdisciplinary
scholars argue for attention to the "outside," to "context," or to a "wider reality" beyond the law, I think they are conflating the metaphors we use to describe this reflexive mode

. Yet every author also understands him or herself to be


of knowledge -- metaphors of expansion and movement -- with an "actual" outside

staking out a normative claim. Maine is for a more academic tradition of legal scholarship, and he is against the democratization of legal institutions. Leach,
likewise, has a political motive in treating the terrorist bombings of the 1970s and the atom bombing of Hiroshima as commensurates. This kind of normative claim, in contrast
to reflexive knowledge, is achieved precisely by holding things constant, by refusing to move to another perspective even if one understands such movement as possible, and

by constricting rather than

continued

Page 1218 of 1481


Page 1219 of 1481
Normativity Answers: 2AC (7/7)
continued

expanding the scope of inquiry so that a sharp claim can be made. It is no wonder that we describe such normative knowledge using
stationary metaphors -- staking out a position, taking a stand, etc. To make a claim about the future of interdisciplinary work in legal anthropology, then, is to be normative in the

normative and reflexive knowledge. It is worth noting at the outset, however, that these two modes of knowledge
sense of this engagement between

are not logically contradictory. On the contrary -- it is precisely Maine's reflexive reconsideration of modern legal institutions from a broader historical
vantage point that gives rise to his antipopulism, and it is Leach's interest in understanding primitive society on its own terms that leads him to defend the terrorist's world view

. One of the defining aspects of the interplay between reflexive and normative modes of
against the position of international law

engagement is that each slips effortlessly, almost uncontrollably, into the other. There is no resting point
at which one is reflexive or normative: we "know" that every relativism is actually an argument for something or other. Indeed, this knowledge gives rise to one of the classic
modes of critique in the repertoire of both lawyers and anthropologists, as we expose the "position" or "argument" behind a certain reflexive exposition. The same is true of

: we can always understand a normative claim such as a call for the universal protection of rights of expression, for example,
normative argument

to be the expression of a particular point of view, and indeed, as soon as such a normative claim is made, it seems to
engender a reflexive turn. It is not just that a normative argument produces a reflexive one. Rather, the very same knowledge,
effectuated in a reflexive mode, invariably becomes normative. Maine's historicization of Bentham's positivism, for example, in turn
becomes an argument against the universal application of positivism. Leach's reconsideration of the cultural construction of terrorism becomes a normative claim for the
importance of attention to cultural difference itself. One of the defining aspects of the interplay between reflexive and normative knowledge in interdisciplinary scholarship, then,
is the way in which each relativism in turn becomes its own position, which then is open to relativization again. A reflexive observation becomes an argument to stand by, and
that argument then can be reconsidered in a reflexive way. By way of example, we might consider a prominent article by lawyer and anthropologist Sally Falk Moore, Treating
Law as Knowledge: Telling Colonial Officers What to Say to Africans About Running "Their Own" Native Courts. Building on a career-long investigation into the British colonial
legal system, its assumptions about African society, and the response it generated among the Chagga, Moore takes as her point of departure a 1957 British directive
concerning the organization of customary courts among the natives of Tanganyika. The theme of the piece is the conflict between the British administrators and the village
courts over British legal notions, such as res judicata and the Rule of Law as a rule of the written word, and the intended audience of the piece includes both lawyers and
anthropologists. The contribution of the piece is a reflexive reconsideration of what Moore takes as the Anglo-American faith in the rule of law. She writes in the article abstract:
This article is presented at two levels throughout. On the surface it is a straightforward historical analysis of a directive to British officers . . . . On a deeper level the article uses
the British colonial occasion to explore widely held cultural assumptions in Anglo-American law about the definability of "justice," the concept of time and timing in legal affairs,
and the complex place of the idea of legitimate, authoritative, and permanent "knowledge" in legal institutions. *646 Moore's ultimate target is the colonial government's
obsession with rule making, with cataloguing African practices into a codifiable form. In a classic relativizing spirit, she is concerned that we understand that notions of a "rule-
governed judiciary" of the kind she finds in the texts of H.L.A. Hart, and the obsession with written precedent on which it depends, are culturally specific ways of resolving
conflicts, not -- as she quotes her colonial directive to claim -- natural law. This reflexive turn engenders many of the patterns we have observed in other contemporary works of
Legal Anthropology: Moore emphasizes the rationality of African legal systems on their own terms and in so doing discovers a social reality outside the law. She argues that the
architects of the British colonial legal regime failed to understand that "[t]he Africa of reality had its own social and legal logic." This African reality, moreover, is the realm of
expertise of the anthropologist: "The colonials had to cope with the consequences of this 'localism' but did not understand the nature of local rural communities," she notes,
owing partly to the fact that (unlike anthropologists) "most of them did not speak any of the many local languages." She explains that "[t]he colonials did not picture these
villages as they were . . . . Had they known what we now know about the internal political life of African neighborhoods and villages, they might have had a very different
understanding of what was going on." She even notes concerning the 1950s writing of a Restatement of African Law, that the law professor in charge saw the insights of
anthropologists as too imprecise to be useful to courts engaged in modernization and nation building. This reconsideration of law from a wider perspective is also its own
normative argument, a kind of lecture to lawyers about the cultural particularity of their world view. The ultimate point Moore hammers home to her legal audience is the classic
plea for attention to context. As she puts it, "[t]his circumstance raises a question in relation to the colonial instance that has far wider application: Is it possible to 'know' much
about a legal system without knowing the character of the case-generating milieu?" The answer for Moore clearly is no. Text is meaningless without context. This rhetoric in
turn is organized around a severe and confident break between the legal and social spheres -- both of the subject, the colonial administrator and the Chagga, and the subtext,
the lawyer and the anthropologist. "Certainly the difference between the designed judicial institution and the 'event-evolved' set of neighborhood institutions is very great." The
effort of looking at the world of law from a broader perspective now has become the subject of an argument to Moore's legal colleagues. Yet Moore does not stop with the
lessons of anthropology for law. In a fascinating passage, she attacks the "fashion" of anthropological critiques of colonial practices that show the ignorance of colonial
administrators about local practices: "As the colonial period has been safely over for more than thirty years, showing colonial flaws coupled with colonial arrogance is not only
politically risk free, it is a rather conventional version of history for our time." Claiming for herself a more "experimental" territory, she asserts an interest in "the cumulative
historical production of institutions" that lies beyond such simple assertions of colonial failure. Given the symbolic association of the legal academic and the colonial
administrator in her text, one is left to wonder what this might mean for those who, like the vulgar critics of colonialism, engage in vulgar lectures to legal academics about the
weaknesses of legal formalism and rule- based adjudication. The paper cannot come to a close, in other words, until Moore's normative claims on behalf of anthropological
methods engender their own reflexive reconsideration. The transformation of reflexive into normative modes and back again spawns a parallel transformation in the knowledge
it produces. For example, we saw that anthropologists first reflected on law from a wider point of view and discovered relationships by doing so. These relationships soon
became a position in themselves, outside the law. It was only a matter of time, therefore, before that position itself would become the subject of reflexive interpretation, as I
have done in the pages above. Yet if reflexive modes of knowledge engender normative knowledge and vice versa, these modes are not alternatives in the lexicon of *648
lawyers and anthropologists, nor are they opposites. One cannot simply choose to relativize or to argue for something, as one would choose a Law and Economics approach or
a Law and Anthropology approach to a legal problem, because each is understood to negate the possibility of the other. Likewise, it would be nonsensical to try to devise an
approach that would combine normative and reflexive knowledge: one cannot be a relativist and stand for something, it is often said. Each mode engulfs the entire enterprise of
representation, so that if I write in one genre, I cannot invoke the other. This is because unlike disciplines or cultures, normative and reflexive modes of knowledge are not of
the same order. They are not contained in a single frame, as law and anthropology are contained in the frame of disciplinarity, or as Barotse legal systems and Anglo-American
law are contained in the frame of cultural difference. Taking a position and looking at things from a relativizing point of view will not create a relationship even if we want it to.
Reflexive and normative knowledge were not always incommensurable in this way. Henry Maine's peers would not have interpreted his appeal to a wider historical perspective
as negating the possibility of normative argument about legal positivism or practical engagement with contemporary legal problems. Maine's failure to treat his argument and his
reflexive analysis as incommensurable, I think, contributes to the contemporary view of Ancient Law as uninteresting scholarship at best and embarrassingly naive scholarship
at worst. Leach might exemplify an epistemological change, vis-aea-vis Maine, then. Although we saw that Leach quite consciously stakes out claims about the rationality of the
terrorist even as he treats his own arguments about terrorism as objects of reflexive inquiry, there is a marked tension between these two modes of engagement, and the
tension is resolved only by the irony in his assertion that savages are not "dog-headed cannibals" that acknowledges the possibility of relativizing the normative claim even as it
seeks to hold that claim constant. It has become necessary for Leach, as it was not for Maine, to appeal to a rhetorical device such as irony to keep what have become two
incommensurable modes of engagement in view. This incommensurability, still implicit in Leach's case, now itself has become a problem, a topic of furious debate. One hardly
can have a conversation about law these days without arguing about relativism. *649 The transformation of normativity into reflexivity and back again has become its own topic
of normative engagement, in other words. We might consider this a key aspect of the contemporary epistemological moment for both disciplines. The effect of this development
is that being in favor of an interdisciplinary method of legal studies today means having faith in this transformation of one mode of knowledge into another. Or to rephrase the
claim in more normative terms, what is best about contemporary interdisciplinary scholarship is the transformation of knowledge it engenders. Although this movement is not
"real" in the sense of a reality outside the law, I am suggesting that it is worth taking seriously in its own right. In this sense, Maine's appeal to movement and change, in which
structure appears as reflection after the fact on the path of such movement, can be as much a model to us as Leach's more contemporary arguments in which structure is
prefigured as an organizing frame. Yet this transformation of modes of knowledge differs from the movement both Maine and Geertz advocate in that normativity and reflexivity
are not positions, places of the same order that occupy a single plane. At least at this juncture, no linear connection can be drawn between them nor can any descriptive thesis
summarize the transformation of one into the other. I do not mean to imply that this kind of transformation is unique to anthropological approaches to law. On the contrary,
lawyers know that slippage from normativity to reflexivity and back again pervades legal thinking as well. Yet perhaps the tension between disciplines provides an apt metaphor
for describing what we do not yet have other language to describe. Perhaps this incommensurablity becomes concretized, or institutionalized in the gulf between disciplines that
both lawyers and anthropologists celebrate, so that interdisciplinary engagement between law, as the metaphorical province of normativity and politics, and anthropology as the
metaphorical province of reflection and difference, provides a technology for experiencing and elaborating the incommensurability of reflexive and normative thought. In the
pages above, I have endeavored to trace a path through a series of claims for an anthropological, ethnological, or interdisciplinary study of the law. A consideration of this
tradition leaves us with a number of possible observations. First, it leads to an appreciation of the extent to which contemporary anthropological appeals to reality outside the
law, discovered through empirical observation of context, and through emphasis on real people rather than the theoretical structures of law, is predicated on shared notions
among lawyers and anthropologists about the salience of the disciplinary divide. Ironically, *650 however, if the success of the rhetoric is predicated on a shared epistemology,
arguing for attention to context against the legal text, for example --
then simply defending one side or another of a shared dichotomy --

can never offer an escape from the theoretical impasse created by the dichotomy precisely because the

Page 1220 of 1481


move is prefigured in the very structure of the dichotomy itself. Such an earnest -- even in some cases strategically self-
righteous -- plea on behalf of the outside, whether it be the new methodological innovation or the "real world out there," may find itself welcome in
both legal and anthropological circles but hardly seems poised to make ground-breaking contributions to either. We need an

alternative to a move to the periphery that always prefigures a return to the center. Second, in tracing the emergence
of the project of discovering and elaborating relationships as the modern project of interdisciplinary work, we come to appreciate why this project also now fails to satisfy. This
elaboration of relationships between disciplines, between law and society, or between ever smaller fragments of each seems predictable because it is. In order to work, the
entities to be combined must already exist in a prefigured frame -- disciplinary or cultural difference, for example -- so that we know at the outset the parameters within which
the new mix will take its form. The recent attempt to show scholarly productivity by finding ever more intricate, indeterminate, or subtle connections only heightens the sense of
a project that now is spent.

Page 1221 of 1481


Page 1222 of 1481
#3 Permutation: 1AR
SCHLAGS CRITICISM ONLY GETS IT HALF RIGHT- THE BUREAUCRACY CERTAINLY
OPERATES ON A FIELD OF PAIN AND DEATH, BUT WE SHOULD NOT BREAK FROM
THE LAW, BUT INSTEAD EMBRACE IT

CARLSON & SCHROEDER IN 2003


(JEANNE AND DAVID, CARDOZO LAW PROFESSORS, 57 U MIAMI L. REV 767)

Beyond laying down the law, another normative program emerges from Schlag's work: "What is missing in
normative legal thought is any serious questioning, let alone tracing, of the relations that the practice, the
rhetoric, the routine of normative legal thought have (or do not have) to the field of pain and death." 18
The suggestion is that we should come to realize that law itself is the very ground for the field of pain and
death. When this is realized, the normative program to lay down the law becomes a high moral
imperative. It appears from Schlag's work that the proper project for legal scholarship is to expose law's
responsibility for pain and death. This is what we should do. When legal scholarship has achieved this
task, presumably pain and death will have been eliminated. Turning the tables on law and economics,
Pierre implies that it would be efficient (i.e., useful to human utility) if law would abolish itself. But,
stranded on a field exfoliated of pain and death, what next? The implicit program seems to be that,
once the distortions of law are removed, the subject simply does not have to be told to do anything.
Whatever the subject does will be authentic. This is the free, liberated subject that Schlag's normativity
implies--a natural subject from whom completeness and authenticity has been unfairly denied by the
legal bureaucracy. If we are right, then underlying Schlag's polemic against law is an uncritical romantic
psychology. This would in turn mean that Schlag is not so much a critical scholar as a romantic one. This
implicit psychology means that Schlag has something in common with the political liberals he attacks.
Both Schlag and liberals believe in the autonomy of the human subject--and the possibility that the
subject can achieve this desired state of freedom. Furthermore, they both believe in the existence of subjectivity in a state of nature on which
positive law or social engineering cannot possibly improve. Law, then, has become a tool for oppressing the bureaucratic society that legal academia unwittingly serves. Legal
subjects, subjected to the law, are alienated from themselves by the law. The corollary to this [*771] is that there must be at least the possibility that subjectivity could be other
than it is now--distorted by law. Lest we be misunderstood, we emphasize that we agree with much of the above account. We agree with Schlag's suggestion that normativity
cannot succeed. Virtually every observation that Schlag makes about law and policy scholarship (normativity) is correct. Where we disagree is that there is a subject left
standing once legal normativity is abolished. Unfortunately, although Schlag ostensibly bases much of his analysis on the post-modern critique of the liberal conception of the
autonomous self-identical subject, he, in fact, falls back on a liberal conception of a natural self. Romanticism implies that the self-identical individual of liberalism is real--but
disfigured by law and hence on a field of pain and death. The post-modern position is quite different. It denies the pre-legality of personality and suggests that personality is

Lacanian psychoanalysis agrees with half


itself a legal idea. On this view, the self-identical subject of liberalism cannot exist as a theoretical matter.

of Schlag's proposition. The subject is on a field of pain and death, where it is not self-identical, but
severely wounded by law. It is precisely law (broadly understood as the symbolic order) that castrates
the subject, as Schlag maintains. Breaking the chains of the law, however, would not free but would
obliterate the subject. Subjectivity is nothing but the split, the gap, the rift in the natural subject torn by
law. If law is removed, the rift that creates subjectivity is obliterated. What was Lacan's name for a
person who successfully follows Schlag's normative program and slips the chains of law? His term for
such a person was "psychotic." 19 For Lacan, the normative program is precisely not to let go of the
symbolic order, for that would be the death of subjectivity, not its liberation.

Page 1223 of 1481


Page 1224 of 1481
#3 Permutation: Ext
DISCOURSE RELIES ON INFORMATION FROM THE OUTSIDE; WITHOUT ENGAGING
IN THE REAL WORLD, CHANGE IS IMPOSSIBLE.

Habermas, Prof @ Goethe U in Frankfurt, 90 (Jurgen, Discourse Ethics: Notes on a Program of Philosophical
Justification, The Communicative Ethics Controversy, Ed. Benhabib and Dallmayr, P. 100-101)

The principle of discourse ethics makes reference to a procedure, namely, the discursive redemption of normative claims to validity. To that extent,
discourse ethics can properly be characterized as formal, for it provides no substantive guidelines but only
a procedure: practical discourse. Practical discourse is not a procedure for generating justified norms but
a procedure for testing the validity of norms that are being proposed and hypothetically considered for
adoption. This means that practical discourses depend on content brought to them from outside. It
would be utterly pointless to engage in a practical discourse without a horizon provided by the life-world
of a specific social group and without real conflicts in a concrete situation in which the actors considered
it incumbent upon them to reach a consensual means of regulating some controversial social matter .
Practical discourses are always related to the concrete point of departure of a disturbed normative agreement. These antecedent disruptions determine the
topics that are "up" for discussion. This procedure, then, is not formal in the sense that it abstracts from content. Quite the contrary is true. In its openness,
practical discourse is dependent upon contingent content being "fed" into it from outside. In discourse this
content is subjected to a process in which particular values are ultimately discarded as being not susceptible to consensus. The question now arises whether
this very selectivity might not make the procedure unsuitable for resolving practical questions.

Page 1225 of 1481


Page 1226 of 1481
#5 Sublime Justice: 1AR
INSOFAR AS PEOPLE THINK OF THE LAW AS REAL, IT EXISTS, ALLOWING FOR
POWERFUL CHANGE DETAINMENT PROVES

Carlson 99
[David Gray, Prof Law @ Cardozo, Duellism in Modern American Jurisprudence, 99 Colum. L. Rev. 1908,
November, LN//uwyo-ajl]

Conceived as that which causes the judge to decide (per Schlag's definition), law is dynamic. It is indeed "performative," as Schlag maintains. n162 This future anterior
grammar of law doesn't make it purely subjective, however. n163 Law exists, and so it animates the judge who pronounces
judgment. It bears not a circular but a linear relation to the judge. Law animates when the judge's free will suppresses the
judge's pathological criteria and lets the judge be the law's oracle. n164 True, the empirical judge is capable of bad faith. Perhaps
what the judge had for breakfast rather than the law caused a judicial decision to be pronounced. But law's possibility, at least, is affirmed by license

of free will. n165

Law's consequence (which Schlag concedes)


Consequence, Schlag maintains, cannot prove that law exists. n166 But quite the opposite is true.

proves law's existence and its suitability for scientific study. n168 Schlag is prepared to concede that the law causes
n167

human beings to act, as when they execute or incarcerate a prisoner. It then follows that either (a) the law has a mechanical effect
on human beings - an absurdity n169 - [*1932] or (b) human beings have the capacity to choose to obey law. The second possibility is the only plausible one, because Schlag
effectively admits the existence of free will and moral capacity. Thanks to this concession, we can affirm that law exists and that human beings can choose to follow the law.

n170 Admittedly, we can never confirm legal effect directly, because it must be mediated by free will, which can only be postulated. n171 We can, however, confirm
its possibility and rule out its impossibility.

If thoughts (such as law) induce free human beings to act, then thoughts are things - and powerful things
at that. To the extent we indulge in a belief in free will, law is potentially effective. When it is, when human beings
execute the law, law's effects are rendered "tangible" and "visible" - the very attributes of the super-realist metaphysics that seem to underwrite Schlag's work. Although law

cannot be felt directly, its indirect effects are sensual indeed .

LEGAL THOUGHT EMBRACES THE NEGATIVITY OF ETHICS, CREATING SUBJECTIVE


FREEDOM

Carlson 99
[David Gray, Prof Law @ Cardozo, Duellism in Modern American Jurisprudence, 99 Colum. L. Rev. 1908,
November, LN//uwyo-ajl]

Schlag criticizes legal academics for unwitting indulgence in a contradiction. The self is supposed to be
sovereign. Yet the self bows down [*1943] to the rule of law. n244 The choice to be bound is supposed
to be a contradiction in terms. n245

From what has been said, it should be clear that there is no contradiction here. The self that stands
against the natural world, and the animal inclinations that afflict its body, is a negative entity. At heart,
the subject is nothing at all. n246 Yet, if it is to "exist," it must have externally observable properties. It
must do something, and the things it does become an attribute of the self. We are what we do. n247

The subject that lawfully follows its passion achieves existence and so perpetuates itself. n248 This is the
positive freedom of the self. Any self choosing to conform to the law has put forth its moral character in

Page 1227 of 1481


the world. It was the free choice of the self to do this. n249 Hence, the free self can choose to be bound,
without contradiction. n250

This concrete subject is likewise free to violate the law and to perpetuate itself by crime. This is the
negative freedom of the concrete subject. It is not properly freedom at all, but slavery to inclination.
Crime constitutes inclination speaking in defiance of the moral side, thereby committing a crime on the
subject's own self. The particularity of the criminal is therefore not freedom but slavery. n251

In fact, tied into the very idea of following the law is the idea of a free will that might choose not to follow
the law. The free will that aspires to follow the law never truly binds itself. A subject that puts itself
forward as lawful could give into impulse tomorrow and is therefore "free" (in the negative sense) to
violate the law. Lawfulness is therefore a constant struggle - the ongoing achievement of the concrete
self. Furthermore, it is a struggle in which the subject must fail:

Freedom realizes itself through a series of failures: every particular attempt to realize freedom may fail;
from its point of view, freedom remains an empty possibility; but the very continuous [*1944] striving of
freedom to realize itself bears witness to its "actuality." n252

Freedom is thus "powerful." It exhibits the "primacy of possibility over actuality." n253 Forever potential,
it is nevertheless a possibility that transforms the world.

Page 1228 of 1481


Page 1229 of 1481
#7 Alt Reinscribes Subject: 1AR
(1/2)
THE ALTERNATIVE IS A BEFORE THE FALL FANTASY OF A PRE-LEGAL SUBJECT

Carlson 99
[David Gray, Prof Law @ Cardozo, Duellism in Modern American Jurisprudence, 99 Colum. L. Rev. 1908,
November, LN//uwyo-ajl]

In contrast to this view, Professor Schlag wants to say that freedom means the concrete self can do what
it feels like. But he should know better than to exalt the authenticity of the pre-legal natural self, and he
has on occasion chastised others for doing just that. n254 To exalt the sovereignty of such a self (that may
be in the thrall of criminal passion) instead of the liberal self is to permit the contingent side of the self to
govern in its moral arbitrariness. n255 In other words, the essence of personality is the rationality of the
liberal self. Negative freedom denies the essence of personality and therefore ends up destroying its own
self. n256

To summarize, Schlag's work is based on a romantic psychology. If only the concrete self were freed from
law, Schlag implies, it would know what to do. Law offers mere "norms" and presents the subject with
empty choices. Such a theory of the self ignores the fact that human nature has two sides - the natural
and the moral. One side cannot be privileged at the expense of the other.

To be sure, many of Schlag's criticisms of liberal psychology n257 are well taken. Liberal psychology
absolutely denies a place for the unconscious and irrational. His accusation that liberal philosophy does
not consider the challenge of deconstruction to liberal psychology is an excellent contribution. Liberal
philosophy in recent times deserves criticism for not peering very deeply into the soul of the legal subject.
n258 But liberal philosophy is also on to something: The moral dimension of personality [*1945] is
constitutive and cannot be abolished without destroying personality entirely.

THE ALTERNATIVE IS A FANTASMIC ATTEMPT TO RESTORE A UNITARY SUBJECT,


SHORING UP ENJOYMENT STOLEN BY THE LAW

Carlson 99
[David Gray, Prof Law @ Cardozo, Duellism in Modern American Jurisprudence, 99 Colum. L. Rev. 1908,
November, LN//uwyo-ajl]

In his disenchantment with reason, Schlag has written that, just because lawyers pursue their profession
"does not establish whether liberal categories such as 'individual rights' are on the order of rocks, trees,
dollar bills, rubles, words, advertising images, or angels." n69 Within the gross and scope of this ontic
spectrum, rocks and trees are trenchantly existential. They can be felt. Dollars are perhaps less so, on
most measures of the money supply, but rubles, words, advertising images, angels and liberal category
drift into the realm of "ontological entities" n70 - mere figments of the imagination. These latter items do
not "exist." Perception mediated by thought is not to be trusted.

Page 1230 of 1481


Law's defect, then, is that, like Macbeth's dagger, it is insensible to feeling. Law is nothing but thought.
Thought (mediation) does not exist, and neither does law. n71 Tangibility - immediacy of intuition - is, I
infer, Schlag's criterion of epistemic certainty. What is tangible does not rely on language for its integrity.
n72 Tangibility transcends the legal order. It is quite alegal and for this very reason valid. n73

Such a criterion of reality means that, in the end, Schlag's program is a romantic one. Law has deprived
the subject of its jouissance. If law would kindly step aside, the subject could enjoy an immediate
restitution of its lost parts - a unity that would be certified by feeling. Therefore, justice supposedly
demands that law abolish itself so that the concrete subject in its negative freedom can be guided by its
natural, uncomplicated [*1921] dimension - by feeling - towards wholeness. n74 But for law, the subject
could enjoy itself all the time. n75

Page 1231 of 1481


Page 1232 of 1481
#7 Alt Reinscribes Subject: 1AR
(2/2)
SCHLAGS PARANOID VISION OF LEGAL BUREACRACY IS A FANTASY ATTEMPT TO
RATIONALIZE THE FAILURE OF THE SYMBOLIC

Carlson 99
[David Gray, Prof Law @ Cardozo, Duellism in Modern American Jurisprudence, 99 Colum. L. Rev. 1908,
November, LN//uwyo-ajl]

Lacanian theory allows us to interpret the meaning of this anti-Masonic vision precisely. Schlag's
bureaucracy must be seen as a "paranoid construction according to which our universe is the work of art
of unknown creators." n273 In Schlag's view, the bureaucracy is in control of law and language and uses it
exclusively for its own purposes. The bureaucracy is therefore the Other of the Other, "a hidden subject
who pulls the strings of the great Other (the symbolic order)." n274 The bureaucracy, in short, is the
superego (i.e., absolute knowledge of the ego), n275 but rendered visible and projected outward. The
superego, the ego's stern master, condemns the ego and condemns what it does. Schlag has transferred
this function to the bureaucracy.

As is customary, n276 by describing Schlag's vision as a paranoid construction, I do not mean to suggest
that Professor Schlag is mentally ill or unable to function. Paranoid construction is not in fact the illness. It
is an attempt at healing what the illness is - the conflation of the domains of the symbolic, imaginary, and
real. n277 This conflation is what Lacan calls "psychosis." Whereas the "normal" subject is split between
the three domains, the psychotic is not. He is unable to keep the domains separate. n278 The symbolic
domain of language begins to lose place to the real domain. The psychotic raves incoherently, and things
begin to talk to [*1947] him directly. n279 The psychotic, "immersed in jouissance," n280 loses desire
itself.

Paranoia is a strategy the subject adopts to ward off breakdown. The paranoid vision holds together the
symbolic order itself and thereby prevents the subject from slipping into the psychotic state in which "the
concrete 'I' loses its absolute power over the entire system of its determinations." n281 This of course
means - and here is the deep irony of paraonia - that bureaucracy is the very savior of romantic
metaphysics. If the romantic program were ever fulfilled - if the bureaucracy were to fold up shop and let
the natural side of the subject have its way - subjectivity would soon be enveloped, smothered, and killed
in the night of psychosis. n282

Paranoid ambivalence toward bureaucracy (or whatever other fantasy may be substituted for it) is very
commonly observed. Most recently, conservatives "organized their enjoyment" by opposing communism.
n283 By confronting and resisting an all-encompassing, sinister power, the subject confirms his existence
as that which sees and resists the power. n284 As long as communism existed, conservatism could be
perceived. When communism disappeared, conservatives felt "anxiety" n285 - a lack of purpose. Although
they publicly opposed communism, they secretly regretted its disappearance. Within a short time, a new
enemy was found to organize conservative jouissance - the cultural left. (On the left, a similar story could
be told about the organizing function of racism and sexism, which, of course, have not yet disappeared.)
These humble examples show that the romantic yearning for wholeness is always the opposite of [*1948]
what it appears to be. n286 We paranoids need our enemies to organize our enjoyment.

Paranoid construction is, in the end, a philosophical interpretation, even in the clinical cases. n287 As
Schlag has perceived, the symbolic order of law is artificial. It only exists because we insist it does. We all

Page 1233 of 1481


fear that the house of cards may come crashing down. Paradoxically, it is this very "anxiety" that shores
up the symbolic. The normal person knows he must keep insisting that the symbolic order exists precisely
because the person knows it is a fiction. n288

The paranoid, however, assigns this role to the bureaucracy (and thereby absolves himself from the
responsibility). Thus, paranoid delusion allows for the maintenance of a "cynical" distance between the
paranoid subject and the realm of mad psychosis. n289 In truth, cynicism toward bureaucracy shows
nothing but the unconfronted depth to which the cynic is actually committed to what ought to be
abolished.

Page 1234 of 1481


Page 1235 of 1481
#9 Normativity Good: 1AR
THE CRITIQUE OF NORMATIVITY IS SIMPLY WRONGWE MUST EMBRACE
NORMATIVE THEORY INSTEAD OF ATTEMPTING TO DECONSTRUCT ITS CLAIMS

Tushnet, Prof of Law @ Georgetown U, 92 (Mark V., The left critique of Normativity: A comment,
Michigan Law Review, August, Lexis)

one might find another tool for rebuilding normative discourse. It is to relinquish
To use Delgado's terms, though,
any normative [*2347] claims for leftist inclinations. n92 Left legal scholarship would be exclusively critical,
deconstructing the normative claims made elsewhere in legal scholarship but offering nothing at all in
their place. This project, too, seems difficult to sustain. Left legal academics walk into classrooms every day in which students
demand that we say what our views are on controverted issues. A stance of unremitting critique will not satisfy them. To face such dissatisfaction routinely is
simply uncomfortable. Thus, even a leftist teacher committed to "only critique" is likely to succumb in the classroom. n93 Because the classroom is where we
try out many of our ideas, it seems likely that the normativity to which this teacher is pushed in the classroom will come to infect his or her scholarship.
There is, of course, an alternative. Perhaps the critique of normativity goes all the way down, in which
case the "only critique" stance is the only one an intellectually honest legal academic can take. But
perhaps the critique of normativity is wrong. Legal academics might then remain committed to the
project of comprehensive normative rationality, and their modest normative gestures would be
promissory notes to be cashed in elsewhere, in the development of a comprehensive normative theory .
n94

Page 1236 of 1481


Page 1237 of 1481
#10 Simulation/Roleplaying
Good: 1AR (1/3)
INFORMAL DELIBERATIVE SITUATIONS, SUCH AS THIS DEBATE ARE A USEFUL
STARTING POINT TOWARDS INFORMING PUBLIC DISCOURSE AND EFFECTUATING
SOCIAL CHANGE.

KULYNYCH IN 1997
(JESSICA, Performing politics: Foucault, Habermas, and postmodern participation Polity, Winter 1997 v30
n2)

Participation equals discursive participation; it is communication governed by rational, communicatively


achieved argument and negotiation. Habermas distinguishes two types of discursive participation:
problem-solving or decision-oriented deliberation, which takes place primarily in formal democratic
institutions such as parliaments and is regulated or governed by democratic procedures; and informal
opinion-formation, which is opinion-formation "uncoupled from decisions... [and] effected in an open and
inclusive network of overlapping, subcultural publics having fluid temporal, social and substantive
boundaries."(11) In many ways this two-tiered description of discursive participation is a radically
different understanding of political participation, and one better suited to the sort of societies we
currently inhabit. Habermas moves the focus of participation away from policymaking and toward
redefining legitimate democratic processes that serve as the necessary background for subsequent
policymaking. While only a limited number of specially trained individuals can reasonably engage in
decisionmaking participation, the entire populous can and must participate in the informal deliberation
that takes place outside of, or uncoupled from, formal decisionmaking structures. This informal
participation is primarily about generating "public discourses that uncover topics of relevance to ail of
society, interpret values, contribute to the resolution of problems, generate good reasons, and debunk
bad ones."(12) Informal participation has two main functions. First, it acts as a "warning system with
sensors that, though unspecialized, are sensitive throughout society."(13) This system communicates
problems "that must be processed by the political system."(14) Habermas labels this the "signal"
function. Second, informal participation must not only indicate when problems need to be addressed, it
must also provide an "effective problematization" of those issues. As Habermas argues, from the
perspective of democratic theory, the public sphere must, in addition, amplify the pressure of
problems, that is, not only detect and identify problems but also convincingly and influentially
thematize them, furnish them with possible solutions, and dramatize them in such a way that they are
taken up and dealt with by parliamentary complexes.

Page 1238 of 1481


Page 1239 of 1481
#10 Simulation/Roleplaying
Good: 1AR (2/3)
THE PROCESS OF DELIBERATION IS AN END IN AND OF ITSELF- EVEN WHEN WE
CANT DIRECTLY INFLUENCE POLICY

KULYNYCH IN 1997
(JESSICA, Performing politics: Foucault, Habermas, and postmodern participation Polity, Winter 1997 v30
n2)

VII. The Politics of Deliberation in a Performative Perspective A performative perspective on participation


enriches our understanding of deliberative democracy. This enlarged understanding can be demonstrated
by considering the examination of citizen politics in Germany presented in Carol Hager's Technological
Democracy: Bureaucracy and Citizenry in the West German Energy Debate.(86) Her work skillfully maps
the precarious position of citizen groups as they enter into problemsolving in contemporary democracies.
After detailing the German citizen foray into technical debate and the subsequent creation of energy
commissions to deliberate on the long-term goals of energy policy, she concludes that a dual standard of
interpretation and evaluation is required for full understanding of the prospects for citizen participation.
Where traditional understandings of participation focus on the policy dimension and concern themselves
with the citizens' success or failure to attain policy preferences, she advocates focusing as well on the
discursive, legitimation dimension of citizen action. Hager follows Habermas in reconstituting
participation discursively and asserts that the legitimation dimension offers an alternative reason for
optimism about the efficacy of citizen action. In the discursive understanding of participation, success is
not defined in terms of getting, but rather in terms of solving through consensus. Deliberation is thus an
end in itself, and citizens have succeeded whenever they are able to secure a realm of deliberative
politics where the aim is forging consensus among participants, rather than achieving victory by some
over others. Through the creation of numerous networks of communication and the generation of
publicity, citizen action furthers democracy by assuming a substantive role in governing and by forcing
participants in the policy process to legitimate their positions politically rather than technically. Hager
maintains that a sense of political efficacy is enhanced by this politically interactive role even though
citizens were only minimally successful in influencing or controlling the outcome of the policy debate,
and experienced a real lack of autonomy as they were coerced into adopting the terms of the technical
debate. She agrees with Alberto Melucci that the impact of [these] movements cannot.., be judged by
normal criteria of efficacy and success .... These groups offer a different way of perceiving and naming the
world. They demonstrate that alternatives are possible, and they expand the communicative as
opposed to the bureaucratic or market realms of societal activity.(87)

INFORMAL PARTICIPATION REDUCES OUR ROLES AS CONSUMING CITIZENS,


INCREASING THE LIKELIHOOD THAT WE WONT BE PASSIVE TO THE REGIMENTING
PROCESSES OF THE BUREAUCRACY.

KULYNYCH IN 1997

Page 1240 of 1481


(JESSICA, Performing politics: Foucault, Habermas, and postmodern participation Polity, Winter 1997 v30
n2)

When we look at the success of citizen initiatives from a performative perspective, we look precisely at
those moments of defiance and disruption that bring the invisible and unimaginable into view. Although
citizens were minimally successful in influencing or controlling the out come of the policy debate and
experienced a considerable lack of autonomy in their coercion into the technical debate, the goal-
oriented debate within the energy commissions could be seen as a defiant moment of performative
politics. The existence of a goal-oriented debate within a technically dominated arena defied the
normalizing separation between expert policymakers and consuming citizens. Citizens momentarily
recreated themselves as policymakers in a system that defined citizens out of the policy process,
thereby refusing their construction as passive clients. The disruptive potential of the energy
commissions continues to defy technical bureaucracy even while their decisions are non-binding.

Page 1241 of 1481


Page 1242 of 1481
#10 Simulation/Roleplaying
Good: 1AR (3/3)
INSULAR DELIBERATION AND SIMULATING THE POLITICAL PROCESS CAN
GENERATE IMPORTANT VIEWPOINTS ON ISSUES FACING SOCIETY

MITCHELL AND SUZUKI IN 2004


(GORDON AND TAKESHI, UNIV OF PITTSBURGH COMMUNICATIONS PROFESSOR AND TSUDA COLLEGE
COMMUNICATIONS PROFESSOR, BEYOND THE DAILY ME: ARGUMENTATION IN AN AGE OF ENCLAVE
DELIBERATION, PAPER PRESENTED AT THE SECOND TOKYO CONFERENCE ON ARGUMENTATION AUGUST
2-5, 2004, TOKYO, JAPAN)

One should not be too quick to dismiss the value of tournament debating purely on the grounds that it
unfolds in obscure enclaves. Such activity benefits greatly the modest number of debaters who are able
to learn the games arcane rules and invest the substantial resources required for tournament travel
(Muir, 1990; Panetta, 1993). Recall Sunsteins stipulation enclave deliberation is not intrinsically bad
it all depends on whether the walls insulating particular discourse communities are temporary or
permanent. While it is true that insular deliberative groups can generate truly novel viewpoints on
important issues facing society, such views can only deepen societys overall argument pool if
eventually, such groups turn outward to communicate with those beyond their tight circle of members
(Cox & Jensen, 1989; Weiss, 1987).

Page 1243 of 1481


Page 1244 of 1481
#11 Alt Lapses Back into NLT:
1AR
SUBJECT FREE FROM LAW CANNOT EXIST

CARLSON & SCHROEDER IN 2003


(JEANNE AND DAVID, CARDOZO LAW PROFESSORS, 57 U MIAMI L. REV 767)

Lacanian theory shows the defects in both normative policy scholarship and romanticism--the two
dominant modes of thinking in American law schools. As for the latter, the Lacanian concept of the
subject's false autobiography helps explain why a romantic faith in the wholeness of the subject apart
from the law cannot be accepted. We find that although Pierre Schlag intuits the Lacanian insight of the
split subject castrated by artificial law, he implies a romantic liberal vision of a self-identical,
uncastrated subject who could exist in a mythical state of nature free from law's corrupting influence.
Lacan teaches otherwise. He suggests that law is a constituent part of the constitution of the subject. To
lay down the law, as Schlag suggests, is to lay down our subjectivity. The law cannot be escaped. Better
to make it our work product, so that we recognize ourselves in the law.

SCHLAGS CRITIQUE IS BOUND BY THE RHETORIC HE CRITICIZESHE FAILS TO


BREAK FROM THE NARROWNESS OF THE LAW

Conaghan, Prof @ Kent Law School, 2K3 (Joanne, Beyond Right and Reason: Pierre Schlag, the
Critique of Normativity, and the Enchantment of Reason: Schlag in Wonderland, Miami Law Review,
April, Lexis)

A final concern emerging from the confines of Schlag's selective mimicry of the mainstream lies in its resolutely legal character. American legal scholars do not,
by and large, like to stray too far beyond the boundaries of what is acceptably "legal" n65 and interestingly, neither doesSchlag. He/they prefers the
snug confines of traditional legal discourse and its discontents, modestly professing ignorance and lack of expertise beyond the
terrain of law, narrowly understood as judicial decisions and the doctrines and theories legal scholars derive from them. Schlag bemoans this
narrowness repeatedly but seems in no great hurry to escape it. Indeed, one sometimes wonders whether or not his insistence
on so limited an enquiry masks a fear of his moving beyond what he has experienced as safe and steady ground. By his own admission, this is the critique of
"an insider," n66 but does it simultaneously affirm the attractions of remaining "inside"? This dogged determination to steer clear of the complexities that an
might introduce is also manifest in Schlag's exclusive preoccupation with reason's aesthetic
extra-legal dimension
appeal. While I applaud his efforts to draw attention to the coercive power of particular aesthetic forms--in the context of law, the compelling effects of
grid-like manifestations of reason--his neglect of, indeed total silence in relation to, other features of law's coerciveness puts him at risk of overstating his case.
This is particularly so when what is neglected is so closely bound up with what he addresses at such length. Here, I am thinking in particular of the ideological
context within which law operates and upon which reason seeks to make her mark. In my view, there is an ideological dimension to the effective deployment
of reason that is not, or is only secondarily, dependent upon its aesthetic form. There
is a detectable distinction (not always but sometimes)
between invocations of reason that are dependent upon the political and ideological landscape for their
validity and deployments of reason that [*557] draw upon (or seek to develop) our aesthetic inclinations,
particularly our attraction to order and coherence. n67 Often, what seems reasonable is inextricably related
to our understanding of what is possible, and yet, it is not always the case that what is possible is
determined by the boundaries of reason. The ideological landscape abounds with all of the "sources of belief" making an appearance in
Schlag's critique. The point is that reason as a particular aesthetic does not always work to disqualify reason as

Page 1245 of 1481


a repository for widely held ideological beliefs. Although the former may contribute to understandings of the latter, it may not wholly
determine (or be determined by) them. A failure to acknowledge this explicitly arguably serves to weaken the power of
Schlag's critique. There are times when he invokes a primarily ideological concept of reason--one that
relies on notions of truth, self-evidence, and righteousness--and then proceeds to critique it for its failure
to adhere to an aesthetic form. Sometimes, this is effective, and it is almost always amusing. n68 At other times, one has a sense that the boot
does not fit, that he is over-emphasizing the importance of the schematic structure of the argument in circumstances where its success has little to do with its
schematic structure and everything to do with its correspondence to the ideological status quo. Put bluntly, if
reason's appeal to self-evidence
(Sunstein) or virtue (Nussbaum) is dependent upon factors beyond its internal logic, it is not thereby
significantly diminished by demonstrating that that logic has reached its limits. Schlag's account of the wonderland of
American legal scholarship is undoubtedly perceptive; his dissection of the stances adopted by those who typify it both
masterly and liberating, and his representation of his own alienation intensely resonant of the
experiences of many who occupy the margins of the legal academy. Indeed, therein lies its appeal. But by the same token, it is
at times injudicious in its forays into "hostile" terrain. It fails adequately to guard against the dangers of importation, co-
option, domestication, and reproduction. It constitutes even as it deconstructs. In Schlagean terms, the
power of his critique is diminished by neglect of aspects of the "rhetorical economy" with which he is
engaging. n69 In simpler terms, there appear to be dimensions to his enchantment of which he is unaware.

Page 1246 of 1481


Page 1247 of 1481
#11 Alt Lapses Back into NLT:
Ext
THE ALTERNATIVE LAPSES BACK INTO REASON AND NLT

University of Miami 2003


[Pierre Schlag, the Critique of Normativity, and the Enchantment of Reason: Smoking in Bed 57 U.
Miami L. Rev. 827, April, LN//uwyo-ajl]

However, Schlag knows all this. Indeed, he writes, "critical reflexivity is not invariably or even intrinsically
liberating or empancipatory. On the contrary, pushed to its limits, it is single-minded and formalistic." n54
Exactly. And critical reflexivity can not tell you when you are onto a good thing versus a bad one, n55 or in
a "good practice" [*838] versus a "bad one." So? Why so hard on the proponents of Reason? After all,
they are just trying to get somewhere. For them, reason is the means to the end. Yet Schlag suggests they
want more. The proponents of Reason, he claims, want to have their cake and eat it too. n56 He indicts
this fantasy and says that the pretense that one can have it both ways is what keeps academics focusing
on the wrong questions. This can lead to the question, "Why is critical reflexivity so unrewarded?" Well, it
could be because if belief in Reason is a faith that we believe can answer "the big question," and that
belief simultaneously rejects faith as an answer, critical reflexivity will expose the very thing rejected by
the faithful - the inadequacy of their "answer" by the dictates of their faith. In other words, Schlag uses
reason to expose unreason in Reason. n57

By using reason to expose unreason, however, Schlag too arguably asks the "wrong" questions, making
the case of "what is a legal academic to do?" seem more desperate than it is. The questions he appears to
think are the ones worth pursuing seem to me to be precisely the ones that can not be answered. At least
not with any more reliability than the questions he claims are the wrong questions. Moreover, the
difference between those questions that Schlag claims are the "wrong" questions and the ones he claims
are the "right" questions is that the "wrong" ones are a prelude to or a call to action (even if no real action
follows). That is, even if "advocating 'progressive legal change' <noteq> advancing progressive legal
change" n58 the question of how to do so appears to be one about what actions to take.

Page 1248 of 1481


Page 1249 of 1481
Normative Thought Inevitable
(1/3)
IT IS IMPOSSIBLE TO ESCAPE NORMATIVE CLAIMS-MIGHT AS WELL TRY TO LIVE
WITH IT.

RADIN AND MICHELMAN IN 1991


( MARGARET JANE AND FRANK, STANFORD AND HARVARD LAW PROFESSORS, 139 U PA. L. REV 1019,
APRIL)

What should we do? What should the law be? What do you propose?' . . . asks normative legal thought."
n11 Normative, we thus understand, is what every prescriptive utterance is; normativity marks every
saying addressed to a question of what someone should (or should not) do. Now, it seems obviously
correct that normativity, thus sweepingly defined, is pandemic in legal thought and writing. But so is it
pandemic, we would say, in thought and writing about legal thought -- as represented, say, by the articles
in this symposium.

[*1021] To work, in writing, at the displacement or destabilization of some named practice of writing
(like normative legal thought) n12 is already to exemplify and thereby to commend some different,
some critically chastened, practice. n13 Moreover, it is extremely difficult to carry on the work of
destabilization without appearing to lapse into normative modes of discourse. Take, for example, this
passage from an article by Schlag:

[T]his [talk-talk genre] simply argues that we should talk [some] new talk. . . . Variations on this old
talk/new talk include the following: we should talk . . . more normatively, [or] more contextually . . . [etc.]
or in that hopeful humanist way until we figure out what the hell we're doing up here 30,000 feet from
earth arguing about how we should land. n14

"We should talk more normatively" (WSTMN, for short) is the name of a certain sentence -- the one that
says we should talk more normatively. If uttering WSTMN is contemptible as just talk or as normative talk
(and, to boot, as naively presupposing that how we talk, what we do, is within our power to decide n15 ),
then what is a reader supposed to make of the sentence that says that uttering WSTMN is contemptible
on those grounds? It seems that saying that cannot (coherently) be an argument about whether or how
we should (or should not) talk. How can one argue that what makes an utterance (or a genre) unworthy
of attention or respect is that it is normative talk? To argue is to invoke the practice of argument, and
that practice consists of normative talk. (Maybe you could try by some other means to remove that
practice from society's repertoire, but you can't well do that by arguing about it.) But if this utterance of
Schlag's is not argument, then what is it?

Page 1250 of 1481


Page 1251 of 1481
Normative Thought Inevitable
(2/3)
NORMATIVE THOUGHT IS INEVITABLE DISCUSSING THE RULE OF LAW IS THE
ONLY WAY TO PREVENT COMPLETE DESTRUCTION OF RIGID CONCEPTIONS OF
LEGAL THEORY

Mootz, Assoc Prof of Law @ Western New England College School of Law, 94 (Francis J., The Paranoid Style in
Contemporary Legal Scholarship, Houston Law Review, Fall, Lexis)

The differences between my conception of postmodern legal theory and Schlag's are highlighted by our very different reactions to the idea of the rule of law.
Schlag regards the rule of [*883] law as a "virtually empty" signifier whose sole purpose is "simply to arrest thought upon impact." n36 Schlag does not
propose to reformulate the idea of the rule of law, or even to replace it with a more fitting concept, because such moves would circle within the same vacuous
maze of normative legal thought. n37 Schlag's disengagement from the language used by lawyers and judges is so stark and unrepentant that its significance
easily is underestimated. In an important sense, the ongoing struggle over the terms and conditions of social organization defines Western history. A significant
Schlag bifurcates the
feature of this struggle has been the ongoing effort to describe what it means for a society to be governed by the rule of law.
operation of the legal system from the discourse of its participants, arguing that the normative claims
made by those attempting to describe what the rule of law entails is superfluous to the reality of law. By
doing so, he openly places in question whether discourse can describe, not to mention influence, practice .
n38 Admittedly, much of the "fancy" scholarship of the academy is removed from the everyday language of legal practice, but the assertion that every
theoretical invocation of the rule of law is detached from some deeper, hidden, nonlinguistic realm of legal reality greatly overstates the case. The extent of
critical detachment presumed by Schlag's total rejection of the usefulness of discussing the rule of law is quite fantastic. An individual who truly could achieve
this detachment would be exhibiting the paranoid style. n39 I [*885] wholeheartedly share Schlag's assessment that the justificatory efforts of judges and
scholars alike to define the rule of law has been framed by the unhelpful polarity of justify and redeem and constrain and control strategies. n40 Yet the
recognition that past formulations no longer suffice leads me to attempt to articulate a new conception of
the rule of law that accords with our experience. n41 It is possible to destroy rigid conceptions of the rule
of law without embracing endless deconstruction that renders further discussion moot. Schlag is correct that the
traditional accounts of the rule of law often are caricatures that arrest thought and discussion, n42 but I argue that we should resume a vital
discussion rather than conclude that all discussion inherently is vacuous. The criticism that rule of law talk doesn't capture
reality reveals a wistfulness for the foundationalist hope of discovering a political truth that is not subject to a contingent, ongoing dialogue among members of
By claiming that everyone else is trapped in a meaningless maze, Schlag conveniently avoids placing
society.
himself at risk in normative dialogue. By asserting that normative legal dialogue is irrelevant, Schlag
eliminates the possibility that he might have to change his mind in light of the force of a better argument,
and he avoids an obligation to rescue the hoi polloi from the maze. In sum, Schlag's approach insulates him from the
contingent and provisional language of social discourse. Such an insulating move runs contrary to antifoundational accounts of the rule of law, which
that the law never operates outside the context of wider social struggles to define the terms of
emphasize
sociopolitical organization. Traditional normative legal thought ordinarily is criticized as being unhelpful
because it offers a constricted and artificial conception of legal norms, not because normative legal
thought is by nature irrelevant to legal practice. Quite the opposite seems true: every assertion of legal power is
predicated on a normative conception of politics that always is subject to attack and reassessment.
Escape from the maze of normative legal thinking is the [*886] familiar dream of empiricists and rationalists alike, but it simply is
not possible. Talking about the reality of law as distinct from our representation of this reality in normative legal dialogue constitutes a performative
contradiction. n43 This is not to say that reality is wholly linguistic, but rather that our experience and understanding of reality is always linguistically mediated
in a shared realm of normative public dialogue. n44

NORMATIVE THOUGHT CANNOT BE COMPLETELY DESTROYEDWE SHOULD


FOCUS ON CLEARING A WAY THROUGH THE MAZE INSTEAD OF REJECTING IT

Page 1252 of 1481


Mootz, Assoc Prof of Law @ Western New England College School of Law, 94 (Francis J., The Paranoid Style in
Contemporary Legal Scholarship, Houston Law Review, Fall, Lexis)

As Hilary Putnam concisely states, "the elimination of the normative is attempted mental suicide." n49 I would refine Putnam's
Schlag writes powerfully, invariably capturing my interest and
observation by including paranoid distanciation within the scope of mental suicide. Professor
leading me to important new insights. However, his effort to distance himself from the normative legal language that is our
heritage falls short, as it must. I congratulate Schlag for his skill in destroying some of the most cherished talismans in our legal vocabulary,
including the rule of law. But destruction is never total. In the wake of destruction we inevitably chart new paths in
the maze. Legal theory properly is viewed not as an attempt to escape the maze of normative legal
thought, but as an effort to develop shared strategies for navigating through the maze. Forging a path,
rather than finding an exit, is the goal. That is enough for me.

Page 1253 of 1481


Page 1254 of 1481
Normative Thought Inevitable
(3/3)
SCHLAGS CHARACTERIZATION OF THE MAZE FAILS TO TAKE ITS FUNCTION WITHIN
CRITICAL THEORY INTO ACCOUNTESCAPE FROM THE MAZE IS IMPOSSIBLE

Mootz, Assoc Prof of Law @ Western New England College School of Law, 94 (Francis J., The Paranoid Style in
Contemporary Legal Scholarship, Houston Law Review, Fall, Lexis)

The epistemological problems posed by modernist critical projects are only partially answered by adding a postmodern gloss. Schlag's effort to
analyze legal scholarship from outside the maze is extremely problematic. Schlag believes that most
scholars reside within a maze characterized by "dreariness," but that a select few have found a way out,
gained perspective [*879] on the maze, and now engage in a fruitful questioning that reveals rather than
obscures the law. n20 In sharp contrast, I reject the idea that such a dramatic escape can take place. Just when a
scholar believes that she has scaled the last wall of the maze, she will be confronted by a boundless
horizon of paths endlessly circling within the ambit of the same maze. Hope for escape must always be dashed in the end,
but this does not mean that an individual's comportment within the maze is without ethical or political significance. The central problem for
contemporary jurisprudence is not the maze of normative legal discourse, but the failure to recognize the maze as an
unavoidable condition that is productive of knowledge. Postmodern thought is a stimulating force, but it has been overused and
abused by more than one scholar in search of a truly radical break from the politics of normalcy. The questions raised by the maze are much more subtle and
complex than Schlag allows. Schlag's
confusion over what the maze represents, how it operates, and the
consequential function of critical theory, exemplifies the postmodern crisis in legal theory . Put differently, Schlag's
characterization of the maze, offered with a sly wink and a conspiratorial nod to others in the know, comes off sounding just a bit paranoid.

Page 1255 of 1481


Page 1256 of 1481
Alternative Fails
SCHLAGS REFUSAL TO DELINEATE A PRECISE OBJECT OF HIS CRITIQUE CAUSES HIS
KRITIK TO BE CO-OPTED INTO THE VERY NORMATIVE SYSTEM HE CHALLENGES
WHILE HE IGNORES KEY NORMATIVE STRUCTURES WE NEED TO CRITICIZE

Conaghan, Professor @ Kent Law School, 2K3 (Joanne, Beyond Right and Reason: Pierre Schlag, the Critique of
Normativity, and the Enchantment of Reason: Schlag in Wonderland, Miami Law Review, April, Lexis)

Schlag's refusal to delineate with any precision the object of his critique is not a risk-free
Nevertheless,
strategy. One difficulty arising is that reason remains deliciously ephemeral throughout, assuming a [*550] dream-
like, shadowy quality that at times heightens its allure and triggers a desire to capture and contain it. This is of course
a reflection of Schlag's own ambivalence towards reason, signalled in particular by his use of the word "enchantment" n29 to denote our (his?) affinity to it.
Schlag's portrayal of reason is that of a siren, a femme fatale, who simultaneously entices and deceives.
And, while he urges us endlessly to recognize her pathological tendencies, we remain suspicious that he is
still in her thrall. More importantly, however, the nebulous quality of Schlag's invocations of reason is misleading
and belies the prescriptive content of the notion(s) he deploys. Reason, for Schlag's purposes, is bounded in ways
he does not openly acknowledge. Woven within the fabric of his critique is a particular perspective from
which reason's purposes are derived and its shortcomings identified and assessed.

ALT CANT SOLVE THE NORMS YOU TRY AND CHANGE WONT TRANSFER TO THE
PUBLIC SPHERE YOU CAN ONLY CHANGE ONE INSTANCE OF BAD DISCOURSE

Habermas, Prof @ Goethe U in Frankfurt, 90 (Jurgen, Discourse Ethics: Notes on a Program of


Philosophical Justification, The Communicative Ethics Controversy, Ed. Benhabib and Dallmayr, P. 82-83)

Admittedly, a second objection can be raised against such arguments, one that is not so easily refuted. True
as it may be that freedom of
opinion in the sense of freedom from external interference in the process of opinion formation is one of
the inescapable pragmatic presuppositions of every argumentation, the fact remains that what the
skeptic is now forced to accept is no more than a the notion that as a participant in a process of
argumentation he has implicitly recognized a principle of freedom of opinion'. This argument does not go far enough to
convince him in his capacity as an actor as well. The validity of a norm of action, as for example a publicly guar- anteed constitutional right to freedom of
expression, cannot be justified in this fashion. Itis by no means self-evident that rules which are unavoidable within
discourses can also claim to be valid for regulating, action outside of discourses. Even if participants in an
argumentation are forced to make substantive normative presuppositions (e.g., to respect one another as Competent
subjects; to treat one another as equal partners; to assume one another's truthfulness; and to cooperate with one another),34 they could still shake
off this transcendental pragmatic compulsion when they leave the field of argumentation. The necessity of making
such presuppositions is not transferred directly from discourse to action. In any case, a separate justification would be required to
explain why the normative content discovered in the pragmatic presuppositions of argumentation should
have the power to regulate action.

Page 1257 of 1481


Page 1258 of 1481
Pragmatism Good
PRAGMATICALLY COMBINING THE INSIGHTS OF THE CRITICISM WITH THE AFF
SOLVES BEST

RADIN AND MICHELMAN IN 1991


( MARGARET JANE AND FRANK, STANFORD AND HARVARD LAW PROFESSORS, 139 U PA. L. REV 1019,
APRIL)

The poststructuralist moment in critical practice is conceptual, diagnostic, and global. It fastens on
intellectual structures and denies their analytic probity. It indicts whole discourses and all their works
by showing their conceptual, categorical frameworks in a state of collapse. In the poststructuralist
frame of mind, we search for dialectical fault lines implanted in discursive frameworks. We deflate
argumentative paradigms built around a characteristic set (one for each target jurisprudence) of
categories, distinctions, and oppositions. We show their failures of closure -- perhaps by exposing
addiction to a "fundamental contradiction," n51 perhaps by exposing tactics of recursion and deferral.
n52

The pragmatist moment in critical practice is, by contrast, empirical, epidemiological, and local. It
notices characteristic kinds of errors or biases that recur when target discourses are deployed by
nonideal -- incompletely committed and assiduous -- practitioners caught in specific cultural
environments. n53 The pragmatically minded critic does not deny or ignore conceptual instability.
Neither does she hold that conceptual instability per se discredits a framework. Indeed, she does not
especially care to discredit any discourse intrinsically or holistically. She rather seeks to evaluate the
discourse in use (given its conceptual instabilities) by ordinarily complacent, culturally bound
practitioners. She asks, for example, about the tendency of the discourse, in its cultural setting, to focus
[*1032] on some problems and blur others. Pragmatically successful critique does not necessarily mean
that practitioners give up use of the framework. It may mean, rather, that they watch out and correct
for biases to which the culturally situated framework is prone.

Page 1259 of 1481


Page 1260 of 1481
**Nuclearism**

Page 1261 of 1481


Nuclearism Answers: 2AC (1/3)
FIRST, PERM DO BOTH
NUCLEARISM CANT SOLVE WITHOUT A POLITICS

Lifton & Falk 82


[Robert Jay & Richard, Prof. Psychiatry * Prof Intl Affairs, Indefensible Weapons: The Political and
Psychological Case Against Nuclearism, New York: Basic Books, 133]

. The entrenched forces that stand behind nuclearism are powerful and wily, and, if
yet we must not be too encouraged

necessary, ruthless. Popular movements are notoriously easy to coopt, divert, infiltrate, bore, and outlast.
For the antinuclear movement to succeed, it desperately needs a politics, that is, a clear understanding of what must be
changed and how to do it. This understanding of what must be changed and how to do it. This understanding must also include an alternative idea of security. The antinuclear
ranks are not composed of idealists who believe that peace on earth, goodwill to men and women is an idea whose time has come. Overwhelmingly they are acting out of fear

of the nuclear menace, increasingly deciding that this fear takes precedence over their more traditional concerns about national defense and preserving a way of life. But in
the end this movement will not succeed unless it combines a negation of nuclearism with the persuasive
creation of new ways to protect independence and territorial integrity of the states that make up world
society. At this time, then, it is crucial to initiate discussions of the politics of antinuclearism. My hope is that this book is read primarily as a contribution to this work.

SECOND, THEY HAVENT DISPROVED OUR TRUTH CLAIMS IN THE STATUS QUO
THERE REALLY IS A THREAT OF NUCLEAR DISASTER. UNLESS THEY TAKE OUT THE
IMPACT, VOTE AFF.

THIRD, REALISM SOLVES THEIR ARGUMENT NUMBING IS IRRELEVENT IF


DETERRENCE AND SELF-INTEREST PREVENT AGENTS FROM USING NUCLEAR
WEAPONS. CROSS-APPLY KHALILZAD

FOURTH, NUCLEARISM IS INEVITABLE MAINTENANCE AND DETERRENCE ARE


NECESSARY FOR WORLD PEACE

Robinson 2001
[C. Paul, Sandi National Laboraties, A White Paper:Pursuing a New Nuclear Weapons Policy for the 21st Century,
March 22, www.mindfully.org/Nucs/Nuclear-Weapons-Policy-21stC.htm, 9-23-06//uwyo-ajl]

I served as an arms negotiator on the last two agreements before the dissolution of the Soviet Union and have spent most of my career enmeshed in the complexity of nuclear
weapons issues on the government side of the table. It is abundantly clear (to me) that formulating a new nuclear weapons policy for the start of the 21st Century will be a most
difficult undertaking. While the often over-simplified picture of deterrence during the Cold War-two behemoths armed to the teeth, staring each other down-has thankfully

, there are nevertheless huge arsenals of nuclear weapons and delivery systems, all in quite usable
retreated into history

states, that could be brought back quickly to their Cold War postures. Additionally, throughout the Cold War and ever since, there
has been a steady proliferation of nuclear weapons and other weapons of mass destruction by other nations around the globe. The vast majority of these newly armed states

Page 1262 of 1481


are not U.S. allies, and some already are exhibiting hostile behaviors, while others have the potential to become aggressors toward the U.S., our allies, and our international
interests.

Russia has already begun to emphasize the importance of its arsenal of nuclear weapons to compensate for its limited conventional capabilities to deal with hostilities that
appear to be increasing along its borders. It seems inescapable that the U.S. must carefully think through how we should be preparing to deal with new threats from other
corners of the world, including the role that nuclear weapons might serve in deterring these threats from ever reaching actual aggressions.

I personally see the abolition of nuclear weapons as an impractical dream in any foreseeable future. I came to this view

from several directions. The first is the impossibility of ever "uninventing" or erasing from the human
mind the knowledge of how to build such weapons. While the sudden appearance of a few tens of nuclear weapons causes only a small stir in a world where
several thousands of such weapons already exist, their appearance in a world without nuclear weapons would produce huge

effects. (The impact of the first two weapons in ending World War II should be a sufficient example.) I believe that the words of Winston Churchill, as quoted by Margaret
Thatcher to a special joint session of the U.S. Congress on February 20, 1985, remain convincing on this point: "Be careful above all things not to let go of the atomic weapon
until you are sure, and more sure than sure, that other means of preserving the peace are in your hands."

the majority of the nations who have now acquired arsenals of nuclear weapons
Similarly, it is my sincere view that

believe them to be such potent tools for deterring conflicts that they would never surrender them. Against this
backdrop, I recently began to worry that because there were few public statements by U.S. officials in reaffirming the unique role which nuclear weapons play in ensuring U.S.
and world security, far too many people (including many in our own armed forces) were beginning to believe that perhaps nuclear weapons no longer had value. It seemed to
me that it was time for someone to step forward and articulate the other side of these issues for the public: first, that nuclear weapons remain of vital importance to the security

nuclear weapons will likely have an enduring role in


of the U.S. and to our allies and friends (today and for the near future); and second, that

preserving the peace and preventing world wars for the foreseeable future. These are my purposes in writing this paper.

Page 1263 of 1481


Page 1264 of 1481
Nuclearism Answers: 2AC (2/3)
FIFTH, IMAGINING NUCLEAR ANNIHILATION IS A PROJECT OF SURVIVAL THEIR
ALTERNATIVE CREATES REPRESSION AND DENIAL WHICH MAKES NUCLEAR WAR
MORE LIKELY

Lenz, Science and Policy Professor at SUNY, 90 (Nuclear Age Literature For Youth, p. 9-10)

A summary of Franks thought in Psychological Determinants of the Nuclear Arms Race notes how all
people have difficulty grasping the
magnitude and immediacy of the threat of nuclear arms and this psychological unreality is a basic obstacle to eliminating
that threat. Only events that people have actually experienced can have true emotional impact. Since Americans have escaped the devastation of nuclear
weapons on their own soil and nuclear weapons poised for annihilation in distant countries cannot be seen, heard, smelled, tasted, or touched, we find
it easy to imagine ourselves immune to the threat. Albert Camus had the same phenomenon in mind when he wrote in his essay
Neither Victims nor Executioners of the inability of most people really to imagine other peoples death (he might have added or their own). Commenting on
Camus, David P. Barash and Judith Eve Lipton observed that this distancing from deaths reality is yet another aspect of our
insulation from lifes most basic realities. We make love by telephone, we work not on matter but on machines, and we kill and are killed
by proxy. We gain in cleanliness, but lose in understanding. If we are to heed Camuss call to refuse to be either the victims of
violence like the Jews of the Holocaust, or the perpetrators of it like the Nazi executioners of the death camps, we must revivify the
imagination of what violence really entails. It is here, of course, that the literature of nuclear holocaust can
play a significant role. Without either firsthand experience or vivid imagining, it is natural, as Frank points out, to deny the
existence of death machines and their consequences. In psychiatric usage denial means to exclude from
awareness, because letting [the instruments of destruction] enter consciousness would create too strong a level of anxiety or other painful emotions. In
most life-threatening situations, an organisms adaptation increases chances of survival, but ironically, adapting ourselves to nuclear fear is
counterproductive. We only seal our doom more certainly. The repressed fear, moreover, takes a psychic toll.

SIXTH, WE DO NOT REALLY KNOW THE IMPACT TO NUCLEAR WAR- DENYING THAT
DESTRUCTION CAN OCCUR THROUGH THE CRITICISM FURTHERS NUMBING

Lifton and Markusen, Prof of International Relations @ Princeton U and Assist Researcher @ U of New
York, 90 (Robert Jay and Eric, The Genocidal Mentality, P. 203)

Not only are we much more ignorant about what we call nuclear
Dissociation is called forth to cover over and deny ignorance.
war than we care to admit, but "we don't know how much we do or do not know about it." Since, as the Israeli
philosopher Avner Cohen points out, "we do not really know how to conceive of nuclear warfare as a concrete actuality, how it could be properly kept under
control and how it might be brought to termination," it is less than responsible to claim how such an event could be "managed, controlled or concluded." But
all evidence suggests that "no matter what nuclear war might be, it would not be the kind of rule-
governed practice" often assumed on the basis of past wars. And while the principle of deterrence has a long history in political
and military practice going back to the time of the Greek city-states, the consequences, should deterrence fail and the deterrer act on his threat, were always
limited: after the war and destruction, there would be recovery and resumption of life. Precisely
the present absence of those limits
"should deterrence fail," the uncertainty or unlikelihood of any significant amount of human life
remaining, radically distinguishes nuclear deterrence from that tradition. Dissociation, especially in the
form of psychic numbing, helps blur that distinction by denying not only our ignorance but also what we
can be expected to know.

Page 1265 of 1481


Page 1266 of 1481
Nuclearism Answers: 2AC (3/3)
SEVENTH, CRITICIZING NUCLEAR REPRESENTATIONS DOESNT PRECLUDE THE NEED
FOR CONCRETE ACTION

Richard Rorty, Professor of Humanities, University of Virginia, Truth, Politics, and Postmodernism, Spinoza Lectures,

1997, p. 51-2

This distinction between the theoretical and the practical point of view is often drawn by Derrida, another writer who enjoys demonstrating that something very important
meaning, for example, or justice, or friendship is both necessary and impossible. When asked about the implications of these paradoxical fact, Derrida usually replies that

the paradox doesn't matter when it comes to practice. More generally, a lot of the writers who are labeled
`post-modernist; and who talk a lot about impossibility, turn out to be good experimentalist social democrats when it
comes to actual political activity. I suspect, for example, that Gray, Zizek, Derrida and I, if we found ourselves citizens of the same country, would all be
voting for the same candidates, and supporting the same reforms. Post-modernist philosophers have gotten a bad name because of their paradox-mongering habits, and their
constant use of terms like `impossible; `self-contradictory' and `unrepresentable'. They have helped create a cult of inscrutability, one which defines itself by opposition to the
Enlightenment search for transparency - and more generally, to the `metaphysics of presence; the idea that intellectual progress aims at getting things clearly illuminated,

. I am all for getting rid of the metaphysics of presence, but I think that the rhetoric of
sharply delimited, wholly visible

impossibility and unrepresentability is counterproductive overdramatization. It is one thing to say that we need to get rid of the
metaphor of things being accurately represented, once and for all, as a result of being bathed in the light of reason. This metaphor has created a lot of headaches for
philosophers, and we would be better off without it. But that does not show that we are suddenly surrounded by unrepresentables; it just shows that `more accurate

Even if we agree that we shall never have what Derrida calls "a full
representation' was never a fruitful way to describe intellectual progress.

presence beyond the reach of play"; our sense of the possibilities open to humanity will not have
changed. We have learned nothing about the limits of human hope from metaphysics, or from the philosophy of history, or from psychoanalysis. All that we have learned
from `post-modern' philosophy is that we may need a different gloss on the notion of `progress' than the rationalistic gloss which the Enlightenment offered. We have

been given no reason to abandon the belief that a lot of progress has been made by carrying out the
Enlightenment's political program. Since Darwin we have come to suspect that whether such progress is made will be largely a matter of luck. But we
have been given no reason to stop hoping to get lucky.

EIGHTH, MEDIA IMAGES PLAY THE CRUCIAL ROLE OF REVEALING THEIR OWN
ILLUSIONS

Jean Baudrillard, professor of philosophy of culture and media at Univ. or Paris, 1994, Illusion of the End,
pg. 60-61

And yet there will, nonetheless, have been a kind of verdict in this Romanian affair, and the artificial
heaps of corpses will have been of some use, all the same. One might ask whether the Romanians, by the
very excessiveness of this staged event and the simulacrum of their revolution, have not served as
demystifiers of news and its guiding principle. For, if the media image has put an end to the credibility of
the event, the event will, in its turn, have put an end to the credibility of the image. Never again shall we
be able to look at a television picture in good faith, and this is the finest collective demystification we
have ever known. The finest revenge over this new arrogant power, this power to blackmail by events.
Who can say what responsibility attaches to the televisual production of a false massacre (Timisoara), as
compared with the perpetrating of a true massacre? This is another kind of crime against humanity, a
hijacking of fantasies, affects and the credulity of hundreds of millions of people by means of television
a crime of blackmail and simulation. What penalty is laid down for such a hijacking? There is no way to
rectify this situation and we must have no illusions: there is no perverse effect, nor even anything
scandalous in the Timisoara syndrome. It is simply the (immoral) truth of news, the secret purpose
[destination] of which is to deceive us about the real, but also to undeceive us about the real. There is no

Page 1267 of 1481


worse mistake than taking the real for the real and, in that sense, the very excess of media illusion plays a
vital disillusioning role. In this way, news could be said to undo its own spell by its effects and the violence
of information to be avenged by the repudiation and indifference it engenders. Just as we should be
unreservedly thankful for the existence of politicians, who take on themselves the responsibility for that
wearisome function, so we should be grateful to the media for existing and taking on themselves the
triumphant illusionism of the world of communications, the whole ambiguity of mass culture, the
confusion of ideologies, the stereotypes, the spectacle, the banality soaking up all these things in their
operation. While, at the same time, constituting a permanent test of intelligence, for where better than
on television can one learn to question every picture, every word, every commentary? Television
inculcates indifference distance, scepticism and unconditional apathy. Through the worlds becoming-
image, it anaesthetizes the imagination, provokes a sickened abreaction, together with a surge of
adrenalin which induces total disillusionment. Television and the media would render reality [le reel]
dissuasive, were it not already so. And this represents an absolute advance in the consciousness or the
cynical unconscious of our age.

Page 1268 of 1481


Page 1269 of 1481
#1 Permutation: 1AR
THE PERMUTATION TO DO THE PLAN WHILE RETHINKING SOLVES BEST THEIR
OWN AUTHOR SAYS THAT THERE IS NO SINGLE TRUTH ENGAGING IN POLITICAL
ACTION AND RECOGNIZING THE POWER OF THE HUMAN RACE ALLOWS US TO
RESIST NUCLEAR AGGRESSION

Lifton and Markusen, Prof of International Relations @ Princeton U and Assist Researcher @ U of New
York, 90 (Robert Jay and Eric, The Genocidal Mentality, P. 278-279)

Species awareness means awareness of human choice: "This is not the End of Timeunless we choose to
make it so. We need not accept the death sentence . . . .We are not powerless." By choosing instead a human future,
we arein the words of the Polish Solidarity leader Adam Michnik"defending hope." And "hope is important. Perhaps more important than anything else."
Hope is greatly enhancedas is the acceptance of individual mortalityby the sense of reasserting the immortality of the species. The task is intensified by the
psychological upheavals we can expect in connection with the millennial transition of the year 2000. Whatever the millennial imagery, we
must
recognize that the hopeful future is not an apocalyptic heavenly peace but rather expanded awareness on
behalf of human continuity. This adaptation will not eliminate peoples need to define themselves in
relation to otherness, but it can begin to subsume that otherness to larger human commonality. It must
include struggles against widespread oppression and drastic human inequities by invoking the kind of
originality in political action that has taken place in the Solidarity movement in Polandand in related movements in Hungary, East Germany,
Czechoslovakia, and Bulgariaand was so cruelly frustrated in the student movement in China: Political action that enlarges, rather than blights or destroys,
This species-oriented approach would defy the given models of defiance. No one can
human possibilities.
claim knowledge of a single, correct path. Rather, there must be endless combinations of reflection and
action and, above all, the kind of larger collective adaptation we have been discussing. At the same time, we
must remain aware of persisting genocidal arrangements and expressions of genocidal mentality. We
cannot afford to stop thinking. Nor can we wait for a new Gandhi or Saint Joan to deliver us. Rather, each of us must join in a
vast projectpolitical, ethical, psychologicalon behalf of perpetuating and nurturing our humanity. We
are then people getting up from their knees to resist nuclear oppression. We clear away the thick
glass that has blurred our moral and political vision. We become healers, not killers, of our species.

Page 1270 of 1481


Page 1271 of 1481
#4 Nuclear Weapons Key to
Peace: 1AR
NUCLEAR WEAPONS ARE NECESSARY FOR WORLD PEACE, ELIMINATING EVILS,
AND ENSURING PROTECTIONS OF FREEDOMS THEIR ARGUMENT IGNORES THE
RATIONAL BASIS FOR THE CREATION OF NUCLEAR WEAPONS

J. A. H. Futterman, Ph.D. from UT-Austin and Physicist at the University of California's Lawrence Livermore
National Laboratory, Obscenity and Peace: Meditations on the Bomb, 1990-94,
http://www.dogchurch.com/scriptorium/nuke.html, UK: Fisher

With the above statement as background I observe that many peace activists confront the evil impulse in
the powers of war with the evil impulse in themselves. They rightly see nuclear war as a threat to the
planet, and therefore a threat to themselves and humanity, and so confront the threat of violence with
anger. Such an attitude is self-defeating, because acting from it creates more conflict, rather than less.
Rather than making peace, such action merely makes war on war.

Now the peace activists didn't invent this type of response. In the same spirit, nuclear weapons were first
invented by good people who were confronting the evil of the Nazis (who were trying to develop their
own atomic bomb) with the evil impulse in themselves. And by continuing to develop and/or maintain a
stockpile of them we give our assent to this evil impulse. I give my assent.

I give it because in response to the Nazis, I would have done the same thing. In response to Stalinism, I
would also have done as my predecessors did. I believe that Nazism had to be defeated at all costs, and
Stalinism had to be contained, in order to preserve and enlarge the freedoms that I hold dear for myself
and for all people. Such a response satisfies the criterion of Utilitarianism -- the greatest good for the
greatest number -- at least in its outcome so far. Even the atomic bombing of Hiroshima and Nagasaki,
which hastened the defeat of the evil of Japanese Imperialism, satisfies the criterion of Utilitarianism in
that it spared the loss of American lives and the even worse devastation of Japan and loss of Japanese
lives that would have resulted from a conventional invasion. And I suppose I would have supported it for
that reason. (And if you think there we could have demonstrated the bomb over an unpopulated area,
remember that we used our entire stockpile of two bombs, and that it took two cities, to bring about the
surrender.) [29] But the image of an orphaned baby, burned and screaming, annihilates forever the
argument that it was good. [30] It was an evil response of good people to evil, and it was the best that we
humans could do at the time.

[30] Ironically, what I had remembered as an image of Hiroshima turns out to be H. S. Wong's photo
taken after the Japanese conventional bombing of Nanking on August 29, 1937.

And so the question of whether I am good or evil in my participation in the nuclear weapons business is
already contained in the discussion of yezer tov and yezer ra, above. Or in the Christian idea that we are
simultaneously sinners and saints. I am neither one nor the other -- like you, I am both. In associating with
a nuclear weapons program, I confront the evil of potential aggressors against America with my own evil
impulse. On the other hand, it is necessary (but not sufficient) for us to defend our turf, even in this
outrageous manner, if we are to defend our freedom. (Otherwise we risk being attacked just for being
vulnerable. And if the old enemy is no longer visible on our horizon, all we need do is to become
complacent for a new one to appear.) Just as an individual needs his evil impulse to live, so does a nation.
The question is not how to eliminate the evil impulse -- the question is how to harness it. How can we use
it for good?[31]

Page 1272 of 1481


Page 1273 of 1481
#5 Fear of Nuc Weapons Solves
Usage: 1AR
FEAR OF NUCLEAR WEAPONS HAS PREVENTED THEIR USE DETERRENCE HAS
CHECKED CONFLICT

Rajaraman, Professor of Theoretical Physics at JNU, 2K2 (R., Ban battlefield nuclear weapons, The Hindu,
April 22, http://www.hinduonnet.com/thehindu/2002/04/22/stories/2002042200431000.htm[

There were a variety of different reasons behind each of these examples of abstinence from using nuclear weapons. But one major common factor
contributing to all of them has been an ingrained terror of nuclear devastation. The well documented images of Hiroshima and Nagasaki, the awesome
Armageddon
photographs of giant mushroom clouds emerging from nuclear tests in the Pacific and the numerous movies based on nuclear
scenarios have all contributed to building up a deep rooted fear of nuclear weapons. This is not limited just to the
abhorrence felt by anti-nuclear activists. It permeates to one extent or another the psyche of all but the most
pathological of fanatics. It colours the calculations, even if not decisively, of the most hardened of military
strategists. The unacceptability of nuclear devastation is the backbone of all deterrence strategies. There is
not just a fear of being attacked oneself, but also a strong mental barrier against actually initiating nuclear attacks on
enemy populations, no matter how much they may be contemplated in war games and strategies. As a
result a taboo has tacitly evolved over the decades preventing nations, at least so far, from actually
pressing the nuclear button even in the face of serious military crises.

Page 1274 of 1481


Page 1275 of 1481
#5 Fear of Nuc Weapons Solves
Usage: Ext
FEAR AND HORROR FORCE PEOPLE TO TAKE THE PATH TOWARDS PEACE

J. A. H. Futterman, Ph.D. from UT-Austin and Physicist at the University of California's Lawrence Livermore
National Laboratory, Obscenity and Peace: Meditations on the Bomb, 1990-94,
http://www.dogchurch.com/scriptorium/nuke.html, UK: Fisher

But the inhibitory effect of reliable nuclear weapons goes deeper than Shirer's deterrence of adventurer-
conquerors. It changes the way we think individually and culturally, preparing us for a future we cannot
now imagine. Jungian psychiatrist Anthony J. Stevens states, [15]

"History would indicate that people cannot rise above their narrow sectarian concerns without some
overwhelming paroxysm. It took the War of Independence and the Civil War to forge the United States,
World War I to create the League of Nations, World War II to create the United Nations Organization and
the European Economic Community. Only catastrophe, it seems, forces people to take the wider view.

Or what about fear? Can the horror which we all experience when we contemplate the possibility of
nuclear extinction mobilize in us sufficient libidinal energy to resist the archetypes of war? Certainly, the
moment we become blas about the possibility of holocaust we are lost. As long as horror of nuclear
exchange remains uppermost we can recognize that nothing is worth it. War becomes the impossible
option. Perhaps horror, the experience of horror, the consciousness of horror, is our only hope. Perhaps
horror alone will enable us to overcome the otherwise invincible attraction of war."

Thus I also continue engaging in nuclear weapons work to help fire that world-historical warning shot I
mentioned above, namely, that as our beneficial technologies become more powerful, so will our
weapons technologies, unless genuine peace precludes it. We must build a future more peaceful than our
past, if we are to have a future at all, with or without nuclear weapons a fact we had better learn
before worse things than nuclear weapons are invented. If you're a philosopher, this means that I regard
the nature of humankind as mutable rather than fixed, but that I think most people welcome change in
their personalities and cultures with all the enthusiasm that they welcome death thus, the fear of
nuclear annihilation of ourselves and all our values may be what we require in order to become peaceful
enough to survive our future technological breakthroughs.[16]

Of course, we could just try for a world-wide halt to scientific research and technological change. This is
obviously not desirable because technological change serves humanity like biological diversity serves life
in general -- it gives us ways to cope with new challenges to our existence. For example, medical scientists
deliberately forced the smallpox virus into virtual extinction. Nor is halting technological change possible,
because the demand for such change is so great people want the new stuff so much that they actually
buy it.

The fear of nuclear annihilation may be what we require in order to become peaceful enough to survive
our future technological breakthroughs.

In other words, when the peace movement tells the world that we need to treat each other more kindly, I
and my colleagues stand behind it (like Malcolm X stood behind Martin Luther King, Jr.) saying, "Or else."
We provide the peace movement with a needed sense of urgency that it might otherwise lack.

Page 1276 of 1481


Page 1277 of 1481
Fear of Nuclear Weapons Good:
Ext (1/4)
FEAR IS OKAY IN THE CONTEXT OF A DEBATE ROUND DISCUSSION HELPS
ALLEVIATE THE NUMBING CAUSED BY FEAR

Dr. Peter M. Sandman is a preeminent risk communication speaker and consultant in the United States and has
also worked extensively abroad, Ph.D. in Communication from Stanford University in 1971, and Dr. JoAnn M.
Valenti, a founding member of SEJ and elected Fellow of the American Association for the Advancement of
Science, Scared stiff or scared into action, Bulletin of the Atomic Scientists, January 1986, pp. 1216, Winner of
the 1986/1987 Olive Branch Award for Outstanding Coverage of the Nuclear Arms Issue, given by New York Universitys
Center for War, Peace, and the News Media, http://www.psandman.com/articles/scarstif.htm, UK: Fisher

Numerous testimonials indicate that the shock therapy of a fear appeal may sometimes cut through
paralysis. But such testimonials are usually from activists who were neither paralyzed nor numb in the
first place, whose fear was maintained at reasonable levels by their own activism, and who derived new
energy and reinforcement from what people in the adjacent seats may well have found intolerable. Our
wager is that the fear speeches revitalize the committed into renewed action, startle the apathetic into
fresh attention, and torment the terrorized and the numb into starker terror and deeper numbness.

In a set of guidelines for Helping People Deal With Terrifying Films, Frances Peavey advised readers in
1981: Do not stand up after the film is over and try to scare people with further horrifying facts. This is a
violent act and does not encourage peace. When people are subjected to too much fear-provoking
material, they tend toward numbing, forgetting or feeling so violated that they are hostile to the overall
message.(12) At that time Peavey still saw value in terrifying films, so long as the discussion afterward
helped people deal with the feelings they aroused. In 1985, when few are apathetic but many are numbed
by terror, the value of the films themselves is much reduced.

FEAR MOTIVATES PEOPLE TO PURSUE CONSTRUCTIVE MEANS TO SUSTAIN PEACE


AND PREVENT LARGE-SCALE CATASTROPHE

Lifton, Distinguished Prof of Psychiatry and Psychology @ John Jay College, 2K1 (Robert Jay,
Illusions of the second nuclear age, World Policy Journal, Spring, Vol. 18, Iss. 1, P. 25)

The trouble is that in other ways the dangers associated with nuclear weapons are greater than ever: the continuing weapons--
centered policies in the United States and elsewhere; the difficulties in controlling nuclear weapons that exist under unstable conditions (especially in Russia
and other areas of the former Soviet Union);2 and the eagerness and potential capacity of certain nations and "private"
groups to acquire and possibly use the weapons. In that sense, the nuclear quietism is perilous. Or, to put the matter
another way, we no longer manifest an appropriate degree of fear in relation to actual nuclear danger. While fear
in itself is hardly to be recommended as a guiding human emotion, its absence in the face of danger can lead to catastrophe. We
human animals have built-in fear reactions in response to threat. These reactions help us to protect
ourselves-to step back from the path of a speeding automobile, or in the case of our ancestors, from the path of a wild animal.

Page 1278 of 1481


Fear can be transmuted into constructive planning and policies: whether for minimizing vulnerability to attacks by wild animals,
or for more complex contemporary threats. Through fear, ordinary people can be motivated to pursue constructive means
for sustaining peace, or at least for limiting the scope of violence. Similarly, in exchanges between world leaders
on behalf of preventing large-scale conflict, a tinge of fear-sometimes more than a tinge- can enable each to feel the
potential bloodshed and suffering that would result from failure. But with nuclear weapons, our psychological circuits are
impaired. We know that the weapons are around-and we hear talk about nuclear dangers somewhere "out there" -but our minds no longer connect with the
dangers or with the weapons themselves. That blunting of feeling extends into other areas. One of the many sins for which advocates of large nuclear
In the
stockpiles must answer is the prevalence of psychic numbing to enormous potential suffering, the blunting of our ethical standards as human beings.
absence of the sort of threatening nuclear rhetoric the United States and Russia indulged in during the
1980s, we can all too readily numb ourselves to everything nuclear, and thereby live as though the
weapons pose no danger, or as though they don't exist. To be sure, we have never quite been able to muster an appropriate level of
fear with respect to these weapons-one that would spur us to take constructive steps to remove the threat. We have always been able to numb ourselves in
this regard, which must be seen as a basic human response to a threat that is apocalyptic in scope and so technologically distanced as to be unreal. But there
were at least brief moments when we would awaken from our nuclear torpor.

Page 1279 of 1481


Page 1280 of 1481
Fear of Nuclear Weapons Good:
Ext (2/4)
THE AFFIRMATIVES ACTIVISM IS CRITICAL TO EMPOWERING INDIVIDUALS
ALLOWING THEM TO BREAK ANY FEAR CAUSED BY NUCLEAR WEAPONS

Dr. Peter M. Sandman is a preeminent risk communication speaker and consultant in the United States and has
also worked extensively abroad, Ph.D. in Communication from Stanford University in 1971, and Dr. JoAnn M.
Valenti, a founding member of SEJ and elected Fellow of the American Association for the Advancement of
Science, Scared stiff or scared into action, Bulletin of the Atomic Scientists, January 1986, pp. 1216, Winner of
the 1986/1987 Olive Branch Award for Outstanding Coverage of the Nuclear Arms Issue, given by New York Universitys
Center for War, Peace, and the News Media, http://www.psandman.com/articles/scarstif.htm, UK: Fisher

The main obstacle to action, writes Frank, is neither apathy nor terror but simply a feeling of
helplessness. To combat it, I have perhaps overemphasized the small signs that antinuclear activities are
at last beginning to influence the political process.(19) Helplessness, hopelessness, futility, and despair
are words one hears even more often than fear from the barely active and the formerly active. And like
fear, these emotions can easily lead to psychic numbing. Those who feel powerless to prevent nuclear war
try not to think about it; and it serves the needs of those who do not wish to think about nuclear war to
feel powerless to prevent it. Messages of hope and empowerment, however, break this vicious circle.

The label hope, as we use it, subsumes a wide range of overlapping concepts: for example, optimism, a
sense of personal control and efficacy, confidence in methods and solutions, a sense of moral
responsibility, and a vision of the world one is aiming for.

It is well established (and hardly surprising) that hope is closely associated with willingness to act.
Activism appeals most to people who feel positive about both the proposed solution and their personal
contribution to its achievement. Over the long term, this means that antinuclear organizers must
communicate a credible vision of a nuclear-free world. Meanwhile, they must offer people things to do
that seem achievable and worthwhile. The nuclear-weapons-freeze campaign attracted millions of new
activists in 1982 because it offered credible hope. By 1985 many of those millions could no longer ground
their hope in the freeze; some found other approaches and some returned to inactivity.

Most social psychologists today see the relationship between hope and action as independent of fear or
other feelings. For example, Kenneth H. Beck and Arthur Frankel conclude that three cognitions (not
emotions) determine whether people will do something about a health risk: recognizing the danger as
real, believing the recommended plan of action will reduce the danger, and having confidence in their
ability to carry out the plan.(20) Similarly, Suttons review of the fear-appeal literature finds inconsistent
support for the notion that people can accept higher levels of fear if they feel the proposed solution will
remedy the problem, but strong evidence that, regardless of fear, people are more inclined to act on
solutions they see as more effective.(21)

Page 1281 of 1481


Page 1282 of 1481
Fear of Nuclear Weapons Good:
Ext (3/4)
NUCLEAR WEAPONRY CHECKS DICTATORIAL CONQUEST AND GLOBAL WAR

J. A. H. Futterman, Ph.D. from UT-Austin and Physicist at the University of California's Lawrence Livermore
National Laboratory, Obscenity and Peace: Meditations on the Bomb, 1990-94,
http://www.dogchurch.com/scriptorium/nuke.html, UK: Fisher

I could say that if I didn't do it, someone else would, but that answer was rejected at Nuremberg. (It's also
a better reason to leave the weapons program than to stay.) I continue to support the nuclear weapons
business with my effort for many reasons, which I discuss throughout this piece. But mostly, I do it
because the fear of nuclear holocaust is the only authority my own country or any other has respected so
far when it comes to nationalistic urges to make unlimited war. As William L. Shirer states in his preface
to The Rise and Fall of the Third Reich (Touchstone Books, New York, 1990),

"Adolf Hitler is probably the last of the great adventurer-conquerors in the tradition of Alexander, Caesar,
and Napoleon, and the Third Reich the last of the empires which set out on the path taken earlier by
France, Rome and Macedonia. The curtain was rung down on that phase of history, at least, by the
sudden invention of the hydrogen bomb, of the ballistic missile, and of rockets which can be aimed to hit
the moon."

Now this contrasts with the argument of those who would "reinvent government" by putting up
bureaucratic roadblocks to maintaining the reliability of the US nuclear arsenal through research and
testing. They reason that if the reliability of everyone's nuclear arsenals declines, everyone will be less
likely to try using them. The problem is that some "adventurer-conqueror" may arise and use everyone's
doubt about their arsenals to risk massive conventional war instead. An expansionist dictatorship might
even risk nuclear war with weapons that are simpler, cruder, less powerful, much riskier (in terms of the
possibility of accidental detonation) but much more reliable than our own may eventually become
without adequate "stockpile stewardship."[14]

Page 1283 of 1481


Page 1284 of 1481
Fear of Nuclear Weapons Good:
Ext (4/4)
NATIONS WILL INEVITABLY SEEK THE DEADLIEST WEAPONS INGRAINED IN
HUMAN NATURE EXISTENCE OF NUCLEAR WEAPONS CHECKS DEVELOPMENT OF
WORSE WEAPONS IN THE FUTURE

J. A. H. Futterman, Ph.D. from UT-Austin and Physicist at the University of California's Lawrence Livermore
National Laboratory, Obscenity and Peace: Meditations on the Bomb, 1990-94,
http://www.dogchurch.com/scriptorium/nuke.html, UK: Fisher

Some people argue that the goal of civilization is to raise our children so that wars don't happen.
Unfortunately, we've had civilization for six thousand years, and our history has been as dysfunctional as
our families. The only thing that's ever made us pause in our societal "addiction" to war is nuclear
weaponry, and the realization that the next big war may kill us all.

But if war is humanity's heroin, nuclear weaponry is its methadone. That is, the treatment has
potentially dangerous side effects. I am partly referring to the doctrine of deterrence by Mutual Assured
Destruction, MAD. It is MAD, because it is intrinsically unstable, as those who lived through the Cuban
Missile Crisis may recall. The Strategic Defense Initiative, (or Star Wars) was an attempt to move toward
something more stable, and its successor, the Ballistic Missile Defense Organization (BMDO), may in time
succeed, provided it is managed as a research program rather than as a political football. But even a
successful BMD will not make the world stable against massively destructive war -- it will merely make it
more stable than it is now. BMD is a technical fix that does not address the real cause of the instability.

As long as war is the ultimate arbiter of international disputes, nations will arm themselves with ultimate
weapons. And that means, that if something worse than nuclear weapons can be discovered and
developed, it will be. And then we will find something worse than that, and so on perhaps until we,
ourselves, prematurely punctuate the end of our universe with as big a bang as the one which began it.
Nuclear weapons may actually be giving us a chance to learn to get along with each other before we get
something really dangerous, a kind of world-historical warning shot.[8] The problem is not nuclear
weapons, the problem is war.

Page 1285 of 1481


Page 1286 of 1481
#5 Nuclear Imagery Good: 1AR
DISCUSSING NUCLEAR WAR IS KEY TO PREVENTIN GIT DENYING THE POSSIBILITY
OF NUCELAR WAR INCREASES POWERLESSNESS

Goodman & Hoff 90


[Lill & Lee, authors, Omnicide]

Why then do we feel so powerless? One of our problems is a partly voluntary lack of imagination. It
protects us from fully visualizing the unthinkable horrors of such a war. It also prevents us from insisting
that government serve our mandates (which, after all, defines democracy) rather than the other way
around. Instead of searching for ways to prevent those who consider nuclear war a viable course of action
from every carrying it out, we engage in self-deceptive maneuvers as a protection against feeling
inadequate, helpless, and anxious. Unfortunately, distorting or denying the gravity of our present
condition does not change its reality. On the contrary, as already noted, it may even be instrumental in
bringing about the disaster.

Page 1287 of 1481


Page 1288 of 1481
A2 Nuclear Numbing: 2AC
NUMBING CAN ONLY BE SOLVED BY EXPOSING THE TRUTH OF NUCLEARISM
PROTECTING PEOPLE FROM EXPOSURE ONLY DOOMS THEM

Gallagher 93
[Carole, photographer, American Ground Zero: the Secret Nuclear War]

The phrase nuclear numbing was coined by Dr Robert jay Lifton, a psychiatrist whose definitive studies
of the emotional health effects of nuclearism (beginning most notably with the interviews of the
hibakusha of Hiroshima and Nagasaki for his book Death in Life) have been a major part of his lifes work.
His writings suggest that the passivity, the numbness that I witnessed may have been a natural byproduct
of both the trauma of the atomic bomb tests conducted so close by (near enough for the shock waves to
throw people from their beds) and the cover-up, the web of lies in which the downwinders were tangled.
To cope with the psychic damage from their betrayal, perhaps it was necessary to avoid acquiring further
information that would make their vague fears specific enough to require decisive action. In this warping
of awareness and crippling of grass roots political potency, the downwinders were the cold War
bellwether of an increasingly unquestioning nation. They were the unwitting casualties of an American
nuclear jihad the Cold War against Communism. Each society produces its own slaughter of innocents, of
those who are most expendable in dangerous times, whether the danger is falsely manufactured to
achieve a political end or truly exists. My country right or wrong: the issue of blind obedience to
authority is germane to all societies that value abstract ideals above life itself. The active or passive role of
absolute repression of individual choice, by either a government or a religion, seems to be a repetition of
the human holocaust syndrome, genocide. And at the heart of this planetary misery is a manipulative net
of what Erasmus called deceitful fictions for the rabble reaching back into recorded history, with which
the powerful few of both the church and the state control the lives of the many, always for their own
good.

Page 1289 of 1481


Page 1290 of 1481
A2 Nuclear Deterrence
Immoral: 2AC (1/2)
NUCLEAR WEAPONS ARE DESIGNED TO DETER WARS THEY ARE MORALLY
ACCEPTABLE BECAUSE THEY SECURE THE WORLD

Gusterson, Assoc Prof of Anthropology and Science Studies @ MIT and Adjunct Fellow @ Harvard Us Center for
Psychology and Social Change, 93 (Hugh, Ethnographic Writing on Militarism, Journal of Contemporary Ethnology,
April, Vol. 22, No.1, P.72)

How can the anthropologist and the political citizen learn to live together in the same person in such a situation? How, for ex- ample, should one write about
Lester,3 who told me that, although his university colleagues tried to talk him out of working at a nuclear weapons laboratory,
an interview subject like
their objec- tions did not trouble him? Hebelieves that it is more ethical to work on nuclear weapons than on less
destructive conventional weapons because nuclear weapons are designed to deter wars rather than to
fight them. He says that he could never work as a lawyer defending murderers or other criminals but feels mor- ally comfortable with his work as a
nuclear warhead designer, and even wonders if it might be morally reprehensible not to work on nuclear weapons
because, as he sees it, they make the world more stable. Lester is puzzled by those who cannot see that nuclear weapons make us safer
by making war unthink- able. Like most of his colleagues, he is confident that nuclear weapons can be controlled by humans, that
technological progress is unavoidable and beneficial, and that nuclear weapons are the embodiment of a
transcendent rationality, which alone can discipline the dark impulses leading humans to make war.
Everything in his life, where he sees the atom bent to the experimental will of human rationality on a daily basis, confirms those beliefs. Lester does not worry
that the United States will misuse the hydrogen bombs he designs, bombs he describes as "no more strange than a vacuum cleaner. You don't feel a fear for
them at all." In fact, he sees weapons technology as "beautiful." "How do I explain that?" he asked me. "To me, a spectrometer is a very pretty thing ... and
you feel badly that it's going to be destroyed [in a nuclear test]."

Page 1291 of 1481


Page 1292 of 1481
A2 Nuclear Deterrence
Immoral: 2AC (2/2)
DETERRENCE IS MORAL
A) NECESSARY TO THWART IMMORAL REGIMES
B) ALL OTHER SECURITY REGIMES RELY ON DETERRENCE, MAKING THE MORAL
DILEMMA INEVITABLE

Joseph, Under Secretary Arms Control & International Security and Former Professor of National
Security Studies & Director of the Center for Counterproliferation Research at the National Defense
University, 98 (Robert, THE CASE FOR NUCLEAR DETERRENCE TODAY, Orbis, Winter, Volume 42, Issue
1)

Morality and ethics. In terms of morality, the


blanket charge that any use of nuclear weapons--and even reliance on the threat of
nuclear retaliation for deterrence--would be immoral goes beyond past proclamations, such as those contained in the 1983 Catholic
bishops' pastoral letter which, while calling for general disarmament and condemning the first use of nuclear weapons, left ambiguous the role of nuclear
weapons for deterrence. If allowed to stand unchallenged, such a charge could carry substantial weight in the policy debate, especially in a democracy (and
perhaps only in a democracy) built upon moral principles. But it does not take a trained ethicist to recognize that such blanket moral assertions are
at best
simplistic, and perhaps--in light of what we know about human nature and history--dangerous in
themselves. The use, or even threat of use, of any weapon may contain elements of moral ambiguity. And like other weapons--whether a club in Rwanda
or artillery surrounding Sarajevo--nuclear weapons could be used in ways that are clearly immoral. Moreover, the scale of destruction that could result from
the employment of even a few nuclear weapons makes imperative the need to consider carefully the full range of moral issues associated with the possession
of these weapons. Perhaps for this reason, well-intentioned people have for decades debated where ethical lines should be drawn regarding the possession
and use of nuclear weapons. Yet, within this realm of considerable ambiguity, policymakers during the Cold War were
forced to decide
where the greater risk lay and make decisions with real consequences. Given the awful consequences of failure, the choice
was not simple. On the one hand, nuclear deterrence could fail. In the aftermath of such failure, it was possible (but by no means certain, insofar as a conscious
choice for use would have to be made by political authorities) that nuclear weapons would be unleashed on civilian populations with truly catastrophic
consequences. On the other hand, in
the absence of a credible nuclear deterrent, conventional deterrence could fail,
as it had so often in the past, twice globally, resulting in another devastating war with casualties perhaps
even greater than those in World War II. Looking back, one might even argue that those who condemned
nuclear weapons as immoral were simply wrong. The Western alliance's nuclear weapons were in fact the moral
weapon of choice. They worked precisely as intended by deterring an immoral totalitarian state from
attacsking Western Europe and undermining the peace, values, and freedom which the democracies
cherished. Indeed, given the tens of millions of innocent noncombatants killed in two world wars, one can
argue that the possession of nuclear weapons to deter yet another outbreak of mass slaughter by
conventional weapons, either in Europe or Asia, was squarely in the just war tradition. The argument that the external
environment has changed so much with the end of the Cold War that no ethical or moral basis for nuclear arms remains is likewise unconvincing. American
lives and interests remain threatened. In fact, the proliferation of chemical and biological weapons have made the likelihood of conflict and the prospect of the
use of weapons of mass destruction even greater than in the past in several key regions. But just as before, sound public and defense policy will emerge only
from a prudent calculation of risks and benefits, not from sweeping generalizations about the morality or immorality of possession or use of nuclear weapons.
The "new eliminationists" who wrap themselves in the cloak of moral superiority and certainty should be
asked to address the consequences of disarming the great democracies in a world in which advanced
conventional, chemical, and biological weapons (and in some cases nuclear capabilities) continue to spread among
states explicitly hostile to democratic values.

Page 1293 of 1481


Page 1294 of 1481
A2 Proliferation K: 2AC
CRITICISM OF THIRD WORLD NUCLEARPOWERS NOT ETHNOCENTRIC WE THINK
ALL NUCLEAR POWERS ARE IRRESPONSIBLE

Rao & Vanaik 2002


[Parsa & Achin, All Nuclear Powers are Irresponsible, Gulf News, June 10,
http://archive.gulfnews.com/articles/02/06/10/53954.html, acc 9-38-06//uwyo-ajl]

Does the nuclear belligerence of India and Pakistan confirm Western criticism that Third World countries
possessing nuclear weapons cannot be expected to behave responsibly?

All nuclear powers, whether they belong to the West or to the Third World, are irresponsible. How else
can you explain the stockpiling of nuclear weapons by the US and the Soviet Union during the Cold War?
It was sheer madness because they did not make hundreds of nuclear warheads for deterrence. They had
the capacity to destroy not only each other but the whole world many times over. It was sheer
irresponsibility.

Page 1295 of 1481


Page 1296 of 1481
**Religion**

Page 1297 of 1481


Wrath of God Answers: 2AC
(1/6)
1. NO LINK THERES NO WARRANT FOR WHY PLAN MAKES ANY METAPHYSICAL
ENTITY ANGRY

2. SOULS DONT EXIST. HUMAN IDENTITY IS NOTHING MORE THAN AN


ARRANGEMENT OF FINITE QUANTUM STATES

Tipler 94
[Frank J., Professor of Mathematical Physics at Tulane University, The Physics of
Immortality: Modern Cosmology, God and the Resurrection of the Dead, New York:
Doubleday, 1994, 221-3//uwyo-ajl]

The Bekenstein Bound follows from the basic postulates of quantum theory combined with the
further assumptions that (1) the system is bounded in energy, and (2) the system is
bounded, or localized, in space. A rigorous proof of the Bekenstein Bound would require
quantum field theory, but it is easy to describe in outline why quantum mechanics leads to
such a bound on the information coded in a bounded region. In essence, the Bekenstein
Bound is a manifestation of the uncertainty principle. Recall that the uncertainty principle tells
us that there is a limit to the precision with which we can measure the momentum of a particle
and its position. More precisely, the uncertainty principle says that the location of a point in
phase space-a concept I defined in Chapter III-cannot be defined more closely thal1 Planck's
constant h. Since a system's state is defined by where it is located in phase space, this
means that the number of possible states is less than or equal to the size of the phase
space region the system could be in, divided by the size of the minimum phase space
size, Planck's constant. (I've given a mathematical expression of this argument in the Appendix
for Scientists.) This state counting procedure, based on there being an absolute minimum size h
to a phase space interval, is an absolutely essential method of quantum statistical mechanics.
We have already used it in Chapter III to prove the almost periodicity of a bounded quantum
system. It is confirmed by the thousands of experiments which have been based on this
counting method.9 In high energy particle physics, any calculation of the "cross section"
requires counting the possible number of particle initial and final states, and the above state
counting method is used.lO The cross section, which is the measure of how many particles
scatter in a particular direction when they collide in particle accelerators, is the basic quantity
tested in particle physics. The Bekenstein Bound on the number of possible states is thus
confirmed by the correctness of the calculated cross sections. In summary, the Bekenstein
Bound on the total information that can be coded in a region is an absolute solid
conclusion of modern physics, a result as solid as the Rock of Gibraltar.

One can also use the Bekenstein Bound to deduce an upper bound to the rate of
information processing. The time for light to cross a sphere of a given diameter is equal to the
diameter of the sphere divided by the speed of light. Since a state inside the sphere cannot
completely change until a signal has time to travel trom one side to the other, the rate of
information processing is bounded above by the above Bekenstein Bound divided by this
time interval. Putting in the numbers (details in the Appendix for Scientists), we calculate that

Page 1298 of 1481


the rate of state change is less than or equal to 4 X 1051 bits per second, multiplied by the mass
of the system in kilograms. That is, the rate of information processing possible for a system
depends only on the mass of the system, not on its spatial size or on any other variable. So a
human being of mass 100 kilograms cannot change state more rapidly than about 4 X 1053
times per second. This number is of course enormous-and in fact a human will probably change
state much, much more slowly than this-but it's finite.

Page 1299 of 1481


Page 1300 of 1481
Wrath of God Answers: 2AC
(2/6)
3. TURN FUNDAMENTALIST CHRISTIANITY REQUIRES STONING OF DIVORCEES
AND HELLFIRE FOR GLBT INDIVIDUALS, JUSTIFYING THE MURDER OF INNOCENT
INDIVIDUALS LIKE MATTHEW SHEPARD AND IS AN INDEPENDENT REASON TO
REJECT THE NEG

4. THERE ARE AN INFINITE NUMBER OF POSSIBLE DEITIES AND YOU CANT BE


CERTAIN OF WHICH ONE IS REAL. TAKING A POSITION ON ONE INCREASES THE
CHANCE OF PISSING OFF ONE OF THE OTHER ONES, MEANING THE SAFEST
POSITION IS ONE OF NEUTRALITY

5. IF GOD EXISTS, ITS PROVEN THAT IT WONT INTERACT WITH OUR REALITY,
MEANING THERES NO IMPACT

Infidels.org 2003
[An Introduction to Atheism, February 24,
www.infidels.org/news/atheism/intro.html, acc 1-20-05//uwyo-ajlo]

If God interacts with our universe in any way, the effects of his interaction must have some
physical manifestation. Hence his interaction with our universe must be in principle
detectable.

If God is essentially non-detectable, it must therefore be the case that he does not
interact with our universe in any way. Many atheists would argue that if God does not
interact with our universe at all, it is of no importance whether he exists or not. A thing
which cannot even be detected in principle does not logically exist.

Of course, it could be that God is detectable in principle, and that we merely cannot detect him
in practice. However, if the Bible is to be believed, God was easily detectable by the
Israelites. Surely he should still be detectable today? Why has the situation changed?

Note that I am not demanding that God interact in a scientifically verifiable, physical way. I
might potentially receive some revelation, some direct experience of God. An experience like
that would be incommunicable, and not subject to scientific verification -- but it would
nevertheless be as compelling as any evidence can be.

But whether by direct revelation or by observation, it must surely be possible to perceive some
effect caused by God's presence; otherwise, how can I distinguish him from all the other things
that don't exist?

Page 1301 of 1481


Page 1302 of 1481
Wrath of God Answers: 2AC
(3/6)
6. PERM DO BOTH. YOU CAN ACKNOWLEDGE GODS EXISTENCE AND STILL VOTE
AFF.

7. RELIGION HAS CAUSED MORE EARTHLY DESTRUCTION, CONFLICT, AND


SUFFERING THAN ANY OTHER FORCE

Infidels.org 2003
[An Introduction to Atheism, February 24,
www.infidels.org/news/atheism/intro.html, acc 1-20-05//uwyo-ajlo]

Religion represents a huge financial and work burden on mankind. It's not just a matter of
religious believers wasting their money on church buildings; think of all the time and effort
spent building churches, praying, and so on. Imagine how that effort could be better spent.

Many theists believe in miracle healing. There have been plenty of instances of ill people
being "healed" by a priest, ceasing to take the medicines prescribed to them by doctors,
and dying as a result. Some theists have died because they have refused blood transfusions
on religious grounds.

It is arguable that the Catholic Church's opposition to birth control -- and condoms in particular
-- is increasing the problem of overpopulation in many third-world countries and
contributing to the spread of AIDS world-wide.

Religious believers have been known to murder their children rather than allow their
children to become atheists or marry someone of a different religion. Religious leaders have
been known to justify murder on the grounds of blasphemy.

There have been many religious wars. Even if we accept the argument that religion was not
the true cause of those wars, it was still used as an effective justification for them.

8A. IF EVOLUTION IS TRUE, THEN EVERYTHING IN THE BIBLE IS A LIE

Gitt 95
[Werner, Creationist Information Scientist, 10 Dangers of theistic evolution,
Creation Ex Nihilo, vol 17 no 4, September-November,
www.answersingenesis.org/docs/1305.asp, acc 1-20-05//uwyo-ajl]

Page 1303 of 1481


The entire Bible bears witness that we are dealing with a source of truth authored by God
(2 Timothy 3:16), with the Old Testament as the indispensable 'ramp' leading to the New
Testament, like an access road leads to a motor free way (John 5:39). The biblical creation
account should not be regarded as a myth, a parable, or an allegory, but as a historical
report, because:

Biological, astronomical and anthropological facts are given in didactic [teaching] form.

In the Ten Commandments God bases the six working days and one day of rest on the
same time-span as that described in the creation account (Exodus 20:8-11).

In the New Testament Jesus referred to facts of the creation (e.g. Matthew 19:4-5).

Nowhere in the Bible are there any indications that the creation account should be understood
in any other way than as a factual report.

The doctrine of theistic evolution undermines this basic way of reading the Bible, as
vouched for by Jesus, the prophets and the Apostles. Events reported in the Bible are
reduced to mythical imagery, and an understanding of the message of the Bible as being
true in word and meaning is lost.

Page 1304 of 1481


Page 1305 of 1481
Wrath of God Answers: 2AC
(4/6)

B. THE OVERWHELMING WEIGHT OF EXPERIMENTAL AND EMPIRICAL EVIDENCE


VERIFIES EVOLUTION

Talkorigins.org 97
[The absolute best site on evolution on the internet, period, contributed to by
biological scientists, Five Major Misconceptions about Evolution, October 1,
talkorigins.org/faqs/faq-misconceptions.html, acc 1-20-05//uwyo-ajl]

Biologists define evolution as a change in the gene pool of a population over time. One
example is insects developing a resistance to pesticides over the period of a few years.
Even most Creationists recognize that evolution at this level is a fact. What they don't
appreciate is that this rate of evolution is all that is required to produce the diversity of all living
things from a common ancestor.

The origin of new species by evolution has also been observed, both in the laboratory
and in the wild. See, for example, (Weinberg, J.R., V.R. Starczak, and D. Jorg, 1992,
"Evidence for rapid speciation following a founder event in the laboratory." Evolution 46:
1214-1220). The "Observed Instances of Speciation" FAQ in the talk.origins archives gives
several additional examples.

Even without these direct observations, it would be wrong to say that evolution hasn't been
observed. Evidence isn't limited to seeing something happen before your eyes. Evolution
makes predictions about what we would expect to see in the fossil record, comparative
anatomy, genetic sequences, geographical distribution of species, etc., and these
predictions have been verified many times over. The number of observations supporting
evolution is overwhelming.

What hasn't been observed is one animal abruptly changing into a radically different one, such
as a frog changing into a cow. This is not a problem for evolution because evolution doesn't
propose occurrences even remotely like that. In fact, if we ever observed a frog turn into a cow,
it would be very strong evidence against evolution.

9. THERES NO EVIDENCE THAT GOD WILL TAKE CARE OF THE PROBLEMS OF CASE.
ITS EMPIRICALLY DENIED BY THE MANY ECO-CATASTROPHES LIKE THE TSUNAMI
THAT SHE OR HE MANAGED TO OVERLOOK

Page 1306 of 1481


Page 1307 of 1481
Wrath of God Answers: 2AC
(5/6)
10. GOD DOESNT EXISTPARADOX OF CAUSATION PROVES

Russell no date
Bertrand, philosopher, http://members.aol.com/JAlw/joseph_alward.html

"If everything must have a cause, then God must have a cause. If there can be anything without
a cause, it may just as well be the world as God, so that there cannot be any validity in that
argument. It is exactly of the same nature as the Hindu's view, that the world rested upon an
elephant and the elephant rested upon a tortoise; and when they said, "How about the
tortoise?" the Indian said, "Suppose we change the subject." The argument is really no better
than that. There is no reason why the world could not have come into being without a cause;
nor, on the other hand, is there any reason why it should not have always existed. There is no
reason to suppose that the world had a beginning at all. The idea that things must have a
beginning is really due to the poverty of our imagination. Therefore, perhaps, I need not waste
any more time upon the argument about the First Cause."

11. EMOTIONS ARGUMENT PROVES GOD DOESNT EXIST

EVILBIBLE.COM 03

A God who knows everything cannot have emotions. The Bible says that God experiences all of
the emotions of humans, including anger, sadness, and happiness. We humans experience
emotions as a result of new knowledge. A man who had formerly been ignorant of his wife's
infidelity will experience the emotions of anger and sadness only after he has learned what had
previously been hidden. In contrast, the omniscient God is ignorant of nothing. Nothing is
hidden from him, nothing new may be revealed to him, so there is no gained knowledge to
which he may emotively react.

We humans experience anger and frustration when something is wrong which we cannot fix.
The perfect, omnipotent God, however, can fix anything. Humans experience longing for things
we lack. The perfect God lacks nothing. An omniscient, omnipotent, and perfect God who
experiences emotion is impossible.

Page 1308 of 1481


Page 1309 of 1481
Wrath of God Answers: 2AC
(6/6)
12A. SEX ABUSE IS ENDEMIC TO CHRISTIANITY

Rice 01
(Fredric, The Skeptic Tank January 17, 2001

http://www.skeptictank.org/clrabuse.htm)

The problem of Christian clergy child sexual abuse is so epidemic in Westernized socities that the
"Sodomizing Priest" has become both stereotypical and cliche. It's not limited to one particular brand
name of Christianity; it's not an aspect of ideological differences; it's epidemic to all Christian brand
names.

B. SEX ABUSE DESTROYS SPIRITUALITY

Franz 02
(Thaeda, Liberty University, Power, Patriarchy and Sexual Abuse in the Christian
Church,http://www.fsu.edu/~trauma/v8/Church.pdf)

In the book, Sexual Abuse in Christian Homes and Churches (1993), Carol

Heggen describes the manner in which sexual abuse can leave victims feeling spiritually

bereft. Her discussion of the problems between victims of sexual abuse and the church is
threefold. First, according to Heggen, the church has ignored the problem of sexual

abuse. Second, the church has ignored victims of sexual abuse. Third, the church through
policies and subtle patriarchal language- has enveloped perpetrators in a web of

safety where their violations will be forgiven and forgotten under the guise of grace.

Heggen (1993) claims there can be profound spiritual damage in the instances of

sexual abuse where the abuser and the victim are both religious. If the victim prayed to

God for protection, and the abuse continued, the victim may see God as uncaring

(Heggen, 1993). If the perpetrator is a church leader, the violation is even worse. For

example, if a little girl is being molested by her father, who is a minister, both the

violation of the father/daughter trust, and the pastor/church member trust occurs. This

Page 1310 of 1481


can hinder the victims ability to develop a close relationship with God in the future. God

begins to be associated with the experience of molestation, and can keep the victim from
seeking out religious spiritual help later in life. The fact that the church at worst

unwilling, and at best unable to discuss sexual abuse has not made it any easier for these victims
to have their questions answered regarding God and His not having protected them.

Page 1311 of 1481


#1 Finite Quantum States: 1AR

EXTEND 2AC NUMBER 2 TIPLER 94 EVIDENCE. QUANTUM MECHANICS PROVES


THAT YOUR IDENTITY IS NOTHING MORE THAN A SERIES OF QUANTUM PARTICLE
STATES. YOU SHOULD PREFER OUR EVIDENCE ON THIS POINT BECAUSE ITS BASED
ON EMPIRICAL EVIDENCE IN THE LAB AND IS FALSIFIABLE, WHEREAS THEIR
EVIDENCE IS UNEVIDENCED CONJECTURE THAT YOU COULD NEVER DETERMINE
WHETHER IT WAS TRUE OR NOT. THIS HAS TWO IMPACTS:

A. IT TAKES OUT THE INTERNAL LINK TO ALL OF THEIR OFFENSE BECAUSE YOU
NO LONGER EXIST AFTER YOU DIE, MEANING NO RISK OF HELLFIRE
B. IT MEANS CASE OUTWEIGHS BECAUSE DEATH IS FINAL, ANNIHILATING YOUR
IDENTITY. PLAN IS THE ONLY WAY TO CONTINUE YOUR EXISTENCE

AND, COMPLEX INFORMATION PROCESSING IS THE BASIS OF LIFE, MEANING THAT


DEATH CAUSES ANNIHILATION OF CONSCIOUSNESS

Tipler 94
[Frank J., Professor of Mathematical Physics at Tulane University, The Physics of
Immortality: Modern Cosmology, God and the Resurrection of the Dead, New York:
Doubleday, 1994, 124-5//uwyo-ajl]

IN ORDER TO INVESTIGATE WHETHER LIFE can continue to exist forever, I shall need to define
"life" in physics language. I claim that a "living being" is any entity which codes information (in
the physics sense of this word) with the information coded being preserved by natural selection.
Thus "life" is a form of information processing, and the human mind-and the human soul-is
a very complex computer program. Specifically, a "person" is defined to be a computer
program which can pass the Turing test, which was discussed in Chapter II.

This definition of "life" is quite different from what the average person-and the average
biologist-would think of as "life." In the traditional definition, life is a complex process
based on the chemistry of the carbon atom. However, even supporters of the traditional
definition admit that the key words are "complex process" and not "carbon atom."
Although the entities everyone agrees are "alive" happen to be based on carbon chemistry,
there is no reason to believe that analogous processes cannot be based on other
systems. In fact, the British biochemist A. G. Cairns-Smith! has suggested that the first living
beings--':our ultim:ate ancestors-were based on metallic crystals, not carbon. If this is true,
then if we insist that living beings must be based on carbon chemistry, we would be forced to
conclude that our ultimate ancestors were not alive. In Cairns-Smith's theory, our ultimate
ancestors were self-replicating patterns of defects in the metallic crystals. Over time, the
pattern persisted, but was transferred to another substrate: carbon molecules. What is
important is not the substrate but the pattern, and the pattern is another name for
information.

Page 1312 of 1481


But life of course is not a static pattern. Rather, it is a dynamic pattern that persists overtime.
It is thus a process. But not all processes are alive. The key feature of the "living" patterns is that
their persistence is due to a feedback with their environment: the information coded in
the pattern continually varies, but the variation is constrained to a narrow range by this
feedback. Thus life is, as I stated, information preserved by natural selection.

Page 1313 of 1481


A2 Cant Disprove Gods Existence: 1AR

FIRST, THATS BECAUSE NO NEGATIVE IS PROVABLE. HOWEVER, IN LIGHT OF AN


ABSENCE OF ANY EMPIRICAL EVIDENCE, YOU MIGHT AS WELL ASSUME THAT IT
DOESNT EXIST. I CANT PROVE PINK UNICORNS DONT EXIST, BUT THERES STILL
NO RATIONAL BASIS TO ASSUME THAT THEY DO

SECOND, EVEN IF GOD DOES EXIST, THERES NO INDICATION THAT IT INTERACTS


WITH OUR WORLD, MAKING THIS IRRELEVENT

Page 1314 of 1481


#7 Religious Suffering: 1AR (1/3)

RELIGIOUS DOGMATISM BREEDS INTOLERANCE AND VIOLENCE

Nussbaum 2004
[Martha, Ernst Freund distinguished service prof of law and ethics at U of

chicago, Relgious Intolerance, Foreign Policy, Sept/Oct, 44//uwyo-ajl]

Sometimes old ideas are the most dangerous, and few ideas are older than those that undergird
religious intolerance. Lamentably, these ideas are acquiring new life. In 2002, Hindus in Gujarat,
India, killed several hundred Muslims, with the collaboration of public officials and the police.
Europe has recently seen a frightening rebirth of anti-Semitism, while the appeal of radical
forms of Islam appears to be increasing in the Muslim world. Prejudice against Muslims and a
tendency to equate Islam with terrorism are too prominent in the United States. On and on it
goes. Intolerance breeds intolerance, as expressions of hatred fuel existing insecurities and
permit people to see their own aggression as legitimate self-defense.

CHRISTIANITY CAUSES HATRED AND WAR

Dolgorukii 97
(Alexis, columnist, Associated press: 26 May 1997)

I would like to have "the undoubted blessings brought by Christianity" demonstrated to me. As for myself,
as an historian I can find nothing at all that Christianity has done to make life on this planet better for the
greatest mass of people. Christians have always babbled blithely about "love and peace" but I see
absolutely no historical indication that they have produced anything but hatred and war! Christians also
babble on about "Christian Charity", but that too is a lie. Charity existed long before Christianity and
Christian Charity (such as it is) has always come with chains of adamant fastened to it. "Christian Charity"
is best exemplified by "The Salvation Army", a militant Christian Organization that feeds the hungry but
insists that they be evangelized along with dinner!

RELIGIOUS INTOLERANCE GIVES WAY TO NIHILISM AND THE JUSTIFICATION OF


SUFFERING AND INEQUALITY

Nussbaum 2004
[Martha, Ernst Freund distinguished service prof of law and ethics at U of

chicago, Relgious Intolerance, Foreign Policy, Sept/Oct, 44//uwyo-ajl]

Page 1315 of 1481


The appeal of religious intolerance is easy to understand. From an early age, humans are aware
of helplessness toward things of the highest importance, such as food, love, and life itself.
Religion helps people cope with loss and the fear of death; it teaches moral principles and
motivates people to follow them. But precisely because religions are such powerful sources of
morality and community, they all too easily become vehicles for the flight from helplessness,
which so often manifests itself in oppression and the imposition of hierarchy. In todays
accelerating world, people confront ethnic and religious differences in new and frightening
ways. By clinging to a religion they believe to be the right one, surrounding themselves with
coreligionists, and then subordinating others who do not accept that religion, people can forget
for a time their weakness and mortality.

Page 1316 of 1481


Page 1317 of 1481
#7 Religious Suffering: 1AR (2/3)
CHRISTIANITY KILLED MORE PEOPLE THAN THE NAZIS

Dolgorukii 97 (Alexis, columnist, Associated press: 26 May 1997)

What has Christianity given humanity? Well, there's the Inquisition, which murdered more
people than the Nazi death camps. There's the various Crusades, which murdered millions in the
name of the "Prince of Peace". There's the religious Wars of the Reformation which killed untold
millions of people in Europe. And there's the Nazis themselves, who never would have been
able to preach their doctrine of racial hatred had that hatred not been fostered by a millennium
of anti-Jewish preachments from the pulpits of Christian Churches. And lastly, and far more
insidiously there's the unending grinding oppression administered by Christianity and its
minions on all the people over whom they held sway. What else has it given us? Well with its
totally insane views on sexuality and sin, Christianity has given the human race guilt and
psychosis to an absolutely astonishing degree.

ALL DEFENDERS OF CHRISTIANITY ARE BIASED AND LIE

Schnook 03 (Charlotte, EVILBIBLE.COM, http://www.evilbible.com/hitler_was_christian.htm)

Considering that Christianity has thus far been incapable of producing an unbiased, educated follower
which speaks the truth, (I havent encountered any), I have been forced to dispel the myth by writing this
essay.

CHRISTIANITY ENCOURAGES PARENTS TO STARVE THEIR CHILDREN TO DEATH

Rice 01 (Fredric, The Skeptic Tank January 17, 2001


http://www.skeptictank.org/clrabuse.htm)

Parents who murder their own children by starving them to death or by allowing them to die from easilly
treatable diseases and other medical problems are doing so because their religious masters tell them to.
As followers, the parents have no cognitive volition of their own when the health and safety of their
children come second to obeying the dictates of their religious masters. It is the priesthood which should
be held accountable for the murder of children first and foremost; then the parents of the murdered child
must be held accountable.

Page 1318 of 1481


Page 1319 of 1481
#7 Religious Suffering: 1AR (3/3)
CHRISTIANITY WAS RESPONSIBLE FOR THE HOLOCAUST

Schnook 03
(Charlotte, EVILBIBLE.COM, http://www.evilbible.com/hitler_was_christian.htm)

History is currently being distorted by the millions of Christians who lie to have us believe that the
Holocaust was not a Christian deed. Through subterfuge and concealment, many of todays Church
leaders and faithful Christians have camouflaged the Christianity of Adolf Hitler and have attempted to
mark him an atheist, a pagan cult worshipper, or a false Christian in order to place his misdeeds on those
with out Jesus. However, from the earliest formation of the Nazi party and throughout the period of
conquest and growth, Hitler expressed his Christian support to the German citizenry and soldiers. Those
who would make Hitler an atheist should turn their eyes to history books before they address their pews
and chat rooms.

CHRISTIANITY INSPIRED HITLERS ANTI-SEMITISM

Schnook 03
(Charlotte, EVILBIBLE.COM, http://www.evilbible.com/hitler_was_christian.htm)

Hitlers anti-Semitism grew out of his Christian education. Austria and Germany were majorly Christian
during his time and they held the belief that Jews were an inferior status to Aryan Christians. The
Christians blamed the Jews for the killing of Jesus. Jewish hatred did not actually spring from Hitler, it
came from the preaching of Catholic priests and Protestant ministers throughout Germany for hundreds
of years. The Protestant leader, Martin Luther, himself, held a livid hatred for Jews and their Jewish
religion. In his book, On the Jews and their Lies, Luther set the standard for Jewish hatred in Protestant
Germany up until World War 2. Hitler expressed a great admiration for Martin Luther constantly quoting
his works and beliefs.

Page 1320 of 1481


A2 Those Ppl Werent Real Christians: 1AR

FIRST, EVEN IF THE PEOPLE WHO CAUSE RELIGIOUS WARS ARENT REAL
CHRISTIANS, THE NEGS ARGUMENT ENGAGES IN THAT SAME FALSE CHRISTIANITY
BY CALLING FOR THE NIHILISTIC REJECTION OF EVERY PLAN THAT TRIES TO SAVE
LIVES

THAT ALSO MEANS THAT THEY DONT ACTUALLY DO THEIR ALTERNATIVE,


MEANING IT CANT SOLVE FOR THE CRITICISM AND THERES AN EQUAL RISK
WHETHER YOU VOTE AFF OR NEG

ALSO, THIS ARGUMENT IS FALLACIOUS THEYRE MANIFESTATIONS OF THE SAME


RELIGIOUS IDEOLOGY, PROVING HOW BANKRUPT IT IS

Infidels.org 98

This is rather like the No True Scotsman fallacy.

What makes a real believer? There are so many One True Religions it's hard to tell. Look at Christianity:
there are many competing groups, all convinced that they are the only true Christians. Sometimes
they even fight and kill each other. How is an atheist supposed to decide who's a real Christian and
who isn't, when even the major Christian churches like the Catholic Church and the Church of England
can't decide amongst themselves?

In the end, most atheists take a pragmatic view, and decide that anyone who calls himself a Christian,
and uses Christian belief or dogma to justify his actions, should be considered a Christian. Maybe
some of those Christians are just perverting Christian teaching for their own ends -- but surely if the Bible
can be so readily used to support un-Christian acts it can't be much of a moral code? If the Bible
is the word of God, why couldn't he have made it less easy to misinterpret? And how do you know
that your beliefs aren't a perversion of what your God intended?

If there is no single unambiguous interpretation of the Bible, then why should an atheist take one
interpretation over another just on your say-so? Sorry, but if someone claims that he believes in Jesus and
that he murdered others because Jesus and the Bible told him to do so, we must call him a Christian.

AND, FOISTING YOUR RELIGION UPON OTHERS THROUGH STATE ACTION IS


RELIGIOUS INTOLERANCE THAT JUSTIFIES VIOLENCE

Nussbaum 2004
[Martha, Ernst Freund distinguished service prof of law and ethics at U of

Page 1321 of 1481


chicago, Relgious Intolerance, Foreign Policy, Sept/Oct, 44//uwyo-ajl]

Two ideas typically foster religious intolerance and disrespect. The first is that ones own
religion is the only true religion and that other religions are false or morally incorrect. But
people possessed of this view can also believe that others deserve respect for their committed
beliefs, so long as they do no harm. Much more dangerous is the second idea, that the state and
private citizens should coerce people into adhering to the correct religious approach. Its an
idea that is catching on, even in many modern democracies. Frances reluctance to tolerate
religious symbols in schools and the Hindu right wings repeated claims that minorities in India
must become part of Hindu culture are disturbing recent examples. The resurgence of this kind
of thinking poses a profound threat to liberal societies, which are based on ideas of liberty and
equality.

Page 1322 of 1481


#8 Evilution Disproves Religion: 1AR

EXTEND 2AC NUMBER 8. IF THE EARTH IS REALLY MILLIONS OF YEARS OLD AND
LIFE GRADUALLY EVOLVED, THEN THAT DISPROVES THE CREATION ACCOUNT IN
THE BIBLE, UNDERMINING THE ENTIRETY OF ITS VALIDITY, AS PROVEN BY GITT 95,
AN EXPERT SOURCE FROM THE CHRISTIAN MOVEMENT.

ALSO, THEYLL NEVER WIN THAT EVOLUTION IS FALSE. ITS BEEN REPEATEDLY
PROVEN BY EXPERIMENTAL TESTS AND REAL WORLD DATA, AS PROVEN BY THE
TALK ORIGINS EVIDENCE.

ALSO, EVOLUTION UNDERMINES THE ENTIRE BELIEVABILITY OF THE CHRISTIAN


EDIFICE

Creation Magazine 89
[The Atheists KnowWhy Christianity has to Fight Evolution, Vol 11 Issue 4, September,
www.answersingenesis.org/creation/v11/i4/bozorath.asp, acc 1-20-2005//uwyo-ajl]

Christianity has fought, still fights, and will continue to fight science to the desperate end over
evolution, because evolution destroys utterly and finally the very reason Jesus earthly life
was supposedly made necessary. Destroy Adam and Eve and the original sin, and in the
rubble you will find the sorry remains of the Son of God. If Jesus was not the redeemer
who died for our sins, and this is what evolution means, then Christianity is nothing.

Page 1323 of 1481


Evolution Contradicts Christianity: Ext (1/2)

EVOLUTION MAKES THE CREATION STORY IMPOSSIBLE IN SO MANY WAYS THAT IT


HURTS

Creation Ex Nihilo 89
[Allan Rosser, March-May 1989, vol 11, no. 2,
www.answersingenesis.org/docs/3855.asp, acc 1-20-05//uwyo-ajl]

if the Bible is the God-breathed Word of God, authoritative and correct, then the theistic
evolutionist who 'accepts' the scriptural account of man's creation does have to stretch it a
little to say that man's creation from dust just took millions of years through transforming life-
forms. If it happened this way, God must have been deceiving us when He said He made man
from dust. What prevented Him from telling it like it was?

The fact that death came by one man. Adam (Romans 5:12) is a serious challenge to
theistic evolution, as many creatures already would have died in the evolutionary
process. The death that came through Adam was two-fold, even as Christ's death was twofold:

(i) physical death: and

(ii) spiritual death-separation from God.

It was from physical death that Jesus rose. Let us not think that this death that Adam brought in
was only spiritual. The result of his sin was that he was not allowed to eat of the tree of life. As a
result of this, he died physically many years later. Chapter 5 of Genesis tells us 'And he died . . .'
some eight times, no doubt to emphasize the consequences of Adam's sin.

As some degree of ape-man. Adam was going to die, so what was the use of God's warning to
Adam, 'In the day you eat of it, dying you will die'? (literal translation). Did God give Adam the
ability to live for ever and then after Adam's sin take it away?

In Scripture we read 'for since by man came death, by man came also the resurrection of the
dead, for as in Adam all die, even so in Christ shall all be made alive' (1 Corinthians 15:22,23). If
Adam was the end of the evolutionary line, then thousands of evolving men had already
died, and death did not come by Adam. Chapter 15 also tells of the second Adam, who was
Christ. If the first Adam " ex ape-man " was as real a person as the second Adam, then there
came a day when God must have said: 'You are of this moment man, Adam!'

APE-MAN'S MATES?

Suddenly, everything was different. Now he is sinless and can sin, but as an 'ape-man'or
part 'ape-like creature', be couldn't have sinned. Now he couldn't take the fruit of the tree
of the knowledge of good and evil, or he would be sinning, and would die. A moment ago
these were no restrictions; now there are. For years he had gone without clothes, and of course
he would not have been ashamed. But now he is a man. When did he lose his ape hair? A
moment ago be had mates, now he has none!

If we make some allowances and jump these hurdles, the women of that day would present a
problem. Let us set the stage again. If the theistic evolutionist believes Adam to be descended
from ape-like creatures, but a creation of God, then what about the woman Eve? If Adam was a
theistic evolution 'creation' " a literal, though stretched, interpretation of the Genesis account-
then what about Eve? God Himself said, 'It is not good for man to be alone'.

Page 1324 of 1481


What an incredible situation Adam's mother and father, sissy and brothers, aunts and uncles,
cousins, nephews and nieces, and his grandparents perhaps, were all around him, and be was
lonely! Maybe God left them out of Eden, or was this first man 'called out' even as Abraham was
in Genesis chapter 12?

ANIMAL RELATIVES?

God brought all the animals before Adam, and the Bible recounts that there was not found
among the animals a suitable mates or helper for Adam. Did all the animals not include his
mother and father, sisters and brothers, aunts and uncles, cousins, nephews and nieces, and his
grandparents? Did God only bring a couple of every kind of animal and did he leave Adam's
relatives out? Why couldn't he marry one of them? What was wrong with one of his distant
relatives, or the closer ones?

continued

Page 1325 of 1481


Page 1326 of 1481
Evolution Contradicts
Christianity: Ext (2/2)
continued

Even if, amazingly, only one family had become the proto-man type, surely there must have
been others near enough, well up in the evolutionary tree. Surely if the line-up of eligible
spinsters included his unmarried female relatives, Adam would have said, 'This one will
do!'. And God would have said, 'No Adam. you can't marry that sort, you are a new sort of
creature, you are a new creation. Or rather a new evolution.. She is not your sort!'

CHIMP OFF THE OLD BLOCK?

Adam would have said 'But she is just like my mother and my sister.' God would have replied
'They are no longer your kith and kin.'

Or did God erase from Adam's mind who he used to be? Did God also remove from his parents'
and relatives' memories all knowledge of Adam before he became Adam? Or did God suddenly
and completely so transform him that he realized that he was no longer a 'chip off the old
block', and was determined to start his own family tree?

If, though, it was because through a special creative act of God he was now different, then why
couldn't God have started from scratch-scratching dirt up to make the man, not just rehashing
an existing creature?

So, some allow that God evolved man, yet at a definite point declared: 'Ape, you are now man!
Adam is your name!' And at that point, God invested him with God-likeness and the opportunity
to live for ever as well. But, did he omit to evolve Eve? Is this why he had to create Eve? The
Bible is very explicit as to how God made Eve. She was made from Adam's side.

CAIN AND THE APE HYBRIDS?

Years later, their son Cain, having killed Abel, is banished to the land of Nod. And there he
marries one of the daughters of the land. Where did she come from? Was she one of the ape-
men family? One of his ancestors' group? If Cain could have married an ancestor type, then
surely Adam could have. Cain certainly wasn't in the Garden of Eden, but were none of his
relatives suitable, if he was only two generations away from them? If he couldn't have married
an ape-woman, had God made a hybrid variety, one that wasn't sterile?

If we accept the Bible account, then Adam and Eve were a special creation, made on
the sixth day of Creation weak. We find also that Adam lived to see Noah's father, and
Noah probably saw Abraham. In the days of Abraham there was writing. Was Adam's story not
written? Why has no trace of Adam's ancestry been revealed? Has God hidden it from us and
deceived us? Did Adam not tell his children even till the eighth generation, or did God take it
from his mind?

If there is anything miraculous about the creation of man, we must accept it by faith. If there is
nothing miraculous, who says these isn't? Will we believe man who doesn't know everything of
God? If God is God and His word is truth, then let us accept the plain sense of Scripture
by faith in God, the holy One who does not lie.

Page 1327 of 1481


A2 Evolution Is Only a Theory: 1AR

EVOLUTION IS A THEORY AND A FACT. YOUR AUTHORS HAVE NO IDEA HOW


SCIENCE OPERATES

Gould 93
[Steven J., scientific genius, Evolution is a fact and a theory, talkorigins, January 22,
talkorigins.org/faqs/evolution-fact.html, acc 1-20-05//uwyo-ajl]

In the American vernacular, "theory" often means "imperfect fact"--part of a hierarchy of


confidence running downhill from fact to theory to hypothesis to guess. Thus the power of the
creationist argument: evolution is "only" a theory and intense debate now rages about many
aspects of the theory. If evolution is worse than a fact, and scientists can't even make up their
minds about the theory, then what confidence can we have in it? Indeed, President Reagan
echoed this argument before an evangelical group in Dallas when he said (in what I devoutly
hope was campaign rhetoric): "Well, it is a theory. It is a scientific theory only, and it has in
recent years been challenged in the world of science--that is, not believed in the scientific
community to be as infallible as it once was."

Well evolution is a theory. It is also a fact. And facts and theories are different things, not rungs
in a hierarchy of increasing certainty. Facts are the world's data. Theories are structures of ideas
that explain and interpret facts. Facts don't go away when scientists debate rival theories to
explain them. Einstein's theory of gravitation replaced Newton's in this century, but apples
didn't suspend themselves in midair, pending the outcome. And humans evolved from ape-like
ancestors whether they did so by Darwin's proposed mechanism or by some other yet to be
discovered.

Page 1328 of 1481


A2 Evolution Contradicts Thermodynamics: 1AR

THAT LAW ASSUMES CLOSED SYSTEMS. THE EARTH ISNT BECAUSE OF SOMETHING
CALLED THE SUN

Talkorigins.org 97
[The absolute best site on evolution on the internet, period, contributed to by
biological scientists, Five Major Misconceptions about Evolution, October 1,
talkorigins.org/faqs/faq-misconceptions.html, acc 1-20-05//uwyo-ajl]

This shows more a misconception about thermodynamics than about evolution. The
second law of thermodynamics says, "No process is possible in which the sole result is the
transfer of energy from a cooler to a hotter body." [Atkins, 1984, The Second Law, pg. 25] Now
you may be scratching your head wondering what this has to do with evolution. The confusion
arises when the 2nd law is phrased in another equivalent way, "The entropy of a closed system
cannot decrease." Entropy is an indication of unusable energy and often (but not always!)
corresponds to intuitive notions of disorder or randomness. Creationists thus misinterpret the
2nd law to say that things invariably progress from order to disorder.

However, they neglect the fact that life is not a closed system. The sun provides more than
enough energy to drive things. If a mature tomato plant can have more usable energy
than the seed it grew from, why should anyone expect that the next generation of
tomatoes can't have more usable energy still? Creationists sometimes try to get around this
by claiming that the information carried by living things lets them create order. However, not
only is life irrelevant to the 2nd law, but order from disorder is common in nonliving
systems, too. Snowflakes, sand dunes, tornadoes, stalactites, graded river beds, and
lightning are just a few examples of order coming from disorder in nature; none require an
intelligent program to achieve that order. In any nontrivial system with lots of energy flowing
through it, you are almost certain to find order arising somewhere in the system. If order from
disorder is supposed to violate the 2nd law of thermodynamics, why is it ubiquitous in nature?

The thermodynamics argument against evolution displays a misconception about evolution as


well as about thermodynamics, since a clear understanding of how evolution works should
reveal major flaws in the argument. Evolution says that organisms reproduce with only
small changes between generations (after their own kind, so to speak). For example, animals
might have appendages which are longer or shorter, thicker or flatter, lighter or darker than
their parents. Occasionally, a change might be on the order of having four or six fingers instead
of five. Once the differences appear, the theory of evolution calls for differential reproductive
success. For example, maybe the animals with longer appendages survive to have more
offspring than short-appendaged ones. All of these processes can be observed today. They
obviously don't violate any physical laws.

Page 1329 of 1481


A2 No Transitional Fossils: 1AR

FIRST, EVERY SPECIES IS TRANSITIONAL. ITS NOT LIKE A COMPLETELY NEW


SPECIES IS CREATED OVERNIGHT BECAUSE ITS SO GRADUAL.

SECOND, THEYVE BEEN FOUND

Talkorigins.org 97
[The absolute best site on evolution on the internet, period, contributed to by
biological scientists, Five Major Misconceptions about Evolution, October 1,
talkorigins.org/faqs/faq-misconceptions.html, acc 1-20-05//uwyo-ajl]

To say there are no transitional fossils is simply false. Paleontology has progressed a bit
since Origin of Species was published, uncovering thousands of transitional fossils, by both
the temporally restrictive and the less restrictive definitions. The fossil record is still spotty
and always will be; erosion and the rarity of conditions favorable to fossilization make
that inevitable. Also, transitions may occur in a small population, in a small area, and/or in a
relatively short amount of time; when any of these conditions hold, the chances of finding the
transitional fossils goes down. Still, there are still many instances where excellent
sequences of transitional fossils exist. Some notable examples are the transitions from
reptile to mammal, from land animal to early whale, and from early ape to human. For many
more examples, see the transitional fossils FAQ in the talk.origins archive, and see
http://www.geo.ucalgary.ca/~macrae/talk_origins.html for sample images for some
invertebrate groups.

THIRD, THIS BETRAYS A MISCONCEPTION ABOUT CATEGORIES

Talkorigins.org 97
[The absolute best site on evolution on the internet, period, contributed to by
biological scientists, Five Major Misconceptions about Evolution, October 1,
talkorigins.org/faqs/faq-misconceptions.html, acc 1-20-05//uwyo-ajl]

The misconception about the lack of transitional fossils is perpetuated in part by a common way
of thinking about categories. When people think about a category like "dog" or "ant," they often
subconsciously believe that there is a well-defined boundary around the category, or that there
is some eternal ideal form (for philosophers, the Platonic idea) which defines the category. This
kind of thinking leads people to declare that Archaeopteryx is "100% bird," when it is clearly a
mix of bird and reptile features (with more reptile than bird features, in fact). In truth,
categories are man-made and artificial. Nature is not constrained to follow them, and it doesn't.

Page 1330 of 1481


Page 1331 of 1481
#12 Sexual Abuse: 1AR

SEX ABUSE=DEHUMANIZATION

Clark 03
(Peg, The Daily News September 14, 2003

http://www.bishop-accountability.org/news2003_07_12/2003_09_14_Clark_GuestCommentary.htm)

In sexual abuse a victim is denied his or her full humanity; this is dehumanization. In
dehumanization, paradoxically, the oppressor too becomes less human in the denial.

DEHUMYNIZATION IS WORSE THAN NUCLEAR WAR

Montagu and Matson 83


(Author and Professor of American at the University of Hawaii, Ashley and Floyd, THE
DEHUMANIZATION OF MAN, preface)

It neither kills outright nor inflicts apparent physical harm, yet the extent of its destructive toll is
already greater than that of any war, plague, famine, or natural calamity on record - and its
potential damage to the quality of human life and the fabric of civilized society is beyond
calculation. For that reason this sickness of the soul might well be called the 'Fifth Horseman of
the Apocalypse.' Its more conventional name, of course, is dehumanization.

Page 1332 of 1481


Page 1333 of 1481
Christianity = Sex Abuse: Ext
(1/3)
SEXUAL ABUSE INEVITABLE IN CHRISTIANITY, SIX REASONS:

Franz 02 (Thaeda, Liberty University, Power, Patriarchy and Sexual Abuse in the Christian
Church,http://www.fsu.edu/~trauma/v8/Church.pdf)

There are six beliefs present within the church that have aided perpetrators in rationalizing their
behavior when it comes to abuse (Heggen, 1993). Some of these beliefs are not obvious parts of
any Christian doctrine. They are found in the subtle subtext of sermons, Bible studies, and
Sunday school lectures throughout the church. Then they are taken a step further in the minds
of the perpetrator to justify his actions.

The first belief is that God intends for men to dominate and for women and children to submit
(Heggen, 1993; Kroeger & Beck, 1996). As head of the family, the authority of the
husband/father is not to be questioned, under any circumstances, regardless of how outrageous
his behavior may be (LaHaye, 1980). If he beats his wife, it is his business. If he molests his
children, it is his business. He is the unquestioned authority and lord over his domestic
domain. The second belief is that because of her role in the Fall, woman is morally inferior to
man (Heggen, 1993). She requires his guidance and is unable to stay on the straight

and narrow without it. After all, according to the Bible, it was Eve who sinned first and led her
husband, Adam, into sin with her. Some men in the church believe that this is proof that women
cannot be kept from sin without the control of their husbands.

Additionally, this belief encourages women to trust their husbands sense of right and wrong,
over their own internal set of values (Kreoger & Beck, 1996). In some cases, this leads a woman
to allow her husband to convince her that it is morally just for him to molest their daughter.
Even if the woman knows such a thing is wrong, she defers to her husbands moral compass,
because she has been told by the church that hers is defective (Kroeger & Beck, 1996). The third
belief is that children are inherently evil and must have their wills broken (Heggen, 1993). The
idea is that children must be forced to submit to their parents or they will never learn to submit
to God (Heggen, 1993). Children are seen as willful, and forcing them into submission is seen as
a parents duty--rather than an act of abuse. If a mother feels the actions of her husband are too
extreme, she may not say anything--not because she doesnt want to protect her children, but
because the church tells her that her opinion comes second to that of her husband. The fourth
belief is that marriage is to be preserved at all costs (Heggen 1993). If the husband/father is
abusive, it is the wife/mothers responsibility to find a way to help herself and her children
endure as an act commitment to the marital covenant.

The fifth belief is that suffering is a Christian virtue (Heggen, 1993). Traditionally, the role of the
ideal Christian woman is to be a suffering servant (Fortune, 1983; Kroeger & Beck, 1996). A
woman who decides to step out from under a yoke of suffering is oftentimes seen as weak and
lacking in faith (Fortune, 1983). Women who complain about their marital situation are
sometimes seen as lacking commitment to their family and to their faith. Fellow congregants
may suggest if she would only pray more, and complain less, then all would be well. The final
belief is that Christians must promptly forgive those who sin against them (Heggen, 1993).
Victims of sexual abuse have been told to forgive and forget-- and to give it to God, as if the

Page 1334 of 1481


responsibility for reconciling with the perpetrator lays squarely on the shoulders of the victim
(Heggen, 1993; Kreoger & Beck, 1996).

Page 1335 of 1481


Page 1336 of 1481
Christianity = Sex Abuse: Ext
(2/3)

CONSERVATIVE CHRISTIANITY IS BEST PREDICTOR OF SEXUAL ABUSE

Heggen 93 (Carolyn Holderread, author, "Sexual Abuse in Christian Homes and Churches", p. 73:
http://www.skeptictank.org/cabuse6.htm)

"A disturbing fact continues to surface in sex abuse research. The

first best predictor of abuse is alcohol or drug addiction in the

father. But the second best predictor is conservative religiosity,

accompanied by parental belief in traditional male-female roles.

This means that if you want to know which children are most likely

to be sexually abused by their father, the second most significant

clue is whether or not the parents belong to a conservative

religious group with traditional role beliefs and rigid sexual

attitudes.

ALL ATTEMPTS TO SOLVE ONLY MOVE THE ABUSE ELSEWHERE

Rice 01 (Fredric, The Skeptic Tank January 17, 2001


http://www.skeptictank.org/clrabuse.htm)

After they go through their period of "therapy," they often get shipped off to
yet another church where, since their new congregations are never informed
of their master's past, the cycle of abuse continues. (NOTE: "Megan's Law"
now makes the location of convicted sex offenders public knowledge. THIS IS A
WIN FOR THE GOOD SIDE! Everyone who has worked to get Megan's Law
passed has made it tougher for Christian clergy to hide their convicted child
moslesters within our communities.) Thus -- whether unintentionally or not --
the Christian clergy ends up being a safe dating service for pedophiles.
Pedophiles may safely gravitate toward the Christian clergy fairly confident in
the knowledge that even if they're ever reported or get caught, they'll simply
be moved to yet another location and be provided with new children to abuse.

Page 1337 of 1481


CHRISTIAN RHETORIC IS A COVER-UP FOR SEX ABUSE
Rice 01 (Fredric, The Skeptic Tank January 17, 2001
http://www.skeptictank.org/clrabuse.htm)

The inescapable conclusion is that the Christian clergy screams from the pulpit
in an attempt to draw attention away from their own horrid criminal activities
and desires. It's also an inescapable conclusion -- due to the epidemic problem
they create -- that they are successful.

Page 1338 of 1481


Page 1339 of 1481
Christianity = Sex Abuse: Ext
(3/3)
REALITY OF SEX ABUSE IS EXCLUDED FROM CHRISTIAN WORLDVIEW

Franz 02 (Thaeda, Liberty University, Power, Patriarchy and Sexual Abuse in the Christian
Church,http://www.fsu.edu/~trauma/v8/Church.pdf)

Considering the prevalence of sexual abuse, it is quite likely that there are at
least

a few survivors in any one church congregation. The church has a history of
speaking

only in generalities regarding sexual sin (Heggen, 1993). The discomfort of the
church in

dealing with sexual matters can make victims feel very isolated (Heggen,
1993). For

victims who have been sexually abused, it is doubly painful to hear that their
reality does

not have a place within the confines of their church . Many victims are told
that such

things just do not happen among good Christian people- particularly if they
identify

their perpetrator as someone who is a leader in the church (Heggen, 1993).

PATRIARCHAL ORDER OF CHRISTIANITY MAKES ABUSE ENDEMIC

Franz 02 (Thaeda, Liberty University, Power, Patriarchy and Sexual Abuse in the Christian
Church,http://www.fsu.edu/~trauma/v8/Church.pdf)

Additionally, the majority of pedophiles are men (E. Schrader LSW, personal
communication, January 2002), and the majority of people holding positions of
leadership in the church are men (Neuger, 1993). So, it would make sense that
since men are leading the church, and men are more often perpetrators rather

Page 1340 of 1481


than victims, that the topic of sexual abuse has thus far been ignored (Fortune,
1983). However, the gender of church leadership is only the tip of the iceberg
in determining why dealing with sexual abuse has taken such a low priority
within the church. The church has embraced the notion that women are
subject to the dominance of men (Neuger, 1993). There are few stories of
women in the Bible (Neuger, 1993). When women are portrayed in the Bible,
they are described as either evil and seductive, or as impossible ideals of self-
sacrifice and love (Neuger, 1993). It is possible that religious women may be
afraid to confront sexism in the church because they fear male protection and
approval will be withdrawn from them (Rayburn, 1982). This fear can also
carry over to God, and to the withholding of divine blessing and acceptance
(Rayburn, 1982).

Page 1341 of 1481


A2 Life Without God Pointless: 1AR

FIRST, THIS IS RIDICULOUS EACH INDIVIDUAL CREATES CONTINGENT MEANING


FOR THEIR OWN LIVES. ARBITRARILY CLAIMING THAT NO NON-THEIST HAS VALUE
TO THEIR LIVES IS OFFENSIVE

SECOND, LIFES MEANING IS CREATED BY THE INDIVIDUAL FROM THE BEAUTY


AVAILABLE TO EXISTENCE, MAKING GOD IRRELEVENT

Infidels.org 2003
[An Introduction to Atheism, February 24,
www.infidels.org/news/atheism/intro.html, acc 1-20-05//uwyo-ajlo]

Perhaps it is to some, but still, many atheists live a purposeful life. They decide what they
think gives meaning to life, and they pursue those goals. They try to make their lives
count, not by wishing for eternal life, but by having an influence on other people who will
live on. For example, an atheist may dedicate his life to political reform, in the hope of leaving
his mark on history.

It is a natural human tendency to look for "meaning" or "purpose" in random events. However,
it is by no means obvious that "life" is the sort of thing that has a "meaning".

To put it another way, not everything which looks like a question is actually a sensible thing to
ask. Some atheists believe that asking "What is the meaning of life?" is as silly as asking
"What is the meaning of a cup of coffee?". They believe that life has no purpose or
meaning, it just is.

Also, if some sort of mystical external force is required to give one's existence a
"meaning", surely that makes any hypothetical god's existence meaningless?

THIRD, LIFE ONLY BECOMES VALUELESS WHEN IT IS DECLARED AS SUCH [author is


describing specific men who were in Auschwitz with him]

Victor Frankl, Professor of Neurology and Psychiatry at the University of Vienna, Mans Search for
Meaning, 1946, p. 90-93

We have stated that that which was ultimately responsible for the state of the prisoners inner self was not so much the enumerated psychophysical causes as it was the result
only the men who allowed their inner hold on their moral
of a free decision. Psychological observations of the prisoners have shown that

and spiritual selves to subside eventually fell victim to the camps degenerating influences. The question now arises, what
could, or should, have constituted this inner hold? Former prisoners, when writing or relating their experiences, agree that the most depressing influence of all was that a
prisoner could not know how long his term of imprisonment would be. He had been given no date for his release. (In our camp it was pointless even to talk about it.) Actually a
prison term was not only uncertain but unlimited. A well-known research psychologist has pointed out that life in a concentration camp could be called a provisional existence.
We can add to this by defining it as a provisional existence of unknown limit. New arrivals usually knew nothing about the conditions at a camp. Those who had come back
from other camps were obliged to keep silent, and from some camps no one had returned. On entering camp a change took place in the minds of the men. With the end of
uncertainty there came the uncertainty of the end. It was impossible to foresee whether or when, if at all, this form of existence would end. The latin word finis has two

meanings: the end or the finish, and a goal to reach. A man who could not see the end of his provisional existence was not

Page 1342 of 1481


able to aim at an ultimate goal in life. He ceased living for the future, in contrast to a man in normal life. Therefore the whole
structure of his inner life changed; signs of decay set in which we know from other areas of life. The unemployed worker, for example, is in a
similar position. His existence has become provisional and in a certain sense he cannot live for the future or aim at a goal. Research work done on unemployed miners has
shown that they suffer from a peculiar sort of deformed timeinner time-which is a result of their unemployed state. Prisoners, too, suffered from this strange time-experience.
In camp, a small time unit, a day, for example, filled with hourly tortures and fatigue, appeared endless. A larger time unit, perhaps a week, seemed to pass very quickly. My
comrades agreed when I said that in camp a day lasted longer than a week. How paradoxical was our time-experience! In this connection we are reminded of Thomas Manns
The Magic Mountain, which contains some very pointed psychological remarks. Mann studies the spiritual development of people who are in an analogous psychological
position, i.e., tuberculosis patients in a sanatorium who also know no date for their release. They experience a similar existencewithout a future and without a goal. One of the
prisoners, who on his arrival marched with a long column of new inmates from the station to the camp, told me later that he had felt as though he were marching at his own
funeral. His life had seemed to him absolutely without future. He regarded it as over and done, as if he had already died. This feeling of lifelessness was intensified by other
causes: in time, it was the limitlessness of the term of imprisonment which was most acutely felt; in space, the narrow limits of the prison. Anything outside the barbed wire
became remoteout of reach and, in a way, unreal. The events and the people outside, all the normal life there, had a ghostly aspect for the prisoner. The outside life, that is,
as much as he could see of it, appeared to him almost as it might have to a dead man who looked at it from another world. A man who let himself decline because he could not
see any future goal found himself occupied with retrospective thoughts. In a different connection, we have already spoken of the tendency there was to look into the past, to

danger. It became easy to overlook the


help make the present, with all its horrors, less real. But in robbing the present of its reality there lay a certain

opportunities to make something positive of camp life, opportunities which really did exist. Regarding our provisional
existence as unreal was in itself an important factor in causing the prisoners to lose their hold on life;
everything in a way became pointless. Such people forget that often it is just such an exceptionally difficult external situation which gives man the
opportunity to grow spiritually beyond himself. Instead of taking the camps difficulties as a test of their inner strength, they did not take their life seriously and despised it as

something of no consequence. They preferred to close their eyes and to live in the past. Life for such people became meaningless.

Page 1343 of 1481


A2 Life Without God is Terrifying: 1AR

FIRST, THIS IS INEVITABLE. SOME SORT OF FEAR WILL EXIST NO MATTER WHAT.
CHRISTIANS TRY TO SURVIVE JUST LIKE ANYONE ELSE

SECOND, TURN - THE BELIEF IN GOD CREATES MORE TERROR BECAUSE YOU HAVE
TO WORRY ABOUT THE THE ETERNAL FATE OF YOUR SOUL. AT LEAST OUR FEAR IS
FINITE

THIRD, THERE ARE A NUMBER OF WAYS TO COPE WITH TERROR IN A GODLESS


FRAMEWORK LIKE KITTENS, THE NEW WILLIAM SHATNER ALBUM, OR ALCOHOL

Infidels.org 2003
[An Introduction to Atheism, February 24,
www.infidels.org/news/atheism/intro.html, acc 1-20-05//uwyo-ajlo]

There are many ways of obtaining comfort:

Your family and friends

Pets

Food and drink

Music, television, literature, arts and entertainment

Sports or exercise
Meditation

Psychotherapy

Drugs

Work

That may sound like rather an empty and vulnerable way to face danger, but so what? Should
individuals believe in things because they are comforting, or should they face reality no matter
how harsh it might be?
In the end, it's a decision for the individual concerned. Most atheists are unable to believe
something they would not otherwise believe merely because it makes them feel comfortable.
They put truth before comfort, and consider that if searching for truth sometimes makes
them feel unhappy, that's just hard luck. Often truth hurts.

Page 1344 of 1481


Alternative Hurts Religion

RELIGION SHOULD BE A PRIVATE MATTER ENTANGLEMENT ALLOWS


ENROACHMENT ON THE RELIGIOUS

Stevenjaygould.org no date
[Religious Court Rulings, www.stephenjaygould.org/ctrl/courtrulings.html, acc 1-
20-05//uwyo-ajl]

In the 1992 Lee v. Weisman case, the Court ruled that public schools may not sponsor invocations at
graduation ceremonies. Justice Anthony M. Kennedy wrote: "The First Amendment's Religion Clauses
mean that religious beliefs and religious expression are too precious to be either proscribed or prescribed
by the State. The design of the Constitution is that preservation and transmission of religious beliefs and
worship is a responsibility and a choice committed to the private sphere, which itself is promised freedom
to pursue that mission. It must not be forgotten then, that while concern must be given to define the prot
ection granted to an objector or a dissenting nonbeliever, these same Clauses exist to protect religion
from government interference."

Page 1345 of 1481


Page 1346 of 1481
**Securitization**

Page 1347 of 1481


Security Good: Helps
Marginalized People
SECURITIZATION IS EMANCIPATINGGIVING MARGINALIZES ISSUES LIKE HUMAN
RIGHTS VISIBILITY

Jeff Huysmans, Lecturer in politics at the department of government at Open University, Alternatives
Defining Social Constructivism in Security Studies: The Normative Dilemma of Writing Security Feb 2002
p. 59-60.

There is no solution for the normative dilemma in the social-constructivist security analyses defined
above. The particular understanding of language makes any security utterance potentially securitizing.
Consequently, enunciating security is never innocent or neutral. Of course, this does not have to result in
a normative dilemma; it does so only if one wants to or has to utter security in a political context while
wanting to avoid a securitization of a particular area. Someone may also employ security language with
the intention of securitizing an area. This does not necessarily require a conservative interest in keeping
the status quo or in establishing law and order. Securitization can also be performed with an
emancipatory interest. Given the capacity of security language to prioritize questions and to mobilize
people, one may employ it as a tactical device to give human-rights questions a higher visibility, for
example. It is also possible to mobilize security questions in nonsecurity areas with the intention to
change the conservative bias of the security language. This would require a positive concept of security
that defines liberation from oppression as a good that should be secured.

Page 1348 of 1481


Alt Bad: Allows Suffering to Continue

REJECTING OUR PLAN IN FAVOR OF THE CRITIQUE PRIVILEGES SEMANTICS OVER


REAL HUMAN SUFFERING WE SHOULD BE ABLE TO RESPOND TO THREATS TO
HUMAN LIFE AND DIGNITY IN THE STATUS QUO WITHOUT WORRYING ABOUT
WHETHER WE OVERLY SECURITIZE SUCH THREATS.

Nicholas Onuf, Professor, International Relatoins, Florida International University, Symposium on the
Norms and Ethics of Humanitarian Intervention, Center for Global Peace and Conflict Studies Working Paper
00-03, May 5, 2000, http://hypatia.ss.uci.edu/gpacs/OnufHumanitarian.pdf.

Paradoxically, if an emergency is defined as a situation calling for immediate action, then these
situations cease to be emergenciesimmediate action remedied nothing. In the meantime, human misery
deepens. It is no wonder, then, that suffering becomes secondary, as violations of human rights take
priority. At least this is a tendency among progressive liberals for whom the situation has become an
inescapable morass, and for whom human rights are the great project of social reform in our time.

Critics of liberalism think little of the human rights movement. They are disposed to see social reform
activism, and more generally the development of civil society, as a manifestation of global liberal
governance or, more scornfully, liberal peace. According to Michael Dillon and Julian Reid, liberal
peace finds itself deeply implicated in a terrain of disorder in which some states are powerful, some states
are in radical dissolution, traditional societies are collapsing and civil conflict is endemic, where
international corporations and criminal cartels are also involved, and where international organizations
and nongovernmental organizations are inextricably committed as well. Dillon and Reid have argued
against calling the more striking manifestations of global disorder complex emergencies because doing
so unduly simplifies their vexed political character and masks the degree to which global liberal
governance is implicated in making them so vexed. Their alternative descriptionemerging political
complexesimplies that the people who want to call these situations emergencies are cynically
motivated. Perhaps some humanitarian liberals are cynically motivated; others no doubt have complex
political motivationspeople always do. Yet banishing emergency from our vocabulary because people
have mixed motives in calling for immediate action has the untoward result of forestalling action that
could help the many victims of the liberal peace and its global disorder. Progressive liberals and their
critics both end up making suffering secondary to their own programmatic concerns.

Page 1349 of 1481


Alt Fails: Engagement/Nonengagement Doublebind

ITS A DOUBLE-BIND: EITHER THE ALTERNATIVE LINKS OR DOESNT SOLVE. IN


ISOLATION, ALTERNATIVES CANT INDIVIDUALLY DECONSTRUCT DOMINANT
SECURITY DISCOURSE. THEY MUST ENGAGE WITH SECURITY, BUT IN DOING SO
LEGITIMIZE THE PRACTICES BEING CRITIQUED.

Jeff Huysmans, Lecturer in politics at the department of government at Open University, Alternatives
Defining Social Constructivism in Security Studies: The Normative Dilemma of Writing Security Feb 2002
p. 50-51.

Although the critical edge of this literature cannot be ignored, denaturalizing security fields is not
necessarily successful in moderating the normative dilemma. The research continues to map the security
discourses, therefore repeating, in an often highly systematic way, a security approach to, for example,
migration or drugs. Demonstrating the contingent character of the politicization does question the
foundational character of this contingent construction, but it does not necessarily undermine the real
effects. It does this only when these discourses rely heavily for their effects on keeping the natural
character of its foundations unquestioned. This points to a more general issue concerning this kind of
analysis. Although it stresses that language makes a difference and that social relations are constructed, it
leaves underdeveloped the concept of security formation that heavily prestructures the possibilities to
speak differently through rarifying who can speak security, what security can be spoken about, how one
should speak about security, and so on.27 Another related problem is that the approach assumes that in
dictating the mere existence of alternative practices challenges the dominance of the dominant discourse.
This is problematic since the alternative constructions do not exist in a vacuum or in a sheltered space. To
be part of the game, they must, for example, Coy test political constructions of migration. Alternative
practices are thus not isolated but engage with other, possibly dominant, constructions. This raises the
question of how the engagement actually works. It involves relations of power, structuring and
restructuring the social exchanges. Staging alternative practices does not necessarily challenge a
dominant construction. The political game is more complex, as Foucaults interpretation of the sexual
revolution the liberation from sexual repressionof the second half of the twentieth century
showed.28 In a comment on human-rights approaches of migration, Didier Bigo raises a similar point
that opposing strategies do not necessarily radically challenge established politicizations: It is often
misleading to counterpose the ideology of security to human rights because they sometimes have more in
common than their authors would like to admit. They often share the same concept of insecurity and
diverge only in their solutions.29 The main point is that alternative discourses should not be left in a
vacuum. The way they function in the political struggle should be looked at. How are the alternative
discourses entrenched in a specific political game? Are they possibly a constitutive part of the mastery of
the dominant construction?

Page 1350 of 1481


Alt Fails: Securitizes Itself

USELESSTHE ALTERNATIVE ACKNOWLEDGES BUT CANT OVERCOME THE


NORMATIVE SECURITY DILEMMA

Jeff Huysmans, Lecturer in politics at the department of government at Open University, Alternatives
Defining Social Constructivism in Security Studies: The Normative Dilemma of Writing Security Feb 2002
p. 52-53.

As already said, theorization means that authors explain the structuring work of the discursive formation.
They interpret the power-knowledge nexus by locating it in symbolic and institutional contexts. The first
question is therefore a heuristic one of how to understand what is happening, rather than a critical
question of how to intervene in the securitization of societal areas. To some extent, this theoretical
agenda engages with the dilemma in a traditional way: at some point, it separates the research question
from the question What is to be done? This does not mean that the agenda ignores the latter question;
rather, the interpretation of why and how an issue is structured into a security question is a precondition
for answering the practical question.

But this more traditional way of dealing with the normative dilemma is only one side of the theoretical
game. The theoretical approach also engages with social relations in a more direct way that is, without
separating the research question from the practical one. A theorization of power relations and the
symbolic dimensions of the security formation can be critical in itself. By explicitly uncovering
dimensions of the security formation that are commonly left implicit, it performs a critical practice. More-
over, explaining the work of power relations involved in the securitization of societal questions is a
politicizing act in itself. As Stefano Guzzini remarks: integrating social relations in a power analysis
politicizes the issue in question since power is a concept that is generally used to define what counts as
a political issue, what it is possible to change.33 This does not imply that this form of social
constructivisrn claims that it escapes the normative dilemma. Due to its interpretation of language, it
cannot but accept that security enunciations risk the opening of space for successful securitizing practices.
The bottom line is, then, that the agenda has to accept the normative dilemma as a dilemma. It cannot
escape that its own security writing risks contributing to the securitization of an area. As a general
statement, it shares this position with the other research projects I have sketched. It differs from the
others in the specific way in which it hopes to moderate the risk of reifying security threats that is, by
theorizing the power-knowledge nexus and interpreting securitization as a specific political strategy.

Page 1351 of 1481


Perm Solves: Starting Point

SECURITY HAS MEANING. WE MUST ASSUME THAT SECURITY IS UNIVERSAL IN


ORDER TO DECONSTRUCT IT.

Anthony Burke, Prof at the School of Political Science and International Studies at University of
Queensland, Alternatives Aporias of Security 2002 p. 2

While this article argues strongly that security has no essential ontological integrity, it also argues that if
the power and sweep of security are to be understood and challenged, its claims to universality must be
taken seriously. They underpin and animate sweeping forms of power, subjectivity, force, and economic
circulation and cannot be dismissed out of hand. Nor, in the hands of some humanist writerswho have
sought to think human and gender security in radical counterpoint to realist images of national and
international securityare such claims always pernicious. They have a valuable moral and political force
that undermines, perhaps unwittingly, the logocentric presuppositions of the realist discourses they
question. Yet a common assumption that security can be ontologically completed and secured does
present a hurdle for the kind of ontopolitical critique that we really need.2 The answer is not to seek to
close out these aporias; they call to us and their existence presents an important political opening. Rather
than seek to resecure security, to make it conform to a new humanist idealhowever laudablewe need
to challenge security as a claim to truth, to set its meaning aside. Instead, we should focus on security as
a pervasive and complex system of political, social, and economic power, which reaches from the most
private spaces of being to the vast flows and conflicts of geopolitics and global economic circulation. It is
to see security as an interlocking system of knowledges, representations, practices, and institutional
forms that imagine, direct, and act upon bodies, spaces, and flows in certain waysto see security not as
an essential value but as a political technology. This is to move from essence to genealogy: a genealogy
that aims, in William Connollys words, to open us up to the play of possibility in the present . . . [to]
incite critical responses to unnecessary violences and injuries surreptitiously imposed upon life by the
insistence that prevailing forms are natural, rational, universal or necessary. 3

Page 1352 of 1481


Perm Solves: Must Act

USING THE STATE DOESNT MEAN WE THINK IT IS PERFECT. WE WORK WITH IT


BECAUSE THERES NO OTHER OPTION

Eric Mazur, doctoral candidate in the Department of Religious Studies at the University of California Santa Barbara,

1997, American Indian Studies, p. 251

We might add also that notions of authority, sovereignty, and political participation are not necessarily constructed on a single intellectual foundation. In the case of Nathan Jim,
our introduction to this wide- ranging conflict over authority, as well as in the broader historical development of the relationship of Native American religious traditions and the
American constitutional order, there are clear differences over how authority is determined, and by whom and under what circumstances. Native traditions, centered (at least in
part) on the cultural orientation toward land, cannot but conflict with the American constitutional order's orientation toward the same land. Not as easily integrated into American
culture as Christianity's symbolic emphasis on "The Word" (and its parallel relationship to the Constitution as symbolic of the federal government's authority dependent on
territoriality), Native American religious traditions expose the very real and tangible conflict that lies at the heart of the American constitutional order. The strengths behind the

. Nathan Jim may not


Constitution are grounded in the control of the land, and any challenge to that control can be met with subtle, but immeasurable resistance

see the legal system of the American constitutional order as his law, but he has understood the power it
holds over him, and has agreed to abide by it. So, too, in many ways, have Native American religious traditions
agreed to abide by the American constitutional order. They may not accept the source of its authority, but
in the face of overwhelming power, they may have had no other choice but to accept it.

WE GOTTA DO SOMETHING AND CANNOT TOTALLY KRITIK SUBJECTIVITY

Anthony Burke, Prof at the School of Political Science and International Studies at University of
Queensland, Alternatives Aporias of Security 2002 p. 22

It is perhaps easy to become despondent, but as countless struggles for freedom, justice, and social
transformation have proved, a sense of seriousness can be tempered with the knowledge that many tools
are already availableand where they are not, the effort to create a productive new critical sensibility is
well advanced. There is also a crucial political opening within the liberal problematic itself, in the sense
that it assumes that power is most effective when it is absorbed as truth, consented to and desired
which creates an important space for refusal. As Cohn Gordon argues, Foucault thought that the very
possibility of governing was conditional on it being credible to the governed as well as the governing.60
This throws weight onto the question of how security works as a technology of subjectivity. It is to take up
Foucaults challenge, framed as a reversal of the liberal progressive movement of being we have seen in
Hegel, not to discover who or what we are so much as to refuse what we are.61 Just as security rules
subjectivity as both a totalizing and individualizing blackmail and promise, it is at these levels that we can
intervene. We can critique the machinic frameworks of possibility represented by law, policy, economic
regulation, and diplomacy, while challenging the way these institutions deploy language to draw
individual subjects into their consensual web. This suggests, at least provisionally, a dual strategy. The first
asserts the space for agency, both in challenging available possibilities for being and their larger
socioeconomic implications. Roland Bleiker formulates an idea of agency that shifts away from the lone
(male) hero overthrowing the social order in a decisive act of rebellion to one that understands both the
thickness of social power and its fissures, fragmentation, and thinness. We must, he says, observe
how an individual may be able to escape the discursive order and influence its shifting boundaries. . . . By
doing so, discursive terrains of dissent all of a sudden appear where forces of domination previously
seemed invincible.62 Pushing beyond security requires tactics that can work at many levelsthat
empower individuals to recognize the larger social, cultural, and economic implications of the everyday
forms of desire, subjection, and discipline they encounter, to challenge and rewrite them, and that in turn

Page 1353 of 1481


contribute to collective efforts to transform the larger structures of being, exchange, and power that
sustain (and have been sustained by) these forms. As Derrida suggests, this is to open up aporetic
possibilities that transgress and call into question the boundaries of the self, society, and the international
that security seeks to imagine and police.

Page 1354 of 1481


A2 Dillon: 2AC

DILLON DOESNT ADVOCATE REJECTION ENDORSING THE POLITICAL ACT OF


PLAN IS CONSISTENT WITH IS CALL FOR ANOTHER FORM OF JUSTICE

Dillon 99
[Michael, Prof. IR @ Lancaster, Another Justice, Political Theory 27: 2, April, Sage//uwyo]

Inordertobeatall,then,thiswayofbeinghastoposeandrespondtothe

questionwhatitistobe.Indoingsoittakesitsbearingcomposureoftran-

sits,plots,courses,andfixesfromtheconnectednessinthemidstofwhich

italwaysalreadyfindsitself.Moreoftenthannot,itisonlywhenthosenavi-

gationalaidsaredisrupted,anditsautomaticpilotsbreakdown,thatitfully

recognises its radically hermeneutical condition. It is at these points,

especially,thatthecallofanotherJusticeresoundsmostloudlythroughoutits

hermeneuticism. Here the bearing of a new bearing may be assumed. Each

alwayshastobeassumedquestioningly,however,withinagivenworld;and

none ever exhausts the task of having to do so. For another Justice always

already arises within and alongside is vented through the legislation, exe-

cution, and adjudication of existing distributive regimes.

This making way for other ways of being to be is a political art. Other jus-

tices emerge out of the injustices of regimes of distributive justice in response

to the call of another Justice. That is why there is an intimate link between

another Justice and politics. Such a politics isneitherasupposedlyhabitual

tradition,acontractualnegotiation,noranepistemicallyrealistcomputation

ofthecorrelatesofrigorouslyself-interestedbehaviour.It is an irruptive and

inventive practice called up by specific historical circumstances. Politics

becomes that way of being (politeia) whose composure is an art of intima-

tion,articulation,intervention,andjudgment.It is a practice that responds to

the call ofanotherJustice.There is no guarantee that it will be available when

required, just as there is no guarantee that it will be successful should it be

exercised,orthateverybodyisabletopracticeitondemand.Toooftenrule,

managementdecision,andviolenceoccludeit.Recognisablewhenitmakes

Page 1355 of 1481


itsappearance, we have to bear witness to it.

ETHICS AND POLITICS CAN CO-EXIST THE PERM IS OPTIMAL

Dillon 99
[Michael, Prof. IR @ Lancaster, Another Justice, Political Theory 27: 2, April, Sage//uwyo]

Philosophys task, for Levinas, is to avoid conflating ethics and politics.

The opposition of politics and ethics opens his first major work, Totality and

Infinity, and underscores its entire reading. This raises the difficult question

of whether or not the political can be rethought against Levinas with Levinas.

Nor is this simpl y a matter ofaskingwhetherornotpoliticscanbeethical.It

embracesthequestionofwhetherornottherecanbesuchathingasanethic

ofthepolitical.Herein,then,liesanimportantchallengetopoliticalthought.

Itarisesasmuchfortheontopoliticalinterpretationasitdoesfortheunder-

standingofthesourceandcharacterofpoliticallifethatflowsfromthereturn

oftheontological. For Levinas the ethical comes first and ethics is first phi-

losophy. But that leaves the political unregenerated, as Levinass own defer-

ral to a Hobbesian politics, as well as his very limited political interventions,

indicate. In this essay I understand the challenge instead to be the necessity

of thinking the co-presence of the ethical and the political. Precisely not the

subsumption of the ethical by the political as Levinas charges, then, but the

belonging together of the two which poses, in addition, the question of the

civil composure required of a political life.

Page 1356 of 1481


**Speaking for Others**
A2 Speaking for Others: 2AC (1/2)

FIRST, THERES NO WAY TO DETERMINE THE LINK. IDENTITY IS FLUID AND YOU
DONT KNOW HOW WE IDENTIFY

SECOND, TURN THEY ASSUME A STATIC NOTION OF GROUP IDENTITY BY


ISOLATING IT AS BEING ABSOLUTELY OTHER, DENYING WITHIN GROUP
DIFFERENCE AND UNSTABLE IDENTIFICATION OF INDIVIDUALS

--THAT FLIPS THE INTERNAL LINK

Butler 99
[Judith, prof. of rhetoric at UC Berkeley, Gender Trouble: Feminism and the
Subversion of Identity, New York: Routledge, 1999, 18-19//uwyo-ajl]

Feminist critique ought to explore the totalizing claims of a masculinist signifying economy, but also remain self-critical with respect to the totalizing gestures of feminism. The effort
to identify the enemy as singular in form is a reverse-discourse that uncritically mimics the strategy of the
oppressor instead of offering a different set of terms. That the tactic can operate in feminist and antifeminist contexts alike suggests that the colonizing gesture is not primarily or
irreducibly masculinist. It can operate to effect other relations of racial, class, and heterosexist subordination, to name but a few. And clearly, listing the varieties of oppression, as I began to
do, assumes their discrete, sequential coexistence along a horizontal axis that does not describe their convergences within the social field. A vertical model is similarly insufficient; oppressions
cannot be summarily ranked, causally related, distributed among planes of originality and derivativeness. Indeed, the field of power structured in part by the imperializing gesture of
dialectical appropriation exceeds and encompasses the axis of sexual difference, offering a mapping of intersecting differentials which cannot be summarily hierarchized either within the

Rather than an exclusive tactic of masculinist


terms of phallogocentrism or any other canddidate for the position of primary condition of oppression.

signifying economies, dialectical appropriation and suppression of the Other is one tactic among many
deployed centrally but not exclusively in the service of expanding and rationalizing the masculinist domain.

THIRD, TURN RETREAT FROM SPEAKING FOR OTHERS IS ANOTHER FORM OF


PRIVILEGE THAT ALLOWS VOICES TO BE TRAMPLED SPEAKING EXCLUSIVELY FOR
YOURSELF IS IMPOSSIBLE

Alcoff 92
[Linda, Prof. of Feminist Studies at the University of Syracuse, The Problem of Speaking for Others,
Cultural Critique, Winter 91-2, 20//uwyo]

This problem is that Trebilcots position, as well as a more general retreat position, presumes an ontological
configuration of the discursive context that simply does not obtain. In particular, it assumes that one can
retreat into ones discrete location and make claims entirely and singularly based on that location that do
not range over others, that one can disentangle oneself from the implicating networks between ones discursive
practices and others locations, situations, and practices. (In other words, the claim that I can speak only for myself assumes the
autonomous conception of the self in Classical Liberal theory that I am unconnected to other in my authentic self or that I can achieve an autonomy from others given certain

But there is no neutral place to stand free and clear in which ones words do not prescriptively
conditions.)

affect or mediate the experience of others, nor is there a way to decisively demarcate a boundary

Page 1357 of 1481


between ones location and all others. Even a complete retreat from speech is of ocurse not neutral since
it allow the continued dominance of current discourses and acts by omission to reinforce dominance.

. The declaration
As my practices are made possible by events spatially far from my body so too my own practices make possible or impossible practices of others

that I speak only for myself has the sole effect of allowing me to avoid responsibility and accountability
for my effects on others; it cannot literally erase those effects.

FOURTH, ALCOFF ONLY SAYS THAT CLAIMING TO SPEAK ON BEHALF OF THE OTHER
IS A BAD THING, NOT THAT MAKING ANY CLAIM ABOUT THEM IS BAD. WE DONT
CLAIM TO REPRESENT OR EVEN KNOW WHAT ALL OF __________ THINK.

Page 1358 of 1481


Page 1359 of 1481
A2 Speaking for Others: 2AC
(2/2)
FIFTH, PERFORMATIVE CONTRADICTION THE VERY CLAIM THAT ACTING ON
BEHALF OF A GROUP HURTS THAT GROUP IS AN ATTEMPT TO ACT ON BEHALF OF
THAT GROUP, DESTROYING ALTERNATIVE SOLVENCY, SKEWING THE 2AC BECAUSE
WE HAVE TO ANSWER MULTIPLE WORLDS, AND DESTROYIGN EDUCATION
BECAUSE OF A LACK OF ACCOUNTABILITY WHICH IS A VOTER FOR FAIRNESS AND
EDUCATION

SIXTH, PERM DO BOTH AFFIRM THE 1AC AND ENGAGE IN CRITICISM OF


REPRESENTATIONS THAT CLAIM TO ACT ON BEHALF OF THE OTHER

SEVENTH, THE PERM SOLVES SPEAKING ERRORS ARE INEVITABLE AND GOOD
BECAUSE THEY PROVIDE A LOCUS FOR CONSTANT CRITICISM, SOMETHING THE
NEG BY ITSELF PRECLUDES

Alcoff 92
[Linda, Prof. of Feminist Studies at the University of Syracuse, The Problem of Speaking for Others,
Cultural Critique, Winter 91-2, 22//uwyo]

But surely it is both morally and politically objectionable to structure ones actions around the desire to
avoid criticism, especially if this outweighs other questions of effectivity. In some cases perhaps the
motivation is not so much to avoid criticism as to avoid errors, and the person believes that the only way
to avoid errors is to avoid all speaking for others. However, errors are unavoidable in the theoretical
inquiry as well as political struggle, and moreover they often make contributions. The desire to find an
absolute means to avoid making errors comes perhaps not from a desire to advance collective goals but a
desire for personal mastery, to establish a privileged discursive posotion wherein one cannot be
undermined or challenged and thus is master of the situation. From such a position ones own location
and positionality would not require constant interrogation and critial reflection; one would not hae to
constantly engage in this emotionally troublesome endeavor and would be immune from the interrogaton
of others. Such a desire of rmastery and immunity must be resisted.

EIGHTH, NO SPECIFIC LINK STORY POSITIONALITY UNDERDETERMINES THE


EFFECT OF A SPEECH ACT ABSENT SPECIFIC ANALYSIS OF HOW WE DISEMPOWER,
THEIR ARGUMENT IS REDUCTIONIST AND NOT A REASON TO REJECT

Page 1360 of 1481


Alcoff 92
[Linda, Prof. of Feminist Studies at the University of Syracuse, The Problem of Speaking for Others,
Cultural Critique, Winter 91-2]

The first response I will consider is to argue that the formulation of the problem with speaking for others
involves a retrograde metaphysically unsupportable essentialism that assumes one can read the truth
and meaning of what one says straight from the discursive context. This response I will call the charge of
reductionism response, because it argues that a sort of reductionist theory of justification (or evlauation)
is entailed by premises 1 and 2. Such a reductionist theory might, fo rexample, reduce evaluation to a
political assessment of the speakers location where that location is seen as an insurmountable essence
that fixes one, as if ones feet are superglued to a spot on the sidewalk.

Page 1361 of 1481


#3 Retreat: 1AR

EXTEND THE 2AC #3 ALCOFF 92 EVIDENCE. THIS DOES TWO THINGS FOR US

IT PROVES THAT THERES NO ALTERNATIVE TO SPEAKING FOR OTHERS. EVERY


DISCURSIVE POSITION PRESUPPOSES ENGAGEMENT WITH THE WORLD, MEANING
THAT EVEN IF YOU VOTE NEGATIVE, YOU STILL SPEAK FOR OTHERS INTERESTS,
ONLY IN A MORE IMPLICIT WAY, PROVING THAT THE ALTERNATIVE LINKS JUST AS
BADLY

IT DEMONSTRATES HOW A RETREAT FROM SPEAKING FOR OTHERS CREATES NEW


FORMS OF OPPRESSION BY OMMITTING DISCUSSION OF OPPRESSION, ALLOWING
ONE TO ESCAPE REAL WORLD VIOLENCE INTO A SELF-IMPORTANT YUPPIE
LIFESTYLE, ALLOWING STATUS QUO DOMINATION TO OCCUR, UNCHECKED,
TURNING THEIR ARGUMENT

ALSO, FALLING BACK TO ACADEMIC CRITICISM ALLOWS A RETREAT FROM


POLITICAL ENGAGEMENT AND INCREASED EXPLOITATION OF THE OPPRESSED FOR
PERSONAL GAIN

Alcoff 92
[Linda, Prof. of Feminist Studies at the University of Syracuse, The Problem of Speaking for Others,
Cultural Critique, Winter 91-2, 13//uwyo\

Neither premise 1 nor premise 2 entail reductionism or essentialism. They argue for the relevance of
location, not its singular power of determination. Since they do not specify how we are to understand the
concept of location, it can certainly be given a nonessential meaning.

While the charge of reductionism response has been popular among academic theorists, a second
response which I will call the retreat response has been popular among some sectionso f the US feminist
movement. This response is simply to retreat from all practices of speaking for and assert that one can
only know ones own narrow individual experience and ones own truth and can enver make claims
beyond this. This response is motivated in part by the desire to recognize difference, for example,
different priorities, without organizing these differences into hierarchies.

Now, sometimes I think this is the proper response ot the problem of speaking for others, depnding on
who is making it. We certainly want to encourage a more receptive listening on the part of the
discursively privileged and discourage presumptuosu and oppressive practices of speaking for. But a
retreat from speaking for will not result in an increase in receptive listening in all cases; it may resul
tmerely in a retreat into a narcissistic yuppie lifestyle in which a privileged person takes no

Page 1362 of 1481


responsibility for her society whatesoever. She may even feel justified in exploiting her priveleged
capacity for personal happiness at the expense of others on the grounds that she has no alternative.

Page 1363 of 1481


#3 Retreat: Ext

AND RETREAT FROM POLITICAL ENGAGEMENT FOR FEAR OF VIOLENCE IN SEARCH


OF AUTHENTICITY ALLOWS US TO SPEND HOURS DEBATING THE FINE POINTS OF
ETHICS TOWARDS THE OTHER WHILE GAS CHAMBERS ARE BUILT

Bewes 97
[Timothy, doctorate in English Literature at the University of Sussex, Cynicism and Postmodernity, New
York City: Verso, 1997,146-7//uwyo-ajl]

If it is unreasonable to suppose that the Final Solution was potentiated or even necessarily facilitated by
Schmitt's theories, it is certainly the case that this metaphysical structure of domination in the Third
Reich, whereby the status of public citizens is reduced to a level determined entirely in the 'natural' or
biological realm of necessity, is foreshadowed in his 1927 essay. In an abstract and insidious way Schmitt
introduces the idea that the 'transcendent' realm of the political, as a matter of course, will not
accommodate a people with insufficient strength to ensure its own participation, and that such a fact is
ipso facto justification for its exclusion. 'If a people no longer possesses the energy or the will to maintain
itself in the sphere of politics, the latter will not thereby vanish from the world. Only a weak people will
disappear.'130 Schmitt's concept of the 'political', quite simply, is nothing of the sort - is instead weighed
down by necessity, in the form of what Marshall Berman calls German-Christian interiority - by its
preoccupation with

authenticity, that is to say, and true political 'identity'. Auschwitz is a corollary not of reason,
understood as risk, but of the fear of reason, which paradoxically is a fear of violence. The
stench of burning bodies is haunted always by the sickly aroma of cheap metaphysics.

Page 1364 of 1481


#6 Perm: 1AR

EXTEND THE 2AC #6 JUXTAPOSITION PERM. ENGAGING IN CRITICISM OF SPEAKING


FOR OTHERS, BY ITSELF, FAILS BECAUSE IT MERELY FLIPS THE BINARISM AND FAILS
TO ACTUALLY ENGAGE THE DISCOURSE THAT IT CRITICIZES, CREATING A NEW
FORM OF MONOLITHIC HEGEMONY IN WHICH NOTHING IS CHALLENGED.
HOWEVER, COMBINING THE 1AC AND THE CRITICISM ALLOWS FOR CONSTANT
CRITICISM, USING THE AFFS REPRESENTATIONS AS A TARGET FOR CRITICAL
INTERROGATION, LEADING TO BETTER SOLVENCY THAN THE ALTERNATIVE BY
ITSELF. CROSS-APPLY THE ALCOFF 92 SOLVENCY EVIDENCE.

ALL OF THEIR PERM THEORY AND LINK ARGUMENTS DONT APPLY BECAUSE THIS
ISNT A STANDARD PERM. IT COMBINES THE ENTIRETY OF THE 1AC AND THE
CRITICISM AND USES THAT CONTRADICTION TO ALLOW A CONSIDERATION OF
BOTH SIDES AND THE ISSUE AND A MORE CRITICAL EXAMINATION OF SPEAKING
FOR, FUNCTIONING AS AN IMPACT TURN TO THEIR ADVOCACY OF ONE-SIDED
CRITICISM.

Page 1365 of 1481


#9 Reductionism: 1AR

EXTEND THE 2AC #9. THE ARGUMENT THAT POSITIONALITY DETERMINES


WHETHER A REPRESENTATION IS GOOD OR NOT IGNORES THE MORE
COMPLICATED ISSUE OF HOW OUR SPEECH ACT ACTUALLY OPERATES IN
DISCURSIVE SPACE

THIS HAS TWO IMPLICATIONS

IT DESTROYS THE LINK. WITHOUT AN EXPLANATION OF HOW OUR ACT


FUNCTIONS, YOU DONT HAVE ENOUGH INFORMATION TO DETERMINE THAT AN
INTERNAL LINK EXISTS

IT LOCKS THEIR CRITICISM INTO SUBJECT ESSENTIALISM THAT RESULTS IN THE


VERY OTHERIZATION THAT THEYRE CRITICIZING, TURNING THE ARGUMENT

AND, THEIR METAPHYSICS OF PRESENCE IS JUST WRONG THERE IS NO STABLY


EXISTING OTHER, ATTEMPTING TO FIT ONE INTO A DISCREET LABEL MAGNIFIES
OPPRESSION

Bewes 97
[Timothy, doctorate in English Literature at the University of Sussex, Cynicism and Postmodernity, New
York City: Verso, 1997, 48//uwyo-ajl]

In this light, to begin to use again terms and concepts which had seemed to be theoretically proscribed
(the author, the subject, reality, sexual and cultural identity, the universal) is not neces-sarily to betray a
reactionary or a nostalgic desire for 'presence'; on the contrary, what the critical insights of post-
structuralism (more specifically, deconstruction) reveal is not only the possibil-ity but the imperative that
such terms continue to be used. There are no others - and if there were, they would by definition not only
be liable to but would comprise exactly the same catachrestic abuses

AND HERES THE ALTERNATIVE (OPTIONAL)


LACANIAN ETHICS RESISTS ATTEMPTS TO REDUCE THE SUBJECT TO
IDENTITARIANISM BY FOCUSING ON CONSTITUTIVE LACK, NOT WHO IS LACKING

Page 1366 of 1481


Stavrakakis 99
[Yannis, New Age composer, Lacan and the Political, 1999, NY: Routledge, 37//uwyo-ajl]

By locating, at the place previously assigned to an essence of the individual psyche, a constitutive lack,
Lacanian theory avoids the essentialist reductionism of the social to the individual level and opens the
way to the confluence of psychoanalysis and socio-political analysis, since this lack can only be filled by
socio-political objects of identification. The point here is that analytic theory is not only concerned with
lack but also with what attempts to fill this lack: Psychoanalysis is otherwise directed at the effect of
discourse within the subject' (Ill: 135). In that sense, `Lacan.. believed in the priority of social discourses, of language, over the subject' (Copjec, 1994: 53).
This is the meaning of the constitutivity of the symbolic in the emergence of the subject that we have been describing up to now. Michelman is correct then when asserting that
`Durkheim and Lacan are thus allied in their critiques of various forms of psychological and biological reductionism that deny the existence and efficacy of facts of this order [the
symbolic/social order]' (Michelman, 1996: 127). Thus Lacan not only seems aware of the dangers pointed by Durkheim and reiterated by Jameson with which we started this
book but avoids them in the most radical way: ~there is no subject according to Lacan which is not always already a social subject' (Lacoue-Labarthe and Nancy, 1992: 30)27

Page 1367 of 1481


The Alternative is a Fantasy

CRITICIZING OUR SPEAKING IS AN ATTEMPT TO RESTORE THE SYMBOLIC


HIERARCHY. MARKING OUR DISCOURSE AS THE CONTINGENT BARRIER TO
STOPPING OPPRESSION IS A NOSTALGIC UTOPIAN FANTASY, MAKING
TOTALITARIANISM POSSIBLE

Aleman 97
[Jorge, Spanish psychoanalyst, Lacan: The End, Lacanian Ink #11, www.lacan.com/frameX16.htm acc 9-
21-04//uwyo]

The task of non-metaphysical thinking and psychoanalysis thus come to a new crossroads. They
both discuss the localization of the void and how to handle it, and with Lacan the way of writing it.
This crossroads has a political scope: the key to totalitarianism becomes intelligible while
revealing the way the Master tries to fill up the void with a law of history or of nature whose
temporality is assured in progress. This is about giving substance to the void in such a way that
everything which is not involved in this project is viewed as dregs to be eliminated. This might be
why Lacan reminded left wing militants in the mid-'60s: "I sustain that psychoanalysis has no right
to interpret revolutionary practice, rather revolutionary theory would do well to take responsibility
for leaving empty the place of truth as cause."
Lacan destroys the sphere, a privileged manner of hiding the "void of Being" while setting up a
topology of the speaking being aimed at a non-metaphoric writing, lest topology not convey the
nostalgia forever attempting to restore a certain symbolic hierarchy, a specific last word on the
real of jouissance and its empty place in the symbolic.
The emphasis of this article is not only on the fact that Heidegger unintentionally broached
psychoanalysis before it was conceived. We have yet tried to exacerbate the experience of
psychoanalysis, interrogating the fact that the speaking being may be "cured" in its core of the
most subtle form of his fantasme-the metaphysics that always returns with the meaning that may
hide its contingency.

Page 1368 of 1481


Page 1369 of 1481
**State Bad, Juhdge**

Page 1370 of 1481


Strategic Use of State Good
MORALISTIC CRUSADES AIMED AT CHANGING THE STATE ARE MISLEADING ABOUT
THE NATURE OF OPPRESSIVE FORCES. THIS OBFUSCATION DENIES THE POTENTIAL
FOR RADICAL TRANSFORMATION

Brown, Professor Political Science UC Berkeley, 2K1


(Wendy, Politics Out of History, pg. 35-37)

But here the problem goes well beyond superficiality of political analysis or compensatory gestures in the
face of felt impotence. A moralistic, gestural politics often inadvertently becomes a regressive politics.
Moralizing condemnation of the National Endowment for the Arts for not funding politically radical art, of
the U.S. military or the White House for not embracing open homosexuality or sanctioning gay marriage,
or even of the National Institutes of Health for not treating as a political priority the lives of HIV target
populations (gay men, prostitutes, and drug addicts) conveys at best naive political expectations and at
worst, patently confused ones. For this condemnation implicitly figures the state (and other
mainstream institutions) as if it did not have specific political and economic investments, as if it were
not the codification of various dominant social powers, but was, rather, a momentarily misguided
parent who forgot her promise to treat all her children the same way. These expressions of moralistic
outrage implicitly cast the state as if it were or could be a deeply democratic and nonviolent institution;
conversely, it renders radical art, radical social movements, and various fringe populations as if they
were not potentially subversive, representing a significant political challenge to the norms of the
regime, but rather were benign entities and populations entirely appropriate for the state to equally
protect, fund, and promote. Here, moralisms objection to politics as a domain of power and history
rather than principle is not simply irritating: it resuits in a troubling and confused political stance. It
misleads about the nature of power, the state, and capitalism; it misleads about the nature of
oppressive social forces, and about the scope of the project of transformation required by serious
ambitions for justice. Such obfuscation is not the aim of the moralists but falls within that more general
package of displaced effects consequent to a felt yet unacknowledged impotence. It signals disavowed
despair over the prospects for more far-reaching transformations.

WE MUST NOT REJECT THE STATE- LIMITED AND STRATEGIC USE OF THE STATE IS
VITAL TO SUCCESSFUL POLITICS

Derrida, French philosopher, 2K


(Jacques, Intellectual Courage: An Interview Culture Machine
http://culturemachine.tees.ac.uk/Cmach/Backissues/j002/articles/art_derr.htm)

Q: Two essential problems of globalisation are the dissolution of the state and the impotence of politics. In your recently published text 'Cosmopolites de tous les pays, encore
un effort!', you develop certain ideas concerning a new right to asylum and a new balance of power between the different places of the political in view of a possible new role of
the city. How do you think philosophy could and should react to the problems mentioned with a kind of institutional fantasy?

JD: I am not sure I understand what you call 'institutional fantasy'. All political experimentation like the initiative of the 'refugee city', despite its limits and its inevitably preliminary
character, has in it a philosophical dimension. It requires us to interrogate the essence and the history of the state. All political innovation touches on philosophy. The 'true'

political action always engages with a philosophy. All action, all political decision making, must invent its norm or rule. Such a
one must fight against that which you
gesture traverses or implies philosophy. Meanwhile, at the risk of appearing self-contradictory, I believe that

Page 1371 of 1481


call the 'dissolution of the state' (for the state can in turn limit the private forces of appropriation, the
concentrations of economic power, it can retard a violent depoliticisation that acts in the name of the
'market'), and above all resist the state where it gives in too easily to the nationalism of the nation
state or to the representation of socio-economic hegemony. Each time one must analyse, invent a new
rule: here to contest the state, there to consolidate it. The realm of politics is not co-extensive with the
state, contrary to what one believes nowadays. The necessary repoliticisation does not need to serve a
new cult of the state. One ought to operate with new dissociations and accept complex and
differentiated practices.

Page 1372 of 1481


Page 1373 of 1481
State is Key to Solving
Oppression (1/2)
INDIVIDUAL ACTION ALONE IS NOT ENOUGHTHE STATE IS CRITICAL TO
JUMPSTART HUMAN RIGHTS RECORDS THAT PREVENT WAR AND OPPRESSION

William W. Burke-White, Lecturer, Public and International Affairs and Senior Special Assistant to the
Dean, Woodrow Wilson School of Public and International Affairs, Princeton University, Human Rights and
National Security: The Strategic Correlation, THE HARVARD ENVIRONMENTAL LAW REVIEW v. 17, Spring
2004, p. 266-267.

The social beliefs explanation begins from the proposition that individuals within human rights protecting states share a preference for a minimum set of protections of human rights. This

If the
assumption is appropriate for two reasons. First, according to liberal political science theory, state policy represents the preferences of some subset of the domestic polity. n100

observed state policy is to protect human rights, then at least some subset of the domestic polity must share that preference. Second, even if
individuals within a domestic polity seek a variety of differentiated ends, basic respect for human rights
allows individuals to pursue--to some degree at least--those ends as they define them. Liberal theory thus suggests that
individuals within a human rights respecting state tend to support basic human rights provisions. The next step in the social beliefs argument is to recognize that respect for human rights has
an inherently universalist tendency. n101 Unlike cultural or national rights, human rights are just that--human. They apply as much [*267] to those individuals within a domestic polity as to
those outside the polity. Such cosmopolitan liberalism indicates that "the more people are free, the better off all are." n102 The net result is that individuals within a human rights respecting

Given a set of universalist human rights values in states that


state tend, on the average, to support the human rights of individuals in other states as well.

, the policy articulated by the government may be one which respects human rights at home
respect human rights

and demands their protection abroad. This belief in a thin set of universal human rights may cause the
leadership of the state to frame its security policy around that belief structure and to refrain from
aggressive acts that would violate the human rights of citizens at home or abroad. As Peter Katzenstein argues, "security
interests are defined by actors who respond to cultural factors." n103 Acts of international aggression tend to impinge on the human rights of individuals in the target state and, at least
temporarily, limit their freedom. After all, bombs, bullets, death and destruction are not consistent with respect for basic human rights. n104 Framed in the liberal international relations
theory terms of policy interdependence, international aggression by State A imposes costs on State B, whose citizens' human rights will be infringed upon by the act of aggression. This
infringement in turn imposes costs on citizens in State A, whose citizens have a preference for the protection of the human rights of citizens in both states. This shared value of respect for
human rights thus may restrain State A from pursuing international aggression. n105 By contrast, a state which commits gross human rights violations against its own people will not be
subject to this restraint. Such violations often occur when the government has been "captured" by a select minority that chooses to violate human rights. If the citizens themselves are not in

Where capture occurs, the government is


favor of human rights at home, they are unlikely to be committed to the enforcement of human rights abroad.

not responsive to the preferences of the domestic polity. In such cases, even if there is a strong
preference among citizens to protect human rights at home and abroad, the government is unlikely to
respond to those interests and its policies will not be constrained by them.

CALLS UPON THE STATE ARE THE ONLY WAY TO ACHIEVE SOCIAL PROGRESS THE
ALTERNATIVE IS A COMBINATION OF ANARCHY AND NIHILISM WE END UP
DITHERING IN THE FACE OF ATTROCITIES

Walzer, Professor of Social Sciences at the Institute for Advanced Studies & Former Professor at Harvard, 1983
(Michael, The Politics of Michel Foucault, Dissent, Fall)

Here again a comparison with Hobbes is illuminating. Hobbes thought that political sovereignty was a literal necessity--else life was nasty, brutish, and short. He supported
every sort of sovereignty, and so for him tyranny was nothing more than "monarchy misliked." Foucault believes that discipline is necessary for this particular society-capitalist,

for him liberalism is nothing more than discipline


modern, or whatever; he abhors all its forms, every sort of confinement and control, and so

concealed. For neither Hobbes nor Foucault does the constitution or the law or even the actual workings
of the political system make any difference. In fact, I think, these things make all the difference. One of Foucault's
followers, the author of a very intelligent essay on Discipline and Punish, draws from that book and the related interviews the extraordinary conclusion that the Russian

Page 1374 of 1481


: the Bolsheviks
Revolution failed because it "left intact the social hierarchies and in no way inhibited the functioning of the disciplinary techniques." Exactly wrong

created a new regime that overwhelmed the old hierarchies and enormously expanded and intensified
the use of disciplinary techniques. And they did this from the heart of the social system and not from what Foucault likes to cal the capillaries, from the
center and not the extremities. Foucault desensitizes his reader to the importance of politics; but politics matters. Power
relations, he says, "are both intentional and nonsubjective." I don't know what that sentence means, but I think that the contradictory words are intended (nonsubjectively?) to

Every disciplinary act is planned and calculated; power is intentional at the tactical
apply to different levels of power

level where guard confronts prisoner; doctor, patient; lecturer, audience. But the set of power relations, the strategic connections, the deer -
functionalism of power has no subject and is the product of no one's plan.

continued

Page 1375 of 1481


Page 1376 of 1481
State is Key to Solving
Oppression (2/2)
continued

Foucault seems to disbelieve in principle in the existence of a dictator or a party or a state that shapes the
character of disciplinary institutions. He is focused instead on what he thinks of as the "micro-fascism" of
everyday life and has little to say about authoritarian or totalitarian politics-that is, about the forms of
discipline that -are most specific to his own lifetime. But these are not the forms most specific to his own country, and Foucault does believe in sticking
close to the local exercise of power. Nor does he often use terms like "micro-fascism." He is not a "general intellectual" of the old sort-so he tells us-who provides an account
and critique of society as a whole.' The general intellectual belongs to the age of the state and the party, when it still seemed possible to seize power and reconstruct society.

He is, in the world of political knowledge, what the king is in the world of political power. Once we have cut off the king's head, power and knowledge alike take different forms .
Foucault's more recent work is an effort to explain these forms, to work out what can be called a political epistemology. I now want to examine this
epistemology, for it is the ultimate source of his anarchism/nihilism. Sometimes Foucault seems to be committed to nothing more than an
elaborate pun on the word "discipline"-which means, on the one hand, a branch of knowledge and on the other a system of correction and control. This is his argument: social
life is discipline squared. Discipline makes discipline possible (the order of the two nouns can be reversed). Knowledge derives from and provides the grounds for social control;
every particular form of social control rests on and makes possible a particular form of knowledge. It follows that power isn't merely repressive but also creative (even if all it
creates is, say, the science of penology); and similarly, knowledge isn't merely ideological but also true. But this doesn't make either power or knowledge terribly attractive.
Penology is "constituted" by the prison system in the obvious sense that there could not be a study of prisoners or of the effects of imprisonment if there were no prisons. One
form of discipline generates the data that makes the other possible. At the same time, penology provides both the rationale and the intellectual structure of the prison system.
There could be no exercise of discipline, at least no sustained and organized exercise, without disciplinary knowledge. It is a nice model, though perhaps a little too easy. In
any case, Foucault proceeds to generalize it. "Truth is a thing of this world: it is produced only by virtue of multiple forms of constraint. And it induces regular effects of power."
So for every society, for every historical age, there is a regime of truth, unplanned but functional, generated somehow out of the network of power relations, out of the multiple
forms of constraint, and enforced along with them. There are certain types of discourse that the society accepts "and makes . . . true," and there are mechanisms that enable us
to distinguish true and false statements-and sanctions, so that we won't make mistakes. Foucault believes that truth is relative to its sanctions and knowledge to the constraints
that produce it. There would appear to be no independent standpoint, no possibility for the development of critical principles. Of course, one can ask the obvious questions:
what is Foucault's standpoint? to what set of power relations is the genealogical antidiscipline connected'? Foucault is far too intelligent not to have worried about these
questions. They are standard for any relativism. He responds in two ways: first by saying, as I have already noted, that his genealogies are fictions waiting for the "political

realities" that will make them true. Each present invents its own past, but Foucault has invented a past for some future present. At other times, Foucault says more simply
that his work is made possible by the events of '68 and by subsequent local revolts here and there along the disciplinary continuum. As the conventional disciplines are
generated and validated by the conventional uses of power, so Foucault's antidiscipline is generated by the resistance to those uses. But I don't see, on Foucault's terms, how it
can be validated by resistance until the resistance is successful (and it's not clear what success would mean). But perhaps, after all, the demand that Foucault show us the

makes no demands on us that we adopt this or that


ground on which he stands, display his philosophical warrants, is beside the point. For he

critical principle or replace these disciplinary norms with some other set of norms. He is not an advocate.
We are to withdraw our belief in, say, the truth of penology and then support ..- what? Not every prison revolt, for there may be some that we have "good reason" not to support.
At this point, it seems to me , Foucault's position is simply incoherent. The powerful evocation of the disciplinary
system gives way to an antidisciplinarian politics that is mostly rhetoric and posturing. But there is more that has to be
said. In those prison revolts with which we might rightly sympathize, the prisoners don't in fact call into

question the line between guilt and innocence or the truth value of jurisprudence or penology. Their
"discourse" takes a very different form: they describe the brutality of the prison authorities or the
inhumanity of prison conditions, and they complain of punishments that go far beyond those to which they were legally condemned. They denounce
official arbitrariness, harassment, favoritism, and so on. They demand the introduction and enforcement of what we might best

call the rule of law. And these descriptions, complaints, denunciations, and demands make an important
point. Foucault is certainly right to say that the conventional truths of morality, law, medicine, and psychiatry are
implicated in the exercise of power; that is a fact too easily forgotten by conventionally detached scientists, social scientists, and even philosophers.
But those same truths also regulate the exercise of power. They set limits on what can rightly be done,
and they give shape and conviction to the arguments the prisoners make. The limits are important even if
they are in some sense arbitrary. They aren't entirely arbitrary, however, insofar as they are intrinsic to the
particular disciplines (in both senses of the word). The truths of jurisprudence and penology, for example, distinguish punishment from preventive detention. And
the truths of psychiatry distinguish the internment of madmen from the internment of political dissidents. A liberal state is one that maintains the

limits of its constituent disciplines and disciplinary institutions and that enforces their intrinsic principles.
Authoritarian and totalitarian states, by contrast, override those limits, turning education into
indoctrination, punishment into repression, asylums into prisons, and prisons into concentration camps .
These are crude definitions; I won't insist upon them; amend them as you will. I only want to suggest the enormous importance of the political regime, the sovereign state. For
it is the state that establishes the general framework within which all other disciplinary institutions
operate. It is the state that holds open or radically shuts down the possibility of local resistance. The
agents of every disciplinary institution strive, of course, to extend their reach and augment their discretionary
power. Ultimately, it is only state power that can stop them. Every act of local resistance is an appeal for
political or legal intervention from the center. Consider, for example, the factory revolts of the 1930s that led (in this country) to the establishment
of collective bargaining and grievance procedures, critical restraints on scientific management, which is one of Foucault's disciplines, though one that he alludes to only
occasionally. Success required not only the solidarity of the workers but also at least some support from the liberal and democratic state. And success was functional not to any
state but to a state of that sort; we can easily imagine other "social wholes" that would require other kinds of factory discipline. A genealogical account of this discipline would
be fascinating and valuable, and it would undoubtedly overlap with Foucault's accounts of prisons and hospitals. But if it were complete, it would have to include a genealogy of

Page 1377 of 1481


Here is a kind of
grievance procedures too, and this would overlap with an account, which Foucault doesn't provide, of the liberal state and the rule of law.

knowledge-political philosophy and philosophical jurisprudence-that regulates disciplinary arrangements


across our society. It arises within one set of power relations and extends toward the others; it offers a
critical perspective on all the networks of constraint. This suggests that whatever the value of detailed
analyses and critiques of local discipline, we still require-I don't mean that society requires, or capitalism or even socialism requires, but you
and I require-what Foucault calls "general intellectuals." We need men and women who tell us when state power is

corrupted or systematically misused, who cry out that something is rotten, and who reiterate the
regulative principles with which we might set things right. But I don't want to end on this last note. I don't want to ask Foucault to be
uplifting. That is not the task he has set himself. The point is rather that one can't even be downcast, angry, grim, indignant, sullen, or embittered with reason unless one
inhabits some social setting and adopts, however tentatively and critically, its codes and categories. Or unless, and this is much harder, one constructs a new setting and
proposes new codes and categories. Foucault refuses to do either of these things, and that refusal, which makes his genealogies so powerful and so relentless, is also the
catastrophic weakness of his political theory.

Page 1378 of 1481


Page 1379 of 1481
State Key to Solving War (1/2)
STRONG HUMAN RIGHTS RECORDS FOSTER PEACE--HUMAN RIGHTS PROTECTIONS
CREATE INSTITUTIONAL CONSTRAINTS ON AGGRESSION

William W. Burke-White, Lecturer, Public and International Affairs and Senior Special Assistant to the
Dean, Woodrow Wilson School of Public and International Affairs, Princeton University, Human Rights and
National Security: The Strategic Correlation, THE HARVARD ENVIRONMENTAL LAW REVIEW v. 17, Spring
2004, p. 265-266.

aggression
One causal pathway rooted in liberal international relations theory that may explain the observed correlation between systematic human rights violations and interstate

is the institutional constraint that accompanies human rights protections. n97 Institutionalization of human rights norms has at
least two powerful effects on state behavior. First, human rights protections govern how broad a spectrum of the community

has at least some voice in the political decisions of the state. Even if the state is not a democratic polyarchy, if it provides basic protections for the human rights of all or most
citizens, then a very broad spectrum of the polity is represented in political affairs. Freedom of thought and freedom from extrajudicial bodily harm, for example, allow citizens to develop

voices, in turn, increases the level of


their own views on political issues and, often, to express those views through public channels. A wider spectrum of

political competition--one of the key structural explanations for the democratic peace--even without the establishment
of a democratic form of government. n98 Of course, in a non-democratic, but human rights respecting state, the views of individual

interests may not have a direct effect on state policy, but, arguably, they can still increase the level of political competition by facilitating

debate and the exchange of ideas. The second effect of institutionalized protections of human rights is to set a minimum floor of treatment for all citizens within
the domestic polity. Even in a non-democracy, minimum human rights protections ensure that [*266] rights are accorded to

individuals not directly represented by the government. By ensuring a minimum treatment of the unrepresented, human rights protections prevent the
government from externalizing the costs of aggressive behavior on the unrepresented. In human rights
respecting states, for example, unrepresented individuals cannot be forced at gunpoint to fight or be bound
into slavery to generate low-cost economic resources for war, and thus restrain the state from engaging in
aggressive action. On the other hand, in a state where power is narrowly concentrated in the hands of a political elite that systematically represses its own people, the state will
be more able to bear the domestic costs of war. By violating the human rights of its own citizens, a state can force individuals to fight or support the military apparatus in its war-making

denial of freedom of
activities. Similarly, by denying basic human rights, a state may be better able to bear the political costs of war. Even if such a state had fair elections,

thought and expression might well insulate the government from the electoral costs of an aggressive
foreign policy. n99

HUMAN RIGHTS CONCIOUSNESS AT THE STATE LEVEL CHECKS CONFLICT: (1)


FOSTERS HUMAN RIGHTS CULTURE; (2) EXPANDS CITIZEN OPPOSITION; (3)
UNDERMINES STATE COERCION TOWARDS WAR

William W. Burke-White, Lecturer, Public and International Affairs and Senior Special Assistant to the
Dean, Woodrow Wilson School of Public and International Affairs, Princeton University, Human Rights and
National Security: The Strategic Correlation, THE HARVARD ENVIRONMENTAL LAW REVIEW v. 17, Spring
2004, p. 271-272.

The institutionalization of human rights protections is not only a means of signaling benign intent, but is also inversely correlated with a
state's ability to engage in aggressive conduct. As a state embeds human rights protections in its domestic
system--even without democratization--a number of structural changes occur within the society that limit
aggressive potential. First, as Thomas Risse and Kathryn Sikkink have argued, a culture of human rights may develop within the population and become institutionalized

Page 1380 of 1481


institutionalization of
domestically. n121 Such a human rights culture would reject international aggression as a threat to the human rights of citizens in other states. Second,

human rights protections expands the ability of citizens to voice opposition to aggressive state policy through
freedoms of belief, speech, and assembly. Third, institutionalization erodes the ability of the state to coerce its citizens into

providing the resources and human capital necessary for aggressive war. n122

Page 1381 of 1481


Page 1382 of 1481
State Key to Solving War (2/2)

STATE ENFORCEMENT OF HUMAN RIGHTS SOLVES WAR

William W. Burke-White, Lecturer, Public and International Affairs and Senior Special Assistant to the
Dean, Woodrow Wilson School of Public and International Affairs, Princeton University, Human Rights and
National Security: The Strategic Correlation, THE HARVARD ENVIRONMENTAL LAW REVIEW v. 17, Spring
2004, p. 271.

human rights informed foreign policy would include


In dealing with states of concern, improving a given state's human rights policy is almost never a primary goal of U.S. policy. A

should be advocated not just for the traditional


far more active advocacy for improvement in some states' human rights records. Such policies

human rights reasons of life and human dignity, n115 but also because improved human rights records may
enhance national and global security by preventing states from engaging in international aggression in the
future. Even for skeptics of the universal duty to promote human rights on grounds of individual dignity, this second argument should have persuasive weight in asserting the strategic
importance of human rights in U.S. foreign policy.

Page 1383 of 1481


Page 1384 of 1481
Alternative Creates Worse
Oppression (1/2)
DIRECT OPPOSITION TO THE STATE CREATES NEW, WORSE FORMS OF MAOIST
REPRESSION ONLY ALLOWING THE STATE TO FAIL UNDER ITS OWN WEIGHT CAN
CHALLENGE THE STATUS QUO

BAD Press 92
[Anarchism and Civility, BAD Broadside #6, June, http://world.std.com/~bbrigade/badbsd6.htm, acc. 10-
2-06//uwyo-ajl]

.
A generally accepted anarchist tenet is that the State can only be effectively dismantled by a voluntary, cooperative and spontaneous insurrection by the people

Authoritarian revolutions gotten up by manipulative vanguardists are rejected as inconsistent with the
anarchist belief that the means must be consistent with the ends. History has plenty of examples to show
that seizure of power through elitist revolt, rather than furthering the goals of the revolution, actually becomes a process for the
strengthening of the State in a new and more vicious form. From an evanescent moment of exultant freedom one inevitably
wakes up to the hangover of a Napoleon or a Lenin or a Mao.

The only
Nevertheless, contemporary anarchists are often still mesmerized by the call to arms, even when the chance of such a romantic gesture succeeding is nil.

real revolutions occur when popular discontent causes the state to collapse under the weight of its own
folly, not when some bloody vanguard, following whatever destructive fantasy its leaders concoct, meets
the modern state head-on. This inevitably results in meaningless hardship for the people involved, with
the greatest misery reserved for innocents who gets in the way of either side's fallacious ideology. Being a
"rebel" and antagonizing the flatulent powers-that-be in a modern state can be an exciting game, but it is only bluster and puerile self-gratification when genuine revolt is
implausible. In the end the most radical "revolutionaries" either end up as bitter, dead-end martyrs or become the next generations' "born-again" capitalists. Having had their
fling, they come to believe in their new "realism" as solipstically as they embraced rebellion. None of this brings us any closer to a solution to the problem of the State.

Everyone is pre-judged by
The fallacy of revolutionary adventurism is mirrored on a personal level by the intolerant and abusive discourse of identity politics.

their race, gender, sexual or religious affiliation, and socially compartmentalized in some politically correct
egg basket. The goal of the anarchist movement is to establish a free, tolerant and cooperative society which will embrace diversity and celebrate difference. If the
means are to be consistent with the ends, then how can such a abrasive and bigoted practice as identity politics possibly achieve that end? Identifying the

"enemy" by birth or predilection, regardless of an individual's actual beliefs or actions, is simple bigotry. Awarding moral virtue on the same grounds is simple
stupidity. Similarly, essaying to act as a unwarranted spokesperson for a diverse grouping of individuals who by chance share a single basic characteristic is the most

Real people, stripped of their individual identities, are thus subsumed in some hypothetical
arrogantsort of elitism.

single-dimensional construct that effectively denies them any complexity of character. This isn't an
answer to institutionalized racism and bigotry, but rather its mirror image.
This sort of prejudicial activity has appeal for the simpleminded. It's easy to either attack or adulate a stranger on the grounds of appearance. A similar anxiety powered the old
Sumptuary laws which punished anyone who dressed above their social class -- it was too unnerving for the elite to think they might make a mistake and treat an inferior as an
equal, thanks to illicit appearances. Political prejudice makes it simple to get through the difficulty of rootless modern life where there are no clear cut exterior indications of
what a person might really be like. All white males (unless, perhaps, gay) are dangerous, power-driven and bigoted. All women (unless, perhaps, Republican) are intuitive,
nurturing and empathetic with Nature. Members of minorities (take your pick) are morally superior to members of majorities. Classifications and labels which assist us in making
such decisions are more real (and more important) than the people they describe. Et cetera. Bullshit.

The goal of a tolerant and cooperative society of free individuals can only be achieved by those very means -- by
being tolerant, cooperative and free. We must be better companions to our fellow mortals, whatever their outward characteristics. Civility,
which facilitates cooperation, is imperative if anarchy is to really work. Pigheaded and self-important
aggressiveness, hypercriticism and easy intolerance is a recipe for the status quo. We don't mean to suggest some sort of
all accepting, "turn-the-other-cheek" bourgeois crap, either. Once you get beyond the labels, there are still unfortunately plenty of folks that it makes sense to despise. Arrogant,
violent, intolerant, fanatical, bigoted, manipulative, rapacious... individuals with these characteristics must be guarded against, but they are not all found in one easily
recognized group identity. These adjectives equally describe individual men, women, blacks, whites, handicapped people -- the whole gamut of the human race. Nor is anyone
as morally pure as some of our new puritan idealists would insist that they be. A person is the sum of their character traits, not a distillation of the most pronounced ones.
Radicals are just as prone to frailties of character as industrialists. It is by their actual effect on their community and environment that we should evaluate our fellow beings, not
by some dominant virtue or fault which particularly excites us. It would be far preferable to tolerate a insensitive verbal bigot who in practice actually helped people than a pious
hypocrite who mouthed politically correct platitudes and then went home and beat his lover.

Page 1385 of 1481


Page 1386 of 1481
Alternative Creates Worse
Oppression (2/2)
THEY TOTALIZE THE STATE IN REVERSE, REINSCRIBING ITS FLAWS AND
PREVENTING REORIENTATION NECESSARY FOR EFFECTIVE POLITICS

Williams & Krause 97


[Michael, Asst. Prof. of Poli Sci @ Southern Maine, & Keith, Prof. of Polic Sci @ Grad Inst. Intl Stud, Critical
Security Studies, xvi//uwyo-ajl]

Many of the chapters in this volume thus retain a concern with the centrality of the state as a locus not only of obligation but of effective political action. In the realm of

. The task of a critical approach is not to deny the centrality of the


organized violence, states also remain the preeminent actors

state in this realm but rather, to understand more fully its structures, dynamics, and possibilities for reorientation. From a critical
perspective, state action is flexible and capable of reorientation, and analyzing state policy need not therefore

be tantamount to embracing the statist assumptions of orthodox conceptions. To exclude focus on state action from a
critical perspective on the grounds that it plays inevitably within the rules of existing conceptions simply reverses

the error of essentializing the state. Moreover, it loses the possibility of influencing what remains the most
structurally capable actor in contemporary world politics.

Page 1387 of 1481


Page 1388 of 1481
Alternative Causes Power
Vaccuum
STATISM IS NECESSARY TO PREVENT A POWER VACCUUM ALLOWING WORSE
OPPRESSION BY CORPORATE POWER

Knox 2000
[R. Redemocracy, http:///www.marininternet.com/rknox/car15.html, acc. 10-2-06//uwyo-ajl]

We are fools to think that free markets will do anything other than to consolidate and monopolize the
marketplace, which is their nature to do if uncontrolled or unchecked by responsible democratic
government. Uncontrolled and unchecked top-down hyper-capitalism is as anti-social,-- as harmful to
individuals, to families and to our village values as totalitarian communism and its top-down central
control proved to be for countries in the Eastern Block.
This is precisely why our so-called "FOUNDERS" did not grant Constitutional rights to corporations. Rather they required corporations to petition for Legislative Charters to
incorporate for specific social benefit for time certain periods, after which they were un-incorporated. Our founders held corporations in the same low or suspect repute as they
did the Crown of England for the same economic reasons... the Crown and corporations shared an equal potential for evil anti-individual behaviors. Fortunately, although there
is an obvious corporate bias in law, corporate speech does not rise to a Constitutionally guaranteed right---at least not yet. But you can bet that corporations are working to
achieve this, too. After all, corporations have won through politics and legal machinations what it was not granted by the Founders,---the status, protections and rights of
individual personhood, actually super-personhood, as corporations (fake, pseudo or contrived persons) enjoy far more benefits and protections in many respects than do
individuals (real persons).

CORPORATIONS = Viritual Persons Who Enjoy More Power and Benefits than Real People, by design.

Corporations enjoy many benefits that have correctly been labeled as super-personhood, and have won legislative benefits that are not extened to individuals which are

. Corporations are now in a position of usurping individual rights essentially


granted specific rights under the Constitution

becoming a private government equally or more insidious than any undemocratic form of government.

Page 1389 of 1481


Page 1390 of 1481
Alternative Causes Nuclear War
AVOIDING STRATEGIC USE OF THE STATE LEADS TO NUCLEAR COUNTER-
REVOLUTIONS

Martin Shaw, 2001


[Review of International Studies, The unfinished global revolution: intellectuals and the new politics of
international relations,

http://nationalism.org/library/science/ir/shaw/shaw-ris-2001-27-04.pdf]

The new politics of international relations require us, therefore, to go beyond the anti-imperialism of the
intellectual left as well as of the semi-anarchist traditions of the academic discipline. We need to
recognize three fundamental truths. First, in the twenty-first century people struggling for democratic
liberties across the non- Western world are likely to make constant demands on our solidarity.
Courageous academics, students and other intellectuals will be in the forefront of these movements. They
deserve the unstinting support of intellectuals in the West. Second, the old international thinking in which
democratic movements are seen as purely internal to states no longer carries convictiondespite the
lingering nostalgia for it on both the American right and the anti-American left. The idea that global
principles can and should be enforced worldwide is firmly established in the minds of hundreds of millions
of people. This consciousness will become a powerful force in the coming decades. Third, global state-
formation is a fact. International institutions are being extended, and (like it or not) they have a symbiotic
relation with the major centre of state power, the increasingly internationalized Western conglomerate.
The success of the global-democratic revolutionary wave depends first on how well it is consolidated in
each national contextbut second, on how thoroughly it is embedded in international networks of
power, at the centre of which, inescapably, is the West.

From these political fundamentals, strategic propositions can be derived. First, democratic movements
cannot regard non-governmental organizations and civil society as ends in themselves. They must aim to
civilize local states, rendering them open, accountable and pluralistic, and curtail the arbitrary and violent
exercise of power. Second, democratizing local states is not a separate task from integrating them into
global and often Western-centred networks. Reproducing isolated local centres of power carries with it
classic dangers of states as centres of war.84 Embedding global norms and integrating new state centres
with global institutional frameworks are essential to the control of violence. (To put this another way: the
proliferation of purely national democracies is not a recipe for peace.)

Third, while the global revolution cannot do without the West and the UN, neither can it rely on them
unconditionally. We need these power networks, but we need to tame them too, to make their messy
bureaucracies enormously more accountable and sensitive to the needs of society worldwide. This will
involve the kind of cosmopolitan democracy argued for by David Held.85 It will also require us to
advance a global social-democratic agenda, to address the literally catastrophic scale of world social
inequalities. This is not a separate problem: social and economic reform is an essential ingredient of
alternatives to warlike and genocidal power; these feed off and reinforce corrupt and criminal political
economies. Fourth, if we need the global-Western state, if we want to democratize it and make its
institutions friendlier to global peace and justice, we cannot be indifferent to its strategic debates. It
matters to develop international political interventions, legal institutions and robust peacekeeping as
strategic alternatives to bombing our way through zones of crisis. It matters that international
intervention supports pluralist structures, rather than ratifying Bosnia-style apartheid.86

As political intellectuals in the West, we need to have our eyes on the ball at our feet, but we also need to
raise them to the horizon. We need to grasp the historic drama that is transforming worldwide

Page 1391 of 1481


relationships between people and state, as well as between state and state. We need to think about how
the turbulence of the global revolution can be consolidated in democratic, pluralist, international
networks of both social relations and state authority. We cannot be simply optimistic about this prospect.
Sadly, it will require repeated violent political crises to push Western and other governments towards the
required restructuring of world institutions.87 What I have outlined is a huge challenge; but the
alternative is to see the global revolution splutter into partial defeat, or degenerate into new genocidal
warsperhaps even nuclear conflicts. The practical challenge for all concerned citizens, and the
theoretical and analytical challenges for students of international relations and politics, are intertwined.

Page 1392 of 1481


Page 1393 of 1481
Permutation Solvency (1/3)
WE MUST USE THE INSTITUTIONS THAT EXERCISE POWER TO CHANGE THEM

Lawrence Grossburg, Professor, University of Illinois, WE GOTTA GET OUTTA THIS PLACE, 1992, p. 391-
393.

The Left needs institutions which can operate within the systems of governance, understanding that such
institutions are the mediating structures by which power is actively realized. It is often by directing
opposition against specific institutions that power can be challenged. The Left has assumed from some
time now that, since it has so little access to the apparatuses of agency, its only alternative is to seek a
public voice in the media through tactical protests. The Left does in fact need more visibility, but it also
needs greater access to the entire range of apparatuses of decision making and power. Otherwise, the
Left has nothing but its own self-righteousness. It is not individuals who have produced starvation and the
other social disgraces of our world, although it is individuals who must take responsibility for eliminating
them. But to do so, they must act within organizations, and within the system of organizations which in
fact have the capacity (as well as the moral responsibility) to fight them. Without such organizations, the
only models of political commitment are self-interest and charity. Charity suggests that we act on behalf
of others who cannot act on their own behalf. But we are all precariously caught in the circuits of global
capitalism, and everyones position is increasingly precarious and uncertain. It will not take much to
change the position of any individual in the United States, as the experience of many of the homeless, the
elderly and the fallen middle class demonstrates. Nor are there any guarantees about the future of any
single nation. We can imagine ourselves involved in a politics where acting for another is always acting for
oneself as well, a politics in which everyone struggles with the resources they have to make their lives
(and the world) better, since the two are so intimately tied together! For example, we need to think of
affirmation action as in everyones best interests, because of the possibilities it opens. We need to think
with what Axelos has described as a planetary thought which would be a coherent thoughtbut not a
rationalizing and rationalist inflection; it would be a fragmentary thought of the open totalityfor what
we can grasp are fragments unveiled on the horizon of the totality. Such a politics will not begin by
distinguishing between the local and the global (and certainly not by valorizing one over the other) for the
ways in which the former are incorporated into the latter preclude the luxury of such choices. Resistance
is always a local struggle, even when (as in parts of the ecology movement) it is imagined to connect into
its global structures of articulation: Think globally, act locally. Opposition is predicated precisely on
locating the points of articulation between them, the points at which the global becomes local, and the
local opens up onto the global. Since the meaning of these terms has to be understood in the context of
any particular struggle, one is always acting both globally and locally: Think globally, act appropriately!
Fight locally because that is the scene of action, but aim for the global because that is the scene of agency.
Local struggles directly target national and international axioms, at the precise point of their insertion
into the field of immanence. This requires the imagination and construction of forms of unity,
commonality and social agency which do not deny differences. Without such commonality, politics is too
easily reduced to a question of individual rights (i.e., in the terms of classical utility theory); difference
ends up trumping politics, bringing it to an end. The struggle against the disciplined mobilization of
everyday life can only be built on affective commonalities, a shared responsible yearning: a yearning out
towards something more and something better than this and this place now. The Left, after all, is defined
by its common commitment to principles of justice, equality and democracy (although these might
conflict) in economic, political and cultural life. It is based on the hope, perhaps even the illusion, that
such things are possible. The construction of an affective commonality attempts to mobilize people in a
common struggle, despite the fact that they have no common identity or character, recognizing that they
are the only force capable of providing a new historical and oppositional agency. It strives to organize
minorities into a new majority.

Page 1394 of 1481


Page 1395 of 1481
Permutation Solvency (2/3)
REFORMIST STRATEGIES CHALLENGE THE STATE TOO THEY ARENT MUTUALLY
EXCLUSIVE WITH RADICAL APPROACHES

Dixon, Founding Member of Direct Action Network, 2K5 (Chris, Reflections on Privilege, Reformism, and
Activism, http://www.geocities.com/kk_abacus/ioaa/dixon2.html)

Evidently sasha doesn't grasp my argument in "Finding Hope." Or else he disagrees. It's difficult to tell because, while skillfully sidestepping engagement with my discussion of privilege, he also

we have to move beyond the myopic view--


sidesteps the main thrust of my essay: rethinking radicalism, particularly in the context of privilege. As I wrote, "

often endemic among anarchists--that the most 'important' activism only or mainly happens in the streets,
enmeshed in police confrontations." In other words, spheres of traditional 'radical action' are limited and limiting. And though I don't believe that sasha fundamentally disagrees with this
criticism, he refuses to accept its broader consequences. For instance, where I question the bounds of 'radicalism' with examples of struggles like opposing prison construction and establishing
community and cultural centers, he conclusively points to "a set of demands and goals of which none suggest any serious critique of capitalism and the state in their totality." There is much
more to the "totality" that we all confront than capitalism and the state. That's unequivocal. Furthermore, a "totality" has an undeniable physical presence, and people do in fact contest and
resist it every day through a variety of struggles using a variety of means--not all containing the "serious critique" necessary to satisfy sasha. J. Kellstadt nicely observes this, noting that an
'activist' perspective (not unlike sasha's) overlooks a whole layer of more "everyday" forms of resistance - from slacking off, absenteeism, and sabotage, to shopfloor "counter-planning" and
other forms of autonomous and "unofficial" organizing - which conventional activists and leftists (including most anarchists) have a bad track record of acknowledging. And this still leaves out
all of those modes of struggle which take place beyond the shopfloor, such as various forms of cultural and sexual revolution. Unfortunately, sasha doesn't deign to discuss these all-too-
pedestrian realities, many of which potentially embrace the very anarchist ethics he touts. They certainly have bearing on the lives of many folks and speak to a breadth of social struggle, but
they apparently don't constitute a sufficient "critique." Even if sasha were to acknowledge their importance, my sense is that he would erect a rationalized theoretical division between
Kellstadt's "everyday forms of resistance" and 'reformism.' No doubt, he would use a rhetorical sleight of hand on par with the "simple fact of language that those who want to reform the
present system are called reformists." A seemingly irrefutable, self-apparent statement, this actually glosses over legitimate questions: Are 'reformists' so easily discernable and cleanly
categorized? Are all 'reforms' equal? Can they be part of a long-term revolutionary strategy? So let's talk plainly about reformism. No matter how much some might wish otherwise, it simply
isn't a cut-and-dry issue. And while it actually deserves a book-length examination, here I'll sketch some general considerations. Principally, I ask, assuming that we share the goal of
dismantling systems of power and restructuring our entire society in nonhierarchical ways, what role does reform play? Must we eschew it, unconditionally embrace it, or is there another
approach? sasha steadfastly represents one rather limited 'radical' view. To bolster his critique of 'reformism,' for instance, he critically cites one of the examples in my essay: demanding
authentic public oversight of police. "[This] might be a small step for social change in some general sense," he argues, "but ultimately it is a step backwards as it strengthens the legitimacy of

in real
the police and of imposed decision." I respect the intent of this critique; it makes sense if one is privileged enough to engage with the police on terms of one's own choosing. Yet

life, it's both simplistic and insulated. Look at it this way: accepting sasha's argument, are we to wait until the coming insurrectionary
upheaval before enjoying an end to police brutality ? More specifically, are African- American men to patiently
endure the continued targeting of "driving while Black"? Should they hold off their demands for police
accountability so as to avoid strengthening "the legitimacy of the police and of imposed decision "? And if they
don't, are they 'reformists'? Many folks who experience daily police occupation understand that ending the "imposed decision " (often

epitomized by police) will require radical change , and they work toward it. At the same time, they demand authentic public

oversight of police forces. The two don't have to be mutually exclusive. I'll even suggest that they can be complementary,
especially if we acknowledge the legacies of white supremacy and class stratification embedded in policing. Ultimately, we need a lucid conception of social

change that articulates this kind of complementarity. That is, we need revolutionary strategy that links
diverse, everyday struggles and demands to long-term radical objectives, without sacrificing either. Of course,
this isn't to say that every so-called 'progressive' ballot initiative or organizing campaign is necessarily radical or strategic. Reforms are not all created equal. But some can

fundamentally shake systems of power, leading to enlarged gains and greater space for further
advances. Andre Gorz, in his seminal book Strategy for Labor, refers to these as "non-reformist" or "structural" reforms. He contends, "a struggle for non-reformist reforms--for anti-
capitalist reforms--is one which does not base its validity and its right to exist on capitalist needs, criteria, and rationales. A non-reformist reform is determined not in terms of what can be,

but what should be." Look to history for examples: the end of slavery, the eight-hour workday, desegregation. All were born
from long, hard struggles, and none were endpoints. Yet they all struck at the foundations of power (in these
cases, the state, white supremacy, and capitalism), and in the process, they created new prospects for revolutionary change . Now

consider contemporary struggles: amnesty for undocumented immigrants, socialized health care, expansive environmental protections,

indigenous sovereignty. These and many more are arguably non-reformist reforms as well. None will single-handedly
dismantle capitalism or other systems of power, but each has the potential to escalate struggles and sharpen
social contradictions. And we shouldn't misinterpret these efforts as simply meliorative incrementalism ,
making 'adjustments' to a fundamentally flawed system. Certainly that tendency exists, but there are plenty of other folks working very consciously within a far more radical strategy, pushing

To fight for alternative solutions," Gorz writes, "and for structural reforms (that is to say, for intermediate
for a qualitative shift in struggle. "

is not to fight for improvements in the capitalist system; it is rather to break it up, to restrict it, to
objectives)

create counter-powers which, instead of creating a new equilibrium, undermine its very foundations ."
Thankfully, this is one approach among a diverse array of strategies, all of which encompass a breadth of struggles and movements. Altogether, they give me hope.

Page 1396 of 1481


Page 1397 of 1481
Permutation Solvency (3/3)
SMALL STEPS FORWARD ARE POSSIBLE AND NECESSARY ITS THE ONLY WAY TO
MAKE THE SYSTEM ANY BETTER WITHOUT THROWING OUR HANDS UP AND
DECIDING THAT NOTHING IS POSSIBLE

Walzer, Professor of Social Sciences at the Institute for Advanced Studies & Former Professor at Harvard, 1983
(Michael, The Politics of Michel Foucault, Dissent, Fall)

I have suggested that all micro-forms of discipline are functional to a larger system. Foucault sometimes calls this system capitalism, but he
also gives it a number of more dramatic names: the disciplinary society, the carceral city, the panoptic regime and, most frightening (and misleading) of all, the carceral archipelago. But
whatever this larger system is, it isn't the political system, the regime or constitution. It isn't determined by a Hobbist sovereign, shaped by a legislator or a founding convention, controlled
through a judicial process. The crucial point of Foucault's political theory is that discipline escapes the world of law and right-and then begins to "colonize" that world, replacing legal
principles with principles of physical, psychological, and moral normality. Thus in his book on prisons: "Although the universal juridicism of modern society seems to fix limits on the exercise of
power, its universally widespread panopticism enables it to operate, on the underside of the law, a machinery both immense and minute...." And the code by which this machinery operates is
a scientific, not a legal code. The function of discipline is to create useful subjects, men and women who conform to a standard, who are certifiably sane or healthy or docile or competent, not

The triumph of professional or scientific norms


free agents who invent their own standards, who, in the language of rights, "give the law to themselves."

over legal rights and of local discipline over constitutional law is a fairly common theme of contemporary
social criticism. It has given rise to a series of campaigns in defense of the rights of the mentally ill, of
prisoners, hospital patients, children (in schools and also in families). Foucault himself has been deeply involved in prison reform or--1 had better be careful--in a
political practice with regard to prisons that might give rise to reforms. And indeed there have been reforms (in this country at least, but I suspect in Europe too):

new laws about consent, confidentiality, access to records; judicial interventions in the administration of
prisons and schools. Foucault has little to say about this sort of thing and is obviously skeptical about its
effectiveness. Despite his emphasis on local struggles, he is largely uninterested in local victories. But what other victories
can he think possible, given his strategic knowledge'? Consider (1) that discipline-in-detail, the precise control of behavior, is necessary to the (unspecified) large-scale
features of contemporary social and economic life; (2) that this kind of control requires the microsetting, the finely meshed network, the local power relation, represented in ideal-typical
fashion by the cellular structure of the prison, the daily timetable of prison events, the extralegal penalties inflicted by prison authorities, the face-to-face encounters of guard and prisoner; (3)
that the prison is only one small part of a highly articulated, mutually reinforcing carceral continuum extending across society, in which all of us are implicated, and not only as captives or
victims; (4) and finally, that the complex of disciplinary mechanisms and institutions constitutes and is constituted by the contemporary human sciences-an argument that runs through all of
Foucault's work, to which I will return. Physical disciplines and intellectual disciplines are radically entangled; the carceral continuum is validated by the knowledge of human subjects that it

how can Foucault expect anything


makes possible. Given all this-leave aside for the moment whether it adds up to a fully satisfactory account of our social life-

more than a small reform here or there, an casing of disciplinary rigor, the introduction of more humane, if no less effective,
methods'? What else is possible? And yet sometimes, not in his books but in the interviews-and especially in a series of interviews of the early 1970s, which still reflect the impact of
May '68-Foucault seems to see a grand alternative: the dismantling of the whole thing, the fall of the carceral city, not revolution but abolition. It's for this reason that Foucault's politics are
commonly called anarchist, and anarchism certainly has its moments in his thought. Not that he imagines a social system different from our own, beyond discipline and sovereignty alike: "I
think that to imagine another system is to extend our participation in the present system." It is precisely the idea of society as a system, a set of institutions, that must give way to something
else-what else, we can't imagine. Perhaps human freedom requires a nonfunctionalist society whose arrangements, whatever they are, serve no larger purpose and have no redeeming social
value. The nearest thing to an account of such arrangements comes in an interview first published in November 1971. "It is possible," says Foucault, "that the rough outline of a future society
is supplied by the recent experiences with drugs, sex, communes, other forms of consciousness, and other forms of individuality." In that same interview, with some such vision in mind, he
repudiates the likely reformist results of his own prison work: "The ultimate goal of [our] interventions was not to extend the visiting rights of prisoners to 30 minutes or to procure flush
toilets for the cells, but to question the social and moral distinction between the innocent and the guilty." As this last passage suggests, when Foucault is an anarchist, he is a moral as well as a

To abolish
political anarchist. For him morality and politics go together. Guilt and innocence are the products of law just as normality and abnormality are the products of discipline.

power systems is to abolish both moral and scientific categories: away with them all! But what will be left'? Foucault does not believe, as
earlier anarchists did, that the free human subject is a subject of a certain sort, naturally good, warmly sociable, kind and loving. Rather, there is for him no such thing as a free human subject,

Foucault's radical abolitionism, if it is serious, is


no natural man or woman. Men and women are always social creations, the products of codes and disciplines. And so

nihilist. For on his own arguments, either there will be nothing left at all, nothing visibly human; or new codes and
not anarchist so much as

disciplines will be produced, and Foucault gives us no reason to expect that these will be any better than
the ones we now live with. Nor, for that matter, does he give us any way of knowing what "better" might mean.

Page 1398 of 1481


Page 1399 of 1481
No Link
PROPOSING REFORMS DOESNT LEGITIMIZE THE STATE

Frost, University of Kent, Mervyn, 96, Ethics in International Relations, p. 90-1)

A first objection which seems inherent in Donelans approach is that utilizing the modern state domain of discourse in effect sanctifies the state: it assumes
that people will always live in states and that it is not possible within such a language to consider alternatives to the system. This objection is not well founded,

by having recourse to the ordinary language of international relations I am not


thereby committed to argue that the state system as it exists is the best mode
of human political organization or that people ought always to live in states as we know them. As I
have said, my argument is that whatever proposals for piecemeal or large-scale reform of the

state system are made, they must of necessity be made in the language of the
modern state. Whatever proposals are made, whether in justification or in
criticism of the state system, will have to make use of concepts which are at
present part and parcel of the theory of states. Thus, for example any proposal for a new
global institutional arrangement superseding the state system will itself have to
be justified, and that justification will have to include within it reference to a
new and good form of individual citizenship, reference to a new legislative machinery equipped with
satisfactory checks and balances, reference to satisfactory law enforcement procedures, reference to a satisfactory arrangement for

distributing the goods produced in the world, and so on. All of these notions are notions which have been developed and finely honed within the

theory of the modern state. It is not possible to imagine a justification of a new world order

succeeding which used, for example, feudal, or traditional/tribal, discourse. More generally there is no worldwide
language of political morality which is not completely shot through with state-related notions such as citizenship, rights under law, representative government
and so on.

Page 1400 of 1481


Page 1401 of 1481
No Alternative
THE NEGATIVES PROBLEMATIZING OF STATE IDENTITY HAS NO ALTERNATIVE

Cole, professor of History @ Univ of Michigan, 95


(Juan R. I. Feature Review: Power, Knowledge, and Orientalism Diplomatic History Vol. 19 No. 3
Summer)

In short, Campbells imaginative and innovative approach places the politics of identity at the very core
of U.S. Foreign Policy. Nevertheless, this reviewer must express a few doubts about his inflection of
poststructuralist principles and Possibilities. Even if the struggle over identity formed the core of
contemporary politics on the national and international levels, the crisis of politics could not be reduced
to the crisis of representation. As much as we learn from Writing Security about the production of
identity, as little do we learn about the reconstitution of politics. Diplomats, policymakers, industrialists,
intellectuals, and social activists, to name but a few, enter the arena of identity politics under conditions
that are uneven and change over time. Campbell, however, treats identity struggles, and the strategies
of otherness and particular forms of representation that go along with them, as having neither origins
nor agency and as being unaccountable to multiple patterns of causality and specific historical
moments. Some might argue that the omissions of the question of agency and of conventional causal
explanations are the very trademarks of poststructuralism The lack of attention to historical details and
peculiarities, and to the nonprogressive movement of history through time, however, is certainly not an
inevitable price of poststructuralist analysis. Campbells alternative to the realist notion of an
essentialist and universalist search for power is a universal and ahistorical search for identity and
differentiation from the Other. Images of the American frontier, for instance, have no doubt a different
purpose and significance in an emerging as opposed to a late capitalist order. Furthermore, Campbells
critique of state- and nation-centered politics is curiously at odds with his focus on the American
identity.20 Such a systemic approach toward the history of identity struggles is perhaps natural to
political science, but not to poststructuralism. By claiming that an only vaguely specified2l
poststructuralist attitude sees theory w practice (emphasis in original) (p. i), Campbell takes a shortcut
and tends to deny any meaningful understanding of the mediation between theory and practice, or
between the discursive and the non-discursive.

Page 1402 of 1481


Page 1403 of 1481
A2 Borders: 2AC
SOVEREIGNTY IS NECESSARY FOR COALITIONS OF RESISTANCE

Gupta 92
[Akhil, Prof. Anthro @ Stanford, Cultural Anthropology 7(1), JSTOR//uwyo-ajl]

Second, just as formal equality of citizens in the nation-state often constitutionally enshrined (Andersons
deep horizontal comradeship), so the equality of nation-states in the world system is given concrete
expression the charter and functioning of international organizations such as the United Nations. The
independence of third world countries, dependent as it is on the international order of the United
Nations, thus redirects spatial identity from the nation at the same time that it produces it.

Last, independence from colonial rule made it imperative for postcolonial third world nation-states to
examine the nature and meaning of sovereignty. They soon realized that the independence they had
fought so hard to obtain could not be sustained under the pressure exerted by the superpowers to
incorporate them into clientistic relationships. The only way to resist this pressure was to band together
and form a common front and to use this union strategically to prevent absorption into either bloc.
Sovereignty not only depends on the protection of spatial borders, but it is above all the ability of state
elites to regulate activities that flow across those borders, such as the crossing of commodities and
surpluses, the passage of people in the form of labor, tourists, et cetera, and the movement of cultural
products and ideas. It is significant that the agenda of successive meetings of nonaligned nations moved
from an initial emphasis on the Cold War and colonialism to questions of imperialism, unequal trading
relationships, and the new information order. It was realized that economic dependence, indebtedness,
and cultural imperialism were as great, if not greater, dangers to sovereignty as was military invasion. The
Nonaligned Movement thus represented an effort on the part of economically and militarily weaker
nations to use the interstate system to consolidate the nation-state.

Page 1404 of 1481


Page 1405 of 1481
**Terror Talk**

Page 1406 of 1481


Terror Talk Answers: 2AC (1/5)
FIRST, NO LINK WE DONT SAY THAT TERROR IS INTRINSICALLY CONNECTED TO
ISLAM, WHICH IS WHAT THEIR EV DESCRIBES. ALL INTENTIONAL KILLING OF
INNOCENTS IS BAD

SECOND, NO IMPACT - RHETORIC DOESNT SHAPE REALITY

Fram-Cohen 85
[Michelle, Reality, Language, Translation: What Makes Translation Possible? American Translators Association Conference,
enlightenment.supersaturated.com/essays/text/michelleframcohen//possibilityoftranslation.html, 9-24-06//uwyo-ajl]

Nida did not provide the philosophical basis of the view that the external world is the common source of all languages. Such a basis can be found in the philosophy of

reality is independent of
Objectivism, originated by Ayn Rand. Objectivism, as its name implies, upholds the objectivity of reality. This means that

consciousness, consciousness being the means of perceiving ?reality, not of creating it. Rand defines language as "a
code of visual-auditory symbols that denote concepts." (15) These symbols are the written or spoken words of any language. Concepts are defined as the "mental integration of
two or more units possessing the same distinguishing characteristic(s), with their particular measurements omitted." (16) This means that concepts are abstractions of units

Since
perceived in reality. Since words denote concepts, words are the symbols of such abstractions; words are the means of representing concepts in a language.

reality provides the data from which we abstract and form concepts, reality is the source of all words--and of
all languages. The very existence of translation demonstrates this fact. If there was no objective reality, there could be no

similar concepts expressed in different verbal symbols. There could be no similarity between the content of different languages, and so, no
translation.

Translation is the transfer of conceptual knowledge from one language into another. It is the transfer of one set of symbols denoting concepts into another set of symbols

denoting the same concepts. This process is possible because concepts have specific referents in reality. Even if a certain word
and the concept it designates exist in one language but not in another, the referent this word and concept stand for nevertheless exists in reality, and can be referred to in
translation by a descriptive phrase or neologism. Language is a means describing reality, and as such can and should expand to include newly discovered or innovated objects
in reality. The revival of the ancient Hebrew language in the late 19th Century demonstrated the dependence of language on outward reality. Those who wanted to use Hebrew
had to innovate an enormous number of words in order to describe the new objects that did not confront the ancient Hebrew speakers. On the other hand, those objects that
existed 2000 years ago could be referred to by the same words. Ancient Hebrew could not by itself provide a sufficient image of modern reality for modern users.

THIRD, TURN: TERRORIST DISCOURSE PREVENTS DEVOLUTION INTO MORAL


NIHILISM

Jean Berthke Elshtain, Professor, Social and Political Ethics, University of Chicago,THINKING ABOUT
SEPTEMBER 11: DEFINING TERRORISM AND TERRORISTS, 2003, p. 19-20.

This line of reasoning pertains directly to how we talk about terror and terrorists. Just as the words martyr and martyrdom are distorted, whether in the Western or the Islamic
tradition, when applied not to those prepared to die as witnesses to their faith but instead to those who commit suicide while killing as many civilians as possible. So terrorist is
twisted beyond recognition if it is used to designate anyone anywhere fighting for a cause. Terrorists are those who kill people they consider their "objective enemy," no matter

what those people may or may not have done. Terrorist and terrorism entered ordinary language to designate a specific
phenomenon: killing directed against all ideological enemies indiscriminately and outside the context of a war between
combatants. According to the logic of terrorism, enemies can legitimately be killed no matter what they are doing, where they are, or
how old they are. The word terror first entered the political vocabulary of the West during the French Revolution. Those who guillotined thousands in the Place de la Concorde
a complex, subtle, and generally
in Paris were pleased to speak of revolutionary terror as a form of justice. Since the era of the French Revolution,

accepted international language has emerged to make critical distinctions between different kinds of
violent acts. Combatants are distinguished from noncombatants. A massacre is different from a battle. An ambush is different from a
firefight. When Americans look back with sadness and even shame at the Vietnam War, it is horrors like the

My Lai massacre they have in mind. Those who called the slaughter of more than 400 unarmed men, women, and children a battle were regarded as
having taken leave of their senses, perhaps because they were so determined to justify anything that Americans did in the Vietnam War that they had lost their moral

moorings.2 A terrorist is one who sows terror. Terror subjects its victims or would-be victims to paralyzing fear. In the words of the political theorist
Michael Walzer, terrorisms "purpose is to destroy the morale of a nation or a class, to undercut its solidarity; [terrorisms] method is the random murder of innocent people.
Randomness is the crucial feature of terrorist activity. If one wishes fear to spread and intensify over time, it is not desirable to kill specific people identified in some particular

way with a regime, a party, or a policy. Death must come by chance."3 Terrorism is "the random murder of innocent people." The reference is not to moral

Page 1407 of 1481


innocence, for none among us are innocent in that way, but to our inability to defend ourselves from murderous attacks as
we go to work, take a trip, shop, or ride a bus. In other words, civilians are not combatants. The designation of terrorism becomes contested because terrorists and
their apologists would prefer not to be depicted accurately. It is important to distinguish between two cases here. In some hotly contested political situations, it may be in the
interest of one side to try to label its opponents as "terrorists" rather than "combatants" or "soldiers" or "fighters." We must ask who such men (and women) are attacking. Do
they target soldiers at outposts or in the field? Do they try to disable military equipment, killing soldiers in the process? As they carry out such operations, are they open to
negotiation and diplomacy? If so, it seems reasonable to resist a blanket label of "terrorism" for what they are up to. In a situation in which noncombatants are deliberately

using terms like "fighter" or "soldier" or "noble warrior" is not only beside
targeted and the murder of the maximum number of noncombatants is the explicit aim,

collapses the distance between those who plant bombs in cafs or fly civilian
the point but pernicious. Such language

aircraft into office buildings and those who fight other combatants, taking the risks attendant upon military forms of fighting. There
is a nihilistic edge to terrorism: It aims to destroy, most often in the service of wild and utopian goals that make no sense at all in the usual political ways. The distinction
between terrorism, domestic criminality, and what we might call "normal" or "legitimate" war is vital to observe. It helps us to assess what is happening when force is used. This
distinction, marked in historic, moral, and political discourses about war and in the norms of international law, seems lost on those who call the attacks of September 11 acts of
"mass murder" rather than terrorism and an act of war under international law. It is thus both strange and disheartening to read the words of those distinction-obliterators for
whom, crudely, a dead body is a dead body and never mind how it got that way. Many of these same individuals would, of course, protest vehemently, and correctly, were
commentators, critics, and political actors to fail to distinguish between the great world religion that is Islam and the terrorists who perpetrated the events of September 11. One
cannot have it both ways, however, by insisting on the distinctions one likes and heaping scorn on those who put pressure on ones own ideological and political commitments.

if we cannot
If we could not distinguish between a death resulting from a car accident and an intentional murder, our criminal justice system would fall apart. And

distinguish the killing of combatants from the intended targeting of peaceable civilians and the deliberate
and indiscriminate sowing of terror among civilians, we live in a world of moral nihilism. In such a world, everything
reduces to the same shade of gray and we cannot make distinctions that help us take our political and moral bearings. The victims of September 11 deserve more from us.

Page 1408 of 1481


Terror Talk Answers: 2AC (2/5)
FOURTH, LANGUAGE DOESNT HAVE A DETERMINATE EFFECT WORDS ARE
EMPTY ABSENT CONTEXT, MEANING OUR RHETORIC CAN BE READ IN A
HETERODOX MANNER TO CHALLENGE VIOLENCE

FIFTH, PERM DO PLAN AND THE ALTERNATIVE REPRESENTATIONAL VIOLENCE


DOESNT PRECLUDE THE NEED FOR CONCRETE ACTION

Richard Rorty, Professor of Humanities, University of Virginia, Truth, Politics, and Postmodernism, Spinoza Lectures,

1997, p. 51-2

This distinction between the theoretical and the practical point of view is often drawn by Derrida, another writer who enjoys demonstrating that something very important
meaning, for example, or justice, or friendship is both necessary and impossible. When asked about the implications of these paradoxical fact, Derrida usually replies that

the paradox doesn't matter when it comes to practice. More generally, a lot of the writers who are labeled `post-modernist; and
who talk a lot about impossibility, turn out to be good experimentalist social democrats when it comes to actual

political activity. I suspect, for example, that Gray, Zizek, Derrida and I, if we found ourselves citizens of the same country, would all be voting for the same
candidates, and supporting the same reforms. Post-modernist philosophers have gotten a bad name because of their paradox-mongering habits, and their constant use of
terms like `impossible; `self-contradictory' and `unrepresentable'. They have helped create a cult of inscrutability, one which defines itself by opposition to the Enlightenment
search for transparency - and more generally, to the `metaphysics of presence; the idea that intellectual progress aims at getting things clearly illuminated, sharply delimited,

I am all for getting rid of the metaphysics of presence, but I think that the rhetoric of impossibility and
wholly visible.

unrepresentability is counterproductive overdramatization. It is one thing to say that we need to get rid of the metaphor of things being
accurately represented, once and for all, as a result of being bathed in the light of reason. This metaphor has created a lot of headaches for philosophers, and we would be
better off without it. But that does not show that we are suddenly surrounded by unrepresentables; it just shows that `more accurate representation' was never a fruitful way to

Even if we agree that we shall never have what Derrida calls "a full presence beyond the reach
describe intellectual progress.

of play"; our sense of the possibilities open to humanity will not have changed. We have learned nothing about the limits of
human hope from metaphysics, or from the philosophy of history, or from psychoanalysis. All that we have learned from `post-modern' philosophy is that we may need a

We have been given no reason to abandon


different gloss on the notion of `progress' than the rationalistic gloss which the Enlightenment offered.

the belief that a lot of progress has been made by carrying out the Enlightenment's political program. Since
Darwin we have come to suspect that whether such progress is made will be largely a matter of luck. But we have been given no reason to stop hoping to get lucky.

SIXTH, CRITICISM OF TERROR RHETORIC RENATURALIZES ITS CAUSES, INSTILLING


POWERLESSNESS

Rodwell 2005
[Jonathan, PhD Cand. @ Manchester Metropolitan University, Trendy But Empty: A Response to Richard
Jackson, 49th Parallel, Spring, www.49thparallel.bham.ac.uk/back/issue15/rodwell1.htm, 9-23-06//uwyo-
ajl]

The larger problem is that without clear causal links between materially identifiable events and factors
any assessment within the argument actually becomes nonsensical. Mirroring the early inability to
criticise, if we have no traditional causational discussion how can we know what is happening? For
example, Jackson details how the rhetoric of anti-terrorism and fear is obfuscating the real problems. It is
proposed that the real world killers are not terrorism, but disease or illegal drugs or environmental issues.

Page 1409 of 1481


The problem is how do we know this? It seems we know this because there is evidence that illustrates as
much Jackson himself quoting to Dr David King who argued global warming is a greater that than
terrorism. The only problem of course is that discourse analysis has established (as argued by Jackson)
that Kings argument would just be self-contained discourse designed to naturalise another arguments for
his own reasons. Ultimately it would be no more valid than the argument that excessive consumption of
Sugar Puffs is the real global threat. It is worth repeating that I dont personally believe global terrorism is
the worlds primary threat, nor do I believe that Sugar Puffs are a global killer. But without the ability to
identify real facts about the world we can simply say anything, or we can say nothing.

Page 1410 of 1481


Page 1411 of 1481
Terror Talk Answers: 2AC (3/5)
SEVENTH, CRITIQUES OF SPEECH PRODUCES A REACTIONARY POLITICS IN WHICH
CHANGE IS FOCUSED ON LANGUAGE DIRECTLY TRADING OFF WITH EFFORTS TO
REFORM THE SOCIOECONOMIC ROOT CAUSES OF INJUSTICE

Brown, Professor Political Science UC Berkeley, 2K1


(Wendy, Politics Out of History, pg. 35-37)

hate speech
Speech codes kill critique, Henry Louis Gates remarked in a 1993 essay on hate speech.14 Although Gates was referring to what happens when

regulations, and the debates about them, usurp the discursive space in which one might have offered a
substantive political response to bigoted epithets, his point also applies to prohibitions against questioning from within selected political practices or institu-
tions. But turning political questions into moralistic onesas speech codes of any sort donot only prohibits certain

questions and mandates certain genuflections, it also expresses a profound hostility toward political life
insofar as it seeks to preempt argument with a legislated and enforced truth. And the realization of that patently
undemocratic desire can only and always convert emancipatory aspirations into reactionary ones. Indeed, it insulates those aspirations from questioning at the very moment
that Weberian forces of rationalization and bureaucratization are quite likely to be domesticating them from another direction. Here we greet a persistent political paradox: the
moralistic defense of critical practices, or of any besieged identity, weakens what it strives to fortify precisely by sequestering those practices from the kind of critical inquiry out
identity-based
of which they were born. Thus Gates might have said, Speech codes, born of social critique, kill critique. And, we might add, contemporary

institutions, born of social critique, invariably become conservative as they are forced to essentialize the
identity and naturalize the boundaries of what they once grasped as a contingent effect of historically specific social
powers.

But moralistic reproaches to certain kinds of speech or argument kill critique not only by displacing it with arguments about abstract
by configuring political injustice and political righteousness as a problem of remarks, attitude,
rights versus identity-bound injuries, but also

and speech rather than as a matter of historical, political-economic, and cultural formations of power. Rather than offering
analytically substantive accounts of the forces of injustice or injury, they condemn the manifestation of these forces in particular remarks or events. There is, in the inclination to
ban (formally or informally) certain utterances and to mandate others, a politics of rhetoric and gesture that itself symptomizes despair over effecting change at more significant
levels. As vast quantities of left and liberal attention go to determining what socially marked individuals say, how they are represented, and how many of each kind appear in

, the sources that generate racism, poverty, violence against women, and other elements of
certain institutions or are appointed to various commissions

social injustice remain relatively unarticulated and unaddressed. We are lost as how to address those sources; but rather than examine this loss or
disorientation, rather than bear the humiliation of our impotence, we posture as if we were still fighting the big and good fight in our clamor over words and names. Dont mourn,
moralize.

EIGHTH, REJECTING DISCOURSE DOES NOTHING AND LEAVES ATTITUDES


UNCHANGED.

Kelly, 12/98 Peace Review

One might ask, in "listening" to violent language and to the people who use it, whether we are actually condoning such language. This is far from the case. To listen is not to

When I listen to a person who, for example, uses sexist language, I am not lending my approval to
pass judgment.

sexist language. Instead, what I am saying is that the person behind the language, and my desire to make a
connection with that person, are more important than the sexist language. If I refuse to listen to the
person who uses sexist language, then I might prevent one particular case where sexist language is used.
But I do nothing to overcome the person's sexist attitudes. She will continue to use sexist language long after I am out of sight. But if I
give her a voice, if I show her respect, if I try to take her seriously as a person, then In the future pershapes she will be more apt to take what I say about sexism seriously. If
she knows that sexist language bothers me, then perhaps she will be less likely to use it around me.

Page 1412 of 1481


Page 1413 of 1481
Terror Talk Answers: 2AC (4/5)
NINTH, LABEL POLITICS MISIDENTIFY THE CAUSE OF PREJUDICE IN AGGRESSIVITY,
SO THE NEW SYMBOL FILLS IN THE SAME ROLL
APPROPRIATIONS OF OLD LABELS RECONCEIVE THEIR MEANING

Zizek 97
[Slavoj, Moving away from the darkness, The Plague of Fantasies, New York: Verso, 1997, 111-2//uwyo-ajl]

In his formidable Fear in the Occident,7 Jean Delumeau draws attention to the unerring succession of atutudes in a medieval city infested by plague: first, people ignore it and
behave as if nothing terrible is really going on; then they withdraw into privacy, avoiding contact with each other; then they start to resort to religious fervour, staging
processions, confessing their sins, and so on; then they say to themselves 'What the hell, let's enjoy it while it lasts!', and indulge passionately in orgies of sex, eating, drinking
and dancing; finally, they return to life as usual, and again behave as if nothing terrible is going on. However, this second 'life as usual' does not occupy the same structural role
as the first: it is, as it were, located on the other side of the Moebius band, since it no longtt signals the desperate attempt to ignore the reality of plague, but, rather its exact

opposite: resigned acceptance of it . . . . Does not the same go for the gradual replacement of (sexually, racially...)
aggressive with more 'correct' expressions, like the chain nigger - Negro - black - African American or crippled - disabled -
bodily challenged? This replacement functions as a metaphorical substitution which potentially proliferate
and enhances the very (racist, etc.) effect it tries to banish, adding insult to injury. In analogy to Delumeau, one should
therefore claim that the only way actually to abolish the hatred-effect is, paradoxically, to create the

circumstances in which one can return to the first link in the chain and use it in a non-aggressive way -like
following the patterns of 'life as usual' the second time in the case of plague. That is to say: as long as the expression 'crippled' contains a

surplus, an indelible mark, of aggressivity this surplus will not only be more or less automatically
transferred on to any of its 'correct' metaphorical substitutes, it will even be enhanced by dint of this
substitution. The strategy of returning to the first link, of course, is risky; however, the moment it is fully
accepted by the group targeted by it, it definitely can work. When radical African-Americans call each other 'niggers', it is wrong to
dismiss this strategy as a mere ironic identification with the aggressor; rather, the point is that it functions as an autonomous act of dismissing the aggressive sting.

TENTH, SPEAKING ERRORS ARE INEVITABLE AND GOOD BECAUSE THEY PROVIDE A
LOCUS FOR CONSTANT CRITICISM, SOMETHING THE NEG BY ITSELF PRECLUDES

Alcoff 92
[Linda, Prof. of Feminist Studies at the University of Syracuse, The Problem of Speaking for Others,
Cultural Critique, Winter 91-2, 22//uwyo]

But surely it is both morally and politically objectionable to structure ones actions around the desire to avoid
criticism, especially if this outweighs other questions of effectivity. In some cases perhaps the motivation is not so much to avoid criticism as to avoid errors, and the
person believes that the only way to avoid errors is to avoid all speaking for others. However, errors are unavoidable in the theoretical inquiry

as well as political struggle, and moreover they often make contributions. The desire to find an absolute
means to avoid making errors comes perhaps not from a desire to advance collective goals but a desire
for personal mastery, to establish a privileged discursive posotion wherein one cannot be undermined or challenged and thus is
master of the situation. From such a position ones own location and positionality would not require constant

interrogation and critial reflection; one would not hae to constantly engage in this emotionally
troublesome endeavor and would be immune from the interrogaton of others. Such a desire of rmastery
and immunity must be resisted.

Page 1414 of 1481


Page 1415 of 1481
Terror Talk Answers: 2AC (5/5)
ELEVENTH, COUNTERSPEECH SOLVES BETTER THAN CENSORSHIP

Calleros 95
[Charles R., Prof. of Law @ ASU, Paternalism, Counterspeech, and Campus Hate-Speech Codes: A Reply
to Delgado and Yun, 27 Ariz. St. L.J. 1249, Winter, LN//uwyo-ajl]

The purveyor of hate speech indeed had made a point about the power of speech, just not the one he had
intended. He had welcomed disciplinary sanctions as a form of empowerment, but the Stanford
community was alert enough to catch his verbal hardball and throw it back with ten times the force.

Thus, the argument that counterspeech is preferable to state suppression of offensive speech is stronger
and more fully supported by experience than is conceded by Delgado and Yun. In both of the cases
described above, the targets of hateful speech were supported by a community united against bigotry.
The community avoided splitting into factions because the universities eliminated the issue of censorship
by quickly announcing that the hateful speakers were protected from disciplinary retaliation. Indeed, the
counterspeech against the bigotry was so powerful in each case that it underscored the need for top
administrators to develop standards for, and some limitations on, their participation in such partisan
speech. n72

Of course, the community action in these cases was effective and empowering precisely because a
community against bigotry existed. At A.S.U. and Stanford, as at most universities, the overwhelming
majority of students, faculty, and staff are persons of tolerance and good will who deplore at least the
clearest forms of bigotry and are ready to speak out [*1262] against intolerance when it is isolated as an
issue rather than diluted in muddied waters along with concerns of censorship. Just as the nonviolent
demonstrations of Martin Luther King, Jr., depended partly for their success on the consciences of the
national and international audiences monitoring the fire hoses and attack dogs on their television sets and
in the print media, n73 the empowerment of the targets of hateful speech rests partly in the hands of
members of the campus community who sympathize with them. One can hope that the counterspeech
and educational measures used with success at A.S.U. and Stanford stand a good chance of preserving an
atmosphere of civility in intellectual inquiry at any campus community in which compassionate, open
minds predominate.

TWELFTH, THE CRITICISM ASSUMES STABLE SPEECH ACTS, PREVENTING US FROM


TAKING BACK HURTFUL WORDS AND COLLAPSING INTO A JURIDICAL MODEL OF
STABLE SUBJECTIVITY THAT KILLS ACTIVISM

Judith Butler, Professor of Rhetoric and Comparative Literature, UC Berkeley, Performativity and Performance, Ed.

Parker and Sedgwick, 1995, p. 204

That words wound seems incontestably true, and that hateful, racist, misogynist, homophobic speech
should be vehemently countered seems incontrovertibly right. But does understanding from where
speech derives its power to wound alter our conception of what it might mean to counter that wounding

Page 1416 of 1481


power? Do we accept the notion that injurious speech is attributable to a singular subject and act? If we
accept such a juridical constraint on thought - the grammatical requirements of accountability - as a point
of departure, what is lost from the political analysis of injury when the discourse of politics becomes fully
reduced to juridical requirements?? Indeed, when political discourse is collapsed into juridical discourse,
the meaning of political opposition runs the risk of being reduced to the act of prosecution. How is the
analysis of the discursive historicity of power unwittingly restricted when the subject is presumed as the
point of departure for such an analysis? A clearly theological construction, the postulation of the subject
as the causal origin of the performative act is understood to generate that which it names; indeed, this
divinely empowered subject is one for whom the name itself is generative.

Page 1417 of 1481


Page 1418 of 1481
Terror Discourse Good: 1AR
MORAL CONDEMNATIONS OF TERRORISM IS APPROPRIATE

Robert Phillips, Director of the Program for War and Ethics, University of Connecticut, TERRORISM,

PROTEST AND POWER, Martin Warner and Roger Crisp, eds., 1990, p. 68-9.

Moral denunciations of terrorism are appropriate and mandatory. Terrorist acts are profoundly immoral
and, in addition, tend not to be the short cut which their practitioners advertise. One has only to look at
the areas of the world where terror has held sway to see that violence is typically prolonged, sometimes
indefinitely. The reason for this is not difficult to discern. Each side comes to perceive the other as
'criminal' and thus as beyond the pale of civilized negotiation.

Page 1419 of 1481


Page 1420 of 1481
Counterspeech Solves: 1AR
COUNTERSPEECH SOLVES BEST

Strossen 2001
[Nadine, Pres. ACLU & Prof. Law @ NYU, Incitement to Hatred: Should There Be a Limit? 25 S. Ill. U.L.J.
243, Winter, LN//uwyo-ajl]

The viewpoint-neutrality principle reflects the philosophy, first stated in pathbreaking opinions by former
United States Supreme Court Justices Oliver Wendell Holmes and Louis Brandeis, that the appropriate
response to speech with which one disagrees in a free society is not censorship but counterspeech-more
speech, not less. Persuasion, not coercion, is the solution. n38 Accordingly, the appropriate response to
hate speech is not to censor it, but to answer it. Recall, as I discussed earlier, that this is the strategy that
the Anti-Defamation League has been pursuing so effectively in response to Internet hate speech. [*255]

This counterspeech strategy is better than censorship not only in principle, butalso from a practical
perspective. That is because of the potentially empoweringexperience of responding to hate speech with
counterspeech. I say "potentially,"since I realize that the pain, anger and other negative emotions
provoked by being the target of hate speech could well have an incapacitating effect on some targeted
individuals, preventing them from engaging in counterspeech. Even in such a situation, though, other
members of the community who are outraged by the hate speech could engage in counterspeech, and
that is likely to have a more positive impact than a censorial response. Furthermore, once other
community members denounce the hate speech, it should be easier for the target to join them in doing
so.

Page 1421 of 1481


Page 1422 of 1481
**Threat Construction**

Page 1423 of 1481


Threat Construction Answers:
2AC (1/3)
FIRST, NO LINK PLAN DOESNT OVERTLY IDENTIFY ANY NATION AS A THREAT IT
ONLY ENDS EXECUTIVE DETAINMENT, MEANING THERES NO RISK OF
CONSTRUCTING A THREAT

SECOND, THE ENEMY IMAGE DETERS INSTEAD OF PROVOKING ATTACK,


CIRCUMVENTING ANY RISK OF AN IMPACT

Hermann 95
[Richard, Prof. Poli Sci @ Ohio State, International Organization, Summer, 431//uwyo]

The logic behind the association of particular strategies with particular images is grounded in the
dimensions and attributes of each image. For example, if an actor perceives a target as an enemy, it
perceives the target as a powerful, aggressive threatening actor that constantly probes for weakness in its
efforts to expand its influence in the international system. Since the perceivers primary interests are
threatened by the perceived revisionist motivation of the target, the perceiver will seek to bridle the
targets expansionist designs. It will not cooperate with the target in any substantial way since it perceives
that the target would take advantage of cooperative initiatives. Furthermore, it will not directly attack the
target because it perceives it as having a capability base similar to its own. This suggests a cautious,
resisting strategy to counter the probes of the target.

THIRD, MULTILAT SOLVES BY ALLOWING US TO ADDRESS PROBLEMS WITH


INTERNATIONAL COOPERATION, SOLVING BAD FORMS OF VIOLENCE. CROSS-
APPLY NYE

FOURTH, HISTORY IS ON OUR SIDE. WE CONSTRUCTED THE USSR AS AN ENEMY


FOR OVER HALF A CENTURY, BUT DETERRENCE AND SELF-INTEREST PREVENTED
CONFLICT

FIFTH, WAR AND VIOLENCE ARE ENDEMIC TO IR POLITICS, MOVING AWAY WILL
INEVITABLY RESULT IN GREAT POWER WARS

Page 1424 of 1481


MEARSHEIMER 2001
[John, Co-Director of IR Policy at University of Chicago and Former research fellow at the Brookings
institute, The Tragedy of Great Power Politics, pg xi-xii. )

The twentieth century was a period of great international violence.In World War I (1914-18), roughly nine million
people died on European battlefields. About fifty million people were killed duringWorld War 11(1939-45), well over half of them civilians. Soon after the end of
World War II, the Cold War engulfed the globe. During this con-frontation, the Soviet Union and its Warsaw Pact allies never directly fought the United States
and its North Atlantic Treaty Organization allies,but many millions died in proxy wars in Korea, Vietnam, Afghanistan, Nicaragua, Angola, El Salvador, and
elsewhere. Millions also died in the century's lesser, yet still fierce, wars, including the Russo-Japanese con-flicts of 1904-5 and 1939, the Allied intervention in
the Russian Civil War from 1918 to 1920, the Russo-Polish War of 1920-21, the various Arab-Israeli wars, and the han-Iraq War of 1980-88. This cycle of
Hopes for peace will probably not be realized, because the great
violence will continue far into the new millennium.

powers that shape the international system fear each other and compete for power as a result.
Indeed, their ultimate aim is to gain a position of dominant power over others, because having
dominant power is the best means to ensure one's own survival. Strength ensures safety, and
the greatest strength is the greatest insurance of safety. States facing this incentive are fated to clash as each competes
for advantage over the others. This is a tragic situation, but there is no escaping it unless the states that make up the system

agree to form a world government. Such a vast transformation is hardly a realistic prospect, however, so conflict and war are bound to

continue as large and enduring features of world politics.

Page 1425 of 1481


Page 1426 of 1481
Threat Construction Answers:
2AC (2/3)
SIXTH, PERM DO PLAN AND THE ALTERNATIVE REPRESENTATIONAL VIOLENCE
DOESNT PRECLUDE THE NEED FOR CONCRETE ACTION

Richard Rorty, Professor of Humanities, University of Virginia, Truth, Politics, and Postmodernism, Spinoza Lectures,

1997, p. 51-2

This distinction between the theoretical and the practical point of view is often drawn by Derrida, another writer who enjoys demonstrating that something very important
meaning, for example, or justice, or friendship is both necessary and impossible. When asked about the implications of these paradoxical fact, Derrida usually replies that

the paradox doesn't matter when it comes to practice. More generally, a lot of the writers who are labeled `post-modernist; and
who talk a lot about impossibility, turn out to be good experimentalist social democrats when it comes to actual

political activity. I suspect, for example, that Gray, Zizek, Derrida and I, if we found ourselves citizens of the same country, would all be voting for the same
candidates, and supporting the same reforms. Post-modernist philosophers have gotten a bad name because of their paradox-mongering habits, and their constant use of
terms like `impossible; `self-contradictory' and `unrepresentable'. They have helped create a cult of inscrutability, one which defines itself by opposition to the Enlightenment
search for transparency - and more generally, to the `metaphysics of presence; the idea that intellectual progress aims at getting things clearly illuminated, sharply delimited,
I am all for getting rid of the metaphysics of presence, but I think that the rhetoric of impossibility and
wholly visible.

unrepresentability is counterproductive overdramatization. It is one thing to say that we need to get rid of the metaphor of things being
accurately represented, once and for all, as a result of being bathed in the light of reason. This metaphor has created a lot of headaches for philosophers, and we would be
better off without it. But that does not show that we are suddenly surrounded by unrepresentables; it just shows that `more accurate representation' was never a fruitful way to
Even if we agree that we shall never have what Derrida calls "a full presence beyond the reach
describe intellectual progress.

of play"; our sense of the possibilities open to humanity will not have changed. We have learned nothing about the limits of
human hope from metaphysics, or from the philosophy of history, or from psychoanalysis. All that we have learned from `post-modern' philosophy is that we may need a

We have been given no reason to abandon


different gloss on the notion of `progress' than the rationalistic gloss which the Enlightenment offered.

the belief that a lot of progress has been made by carrying out the Enlightenment's political program. Since
Darwin we have come to suspect that whether such progress is made will be largely a matter of luck. But we have been given no reason to stop hoping to get lucky.

SEVENTH, OUR ENGAGEMENT IN SCENARISM OVERCOMES THE TRAP OF THREAT


CONSTRUCTION WE AVOID THE ILLUSION OF CERTAINTY IN IDENTIFYING FUTURE
THREATS, AND INSTEAD USE SCENARIOS TO CRITICALLY EXAMINE POSSIBLE
OUTCOMES TO DETERMINE THE BEST COURSE OF ACTION. THE OTHER TEAMS
REFUSAL TO ENGAGE IN THIS PROCESS ONLY REINFORCES THE PRACTICE OF
SHALLOW RISK ANALYSIS.

P.H. Liotta, Jerome E. Levy Cahir, Economic Geography and National Security, Naval War College and
Timothy E. Somes, Professor Emeritus, The Art of Perceiving Scenarios and the Future, NAVAL WAR
COLLEGE REVIEW v. 56 n. 4, Autumn 2003, http://www.nwc.navy.mil/press/Review/2003/Autumn/cy1-
a03.htm.

, the scenarios we are talking about are not the limited threat-based planning scenarios common in
Finally

defense planning. Threat-based scenarios, generally based on assessments of current or postulated threats or enemy capabilities, determine

Page 1427 of 1481


only the amount and types of force needed to defeat an adversary. (Similarly, capabilities-based planning seeks to avoid the
perceived limits of threat-derived scenarios.)6 In contrast, the scenarios we want to consider should look well beyond

current evaluations of threats. If future military force capabilities are derived from the kind of scenarios
we are discussing, they must encompass the full range of possibilities, with a commensurate weighing of
benefits, costs, and risks. Accomplishing this is a difficult but essential challenge, if decision makers are to
come to any informed, perceptive conclusions for the future. In Wacks words, Scenarios serve two purposes. The first is protective
anticipating and understanding risk. The second is entrepreneurialdiscovering strategic options of which one was previously unaware.7 Often, and probably naturally,

decision makers prefer the illusion of certainty to understanding risk and realities. But the scenario
builder and analyst should strive to shatter the decision makers confidence in his or her ability to look
ahead with certainty at the future. Scenarios should allow a decision maker to say, I am prepared for
whatever happens, because we have thought through complex choices with a knowledgeable sense of
risk and reward.8

Page 1428 of 1481


Page 1429 of 1481
Threat Construciton Answers:
2AC (3/3)
EIGHTH, NO IMPACT - RHETORIC DOESNT SHAPE REALITY

Fram-Cohen 85
[Michelle, Reality, Language, Translation: What Makes Translation Possible? American Translators Association Conference,
enlightenment.supersaturated.com/essays/text/michelleframcohen//possibilityoftranslation.html, 9-24-06//uwyo-ajl]

Nida did not provide the philosophical basis of the view that the external world is the common source of all languages. Such a basis can be found in the philosophy of

reality is independent of
Objectivism, originated by Ayn Rand. Objectivism, as its name implies, upholds the objectivity of reality. This means that

consciousness, consciousness being the means of perceiving ?reality, not of creating it. Rand defines language as "a
code of visual-auditory symbols that denote concepts." (15) These symbols are the written or spoken words of any language. Concepts are defined as the "mental integration of
two or more units possessing the same distinguishing characteristic(s), with their particular measurements omitted." (16) This means that concepts are abstractions of units

Since
perceived in reality. Since words denote concepts, words are the symbols of such abstractions; words are the means of representing concepts in a language.

reality provides the data from which we abstract and form concepts, reality is the source of all words--and of
all languages. The very existence of translation demonstrates this fact. If there was no objective reality, there could be no

similar concepts expressed in different verbal symbols. There could be no similarity between the content of different languages, and so, no
translation.

Translation is the transfer of conceptual knowledge from one language into another. It is the transfer of one set of symbols denoting concepts into another set of symbols
denoting the same concepts. This process is possible because concepts have specific referents in reality. Even if a certain word
and the concept it designates exist in one language but not in another, the referent this word and concept stand for nevertheless exists in reality, and can be referred to in
translation by a descriptive phrase or neologism. Language is a means describing reality, and as such can and should expand to include newly discovered or innovated objects
in reality. The revival of the ancient Hebrew language in the late 19th Century demonstrated the dependence of language on outward reality. Those who wanted to use Hebrew
had to innovate an enormous number of words in order to describe the new objects that did not confront the ancient Hebrew speakers. On the other hand, those objects that
existed 2000 years ago could be referred to by the same words. Ancient Hebrew could not by itself provide a sufficient image of modern reality for modern users.

NINTH, PREFER OUR EV THEIR K IS UTOPIAN THEORY WITH ZERO GROUNDING IN


PRAXIS OR CONSEQUENCE

Mearsheimer 95
[John, Prof. Poli Sci @ Chicago, International Security, Winter, 38//uwyo]

, critical theory per se has little to say about the future shape of international politics. In fact,
Very significantly, however

Critical awareness of potentiality for change


critical theory emphasizes that, It is impossible to predict the future. Robert Cox explains this point:

must be distinguished from utopian planning, i.e. the laying out of the design of a future society that is to be the end goal of change. Critical
understanding focuses on the process of change rather than on its ends; it concentrates on the possibilities of launching a social
movement rather than on what that movement might achieve. Nevertheless, international relations scholars who use critical theory to

challenge and subvert realism certainly expect to create a more harmonious and peaceful international system.
But the theory itself says little about either the desirability or feasibility of achieving that particular end.

Page 1430 of 1481


Page 1431 of 1481
#2 Threat Rhetoric Deters War:
1AR
NEGLECTING SECURITY AND DETERRENCE CAUSES WAR

Doran 99
[Charles, Prof. IR @ John Hopkins, Survival, Summer, 148-9//uwyo]

And by neglecting the underlying problem of security, the probability of war perversely increases: as
governments fail to provide the kind of defence and security necessary to maintain deterrence, one opens
up the possibility of new challenges. In this regard it is worth recalling one of Clauswitzs most important
insights: A conqueror is always a lover of peace. He would like to make his entry into our state
unopposed. That is the underlying dilemma when one argues that a major war is not likely to occur and,
as a consequence, on need not necessarily be so concerned about providing the defences that underlie
security itself. History shows that surprise threats emerge and rapidly destabilising efforts are made to try
to provide that missing defence, and all of this contributes to the spiral of uncertainty that leads in the
end to war.

WORLD WAR II PROVES ITS BETTER TO BELIEVE IN THREATS THAN IGNORE THEM
AND RISK WAR

Thompson 85
[Kenneth, Prof. poli sci @ Virginia, Moralism and morality in politics and diplomacy, 1985, 130//uwyo]

We need also to recall that the failures leading up to World War II were not alone failures of military
preparation and military action. They were also political failures, as Arnold Wolfers and Hans J.
Morgenthau pointed out, of the allies and of France and Britain in particular, to concert their foreign
policies and present any kind of united, consistent and coherent opposition which carried weight with
Hitler, rather than tempting him with the disunity of the West.

Page 1432 of 1481


Page 1433 of 1481
#5 Realism Inevitable: 1AR
POWER IS ZERO SUM THE ALTERNATIVE ONLY SHIFTS POWER ELSEWHERE

John Mearsheimer, Professor at University of Chicago, 2001 (The Tragedy of Great Power Politics p. 34)

Consequently, states pay close attention to how power is distributed among them, and they make a special effort to maximize
their share of world power. Specifically, they look for opportunities to alter the balance of power by acquiring
additional increments of power at the expense of potential rivals. States employ a variety of meanseconomic,
diplomatic, and militaryto shift the balance of power in their favor, even if doing so makes other states suspicious or even
hostile. Because one states gain in power is another states loss, great powers tend to have a zero-sum
mentality when dealing with each other. The trick, of course, is to be the winner in this competition and to dominate
the other states in the system. Thus, the claim that states maximize relative power is tantamount to arguing that states are
disposed to think offensively toward other states, even though their ultimate motive is simply to survive. In short, great
powers have aggressive intentions.

Page 1434 of 1481


Page 1435 of 1481
#7 Scenario Analysis Good: 1AR
(1/3)
OUR SCENARIOS ARE NOT FALSE CONSTRUCTION OF THREATS BUT RATHER AN
EXAMINATION OF CRITICAL UNCERTAINTIES WHICH IS VITAL IN MAKING
RESPONSIBLE POLICY DECISIONS.

P.H. Liotta, Jerome E. Levy Cahir, Economic Geography and National Security, Naval War College and
Timothy E. Somes, Professor Emeritus, The Art of Perceiving Scenarios and the Future, NAVAL WAR
COLLEGE REVIEW v. 56 n. 4, Autumn 2003, http://www.nwc.navy.mil/press/Review/2003/Autumn/cy1-
a03.htm.

Scenarios structure the future into both predetermined and uncertain elements. Any good scenario
reading explores and seeks to comprehend these elements. Often, events that are already in the
pipeline, such as demographic shifts or energy dependency, bring consequences that have yet to unfold,
and these consequences may have immense impact.

Schwartz provides one example to illustrate the shortcomings of conventional forecasting and trend
analysis:

[Consider] the U.S. birthrate. In the early 1970s it hovered around 3 million births per year; forecasters
at the U.S. Census Bureau projected that this trend would continue forever. Schools, which had been
rushed into construction during the baby boom of the fifties and early sixties, were now closed down and
sold. Policymakers did not consider that the birthrate might rise again suddenly. But a scenario might
have considered the likelihood that original baby boom children, reaching their late thirties, would
suddenly have children of their own. In 1979, the U.S. birthrate began to rise . . . in 1990 [it was] almost
back to the 4 million of the fifties. Demographers also failed to anticipate that immigration would
accelerate. To keep up with demand, the state of California (which had been closing schools in the late
1970s) . . . [had to] build a classroom every day for the next seven years.16

Assessing and developing the two fundamentalspredetermined elements and critical uncertainties
when building a scenario may be among the more valuable aspects of this process, or at least on what
strategic planners spend much of their time. Yet experience tells us that many of our war college
students, initially introduced to this art of scenario reading, find of particular value the process of
deciding what are predetermined elements, as opposed to critical uncertainties. When we examine
geostrategic regions, for example, we may strive to recognize which elements of each region are
predetermined, such as geography, and which may be critical but uncertain identities, such as how the
predetermined importance of geography can be made less important, or even irrelevant, by the
uncertainty and influence of technology.

Page 1436 of 1481


Page 1437 of 1481
#7 Scenario Analysis Good: 1AR
(2/3)
ENGAGING IN SCENARIOS ALLOWS US TO OVERCOME UNDECIDABILITY BY
LEADING THE DECISION MAKER TO CHANGE THEIR FUNDAMENTAL PERCEPTIONS
OF REALITY.

P.H. Liotta, Jerome E. Levy Cahir, Economic Geography and National Security, Naval War College and
Timothy E. Somes, Professor Emeritus, The Art of Perceiving Scenarios and the Future, NAVAL WAR
COLLEGE REVIEW v. 56 n. 4, Autumn 2003, http://www.nwc.navy.mil/press/Review/2003/Autumn/cy1-
a03.htm.

The challenge for strategic planners is to help decision makers understand what the future security
environment might look like, to affect their perceptions, in essence, to help them reperceive. Wack,
who gained some fame as a strategic planner during the oil crises of the 1970s with his ability to get the
senior executives in Shell Oil to understand what might happen in the energy business, wrote in the
Harvard Business Review some years later:

Scenarios deal with two worlds: the world of facts and the world of perceptions. They explore the facts
but they aim at perceptions inside the heads of decision makers. Their purpose is to gather and transform
information of strategic significance into fresh perceptions. This transformation process is not trivial
more often than not it does not happen. When it works, it is a creative experience that generates a
heartfelt Aha! from you . . . [decision makers] and leads to strategic insights beyond the minds
previous reach.3

In short, to think and act effectively in an uncertain world, people need to learn to reperceiveto
question their assumptions and their understanding about the way the world works. By questioning those
assumptions and rethinking the correct way to operate under uncertainty, we often see the world more
clearly than we otherwise would. Wack summarized his goals as a strategic planner and developer of
scenarios by stating:

I have found that getting to that [decision makers] Aha! is the real challenge of scenario analysis. It
does not simply leap at you when youve been presented all the possible alternatives . . . . It happens
when your message reaches the microcosms of decision makers, obliges them to question their
assumptions about how their . . . world works, and leads them to change and reorganize their inner
models of reality.

Page 1438 of 1481


Page 1439 of 1481
#7 Scenario Analysis Good: 1AR
(3/3)
SCENARIOS LIKE THE 1AC ESCAPE THE TRAP OF UNDECIDABILITY BY LEADING TO A
CONSTANT RE-PERCEPTION OF THE FUTURE THROUGH EXAMINATION OF CRITICAL
UNCERTAINTIES. FAILURE TO EFFECTIVELY ENGAGE IN SCENARIOS WILL LEAD TO
FUTURE DISASTER.

P.H. Liotta, Jerome E. Levy Cahir, Economic Geography and National Security, Naval War College and
Timothy E. Somes, Professor Emeritus, The Art of Perceiving Scenarios and the Future, NAVAL WAR
COLLEGE REVIEW v. 56 n. 4, Autumn 2003, http://www.nwc.navy.mil/press/Review/2003/Autumn/cy1-
a03.htm.

The relationship between driving forces, predetermined elements, and critical uncertainties is complex,
but important to understand, as we learn to read the flow of what is occurring in useful scenarios. As
Schwartz points out, I sometimes think of the relationship between predetermined elements and critical
uncertainties as a choreographed dance. You cannot experience the dance just by knowing the sequence
of steps. Each dancer will interpret them differently, and add his or her unpredictable decisions.19 In
terms of national security and defense, one cannot anticipate the nature of a war merely by looking at the
military orders of battle, even if you know your plans and those of the enemy. In the same fashion, by
developing scenarios oriented to a more distant future, the interrelationship between that which is
predetermined and that which is uncertain may be equally open to interpretation and changing factors.
Pierre Wack offers several thoughts with respect to the use of scenarios as tools:

I have found that scenarios can effectively organize a variety of seemingly unrelated economic,
technological, competitive, political, and societal information and translate it into a framework for
judgmentin a way that no model could do. . . . Decision scenarios describe different worlds, not just
different outcomes in the same world. . . . You can test the value of scenarios by asking two questions: (1)
What do they leave out? In five to ten years . . . [decision makers] must not be able to say that the
scenarios did not warn of important events that subsequently happened. (2) Do they lead to action? If
scenarios do not push managers to do something other than that indicated by past experience, they are
nothing more than interesting speculations.20

We are experiencing a world of dynamic change where even the most mind-numbing, dramatic events
do not impress us for long. Yet any good strategist and planner must be able to help the nations leaders
see more clearly the different futures that may occur. To operate in an uncertain world, we need to
reperceiveto question our assumptions about how the world works, so that we see the world more
clearly. The purpose of this is to help us make better decisions about the future.

Perhaps one way to think about this is to obvert George Santayanas famous saying about learning from
history by changing our perception of things that are yet to come, by suggesting that those who do not
learn from the future are destined to make mistakes in it. To be able to understand that future, we have
to have a mental map flexible enough to consider plausible alternatives and possibilities we might not
otherwise consider.

Page 1440 of 1481


#9 Prefer Our Args: 1AR

THE ALT CANT SOLVE ZERO EXPLANATION OF HOW REALISM IS UNSEATED AS


THE PRIMARY MODE OF POLITICAL DISCOURSE

Mearsheimer 95
[John, Prof. Poli Sci @ Chicago, International Security, Winter, 91-2//uwyo]

The most revealing aspect of Wendts discussion is tha the did not respond to the two main charges
leveled against critical theory in False Promise. The first problem with critical theory is that although the
theory is deeply concerned with radically changing state behavior, it says little about how change comes
about. The theory does not tell us why particular discourses becoem dominant and others fall by the
wayside. Specifically, Wendt does not explain why realism has been the dominant discourse in world
politics for well over a thousand years, although I explicitly raised this question in False Promise (p. 42).
Moreover, he sheds no light on why the time is ripe for unseating realism, nor on why realism is likely to
be replaced by a more peaceful, communitarian discourse, although I explicitly raised both questions.

CRITICAL APPROACHES LACK STRUCTURAL EXPLANATIONS FOR INTERNATIONAL


RELATIONS

Walt 91
[Stephen, Poli Sci @ Chicago, International Studies Quarterly, 1991, 223//uwyo]

In short, security studies must steer between the Scylla of political opportunism and the Charybdis of
academic irrelevance. What does this mean in practice? Among other things, it means that security
studies should remain wary of the counterproductive tangents that have seduced other areas of
international studies, most notably the postmodern approach to international affairs (Ashley, 1984, Der
Derian and Shapiro, 1989; Lapid, 1989). Contrary to their proponents claims, post-modern approaches
have yet to demonstrate much value for comprehending world politics; to date, these works are mostly
criticism and not much theory. As Robert Keohane has noted, until these writers have delineated . . . a
research program and shown . . . that it can illuminate important issues in world politics, they will remain
on the margins of the field (Keohane, 1988:392). In particular, issues of war and peace are too important
for the filed to be diverted into a prolix and self-indulgent discourse that is divorced from the real world.

Page 1441 of 1481


Dillon Supports Acting Against Terrorism

DILLON HAS ADVOCATED MORE EFFICIENT STATE RESPONSES TO TERROR


THREATS.

David Smith, Economics Editor of the Times of London, THE EDGE, March 2003,
http://www.esrc.ac.uk/ESRCContent/downloaddocs/EdgeMarch.pdf.

In the wake of 9/11, the ESRC has allocated more than 600,000 for three Research projects examining
the quality of Britains response to, and preparedness for, terrorist incidents. One of those involved is the
University of Lancasters Professor of Politics, Michael Dillon. What we lack, he says, is a single relevant
department of state like Americas Homeland Security. We seem to take a very British view, its all about
committees. (In Britain there are close to 50 agencies that would be involved in dealing with a major
terrorist attack.) Professor Dillon is also critical of the Governments slowness to modernise the legislation
for Civil Defence, pointing out that it dates back to the Cold War in 1948 and desperately needs updating.
So what needs to be done? Professor Dillon lists three immediate priorities: Firstly, bring in legislation
sooner so local bodies know their responsibilities; secondly, improve their financial resources so they can
cope with a chemical or biological attack, and thirdly, provide more obvious direct political leadership.

MORE EVIDENCE:

David Smith, Economics Editor of the Times of London, THE EDGE, March 2003,
http://www.esrc.ac.uk/ESRCContent/downloaddocs/EdgeMarch.pdf.

Buttressed by ESRC research projects on terrorism, commissioned in the wake of September 11, Gardner
also calls into question Britains state of readiness. One researcher, Professor Michael Dillon of the
University of Lancaster, suggests the government machine is locked into the 50-year old mentality of
dealing with the ColdWar, rather than the new and more diverse risks from terrorism. A government that
has been criticized for too much centralization appears unwilling to centralize enough when it comes to its
civil contingency strategy. Terrorism, by its nature, succeeds partly by action but mainly by fear.

Page 1442 of 1481


Page 1443 of 1481
**Zizek: Psychopolitics**

Page 1444 of 1481


Lacan Destroys Social Change
(1/2)
ALTERNATIVE DOESNT SOLVE CASE RECOGNIZING THE LACK CANT ACCESS
THE REAL-WORLD POLICY IMPACTS INEVITABLE IN THE STATUS QUO

Robinson, PhD @ U of Nottingham, 2K5 (Andy, The Political Theory of Constitutive Lack: A
Critique, Theory and Event 8.1, The Johns Hopkins University Press, Project Muse)

The function of the iekian "Act" is to dissolve the self, producing a historical event. "After the
revolution", however, everything stays much the same. For all its radical pretensions, iek's politics can be summed up in his
attitude to neo-liberalism: 'If it works, why not try a dose of it?'31. The phenomena which are denounced in Lacanian theory are
invariably readmitted in its "small print", and this leads to a theory which renounces both effectiveness
and political radicalism. It is in this pragmatism that the ambiguity of Lacanian political theory resides, for,
while on a theoretical level it is based on an almost sectarian "radicalism", denouncing everything that
exists for its complicity in illusions and guilt for the present, its "alternative" is little different from what it
condemns (the assumption apparently being that the "symbolic" change in the psychological coordinates
of attachments in reality is directly effective, a claim assumed wrongly to follow from the claim that
social reality is constructed discursively). Just like in the process of psychoanalytic cure, nothing actually changes on the
level of specific characteristics. The only change is in how one relates to the characteristics, a process iek terms 'dotting the "i's"' in reality,
recognizing and thereby installing necessity32. All that changes, in other words, is the interpretation: as long as they are
reconceived as expressions of constitutive lack, the old politics are acceptable. Thus, iek claims that de Gaulle's "Act"
succeeded by allowing him 'effectively to realize the necessary pragmatic measures' which others pursued unsuccessfully33.

THE CRITICISM IS PREMISED ON ORIGINARY LACK, DESTROYING ANY HOPE OF


POLITICAL TRANSFORMATION

Robinson, PhD @ U of Nottingham, 2K5 (Andy, The Political Theory of Constitutive Lack: A
Critique, Theory and Event 8.1, The Johns Hopkins University Press, Project Muse)

.
The idea of "constitutive lack" is supposed to entail a rejection of neutral and universal standpoints, and it is this rejection which constructs it as an "anti-essentialist" position

In practice, however, Lacanians restore the idea of a universal framework through the backdoor. Beneath
the idea that "there is no neutral universality" lurks a claim to know precisely such a "neutral universality"
and to claim a privileged position on this basis. A consistent belief in contingency and "anti-essentialism"
entails scepticism about the idea of constitutive lack. After all, how does one know that the appearance that 'experience' shows lack to be
constitutive reflects an underlying universality, as opposed to the contingent or even simulated effects of a particular discourse or episteme? Alongside its opponents, shouldn't
Lacanian theory also be haunted by its own fallibility and incompletion? There is a paradox in the idea of radical choice, for it is unclear whether Lacanians believe this should
be applied reflexively. Is the choice of Lacanian theory itself an ungrounded Decision? If so, the theory loses the universalist status it implicitly claims. If not, it would seem to
be the kind of structural theory it attacks. A complete structural theory would seem to assume an extra-contingent standpoint, even if the structure includes a reference to
constitutive lack. Such a theory would seem to be a radical negation of the incompletion of "I don't know".

# The myth of constitutive lack, like all myths, has a closing role: it limits what can be said through an "order not to
think". On the other hand, the idea that creativity is motivated by a stance that "I-don't-know" has an opening effect. As Callinicos puts it, 'what Badiou and iek calls the
"void" in a situation is rather the set of determinate possibilities it contains, including that of transformation'122. If there is no irreducible "Real" beneath each blockage or lack,
these can be overcome by creative action, as with the creative role of anomalies in paradigm-change in the sciences, and the creative role of "psychotic" philosophies such as

those of Deleuze and Nietzsche . The imperative in Lacanian theory is to "accept" lack, whereas the logic of a non-mythical idea of
contingency is to use opportunities for openness as a basis for creativity.

Page 1445 of 1481


# Furthermore, Lacanian theories involve a strong commitment to slave morality, as exemplified by Laclau's insistence that every
chain of equivalence involve a unity against an external threat123, Norval's advocacy of the use of "apartheid" as a bogeyman in South African politics124 and Mouffe's

demand for submission to rules125, and also in Zizek's "revolutionary" insistence on the need for masochistic self-
degradation, 'subjective destitution' and identification with a Master and a Cause126, not to mention his
directly reactive insistence that self-awareness amounts to awareness of the negative, of death and trauma, prior to any
active identification or articulation127. This is a reterritorializing "contingency" which fits closely with the operation of

capitalist ideology, where 'under conditions we recognize as desperate, we are told to alter ourselves', not
the conditions, because the self is conceived as a decisionist founder128. The alternative is a difference which is not reified into
a "positive" negativity. According to Deleuze, there are two models of contingency: the creative power of the poet, and the politician's denial of difference so as to prolong an
established order. It is for the latter that negation (lack) is primary, 'as if it were necessary to pass through the misfortunes of rift and division in order to be able to say yes'. For
the poet, on the other hand, difference is 'light, aerial and affirmative'. 'There is a false profundity in conflict, but underneath conflict, the play of differences', differences which
should be affirmed as positive and not overcoded by negativity

Page 1446 of 1481


Page 1447 of 1481
Lacan Destroys Social Change
(2/2)
LACANIAN PESSIMISM IS A SELF-FULFILLING PROPHECY THEIR HYPER-SKEPTICISM
LOCKS IN THE STATUS QUO, PREVENTING RADICAL SOCIAL CHANGE

Robinson, PhD @ U of Nottingham, 2K5 (Andy, The Political Theory of Constitutive Lack: A
Critique, Theory and Event 8.1, The Johns Hopkins University Press, Project Muse)

# There is more than an accidental relationship between the mythical operation of the concept of "constitutive lack" and Lacanians' conservative and pragmatist politics.
Myth is a way of reducing thought to the present: the isolated signs which are included in the mythical gesture are thereby attached to extra-historical abstractions. On an

analytical level , Lacanian theory can be very "radical", unscrupulously exposing the underlying relations and assumptions concealed beneath officially-sanctioned
discourse. This radicalism, however, never translates into political conclusions: as shown above, a radical rejection of anti-"crime" rhetoric
turns into an endorsement of punishment, and a radical critique of neo-liberalism turns into a pragmatist endorsement of structural adjustment. It is as if there is a magical
barrier between theory and politics which insulates the latter from the former. One should recall a remark once made by Wilhelm Reich: 'You plead for happiness in life, but

Lacanians have a "radical" theory oriented towards happiness, but politically, their
security means more to you'133.

primary concern is security. As long as they are engaged in politically ineffectual critique, Lacanians will
denounce and criticize the social system, but once it comes to practical problems, the "order not to think"
becomes operative.
# This "magic" barrier is the alibi function of myth. The short-circuit between specific instances and high-level abstractions is politically consequential. A present evil can be
denounced and overthrown if located in an analysis with a "middle level", but Lacanian theory tends in practice to add an "always" which prevents change. At the very most,
, Lacanian theory operates as
such change cannot affect the basic matrix posited by Lacanian theory, because this is assumed to operate above history. In this way

offers a little bit of theoretical radicalism to inoculate the system against the threat posed by a lot
an alibi: it

of politicized radicalism134. In Laclau and Mouffe's version, this takes the classic Barthesian form: "yes, liberal democracy involves violent exclusions, but
what is this compared to the desert of the real outside it?" The iekian version is more complex: "yes, there can be a revolution, but after the

revolution, one must return to the pragmatic tasks of the present". A good example is provided in one of iek's texts. The author
presents an excellent analysis of a Kafkaesque incident in the former Yugoslavia where the state gives a soldier a direct, compulsory order to take a voluntary oath - in other

words, attempts to compel consent. He then ruins the impact of this example by insisting that there is always such a moment of "forced choice", and that one should
not attempt to escape it lest one end up in psychosis or totalitarianism135. The political function of Lacanian theory is to preclude critique
by encoding the present as myth.

# There is a danger of a stultifying conservatism arising from within Lacanian political theory, echoing the
'terrifying conservatism' Deleuze suggests is active in any reduction of history to negativity136. The addition of an "always" to contemporary evils amounts to a "pessimism of
Stavrakakis, for instance, claims that attempts to find causes and
the will", or a "repressive reduction of thought to the present".

thereby to solve problems are always fantasmatic137, while iek states that an object which is perceived as blocking something does nothing
but materialize the already-operative constitutive lack138. While this does not strictly entail the necessity of a conservative attitude to the possibility of any specific reform, it

creates a danger of discursive slippage and hostility to "utopianism" which could have conservative
consequences. Even if Lacanians believe in surplus/contingent as well as constitutive lack, there are no standards for distinguishing the two. If one cannot tell which
social blockages result from constitutive lack and which are contingent, how can one know they are not all of the latter type? And even if constitutive lack exists, Lacanian
theory runs a risk of "misdiagnoses" which have a neophobe or even reactionary effect. To take an imagined example, a Lacanian living in France in 1788 would probably
conclude that democracy is a utopian fantasmatic ideal and would settle for a pragmatic reinterpretation of the ancin regime. Laclau and Mouffe's hostility to workers' councils

The pervasive negativity and cynicism of


and iek's insistence on the need for a state and a Party139 exemplify this neophobe tendency.

Lacanian theory offers little basis for constructive activity. Instead of radical transformation, one is left
with a pragmatics of "containment" which involves a conservative de-problematization of the worst
aspects of the status quo. The inactivity it counsels would make its claims a self-fulfilling prophecy by
acting as a barrier to transformative activity.

Page 1448 of 1481


Page 1449 of 1481
Lacan = Being Towards Death
THE ALTERNATIVES OBSESSION WITH NEGATIVITY LOCKS IN A BEING TOWARDS
DEATH THAT PREVENTS AFFIRMATION OF LIFE

Hallward 2001
[Peter, Nip/Tuck junky, Ethics: An Essay on the Understanding of Evil, Trans. Peter Hallward, New York:
Verso, 2001, xvi-xix//uwyo-ajl]

What distinguishes Badiou's Philosophical ethics from Lacan's own essentially' anti-philosophical
stance is the precise status allocated to the Real in this arrangement.15 Badiou emphasizes
the topological location of the Real, the Real as 'being, in a situation, in any given
symbolic field, the point of impasse, or the point of impossibility, which precisely allows
us to think the situation as a whole'.16 The Real is what seems empty or void from the
perspective of those who re-present and dominate the situation (i.e. from the perspective
assumed by the 'state of the situation'); rejected from any stable assignation of place, it is
thereby that which calls into question the prevailing regime of place and placement tout
court,17 Badiou's Real is always strictly situation-specific. But from a later Lacanian perspective,
the unsymbolizable Real often comes

to indicate general human finitude in its most elementary form, that is, death. As Lacan's most
forceful contemporary disciple puts it:

The whole of Lacan's effort is precisely focused on those limitexperiences in which the subject
finds himself confronted with the death drive at its purest, prior to its reversal into sublimation.
, " What 'Death' stands for at its most radical is not merely the passing of earthly life, but the
'night of the world', the self-withdrawal, the absolute contradiction of subjectivity, the severing
of its links with 'reality' ,18

A Lacanian ethics is designed to enable us to endure this severing without flinching, as the price
to be paid for a 'symbolic New Beginning, the emergence of the "New Harmony" sustained by a
newly emerged Master-Signifier'.

And it is at this point, Zizek continues, that 'Lacan parts company with Badiou' (154). For
confrontation with Lacan's Real here amounts to an experience of the abject,
inarticulable realm of the corpse as such - the 'undead' that is Oedipus after his mutilation, or
Antigone reduced to her 'living death' ,19 Zizek accepts this reduction without hesitation. Since
'modern subjectivity emerges when the subject perceives himself as "out of joint", as excluded
from the order of things, from the positive order of entities', so 'for that reason, the ontic
equivalent of the modern subject is inherently excremental. . . , There is no subjectivity without
the reduction of the subject's positive-substantial being to a disposable "piece of shit''' (157).
From Zizek's perspective, what thus 'remains beyond Badiou's reach ... is this domain "beyond
the Good", in which a human being encounters the death drive as the utmost limit of human
experience, and pays the price by undergoing a radical "subjective destitution", by being
reduced to an excremental remainder' (161).

Badiou would no doubt plead guilty as charged. For the great virtue of his system,
compared with Lacan's, is surely its separation of the merely ineffable, in-significant horror
of death from the generic 'destitution' or subtraction no doubt demanded by every
subjectification. It is Badiou's achievement to have subtracted the operation of truth from
any redemption of the abject, and to have made the distinction between living and unliving,

Page 1450 of 1481


between finite and infinite, a matter of absolute indifference. The 'Real' emergence of 'the
undead-indestructible object, [of] Life deprived of support in the symbolic order'20 is
incapable of provoking the slightest reaction either from within the domain of purely
multiple being-as-being on the one hand, or from the domain of an infinite, properly immortal
subjectivization on the other. From Badiou's perspective, death can never quality as an
event.

Page 1451 of 1481


Page 1452 of 1481
Lacan = Oppression
PSYCHOANALYSIS FORCES SEXUALITY INTO THE JURIDICAL MODEL OF THE FAMILY,
ALLOWING DISCIPLINE OF OTHERNESS

May 93
[Todd, Between Genealogy and Epistemology: Psychology, Politics, and Knowledge in
the Thought of Michel Foucault, Pennsylvania: The Pennsylvania State University
Press, 1993, 48-9//wfi-ajl]

The intrusion of the deployment of sexuality upon the traditional sexual arrangement of the
family and its bonds (kinship structures, codes of name and material transmission, and so on)
forced it to change its shape in order to accommodate this new, powerful set of practices.
Although a more complete analysis of the changes produced by this intrusion is given by
Foucault's colleague Jacques Donzelot (1979), Foucault points out that the deployment of
sexuality entails a set of power relationships that differ from those of the family alliance. For the
family alliance, power was realized on a juridical model of law, right, and possession. For the
new sexuality, however, power is a matter of local and dispersed tactics that run through such
nonfamilial domains as the school and the clinic.

What binds these local and dispersed tactics into a uniform sexuality is both their convergence
upon an expanded set of behaviors that are considered to be sexually relevant and the
development of the normality/abnormality axis to which all sexuality is now referred. At this
intersection psychoanalysis finds its place, structuring the new deployment of sexuality and
grafting it onto the traditional familial alliance. Indeed, the great genius of psychoanalysis lies in
this: that it was able to integrate the dispersed and mobile relations of sexuality into the rigid
codes of familial alliance without causing the breakdown of that alliance. Because
psychoanalysis presented the deployment of sexuality as a matter of juridical power, of law-
specifically the law that prohibits incest-the family, while becoming infused with sexual
strategies, was able to retain a sense of itself as the focal point of those strategies and as their
juridical protector. Thus sexuality, which threatened to burst the bonds of familial alliance by
introducing into it new matrices of power, is coordinated with the familial scheme. Children
have strange desires, it is true; nevertheless, in the end it is their parents they desire, just as
their parents desire one another and their own parents.

What was being constituted in this new sexuality, which psychoanalysis sponsored and to which
it owes such a great debt? Essentially, sexuality itself was being constituted, a modern sexuality
that is often heralded as the deepest truth or, better, as the essence of the modern soul. As the
soul was being created by disciplinary techniques, so its essence was being fashioned by sexual
techniques. And in both strategies psychological thinking, psychological discourse, and
psychotherapeutic intervention were drawing their nourishment and contributing their effects.
In both strategies, moreover, certain social figures were being created, figures that correspond
to contemporary networks of power and that invite contemporary modes of intervention--often
psychological intervention. In prisons, the figure of the delinquent emerged, a criminal not in
the mere authorship of a crime but in an existence that was itself deviant. The delinquent "is
not only the author of his acts. . . but is linked to his offense by a whole bundle of complex
threads (instincts, drives, tendencies, character)" (Foucault 1977a, p. 253). As such, the
delinquent requires observation, intervention, and rehabilitation--or, if these things fail, at least
surveillance and usefulness for intervening with other delinquents.

Page 1453 of 1481


Page 1454 of 1481
Page 1455 of 1481
A2 Stavrakakis: 2AC
ENDORSING THE AFFIRMATIVE AS AN ACT OF HOPE, NOT UTOPIA, IT IS POSSIBLE
TO HAVE POLITICS WITHOUT UTOPIA

Stavrakakis, Teaching Fellow in Government @ U of Essex, 99 (


Yannis, Lacan and the Political, P. 111-112)

What should not be neglected however in Ricoeurs standpoint is the centrality of the element of hope. No
doubt, a society without hope is a dead society. Yet, in reality, to eliminate the element of hope is a dead society. Yet, in reality, to
eliminate the element of hope from human life is not only undesirable but also impossible. As Jacques Derrida has put it:
There is no language without the performative dimension of the promise, the minute I open my mouth I am in the
promise. Even if I say I dont believe in truth or whatever, the minute I open my mouth there is a believe
me at work. Even when I lie, and perhaps especially when I lie, there is a believe me in play. And this I promise you that I am speaking the truth is a
messianic a priori, a promise which, even if it is not kept, even if one knows it cannot be kept, takes place and qua promise is messianic. (Derrida, 1996:82-3) In
addition, for Derrida, this element of hope is not necessarily utopian: I would not call this attitude utopian. The messianic experience of which
I spoke takes place here and now that is the fact of promising and speaking is an event that takes place here and now and is not utopian (ibid.). Is
it then
possible to retain this element of hope without incorporating it into a utopian vision? Can we have
passion in politics without holocausts? Furthermore, is it possible to have a politics of hope, a politics of
change without utopia? The experience of the democratic revolution permits a certain optimism .
Democratization is certainly a political project of hope. But democratic discourse is not (or should not be) based on the vision of a
utopian harmonious society. It is based on the recognition of the impossibility and the catastrophic consequences of such a dream. What
differentiates democracy from other political forms of society is legitimization of conflict and the refusal
to eliminate it through the establishment of an authoritarian harmonious order. Within this framework the antagonistic
diversity between different conceptions of good is not seen as something negative that should be eliminated, but as something to be valued and celebrated.
This requires the presence of institutions that establish a specific dynamic between consensus and dissent This is why democratic politics cannot aim towards
harmony and reconciliation. To believe that a final resolution of conflict is eventually possible, even when it is envisaged as asymptotic approaching to the
regulative idea of a free unconstrained communication, as in Habermas, is to put the pluralist democratic project at risk.(Mouffe, 1996b:8) 14

Page 1456 of 1481


Page 1457 of 1481
Marxism Answers: 2AC (1/2)

Page 1458 of 1481


Page 1459 of 1481
Marxism Answers: 2AC (2/2)

Page 1460 of 1481


Page 1461 of 1481
Brown Turns (1/2)
TURN: THEIR LINK AND REJECTION CLAIMS ARE PART OF A MELANCHOLIC POLITICS
THAT MOURNS THE LEFTS PASTTHIS INTELLECTUAL STRAIGHTJACKET TIES THEM
TO A POLITICS THAT TIME HAS PASSED AND IGNORES THE POLITICAL POSSIBILITIES
TO BE FOUND IN OUR IMPURE POLITICS

Wendy Brown, Professor, Political Science and Womens Studies, University of California-Berkeley, Resisting Left Melancholia, LOSS: THE POLITICS OF MOURNING,
ed. David L. Eng & David Kazanjian, 2002, p. 460-463.

Now our challenge would be to figure out who or what is this substitutive object. What do we hate that we might preserve the idealization of that romantic Left promise? What
do we [460] punish that we might save the old guarantees of the Left from our wrathful disappointment?

Two familiar answers emerge from recent quarrels and reproaches on the Left. The first is a set of social and political formations variously known as cultural politics or identity
politics. Here the conventional charge from one portion of the Left is that political movements rooted in cultural identity racial, sexual, ethnic, or gendered not only elide the
fundamental structure of modernity, capitalism, and its fundamental formation, class, but also fragment Left political energies and interests such that coalition building is
impossible. The second culprit also has various names poststructuralism, discourse analysis, postmodernism, trendy literary theory got up as political analysis. The murder
charges here are also familiar: postfoundational theories of the subject, truth, and social processes undermine the possibility of a theoretically coherent and factually true

Together or separately, these two phenomena are


account of the world and also challenge the putatively objective grounds of Left norms.

held responsible for the weak, fragmented, and disoriented character of the contemporary Left. This
much is old news. But if read through the prism of Left melancholia, the element of displacement in both sets of charges may appear more starkly, since we would
be forced to ask: What aspects of Left analysis or orthodoxy have wilted on the vine for its adherents, but are safeguarded from this recognition through the scornful attention
heaped on identity politics and poststructuralism? Indeed, what narcissistic identification with that orthodoxy is preserved in the lament over the loss of its hold on young Leftists
and the loss of its potency in the political field? What love for the promises and guarantees that a Left analysis once held is preserved, as responsibility for the tattered condition
of those promises and guarantees is distributed onto debased others? And do we here also see a certain thingness of the Left take shape, its reification as something that is,
the fantastical memory that it once was, at the very moment that it so clearly is not/ one? . . . . .

If
Now let us bring these speculations about a melancholic Left back to Stuart Halls more forthrightly political considerations about the troubles of the contemporary Left.

Hall understands our failure as a Left in the last quarter century as a failure within the Left to apprehend
this time, this is a failure that is only reiterated and not redressed by our complaints against those who
are succeeding (liberal centrists, neoconservatives, the Right) or by our complaints against one another
(antiracists, feminists, queer activists, postmodernists, or unreconstructed Marxists). In Halls understanding, this
failure is not simply the consequence of adherence to a particular [461] analytic orthodoxy the
determinism of capital, the primacy of class although it is certainly that. Rather, this failure results as
well from a particular intellectual straitjacket an insistence on a materialism that refuses the
importance of the subject and the subjective, the question of style, the problematic of language. And it is
the combination of these two that is deadly: Our sectarianism, Hall argues in the conclusion of The Hard Road to Renewal,
consists not only of a defensiveness toward the agendas fixed by now anachronistic political-economic
formations (those of the 1930s and 1945), but is also due to a certain notion of politics, inhabited not so
much as a theory, more as a habit of mind. We go on thinking a unilinear and irreversible political logic,
driven by some abstract entity we call the economic or capital, unfolding to its preordained end.
Whereas, as Thatcherism clearly shows, politics actually works more like the logic of language: you can
always put it another way if you try hard enough. 9

Certainly the course of capital shapes the conditions of possibility in politics, but politics itself is either
conducted ideologically, or not at all. 10 Or, in another of Halls pithy formulas, politics does not reflect majorities, it
constructs them. 11
It is important to be clear here. Hall never claims that ideology determines the course of globalization but claims that it harnesses it for one political purpose or another, and
when it is successful, the political and economic strategies represented by a particular ideology will also themselves bring into being certain political-economic formations within
global capitalist developments.

Now we are beginning . . . to move into a post-Fordist society what some

theorists call disorganized capitalism, the era of flexible specialisation. One way of reading present developments is that privatization is Thatcherisms way of harnessing and
appropriating this underlying movement within a specific economic and political strategy and constructing it within the terms of a specific philosophy. It has succeeded, to some
degree, in aligning its historical, political, cultural and sexual logics with some of the most powerful tendencies in the contemporary logics of capitalist development. And this,
in part, is what gives it its supreme confidence, its air of ideological complacency: what makes it appear to have history on its side, to be coterminous with the inevitable
course of the future. The left, however, instead of rethinking its economic, political, and cultural strategies in the light of this deeper, underlying logic of dispersal and
diversification (which after all, need not necessarily be an enemy of greater democratization) simply resists it. If Thatcherism can lay claim to it, then we must have nothing to do
with it. Is there any more certain way of rendering yourself historically anachronistic? 12

If the contemporary Left often clings to the formations and formulations of another epoch, one in which
the notions of unified movements, social [462] totalities, and class-based politics were viable categories of

Page 1462 of 1481


political and theoretical analysis, this means that it literally renders itself a conservative force in history
one that not only misreads the present but also installs traditionalism in the very heart of its praxis, in the
place where commitment to risk and upheaval belongs. Walter Benjamin sketches this phenomenon in his attack on Eric Kastner, the Left-
wing Weimer Republic poet, who is the subject of his Left-Wing Melancholy essay: This poet is dissatisfied, indeed heavy-hearted. But this heaviness of heart

derives from routine. For to be in a routine means to have sacrificed ones idiosyncracies, to have
forfeited the gift of distaste. And that makes one heavy-hearted. 13 In a different tonality, Stuart Hall sketches this problem in the
Lefts response to Thatcherism:

I remember the moment in the 1979 election when Mr. Callaghan, on his last political legs, so to speak, said with real astonishment about the offensive of Mrs. Thatcher that

. The truth is that


She means to tear society up by the roots. This was an unthinkable idea in the social-democratic vocabulary: a radical attack on the status quo

traditionalist ideas, the ideas of social and moral respectability, have penetrated so deep inside socialist
consciousness that it is quite common to find people committed to a radical political programme
underpinned by wholly traditional feelings and sentiments. 14

Page 1463 of 1481


Brown Turns (2/2)
THEIR POLITICS OF MOURNING ARE REALLY JUST MELANCHOLIA IN DRAG (PUN
INTENDED), WHICH FORECLOSES UPON ANALYZING THE POLITICAL
POTENTIALITIES OF OUR CURRENT SITUATION

Wendy Brown, Professor, Political Science and Womens Studies, University of California-Berkeley,
Resisting Left Melancholia, LOSS: THE POLITICS OF MOURNING, ed. David L. Eng & David Kazanjian, 2002,
p. 458-459.

For the last two decades, cultural theorist Stuart Hall has insisted that the crisis of the Left is due neither to internal
divisions in the activist or academic Left nor to the clever rhetoric or funding schemes of the Right. Rather, he has
charged, this ascendancy is consequent to the Lefts own failure to apprehend the character of the age and to develop a
political critique and a moral-political vision appropriate to this character. For Hall, the rise of the Thatcher-Reagan Right
was a symptom rather than a cause of this failure, just as the Lefts dismissive or suspicious attitude toward cultural politics is for Hall a sign not of its unwavering
principles but of its anachronistic habits of thought and its fears and anxieties about revising those habits.

? I want to develop just one thread of this problem through a consideration of


But what are the content and dynamic of these fears and anxieties

the phenomenon named Left melancholia by Walter Benjamin more than half a century ago. What did Benjamin mean by and with this pejorative appellation for a
certain intellectual and political bearing? As most readers will know, Benjamin was neither categorically nor characterologically opposed to the value and valence of sadness as such, nor to the
potential insights gleaned from brooding over ones losses. Indeed, he had a well-developed appreciation of the productive value of acedia, sadness, and mourning for political and cultural work, and

, Benjamin treated melancholia itself as something of a creative wellspring. But Left melancholia is
in his study of Baudelaire

Benjamins unambivalent epithet for the revolutionary hack who is, finally, more attached to a particular political
analysis or ideal even to the failure of that ideal than to seizing possibilities for radical change in the present. In
Benjamins enigmatic insistence on the political value of a dialectical historical grasp of the time of the Now, Left [458] melancholia represents not only a refusal

to come to terms with the particular character of the present, that is, a failure to understand history in terms other
than empty time or progress. It signifies as well a certain narcissism with regard to ones past political attachments
and identity that exceeds any contemporary investment in political mobilization, alliance, or transformation. 1

The irony of melancholia, of course, is that attachment to the object of ones sorrowful loss supersedes any desire to
recover from this loss, to live free of it in the present, to be unburdened by it. This is what renders melancholia a
persistent condition, a state, indeed, a structure of desire, rather than a transient response to death or loss. In Freuds 1917
meditation on melancholia, he reminds us of a second singular feature of melancholy: It entails a loss of a more ideal kind [than mourning]. The object

has not perhaps actually died, but has been lost as an object of love. 2 Moreover, Freud suggests, the melancholic will often not
know precisely what about the object has been loved and lost: This would suggest that melancholia is in some way
related to an object-loss which is withdrawn from consciousness, in contradistinction to mourning, in which there is
nothing about the loss that is unconscious. 3 The loss precipitating melancholy is more often than not unavowed and unavowable. Finally, Freud suggests that the
melancholic subject low in self-regard, despairing, even suicidal has shifted the reproach of the once-loved object (a reproach waged for not living up to the idealization by the beloved) onto itself,
thus preserving the love or idealization of the object even as the loss of this love is experienced in the suffering of the melancholic.

Now why would Benjamin use this term, and the emotional economy it represents, to talk about a particular formation on and of the Left? Benjamin never offers a precise formulation of Left
melancholia. Rather, he deploys it as a term of opprobrium for those more beholden to certain long-held sentiments and objects than to the possibilities of political transformation in the present.
Benjamin is particularly attuned to the melancholics investment in things. In the Trauerspiel, he argues that melancholy betrays the world for the sake of knowledge, here suggesting that the
loyalty of the melancholic converts its truth (every loyal vow or memory) about its beloved into a thing, indeed, imbues knowledge itself with a thinglike quality. 4 Another version of this formulation:
In its tenacious self-absorption [melancholy] embraces dead objects in its contemplation. 5 More simply, melancholia
is loyal to the world of things, 6 suggesting a certain logic of fetishism with all the conservatism and withdrawal
from human relations that fetishistic desire implies contained within the melancholic logic. In the critique of Kastners poems in which
Benjamin first coins Left melancholia, Benjamin suggests that sentiments themselves become things for the Left melancholic who takes as much pride in the [459] traces of former spiritual goods
We come to love our Left passions and reasons, our Left analyses and convictions, more
as the bourgeois do in their material goods. 7

than we love the existing world that we presumably seek to alter with these terms or the future that would be aligned
with them. Left melancholia, in short, is Benjamins name for a mournful, conservative, backward-looking attachment
to a feeling, analysis, or relationship that has been rendered thinglike and frozen in the heart of the putative Leftist. If
Freud is helpful here, then this condition presumably issues from some unaccountable loss, some unavowably crushed ideal, contemporarily signified by the terms Left, Socialism, Marx, or the
Movement.

Certainly the losses, accountable and unaccountable, of the Left are many in our own time. The literal disintegration of socialist regimes and the legitimacy of Marxism may well be the least of it. We
are awash in the loss of a unified analysis and unified movement, in the loss of labor and class as inviolable predicates of political analysis and mobilization, in the loss of an inexorable and scientific
forward movement of history, and in the loss of a viable alternative to the political economy of capitalism. And on the backs of these losses are still others: we are without a sense of international, and

often even local, Left community; we are without conviction about the Truth of the social order; we are without a rich moral-political vision of the Good to guide and sustain political work . Thus

Page 1464 of 1481


we suffer with the sense of not only a lost movement but also a lost historical moment, not only a lost theoretical and
empirical coherence but also a lost way of life and a lost course of pursuits.

in the hollow core of all these losses, perhaps in the place of


This much many on the Left can forthrightly admit, even if we do not know what to do about it. But

our political unconscious, is there also an unavowed loss the promise that Left analysis and Left commitment would
supply its adherents a clear and certain path toward the good, the right, and the true? Is it not this promise that formed
the basis for much of our pleasure in being on the Left, indeed, for our self-love as Leftists and our fellow feeling toward
other Leftists? And if this love cannot be given up without demanding a radical transformation in the very foundation of
our love, in our very capacity for political love or attachment, are we not doomed to Left melancholia, a melancholia
that is certain to have effects that are not only sorrowful but also self-destructive? Freud again: If the love for the object a love which cannot
be given up though the object itself is given up takes refuge in narcissistic identification, then the hate comes into operation on this substitutive object, abusing it, debasing it, making it suffer and
deriving sadistic satisfaction from its suffering. 8

Page 1465 of 1481


Page 1466 of 1481
Permutation Key to Socialism
SOCIALISM MUST EMBRACE THE CAUSES OF OTHER OPPRESSIONS IF IT IS TO EVER
BE SUCCESSFUL

John Bellamy Foster, University of Oregon, The Renewing of Socialism: An Introduction, THE MONTHLY
REVIEW v. 57 n. 3, July-August 2005. Available from the World Wide Web at:
http://www.monthlyreview.org/0705jbf.htm, accessed 4/12/06.

Socialism cannot survive unless it transcends not only class divisions that divide off those who run the
society from those that are compelled to work mainly on their behalf, but also all other major forms of
oppression that cripple human potential and prevent democratic, social alliances. If any lesson was
learned from the experiences of twentieth-century attempts to create socialism it is that class struggle
must be inseparable from the struggles against gender, race, and national oppressionsand against other
forms of domination such as those directed against gays or against those politically designated as the
disabled. Socialism also cannot make any real headway unless it is ecological in the sense of promoting a
sustainable relation to the environment, since any other approach threatens the well-being and even
survival of the human species, along with all other species with which we share the earth. The various
forms of non-class domination are so endemic to capitalist society, so much a part of its strategy of divide
and conquer, that no progress can be made in overcoming class oppression without also fighting
sometimes even in advance of the class strugglethese other social divisions. If the political
emancipation of bourgeois society constituted one of the bases upon which a wider human emancipation
could be built, a major obstacle to the latter has been the fact that political emancipationthe realm of
so-called inalienable human rightshas remained incomplete under capitalism. That obstacle must in all
cases be overcome as a necessary part of the struggle for a socialist society.

Page 1467 of 1481


Page 1468 of 1481
**Miscellaneous**

Page 1469 of 1481


A2 Art (1/2)
AESTHETICS ARENT ENOUGH THEORETICAL ENLIGHTENMENT IS NECESSARY TO
INSTILL SOCIAL REVOLUTION

Best & Kellner 2002


[Steven & Douglas, Richard Rorty and Postmodern Theory,
www.gseis.ucla.edu/faculty/kellner/essays/richardrortypostmoderntheory.pdf, 9-24-06//uwyo-ajl]

Theory is necessary to the extent that the world is not completely and immediately transparent to

consciousness. Since this is never the case, especially in our own hypercapitalist culture where

the shadows flickering on the walls of our caves stem principally from television sets, the

corporate-dominated ideology machines that speak the language of deception and manipulation.

As we show in our book The Postmodern Adventure (Best and Kellner, 2001), which contains

studies of Thomas Pynchon, Michael Herr, Mary Shelley, H.G. Wells, Philip K. Dick, and other

imaginative writers, Rorty is right that fiction can powerfully illuminate the conditions of our

lives, often in more concrete and illuminating ways than theory. Ultimately, we need to grant

power to both theory and fiction, and understand their different perspectives and roles. For just

as novels like Upton Sinclair's The Jungle had dramatic social impact, so too has the discourse of

the Enlightenment, which provided the philosophical inspiration for the American and French

Revolutions, as well as numerous succeeding revolts in history.

MUSIC AND ART HAVE LIMITS IN SPACE AND TIME WHICH ALLOW THEM TO BE
EASILY COMMODIFIED AND USED TO AFFIRM BOURGEOIS CULTURE

John Beverley, The Ideology of Postmodern Music and Left Politics, 1990
(http://muse.jhu.edu/journals/postmodern_culture/v001/1.1beverley.html)

The antidote to Muzak would seem to be something like Punk. By way of a preface to a discussion of Punk
and extending the considerations above on the relation between music and commodification, I want to
refer first to Jackson Pollock's great paintingAutumn Rhythm in the Met, a picture that--like Pollock's work
in general--is particularly admired by Free Jazz musicians. It's a vast painting with splotches of black,
brown and rust against the raw tan of unprimed canvas, with an incredible dancing, swirling, clustering,
dispersing energy. As you look at it, you become aware that while the ambition of the painting seems to
be to explode or expand the pictorial space of the canvas altogether, it is finally only the limits of the
canvas which make the painting possible as an art object. The limit of the canvas is its aesthetic
autonomy, its separation from the life world, but also its commodity status as something that can be

Page 1470 of 1481


bought, traded, exhibited. The commodity is implicated in the very form of the "piece;" as in the jazz
record in Nausea, "The music ends." (The 78 RPM record--the commodity form of recorded music in the
20s and 30s-- imposed a three minute limit per side on performances and this in turn shaped the way
songs were arranged in jazz or pop recording: cf. the 45 and the idea today of the "single.")

Such a situation might indicate one limit of Jameson's cultural hermeneutic. If the strategy in Jameson is
to uncover the emancipatory utopian- communist potential locked up in the artifacts of the cultural
heritage, this is also in a sense to leave everything as it is, as in Wittgenstein's analytic (because that
which is desired is already there; it only has to be "seen" correctly), whereas the problem of the relation
of art and social liberation is also clearly the need to transgress the limits imposed by existing artistic
forms and practices and to produce new ones. To the extent, however, such transgressions can be
recontained within the sphere of the aesthetic-- in a new series of "works" which may also be available as
commodities--, they will produce paradoxically an affirmation of bourgeois culture: in a certain sense they
are bourgeois high culture.

Page 1471 of 1481


Page 1472 of 1481
A2 Art (2/2)
ART AS RESISTANCE CAN GIVE AN AESTHETIC GRATIFICATION WHICH STOPS
FURTHER STRUGGLE

John Beverley, The Ideology of Postmodern Music and Left Politics, 1990
(http://muse.jhu.edu/journals/postmodern_culture/v001/1.1beverley.html)

Adorno and the Frankfurt School make of the Kantian notion of the aesthetic as a purposiveness without
purpose precisely the locus of the radicalizing and redemptive power of art, the sense in which by
alienating practical aims it sides with the repressed and challenges domination and exploitation,
particularly the rationality of capitalist institutions. By contrast, there is Lenin's famous remark--it's in
Gorki'sReminiscences--that he had to give up listening to Beethoven'sAppasionata sonata: he enjoyed it
too much, it made him feel soft, happy, at one with all humanity. His point would seem to be the need to
resist a narcotic and pacifying aesthetic gratification in the name of the very difficult struggle--and the
corresponding ideological rigor--necessary to at least setting in motion the process of building a classless
society. But one senses in Lenin too the displacement or sublation of an aesthetic sensibility onto the field
of revolutionary activism. And in both Adorno and Lenin there is a sense that music is somehow in excess
of ideology.

Page 1473 of 1481


Page 1474 of 1481
A2 Love
LOVE DOESNT BRING PEACEFUL RECONCILIATIONIT RECREATES DIVISION AND
OTHERNESS

Michael Dillon, Senior Lecturer in Politics and International Relations at the Univ. of Lancaster, 1996, the
Politics of Security, 194-5

But do not mistake this love for a peaceful reconciliation, or a reconciliation which brings peace, neither
for a release that brings tranquillity. There is a force in this I love you, I want you to be. Do not be fooled
into thinking, therefore, that the only violence of any consequence is the violence which elides the other,
effaces and defaces the other; as if, if we could, facing up to rather than effacing the other, we might
bring an end to violence. For in this releasing love also lies the power and latent force of an injunction
an injunction that is itself disjointed.128 Issuing from someone who is himself out-of-joint, this
injunctioneffects another disjunction. Oedipus, with love, and through love, releases them to their
inheritance of life. That injunction insists, albeit lovingly, that the other can be no other than other, come
what may; and exhorts it to remain so. Ultimately, he says, you can (must) be no other than other
because that is what you are; haunted, as am I, by an Otherness which I recognise but can never know,
and I love you for it. Smitten as he is with love, Oedipus smites back in the same currency: Pray for life my
children, live where you are free to grow and season. Pray god you find a better life than mine, the father
who begot you. In being loved, and finding love, these lines suggest, you do not find peace. Instead, the
struggle of your being is refreshed. This, therefore, is not the struggle-free love of a happy ending. Love
does not gather everything together in unison and unity here. An expression of their being-in-common, of
their belonging together in virtue of difference, this love also effects a new scission or severance in which
the possibility of the being of these other beings specifically his daughters takes place. It
consequently also splits asunder, severs and divides, allowing for a further duplication of difference. For
only in division does the possibility of possibilities and their multiplication occur. There is violence in it
precisely, therefore, because it is an acceptance of this mortal freedom. Moreover, it is not a freedom
which is paternally, or patriarch-ally, granted by Oedipus, despite his being their father. His off-spring are
being wrenched from his grasp, and it is not a freedom, in any event, that is within his gift. Instead, it is an
antecedent freedom which, in extremity, he now recognises that he shares in common with them. This I
love you, I want you to be therefore says: stand up for yourself and before others take up the
burden of freedom afresh and live it out more fully. Just as he tries to give that which he does not have
the bearings of their latitude to be so Oedipus love amounts to an impossible command. Freeing them
of himself is not possible. He cannot efface what he is and what has happened. He cannot, in fact,
withdraw what has already been given, and nothing can free them of the inheritance into which they have
come.129 In short, they cannot be delivered of their existence. But, is it possible to have it delivered over
to them, freely to assume it as best they might, in a freeing way? Perhaps this is what Oedipus is trying to
do. They alone can bear their inheritance. But, in the way in which he himself comports himself towards
its handing-on saying something like grieve for me, therefore, keep me enough to use me as you
must130 Oedipus tries to contribute somehow to their free reception and assumption of it. Ultimately,
however, it is impossible because (irrespective of the awfulness of the inheritance which he leaves to
them) there is nothing he, or they, can do to secure compliance with a command to find and enjoy a
better life. And, yet, that is precisely the forceful insistent desire of love for the loved one. For its
otherness, and the Otherness it bears, to thrive.

Page 1475 of 1481


Page 1476 of 1481
A2 Poetry
LITERATURE FAILS TO UNDERMINE THE HEGEMONIC MODES OF REPRESENTATION

John Beverley, Professor of Spanish and Latin American Literature and Cultural Studies at Pitt, Subalternity and
Representation: Arguments in Critical Theory, 1999, p.4

Our hypothesis in Literature and Politics was that the dominant forms of modern Central American
literaturepoetry in particularhad become a material forcean ideological practice, in the sense Louis
Althusser gives the termin the construction of the revolutionary movements that were vying for power
in the region. However, as Marc and I struggled to finish the book we were struck with a growing sense of
the limitations of literature as a form of popular empowerment and agencylimitations revealed
dramatically for us in the debates around the poetry workshop experiment in Nicaragua and in the
question of testimonio as a narrative form that resisted in some ways being treated simply as a new kind
of literature. We ended Literoture and Politics with these words: "We return, therefore, in closing to the
paradox that has been with us from the beginning of this book: literature has been a means of national-
popular mobilization in the Central American revolutionary process, but that process also elaborates or
points to forms of cultural democratization that will necessarily question or displace the role of literature
as a hegemonic cultural institution" (207).

POETRY CANNOT BE TRANSFORMATIVE IN THE CURRENT POLITICAL CLIMATE

Jamie Owen Daniel, English Department, The University of Illinois at Chicago, Does "poetry makes nothing
happen?": The Case for Public Poetry as a Counter-Public Sphere, 1997,

http://english.rutgers.edu/does.htm , accessed January 30, 2002.

Thus, as attractive as the idea of public poetry may be as an alternative public sphere, it nonetheless
remains fixated and fixed at the level of changing the final product rather than the process of production.
Merely allowing "diverse relations of power and privilege" to intermingle in public space, whether a
poetry slam or spontaneous street festival or an academic conference such as this one, does not magically
render those constituencies equal, given their various histories of deprivation or exploitation. Our inability
or unwillingness to confront the bigger, less easily manipulable world outside the institutions of culture is,
it seems to me, symptomatic of a key flaw in the celebration of more public poetry as a potential counter-
public sphere, for no matter how we de- or reconstruct the hierarchy of authority, no matter how many
voices we allow or encourage to intermingle, we still can't make a democratic public sphere, or a
democratic culture, in a society based on a system that remains fundamentally anti-democratic.

Page 1477 of 1481


Page 1478 of 1481
A2 Silence
SILENCE IS CONSENT. SPEAKING RESTORES DIGNITY

Nadezhda Mandelstam, Hope Against Hope, 1971, quoted in: In A Dark Time, ed. Robert Lifton, 1986

When a bull is being led to the slaughter, it still hopes to break loose and trample its butchers. Other bulls
have not been able to pass on the knowledge that this never happens and that from the slaughterhouse
there is no way back to the herd. But in human society there is a continuous exchange of experience. I
have never heard of a [hu]man who broke away and fled while being led to his execution. It is even
thought to be a special form of courage if a man about to be executed refuses to be blindfolded and dies
with his eyes open. But I would rather have the bull with his blind rage, the stubborn beast who doesnt
weigh his chances of survival with the prudent dull-wittedness of man, and doesnt know the despicable
feeling of despair. Later I often wondered whether it is right to scream when you are being beaten and
trampled underfoot. Isnt it better to face ones tormentors in a stance of satanic pride, answering them
with contemptuous silence? I decided that it is better to scream. This pitiful sound, which sometimes,
goodness knows how, reaches into the remotest prison cell, is a concentrated expression of the last
vestige of human dignity. It is a mans way of leaving a trace, of telling people how he lived and died. By
his screams [one] he asserts [the] his right to live, sends a message to the outside world demanding help
and calling for resistance. If nothing else is left, one must scream. Silence is the real crime against
humanity.

Page 1479 of 1481


Page 1480 of 1481
A2 Third World Bad
THIRD WORLD, WHILE IMPRECISE AND ESSENTIALIZING, IS A USEFUL PHRASE
AND NECESSARY FOR COMMUNICATION

Martin W. Lewis, associate research professor of geography, co-director of Comparative Area Studies, Duke
University, 1992, Green Delusions, pp. 191-192

One must take care to draw distinctions within this broad zone of global poverty. The environmental
problems and prospects of Mexico, for example, are as different from those of Mali as they are from
those of Germany. Still, terms such as the Third World or the South provide convenient labels for the
earths relatively poor countries. In this chapter Third World will be employed to designate both the
relatively nonindustrialized and the recently industrializing areas of the globe. The term admittedly
obscures almost as much as it reveals, but such imprecision is necessary if we are to avoid using stiflingly
cumbersome forms of expression.

Page 1481 of 1481

Das könnte Ihnen auch gefallen